Sunteți pe pagina 1din 382

y

o
u
rs
m
a
h
b
o
o
b
.w
o
rd
p
re
s
s
.c
o
m
101 SPEED TEST

with
SBI
BANK CLERK
5 Practice Sets for Preliminary & Main Exams
y
o
u
rs
m
a
h
b
o
o
• Head Office : B-32, Shivalik Main Road, Malviya Nagar, New Delhi-110017

b
.w
Sales Office : B-48, Shivalik Main Road, Malviya Nagar, New Delhi-110017

o

rd
Tel. : 011-26691021 / 26691713

p
re
s
s
.c
o
m
Typeset by Disha DTP Team

DISHA PUBLICATION
ALL RIGHTS RESERVED

© Copyright Publisher
No part of this publication may be reproduced in any form without prior permission of the publisher. The author and the
publisher do not take any legal responsibility for any errors or misrepresentations that might have crept in. We have tried
and made our best efforts to provide accurate up-to-date information in this book.

For further information about books from DISHA,


Log on to www.dishapublication.com or email to info@dishapublication.co.in

[ ii ]
y
o
u
rs
m
a
h
b
o
o
Preface

b
.w
o
rd
p
re
101 Speed Tests for SBI Bank Clerk Exam

s
s
101 Speed Tests for SBI Bank Clerk Exam is revised and updated edition on the basis of Current Exam Patten. It contains all the

.c
o
IMPORTANT CONCEPTS which are required to crack this exam. The concepts are covered in the form of 101 SPEED TESTS.

m
No matter where you PREPARE from – a coaching or any textbook/ Guide — 101 SPEED TESTS provides you the right
ASSESSMENT on each topic. Your performance provides you the right cues to IMPROVE your concepts so as to perform better
in the final examination.
It is to be noted here that these are not mere tests but act as a checklist of student’s learning and ability to apply concepts to
different problems.
The book is based on the concept of TRP – Test, Revise and Practice. It aims at improving your SPEED followed by STRIKE RATE
which will eventually lead to improving your SCORE.

How is this product different?


• 1st unique product with 101 speed tests.
• Each test is based on small topics which are most important for the SBI Bank Clerk exam. Each test contains around 25-
30 MCQs on the latest pattern of the exam.
• The whole syllabus has been divided into 5 sections which are further distributed into 96 topics.
1. QUANTITATIVE APTITUDE is distributed into 28 topics.
2. REASONING is distributed into 24 topics.
3. ENGLISH LANGUAGE is distributed into 19 topics.
4. COMPUTER KNOWLEDGE/ MARKETING APTITUDE is distributed into 9 topics.
5. GENERAL AWARENESS is distributed into 16 topics.
• In the end of each section a Sectional Test is provided so as to sum up the whole section.
• Finally at the end 5 FULL TESTS-3 Tests for Prelim and 2 Tests for Mains are provided so as to give the candidates the real
feel of the final exam.
• In all, the book contains 3400+ Quality MCQ’s in the form of 101 tests.
• Solutions to each of the 101 tests are provided at the end of the book.
• The book provides Separate Tests. The book comes with perforation such that each test can be torn out of the book.
• Separate Time Limit, Maximum Marks, Cut-off, Qualifying Score is provided for each test.
• The book also provides a separate sheet, SCORE TRACKER where you can keep a record of your scores and performance.
• It is advised that the students should take each test very seriously and must attempt only after they have prepared that
topic.
• Once taken a test the candidates must spend time in analysing their performance which will provide you the right cues
to IMPROVE the concepts so as to perform better in the final examination.
It is our strong belief that if an aspirant works hard on the cues provided through each of the tests he/ she can improve
his/ her learning and finally the SCORE by at least 20%.
DISHA PUBLICATION

[ iii ]
y
o
u
rs
m
a
h
b
o
Syllabus

o
b
.w
o
rd
p
REASONING QUANTITATIVE APTITUDE

re
s
s
1. ANALOGY 1-2 29. NUMBER SYSTEM - I 59-60

.c
o
30. NUMBER SYSTEM - II 61-62

m
2. CLASSIFICATION 3-4
31. LCM, HCF AND FRACTION 63-64
3. SERIES 5-6
32. SQUARE, CUBE, INDICES & SURDS 65-66
4. ALPHABET TEST 7-8
33. SIMPLIFICATION 67-68
5. CODING - DECODING 9-10
34. AVERAGE 69-70
6. BLOOD RELATION 11-12
35. PERCENTAGE 71-72
7. DIRECTION AND DISTANCE 13-14
36. RATIO & PROPORTION 73-74
8. TIME, SEQUENCE & RANKING TEST 15-16
37. ALLIGATION AND MIXTURE 75-76
9. SITTING ARRANGEMENT 17-18 38. PROFIT, LOSS & DISCOUNT 77-78
10. ANALYTICAL PUZZLE - I 19-20 39. SIMPLE INTEREST 79-80
11. ANALYTICAL PUZZLE - II 21-22 40. COMPOUND INTEREST 81-82
12. VENN DIAGRAMS 23-24 41. DISTANCE, SPEED AND TIME 83-84

13. SYLLOGISM - I 25-26 42. TIME & WORK / PIPE & CISTERN 85-86

14. SYLLOGISM - II 27-28 43. PROBLEM BASED ON AGES 87-88


44. PERMUTATION & COMBINATION 89-90
15. SYMBOLS & CODES 29-30
45. PROBABILITY 91-92
16. ALPHA NUMERIC SEQUENCE PUZZLE 31-32
46. AREA AND PERIMETER 93-94
17. INPUT-OUTPUT 33-34
47. VOLUME AND SURFACE AREA 95-96
18. MATHEMATICAL OPERATIONS 35-36
48. GEOMETRY 97-98
19. CLOCK AND CALENDAR 37-38
49. NUMBER SERIES - I 99-100
20. DATA SUFFICIENCY 39-40
50. NUMBER SERIES - II 101-102
21. STATEMENT & CONCLUSION 51. DATA INTERPRETATION 103-104
(MATHEMATICAL) 41-42
52. SECTION TEST :
22. STATEMENT & CONCLUSION QUANTITATIVE APTITUDE 105-106
(LOGICAL) 43-44
23. PASSAGE BASED CONCLUSION-1 45-46 ENGLISH LANGUAGE
53. READING COMPREHENSION - I 107-110
24. PASSAGE BASED CONCLUSION-2 47-48
54. READING COMPREHENSION - II 111-114
25. CUBE & DICE 49-50
55. READING COMPREHENSION - III 115-118
26. NON-VERBAL REASONING - I 51-52
56. SYNONYMS 119-120
27. NON-VERBAL REASONING - II 53-54
57. ANTONYMS 121-122
28. SECTION TEST : REASONING 55-58
58. SENTENCE COMPLETION - I 123-124
[ iv ]
y
o
u
rs
m
a
h
b
o
o
b
59. SENTENCE COMPLETION - II 125-126 GENERAL AWARENESS

.w
o
60. SIMILAR SUBSTITUTION 127-128

rd
81. HISTORY OF BANKING AND

p
61. CORRECT USAGE OF

re
ITS DEVELOPMENT 171-172
PREPOSITION 129-130

s
s
.c
62. SENTENCE IMPROVEMENT 131-132 82. RBI & ITS MONETARY POLICIES 173-174

o
m
63. SPOTTING THE ERRORS - I 133-134 83. BANKING PRODUCT & SERVICES 175-176
64. SPOTTING THE ERRORS - II 135-136 84. BANKING TERM/ TERMINOLOGY 177-178
65. SPELLING TEST 137-138
85. MICRO FINANCE & ECONOMICS 179-180
66. ONE WORD SUBSTITUTION 139-140
86. FOREIGN TRADE 181-182
67. PARA JUMBLES 141-142
68. IDIOMS AND PHRASES 143-144 87. SOCIO-ECO-POLITICAL
ENVIRONMENT OF INDIA 183-184
69. CLOZE TEST - I 145-146
70. CLOZE TEST - II 147-148 88. APPOINTMENT/ ELECTION/
RESIGNATION 185-186
71. SECTION TEST :
ENGLISH LANGUAGE 149-152 89. EVENTS/ ORGANISATION/
SUMMITS 187-188
COMPUTER KNOWLEDGE/ MARKETING APTITUDE

72. COMPUTER FUNDAMENTAL / 90. AWARDS AND HONOURS 189-190


BINARY SYSTEM/ OPERATING SYSTEM 153-154
91. BOOKS AND AUTHORS 191-192
73. MS OFFICE/ COMMANDS and 155-156
SHORTCUT KEYS 92. SPORTS AND GAMES 193-194
74. SOFTWARES/ PROGRAMMING 157-158
93. SCIENCE AND TECHNOLOGY 195-196
75. INTERNET, NETWORKING AND 159-160
COMPUTER ABBREVIATIONS 94. CURRENT BANKING 197-198
76. FUNDAMENTALS OF MARKETING, 161-162 95. CURRENT AFFAIRS 199-200
PRODUCT AND BRANDING
77. MARKET SITUATIONS BASED ON PRICE, 96. SECTION TEST :
DISTRIBUTION, PROMOTION AND General & Financial Awareness 201-204
ADVERTISING 163-164
97. PRELIM FULL TEST - 1 205-212
78. MARKET SEGMENTATION,
TARGETING AND POSITIONING 165-166 98. PRELIM FULL TEST - 2 213-220
79. MODERN MARKETING /
MARKETING IN BANKING INDUSTRY 167-168 99. PRELIM FULL TEST - 3 221-228

80. SECTION TEST : 100. Full MAIN TEST - 4 229-242


COMPUTER KNOWLEDGE/
MARKETING APTITUDE 169-170 101. Full MAIN TEST - 5 243-256

101 SPEED TEST - SOLUTIONS 1-116

[v]
m
o
.c
s
s

re
p
rd
o
.w
b
o
o
b
h
a

m
rs
u
o
y

[ vi ]
m
o
.c
s
s

re
p
rd
o
.w
b
o
o
b
h
a

m
rs
u
o
y

[ vii ]
y
o
u
rs
m
a
h
b
o
o
b
.w
o
rd
p
– –

re
s
s
.c
o
m
30 15
15
15
30 15
15
15
15
15
25 30

25

25

25
15 23
20 12 15
15 20

135 150
135 150
135 150
135 150
135 150

[ viii ]
y
o
u
rs
m
a
h
b
o
1

o
b
.w
o
rd
Analogy

p
re
s
s
.c
o
m
Max. Marks : 30 No. of Qs. 30 Time : 20 min. Date : ........./......../................

1. A 'Square' is related to 'Cube' in the same way as a 'Circle' is 8. NUMBER : UNBMRE : : GHOST : ?
related to (a) HOGST (b) HOGTS (c) HGOST
(a) Sphere (b) Circumference (c) Diameter
(d) HGSOT (e) HGOTS
(d) Area (e) None of these
9. DRIVEN : EIDRVN : : BEGUM : ?
2. In a certain code BRIGHT is written as JSCSGG. How is
JOINED written in that code? (a) EUBGM (b) MGBEU
(a) HNIEFO (b) JPKEFO (c) JPKMDC (c) BGMEU (d) UEBGM
(d) KPJCDM (e) None of these (e) BGMUE
3. '34' is related to '12' in the same way as '59' is related to 10. PRLN : X Z T V : : J L F H : ?
(a) 45 (b) 14 (c) 42 (a) RTNP (b) NPRT (c) NRPT
(d) 38 (e) 47
(d) NTRP (e) RPNT
4. 'Mustard' is related to 'Seed' in the same way as 'Carrot' is
11. XWV : UTS : : LKJ : ... ?
related to
(a) Fruit (b) Stem (c) Flower (a) IHG (b) JKL (c) STU
(d) Root (e) None of these (d) MNO (e) KIG
DIRECTIONS (Q. 5 - 24) : For each of the following questions 12. QYGO : SAIQ : : UCKS : ?
there is some relationship between the two terms to the left of :: (a) WDMV (b) VFNU (c) WDLU
and the same relationship obtains between the two terms to its
(d) VEMU (e) WEMU
right. In each of these questions the fourth term is missing. This
term is one of the alternatives among the five terms given below. 13. YAWC : UESG : : QIOK : ?
Find out this term. (a) MINC (b) MIKE (c) KOME
5. ADE : FGJ :: KNO : ? (d) MMKO (e) LIME
(a) PQR (b) TPR (c) PQT
14. CFIL : PSVY : : HKNQ : ?
(d) RQP (e) PRS
(a) NPSV (b) LPSY (c) LORU
6. DGPGJ : MPQPS : : KNENQ : ....?
(a) RUFUX (b) RFUFX (c) RXUXF (d) MOQT (e) MPSU
(d) RFUFX (e) None of these 15. 122 : 170 : : 290 : ?
7. UTS : EDC : : WVU : ? (a) 299 (b) 315 (c) 332
(a) XWV (b) WXY (c) SJM (d) 344 (e) 362
(d) RPO (e) SRP

1. a b c d e 2. a b c d e 3. a b c d e 4. a b c d e 5. a b c d e
RESPONSE 6. a b c d e 7. a b c d e 8. a b c d e 9. a b c d e 10. a b c d e
GRID 11. a b c d e 12. a b c d e 13. a b c d e 14. a b c d e 15. a b c d e
y
o
u
rs
m
a
h
2 SPEED TEST 1

b
o
o
16. 42 : 56 : : 110 : ? 25. Which of the following pairs of words have the same

b
.w
(a) 132 (b) 136 (c) 144
relationship as FAN : HEAT?
(d) 148 (e) 156

o
rd
17. 12 : 20 : : 30 : ? (a) Water : Drink (b) Light : Night

p
(a) 15 (b) 32 (c) 35 (c) Teach : Student (d) Air : Breathe

re
(d) 42 (e) 48

s
(e) Food : Hunger

s
.c
18. 3 : 10 : : 08 : ?

o
(a) 10 (b) 13 (c) 17 26. A disease would always necessarily have

m
(d) 14 (e) 16 (a) Medicine (b) Bacteria (c) Cause
19. 08 : 28 : 15 : ?
(d) Cure (e) Fever
(a) 63 (b) 126 (c) 65
(d) 124 (e) 26 27. 'Army' is related to 'Land' in the same way as 'Navy' is related
20. 08 : 09 : : 64 : ? to _____.
(a) 16 (b) 25 (c) 125
(a) Ships (b) Battle (c) Water
(d) 32 (e) 20
21. 6 : 24 : : 5 : ? (d) Admiral (e) Defence
(a) 23 (b) 22 (c) 26 28. A 'Tumbler' is related to 'Empty' in the same way as a 'Seat' is
(d) 20 (e) 19 related to
22. 6 : 35 : : 11 : ?
(a) 120 (b) 115 (c) 122 (a) Occupied (b) Person (c) Chair
(d) 121 (e) 124 (d) Sitting (e) Vacant
23. 3 : 27 : : 4 : ?
29. Secretly is to openly as silently is to__
(a) 140 (b) 75 (c) 100
(d) 80 (e) 64 (a) scarcely (b) impolitely (c) noisily
3 (d) quietly (e) None of these
24. 3:3 :: 5:?
8 30. Spring is to as coil as ring is to____

5 3 1 (a) rope (b) loop (c) cowl


(a) 5 (b) 5 (c) 5
8 8 8 (d) stretch (e) None of these

5 1
(d) 2 (e) 6
8 8

RESPONSE 16. a b c d e 17. a b c d e 18. a b c d e 19. a b c d e 20. a b c d e


21. a b c d e 22. a b c d e 23. a b c d e 24. a b c d e 25. a b c d e
GRID 26. a b c d e 27. a b c d e 28. a b c d e 29. a b c d e 30. a b c d e
y
o
u
rs
m
a
h
b
o
2

o
b
.w
Classification

o
rd
p
re
s
s
.c
o
m
Max. Marks : 30 No. of Qs. 30 Time : 20 min. Date : ........./......../................
1. Four of the following five are alike in a certain way on the 8. Four of the following five are alike in a certain way and so
basis of their positions in English alphabet and so form a form a group. Which is the one that does not belong to that
group. Which is the one that does not belong to that group. group?
(a) HJG (b) PQN (c) DEB (a) Garlic (b) Ginger (c) Carrot
(d) TUR (e) KLI (d) Radish (e) Brinjal
9. Four of the following five are alike in a certain way and so
2. Four of the following five are alike in a certain way and so
form a group. Which is the one that does not belong to that
form a group. Which is the one that does not belong to that
group?
group ?
(a) Clutch (b) Wheel (c) Break
(a) Rose (b) Jasmine (c) Hibiscus (d) Car (e) Gear
(d) Marigold (e) Lotus 10. Four of the following five are alike in a certain way and so
3. Four of the following five pairs of alphas and numerals have form a group. Which is the one that does not belong to that
same relationship between their elements as in the case of group?
the pair PROBLEM : 2948375 and hence form a group. Which (a) 196 (b) 256 (c) 529
one does not belong to the group? (d) 576 (e) 324
(a) BORE : 8497 (b) MOEP : 5972 11. Four of the following five are alike in a certain way and so
(c) LBOR : 3849 (d) OMEP : 4572 form a group. Which is the one that does not belong to that
(e) EROL : 7943 group?
4. Four of the following five pairs are alike in a certain way (a) RPN (b) WSU (c) HDF
(d) LHJ (e) QMO
and hence form a group. Which one does not belong to
12. Four of the following five are alike in a certain way and so
that group?
form a group. Which is the one that does not belong to that
(a) DONE : NOED (b) WANT : NATW group?
(c) WITH : TIHW (d) JUST : SUTJ (a) 169 (b) 441 (c) 361
(e) HAVE : AVEH (d) 529 (e) 289
5. Four of the following five are alike in a certain way and so 13. Four of the following five are alike in a certain way and so
form a group. Which is the one that does not belong to that form a group. Which is the one that does not belong to that
group? group?
(a) Hill (b) Valley (c) Dam (a) PM (b) EB (c) TQ
(c) River (e) Mountain (d) IF (e) VY
6. Four of the following five are alike in a certain way and so 14. Four of the following five are alike in a certain way and so
form a group. Which is the one that does not belong to that form a group. Which is the one that does not belong to that
group? group?
(a) 115 (b) 85 (c) 95
(a) 50 (b) 65 (c) 170
(d) 75 (e) 155
(d) 255 (e) 290
15. Four of the following five are alike in a certain way and so
7. Four of the following five are alike in a certain way and so form a group. Which is the one that does not belong to that
form a group. Which is the one that does not belong to that group?
group? (a) 115 (b) 161 (c) 253
(a) 21 (b) 35 (c) 42 (d) 391 (e) 345
(d) 56 (e) 49

1. a b c d e 2. a b c d e 3. a b c d e 4. a b c d e 5. a b c d e
RESPONSE 6. a b c d e 7. a b c d e 8. a b c d e 9. a b c d e 10. a b c d e
GRID 11. a b c d e 12. a b c d e 13. a b c d e 14. a b c d e 15. a b c d e
y
o
u
rs
m
a
h
4 SPEED TEST 2

b
o
o
16. Four of the following five are alike in a certain way and so 23. Four of the following five are alike in a certain way and

b
.w
form a group. Which is the one that does not belong to that hence form a group. Which is the one that does not belong

o
group? to that group?

rd
(a) OMQ (b) HFJ (c) TPR (a) KP (b) BY (c) DW

p
re
(d) TRV (e) VTX (d) HU (e) GT

s
24. Four of the following five are alike in a certain way and

s
17. Four of the following five are alike in a certain way based on

.c
hence form a group. Which is the one that does not belong

o
the English alphabetical series and so form a group. Which

m
to that group?
is the one that does not belong to that group?
(a) JLNK (b) TVXU (c) ACEB
(a) MLJ (b) WVT (c) OMK (d) PRTQ (e) GJKH
(d) JIG (e) TSQ 25. Four of the following five are alike in a certain way and
18. Four of the following five are alike in a certain way and so hence form a group. Which is the one that does not belong
form a group. Which is the one that does not belong to that to that group?
group? (a) GIJF (b) OQRN (c) KMNL
(a) Diabetes (b) Smallpox (d) UWXT (e) CEFB
(c) Conjunctivitis (d) Chickenpox 26. Four of the following five are alike in a certain way and
(e) Plague hence form a group. Which is the one that does not belong
19. Four of the following five are alike in a certain way and so to the group?
(a) Break (b) Change (c) Split
form a group. Which is the one that does not belong to that
(d) Divide (e) Separate
group?
27. Four of the following five are alike in a certain way and
(a) Mustard (b) Jowar (c) Wheat hence form a group. Which is the one that does not belong
(d) Paddy (e) Bajra to the group?
20. Four of the following five are alike in a certain way and so (a) Train (b) Instruct (c) Educate
form a group. Which is the one that does not belong to that (d) Advocate (e) Coach
group? 28. Four of the following five are alike in a certain way and
(a) 45 (b) 35 (c) 85 hence form a group. Which is the one that does not belong
(d) 25 (e) 75 to the group?
21. Four of the following five are alike in a certain way and (a) Extend (b) Higher (c) Upward
(d) Rise (e) Ascend
hence form a group. Which is the one that does not belong
29. Four of the following five are alike in a certain way and so
to that group?
form a group. Which is the one that does not belong to that
(a) Succeed (b) Victory (c) Triumph
group?
(d) Compete (e) Win (a) Volume (b) Size (c) Large
22. Four of the following five are alike in a certain way and (d) Shape (e) Weight
hence form a group. Which is the one that does not belong 30. Four of the following five are alike in a certain way and so
to that group? form a group. Which is the one that does not belong to that
(a) Fair (b) Impartial (c) Indifferent group?
(d) Unbiased (e) Just (a) Anxiety (b) Worry (c) Inhibition
(d) Curiosity (e) Weariness

RESPONSE 16. a b c d e 17. a b c d e 18. a b c d e 19. a b c d e 20. a b c d e


21. a b c d e 22. a b c d e 23. a b c d e 24. a b c d e 25. a b c d e
GRID 26. a b c d e 27. a b c d e 28. a b c d e 29. a b c d e 30. a b c d e
y
o
u
rs
m
a
h
b
o
3

o
b
.w
Series

o
rd
p
re
s
s
.c
o
m
Max. Marks : 30 No. of Qs. 30 Time : 20 min. Date : ........./......../................

DIRECTIONS (Q. 1 - 12) : What should come in place of question 9. 12 18 36 102 360 (?)
mark (?) in the following number series? (a) 1364 (b) 1386 (c) 1384
1. 1050 420 168 67.2 26.88 10.752 ? (d) 1376 (e) None of these

(a) 4.3008 (b) 6.5038 (c) 4.4015 10. 71 78 99 134 183 (?)
(d) 5.6002 (e) None of these (a) 253 (b) 239 (c) 246
2. 0 6 24 60 120 210 ? (d) 253 (e) None of these

(a) 343 (b) 280 (c) 335 11. 342 337.5 328.5 315 297 (?)
(d) 295 (e) None of these (a) 265.5 (b) 274.5 (c) 270
3. 15 19 83 119 631 (?) (d) 260 (e) None of these

(a) 731 (b) 693 (c) 712 12. 161 164 179 242 497 (?)
(d) 683 (e) None of these (a) 1540 (b) 1480 (c) 1520
4. 19 26 40 68 124 (?) (d) 1440 (e) None of these

(a) 246 (b) 238 (c) 236


(d) 256 (e) None of these DIRECTIONS (Qs. 13 to 17): What will come in place of question
5. 11 10 18 51 200 (?) mark (?) in the following number series?
(a) 885 (b) 1025 (c) 865 13. 12, 30, 56, 90, 132, ?
(d) 995 (e) None of these (a) 178 (b) 182 (c) 185
6. 14 24 43 71 108 (?) (d) 189 (e) 196
(a) 194 ( b) 154 (c) 145 14. 91, 381, 871, 1561, 2451, ?
(d) 155 (e) None of these (a) 3541 (b) 3621 (c) 3681
7. 144 173 140 169 136 (?) (d) 3716 (e) 3772
(a) 157 ( b) 148 (c) 164 15. 110, 440, 990, 1760, 2750, ?
(d) 132 (e) None of these (a) 3680 (b) 3610 (c) 37820
8. 656 352 200 124 86 (?) (d) 3840 (e) 3960
(a) 67 (b) 59 (c) 62
(d) 57 (e) None of these

1. a b c d e 2. a b c d e 3. a b c d e 4. a b c d e 5. a b c d e
RESPONSE 6. a b c d e 7. a b c d e 8. a b c d e 9. a b c d e 10. a b c d e
GRID 11. a b c d e 12. a b c d e 13. a b c d e 14. a b c d e 15. a b c d e
y
o
u
rs
m
a
h
6 SPEED TEST 3

b
o
o
16. 5, 6, 11, 20, 33, 50, ? 24. What will be the next term in ?

b
.w
(a) 64 (b) 71 (c) 78 DCXW, FEVU, HGTS, ...........

o
(d) 81 (e) 84

rd
(a) LKPO (b) ABYZ (c) JIRQ

p
17. 2, 7, 24, 77, 238, 723, ? (d) LMRS (e) None of these

re
s
(a) 1948 (b) 1984 (c) 2010 25. ZXVTR....

s
.c
(d) 2096 (e) 2180 (a) O, K (b) N, M (c) K, S

o
m
(d) M, N (e) P, N
DIRECTIONS (Q.18 - 22) :What should come in place of question 26. C, e, G, i, K....
mark (?) in the following number series? (a) O, K (b) m, O (c) k, M
18. 980 484 260 112 50 ? 3.5 (d) M, k (e) O, p
(a) 25 (b) 17 (c) 21 27. m_ _ l m _ l _ m m _ l
(d) 29 (e) None of these (a) mllml (b) mlmll (c) llmlm
19. 1015 508 255 129 66.5 ? 20.875 (d) mmlml (e) llmll
(a) 34.50 (b) 35 (c) 35.30
(d) 35.75 (e) None of these
28. What should come next in the number series given below ?
20. 4.5 18 2.25 ? 1.6875 33.75 1121231234123451234 56123456
(a) 27 (b) 25.5 (c) 36 (a) 5 (b) 2
(d) 40 (e) None of these (c) 8 (d) 1
21. 59.76 58.66 56.46 52.06 ? 25.66 (e) None of these
(a) 48.08 (b) 46.53 (c) 43.46 29. What should come next in the following letter series?
(d) 43.26 (e) None of these ABC DPQRSA BCDE PQ RSTABCDEFPQ RST
22. 36 157 301 470 ? 891 (a) A (b) V
(a) 646 (b) 695 (c) 639 (c) U (d) W
(d) 669 (e) None of these
(e) None of these
30. What will come in place of question mark (?) in the following
DIRECTIONS (Qs. 23 to 27) : Which one of the letters when series ?
sequentially placed at the gaps in the given letter series shall NDP, QWB, ZFR, ?
complete it?
(a) SVJ (b) AFS
23. a–ca–bc–bcc–bca
(c) IVS (d) SFA
(a) b b a b (b) b a b a (c) a a b b
(e) None of these
(d) b b a a (e) None of these

16. a b c d e 17. a b c d e 18. a b c d e 19. a b c d e 20. a b c d e


RESPONSE
21. a b c d e 22. a b c d e 23. a b c d e 24. a b c d e 25. a b c d e
GRID 26. a b c d e 27. a b c d e 28. a b c d e 29. a b c d e 30. a b c d e
y
o
u
rs
m
a
h
b
o
4

o
b
.w
Alphabet Test

o
rd
p
re
s
s
.c
o
m
Max. Marks : 30 No. of Qs. 30 Time : 20 min. Date : ........./......../................
1. How many such pairs of letters are there in the word 9. If the letters of the word ARROGANCE are interchanged,
EXCURSION, each of which has as many letters between first with fifth, second with sixth, third with seventh, fourth
them in the word as they have in the English alphabet? with eighth and the position of the ninth remains unchanged,
(a) None (b) One (c) Two then what will the new arrangement of letters be?
(d) Three (e) More than three (a) GANACRROE (b) GANCRAROE"
2. How many such pairs of letters are there in the word GOLDEN, (c) GNACORRAE (d) GANCARROE
each of which has as many letters between them in the word (e) None of these
as in the English alphabet? 10. If all the letters in the word 'PRINCE' are rearranged in
(a) None (b) One (c) Two alphabetical order, then how many letter(s) will remain
(d) Three (e) More than three unchanged?
3. How many such pairs of letters are there in the word STRIVE (a) None (b) One (c) Two
each of which has as many letters between them in the word
as in the English alphabet ? (d) Three (e) More than three
(a) One (b) Two (c) Three 11. How many meaningful English words can be formed with
(d) Four (e) None of these ESRO using each letter only once in each word?
4. How many meaningful English words can be formed with the (a) None (b) One (c) Two
letters 'ATN' using each letter only once in each word? (d) Three (e) More than three
(l) One (b) Two (c) Three 12. How many such pairs of letters are there in the word
(d) Four (e) More than four CONSTABLE, each of which has as many letters between
5. The serial order of how many letters in the word CLIENT will them in the word as in the English alphabet?
not differ than their serial order in the arrangement where the (a) None (b) One (c) Two
letters of the word are arranged alphabetically? (d) Three (e) More than three
(a) Four (b) One (c) Three 13. How many such pairs of letters are there in the word
(d) Two (e) None of these PHYSICAL, each of which has as many letters between them
6. How many meaningful English words can be made with the in the word as they have in the English alphabet?
letters ENAL using each letter only once in each word? (a) None (b) One (c) Two
(a) One (b) Two (c) Three
(d) Three (e) More than three
(d) Four (e) More than four
7. How many three - letter meaningful words can be formed 14. How many pairs of letters are there in the word 'SHIFTED'
from the word TEAR beginning with 'A' without repeating each of which has as many letters between its two letters as
any letter within that word? there are between them in the English alphabet?
(a) One (b) Three (c) Five (a) None (c) One (c) Two
(d) Two (e) None of these (d) Three (e) None of these
8. If all the letters in the word ARGUMENT are rearranged in 15. How many meaningful English words can be formed by using
alphabetical order and substituted by the letter immediately any two letters of the word 'GOT'?
following it in the English alphabet, what will be the new (a) Three (b) Two (c) One
arrangement of letters? (d) More than three (e) None of these
(a) BFHNOSUV (b) BFHONSWV
(c) BFHNOUSV (d) BFHNOQUV
(e) None of these

1. a b c d e 2. a b c d e 3. a b c d e 4. a b c d e 5. a b c d e
RESPONSE 6. a b c d e 7. a b c d e 8. a b c d e 9. a b c d e 10. a b c d e
GRID 11. a b c d e 12. a b c d e 13. a b c d e 14. a b c d e 15. a b c d e
y
o
u
rs
m
a
h
8 SPEED TEST 4

b
o
o
16. How many three-letter meaningful English words can be 23. If each vowel of the word WEBPAGE is substituted with the

b
.w
formed from the word NOTE beginning with T and without next letter of the English alphabet, and each consonant is
repeating any letter within that word? substituted with the letter preceding it, which of the following

o
rd
(a) Three (b) One (c) Two letters will appear thrice?

p
(d) None (e) None of these (a) G (b) F (c) Q

re
s
17. If all the letters in the word MERCIFUL are rearranged in (d) V (e) None of these

s
.c
alphabetical order and substituted by the alphabet preceding 24. How many meaningful four letter English words can be

o
them in the English alphabet, what will be the new formed with the letters TPSI using each letter only once in

m
arrangement of letters? each word?
(a) BDFIEKLQT (b) BDEHKLQT (a) One (b) Two (c) Three
(c) BDEHLKQT (d) BDEJMLQT (d) Four (e) More than four
(e) None of these 25. How many meaningful five-letter words can be formed with
18. If it is possible to make only one meaningful word, from the the letters SLIKL using each letter only once ?
first, the third, the fifth and the eighth letters of word (a) One (b) Two (c) Three
ENTERPRISE using each letter only once, first letter of the (d) More than three (e) None
word is your answer. If more than one such word can be 26. If each vowel in the word HABITUAL is changed to the next
made your answer is `X' and if no such word can be made, letter in the English alphabet and each consonant is changed
your answer is Y'. to the previous letter in the English alphabet, which of the
(a) P (b) S (c) T following will be fourth from the left ?
(a) A (b) S (c) J
(d) X (e) Y
(d) H (e) None of these
19. If in the word DISTANCE all the vowels are replaced by the
27. The positions of how many alphabets will remain unchanged
next letter and all the consonants are replaced by the previous
if each of the alphabets in the word WALKING is arranged in
letter and then all the letters are arranged alphabetically, which
alphabetical order from left to right ?
letter will be third from the right?
(a) None (b) One (c) Two
(a) M (b) F (c) R
(d) Three (e) More than three
(d) J (e) None of these
28. If the first three letters of the word COMPREHENSION are
20. After arranging the letters of the word FOLK in alphabetical reversed, then the last three letters are added and then the
order, if each letter is substituted by the letter immediately remaining letters are reversed and added, then which letter
preceding to it in the English alphabet, what will be the will be exactly in the middle ?
resultant form of the word?' (a) H (b) N (c) R
(a) GLMP (b) EJKP (c) EKJN (d) S (e) None of these
(d) EJKN (e) None of these 29. If the first and second letters in the word DEPRESSION were
21. If A is denoted by 1, B by 2, C by 3, D by 1, E by 2, F by 3 and interchanged, also the third and the fourth letters, the fifth
so on, what would be the sum of the digits for the word and the sixth letters and so on, which of the following would
MULE ? be the seventh letter from the right ?
(a) 7 (b) 8 (c) 9 (a) R (b) O (c) S
(d) 10 (e) None of these (d) P (e) None of these
22. How many meaningful English words can be made with the 30. Arrange the given words in alphabetical order. Which one
letters ARTSE using each letter only once in each word? comes in the middle?
(a) None (b) One (c) Two (a) Restrict (b) Rocket (c) Robber
(d) Three (e) More than three (d) Random (e) Restaurant

16. a b c d e 17. a b c d e 18. a b c d e 19. a b c d e 20. a b c d e


RESPONSE
21. a b c d e 22. a b c d e 23. a b c d e 24. a b c d e 25. a b c d e
GRID 26. a b c d e 27. a b c d e 28. a b c d e 29. a b c d e 30. a b c d e
y
o
u
rs
m
a
h
b
o
o
5

b
.w
o
Coding - Decoding

rd
p
re
s
s
.c
o
m
Max. Marks : 30 No. of Qs. 30 Time : 20 min. Date : ........./......../................

1. In a certain code, PROSE is written as PPOQE. How would 9. If ‘light’ is called ‘morning’, ‘morning’ is called ‘dark’, ‘dark’
LIGHT be written in that code ? is called ‘night’, ‘night’ is called ‘sunshine’ and ‘sunshine’ is
(a) LIGFT (b) LGGHT (c) LGGFT called ‘dusk’, when do we sleep ?
(d) JIEHR (e) None of these (a) Dusk (b) Dark (c) Night
2. If Z = 52 and ACT is equal to 48, then BAT will be equal to (d) Sunshine (e) None of these
(a) 39 (b) 41 (c) 44 10. If A + B = C, D – C = A and E – B = C, then what does D + F
(d) 46 (e) None of these stands for ? Provide your answer in letter terms as well as in
3. In a certain code, 15789 is written as EGKPT and 2346 is number terms.
written as ALUR. How is 23549 written in that code ? (a) J & 10 (b) A & 1 (c) C & 3
(a) ALEUT (b) ALGTU (c) ALGUT (d) Q & 17 (e) None of these
(d) ALGRT (e) None of these 11. In a certain code, ‘247’ means ‘spread red carpet’, ‘256’ means
4. In a certain coding system, RBM STD BRO PUS means ‘the ‘dust one carpet’ and ‘234’ means ‘one red carpet’. Which
cat is beautiful’. TNH PUS DIM STD means ‘the dog is digit in that code means ‘dust’?
brown’. PUS DIM BRO PUS CUS means ‘the dog has the (a) 2 (b) 3 (c) 5
cat’. What is the code for ‘has’ ? (d) 6 (e) None of these
(a) CUS (b) BRO (c) DIM 12. If table is called chair, chair is called cot, cot is called pot and
(d) STD (e) None of these pot is called filter, where does a person sit?
5. If NAXALITE is written in a certain code as LYVYJGRC’ how (a) pot (b) cot (c) chair
will INTEGRATE be written in the same code ? (d) filter (e) None of these
(a) LGRECYPRC (b) GLRCEPYRC 13. In a certain code OVER is written as $#%*. and VIST is written
(c) PYWMNOPQR (d) BLACKHOLE as #+×–. How is SORE written in that code?
(a) ×$*% (b) %×$*
(e) None of these
(c) ×*$% (d) None of these
6. ENGLAND is written as 1234526 and FRANCE as 785291.
How will GREECE be written in this coding scheme ? (e) All of these
(a) 381191 (b) 381911 (c) 394132 14. In a certain code language `PULSE' is written as `DRKTO'
(d) 562134 (e) None of these and 'NEW is written as `VDM'. How will 'PROBES' be written
7. In a certain code, CAT is written as SATC and DEAR is written in that code language?
as SEARD. How would SING be written in that code? (a) RDANQO (b) QSPCFT
(a) GNISS (b) SINGS (c) SGNIS (c) TFCPSQ (d) OPNADR
(d) BGINS (e) None of these (e) None of these
8. PROMISE is coded as 1234567. What should be the code for 15. If REMIT is written as *£3 7 and CONSUL is written as
MISER in that code language ? = %8 b $5; then OCELOT will be written as
(a) 45672 (b) 43672 (c) 76543 (a) %=3587 (b) %=£5%7
(d) 14572 (e) None of these
(c) %=35% (d) %35%7
(e) None of these

1. a b c d e 2. a b c d e 3. a b c d e 4. a b c d e 5. a b c d e
RESPONSE 6. a b c d e 7. a b c d e 8. a b c d e 9. a b c d e 10. a b c d e
GRID 11. a b c d e 12. a b c d e 13. a b c d e 14. a b c d e 15. a b c d e
y
o
u
rs
m
a
h
10 SPEED TEST 5

b
o
o
16. If AMONG is written as NAOGM and SPINE is written as 24. In a certain code,'LOCK' is written as `MPBJ'' and `BLOW' is

b
NSIEP, then LAMON will be written as written as `CMNV'. How is 'WINE' written in that code?

.w
(a) OALNM (b) MLONA (c) OLMNA (a) VHOF (b) XJMD (c) XJOR

o
rd
(d) OLNMA (e) None of these (d) VHMD (e) None of these

p
17. In a certain code SEAL is written as $75@ and DOSE is 25. In a certain code SOLDIER is written as JFSCRNK. How is

re
written as #8$7. How is SOLD written in that code? GENIOUS written in that code?

s
s
.c
(a) $8@# (b) #87$ (c) #8$7 (a) PVTHHFO (b) PVTHFDM

o
(d) $5@# (e) None of these (c) PVTHMDF (d) TVPHFDM

m
18. If 'table' is called 'chair'; 'chair' is called `cupboard', 'cupboard' (e) None of these
is called 'chalk', 'chalk' is called 'book', 'book' 26. If blue means green, green means black, black means white,
is called 'duster' and 'duster' is called 'table', what does the white means pink, pink means red and red means orange,
teacher use to write on the black board? then what is the colour of blood?
(a) book (b) cupboard (c) table (a) Red (b) Black (c) White
(d) duster (e) None of these (d) Pink (e) None of these
19. in a certain code `MOTHER' is written as OMHURF. How
will `ANSWER' be written in that code ?
DIRECTIONS (27 and 28) : Study the following information to
(a) NBWRRF (b) MAVSPE
answer the given questions :
(c) NBWTRD (d) NBXSSE
In a certain code ‘colours of the sky’ is written as ’ki la fa so’,
(e) None of these
‘rainbow colours’ is written as ‘ro ki’ and ‘sky high rocket’ is
20. In a certain code 'GROW' is written as '=@%#' and 'WITHIN'
written as ‘la pe jo’ and ‘the rocket world’ is written as ‘pe so ne’.
is written as '# ¸ + Ó ¸ D ' How is 'WING' written in that
27. Which of the following is the code for ’colours sky high’?
code?
(a) ro jo la (b) fa la jo (c) la ki so
(a) # ¸ D = (b) # % D = (c) % ¸ D =
(d) ki jo la (e) fa ki jo
(d) # ¸ Ó = (e) None of these
28. Which of the following will/may represent ‘the’?
21. If 'DO' is written as 'FQ' and 'IN' is written as 'KP' then how
(a) Only ’fa’ (b) Either ‘fa’ or ’la’
would 'AT' be written?
(a) CV (b) BS (c) CU (c) Only ’so’ (d) Only ’la’
(d) DV (e) None of these (e) Either ‘so’ or ’fa’
22. lf 8 is written as B, 1 as R, 6 as K, 9 as O, 4 as M, 7 as W and 29. In a certain code language DREA is written as BFSE, MING
3 as T, then how, would WROMBT be Written in the numeric is written as FMHL and TREA is written as BFSU How will
form? TISE be written in that code ?
(a) 714983 (b) 719483 (c) 769483 (a) DTHS (b) DSTV (c) DSHS
(d) 719486 (e) None of these (d) FUGS (e) None of these
23. In a code language, PINK is written as QHOJ and BOLT is 30. If table is called chair, chair is called cot, cot is called pot
written as CNMS. How would MUST be written in that code? and pot is called filter, where does a person sit?
(a) NVTS (b) NTTS (c) NTRS (a) pot (b) cot (c) chair
(d) NITU (e) None of these (d) filter (e) None of these

16. a b c d e 17. a b c d e 18. a b c d e 19. a b c d e 20. a b c d e


RESPONSE
21. a b c d e 22. a b c d e 23. a b c d e 24. a b c d e 25. a b c d e
GRID 26. a b c d e 27. a b c d e 28. a b c d e 29. a b c d e 30. a b c d e
y
o
u
rs
m
a
h
b
o
6

o
b
.w
Blood Relation

o
rd
p
re
s
s
.c
o
m
Max. Marks : 30 No. of Qs. 30 Time : 20 min. Date : ........./......../................

1. Pointing to a photograph, Arun said, she is the mother of my 10. Ankit is related to Binny and Chinky, Daizy is Chinky’s
brother’s son’s wife’s daughter. How is Arun related to the lady? mother. Also Daizy is Binny’s sister and Aruna is Binny’s
(a) Cousin (b) Daughter-in-law sister. How is Chinky related to Aruna?
(c) Uncle (d) None of these (a) Niece (b) Sister (c) Cousin
(e) None of these (d) Aunt (e) None of these
2. A’s mother is sister of B and has daughter C. How can A be 11. Saroj is mother-in-law of Vani who is sister-in-law of Deepak.
related to B from amongst the following ? Rajesh is father of Ramesh, the only brother of Deepak. How
(a) Niece (b) Uncle (c) Daughter is Saroj related to Deepak?
(d) Father (e) None of these (a) Mother-in-law (b) Wife (c) Aunt
3. Introducing Kamla, Mahesh said : His father is the only son (d) Mother (e) None of these
of my father. How was Mahesh related to Kamla ? 12. A man pointing to a photograph says, “The lady in the
(a) Brother (b) Father (c) Uncle photograph is my nephew’s maternal grandmother.” How is
(d) Son (e) None of these the lady in the photograph related to the man’s sister who
4. Anil, introducing a girl in a party, said, she is the wife of the has only brother and no other sister?
grandson of my mother. How is Anil related to the girl? (a) Sister-in-law (b) Cousin
(a) Father (b) Grandfather (c) Mother (d) Mother-in- law
(c) Husband (d) Father-in-law
(e) None of these
(e) None of these
13. A man pointing to a photograph says, “The lady in the
5. A and B are married couple. X and Y are brother. X is the
photograph is my nephew’s maternal grandmother and her
brother of A. How Y is related to B ?
son is my sister’s brother-in-law. How is the lady in the
(a) Brother (b) Brother-in-law photograph related to his sister who has no other sister?
(c) Son (d) Son-in-law
(a) Mother (b) Cousin
(e) None of these
(c) Mother-in-law (d) Sister-in-law
6. A man said to a woman, “Your mother’s husband’s sister is
(e) None of these
my aunt.” How is the woman related to the man ?
(a) Granddaughter (b) Daughter (c) Sister DIRECTIONS (Qs. 14 - 16) : Read the information given below to
(d) Aunt (e) None of these answer the questions that follow.
7. Showing a lady in the park, Vineet said, ‘She is the daughter Rama and Mohan are a married couple having two daughters
of my grandfather’s only son’. How is Vineet related to that named Smita and Devika. Devika is married to a man who is the
lady ? son of Madhu and Jeewan. Romila is the daughter of Aman.
(a) Father (b) Son (c) Brother Krishna who is Aman’s sister is married to Sunil and has two sons
(d) Mother (e) None of these Anuj and Ankur. Ankur is the grandson of Madhu and Jeewan.
8. X told Y, “Though I am the son of your father, you are not my 14. How is Krishna related to Devika ?
brother”. How is X related to Y ? (a) Sister-in-law (b) Sister
(a) Sister (b) Son (c) Daughter (c) Aunt (d) None of these
(d) None of these (e) None of these
(e) None of these
9. Rahul’s mother is the only daughter of Monika’s father. How
15. What is the relationship between Anuj and Romila?
is Monika’s husband related to Rahul?
(a) Uncle – Niece (b) Father – Daughter
(a) Uncle (b) Father
(c) Grandfather (d) Brother (c) Husband – Wife (d) Cousins
(e) None of these (e) None of these

1. a b c d e 2. a b c d e 3. a b c d e 4. a b c d e 5. a b c d e
RESPONSE 6. a b c d e 7. a b c d e 8. a b c d e 9. a b c d e 10. a b c d e
GRID 11. a b c d e 12. a b c d e 13. a b c d e 14. a b c d e 15. a b c d e
y
o
u
rs
m
a
h
12 SPEED TEST 6

b
o
o
16. Which of the following is true ? 22. How is T related to S in the expression 'T × R ¸ V – S'?

b
.w
(a) Anuj is the son of Smita. (a) Father (b) Sister (c) Daughter
(d) Aunt (e) None of these

o
(b) Romila is the cousin of Krishna.

rd
(c) Madhu is Sunil’s mother-in-law. 23. How is S related to T in the expression 'T + R – V + S'?

p
re
(d) Jeewan is Devika’s maternal uncle. (a) Uncle (b) Nephew

s
(e) None of these (c) Son (d) Cannot be determined

s
.c
DIRECTIONS (Qs. 17-20) : Read the information given below to (e) None of these

o
m
answer these questions: 24. Which of the following means that 'S is the husband of T'?
(i) In a family of six persons, A, B, C, D, E and F, there are two (a) T × R – V + S (b) T – R ¸ V × S
married couples. (c) T – R + V ¸S (d) T ¸ R × V + S
(ii) D is the grandmother of A and the mother of B. (e) None of these
(iii) C is the wife of B and the mother of F. 25. How is V related to T in the expression 'T ¸ R + V × S'?
(iv) F is the granddaughter of E. (a) Aunt (b) Nephew (c) Niece
17. What is C to A? (d) Uncle (e) None of these
(a) Daughter (b) Grandmother 26. D is brother of B. M is brother of B. K is father of M. T is wife
(c) Mother (d) Cannot be determined of K. How is B related to T?
(e) None of these (a) Son (b) Daughter
18. How many male members are there in the Family? (c) Son or Daughter (d) Data inadequate
(a) Two (b) Three (e) None of these
(c) Four (d) Cannot be determined DIRECTIONS (Qs. 27 - 30) : Study the following information
(e) None of these carefully to answer the questions that follow.
19. Which of the following is true? Adhir Mishra has three children : Urmila, Raghu and Sumit. Sumit
(a) A is the brother of F married Roma, the eldest daughter of Mr. And Mrs. Mohan. The
Mohans married their youngest daughter to the eldest son of Mr.
(b) A is the sister of F
and Mrs. Sharma and they had two children Sandeep and Shaifali.
(c) D has two grandsons
The Mohans have two more children, Roshan and Bimla, both
(d) None of these elder to Sheila. Sohan and Shivendar are sons of Sumit and Roma.
(e) None of these Leela is the daughter of Sandeep.
20. Who among the following is one of the couples? 27. How is Mrs. Mohan related to Sumit ?
(a) C D (b) D E (a) Aunt (b) Mother-in-law
(c) E B (d) Cannot be determined (c) Mother (d) Sister-in-law
(e) None of these (e) None of these
DIRECTIONS (Q.21-25) : Read the following information carefully 28. What is the surname of Sohan ?
and answer the questions which follow: (a) Sharma (b) Mohan (c) Mishra
(i) 'P ¸ Q' means 'P is son of Q'. (d) Raghu (e) None of these
(ii) 'P × Q' means 'P is sister of Q'. 29. What is the surname of Leela ?
(iii) 'P + Q' means 'P is brother of Q'. (a) Sharma (b) Mohan (c) Mishra
(iv) 'P – Q' means 'P is mother of Q'. (d) None of these (e) None of these
21. How is T related to S in the expression 'T × R + V ¸ S'? 30. How is Shivendar related to Roma’s father ?
(a) Sister (b) Mother (c) Aunt (a) Son-in-law (b) Cousin (c) Son
(d) Uncle (e) None of these (d) Grandson (e) None of these

16. a b c d e 17. a b c d e 18. a b c d e 19. a b c d e 20. a b c d e


RESPONSE
21. a b c d e 22. a b c d e 23. a b c d e 24. a b c d e 25. a b c d e
GRID 26. a b c d e 27. a b c d e 28. a b c d e 29. a b c d e 30. a b c d e
y
o
u
rs
m
a
h
b
7

o
o
b
Direction and

.w
o
rd
p
Distance

re
s
s
.c
o
m
Max. Marks : 30 No. of Qs. 30 Time : 20 min. Date : ........./......../................
1. At my house I am facing east, then I turn left and go 10 m, (I) Kunal is 40 metres to the right of Atul.
then turn right and go 5 m, and then I go 5 m towards the (II) Dinesh is 60 metres to the south of Kunal
south and from there 5 m towards the west. In which direction (III) Nitin is 25 metres to the west of Atul.
am I from my house ? (IV) Prashant is 90 metres to the north of Dinesh.
(a) East (b) West (c) North 8. Who is to the north-east of the person who is to the left of
(d) South (e) None of these Kunal?
2. My friend and I started walking simultaneously towards each (a) Dinesh (b) Nitin (c) Atul
other from two places 100 m apart. After walking 30 m, my (d) None of these (e)
friend turns left and goes 10 m, then he turns right and goes 9. If a boy walks from Nitin, meets Atul followed by Kunal,
20 m and then turns right again and comes back to the road Dinesh and Prashant, then how many metres has he walked
on which he had started walking. If we walk with the same if he has travelled the straight distance all through?
speed, what is the distance between us at this point of time? (a) 155 metres (b) 185 metres
(a) 50 m (b) 20 m (c) 30 m (c) 215 metres (d) 245 metres
(d) 40 m (e) None of these (e) None of these
3. A watch reads 4:30 O’clock. If minute hand points towards 10. A square field ABCD of side 90 m is so located that its
the East, in which direction does the hour hand point ? diagonal AC is from north to south and the corner B is to the
(a) North-East (b) South-East west of D. Rohan and Rahul start walking along the sides
(c) North-West (d) North from B and C respectievely in the clockwise and anti-
(e) None of these clockwise directions with speeds of 8 km/hr and 10 km/hr.
4. A man travels 3 km to the west, turns left and goes 3 km, Where shall they cross each other the second time ?
turns right and goes 1 km, again turns right and goes 3 km. (a) On AD at a distance of 30 m from A
How far is he from the starting point ? (b) On BC at a distance of 10 m from B
(a) 7 km (b) 6 km (c) 5 km (c) On AD at a distance of 30 m from D
(d) 4 km (e) None of these (d) On BC at a distance of 10 m from C
5. A and B start walking in opposite directions. A covers 3 km and (e) None of these
B covers 4 km. Then A turns right and walks 4 km while B turns 11. One morning after sunrise, Reeta and Kavita were talking to
left and walks 3 km. How far is each from the starting point ? each other face to face at Tilak square. If Kavita’s shadow
(a) 5 km (b) 4 km (c) 10 km was exactly to the right of Reeta, then which direction was
(d) 8 km (e) None of these
Kavita facing?
6. Ram walks 10 m south from his house, turns left and walks
(a) North (b) South (c) East
25 m, again turns left and walks 40 m, then turns right and
(d) Data inadequate (e) None of these
walks 5 m to reach to the school. In which direction the
12. I am facing west. I turn 45° in the clockwise direction and
school is from his house ?
then 180° in the same direction and then 270° anticlockwise.
(a) South-west (b) North-east (c) East
Which direction am I facing now?
(d) North (e) None of these
(a) South-West (b) South (c) West
7. A river flows west to east and on the way turns left and goes
(d) North-west (e) None of these
in a semicircle round a hillock, and then turns left in a right-
13. Bhavika and Sunaina start simultaneously towards each
angle. In which direction is the river finally flowing ?
other from two places 100m apart. After walking 30 m Bhavika
(a) North (b) South (c) East
turns left and goes 10 m, then she turns right and goes 20 m
(d) West (e) None of these
and then turns right again and comes back to the road on
which she had started walking. If both Bhavika and Sunaina
DIRECTIONS (Qs. 8 & 9) : Study the information given below walk with the same speed, what is the distance between
carefully to answer these questions: them at this point of time?
On a playing ground, Dinesh, Kunal, Nitin, Atul and Prashant are (a) 70 metres (b) 10 metres (c) 40 metres
standing, as described below, facing the North. (d) 20 metres (e) None of these

1. a b c d e 2. a b c d e 3. a b c d e 4. a b c d e 5. a b c d e
RESPONSE 6. a b c d e 7. a b c d e 8. a b c d e 9. a b c d e 10. a b c d e
GRID 11. a b c d e 12. a b c d e 13. a b c d e
y
o
u
rs
m
a
h
14 SPEED TEST 7

b
o
o
14. A man starts from a point and moves 3 km north, then turns to (a) North (b) South (c) West

b
west and goes 2 km. He turns north and walks 1 km and then (d) East (e) North-West

.w
moves 5 km towards east. How far is he from the starting point? 23. Town D is to the West of town M. Town R is to the South of

o
(a) 11 km (b) 10 km (c) 5 km town D. Town K is to the East of town R. Town K is towards

rd
(d) 8 km (e) None of these which direction of town D?

p
re
15. Starting from Point X, Joy walked 15 metres towards West. (a) South (b) East

s
He turned left and walked 20 metres. He again turned left (c) North-East (d) South-East

s
.c
and walked 15 metres. After which he turned right and (e) None of these

o
walked for another 12 metres. How far is Joy from point X if 24. Mohan walked 30 metres towards South, took a left turn

m
he is facing North at present? and walked 15 metres. He then took a right turn and walked
(a) 27 m (b) 35 m (c) 32 m 20 metres. He again took a right turn and walked 15 metres.
(d) 42 m (e) None of these How far is he from the starting point?
16. Town D is 12 km towards the North of town A. Town C is 15 (a) 95 metres (b) 50 metres
km towards the West of town D. Town B is 15 km towards (c) 70 metres (d) Cannot be determined
the West of town A. How far and in which direction is town (e) None of these
B from town C? 25. W walked 40 metres towards West, took a left turn and
(a) 15 km towards North (b) 12 km towards North walked 30 metres. He then took a right turn and walked 20
(c) 3 km towards South (d) 12 km towards South metres, He again took a right turn and walked 30 metres.
(e) cannot be determined How far was he from the starting point?
17. Rahul started from point A and travelled 8 km towards the (a) 70 metres (b) 60 metres
North to point B, he then turned right and travelled 7 km to (c) 90 metres (d) Cannot be determined
point C, from point C he took the first right and drove 5 km (e) None of these
to point D, he took another right and travelled 7 km to point 26. Town D is 13 km towards the East of town A. A bus starts
E and finally turned right and travelled for another 3 km to from town A, travels 8 km towards West and takes a right
point F. What is the distance between point F and B? turn. After taking the right turn, it travels 5 km and reaches
(a) 1 km (b) 2 km (c) 3 km town B. From town B the bus takes a right turn again, travels
(d) 4 km (e) None of these 21 km and stops. How far and towards which direction must
18. Meghna drives 10 km towards South, takes a right turn and the bus travel to reach town D?
drives 6 km. She then takes another right turn, drives 10 km (a) 13 km towards South (b) 5 km towards West
and stops. How far is she from the starting point? (c) 21 km towards South (d) 5 km towards South
(a) 16 km (b) 6 km (c) 4 km (e) None of these
(d) 12 km (e) None of these 27. A person travels 12 km due North, then 15 km due East, after
19. Vikas walked 10 metres towards North, took a left turn and that 15 km due West and then 18 km due South. How far is
walked 15 metres, and again took a left turn and walked 10 he from the starting point?
metres and stopped walking. Towards which direction was (a) 6 km (b) 12 km (c) 33 km
he facing when he stopped walking? (d) 60 km (e) 65 km
(a) South (b) South-West 28. In a meeting, the map of a village was placed in such a
(c) South-East (d) Cannot be determined manner that south-east becomes north, north-east becomes
(e) None of these west and so on. What will south become?
(a) North (b) North-east (c) North-west
DIRECTIONS (Q. 20 & 21) : Study the following information (d) West (e) South
carefully to answer these questions. 29. A school bus driver starts from the school, drives 2 km
A vehicle starts from point P and runs 10 km towards North. It towards North, takes a left turn and drives for 5 km. He then
takes a right turn and runs 15 km. It now runs 6 km after taking a takes a left turn and drives for 8 km before taking a left turn
left turn. It finally takes a left turn, runs 15 km and stops at point Q. again and driving for further 5 km. The driver finally takes a
20. How far is point Q with respect to point P? left turn and drives 1 km before stopping. How far and
(a) 16 km (b) 25 km (c) 4 km towards which direction should the driver drive to reach the
(d) 0 km (e) None of these school again?
21. Towards which direction was the vehicle moving before it (a) 3 km towards North (b) 7 km towards East
stopped at point Q? (c) 6 km towards South (d) 6 km towards West
(a) North (b) East (c) South (e) 5 km towards North
(d) West (e) North-East 30. Roma walked 25 metre towards south, took a right turn and
22. Raman starts from point P and walks towards South and walked 15 metre. She then took a left turn and walked 25
stops at point Q. He now takes a right turn followed by a left meter. Which direction is she now from her starting point?
turn and stops at point R. He finally takes a left turn and (a) South-east (b) South (c) South-west
stops at point S. If he walks 5 km before taking each turn, (d) North-west (e) None of these
towards which direction will Raman have to walk from point
S to reach point Q?
14. a b c d e 15. a b c d e 16. a b c d e 17. a b c d e 18. a b c d e
RESPONSE 19. a b c d e 20. a b c d e 21. a b c d e 22. a b c d e 23. a b c d e
GRID 24. a b c d e 25. a b c d e 26. a b c d e 27. a b c d e 28. a b c d e
29. a b c d e 30. a b c d e
y
o
u
rs
m
a
h
b
8

o
8

o
b
.w
Time, Sequence and

o
rd
p
re
Ranking Test

s
s
.c
o
m
Max. Marks : 32 No. of Qs. 32 Time : 20 min. Date : ........./......../................

1. At an enquiry office at a railway station, a passenger was DIRECTIONS (Qs. 8-11) : Read the following information carefully
told that a train for New Delhi has left 15 minutes ago, but and answer the question that follow:
after every 45 minutes a train leaves for New Delhi. The next Six boys A, B, C, D, E and F are marching in a line. They are
train will leave at 8.30 p.m. At what time was this information arranged according to their height, the tallest being at the back
given to the passanger ? and the shortest in front,
(a) 7.45 pm (b) 8.00 pm (c) 8.15 pm F is between B and A.
E is shorter than D but taller than C who is taller than A.
(d) 8.05 pm (e) None of these
E and F have two boys between them.
2. A watch is a minute slow at 1 p.m. on Tuesday and 2 minutes A is not the shortest among them.
fast at 1 p.m. on Thursday. When did it show the correct time?
8. Where is E ?
(a) 1:00 a.m. on Wednesday (a) Between A and B (b) Between C and A
(b) 5:00 a.m. on Wednesday (c) Between D and C (d) In front of C
(c) 1:00 p.m. on Wednesday (e) None of these
(d) 5:00 p.m. on Wednesday 9. Who is the tallest ?
(e) None of these (a) B (b) D (c) F
3. Reaching the place of meeting on Tuesday 15 minutes before (d) A (e) None of these
8.30 hours, Anuj found himself half an hour earlier than the 10. If we start counting from the shortest, which boy is fourth
man who was 40 minutes late. What was the scheduled time one in the line ?
(a) E (b) A (c) D
of the meeting?
(d) C (e) None of these
(a) 8.00 hrs (b) 8.05 hrs (c) 8.15 hrs 11. Who is the shortest ?
(d) 8.45 hrs (e) None of these (a) C (b) D (c) B
4. A clock gaining 2 min every hour was synchronised at midnight (d) F (e) None of these
with a clock losing 1 min every hour. How many minutes behind 12. Five newly born babies were weighed by the doctor. In her
will its minute hand be at eleven the following morning ? report, she stated that child A is lighter than child B, child C
(a) 23 (b) 27 (c) 22 is lighter than D and child B is lighter than child D, but
(d) None of these (e) heavier than child E. Which child is the heaviest?
5. Samant remembers that his brother’s birthday is after fifteenth (a) E (b) D (c) C
(d) A (e) None of these
but before eighteenth of February, whereas his sister
13. Thirty-six vehicles are parked in a parking in a single row.
remembers that her brother’s birthday is after sixteenth but After the first car, there is one scooter. After the second car,
before nineteenth of February. On which date of February is there are two scooters. After the third car, there are three
Samant’s brother’s birthday? scooters and so on. Work out the number of scooters in the
(a) 18th (b) 16th (c) 19th second half of the row.
(d) 17th (e) None of these (a) 10 (b) 12 (c) 15
6. A is shorter than B but much taller than E. C is the tallest and (d) 17 (e) None of these
D is little shorter than A. Who is the shortest ? 14. In a row at a bus stop, A is 7th from the left and B is 9th from
(a) A (b) E (c) C the right. Both of them interchange their positions and thus
A becomes 11th from the left. How many people are there in
(d) D (e) None of these
that row?
7. In a class of 35 students Kiran is placed 7th from the bottom (a) 18 (b) 19 (c) 20
whereas Sohan is placed 9th from the top. Mohan is placed exactly (d) 21 (e) None of these
in between the two. What is Kiran’s position from Mohan? 15. A, B, C, D and E when arranged in descending order of their
(a) 10 (b) 11 (c) 13 weight from the top, A becomes third, E is between D and A while
(d) 12 (e) None of these C and D are not at the top. Who among them is the second?
(a) C (b) B (c) E
(d) Data inadequate (e) None of these
1. a b c d e 2. a b c d e 3. a b c d e 4. a b c d e 5. a b c d e
RESPONSE 6. a b c d e 7. a b c d e 8. a b c d e 9. a b c d e 10. a b c d e
GRID 11. a b c d e 12. a b c d e 13. a b c d e 14. a b c d e 15. a b c d e
y
o
u
rs
m
a
h
16 SPEED TEST 8

b
o
o
DIRECTIONS (Qs. 16 & 17) : Read the relationships given 24. In a row of twenty-five children facing North, W is fifth to

b
.w
below to answer the questions that follow. the right of R, who is sixteenth from the right end of the row.
What is W's position from the right end of the row?

o
Archana is elder than Suman. Arti is elder than Archana but

rd
younger than Kusum. Kusum is elder than Suman. Suman is (a) Eleventh (b) Tenth (c) Twelfth

p
younger than Arti and Gita is the eldest. (d) Data inadequate (e) None of these

re
25. Seema's watch is 6 minutes fast and the train, which should

s
16. Who is the youngest ?

s
have arrived at 7 p.m. was 14 minutes late. What time is it by

.c
(a) Archana (b) Suman (c) Arti

o
(d) Kusum (e) None of these Seema's watch when the train arrived?

m
17. Age wise, who is in the middle ? (a) 7 : 05 pm (b) 7 : 30 pm (c) 7 : 01 pm
(a) Suman (b) Archna (c) Arti (d) 7 : 31 pm (e) None of these
(d) Kusum (e) None of these 26. In a row of children facing North, Ritesh is twelfth from the
18. Five boys took part in a race. Raj finished before Mohit but left end. Sudhir who is twenty-second from the right end is
behind Gaurav. Ashish finished before Sanchit but behind fourth to the right of Ritesh. Total how many children are
Mohit. Who won the race? there in the row?
(a) Raj (b) Gaurav (c) Mohit (a) 35 (b) 36 (c) 37
(d) Ashish (e) None of these (d) 34 (e) None of these
19. Five men A, B, C, D and E read newspaper. The one who
27. Among A, B, C, D and E each reaching school at a different
reads first gives it to C. The one who reads last had taken it
time, C reaches before D and A and only after B. E is not the
from A. E was not the first or the last to read. There were two
readers between B and A. last to reach school. Who among them reached school last?
B passed the newspaper to whom? (a) D (b) A (c) C
(a) A (b) C (c) D (d) Data inadequate (e) None of these
(d) E (e) None of these 28. M is older than R. Q is younger than R and N. N is not as old
20. In a row of children, Bhushan is seventh from the left and as M. Who among M, N, R and Q is the oldest?
Motilal is fourth from the right. When Bhushan and Motilal (a) M (b) R (c) M or R
exchange positions, Bhushan will be fifteenth from the left. (d) Data inadequate (e) None of these
What will be Motilal’s position from the right ? 29. Akshay is 16th from the left end in the row of boys and Vijay
(a) Eighth (b) Fourth (c) Eleventh is 18th from the right end. Avinash is 11th from Akshay
(d) Twelfth (e) None of these towards the right end and 3rd from Vijay towards the right
21. In a queue I am the last person while my friend is seventh end. How many boys are there in the row?
from the front. If the person exactly between me and my (a) Data inadequate (b) 42 (c) 40
friend is on the 23rd position from the front, what is my
(d) 48 (e) None of these
position in the queue ?
30. Geeta is senior to Shilpa but not to Deepa. Gayatri is junior
(a) 37 (b) 36 (c) 38
to Reepa. No one is senior to Fatima. Who is most junior?
(d) 39 (e) None of these
22. Pratap correctly remembers that his mother's birthday is (a) Shilpa (b) Geeta (c) Gayatri
before twentythird April but after nineteenth April, whereas (d) Data inadequate (e) None of these
his sister correctly remembers that their mother's birthday is 31. Among M, N, P, R and T each one has secured different
not on or after twentysecond April. On which day in April is marks in an examination. R secured more marks than M and
definitely their mother's birthday? T. N secured less marks than P. Who among them secured
(a) Twentieth third highest marks?
(b) Twentyfirst (a) N (b) R (c) M
(c) Twentieth or Twentyfirst (d) T (e) Data inadequate
(d) Cannot be determined 32. In a column of girls Kamal is 11th from the front. Neela is
(e) None of these 3 places ahead of Sunita who is 22nd from the front.
23. In a row of forty children, R is eleventh from the right end How many girls are there between Kamal and Neela in the
and there are fifteen children between R and M. What is M's column?
position from the left end of the row?
(a) Six (b) Eight
(a) Fourteenth (b) Fifteenth
(c) Seven (d) Cannot be determined
(c) Thirteenth (d) Cannot be determined
(e) None of these (e) None of these

16. a b c d e 17. a b c d e 18. a b c d e 19. a b c d e 20. a b c d e


RESPONSE 21. a b c d e 22. a b c d e 23. a b c d e 24. a b c d e 25. a b c d e
GRID 26. a b c d e 27. a b c d e 28. a b c d e 29. a b c d e 30. a b c d e
31. a b c d e 32. a b c d e
y
o
u
rs
m
a
h
9

b
8

o
Sitting
9

o
b
.w
o
rd
Arrangement

p
re
s
s
.c
o
m
Max. Marks : 30 No. of Qs. 30 Time : 20 min. Date : ........./......../................

1. A, B, C, D, E, F and G are sitting in a line facing east. C is 7. Who is to the immediate left of H?
immediate right of D. B is at the extreme ends and has E as his (a) W (b) T (c) R
neighbour. G is between E and F. D is sitting third from the (d) Data inadequate (e) None of these
south end. Who are the persons sitting at the extreme ends? (e) None of these
(a) A and E (b) A and B (c) F and B 8. Who is third to the right of H ?
(d) C and D (e) None of these (a) M (b) D (c) Y
DIRECTIONS (Qs. 2-4) : Read the following statements to answer (d) R (e) None of these
the questions that follow. (e) None of these
Nine cricket fans are watching a match in a stadium. Seated in one 9. Who is third to the right of D?
row, they are – J, K, L, M, N, O, P, Q and R. L is at the right of M and (a) M (b) R (c) W
at third place to the right of N. K is at one end of the row. Q is (d) M (e) None of these
immediately next to O and P. O is at the third place to the left of K. (e) None of these
J is right next to left of O. 10. What is Y's position with respect toT?
2. Who is sitting in the centre of the row ? (a) Third to the right (b) Fourth to the left
(a) L (b) O (c) J (c) Third to the left (d) Second of the left
(d) Q (e) None of these (e) None of these
3. Who is at the other end of the row ? DIRECTIONS (Q.11-15): Study the following information carefully
(a) R (b) J (c) P to answer these questions :
(d) N (e) None of these Eight friends J, K, L, M, N, O, P and Q are sitting around a circle
4. Which of the following statements is true ? facing the centre. J is not the neighour of N. L is third to the right
(a) R and P are neighbours. of K. Q is second to the left of N who is next to the right of L. O is
(b) There is one person between L and O. not the neighbour of N or K and is to the immediate left of P.
(c) M is at one extreme end.
11. Which of the following is the correct position of L?
(d) N is two seats away from J.
(a) To the immediate right of N
(e) None of these
(b) To the immediate right of Q
5. A, B, C, D, E and F are sitting around a round table. A is
between E and F, E is opposite D, and C is not in either of the (c) To the immediate left of N
neighbouring seats of E. Who is opposite to B ? (d) To the immediate left of Q
(a) F (b) C (c) D (e) None of these
(d) None of these (e) 12. Which of the following pair of persons represent O's
6. Four girls (G1, G2, G3, G4) and three boys (B1, B2, B3) are to neighbours?
sit for a dinner such that no two boys should sit together (a) L&N (b) P&K (c) M&P
nor two girls. If they are successively sitting, what is the (d) N&P (e) None of these
position of B2 and G3? 13. Which of the following groups has the first person sitting
(a) 5th and 6th (b) 4th and 5th between the other two persons?
(c) 3rd and 4th (d) 2nd and 3rd (a) PKJ (b) JQL (c) QNL
(e) None of these (d) LMN (e) None of these
14. Who is to the immediate right of K?
DIRECTIONS (Q. 7-10) : Study the following information carefully (a) J (b) P
and answer the questions given below: (c) Q (d) Cannot be determined
W, Y, T, M, R, H and D are seven persons, sitting around a circle (e) None of these
facing the centre. T is fourth to the right of M who is second to the 15. Who is to the immediate left of O?
right of R. W is third to the left of R. H is not an immediate neighbour (a) P (b) L (c) Q
of M. D is not an immediate neighbour of W.
(d) M (e) None of these

1. a b c d e 2. a b c d e 3. a b c d e 4. a b c d e 5. a b c d e
RESPONSE 6. a b c d e 7. a b c d e 8. a b c d e 9. a b c d e 10. a b c d e
GRID 11. a b c d e 12. a b c d e 13. a b c d e 14. a b c d e 15. a b c d e
y
o
u
rs
m
a
h
18 SPEED TEST 9

b
o
o
DIRECTIONS (Q. 16 - 20): These questions are based on the 24. Who sits between G & D?

b
.w
basis of following information. Study it carefully and answer the (a) H (b) D (c) F

o
questions. (d) E (e) None of these

rd
Eight executives J, K, L, M, N, O, P and Q are sitting around a 25. Which of the following is the correct position of B with

p
re
circular table for a meeting. J is second to the right of P who is respect to H?

s
I Second to the right

s
third to the right of K. M is second to the left of O who sits

.c
between P and J. L is not a neighbour of K or N. II Fourth to the right

o
m
III Fourth to the left
16. Who is to the immediate left of L?
IV Second to the left
(a) Q (b) O (c) K
(a) Only I (b) Only II (c) Only III
(d) N (e) None of these
(d) Both II & III (e) None of these
17. Who is to the immediate left of K?
(a) N (b) J DIRECTIONS (Q. 26 - 30) : Study the following information
(c) Q (d) Cannot be determined carefully and answer the given questions:
(e) None of these Eight friends P, Q, R, S, T, V, W and Y are sitting around a square
18. Which of the following is the correct position of N? table in such a way that four of them sit at four corners of the
(a) Second to the right of K square while four sit in the middle of each of the four sides. The
(b) To the immediate left of K ones who sit at the four corners face the centre while those who
(c) To the immediate right of M sit in the middle of the sides face outside.
(d) To the immediate right of K P, who faces the centre, sits third to the right of V. T, who faces the
(e) None of these centre, is not an immediate neighbour of V. Only one person sits
19. Who is third to the right of P ? between V and W. S sits second to right of Q. Q faces the centre.
(a) L (b) J (c) Q R is not an immediate neighbour of P.
(d) N (e) None of these 26. Who sits second to the left of Q?
20. Which of the following groups of persons have the first (a) V (b) P (c) T
person sitting between the other two? (d) Y (e) Cannot be determined
(a) PJO (b) OPJ (c) OPM 27. What is the position of T with respect to V?
(d) MPO (e) None of these (a) Fourth to the left (b) Second to the left
(c) Third to the left (d) Third to the right
DIRECTIONS (21-25) : Study the following information
(e) Second to the right
carefully to answer these questions.
28. Four of the following five are alike in a certain way and so
A, B, C, D, E, F, G and H are sitting around a circle facing the form a group. Which is the one that does not belong to that
centre. F is third to the right of C and second to the left of H. D is group?
not an immediate neighbour of C or H. E is to the immediate right (a) R (b) W (c) V
of A, who is second to the right of G. (d) S (e) Y
21. Who is second to the left of C? 29. Which of the following will come in place of the question
(a) A (b) B (c) E mark based upon the given seating arrangement?
(d) D (e) None of these WP TR QW RS ?
22. Who is to the immediate right of C? (a) Y T (b) VY (c) V Q
(a) E (b) B (c) D (d) P Y (e) Q V
(d) B or D (e) None of these 30. Which of the following is true regarding R?
23. Which of the following pairs of persons has first person (a) R is an immediate neighbour of V.
sitting to the right of the second person? (b) R faces the centre.
(a) CB (b) AE (c) FG (c) R sits exactly between T and S.
(d) HA (e) DB (d) Q sits third to left of R
(e) None of these

16. a b c d e 17. a b c d e 18. a b c d e 19. a b c d e 20. a b c d e


RESPONSE
GRID 21. a b c d e 22. a b c d e 23. a b c d e 24. a b c d e 25. a b c d e
26. a b c d e 27. a b c d e 28. a b c d e 29. a b c d e 30. a b c d e
y
o
u
rs
m
a
h
b
o
10

o
8

b
.w
Analytical

o
rd
p
re
Puzzle - I

s
s
.c
o
m
Max. Marks : 30 No. of Qs. 30 Time : 20 min. Date : ........./......../................

DIRECTIONS (Qs. 1 - 5) : The questions below are based on the DIRECTIONS (Qs. 8 - 12) : Study the information given below to
following statements. answer the questions that follow :
Asha and Charu are good in Mathematics and Athletics. Deepa (i) There is a family of 5 persons A, B, C, D and E.
and Asha are good in Athletics and Studies, Charu and Beena are (ii) They are working as a doctor, a teacher, a trader, a lawyer
good in General Knowledge and Mathematics. Deepa, Beena and and a farmer.
Ela are good in Studies and General Knowledge. Ela and Deepa (iii) B, an unmarried teacher, is the daughter of A.
are good in Studies and Arts. (iv) E, a lawyer, is the brother of C.
1. Who is good in Studies, General Knowledge, Athletics & Arts? (v) C is the husband of the only married couple in the family.
(a) Asha (b) Beena (c) Charu (vi) Daughter-in-law of A is a doctor.
(d) Deepa (e) None of these 8. Which of the following is a group of female members in the
2. Who is good in Studies, General Knowledge and Mathematics? family ?
(a) Asha (b) Beena (c) Charu (a) A and D (b) D and E (c) A, C and E
(d) Deepa (e) None of these (d) B and D (e) None of these
3. Who is good in Studies, Mathematics and Athletics ? 9. Which of the following is the married couple ?
(a) Asha (b) Beena (c) Charu (a) A and B (b) C and D (c) A and D
(d) Deepa (e) None of these (d) B and C (e) None of these
4. Who is good in Athletics, General Knowledge and Mathematics? 10. Which of the following is a group of male members in the
(a) Asha (b) Beena (c) Charu family ?
(d) Deepa (e) None of these (a) A, B and C (b) B and D (c) A, C and E
5. Who is good in Studies, General Knowledge and Arts but (d) A, C and D (e) None of these
not in Athletics ? 11. Who is the doctor in the family ?
(a) Asha (b) Beena (c) Charu (a) A (b) B (c) C
(d) Ela (e) None of these (d) D (e) None of these
DIRECTIONS (Qs. 6 & 7) : Study the information given below 12. Who is the trader in the family ?
carefully and then answer the questions that follow : (a) A (b) B (c) C
Students joining certain university can choose from among seven (d) D (e) None of these
courses : Biology, Astronomy, English, Mathematics, History, DIRECTIONS (Qs. 13 - 16) : Study the infomation given below to
Chemistry and Psychology. answer these questions :
A student cannot take both English and Astronomy. There are four friends A, B, C and D. One of them is a Cricketer and
Chemistry is a prerequisite for Biology and cannot be taken studies Chemistry and Biology. A and B play Football. Both
concurrently with Biology. A student must take at least two of the Football players study Maths. D is a Boxer. One football player
three courses, English, History and Psychology. No course may
also studies physics. The Boxer studies Maths and Accounts. All
be repeated.
the friends study two subjects each and play one game each.
6. What is the maximum number of courses that can a student
13. Who is the Cricketer ?
can take without violating any of the conditions given above?
(a) A (b) B (c) C
(a) 3 (b) 4 (c) 5
(d) D (e) None of these
(d) 6 (e) None of these
14. Who studies Accounts and plays Football ?
7. If a student is taking Astronomy and Mathematics and
(a) A alone (b) B alone (c) D
wishes to take four courses, then in how many different
ways can the other two courses be taken? (d) A or B (e) None of these
(a) One (b) Two (c) Three 15. Who studies Physics?
(d) Four (e) None of these (a) A or B (b) A alone (c) B alone
(d) D (e) None of these
1. a b c d e 2. a b c d e 3. a b c d e 4. a b c d e 5. a b c d e
RESPONSE 6. a b c d e 7. a b c d e 8. a b c d e 9. a b c d e 10. a b c d e
GRID 11. a b c d e 12. a b c d e 13. a b c d e 14. a b c d e 15. a b c d e
y
o
u
rs
m
a
h
20 SPEED TEST 10

b
o
o
16. How many games are played and subjects studied by the 22. Who among the teachers was teaching maximum number of

b
four friends ? subjects?

.w
(a) 1 game and 4 subjects (a) A (b) C (c) B

o
rd
(b) 2 games and 3 subjects (d) D (e) None of these

p
(c) 3 games and 4 subjects 23. Which of the following pairs was teaching both Geography

re
(d) 3 games and 5 subjects and Hindi?

s
s
(e) None of these (a) A and B (b) C and A (c) B and C

.c
(d) None of these (e)

o
DIRECTIONS (Qs. 17 - 21) : Study the information given below

m
to answer these questions : 24. More than two teachers were teaching which subject?
(i) Six plays A, B, C, D, E and F are to be organised from (a) History (b) French (c) Hindi
Monday to Saturday, i.e from 5th to 10th-one play each day. (d) Geography (e) None of these
(ii) There are two plays between C and D and one play between 25. D, B and A were teaching which of the following subjects?
A and C. (a) English only (b) Hindi only
(iii) There is one play between F and E and E is to be organised (c) Hindi and English (d) English and Geography
before F. (e) None of these
(iv) B is to be organised before A, not necessarily immediately. DIRECTIONS (26-30) : Study the following information carefully
(v) The organisation does not start with B and answer the given questions.
17. The organisation would start from which play ? Seven flights namely those of Jet Airways, British Airways, Delta,
(a) A (b) F Quantas, Emirates, Lufthansa and Air India are scheduled to fly to
(c) D (d) Cannot be determined London. There is only one flight to London on each of the seven
(e) None of these days of the week, starting from Monday and ending on Sunday.
18. On which date the play E is to be organised ? Delta flies on Wednesday. Air India flies the day next to British
(a) 5th (b) 7th Airways. British Airways does not fly on Monday or Friday. Two
(c) 6th (d) Cannot be determined airlines fly between the days British Airways and Emirates fly.
(e) None of these Emirates does not fly on Sunday. Quantas flies a day before
19. The organisation would end with which of the following Lufthansa.
plays? 26. On which of the following days does Jet Airways fly ?
(a) A (b) D (a) Friday (b) Sunday (c) Tuesday
(c) B (d) Cannot be determined (d) Thursday (e) None of these
(e) None of these 27. How many flights fly between Lufthansa and Delta ?
20. On which day the play B is organised ? (a) None (b) One (c) Two
(a) Tuesday (b) Friday (d) Three (e) Five
(c) Thursday (d) None of these 28. Which of the following flights flies on Friday ?
(e) None of these (a) Air India (b) Quantas (c) Emirates
21. Which of the following is the correct sequence of organising (d) Lufthansa (e) Jet Airways
plays? 29. If Delta postpones its flight to Sunday owing to some technical
(a) AECFBD (b) DFECBA (c) BDEFCA reasons and all the flights scheduled for Thursday to Sunday
(d) None of these (e) are now made to take off a day ahead of the schedule, which of
DIRECTIONS (Qs. 22-25) : Read the following information to the following flights would now fly on Friday?
answer these questions : (a) Lufthansa (b) Jet Airways
In a school, there were five teachers. A and B were teaching Hindi (c) British Airways (d) Air India
and English. C and B were teaching English and Geography. D (e) Quantas
and A were teaching Mathematics and Hindi. E and B were teaching 30. If Emirates is related to British Airways and Delta is related
History and French. to Quantas in a certain way based upon the given flight
schedule, then Jet Airways will be related to which of the
following based upon the same relationship ?
(a) Lufthansa (b) Quantas (c) Delta
(d) Air India (e) None of these

16. a b c d e 17. a b c d e 18. a b c d e 19. a b c d e 20. a b c d e


RESPONSE
GRID 21. a b c d e 22. a b c d e 23. a b c d e 24. a b c d e 25. a b c d e
26. a b c d e 27. a b c d e 28. a b c d e 29. a b c d e 30. a b c d e
y
o
u
rs
m
a
h
11

b
o
o
Analytical

b
.w
o
rd
Puzzle - II

p
re
s
s
.c
o
m
Max. Marks : 30 No. of Qs. 30 Time : 20 min. Date : ........./......../................

DIRECTIONS (Q.1- 5) : Read the following passage carefully (a) None (b) One (c) Two
and answer these questions given below it. (d) Three (e) Four
7. Which of the following plays is scheduled on Saturday?
A group of seven friends A, B, C, D, E, F and G, work as Economist, (a) Q (b) W (c) R
Agriculture Officer, IT Officer, Terminal Operator, Clerk, Forex (d) S (e) T
Officer and Research Analyst, for Banks L, M, N, P, Q, R and S, but 8. R is related to S in a certain way. In the same way P is
not necessarily in the same order. C works for Bank N and is related to V based on the given schedule. Which of the
neither Research Analyst nor a Clerk. E is an IT Officer and works following is W related to the following the same pattern?
for Bank R.A works as Forex Officer and does not work for Bank L (a) P (b) Q (c) R
or Q. The one who is an Agricultural Officer works for Bank M. (d) T (e) Cannot be determined
The one who works for Bank L works as a Terminal Operator. F
9. On which of the following days is Play W scheduled?
works for Bank Q. G works for Bank P as a Research Analyst. D is
not an Agricultural Officer. (a) Monday (b) Tuesday
1. Who amongst the following works as an Agriculture Officer? (c) Wednesday (d) Saturday
(a) C (b) B (c) F (e) Cannot be determined
(d) D (e) None of these 10. Which of the following plays is scheduled on Friday?
2. What is the profession of C? (a) R (b) T (c) Q
(a) Terminal Operator (b) Agriculture Officer (d) W (e) S
(c) Economist (d) Cannot be determined
(e) None of these DIRECTIONS (Q.11-Q.15) : Study the following information
3. For which bank does B work? carefully and answer the questions given below:
(a) M (b) S (c) L P, Q, R, S, T, V and W are seven friends working in a call centre.
(d) Either M or S (e) None of these Each of them has different day offs in a week from Monday to
4. What is the profession of the person who works for Bank S? Sunday not necessarily in the same order. They work in three
(a) Clerk (b) Agriculture Officer different shifts I, II and III with at least two of them in each shift.
(c) Terminal Operator (d) Forex Officer R works in shift II and his day off is not Sunday. P's day off
(e) None of these is Tuesday and he does not work in the same shift with either Q or
5. Which of the following combinations of person, profession W. None of those who work in shift I has day off either on
and bank is correct? Wednesday or on Friday. V works with only T in shift III. S's day
(a) A - Forex Officer - M off is Sunday. V's day off is immediate next day of that of R's day
(b) D - Clerk - L off. T's day off is not on Wednesday. W's day off is not on the
(c) F - Agriculture Officer - Q previous day of P's day off. S works in shift I. Q does not work in
(d) B - Agriculture Officer - S the same shift with R and his day off is not on Thursday.
(e) None of these 11. Which of the following is W's day off?
(a) Tuesday (b) Monday (c) Saturday
DIRECTIONS (Q.6-10) : Study the following information to (d) Data inadequate (e) None of these
answer the given questions. 12. Which of the following is R's day off?
Each of seven plays viz. P, Q, R, S, T, V and W are scheduled to be (a) Friday (b) Thursday (c) Tuesday
staged on a different day of a week starting from Monday and (d) Wednesday (e) None of these
ending on Sunday of the same week. Play V is scheduled on 13. Which of the following groups of friends work in shift II?
Thursday. Two plays are scheduled to be held between Play V (a) RP (b) RV (c) QWS
and Play P. Only one play is scheduled between Play T and Play S. (d) Data inadequate (e) None of these
Play T is not scheduled on the day immediately before or 14. Which of the following is Q's day off?
immediately after the day when Play V is scheduled. Play R is (a) Friday (b) Wednesday (c) Thursday
scheduled the day immediately before the day when Play W is (d) Monday (e) None of these
scheduled. Play S is not scheduled after Play Q. 15. Which of the following groups of friends work in shift I?
6. How many plays are scheduled to be staged between Play R (a) RV (b) RP (c) QWS
and Play S? (d) Data inadequate (e) None of these
1. a b c d e 2. a b c d e 3. a b c d e 4. a b c d e 5. a b c d e
RESPONSE 6. a b c d e 7. a b c d e 8. a b c d e 9. a b c d e 10. a b c d e
GRID 11. a b c d e 12. a b c d e 13. a b c d e 14. a b c d e 15. a b c d e
y
o
u
rs
m
a
h
22 SPEED TEST 11

b
o
o
DIRECTIONS (Q.16-20) : Study the following information 21. Which of the following groups of employees work in

b
Administration department?

.w
carefully and answer the questions given below:
(a) EGH (b) AF (c) BCD

o
(i) A, B, C, D, E, F, G and H are eight students, each having a (d) BGD (e) Data inadequate

rd
different height 22. In which department does E work?

p
re
(ii) D is shorter than A but taller than G. (a) Personnel (b) Marketing

s
(iii) E is taller than H but shorter than C. (c) Administration (d) Data inadequate

s
.c
(iv) B is shorter than D but taller than F. (e) None of these

o
23. Which of the following combinations of employees

m
(v) C is shorter than G.
(vi) G is not as tall as F. department-favourite sport is correct?
16. Which of the following is definitely false? (a) E-Administration-Cricket
(a) G is shorter than F (b) C is shorter than F (b) F-Personnel-Lawn Tennis
(c) F is taller than C. (d) B is taller than E. (c) H-Marketing-Lawn Tennis
(e) All are true (d) B-Administration-Table Tennis
17. If another student J, who is taller than E but shorter than G, (e) None of these
is added to the group, which of the following will be definitely 24. What is E's favourite sport?
true? (a) Cricket (b) Badminton (c) Basketball
(a) C and J are of the same height (d) Lawn Tennis (e) None of these
25. What is G's favourite sport?
(b) J is shorter than D.
(a) Cricket (b) Badminton (c) Basketball
(c) J is shorter than H. (d) Lawn Tennis (e) None of these
(d) J is taller than A.
(e) None of these DIRECTIONS (Qs. 26-30): Study the following information
18. Which of the following will definitely be the third from top carefully and answer the given questions.
when the eight students are arranged in descending order P, Q, R, S, T, V and W are seven friends, all of a different height
of height? and fatness.
(a) B (b) F (c) G (i) Q is the thinnest and tallest among them.
(d) B or G (e) Cannot be determined (ii) S is not as short as T, but shorter than R.
19. How many of them are definitely shorter than F? (iii) W is not as fat as R, but fatter than V.
(a) Three (b) Four (c) Five (iv) R and T are taller than W but shorter than R
(d) Data inadequate (e) None of these (v) V is fatter than T and the shorter among them.
20. Which of the following is redundant to answer all the above (vi) R is third among them in fatness in descending order.
questions?
(a) (ii) only 26. Who is the fattest among them?
(b) (ii) and (iii) only (a) P (b) S
(c) (iii) and (iv) only (c) Either P or S (d) Either T or S
(d) (ii) and (v) only (e) None of these
(e) All are necessary to answer the above questions. 27. Which of the given statements is not required to find out
the thickest among them?
DIRECTIONS (Q.21-Q.25) : Study the following information (a) (i) (b) (vi)
carefully and answer the questions given below: (c) (iii) and (iv) (d) (v)
A, B, C, D, E, F, G and H are eight employees of an organization (e) None of these
working in three departments, viz Personnel, Administration and 28. If they are made to stand in ascending order of their heights,
Marketing with not more than three of them in any department. who will come in the middle?
Each of them has a different choice of sports from football, cricket, (a) R (b) S (c) T
volleyball, badminton, lawn tennis, basketball, hockey and table (d) Data inadequate (e) None of these
tennis, not necessarily in the same order. 29. If they are made to stand in ascending order of their fatness,
D works in Administration and does not like either football or who will be the second from the last?
cricket. F works in Personnel with only A, who likes table tennis. (a) S (b) V (c) W
E and H do not work in the same department as D. C likes hockey (d) Data inadequate (e) None of these
and does not work in Marketing. G does not work in Administration 30. Who obtained the same place in height and fatness among
and does not like either cricket or badminton. One of those who them when arranged in ascending order of their height and
work in Administration likes football. The one who likes volleyball fatness?
works in Personnel. None of those who work in Administration (a) P (b) Q (c) R
likes either badminton or lawn tennis. H does not like cricket. (d) S (e) None of these

16. a b c d e 17. a b c d e 18. a b c d e 19. a b c d e 20. a b c d e


RESPONSE
21. a b c d e 22. a b c d e 23. a b c d e 24. a b c d e 25. a b c d e
GRID 26. a b c d e 27. a b c d e 28. a b c d e 29. a b c d e 30. a b c d e
y
o
u
rs
m
a
h
b
o
11
12

o
10
8

b
.w
o
Venn Diagrams

rd
p
re
s
s
.c
o
m
Max. Marks : 30 No. of Qs. 30 Time : 20 min. Date : ........./......../................
1. The diagram represent the student who are singers, dancers Rectangle represents males
and poets. Triangle represents educated
Circle represents urban
Singers P Dancers
Q Square represents civil servants
V
3. Who among the following is an educated male who is not an
U
T R urban resident?
(a) 4 (b) 5 (c) 11
S (d) 9 (e) None of these
Poets 4. Who among the following is neither a civil servant nor
Study the diagram and identify the region which represent educated but is urban and not a male?
the students who are both poets and singers but not dancer. (a) 2 (b) 3 (c) 6
(a) P + T + S (b) T (d) 10 (e) None of these
(c) T + V + R + S (d) P + T + U + S 5. Who among the following is a female urban resident and
(e) None of these
also a civil servant?
2. School children (a) 6 (b) 7 (c) 10
a (d) 13 (e) None of these
b 6. Who among the following is an educated male who hails
d c from urban, a civil servants?
f (a) 6 (b) 7 (c) 10
Artist
e (d) 13 (e) None of these
7. Who among the following is uneducated and also an urban
g
male?
Singers (a) 2 (b) 3 (c) 11
Above diagram represents school children, artist and singers. (d) 12 (e) None of these
Study the diagram and identify the region. Which represents 8. Who among the following is only a civil servant but neither
those school children who are artist not singers. a male or urban oriented and uneducated?
(a) a (b) b (a) 7 (b) 8 (c) 9
(c) f (d) e (d) 14 (e) None of these
(e) None of these 9. Who among the following is a male urban oriented and also
DIRECTIONS (Qs. 3-9) : The following question are based on a civil servant but not educated?
the diagram given below. (a) 13 (b) 12 (c) 6
(d) 10 (e) None of these
3 10 7 10. Which of the following diagram represents the relationship
among sun, moon and stars?
8 12 6 13

4
9 11 14

5
(a) (b) (c) (d) (e)

RESPONSE 1. a b c d e 2. a b c d e 3. a b c d e 4. a b c d e 5. a b c d e

GRID 6. a b c d e 7. a b c d e 8. a b c d e 9. a b c d e 10. a b c d e
y
o
u
rs
m
a
h
24 SPEED TEST 12

b
o
o
DIRECTIONS (Qs. 11-14) : Refer to the following Venn DIRECTIONS (Qs. 15-30) : In each of these questions, three

b
.w
diagram : words are related in some way. The relationship among the words

o
in question can best represents by one of the five diagram.

rd
HISTORY

p
re
s
16 GEOGRAPHY

s
.c
(a) (b)

o
14 12

m
13
9 18 20 18 9
13 16
15 (c) (d)
MATHEMATICS
19 SCIENCE

11. The number of students who took any three of the above (e)
subjects was
(a) 62 (b) 63 15. Teachers, college, students.
(c) 64 (d) 66 16. Mothers, Homo sapiens, Woman
(e) None of these 17. Cabinet, Home Minister, Minister.
12. The number of students in total, who took History or 18. Parrots, Birds, Mice.
Mathematics or Science, was 19. Professors, Researchers, Scientists.
20. Men, Rodents, Living beings.
(a) 183 (b) 190
21. Parents, Mothers, Fathers.
(c) 424 (d) 430 22. Nitrogen, Ice, Air.
(e) None of these 23. Musicians, Singers, Women
13. The number of students who took both History and 24. Elephants, Carnivore, Tiger.
Geography among other subjects was 25. Fish, Herring, Animal living in water
(a) 62 (b) 63 26. Hospital, Nurse, Patient.
(c) 65 (d) 66 27. Nose, Hand, Body.
28. Rings, Ornaments, Diamond Rings.
(e) None of these
29. Furniture, Table, Books.
14. Which subject was taken by the largest number of students?
30. Indoor games, Chess, Table tennis.
(a) Mathematics (b) Science
(c) Geography (d) History
(e) None of these

11. a b c d e 12. a b c d e 13. a b c d e 14. a b c d e 15. a b c d e


RESPONSE 16. a b c d e 17. a b c d e 18. a b c d e 19. a b c d e 20. a b c d e

GRID 21. a b c d e 22. a b c d e 23. a b c d e 24. a b c d e 25. a b c d e


26. a b c d e 27. a b c d e 28. a b c d e 29. a b c d e 30. a b c d e
y
o
u
rs
m
a
h
13

b
o
o
b
.w
Syllogism - I

o
rd
p
re
s
s
.c
o
m
Max. Marks : 30 No. of Qs. 30 Time : 20 min. Date : ........./......../................
DIRECTIONS (Q.1-5) : In each of the questions below are given DIRECTIONS (6 - 10): In each question below arc two statements
three statements followed by two conclusions numbered I and II. followed by two conclusions numbered I and II. You have to take
You have to take the given statements to be true even if they seem the two given statements to be true even if they seem to be at
to be at variance from commonly known facts. Read both of the variance from commonly known facts and then decide which of
conclusions and then decide which of the given conclusions the given conclusions logically follows from the given statements
logically follows from the given statements disregarding commonly disregarding commonly known facts.
known facts.
Give answer (a) if only conclusion 1 follows.
1. Statements: Some phones are computers. Give answer (b) if only conclusion II follows.
All computers are radios. Give answer (c) if either conclusion I or conclusion II follows.
All radios are televisions. Give answer (d) if neither conclusion I nor conclusion II follows.
Conclusions: I. All televisions are computers.
Give answer (e) if both conclusions I and II follow.
II. Some radios are phones.
6. Statements : Some windows arc grills.
(a) None follows (b) Only I follows
(c) Only II follows (d) Both I and II follow All glasses are grills.
(e) None of these Conclusions : I. All grills are windows.
2. Statements: All rings are fingers. II. At least some grills are glasses.
Some ears are fingers. 7. Statements : Some painters are artists. Some dancers are painters.
All ears are necklaces. Conclusions : I. All artists are dancers.
Conclusions: I. Some necklaces are fingers. II. All painters are dancers.
II. Some necklaces are rings. 8. Statements : All cabins are rooms.
(a) None follows (b) Only I follows All rooms are buildings.
(c) Only II follows (d) Both I and II follow Conclusions: I. All buildings are rooms.
(e) None of these II. All cabins are buildings.
3. Statements: Some bottles are cups. 9. Statements : All rings are necklaces.
Some cups are plates. No necklace is a bracelet.
No spoon is a plate. Conclusions: I. No ring is a bracelet.
Conclusions: I. Some spoons are bottles. II. All necklaces are rings.
II. No bottle is a spoon. 10. Statements : All hands are arms.
(a) None follows (b) Only I follows Some hands are muscles.
(c) Only II follows (d) Either I or II follow Conclusions: I. Some muscles are arms.
(e) None of these II. All muscles are arms.
4. Statements: All pens are erasers. DIRECTIONS (Qs. 11-15): In each question below are two
Some erasers are sharpeners. statements followed by two conclusions numbered I and II. You
Some sharpeners are staples have to take the two given statements to be true even if they seem
Conclusions: I. Some sharpeners are pens.
to be at variance from commonly known facts and then decide
II. Some staples are erasers.
which of the given conclusions logically follows from the given
(a) None follows (b) Only I follows
statements disregarding commonly known facts. Give answer
(c) Only II follows (d) Both I and II follow
(e) None of these (a) if only conclusion I follows.
5. Statements: All hills are trees. (b) if only conclusion II follows.
All trees are jungles. (c) if either conclusion I or II follows.
All jungles are bushes. (d) if neither conclusion I nor II follows.
Conclusions: I. All trees are bushes. (e) if both conclusions I and II follow.
II. Some jungles are hills. 11. Statements: No holiday is a vacation.
(a) None follows (b) Only I follows Some vacations are trips.
(c) Only II follows (d) Both I and II follow Conclusions: I. No trip is a holiday.
(e) None of these II. Some holidays are definitely not trips.
1. a b c d e 2. a b c d e 3. a b c d e 4. a b c d e 5. a b c d e
RESPONSE 6. a c d e 7. a c d e 8. a c d e 9. a c d e 10. a c d e
b b b b b
GRID 11. a b c d e
y
o
u
rs
m
a
h
26 SPEED TEST 13

b
o
o
12. Statements: Some kites are birds. Give answer (c) if either conclusion I or conclusion II follows;

b
No kite is an aeroplane. Give answer (d) if neither conclusion I nor conclusion II follows;

.w
Conclusions: I. All aeroplanes are birds. Give answer (e) if both conclusion I and conclusion II follow.

o
II. Some birds are definitely not kites

rd
21. Statements : Some exams are tests. No exam is a question.
13. Statements: All metals are plastics.

p
Conclusions : I. No question is a test.

re
All plastics are fibres. II. Some tests are definitely not exams.

s
Conclusions: I. Atleast some fibres are metals. 22. Statements : All forces are energies. All energies are

s
.c
II. Some metals are not fibres. powers. No power is heat.

o
14. Statements: All roads are streets. Conclusions : I. Some forces are definitely not powers.

m
No street is a highway. II. No heat is force.
Conclusions: I. No highway is a road. 23. Statements : All forces are energies. All energies are
II. All streets are roads. powers. No power is heat.
15. Statements: Some animals are plants. Conclusions : I. No energy is heat.
All plants are rocks. II. Some forces being heat is a possibility.
Conclusions: I. All plants are animals. 24. Statements : No note is a coin. Some coins are metals.
II. Atleast some rocks are animals. All plastics are notes.
DIRECTIONS (Q.16-20) : In each questions below are two / three Conclusions : I. No coin is plastic.
statements followed by two conclusions numbered I and II. You II. All plastics being metals is a possibility.
have to take the two / three given statements to be true even if they 25. Statements : No note is a coin. Some coins are metals.
seem to be at variance from commonly known facts and then decide All plastics are notes.
which of the given conclusions logically follows from the given Conclusions : I. No metal is plastic.
statements disregarding commonly known facts. Give answer II. All notes are plastics.
(a) if only conclusion I follows. 26. Statements : Some symbols are figures. All symbols are
(b) if only conclusion II follows. graphics.
(c) if either conclusion I or II follows. No graphic is a picture.
(d) if neither conclusion I nor II follows. Conclusions : I. Some graphics are figures.
(e) if both conclusions I and II follows. II. No symbol is a picture.
16. Statements: No holiday is a vacation. 27. Statements : All vacancies are jobs. Some jobs are
Some vacations are trips. occupations.
Conclusions: I. No trip is a holiday. Conclusions : I. All vacancies are occupations.
II. Some holidays are definitely not trips. II. All occupations being vacancies is a
17. Statements: Some kites are birds. possibility.
No kite is an aeroplane. DIRECTIONS (Qs. 28-30) : In each question below are two/three
Conclusions: I. All aeroplanes are birds. statements followed by two conclusions numbered I and II. You
II. Some birds are definitely not kites. have to take the two/three given statements to be true even if
18. Statements: All metals are plastics. they seem to be at variance from commonly known facts and then
All plastics are fibres. decide which of the given conclusions logically follows from the
Conclusions: I. At least some fibres are metals given statements disregarding commonly known facts.
II. Some metals are not fibres.
19. Statements: Some animals are plants. Give answer (a) if only conclusion I follows
All plants are rocks. Give answer (b) if only conclusion II follows.
Conclusions: I. All plants are animals. Give answer (c) if either conclusion I or conclusion II follows.
II. Atleast some rocks are animals. Give answer (d) if neither conclusion I nor conclusion II follows.
20. Statements: Some institutes are banks. Give answer (e) if both conclusion I and conclusion II follow.
All institutes are academies. (Qs. 28-29) :
All academies are schools. Statements : All gliders are parachutes.
Conclusions: I. All banks can never be schools. No parachute is an airplane.
II. Any bank which is an institute in a school. All airplanes are helicopters.
28. Conclusions : I. No glider is an airplane.
DIRECTIONS (Qs. 21-27) : In each group of questions below are II.All gliders being helicopters is a possibility.
two/three statements followed by two conclusions numbered I and 29. Conclusions : I. No helicopter is a glider.
II. You have to take the given statements to be true even if they II. All parachutes being helicopters is a
seem to be at variance from commonly known facts and then decide possibility.
which of the given conclusions logically follows from the two/three 30. Statements : Some mails are chats.
statements disregarding commonly known facts. All updates are chats.
Give answer (a) if only conclusion I follows; Conclusions : I. All mails being updates is a possibility.
Give answer (b) if only conclusion II follows; II. No update is a mail.
12. a b c d e 13. a b c d e 14. a b c d e 15. a b c d e 16. a b c d e
RESPONSE 17. a b c d e 18. a b c d e 19. a b c d e 20. a b c d e 21. a b c d e

GRID 22. a b c d e 23. a b c d e 24. a b c d e 25. a b c d e 26. a b c d e


27. a b c d e 28. a b c d e 29. a b c d e 30. a b c d e
y
o
u
rs
m
a
h
14

b
o
o
b
.w
Syllogism - II

o
rd
p
re
s
s
.c
o
m
Max. Marks : 25 No. of Qs. 25 Time : 18 min. Date : ........./......../................
DIRECTIONS (Q.1-5) : In each of the questions below are given 4. Statements:
three statements followed by three conclusions numbered I , II All trees are gardens.
and III. You have to take the given statements to be true even if All gardens are stones.
they seem to be at variance from commonly known facts. Read All stones are fences.
all the conclusions and then decide which of the given Conclusions:
conclusions logically follows from the given statements I. Some fences are gardens.
disregarding commonly known facts. II. All gardens are stones.
1. Statements: III. Some stones are trees.
Some flowers are bins. (a) Only I and II follows (b) Only I and III follows
Some bins are handles (c) Only II or III follow (d) All follow
All handles are sticks. (e) None of these
Conclusions: 5. Statements:
I. Some sticks are bins. All books are leaves.
II. Some handles are flowers. Some leaves are jungles.
III. Some sticks are flowers. No jungle is box.
(a) Only II follows (b) Only III follows Conclusions:
(c) Only I and II follow (d) Only I and III follow I. Some jungles are books.
(e) None of these II. No book is box.
2. Statements: III. Some leaves are boxes.
Some towers are windows. (a) None follows (b) Only I follows
All windows are houses. (c) Only II follows (d) Only III follows
Some houses are temples (e) Only I and II follow
Conclusions:
I. Some towers are temples. DIRECTIONS (Q.6-25) : In each questions below are given
II. Some houses are towers. two/three statements followed by two conclusions numbered I
III. Some temples are windows. and II. You have to take the given statements to be true even if
(a) Only I follows (b) Only II follows they seem to be at variance with commonly known facts. Read all
the conclusions and then decide which of the given conclusions
(c) Only III follows (d) Only I and II follow
logically follows from the given statements disregarding
(e) None of these
commonly known facts. Give answer.
3. Statements:
Some walls are doors. (a) if only conclusion I follows.
Some doors are cots. (b) if only conclusion II follows.
Some cots are chairs. (c) if either conclusion I or II follows.
Conclusions: (d) if neither conclusion I nor II follows.
I. Some chairs are doors. (e) if both conclusions I and II follows.
II. Some cots are walls. 6. Statements:
III. No chair is door. Some toys are desks.
(a) Only II follows Some desks are pens.
(b) Only III follows All pens are rods.
(c) Only either I or III follows Conclusions:
(d) Only I follows I. Some rods are toys.
(e) None of these II. Some pens are toys.

RESPONSE 1. a b c d e 2. a b c d e 3. a b c d e 4. a b c d e 5. a b c d e

GRID 6. a b c d e
y
o
u
rs
m
a
h
28 SPEED TEST 14

b
o
o
7. Statements: 15. Conclusions:

b
.w
Some table are huts. I. No card is a paper.
No hut is ring. II. Some papers are cards.

o
rd
All rings are bangles. 16. Conclusions:

p
Conclusions: I. All cards being papers is a possibility.

re
I. Some bangles are tables. II. All boards being papers is a possibility.

s
s
.c
II. No bangles is table. 17. Statements: All rings are circles.

o
8. Statements: All squares are rings.

m
Some chairs are rooms. No ellipse is a circle.
All rooms are trees. Conclusions: I. Some, rings being ellipses is a possibility.
All trees are poles. : II. At least some circles are squares.
Conclusions: 18. Statements : No house is an apartment.
I. Some poles are chairs. Some bungalows are apartments.
II. Some trees are chairs. Conclusions: I. No house is a bungalow.
9-10. Statements: II. All bungalows are houses.
All buildings are houses. 19. Statements: Some gases are liquids.
No house is an apartment.
All liquids are water.
All apartments are flats.
Conclusions: I. All gases being water is a possibility.
9. Conclusions:
II. All such gases which are not water can
I. No flat is a house.
never be liquids.
II. No building is an apartment.
20. Statements: All minutes are seconds.
10. Conclusions:
All seconds are hours.
I. All buildings being flats is a possibility.
II. All apartments being building is a possibility. No second is a day.
11-12.Statements: Conclusions: I. No day is an hour.
Some oceans are seas. II. At least some hours are minutes.
All oceans are rivers. (21-22): Statements: Some teachers are professors.
No river is a canal. Some lecturers are teachers.
11. Conclusions: 21. Conclusions: I. All teachers as well as professors being
I. All rivers can never be oceans. lecturers is a possibility.
II. All canals being oceans is a possibility. II. All those teachers who are lecturers are
12. Conclusions: also professors.
I. No ocean is a canal. 22. Conclusions: I. No professor is a lecturer.
II. At least some seas are rivers. II. All lecturers being professors is a
13-14. Statements: possibility.
No day is night. (23-24):
All nights are noon. Statements: Some flowers are red.
No noon is an evening. Some roses are flowers.
13. Conclusions: 23. Conclusions:
I. No day is noon. I. All those flowers which are roses are red.
II. No day is an evening. II. No rose is red.
14. Conclusions: 24. Conclusions :
I. No evening are nights. I. All roses being red is a possibility.
II. All days being noon is a possibility. II. Some flowers can never be roses.
25. Statements: Some hills are mountains.
15-16. Statements:
All mountains are high.
Some papers are boards
Conclusions: I. All hills being high is a possibility.
No board is a card.
II. Some mountains can never be hills.

7. a b c d e 8. a b c d e 9. a b c d e 10. a b c d e 11. a b c d e
RESPONSE 12. a b c d e 13. a b c d e 14. a b c d e 15. a b c d e 16. a b c d e

GRID 17. a b c d e 18. a b c d e 19. a b c d e 20. a b c d e 21. a b c d e


22. a b c d e 23. a b c d e 24. a b c d e 25. a b c d e
y
o
u
rs
m
a
h
b
15

o
o
b
.w
Symbols & Codes

o
rd
p
re
s
s
.c
o
m
Max. Marks : 25 No. of Qs. 25 Time : 20 min. Date : ........./......../................
DIRECTIONS (Qs.1-5): In each question below is given a group
Number/
of letters followed by four combinations of digits/symbols 9 4 & 5 % 3 # 7 6 @ 8 + 2 $
Symbols
numbered (a), (b), (c) and (d). You have to find out which of the
Letter
combinations correctly represents the group of letters based on X P J H B D K F S T N G R L
Codes
the coding system and the conditions given below and mark the
number of that combination as your answer. If none of the
Conditions:
combinations correctly represents the group of letters, mark (e)
(i) If the first element is a symbol and the last element is a
i.e. ‘None of these’ as your answer.
number, then the codes for both are to be interchanged.
Letters P M A E J K D R W H I U T F (ii) If both the first and last elements are symbols, then the
Digits/symbols last element is to be coded as the code for the first
4 $ 1 2 3 # 5 @ © 6 % d 7 9
Conditions element.
(i) If the first letter is a consonant and the last letter is a vowel, (iii) If the group of elements contains only one symbol,
the codes of both these are to be interchanged. then that symbols is to be coded as A.
(ii) If both the first and the last letters are consonants both these 6. 28%956
are to be coded as per the code of the last letter. (a) RNBXHS (b) RNAXSH
(iii) If the first letter is vowel and the last letter is a consonant (c) RNBXSH (d) RNAXHS
both these are to be coded as ‘ ’ (e) RNASHX
Note: All the remaining letters are to be coded as per their original
7. ©62+74
codes.
1. ERWHKA (a) PSRGFT (b) TSRFGP
(a) 2 @ © 6# 1 (b) 1 @ © 6# 2 (c) 1 @ © 6 # I (c) PSRFGT (d) PRSGFT
(d) 2 @ © 6# 2 (e) None of these (e) TSRGFP
2. MPEKDU 8. +5963%
(a) $ 42 #5 d (b) $42 #5$ (c) d 4 2# 5 d (a) GHXSDG (b) GSHXDB
(d) d 42 5# $ (e) None of these (c) GHXDSG (d) GHSXDB
3. TMEIUF (e) GXHSDG
(a) 7$2% d 9 (b) 7$2% d 7 (c) 9$2% d 7 9. In a certain code MODE is written as #8%6 and DEAF is
(d) 9$2% d 9 (e) None of these written as %67$. How is FOAM written in that code?
4. JTAERI (a) $87# (b) $#7% (c) #87%
(a) % 7 1 2@ 3 (b) 3712@3 (c) 71 2@ (d) $87% (e) None of these
(d) %7 12 @ % (e) None of these 10. In a certain code WEAK is written as 5%9$ and WHEN is
5. UKTMIH written as 5*%7. How HANK written in that code?
(a) # 7$ % 6 (b) 6 # 7$ % d (c) # 7 $ % (a) *9$7 (b) 9*$7 (c) $97*
(d) 7 #$ % 6 (e) None of these (d) 9*7$ (e) None of these
DIRECTIONS (Qs.11-15) : In each of these questions a group of
DIRECTIONS (Qs.6-8): In each question below is given a group letters is given followed by four combinations of number/symbol
of numbers/symbols followed by five combinations of letter codes numbered (a), (b), (c) & (d). Letters are to be coded as per the
numbered (a), (b), (c), (d) and (e). You have to find out which of scheme and conditions given below. You have to find out the
the combinations correctly represents the group of numbers/ serial number of the combination, which represents the letter group.
symbols based on the following coding system and the conditions Serial number of that combinations is your answer. If none of the
and mark the number of that combination as your answer. combinations is correct, your answer is (e) i.e. 'None of these'.

RESPONSE 1. a b c d e 2. a b c d e 3. a b c d e 4. a b c d e 5. a b c d e

GRID 6. a b c d e 7. a b c d e 8. a b c d e 9. a b c d e 10. a b c d e
y
o
u
rs
m
a
h
30 SPEED TEST 15

b
o
o
19. 592476

b
Letters D K M B I N P R J A L S E Q G
(a) H COQ$%T (b) Q$% oT

.w
Number /
% 3 7 H 4 @ $ 1 8 5 # 9 2 £ 6 (c) H©Q$%OO (d) CCQ$%o©

o
Symbol Code

rd
(e) None of these

p
Conditions 20. 468910

re
(i) If the first letter is a consonant and the last a vowel, (l) $HJ©KL (b) LHJ@K$

s
s
both are to be coded as the code of the vowel. (c) *HJ@K$ (d) $HJ@K*

.c
(ii) If the first letter is a vowel and the last a consonant, the

o
(e) None of these

m
codes for the first and the last are to be interchanged.
(iii) If no vowel is present in the group of letters, the second DIRECTIONS (Qs. 21-24) : In each of the questions given below,
and the fifth letters are to be coded as ã . a group of digits is given followed by four combinations of letters/
11. KQAPJE symbols numbered (a), (b),(c) and(d). You have to find out which
(a) 3£5$82 (b) 3£58$2 (c) 2£5$82 of the four combinations correctly represents the group of digits
(d) 2£5$83 (e) None of these based on the letter/symbol codes and the conditions given below.
12. EMANRB If none of the four combinations represents the group of digits
(a) *75@12 (b) 275@1* (c) ã 75@2ã correctly, give (e) i.e. `None of these' as the answer.
(d) *75@1* (e) None of these
13. JAQDKP Digit 2 8 3 9 4 7 6 5 1
(a) 85£%38 (b) $5£%3$ (c) $5£%38 Code B = T @ K $ © P C
(d) $5£3%8 (e) None of these
14. QDBGRM Conditions:
(a) £%*617 (b) $ã *6ã 7 (c) £%*167 (i) If the first digit is odd and last digit is even, the codes
(d) % £*61ã (e) None of these for the first and the last digits are to be interchanged.
15. IKQLMS (ii) If the first as well as the last digit is even, both are to
(a) 43£#74 (b) ã 3£#7ã (c) 4£3#74 be coded by the code for last digit.
(d) 93£#74 (e) None of these (iii) If the first as well as the last digit is odd, both are to
be coded as X.
DIRECTIONS (Qs. 16-20): In each of these questions, a group
(iv) If the first digit is even and last digit is odd, both are
of digits is given followed by four combinations of letters and
to be coded by the code for the first-digit.
symbols numbered (a), (b), (c) and (d). The group of digits is to be
21. 2976581
coded as per the scheme and conditions given below. The serial
(a) B@$©P = B (b) C@$©P = C
number of the combination which correctly represents the group
(c) B@$©P = C (d) C@S©P = B
of digits is your answer. If none of the four combinations is correct,
(e) None of these
your answer is (e) i.e., 'None of these'.
22. 7269534
Digits 5 8 4 3 6 2 9 0 7 1 (a) $BC@PTK (b) KB©@PT$
Code T J $ # H Q @ L % K (c) $B©@PT$ (d) KB©@P=$
Conditions: (e) None of these
(i) If the first as well as the last digit is odd, both are to be 23. 8135246
coded as ©. (a) = CTPBK = (b) ©CTP = K©
(ii) If the first as well as the last digit is even, their codes are to (c) ©CTPBK© (d) CTPB$ =
be swapped.
(e) None of these
(iii) If '0' is the last digit, it is to be coded as *.
16. 270514 24. 4352718
(a) ©%LTK© (b) $%LTKQ (a) XTPB$CX (b) KTPB$C=
(c) Q%LTK$ (d) $%*TKQ (c) =TPB$CK (d) KTP$CK
(e) None of these (e) None of these
17. 364279 25. if 1 is coded as$, 5 is coded as %, 9 is coded as , 3 is coded
(a) ©H$Q%© (b) #H$Q%@ as +, 7 is coded as # and 4 is coded as?, what will be the
(c) ©H$Q%# (d) #H$Q%© correct code of the number 435971?
(e) None of these
(a) ? + % # $ (b) ? + % $ #
18. 875306
(a) J%T#Ll1 (b) H%T#LH (c) ? + % # $ (d) $ # % + ?
(c) H%T#LJ (d) J% oT#LJ (e) None of these
(e) None of these
11. a b c d e 12. a b c d e 13. a b c d e 14. a b c d e 15. a b c d e
RESPONSE 16. a b c d e 17. a b c d e 18. a b c d e 19. a b c d e 20. a b c d e
GRID 21. a b c d e 22. a b c d e 23. a b c d e 24. a b c d e 25. a b c d e
y
o
u
rs
m
a
h
b
16

o
16

o
b
Alpha Numeric

.w
o
rd
p
re
Sequence Puzzle

s
s
.c
o
m
Max. Marks : 30 No. of Qs. 30 Time : 20 min. Date : ........./......../................
DIRECTIONS (Q. 1-5) : Answers the questions given below 8. If all the symbols are dropped from the arrangement, which
referring to the following arrangement: of the following will be the twelfth from the left end ?
J * R 3 PL2 # I N 7 O C @ K 5 D = M $ 6 B< AQ 4 (a) 9 (b) 2 (c) S
1. Four of the following five are alike in a certain way as regards (d) 7 (e) None of these
their position in the above arrangement and so form a group. 9. Four of the following five are alike in a certain way based on
Which is the one that does not belong to that group? their positions in the above arrangement and so form a group.
(a) 2 3 # (b) O I C (c) K O 5 Which is the one that does not belong to the group?
(d) # P I (e) B $ < (a) L$8 (b) AKM (c) @!F
2. What will come in the place of the question mark (?) in the (d) 6%G (e) JD©
following series based on the above arrangement? 10. What should come in place of the question mark (?) in the
PRJ#L3712@ ON? following series based on the above arrangement >
(a) D K C (b) 5 @ O (c) D K @ F3U %IT L$ Ù ?
(d) = 5 @ (e) None of these (a) 927 (b) 7&A (c) 7AM
3. It the above series is re-arranged in the reverse order, (d) 2&A (e) 27&
which will be the eleventh element to the left to the
DIRECTIONS (Qs. 11-18): Study the following arrangement of
sixteenth element from the left end?
letters/symbols and answer the questions given below:
(a) J (b) 6 (c) B
(d) < (e) None of these D F JT $ # PR ZQ * C MAB@ HK LS + ?
4. How many such numbers are there in the above arrangement 11. How many such symbols are there each of which is
each of which is immediately preceded by a consonant and immediately preceded by a symbol and immediately followed
not immediately followed by a symbol? by a letter?
(a) None (b) Two (c) Four (a) One (b) Two (c) Three
(d) Three (e) None of these (d) Four (e) None of these
5. How many such symbols are there in the above arrangement 12. If the order of the first half of the arrangement is reversed
each of which is immediately preceded by a number and which of the following letters/symbols will be the fifth to the
immediately followed a consonant? left of the fifteenth letter/symbol from the left?
(a) One (b) Two (c) Three (a) * (b) Q (c) T
(d) More than three (e) None (d) J (e) None of these
13. If all the symbols of the above sequence are denoted by 7
DIRECTIONS (Qs. 6-10) : Study the following arrangement of
and each letter is denoted by 5, then what will be the sum of
consonants, vowels, numbers and symbols carefully and answer
all the elements of the sequence?
the questions given below:
(a) 142 (b) 138 (c) 132
H@ F ! 3 U 6 % G I T * PL 8 $Ù9 S 2 7 &AM K + J © (d) 122 (e) None of these
D4#5&E 14. If all the symbols from the above sequence are dropped,
6. Which of the following is ninth to the right of the twentieth which letter will be seventh to the right of twelfth letter from
from the right end of the above arrangement ? the right?
(a) K (b) M (c) U (a) H (b) B (c) K
(d) A (e) None of these (d) A (e) None of these
7. How many such consonants are there in the above 15. Which of the following is related to ‘FT’ in the same way as
arrangement, each of which is immediately preceded by a ‘DJ’ is related to‘? S’ ?
symbol and also immediately followed by a symbol ? (a) L+ (b) KS (c) HL
(a) None (b) One (c) Two (d) + L (e) None of these
(d) Three (e) More than three
1. a b c d e 2. a b c d e 3. a b c d e 4. a b c d e 5. a b c d e
RESPONSE 6. a b c d e 7. a b c d e 8. a b c d e 9. a b c d e 10. a b c d e
GRID 11. a b c d e 12. a b c d e 13. a b c d e 14. a b c d e 15. a b c d e
y
o
u
rs
m
a
h
32 SPEED TEST 16

b
o
o
16. How many such letters are there in the above sequence 23. If the order of the last fifteen elements in the above

b
.w
each of which occupies the same position from the left in arrangement is reversed, which of the following will be the
the sequence as in the alphabet from left? ninth to the right of the eleventh element from. the left end?

o
rd
(a) None (b) One (c) Two (a) G (b) % (c) 8

p
(d) Three (e) None of these (d) 3 (e) None of these

re
s
17. Four of the following five are alike on the basis of their 24. How many such consonants are there in the above

s
.c
position in the above sequence and hence form a group. arrangement, each of which is immediately preceded by a

o
Which of the following does not belong to that group? symbol but not immediately followed by either a number or

m
(a) DJ ? (b) T # L (c) FT + a vowel?
(d) PZ @ (e) J # S (a) None (b) One (c) Two
18. Which of the following will be exactly midway between fifth (d) Three (e) More than three
element from the left and eighth element from the right? 25. Four of the following five are alike in a certain way based on
(a) C (b) * (c) Q their position in the above arrangement and so form a group.
(d) M (e) None of these Which is the one that does not belong to that group?
19. Select the combination of numbers so that letters arranged (a) A $ E (b) % V N (c) 2 F V
accordingly will form a meaningful word. (d) 4 K 1 (e) 6 Q ÷
R A C E T DIRECTIONS (Qs. 26-30): Study the following arrangement
1 2 3 4 5 carefully and answer the questions given below:
(a) 1, 2, 3, 4, 5 (b) 3, 2, 1, 4, 5 M £ 5 T R E 3 $ PJ 1 7 D 1 2 NA4 F H 6 * U 9 # V B @ W
(c) 5, 2, 3, 4, 1 (d) 5, 1, 2, 3, 4 26. If the positions of the first fourteen characters of the above
(e) None of these arrangement are reversed, which of the following will be the
20. Select the combination of numbers so that the letters twenty-second from the right end?
arranged accordingly will form a meaningful word. (a) J (b) I (c) P
VA R S T E (d) 3 (e) None of these
(a) 2, 3, 1, 6, 4, 5 (b) 4, 5, 2, 3, 1, 6 27. How many such numbers are there in the above arrangement,
(c) 6, 3, 4, 5, 2, 1 (d) 3, 2, 4, 5, 6, 1 each of which is immediately preceded by a vowel and
(e) None of these immediately followed by a consonant?
DIRECTIONS (Qs. 21-25): Study the following arrangement (a) None (b) One (c) Two
carefully and answer the questions given below: (d) Three (e) More than three
J 1 #P 4 E K 3 A D $ R U M 9 N 5 1 % T V * H2 ÷ 28. What should come in place of the question mark (?) in the
F6G8QW series given below based on the above arrangement?
21. How many such numbers are there in the above arrangement, R3 £ PIE ?AF I
each of which is either immediately preceded by or (a) DNJ (b) D21 (c) IN1
immediately followed by a vowel or both? (d) N4D (e) None of these
(a) None (b) One (c) Two 29. How many such consonants are there in the above
(d) Three (e) More than three arrangement each of which is immediately preceded by a
22. Which of the following is exactly in the middle between the symbol but not immediately followed by a number?
tenth from the left and the eighth from the right end in the (a) None (b) One (c) Two
above arrangement? (d) Three (e) More than three
(a) M (b) N (c) 1 30. Which of the following is the fifth towards right of the
(d) 5 (e) None of these seventeenth from the right end?
(a) $ (b) 4 (c) 7
(d) A (e) None of these

16. a b c d e 17. a b c d e 18. a b c d e 19. a b c d e 20. a b c d e


RESPONSE 21. a c d e 22. a c d e 23. a c d e 24. a c d e 25. a c d e
b b b b b
GRID
26. a b c d e 27. a b c d e 28. a b c d e 29. a b c d e 30. a b c d e
y
o
u
rs
m
a
h
b
17

o
17

o
b
.w
Input-Output

o
rd
p
re
s
s
.c
o
m
Max. Marks : 23 No. of Qs. 23 Time : 18 min. Date : ........./......../................
5. Input: paper dry 37 23 height call 62 51
DIRECTIONS (Qs. 1-5): Study the following information carefully
and answer the given questions: Which of the following steps will be the last but one?
(a) V (b) IV (c) VI
A word and number arrangement machine when given an input
(d) III (e) None of these
line of words and numbers rearranges them following a particular
rule in each step. The following is an illustration of input and DIRECTIONS (Qs.6-10) : A word and number arrangement
rearrangement. (All numbers are two-digit numbers.) machine when given an input line of words and numbers rearranges
Input : good for everything 19 37 26 all 65 them following a particular rule in each step. The following is an
Step I : all good for everything 19 37 26 65 illustration of an input and rearrangement.
Step II : all 65 good for everything 19 37 26 Input : 17 put show on 39 27 85 gold
Step III : all 65 everything good for 19 37 26 Step I : show 17 put on 39 27 85 gold
Step IV : all 65 everything 37 good for 19 26 Step II : show 85 17 put on 39 27 gold
Step V : all 65 everything 37 for good 19 26 Step III : show 85 put 17 on 39 27 gold
Step VI : all 65 everything 37 for 26 good 19 Step IV : show 85 put 39 17 on 27 gold
and Step VI is the last step of the rearrangement as the desired Step V : show 85 put 39 on 17 27 gold
arrangement is reached. Step VI : show 85 put 39 on 27 17 gold
As per the rules followed in the above steps, find out in each of Step VII : show 85 put 39 on 27 gold 17
the following questions the appropriate step for the given input. and step VII is the last step of the rearrangement of the above
(All numbers are two-digit numbers.)
input.
1. Input: won 13 now 25 72 please go 47
As per the rules followed in the above steps, find out in each of
How many steps will be required to complete the rearrangement?
the following questions the appropriate step for the given input.
(a) Four (b) Five (c) Six
(d) Three (e) None of these 6. Input : glass full 15 37 water now 85 67
2. Step III of an input is : Which of the following will be step VI of the above input?
car 81 desk 15 42 39 tall more (a) 85 now 67 full glass 15 37
Which of the following will be Step VI? (b) water 85 now 67 glass full 15 37
(a) car 81 desk 42 39 15 tall more (c) water 85 now 67 glass 37 full 15
(b) car 81 desk 42 15 39 tall more (d) There will be no such step.
(c) car 81 desk 42 more 39 15 tall (e) None of these
(d) There will be no such step. 7. Step II of an input is: ultra 73 12 16 mail sort 39 kite. Which of
(e) None of these the following steps will be the last but one?
3. Step II of an input is: (a) VIII (b) IX (c) VII
bell 53 town hall near 27 43 12 (d) VI (e) None of these
How many more steps will be required to complete the 8. Step III of an input is: win 75 voice 15 39 store gap 26. Which
rearrangement? of the following is definitely the input?
(a) Five (b) Four (c) Six (a) voice 15 win 75 39 store gap 26
(d) Three (e) None of these (b) voice win 75 15 39 store gap 26
4. Step II of an input is (c) 15 75 win voice store gap 26
box 93 25 year end 41 32 value (d) Cannot be determined
Which of the following is definitely the input? (e) None of these
(a) 25 year end box 93 41 32 value 9. Step II of an input is: tube 83 49 34 garden flower rat 56. How
(b) 25 year end 93 41 32 value box many steps will be required to complete the rearrangement?
(c) 9325 box year end 41 32 value
(a) Four (b) Five (c) Six
(d) Cannot be determined
(d) Three (e) None of these
(e) None of these

1. a b c d e 2. a b c d e 3. a b c d e 4. a b c d e 5. a b c d e
RESPONSE 6. a b c d e 7. a b c d e 8. a b c d e 9. a b c d e
GRID
y
o
u
rs
m
a
h
34 SPEED TEST 17

b
o
o
10. Input : hunt for 94 37 good 29 48 book. Input : 51 pour 32 start now 23 46 house

b
How many steps will be required to complete the Step I : 23 51 pour 32 start now 46 house

.w
rearrangement? Step II : 23 start 51 pour 32 now 46 house

o
rd
(a) Four (b) Five (c) Six Step III : 23 start 32 51 pour now 46 house

p
(d) Seven (e) None of these Step IV : 23 start 32 pour 51 now 46 house

re
DIRECTIONS (Q.11 to 15) : Study the following information to Step V : 23 start 32 pour 46 51 now house

s
Step VI : 23 start 32 pour 46 now 51 house

s
answer the given questions:

.c
and step VI is the last step of the rearrangement

o
A word and number arrangement machine when given an input

m
As per the rules followed in the above steps, find out in each of
line of words and numbers rearranges them following a particular
the following questions the appropriate steps for the given input.
rule. The following is an illustration of input and rearrangement.
16. Step II of an input is : 18 task bear cold dish 81 63 31
(All the numbers are two-digit numbers.)
How many more steps will be required to complete the
Input : sine 88 71 cos theta 14 56 gamma delta 26
rearrangement?
Step I : cos sine 71 theta 14 56 gamma delta 26 88
(a) Three (b) Four (c) Five
Step II : delta cos sine theta 14 56 gamma 26 88 71
(d) Six (e) None of these
Step III : gamma delta cos sine theta 14 26 88 71 56
17. Input : 72 59 37 go for picnic 24 journey
Step IV : sine gamma delta cos theta 14 88 71 56 26
How many steps will it take to complete the rearrangement?
Step V : theta sine gamma delta cos 88 71 56 26 14
(a) Three (b) Four (c) Five
Step V is the last step of the rearrangement.
(d) Six (e) None of these
As per the rules followed in the above steps, find out in each of
18. Input : nice flower 24 12 costly height 41 56
the following questions the appropriate steps for the given input.
Which of the following will be step III ?
Input for the questions:
(a) 12 nice 34 height flower costly 41 56
Input : for 52 all 96 25 jam road 15 hut 73 bus stop 38 46 (All the
(b) 12 nice 34 height 41 flower costly 56
numbers given in the arrangement are two digit numbers).
(c) 12 nice 34 flower costly height 41 56
11. Which word/ number would be at 8th position from the
(d) 12 nice flower 34 costly height 41 56
right in step IV ?
(e) None of these
(a) 15 (b) road (c) hut
19. Step II of an input is : 16 victory 19 36 53 store lake town.
(d) jam (e) stop
Which of the following will be step V ?
12. Which step number would be the following output? Bus all
(a) 16 victory 19 town store 36 53 lake
for 52 25 jam road 15 hut stop 38 46 96 73.
(b) 16 victory 19 town 36 store 53 lake
(a) There will be no such step.
(c) 16 victory 19 town 36 53 store lake
(b) III (c) II (d) V
(d) There will be no such step
(e) VI
(e) None of these
13. Which of the following would be step VII?
20. Step III of an input is : 15 yes 29 ask for soap 42 37
(a) stop road jam hut for bus all 15 96 73 5246 38 25
Which of the following is definitely the input?
(b) road jam hut for bus all stop 15 25 38 46 52 73 96
(a) ask yes 29 15 for soap 42 37
(c) stop road jam hut for bus all 96 73 52 46 38 25 15
(b) yes ask 15 29 for soap 42 37
(d) jam hut for bus all 25 road stop 15 96 73 52 46 38
(c) 29 15 yes ask for soap 42 37
(e) There will be no such step
(d) Cannot be determined
14. Which word/number would be at 6th position from the left
(e) None of these
in step V?
21. Input : milk pot 18 24 over goal 36 53
(a) 25 (b) stop (c) jam
Which of the following steps will be the last but one?
(d) all (e) road
(a) VI (b) V (c) VII
15. Which of the following would be step III?
(d) VIII (e) None of these
(a) hut for bus all 25 jam road 15 stop 38 96 73 52 46
22. Step III of an input is : 36 win 44 95 86 ultra box queen
(b) for bus all 25 jam road 15 hut 38 stop 96 46 73 52
How many more steps will be required to complete the
(c) hut for bus all jam road 15 stop 38 96 73 52 46 25
rearrangement?
(d) for bus all 25 jam road 15 hut stop 38 46 96 73 52
(a) Three (b) Four (c) Five
(e) None of these
(d) Six (e) None of these
DIRECTIONS (Q. 16 to 23) : Study the following information 23. Input : new 22 model 27 pump 38 11 join
carefully and answer the given questions: How many steps will be required to complete the
A word and number arrangement machine when given an input line rearrangement?
of words and numbers rearranges them following a particular rule in (a) Four (b) Five (c) Six
each step. The following is an illustration of input and rearrangement. (d) Seven (e) None of these

10. a b c d e 11. a b c d e 12. a b c d e 13. a b c d e 14. a b c d e


RESPONSE 15. a c d e 16. a c d e 17. a c d e 18. a c d e 19. a c d e
b b b b b
GRID
20. a b c d e 21. a b c d e 22. a b c d e 23. a b c d e
y
o
u
rs
m
a
h
b
18

o
18

o
b
Mathematical

.w
o
rd
p
Operations

re
s
s
.c
o
m
Max. Marks : 30 No. of Qs. 30 Time : 20 min. Date : ........./......../................
1. If ‘+’ means ‘minus’ ‘–’ means ‘multiplied by’ ‘¸’ means ‘plus’ 8. If L stands for +, M stands for –, N stands for ×, P stands for
and ‘×’ means ‘divided by’, then ¸, then 14 N 10 L 42 P 2 M 8 = ?
10 × 5 ¸ 3 – 2 + 3 = ? (a) 153 (b) 216 (c) 248
53 (d) 251 (e) None of these
(a) 5 (b) 21 (c) 9. It being given that: > denotes +, < denotes –, + denotes ¸, –
3
denotes =, = denotes ‘less than’ and × denotes ‘greater than’.
(d) 18 (e) None of these
Find which of the following is a correct statement.
2. In the following question you have to identify the correct
response from the given premises stated according to the (a) 3 + 2 > 4 = 9 + 3 < 2
following symbols. (b) 3 > 2 > 4 = 18 + 3 < 1
If ‘+’ means ‘¸’ , ‘–’ means ‘×’, ‘¸’ means ‘+’ and ‘×’ means (c) 3 > 2 < 4 × 8 + 4 < 2
‘–’, then 63 × 24 + 8 ¸ 4 + 2 – 3 = ? (d) 3 + 2 < 4 × 9 + 3 < 3
(a) 54 (b) 66 (c) 186 (e) None of these
(d) 48 (e) None of these 10. If P denotes +, Q denotes –, R denotes × and S denotes ¸
3. Which one of the following is correct? which of the following statements is correct?
6 * 4 * 9 * 15 (a) 36 R 4 S 8 Q 7 P 4 = 10
(a) ×, = , – (b) ×, –, = (c) =, ×, – (b) 16 R 12 P 49 S 7 Q 9 = 200
(d) –, ×, = (e) None of these (c) 32 S 8 R 9 = 160 Q 12 R 12
4. If > = ¸, Ú = ×, < = +, Ù = –, + = <, × = =, – = > (d) 8 R 8 P 8 S 8 Q 8 = 57
(a) 6 > 2 > 3 Ù 8 Ú 4 + 13 (e) None of these
(b) 6 Ù 2 < 3 > 8 < 4 – 13 11. If ‘×’ stands for ‘addition’, ‘<’ for substraction, + for division,
(c) 6 Ú 2 < 3 Ù 8 > 4 × 13 > for multiplication, – for ‘equal to’, ¸ for ‘greater than’ and
(d) 6 > 2 Ú 3 < 8 Ù 4 + 13 ‘=’ for ‘less than’, then state which of the following is true?
(e) None of these (a) 3 × 4 > 2 – 9 + 3 < 3
5. Find out the correct answer for the unsolved equation on (b) 5 × 3 < 7 ¸ 8 + 4 × 1
the basis of the given equations. (c) 5 > 2 + 2 = 10 < 4 × 8
If 6 * 5 = 91, 8 * 7 = 169, 10 * 7 = 211, then11 * 10 = ? (d) 3 × 2 < 4 ¸ 16 > 2 + 4
(a) 331 (b) 993 (c) 678
(e) None of these
(d) 845 (e) None of these
6. If ‘–’ stands for division, ‘+’ for multiplication ‘¸’for DIRECTIONS (Qs. 12 - 16): In an imaginary language, the digits
subtraction and ‘×’ for addition. Which one of the following 0, 1, 2, 3, 4, 5, 6, 7, 8 and 9 are substituted by a, b, c, d, e, f, g,
equation is correct? h, i and j. And 10 is written as ba.
(a) 6 ¸ 20 × 12 + 7 – 1 = 70 12. (cd + ef) × bc is equal to
(b) 6 + 20 – 20 ¸ 7 × 1 = 62 (a) 684 (b) 816 (c) 916
(c) 6 – 20 ¸ 12 × 7 + 1 = 57 (d) 1564 (e) None of these
(d) 6 + 20 – 20 ¸ 7 – 1 = 38 13. dc × f – (bf – d) × d is equal to
(e) None of these (a) abb (b) abe (c) bce
7. In an imaginary mathematical operation ‘+’ means (d) bcf (e) None of these
multiplication, ‘×’ means subtraction, ‘¸’means addition and
14. baf + fg – (ca × h/be) is equal to
‘–’ means division. All other rules in mathematical operation
(a) 141 (b) 145 (c) 151
are the same as in the existing system.
Which one of the following gives the result of (d) 161 (e) None of these
175 – 25 ¸ 5 + 20 × 3 + 20 = ? 15. baf ¸ bf × d is equal to
(a) 160 (b) 2370 (c) 7 7 (a) df (b) cb (c) be
(d) 240 (e) None of these (d) d (e) None of these

1. a b c d e 2. a b c d e 3. a b c d e 4. a b c d e 5. a b c d e
RESPONSE 6. a b c d e 7. a b c d e 8. a b c d e 9. a b c d e 10. a b c d e
GRID 11. a b c d e 12. a b c d e 13. a b c d e 14. a b c d e 15. a b c d e
y
o
u
rs
m
a
h
36 SPEED TEST 18

b
o
o
16. If ‘+’ means ‘×’, ‘×’ means ‘–’, ‘¸’ means ‘+’ and ‘–’ means 24. If ‘÷’ means ‘+’ ; ‘–’ means ‘×’ ; ‘+’ means ‘÷’ and ‘×’ means

b
.w
‘¸’, then which of the following gives the result of ‘–’ then 20 ÷ 12 × 4 + 8 – 6 = ?
175 – 25 ¸ 5 + 20 × 3 ¸ 10 = ?

o
2

rd
(a) 77 (b) 160 (c) 240 (a) 8 (b) 29 (c) 32

p
3
(d) 2370 (e) None of these

re
(d) 26 (e) None of these

s
17. If + means ÷ ,– means ×, ¸ means + and × means –, then 36

s
25. If ‘–’ means ‘×’ ; ‘×’ means ‘+’ ; ‘+’ means ‘÷’ and ‘÷’ means

.c
× 8 + 4 ÷ 6 + 2 –3 = ?

o
(a) 2 (b) 18 (c) 43 ‘–’ then 40 × 12 + 3 – 6 ÷ 60= ?

m
(a) 44 (b) 7.95 (c) 16
1
(d) 6 (e) None of these (d) 28 (e) None of these
2 26. If ‘ + ’ means ‘divided by’ ‘–’ means ‘added to ’ ‘x’ means
18. If the given interchanges namely : signs + and ÷ and ‘subtracted from’ and ÷ means ‘multiplied by’ then what is
numbers 2 and 4 are made in signs and numbers, which one the value of 24 ¸ 12 – 18 + 9 ?
of the following four equations would be correct ? (a) – 25 (b) 0.72 (c) 15.30
(a) 2 + 4 ÷ 3 = 3 (b) 4 + 2 ÷ 6 = 1.5 (d) 290 (e) None of these
(c) 4 ÷ 2 + 3 = 4 (d) 2 + 4 ÷ 6 = 8. 27. If ® stands for ‘addition’ ¬ stands for ‘subtraction’ ­
(e) None of these
stands for ‘division ; ¯ stands for’ multiplication' Z stands
19. If L denotes x, M denotes ÷ , P denotes + and Q denotes
–, than 8 P 36 M 6 Q 6 M 2 L 3 = ? for equal to' then which of the following alternatives is
correct?
13 1 1
(a) (b) – (c) 14 (a) 7 43 6 1Z4
6 6 2
(d) 5 (e) None of these (b) 3 6 2 3 6Z5
20. If X stands for’ addition’, < for ‘substraction’, + stands for (c) 5 7 3 2Z5
‘division’, > for ‘multiplication’, –, stands for ‘equal to’,
(d) 2 5 6 2Z6
¸ for ‘greater than’ and = stands for ‘less than’, state which
of the following is true ? (e) None of these
(a) 3 × 2 < 4 ¸ 16 > 2 + 4 28. Of ‘x’ Stands for ‘ addition’ ‘z’ for subtraction’ ‘+’ for division'
(b) 5 > 2 + 2 = 10 < 4 × 2 > for multiplication’ ‘–’ for equal to’ ‘+’ for ‘ greater than’
(c) 3 × 4 > 2 – 9 + 3 < 3 and '=' for ' less than' state which of the following is true. ?
(d) 5 × 3 < 7 ¸ 8 + 4 × 1 (a) 3 x 4 > 2 – 9 + 3 < 3
(e) None of these (b) 5 x 3 < 7 ¸ 8 + 4 x 1
21. If ‘20 – 10’ means 200, ‘8 ÷ 4’ means 12, ‘6 × 2’ means 4 and (c) 5 > 2 + 2 = 10 < 4 x 8
‘12 + 3’ means 4, then (d) 3 x 2 < 4 ¸ 16 > 2 + 4
100 – 10 × 1000 ÷ 1000 + 100 × 10 = ? (e) None of these
(a) 1090 (b) 0 (c) 1900 29. If ¸ means + – means ÷ x means – and + means x then
(d) 20 (e) None of these
(36 x 4 ) - 8 x 4
22. If × means +, ÷ means – , – means × and + means ÷, then 8 × =?
4 + 8 x 2 + 16 + 1
7 – 8 + 40 ÷ 2 = ?
(a) 0 (b) 8 (c) 12
2 3
(a) 1 (b) 7 (c) 8 (d) 16 (e) None of these
5 5 30. If x means +, – means x , ÷ means + and + means – then
(d) 44 (e) None of these (3 – 15 ÷ 19) x 8 + 6 = ?
23. If ‘+’ means ‘×’ ; ‘–’ means ‘÷’ ; ‘ ×’ means ‘–’ and ‘÷’ means (a) – 1 (b) 2 (c) 4
‘+’ then 9 + 8 ÷ 8 – 4 × 9 = ? (d) 8 (e) None of these
(a) 26 (b) 17 (c) 65
(d) 11 (e) None of these

16. a b c d e 17. a b c d e 18. a b c d e 19. a b c d e 20. a b c d e


RESPONSE 21. a c d e 22. a c d e 23. a c d e 24. a c d e 25. a c d e
b b b b b
GRID
26. a b c d e 27. a b c d e 28. a b c d e 29. a b c d e 30. a b c d e
y
o
u
rs
m
a
h
19

b
o
19

o
b
.w
Clock and Calendar

o
rd
p
re
s
s
.c
o
m
Max. Marks : 30 No. of Qs. 30 Time : 20 min. Date : ........./......../................
1. The number of times in a day the Hour-hand and the Minute- 8. At what time between 5.30 and 6 will the hands of a clock be
hand of a clock are at right angles, is at right angles?
(a) 44 (b) 48 (c) 24
5 7
(d) 12 (e) None of these (a) 43min. past 5 (b) 43 min. past 5
2. An accurate clock shows the time as 3.00. After hour hand 11 11
has moved 135°, the time would be (c) 40 min. past 5 (d) 45 min. past 5
(a) 7.30 (b) 6. 30 (c) 8.00 (e) None of these
(d) 9.30 (e) None of these 9. At what time between 4 and 5 o’clock will the hands of a
3. An accurate clock shows 8 O' clock in the morning. watch point in opposite directions?
Throughout how many degrees will the hour hand rotate, (a) 45 min. past 4 (b) 40 min. past 4
when the clock shows 2 O' clock in the afternoon?
(a) 150° (b) 144° (c) 168° 4 6
(c) 50 min. past 4 (d) 54 min. past 4
(d) 180° (e) None of these 11 11
4. March 1, 2008 was Saturday. Which day was it on March 1, (e) None of these
2002? 10. How much does a watch lose per day, if its hands coincide
(a) Thursday (b) Friday (c) Saturday
every 64 minutes?
(d) Sunday (e) None of these
5. How many times are an hour hand and a minute hand of a 8 5
(a) 32 min. (b) 36 min. (c) 90 min.
clock at right angles during their motion from 1.00 p.m. to 11 11
10.00 p.m.? (d) 96 min. (e) None of these
(a) 9 (b) 10 (c) 18 11. The last day of a century cannot be
(d) 20 (e) None of these
(a) Monday (b) Wednesday (c) Tuesday
6. At what approximate time between 4 and 5 am will the
hands of a clock be at right angle? (d) Friday (e) None of these
(a) 4 : 40 am (b) 4 : 38 am (c) 4 : 35 am 12. Which of the following is not a leap year?
(d) 4 : 39 am (e) None of these (a) 700 (b) 800 (c) 1200
7. At what time between 3 and 4 o’clock, the hands of a clock (d) 2000 (e) None of these
coincide? 13. How many days are there in x weeks x days?
4 (a) 7 x2 (b) 8 x (c) 14 x
(a) 16 minutes past 3
11 (d) 7 (e) None of these
5 14. It was Sunday on Jan 1, 2006. What was the day of the week
(b) 15 minutes past 3 on Jan 1, 2010?
61
(a) Sunday (b) Saturday (c) Friday
5 (d) Wednesday (e) None of these
(c) 15 minutes to 2
60 15. On 8th Feb, 2005 it was Tuesday. What was the day of the
4 week on 8th Feb, 2004?
(d) 16 minutes to 4
11 (a) Tuesday (b) Monday (c) Sunday
(e) None of these (d) Wednesday (e) None of these

1. a b c d e 2. a b c d e 3. a b c d e 4. a b c d e 5. a b c d e
RESPONSE 6. a b c d e 7. a b c d e 8. a b c d e 9. a b c d e 10. a b c d e
GRID 11. a b c d e 12. a b c d e 13. a b c d e 14. a b c d e 15. a b c d e
y
o
u
rs
m
a
h
38 SPEED TEST 19

b
o
o
16. The calendar for the year 2007 will be the same for the year. 23. Ashish leaves his house at 20 minutes to seven in the

b
.w
(a) 2014 (b) 2016 (c) 2017 morning reaches Kunal’s house in 25 minutes. They finish

o
(d) 2018 (e) None of these their breakfast in another 15 minutes and leave for their

rd
17. Today is Monday. After 61 days, it will be office which takes another 35 minutes. At what time do

p
re
(a) Wednesday (b) Saturday (c) Tuesday they leave Kunal’s house to reach their office?

s
s
(d) Thursday (e) None of these (a) 7.40 a.m. (b) 7.20 a.m. (c) 7.45 a.m.

.c
o
18. What was the day of the week on 17th June, 1998? (d) 8.15 a.m. (e) None of these

m
(a) Monday (b) Tuesday 24. Reaching the place of meeting on Tuesday 15 minutes
(c) Wednesday (d) Thursday before 8.30 hours, Anuj found himself half an hour earlier
(e) None of these than the man who was 40 minutes late. What was the
19 If 21st July, 1999 is a wednesday, what would have been scheduled time of the meeting?
the day of the week on 21st July, 1947 ? (a) 8.00 hrs (b) 8.05 hrs (c) 8.15 hrs
(a) Monday (b) Sunday (c) Thursday (d) 8.45 hrs (e) None of these
(d) Saturday (e) None of these 25. A clock gaining 2 min every hour was synchronised at
20. At an enquiry office at a railway station, a passenger was midnight with a clock losing 1 min every hour. How many
told that a train for New Delhi has left 15 minutes ago, but minutes behind will its minute hand be at eleven the
after every 45 minutes a train leaves for New Delhi. The following morning ?
next train will leave at 8.30 p.m. At what time was this (a) 23 (b) 27 (c) 22
information given to the passanger ? (d) None of these (e) None of these
(a) 7.45 pm (b) 8.00 pm (c) 8.15 pm 26. Rama remembers that she met her brother on Saturday,
(d) 8.05 pm (e) None of these which was after the 20th day of a particular month. If the
21. A watch is a minute slow at 1 p.m. on Tuesday and 2 minutes 1st day of that month was Tuesday, then on which date did
fast at 1 p.m. on Thursday. When did it show the correct Rama meet her brother ?
time ? (a) 24th (b) 23rd (c) 25th
(d) None of these
(a) 1:00 a.m. on Wednesday
(b) 5:00 a.m. on Wednesday 1
27. In 2 hours the hour hand of a clock rotates through an
(c) 1:00 p.m. on Wednesday 2
(d) 5:00 p.m. on Wednesday angle of
(e) None of these (a) 90° (b) 140° (c) 120°
(d) 75° (e) None of these
22. An application was received by inward clerk in the afternoon
28. On 27 March, 1995 was a Monday. Then what days of the
of a week day. Next day he forwarded it to the table of the
week was 1 November, 1994?
senior clerk, Who was on leave that day. The senior clerk
(a) Monday (b) Sunday
put up the application to the desk officer next day in the
(c) Tuesday (d) Wednesday
evening. The desk officer studied the application and
29. 16 January 1997 was a Thursday. What day of the week
disposed off the matter on the same day i.e., Friday. Which
was 4 January 2000?
day was the application received by the inward clerk ?
(a) Tuesday (b) Wednesday (c) Thursday
(a) Monday
(d) Friday (e) None of these
(b) Wednesday
30. In a year 28th February is Tuesday; if the leap year is
(c) Tuesday
excluded, then 28th March will be a
(d) Previous week’s Saturday
(a) Sunday (b) Tuesday (c) Monday
(e) None of these (d) Saturday (e) None of these

16. a b c d e 17. a b c d e 18. a b c d e 19. a b c d e 20. a b c d e


RESPONSE 21. a c d e 22. a c d e 23. a c d e 24. a c d e 25. a c d e
b b b b b
GRID
26. a b c d e 27. a b c d e 28. a b c d e 29. a b c d e 30. a b c d e
y
o
u
rs
m
a
h
b
20

o
20

o
b
.w
Data Sufficiency

o
rd
p
re
s
s
.c
o
m
Max. Marks : 25 No. of Qs. 25 Time : 20 min. Date : ........./......../................
7. In a row of girls facing North, what is D’s position from the
DIRECTIONS (Qs. 1 - 20) : Each question below is followed by
left end?
two statements I and II. You are to determine whether the data
I. D is twentieth from the right end.
given in the statement is sufficient to answer the question. You
II. There are ten girls between B and D.
should use the data and your knowledge of Mathematics to choose
8. Town M is towards which direction of Town K?
between the possible answers.
I. Town K is towards North-West of Town D.
Give answer (a) if the statement I alone is sufficient to answer the
II. Town M is towards South - East of Town D.
question, but the statement II alone is not
9. How many daughters does P have?
sufficient.
I. K and M are sisters of T.
Give answer (b) if the statement II alone is sufficient to answer
II. T’s father is husband of P’s mother.
the question, but the statement I alone is not
10. Towards which direction is Village M from Village T?
sufficient.
I. Village P is to the south of Village M and Village P is to
Give answer (c) if both statements I and II together are needed to
the west of Village T.
answer the question.
II. Village K is to the east of Village M and Village K is to
Give answer (d) if either the statement I alone or the statement II
the north of Village T.
alone is sufficient to answer the question.
11. How is D related to M?
Give answer (e) if you cannot get the answer from the statements
I. K and D are the only sisters of R.
I and II together, but need even more data.
II. M is married to R’s father.
1. What is the age of C, in a group of A, B, C, D and E, whose
12. What is R’s position from the left end in a row of children
average age is 45 years?
facing South?
I. Average of the ages of A and B is 53 years.
I. There are forty children in the row.
II. Average of the ages of D and E is 47 years
II. D is tenth to the left of R and fifteenth from the right
2. Tower ‘P’ is in which direction with respect to tower ‘Q’?
end of the row.
I. P is to the West of H, which is to the South of Q.
13. Towards which direction was D facing when he started his
II. F is to the West of Q and to the North of P.
journey?
3. How is K related to N?
I. D walked 20 metres after he started, took a right turn
I. N is the brother of M, who is the daughter of K.
and walked 30 metres and again took a right turn and
II. F is the husband of K
faced West.
4. What is Nidhi’s age?
II. D walked 20 metres after he started, took a left turn and
I. Nidhi is 3 times younger than Rani.
walked 30 metres and again took a left turn and faced
II. Surekha is twice the age of Rani and the sum of their
West.
ages is 72.
14. How many daughters does A have?
5. What is Seema’s age?
I. A has four children.
I. Seema’s age is half of Reema age
II. B and C are sisters of D who is son of A.
II. Reema is 5 years younger than her sister.
15. How far is A from the starting point?
6. What is Deepali’s age?
I. A moves 5 km. towards East, then 2 km. towards left,
I. Deepali is two times younger than Nisha.
10 km, towards right and finally. 2 km, towards right
II. Supriya is twice the age of Nisha.
and stops
II. A moves 2 km. towards East, then 2 km. towards right,
13 km, towards left and finally, 2 km. towards left and
stops.

1. a b c d e 2. a b c d e 3. a b c d e 4. a b c d e 5. a b c d e
RESPONSE 6. a b c d e 7. a b c d e 8. a b c d e 9. a b c d e 10. a b c d e
GRID 11. a b c d e 12. a b c d e 13. a b c d e 14. a b c d e 15. a b c d e
y
o
u
rs
m
a
h
40 SPEED TEST 20

b
o
o
16. In a row of 40 students facing North, how many students are (b) if the data in statement II alone are sufficient to answer the

b
.w
there between R and S? question, while the data in statement I alone are not sufficient

o
I. S’s position in the row is 15th from the right end. to answer the question.

rd
II. R’s position in the row is 4th from the left end. (c) If the data either in statement I alone or in statement II alone

p
re
17. How many children does Suneeta have? are sufficient to answer the question.

s
I. X is the only daughter of Suneeta. (d) if the data given in both the statements I and II together are

s
.c
II. Y is brother of X. not sufficient to answer the question.

o
m
18. Pole X is in which direction with respect to pole Y? (e) if the data given in both the statements I and II together are
I. Pole H is to the north-east of pole X and to the north of necessary to answer the question.
pole Y. 21. In a row of girls facing North, what is D’s position from the
II. Pole R is to the east of pole X and to the north of pole left end?
Y. I. D is twentieth from the right end.
19. How many children does Seema have? II. There are ten girls between Band D.
I. Seema, the mother of Varsha’s sister has only one son. 22. Town M is towards which direction of Town K?
II. Varsha has only three siblings. I. Town K is towards North-West of Town D
20. How is Anil related to Sanjay? II. Town M is towards South-East of Town D
I. Sanjay’s son is the brother of only sister of Anil. 23. How many daughters does P have?
II. Radhika, the only daughter of Sanjay has only two I. K and M are sisters of T.
brothers. II. T’s father is husband of P’s mother.
DIRECTIONS (Qs. 21-25) : Each of the questions below consists 24. On which day of the week from Monday to Sunday did Arun
of a question and two statements numbered I and II given below leave for London?
it. You have to decide whether the data provided in the statements I. Arun did not leave for London during the weekend.
are sufficient to answer the question. Read both the statements II. Arun’s brother left for London on Friday two days after
and Give answer Arun left for London.
(a) if the data in statement I alone are sufficient to answer the 25. How is ‘new’ written in a code language?
question, while the data in statement II alone are not sufficient I. ‘new good clothes’ is written as ‘5 3 9’ in that code
to answer the question. language.
II. ‘good clothes are costly’ is written as ‘9673’ in that
code language.

RESPONSE 16. a b c d e 17. a b c d e 18. a b c d e 19. a b c d e 20. a b c d e

GRID 21. a b c d e 22. a b c d e 23. a b c d e 24. a b c d e 25. a b c d e


y
o
u
rs
m
a
h
b
21

o
o
b
.w
Statement & Conclusion

o
rd
p
(Mathematical)

re
s
s
.c
o
m
Max. Marks : 22 No. of Qs. 22 Time : 20 min. Date : ........./......../................
DIRECTIONS (Qs. 1-4): In these questions, relationships between Give answer
different elements is shown in the statements. These statements (a) if only Conclusion I is true.
are followed by two conclusions. (b) if only Conclusion II is true.
Give answer (a) if only conclusion I follows. (c) if either Conclusion I or II is true.
Give answer (b) if only conclusion II follows.
Give answer (c) if either conclusion I or conclusion II follows. (d) if neither Conclusion I nor II is true.
Give answer (d) if neither conclusion I nor conclusion II (e) if both Conclusions I and II are true.
follows. 5. Statements: F @ N, N d R, H @ R
Give answer (e) if both conclusions I and II follow. Conclusions: I. H d N
1. Statement : II. F # R
A< L<T< R £ H>K 6. Statements: M # T, T@ K, K $ N
Conclusions :
Conclusions: I. M # N
I. H > L
II. K > T II. K d M
2. Statement : 7. Statements: T % H, H $ W
P= N > D ³ G< B= J Conclusions: I. W # T
Conclusions : II. W % T
I. G < P 8. Statements: N d K, K # D, D % M
II. G < J
Conclusions: I. M d K
3. Statement :
F£ C³ V=Z<X=U II. D d N
Conclusions : 9. Statements: J $ B, B % R, R d F
I. V < U Conclusions: I. F # B
II. Z < F II. R @ J
4. Statement :
DIRECTIONS (Qs. 10-14): In these questions symbols ©, #, *, $
Q£ E=I>N ³ R ³ S and @ are used with different meanings as follows:
Conclusions :
'A ©B' means ‘A is smaller than B’.
I. E = S
A # B means ‘A is either smaller, than or equal to B’.
II. S £ N
`A * B' means ‘A is greater than B’.
DIRECTIONS (Qs. 5-9): In the following questions, the symbols A $ B means ‘A is either greater than or equal to B’.
d, %, $, # and @ are used with the following meaning as illustrated A @ B means ‘A is neither smaller than nor greater than B’.
below: In each of the following questions assuming the given
‘P $ Q’ means ‘P is not smaller than Q’. statements to be true, find out which of the two conclusions
‘P @ Q’ means ‘P is not greater than Q’. I and II given below them is/are definitely true.
‘P d Q’ means ‘P is neither smaller than nor equal to Q’. Give answer (a) if only conclusion I is true.
‘P # Q’ means ‘P is neither greater than nor equal to Q’. Give answer (b) if onlyconclusion II is true.
‘P % Q’ means ‘P is neither smaller than nor greater than Q’. Give answer (c) if either conclusion I or conclusion II is true.
Give answer (d) if neither conclusion I nor conclusion II is
Now in each of the following questions assuming the given
true.
statements to be true, find which of the two conclusions I and II Give answer (e) if both conclusions I and II are true.
given below them is/are definitely true? 10. Statements:V # S, S © L, L © J
Conclusions: I. V © L
II. S © J

RESPONSE 1. a b c d e 2. a b c d e 3. a b c d e 4. a b c d e 5. a b c d e

GRID 6. a b c d e 7. a b c d e 8. a b c d e 9. a b c d e 10. a b c d e
y
o
u
rs
m
a
h
42 SPEED TEST 21

b
o
o
11. Statements: M # R, R © J, J # H 17. Statements:

b
Conclusions: I. M # H K # T, T $ B, B @ F

.w
II. R © H Conclusions: I. F $ T

o
rd
12. Statements:H $ F, F @ G, G « M II. K # B

p
Conclusions: I. H « M III. T $ F

re
II. H « G (a) None is true (b) Only I is true

s
13. Statements:R © J, J « T, T # L

s
(c) Only I and II are true (d) Only II and III are true

.c
Conclusions: I. R @ T (e) All are true

o
m
II. J @ L 18. Statements:
14. Statements: W @ T, T $ K, K « F Z # F, R @ F, D c R
Conclusions: I. W $ K Conclusions: I. Z # R
II. W @ K II. F # D
DIRECTIONS (Qs.15-19) : In the following questions, the symbols III. D @ Z
(a) None is true (b) Only I is true
@, #, %, $ and c are used with the following meaning as illustrated
(c) Only III is true (d) Only either I or III is true
below:
(e) All are true
'P # Q' means 'P is neither greater than nor equal to Q'.
19. Statements:
'P c Q' means 'P is neither equal to nor smaller than Q'
M c R, R % D, D @ N
'P % Q' means 'P is neither smaller than nor greater than Q.
Conclusions: I. M c N
'P $ Q' means 'P is not smaller than Q'
'P @ Q' means 'P is not greater than Q'. II. N $ R
Now in each of the following questions, assuming the given III. M c D
statements to be true, find which of the three conclusions I, II and (a) Only I and II are true (b) Only II and III are true
III given below them is/are definitely true and give your answer (c) Only I and III are true (d) All are true
accordingly. (e) None of these
15. Statements: R @ D, D c W, B $ W DIRECTIONS (Qs.20-22) : In these questions, relationship
Conclusions: between different elements is shown in the statements. These
I. W # R II. B c D III. W $ R statements are followed by two conclusions.
(a) None is true (b) Only I is true Mark answers if
(c) Only III is true (d) Only either I or III is true (a) Only conclusion I follows.
(e) All are true (b) Only conclusion II follows.
16. Statements: (c) Either conclusion I or II follows.
H $ V, V % M, K c M (d) Neither conclusion I nor II follows.
Conclusions: I. K c V (e) Both conclusion I and II follows.
20. Statement: P ³Q=R>S>T
II. M @ H
Conclusions: I. P ³ T
III. H c K
II. T < Q
(a) Only I and III are true (b) Only II and III are true 21. Statement: L £M<N>O³P
(c) Only I and II are true (d) All are true Conclusions: I. O < M
(e) None of these II. P £ N
22. Statement: A > B, B ³ C = D < E
Conclusions: I. C < A
II. D £ B

11. a b c d e 12. a b c d e 13. a b c d e 14. a b c d e 15. a b c d e


RESPONSE
16. a b c d e 17. a b c d e 18. a b c d e 19. a b c d e 20. a b c d e
GRID
21. a b c d e 22. a b c d e
y
o
u
rs
m
a
h
b
22

o
22

o
Statement &

b
.w
o
rd
Conclusion (Logical)

p
re
s
s
.c
o
m
Max. Marks : 20 No. of Qs. 20 Time : 15 min. Date : ........./......../................
DIRECTIONS (Q. 1 to 18) : In each question below is given a 4. Statement : Population increase coupled with depleting
statement followed by two conclusions numbered I and II. You resources is going to be the scenario of many developing
have to assume everything in the statement to be true, then countries in days to come.
consider the two conclusions together and decide which of them Conclusions :
logically follows beyond a reasonable doubt from the information I : The population of developing countries will not
given in the statement. continue to increase in future.
Give answer (a) if only conclusion I follows. II : It will be very difficult for the governments of
Give answer (b) if only conclusion II follows. developing countries to provide its people decent
Give answer (c) if either I or II follows. quality of life.
Give answer (d) if neither I nor II follows. 5. Statement : Mr. X is one of the probable candidates
Give answer (e) if both I and II follows. shortlisted for the post of Director of K. L. M. Institute.
1. Statement : Although we have rating agencies like Crisil. Conclusions :
ICRA, there is demand to have a separate rating agency for I : Mr. X will be selected as Director of K. L. M. Institute.
IT Companies to protect investors. II : Mr. X will not be selected as Director of K. L. M.
Conclusions : Institute.
I : Assessment of financial worth of IT Companies calls 6. Statement :
for separate set of skills, insight and competencies. ‘We follow some of the best and effective teaching learning
II : Now the investors investing in I. T. Companies will get practices used by leading institutes all over the world’. — A
protection of their investment. statement of professor of MN Institute.
2. Statement : Company "Y" will improve the manufacturing Conclusions :
I. The MN Institute is one of the leading institutes of the
facilities for the production of shaving kits as a result of
world.
which capacity would increase and cost would be reduced –
II. Whatever is being followed by world’s leading institutes
A spokesperson of the Company "Y". will definitely be good and useful.
Conclusions : 7. Statement :
I : The products of Company "Y" will complete the market In the absence of national health insurance or social security
norms in the quality and cost factor. cover, a person with limited resources has to depend on
II : There will be demand of shaving kits of Company "Y"? government hospitals, which are crowded, overburdened
3. Statement : During 1997-98 the total loss incurred by the and understaffed.
111 Public Sector Units was to the tune of ` 6809 crore, Conclusions :
which was converted into paid capitals by the Government I. National health insurance is meant only for the affluent
of its total investment of ` 5129 crore. sections of society.
Conclusions : II. The government hospitals provide treatment on nominal
I : The Government is left with only one option that is to charges or free.
privatise these units. 8. Statement :
II : The Government did not take care in the matter of We do not need today in India extraordinary specialists but
investments in these public sector units. those trained ordinary doctors who are dedicated to their
profession.
Conclusions :
I. We should promote medical profession with dedicated
ordinary doctors rather than promoting high specialised
medical education.
II. Extraordinary specialists are not dedicated to their
profession.

RESPONSE 1. a b c d e 2. a b c d e 3. a b c d e 4. a b c d e 5. a b c d e

GRID 6. a b c d e 7. a b c d e 8. a b c d e
y
o
u
rs
m
a
h
44 SPEED TEST 22

b
o
o
9. Statement : 16. Statements :

b
“The Government will review the present policy of the diesel A study of planning commission reveals boom in revenues.

.w
price in view of further spurt in the international oil prices” However, this has been of little avail owing to soaring

o
expenditure. In the event, there has been a high dose of

rd
— A spokesman of the Government.
deficit financing, leading to marked rise in prices.

p
Conclusions :

re
Large financial outlays year after year had little impact on
I. The Government will increase the price of the diesel after

s
the standard of living.

s
the imminent spurt in the international oil prices.

.c
Conclusions :
II. The Government will not increase the price of the diesel

o
I. A boom in revenues leads to soar in prices.

m
even after the imminent spurt in the international oil II. Large financial outlays should be avoided.
prices. 17. Statements :
10. Statement : The average number of students per teacher is 50 in the urban
Vegetable prices are soaring in the market. area whereas it is 60 in rural areas. The national average is 55.
Conclusions : Conclusions :
I. Vegetables are becoming a rare commodity. I. The student-teacher ratio in the rural areas is higher than
II. People cannot eat vegetables. in the urban areas.
11. Statement : II. More students study with the same teacher in the rural
Being from a business family, Chandan was apparently areas as compared to those in the urban areas.
18. Statement :
convinced by his parents and other family members to join
Morning walks are good for health.
the family trade. Conclusions :
Conclusions : I. All healthy people go for morning walks.
I. People should take up their family profession so that II. Evening walks are harmful.
family prospers. 19. Statement : The cost of manufacturing cars in state A is 30
II. It is necessary to keep in family members happy by per cent less than the cost of manufacturing cars in state B.
choosing family’s business. After transportation fee for the differential distances of states
12. Statement : A and B and the interstate taxes, it is cheaper to manufacture
Global ecological issues have eclipsed local environmental cars in state B than in state A for selling these cars in State C.
problems which are being faced by the poor societies. Which of he following supports the conclusion draw in the
Conclusions : above statement?
I. Poor societies always have to suffer because of their (a) The cost of transportation from state A to state C is
more than 30 per cent of the production cost.
poverty.
(b) The production cost of cars in state B is lower in
II. Global ecological issues are not so important. Rich comparison to state A.
societies can bear with it. (c) Only entry tax at state C is more for the production
13. Statements : originating in state A.
Of the ten fishermen caught in a storm, nine managed to (d) Entry tax at state C is more for the products originating
return to the shore. in state B.
Praveen has not yet returned after four days. (e) The total of transportation cost of cars from state B to
Conclusions : state C and entry tax of cars at state C is less than 30
I. Praveen got killed in the storm. per cent of the production cost of cars in state B.
II. Praveen has survived the storm. 20. Statement : There was a slow decline in the number of
14. Statements : patients with flu-like symptoms visiting various health
Now you don’t need an import licence to own a VCR. facilities in the city during the last fortnight.
Which of the following substantiates the fact mentioned in
Conclusions :
the above statement?
I. VCRs are now manufactured indigenously. (a) Majority of the people suffering from flu visit the health
II. VCRs are now freely permitted to be imported. facilities in the city.
15. Statements : (b) There has been a continuous increase in the sale of
Just about everyone in Germany has been on a diet at one medicines for curing flu in the city limits.
time or the other and millions of them have learned that the (c) People have started visiting the crowded places like
weight they lose is all too easily regained. malls and cinema halls during the last fortnight after a
Still’ despite their frustration, few question the wisdom of cautioned gap of one month.
dieting. (d) There is a sudden increase in the number of deaths
Conclusions : caused by flu-like symptoms followed by respiratory
I. Germany should stop dieting. complications.
II. Germans do not learn from experience. (e) None of these

9. a b c d e 10. a b c d e 11. a b c d e 12. a b c d e 13. a b c d e


RESPONSE
14. a b c d e 15. a b c d e 16. a b c d e 17. a b c d e 18. a b c d e
GRID
19. a b c d e 20. a b c d e
y
o
u
rs
m
a
h
23

b
o
23

o
b
Passage Based

.w
o
rd
p
Conclusion-1

re
s
s
.c
o
m
Max. Marks : 15 No. of Qs. 15 Time : 15 min. Date : ........./......../................
DIRECTIONS (Qs. 1-3): Study the following information carefully 18.2% rural population moved out of poverty whereas another 22.1%
and answer the given questions. fell into it over this period. This net increase of about four percentage
points was seen to have a considerable variation across states and regions.
The prospects for the Indian economy this year will be influenced by the
4. Which of the following is a conclusion which can be drawn from
behaviour of the monsoon and expansion of commerce and trade. The
the facts slated in the above paragraph ?
Eleventh Plan has envisaged a growth target of 8%. If the agriculture
(a) Accurate estimates of number of people living below poverty
sector does well and the world trade conditions improve then it is possible
line in India is possible to be made.
to achieve a growth of 6-7%. We need to improve our economy and aim
(b) Many expert groups in dia are not interested measure poverty
at a higher rate of growth in order to feed our population, maintain the
objectively.
standard of living and improve the quality of life. It is now more than 10
(c) Process of poverty measurement needs to take into account
years since we have adopted reforms. We need to go forward in
various factors to tackle its dynamic nature.
liberalisation but we cannot throw open the market for everything. There
(d) People living below poverty line remain in that position for
are sectors like village industries which need protection.
a very long time.
1. Which of the following is an assumption which is implicit in the
(e) None of these
facts stated in the above paragraph?
5. Which of the following is an assumption which is implicit in the
(a) India should adopt economic policies of developed countries.
facts stated In the above paragraph ?
(b) Free market strategy is beneficial for India, but not in all the
(a) It may not be possible to have an accurate poverty
sectors.
measurement in India.
(c) Over the last few years, we have achieved sustained growth.
(b) Level of poverty in India is static over the years.
(d) A very good monsoon is expected this year.
(c) Researchers avoid making conclusions on poverty
(e) None of these
measurement data in India.
2. Which of the following is an inference which can be drawn from the
(d) Government of India has a mechanism to measure level of
facts stated in the paragraph?
poverty effectively and accurately.
(a) The world trade conditions don't affect Indian economy.
(e) None of these
(b) The world trade conditions have a major impact on Indian
6. Which of the following is an inference which can be made from
economy.
the facts stated in the above paragraph ?
(c) Indian economy has been downgraded since last decade.
(a) Poverty measurement tools in India arc outdated.
(d) Govt should cut the subsidies in order to obtain sustained growth.
(b) Increase in number of persons falling into poverty varies
(e) None of these
considerably across the country over a period of time.
3. Which of the following is a conclusion which can be drawn from
(c) Government of India has stopped measuring poverty related
the facts stated in the above paragraph?
studies.
(a) India may become a super economic power some day.
(d) People living in rural areas are more susceptible to fall into
(b) The standard of living of people has continuously degraded
poverty over the time
in India.
(e) None of these
(c) Growth of Indian economy and a good mon soon are
complement of each other. DIRECTIONS (Qs. 7 to 9) : Study the given information carefully and
(d) Indian economy is on the peak of growth. answer the questions that follow:
(e) None of these Indian Navy's now believes that its worst fear has come true, and believes
it's unlikely to find any survivors onboard INS Sindhurakshak. The Navy
DIRECTIONS (Qs. 4-6) : Study the following Information carefully
also came out with a statement, which said that 'the state of these two
and answer the questions given below :
bodies and conditions within the submarine leads to firm conclusion that
Poverty measurement is an unsettled issue, both conceptually and finding any surviving personnel within the submarine is unlikely'.
methodologically. Since poverty is a process as well as an outcome; This is for the first time that the Indian Navy has got first-hand
many come out of it while others may be falling into it. The net effect of confirmation of fatality since two massive explosions rocked INS
these two parallel processes Is a proportion commonly identified as the Sindhurakshak. A Navy release said that the bodies were shifted to INHS
‘head count ratio’, but these ratios hide the fundamental dynamism that Asvini where a DNA identification test would be carried to identify the
characterises poverty in practice. The most recent poverty reestimates crewmember.
by an expert group has also missed the crucial dynamism. In a study The damage and destruction within the submarine around the control
conducted on 13.000 households which represented the entire country In room area indicates that the feasibility of locating bodies of personnel in
1993-94 and again on 2004-05. it was found that in the ten-year period the forward part of the submarine is also very remote as the explosion

RESPONSE 1. a b c d e 2. a b c d e 3. a b c d e 4. a b c d e 5. a b c d e

GRID 6. a b c d e
y
o
u
rs
m
a
h
46 SPEED TEST 23

b
o
o
and very high temperatures, which melted steel within, would have 11. Which of the following is an Assumption which is implicit in the

b
incinerated the bodies too, said Navy officials. But navy officials said facts stated in the above paragraph?

.w
that the divers would continue to search every inch of the submerged (a) Advertising guarantees the manufacturer in exhausting the

o
submarine till all bodies are either located or it can be stated with finality

rd
stocks he has produced.
that no bodies remain to be found.

p
(b) Advertising stimulates consumers to buy more products.

re
7. Which of the following is the conclusion which can be drawn
(c) Advertising leads to fierce competition.
from the facts stated in the above paragraph?

s
(d) A manufacturer manufactures a product because there is

s
(a) No personnel have survived in the INS Sindhurakshak tragedy.

.c
demand in the market.

o
(b) It is impossible to trace the bodies of all the personnel died in
(e) It is impossible to predict the demand of a product accurately.

m
the incident.
(c) The Navy is presently concentrating on reaching the interiors 12. Which of the following is an Inference which can be made from
of the submarine to locate and extricate any remaining bodies the facts stated in the above paragraph?
that may still be trapped within. (a) Manufacturers shall not try to predict the demand of a
(d) Navy's rescue and salvage effort is proceeding slowly because product.
it lacks expertise in conducting such an operation. (b) These days there are a lot of tools that helps in predicting the
(e) It was a massive accident leading to the death of all the sailing demand of a product accurately.
crew members. (c) Manufacturers must produce only what they can sell.
8. Which of the following is an assumption which is implicit in the (d) Consumer is the king.
facts stated in the above paragraph? (e) Advertising can minimise the risk taken by manufacturers.
(a) Some personnel could be found alive. 13. Our school district should not spend its money on the new Verbal
(b) The submarine and its internal machinery could be reused Advantage reading program. After all, our students get all the reading
thus saving costs for the Navy. practice they need by studying history and science.
(c) The temperature inside the submerged submarine would be The argument above depends on which the following assumptions?
very hot. (a) The Verbal Advantage program would not help the students
(d) There is lack of divers who are equally skilled in using gas learn history and science.
cutters or other machinery required to break open jammed (b) Other reading programs are just as effective but less expensive
hatches. than the Verbal Advantage program.
(e) Looking at the level of destruction more dead bodies would
(c) The Verbal Advantage program involves only reading practice.
be found.
9. Which of the following is an inference which can be made from
(d) Teaching students history and science is more important
the facts stated in the above paragraph?
than teaching them reading skills.
(a) The explosion and very high temperatures, which melted
steel within, would have incinerated the bodies too. (e) None of these
(b) Continuous efforts would lead to extrication of the bodies of 14. Efficiency is all right in its place, in the shop, the factory, the store.
all the personnel. The trouble with efficiency is that it wants to rule our play as well
(c) The bodies are expected to be so badly charred that as our work; it won’t be content to reign in the shop, it follows us
identification will not be possible. home.
(d) Submarines are not safe watercraft capable of carrying It can be inferred from the above passage that
independent operation underwater. (a) Efficiency can become all - pervading
(e) Old vessels and other crafts must be phased out so as to (b) Efficiency does not always pay
avoid such accidents in future. (c) Efficiency can be more of a torture than a blessing
DIRECTIONS (Qs 10 to 12): Study the given information carefully (d) both (b) and (c) (e) None of these
and answer the questions that follow: 15. The fare-paying capacity of people who travel on routes connecting
to small towns is very low. Most successful airlines which operate
Through advertisting, manufacturing exercises a high degree of control
in such regions have a large number of seats.
over consumer's desires. However, the manufacturer assumes enormous
risks in attempting to predict what consumers will want and in producing Which of the following can be inferred from the above information?
goods in quantity and distributing them in advance of final selection by (a) Regional airlines are quite profitable.
the consumers. (b) People from cities are increasingly travelling to small towns.
10. Which of the following is the conclusion which can be drawn (c) Regional airlines have to charge low fares in order to be
from the facts stated in the above paragraph? profitable.
(a) Distribute goods directly to the consumers. (d) The number of people travelling from small towns to cities is
(b) Can eliminate the risk of overproduction by advertising. massive.
(c) Always take moderate and calculated risk. (e) None of these
(d) Can predict with great accuracy the success of any product
they put on the market.
(e) Must depend upon the final consumers for the success of
their undertakings.

RESPONSE 7. a b c d e 8. a b c d e 9. a b c d e 10. a b c d e 11. a b c d e

GRID 12. a b c d e 13. a b c d e 14. a b c d e 15. a b c d e


y
o
u
rs
m
a
h
b
24

o
24

o
13

b
Passage Based

.w
o
rd
p
Conclusion-2

re
s
s
.c
o
m
Max. Marks : 15 No. of Qs. 15 Time : 15 min. Date : ........./......../................
DIRECTIONS (Qs. 1 to 3): Study the given information carefully and DIRECTIONS (Q. 4 and 5) : In the following questions a paragraph is
answer the questions that follow: given. Read the paragraph carefully and answer the questions which
follow each of these paragraph.
In economics, a recession is a business cycle contraction, a general
slowdown in economic activity. Macroeconomic indicators such as GDP Fashion has become one of the largest fads among the youth. The amount
(Gross Domestic Product), employment, investment spending, capacity of time wastage and expenditure on fashion is very large. What bothers,
utilization, household income, business profits, and inflation fall, while however, is the fact that fashion is here to stay despite countless arguments
bankruptcies and the unemployment rate rise. against it. What is required, therefore, is that strong efforts should be
Recessions generally occur when there is a widespread drop in spending made in order to displace the excessive craze of fashion from the minds of
(an adverse demand shock). This may be triggered by various events, such today’s youth.
as a financial crisis, an external trade shock, an adverse supply shock or the 4. Which of the following statements finds the least support by the
bursting of an economic bubble. Governments usually respond to recessions argument made by the author in the given paragraph?
by adopting expansionary macroeconomic policies, such as increasing money (a) Youngsters should be motivated to do constructive business
supply, increasing government spending and decreasing taxation. rather than wasting time on fashion.
1. Which of the following is the conclusion which can be drawn from (b) The world of fashion being glamorous and glittery attracts
the facts stated in the above paragraph? people towards itself.
(a) Recession leads to financial crisis. (c) Following the latest fashion increases the self-efficacy of
(b) Demand and supply play a major role in determining the people, thus increasing their overall mental abilities.
fundamentals of an economy. (d) Many universities have implemented a dress code to put a
(c) Cash is the king. So always keep your money safe. check on the increasing fad amongst the youth which was
(d) In a recession, interest rates are reduced so as to enable people affecting their grades.
to take loans which bring back the money in the economy (e) None of these
thus increasing spending and economic activity. 5. Which of the following can be inferred from the given paragraph?
(e) Recession is a normal (albeit unpleasant) part of the business (a) The author has made strong efforts to wipe out fashion from
cycle and brings in correction in the market. the minds of youth.
2. Which of the following is an assumption which is implicit in the (b) Steps need to be taken in order to control the growing fad of
facts stated in the above paragraph? fashion amongst the youth.
(c) The author is upset with the shift” of fashion from the
(a) Recessions generally occur when there is a widespread drop
traditional ethnic wear to western outfits.
in spending.
(d) Fashion world is responsible for lack of creativity among the
(b) When economic activity slows down, firms may decide to
youth.
reduce employment levels and save money rather than invest.
(e) None of these
(c) A recession generally lasts from six to 18 months.
(d) Recession is the result of adverse supply shock or the bursting DIRECTIONS (Qs. 6 to 8): Study the given information carefully and
of an economic bubble. answer the questions that follow:
(e) A slow growing nation can protect itself from recession.
3. Which of the following is an inference which can be made from Due to enormous profits involved in smuggling, hundreds of persons
the facts stated in the above paragraph? have been attracted towards this anti-national activity. Some of them
(a) Recession leads to decline in consumption, investment, became millionaires overnight. India has a vast coastline both on the
Eastern and Western Coast. It has been a heaven for smugglers who have
government spending, and net export activity.
been carrying on their activities with great impunity. There is no doubt,
(b) In a recession people should generally try to watch their
that from time to time certain seizures were made by the enforcement
spending and not take any undue risks that might put their
authorities, during raids and ambush but even allowing these losses the
future financial goals in jeopardy.
smugglers made huge profits.
(c) Recession affects both the developed and developing nations
6. Which of the following is the conclusion which can be drawn from
of the world.
the facts stated in the above paragraph?
(d) Countries must reduce their imports drastically so as to avoid
(a) smuggling hampers the economic development of a nation.
recession. (b) smuggling ought to the curbed.
(e) Recession is a business cycle contraction which brings a (c) authorities are taking strict measures to curb smuggling.
general slowdown in economic activity. (d) smuggling is fast increasing in our country owing to the quick
profit it entails.
(e) None of these

RESPONSE 1. a b c d e 2. a b c d e 3. a b c d e 4. a b c d e 5. a b c d e

GRID 6. a b c d e
y
o
u
rs
m
a
h
48 SPEED TEST 24

b
o
o
7. Which of the following is an assumption which is implicit in the (c) Increase in income of all leads to a better life style of people.

b
facts stated in the above paragraph? (d) Increase in exports does not guarantee increase in income

.w
(a) Coastlines provide easy escape for the smugglers. levels of all class of people.

o
rd
(b) People are driven by money not by values. (e) The gap between rich and poor determines national
prosperity.

p
(c) Smugglers are smart enough to be caught.

re
(d) Government can earn a lot of money from seizing smuggled DIRECTIONS (Qs. 12 to 14): Study the given information carefully

s
goods.

s
and answer the questions that follow:

.c
(e) Smuggling cannot be stopped as smugglers will find new

o
The consumption of harmful drugs by the people can be prevented not
ways of doing it.

m
only by banning their sale in the market but also by instructing users
8. Which of the following is an inference which can be made from about their dangerous effects which they must understand for their safety.
the facts stated in the above paragraph? Also the drug addicts may be provided with proper medical facilities for
(a) The enforcement authorities lack in their efforts to curb their rehabilitation. This will help in scaling down the use of drugs.
smuggling. 12. Which of the following is the conclusion which can be drawn
(b) Authorities are taking bribe from smugglers there by from the facts stated in the above paragraph?
increasing their potential. (a) are on increase in the society
(c) There is no measure taken by authorities. (b) can always be reduced.
(d) Smuggling hampers the GOD of a nation. (c) are due to lack of medical facilities.
(e) None of these (d) can be eliminated with the help of banning their sale.
DIRECTIONS (Qs. 9 to 11): Study the given information carefully and (e) may be channelised through proper system.
answer the questions that follow: 13. Which of the following is an assumption which is implicit in the
Exports and imports, a swelling favourable balance of trade, investments facts stated in the above paragraph?
and bank-balances, are not an index or a balance sheet of national (a) Such drugs are harmful and can lead to severe depression and
prosperity. Till the beginning of the Second World War, English exports suicidal tendencies.
were noticeably greater than what they are today. And yet England has (b) People using drugs harm the society.
greater national prosperity today than it ever had. Because the income of (c) Scaling down the use of such harmful drugs is beneficial for
average Englishmen, working as field and factory labourers, clerks, the society.
policemen, petty shopkeepers and shop assistants, domestic workers (d) Medical facilities needs to be improved in our country.
and other low-paid workers, has gone up. (e) The government is not taking appropriate actions to curb the
9. Which of the following is the conclusion which can be drawn use of harmful drugs.
from the facts stated in the above paragraph? 14. Which of the following is an inference which can be made from
(a) a country’s economic standard can be best adjudged by per the facts stated in the above paragraph?
capita income. (a) Such harmful drugs should not be available at the first place.
(b) a country’s balance of trade is the main criteria of determining (b) Controlling drug addicts can scale down the use of drugs.
its economic prosperity.
(c) People using these drugs must be rehabilitated.
(c) a nation’s economy strengthens with the increase in exports.
(d) Awareness about the ill-effects of these drugs can scale down
(d) English trade has continually increased since the Second their usage.
World War.
(e) Bad company leads to spread of use of such drugs.
(e) None of these
15. “If you want a hassle-free holiday package for city M, then join
10. Which of the following is an assumption which is implicit in the only our tour. Hurry up; only a few seats available” – An
facts stated in the above paragraph? advertisement of XYZ Tourist Company.
(a) Increasing exports and decreasing imports helps a nation If the above statement is true then which of the following has been
prosper. assumed while making the statement?
(b) Prosperity of a nation is the outcome of prosperity of its (a) No seats may be available with other tour operators for city
people. M.
(c) Greater exports leads to higher GDP which leads to better (b) Nowadays people have a lot of money to spend on their
flow of money in the economy. comforts.
(d) Second World War led to deterioration in the national (c) Travel packages offered by other tour operators are neither
prosperity of England. cheap nor comfortable.
(e) None of these (d) Many people desire convenience and comfort while going
11. Which of the following is an inference which can be made from for a holiday.
the facts stated in the above paragraph? (e) None of these
(a) Countries must focus on increasing income of its people and
not an exports or imports.
(b) Favourable balance of trade will automatically lead to higher
income of people.

RESPONSE 7. a b c d e 8. a b c d e 9. a b c d e 10. a b c d e 11. a b c d e

GRID 12. a b c d e 13. a b c d e 14. a b c d e 15. a b c d e


y
o
u
rs
m
a
h
25

b
o
13

o
b
.w
Cube & Dice

o
rd
p
re
s
s
.c
o
m
Max. Marks : 20 No. of Qs. 20 Time : 15 min. Date : ........./......../................
1. Each of the six faces of a cube is numbered by one of the K H B
digits from 1 to 6. This cube is shown in its four different
positions in the figure I, II, III, and IV. B K H
A M P
6 1 5 6
What is the letter opposite to A?
(a) H (b) P (c) B
3 2 4 2 6 4 2 4 (d) M (e) None of these
DIRECTIONS (Qs. 5 -8) : A cube is coloured red on all faces. It is
cut into 64 smaller cubes of equal size. Now, answer the following
I II III IV questions based on this statement :
Consider the following statements. 5. How many cubes have no face coloured?
1. Figures II and III are sufficient to known as to which (a) 24 (b) 16 (c) 8
face is opposite to the face numbered 6. (d) 0 (e) None of these
2. figures II and III are sufficient to known as to which 6. How many cubes are there which have only one face
coloured?
face is opposite to the face numbered 4. . (a) 4 (b) 8 (c) 16
3. Figures I and IV are sufficient to known as to which (d) 24 (e) None of these
face is opposite to the face numbered 3. 7. How many cubes have two red opposite faces?
Which of the statements given above are correct? (a) 0 (b) 8 (c) 16
(a) 1 and 3 only (b) 1 and 2 only (d) 24 (e) None of these
(c) 2 and 3 only (d) 1, 2 and 3 8. How many cubes have three faces coloured?
(e) None of these (a) 24 (b) 16 (c) 8
2. Six faces of a cube are numbered from 1 to 6, each face (d) 4 (e) None of these
carrying one different number. Further,
1. The face 2 is opposite to the face 6.
2. The face 1 is opposite to the face 5. 9.
3. The face 3 is between the face 1 and the face 5
4. The face 4 is adjacent to the face 2. How many dots are there on the dice opposite to the one
Which one of the following is correct? dot?
(a) The face 2 is adjacent to the face 3 (a) 2 (b) 4 (c) 5
(b) The face 6 is between the face 2 and the face 4 (d) 6 (e) None of these
(c) The face 1 is between the face 5 and the face 6 10. Select from alternative, the box that can be formed by folding
(d) None of the above the sheet shown.
(e) None of these 1
3. Six squares are coloured, front and back, red (R), blue (B),
yellow (Y), green (G), white (W) and orange (O) and are 4 3 6
hinged together as shown in the figure given below. If they
are folded to form a cube, what would be the face opposite 2
the white face?
5
R B
G Y O 2 4 3
W (a) 3 5 (b) 6 3 (c) 2 5
(a) R (b) G (c) B
(d) O (e) None of these 4
4. Three views of a cube following a particular motion are given (d) 3 (e) None of these
below: 1

RESPONSE 1. a b c d e 2. a b c d e 3. a b c d e 4. a b c d e 5. a b c d e

GRID 6. a b c d e 7. a b c d e 8. a b c d e 9. a b c d e 10. a b c d e
y
o
u
rs
m
a
h
50 SPEED TEST 25

b
o
o
11. Select from alternative, the box that can be formed by folding 14. Which number is on the face opposite to 6 on the dice

b
the sheet shown. whose four positions as shown below ?

.w
o
1

rd
6 6 5
2

p
2 4 4 4
2 6

re
3
×

s
s
.c
(a) 1 (b) 2 (c) 3
¸

o
= (d) 4 (e) None of these

m
15. A cube is painted white on all the sides. It is then cut into 64
+ smaller cubes of equal sizes. How many of these smaller
cubes have no paint on any side?
– (a) 8 (b) 6 (c) 4
(d) 1 (e) None of these
× 16. How many cubes are there in the following figure?
(a) ¸ (b) ¸ (c) ¸
+ = ×

¸
(d) – × (e) None of these

12. Select from the alternative, the box that can be formed by
folding the sheet shown in figure (X) :

(a) 6 (b ) 10 (c) 12
(d) 8 (e) None of these
DIRECTIONS (Qs. 17 - 19) : The questions below are to be
answered on the basis of the three views of a cube given as follows:
5 4 2
4 6 6
(X) 1 3 5

17. Which number is on the face opposite to 1 ?


(a) 3 (b) 2 (c) 6
(d) 4 (e) None of these
(A) (B) (C) (D) 18. Which number is on the face opposite to 4 ?
(a) A only (b) A and C only (a) 2 (b) 3 (c) 6
(c) A , C and D only (d) A, B, C and D (d) 1 (e) None of these
(e) None of these 19. Which number is at the bottom face of figure 1?
13. Three positions of a cube are as shown below : (a) 3 (b) 2 (c) 6
(d) 1 (e) None of these
20. Four positions of a cube are shown below. Which symbol
is opposite to the face having ‘ ’?
?
÷ ÷
÷
× + +
×
The figure on the face opposite the triangle is the :
(a) (b) (c) (d)
(a) pentagon (b) circle
(c) question mark (d) rectangle (a) D (b) – (c) ×
(e) None of these (d) ÷ (e) None of these

RESPONSE 11. a b c d e 12. a b c d e 13. a b c d e 14. a b c d e 15. a b c d e

GRID 16. a b c d e 17. a b c d e 18. a b c d e 19. a b c d e 20. a b c d e


y
o
u
rs
m
a
h
26

b
o
26
13

o
Non-verbal

b
.w
o
rd
Reasoning - I

p
re
s
s
.c
o
m
Max. Marks : 15 No. of Qs. 15 Time : 15 min. Date : ........./......../................
DIRECTIONS (Qs. 1 to 5): In each of the questions given below which one of the five Answer Figures on the right should come after
the Problem Figures on the left, if the sequence were continued?

Problem Figures Answer Figures


1.
S C O B E B S O L O O L B
E E B K E S K O S L S S K
C S K L C S L O C E E E L C
O B K L L O B K C E S O L B K C E L C B K C B K C B K E S O

(a) (b) (c) (d) (e)

2.

(a) (b) (c) (d) (e)

3.

(a) (b) (c) (d) (e)

4. o o
c

c
c

o c c o c o o o
c

c o
o o c
c

(a) (b) (c) (d) (e)

5. E L S S S
L L
E E H H L L
=HOLE H K C C L H L L
O H L C H
K H S S H

(a) (b) (c) (d) (e)

RESPONSE 1. a b c d e 2. a b c d e 3. a b c d e 4. a b c d e 5. a b c d e

GRID
y
o
u
rs
m
a
h
52 SPEED TEST 26

b
o
o
DIRECTIONS (Qs. 6 to 10) : In each of the following questions a 12. Problem Figures

b
.w
series begins with an unnumbered figure on the extreme left. One

o
and only one of the five lettered figures in the series does not fit

rd
into the series. The two unlabelled figures, one each on the extreme

p
left and the extreme right, fit into the series. You have to take as

re
Answer Figures

s
many aspects into account as possible of the figures in the series

s
.c
and find out the one and only one of the five lettered figures which

o
does not fit into the series. The letter of that figure is the answer.

m
6.
(a) (b) (c) (d) (e)
s s s s s s s
13. Problem Figures

= ­ ­= ­=
(a) (b)
=
(c)
­­
(d)
= ­=
(e)
= ­
?
Answer Figures
7. N ×
×
N

(a) (b) (c) (d) (e)


(a) (b) (c) (d) (e)
14. Problem Figures
8. ×
×
(a) (b) (c) (d) (e) ?
Answer Figures
s c s c s c
9. c s
c s s c c s

(a) (b) (c) (d) (e)


(a) (b) (c) (d) (e)
15. Problem Figures
10.

(a) (b) (c) (d) (e) ?


DIRECTIONS (Qs. 11-15) : In each of the questions given below Answer Figures
which one of the five answer figures on the bottom should come
after the problem figures on the top if the sequence were
continued?
11. Problem Figures (a) (b) (c) (d) (e)
z s * o =
o z
o s ? =
* *
Answer Figures

o * o * *
z * o s z s * o s o =
(a) (b) (c) (d) (e)

RESPONSE 6. a b c d e 7. a b c d e 8. a b c d e 9. a b c d e 10. a b c d e

GRID 11. a b c d e 12. a b c d e 13. a b c d e 14. a b c d e 15. a b c d e


y
o
u
rs
m
a
h
b
27

o
27

o
13
Non-verbal

b
.w
o
rd
Reasoning - II

p
re
s
s
.c
o
m
Max. Marks : 15 No. of Qs. 15 Time : 15 min. Date : ........./......../................

DIRECTIONS (Qs. 1 to 5): In each of the questions, there are two 3. Problem Figures
sets of figures. The figures on upper side are problem figures × ×
marked by letters (1), (2), (3), (4) and (5), and on the bottom side
are answer figures marked by numbers (a), (b), (c), (d) and (e). A ? ×
series is established, if one of the five answer figures is placed in
place of the (?) sign in the problem figures. That figure is your (1) (2) (3) (4) (5)
answer. Answer Figures
1. Problem Figures

? (a) (b) (c) (d) (e)


4. Problem Figures
(1) (2) (3) (4) (5)
Answer Figures
?
(1) (2) (3) (4) (5)
Answer Figures
(a) (b) (c) (d) (e)
2. Problem Figures
· S ·
= S S 0 C S C ?
= 0 C ? · ? = 0 (a) (b) (c) (d) (e)
? C 0 ? = · 5. Problem Figures
(1) (2) (3) (4) (5)
Answer Figures ?
· S C 0 · S· S ? S C = S C
(1) (2) (3) (4) (5)
0 = C 0 = 0 = ? · Answer Figures
? ? = C ? 0 ·
(a) (b) (c) (d) (e)

(a) (b) (c) (d) (e)

DIRECTIONS (Qs. 6 to 10): In each of the following questions, a related pair of figures is followed by five lettered pairs of figures. Select
the pair that has relationship similar to that in the question figure. The best answer is to be selected from a group of fairly close choices.
6.

I II I II I II I II I II I II
(a) (b) (c) (d) (e)

RESPONSE 1. a b c d e 2. a b c d e 3. a b c d e 4. a b c d e 5. a b c d e
GRID
y
o
u
rs
m
a
h
54 SPEED TEST 27

b
o
o
7.

b
.w
o
rd
I II I II I II I II I II I II

p
(a) (b) (c) (d) (e)

re
s
8.

s
.c
o
m
I II I II I II I II I II I II
(a) (b) (c) (d) (e)
9.

I II I II I II I II I II I II
(a) (b) (c) (d) (e)
10.

I II I II I II I II I II I II
(a) (b) (c) (d) (e)

DIRECTIONS (Qs. 11-15): In each of the questions given below which one of the following five answer figures should come after the
problem figures, if the sequence were continued?
11. Problem Figures Answer Figures
Z O E VP TD SA Z P U

P
P

E V P T D SA ZB U U U
U

P
Answer Figures (a) (b) (c) (d) (e)
B B B = = 14. Problem Figures
C D C D C C
= A Z = A Z B s Dz z s DD s
(a) (b) (c) (d) (e) z o Co s s o o z z o
12. Problem Figures Answer Figures
DZ T DO T z o
C
DO T DZ Z T O
D C D s z C C z C
T Z DZ z o s z o o D s D s o D s
Answer Figures (a) (b) (c) (d) (e)
DZ Z D O 15. Problem Figures
O DO T DZ T Z DZ T
T T O O
(a) (b) (c) (d) (e)
13. Problem Figures Answer Figures
U
P U
U
U

P
P
U

P
P

(a) (b) (c) (d) (e)

RESPONSE 6. a b c d e 7. a b c d e 8. a b c d e 9. a b c d e 10. a b c d e
GRID 11. a b c d e 12. a b c d e 13. a b c d e 14. a b c d e 15. a b c d e
y
o
u
rs
m
a
h
28

b
o
13
28

o
Section Test :

b
.w
o
rd
Reasoning

p
re
s
s
.c
o
m
Max. Marks : 40 No. of Qs. 40 Time : 25 min. Date : ........./......../................
1. How many meaningful three letter English words can be 7. In a certain code DISPLAY is written as RHCQZBM. How is
formed with the letters AER, using each letter only once in GROUPED written in that code?
each word ? (a) PSHTEFQ (b) NQFVCDO
(a) None (b) One (c) NQFVEFQ (d) PSHTCDO
(c) Two (d) Three (e) None of these
(e) Four 8. Among P, Q, R, T and W each having different weight, T is
2. If in a certain language LATE is coded as 8 & 4 $ and HIRE heavier than W and lighter than only P. Q is not the lightest.
is coded as 7*3$ then how will HAIL be coded in the same Who among them is definitely the lightest ?
language ? (a) R (b) W
(a) 7 & 8* (b) &7*8 (c) R or W (d) Data inadequate
(c) 7*& 8 (d) 7&*8 (e) None of these
(e) None of these 9. In a row of thirty children M is sixth to the right of R who is
3. Four of the following five are alike in a certain way and so twelfth from the left end. What is M’s position from the right
form a group. Which is the one that does not belong to that end of the row ?
group ? (a) Twelfth (b) Thirteenth
(a) Stem (b) Tree (c) Fourteenth (d) Data inadequate
(c) Root (d) Branch (e) None of these
(e) Leaf 10. What should come next in the following letter series?
4. Four of the following five are alike in a certain way and so B D F H J L N A C E G I K M B D F H J LA C E G I K B D F H J
form a group. Which is the one that does not belong to that (a) B (b) L
group ? (c) M (d) F
(a) Clutch (b) Wheel (e) None of these
(c) Break (d) Car DIRECTIONS (Qs. 11-15) : Study the following arrangement
(e) Gear carefully and answer the questions given below
5. How many such pairs of letters are there in word SENDING,
M, D, K, R, T, H, W and A are sitting around a circle facing at the
each of which has as many letters between its two letters as
centre. D is second to the right of M who is fifth to the left of T. K
there are between them in the English alphabets ?
is third to the right of R who is second to the right of D. H is
(a) None (b) One
second to the right of W.
(c) Two (d) Three
11. Who is second to the right of A ?
(e) More than three
(a) M (b) D
6. Four of the following five are alike in a certain way and so
(c) K (d) Data inadequate
form a group. Which is the one that does not belong to that
(e) None of the above
group?
12. Who is third to the left of M ?
(a) 169 (b) 441
(a) A (b) T
(c) 361 (d) 529
(c) H (d) D
(e) 289
(e) Data inadequate

1. a b c d e 2. a b c d e 3. a b c d e 4. a b c d e 5. a b c d e
RESPONSE 6. a b c d e 7. a b c d e 8. a b c d e 9. a b c d e 10. a b c d e
GRID 11. a b c d e 12. a b c d e
y
o
u
rs
m
a
h
56 SPEED TEST 28

b
o
o
13. Who is fourth to the right of H ? 19. Statements:

b
.w
(a) A (b) T All puppies are tigers.

o
(c) R (d) K All kittens are tigers.

rd
(e) None of these Conclusions:

p
re
14. In which of the following combinations is the first person I. All puppies are kittens.

s
sitting between the second and the third person ? II. All tigers are puppies.

s
.c
(a) KMW (b) MWD 20. Statements: Some doctors are nurses.

o
m
(c) RHT (d) TAK All nurses are patients.
(e) None of the above Conclusions:
15. In which of the following pairs is the second person sitting I. All doctors are patients.
to the immediate left of the first person ? II. Some patients are doctors.
(a) MW (b) AK DIRECTIONS (Qs.21-25) : Study the following information to
(c) TA (d) RH answer the given questions :
(e) WD Six plays – A, B, C, D, E and F are to be staged on six days of the
DIRECTIONS (Qs. 16 - 20): In each question below, there are week starting from Monday and ending on Saturday. Play C is
three of two statements followed by two conclusions numbered staged on Tuesday. Plays A, F and B are staged one after the
I and II. You have to take the three given statements to be true other in the same order. Play D is not staged on Monday or
even if they seem to be at variance from commonly known facts Wednesday.
and then decide which of the given conclusions logically follow 21. How many plays are staged after play A is staged ?
from the three statements. (a) One (b) Two
Give answer (a) if only conclusion I follows. (c) Three (d) Four
Give answer (b) if only conclusion II follows. (e) Cannot be determined
Give answer (c) if either I or II follows. 22. Four of the following five form a group based on the days
Give answer (d) if neither I nor II follows. that they are staged. Which one of them does not belong to
Give answer (e) if both I and II follow. that group ?
16. Statements: (a) EC (b) FD
All fish are birds. (c) CA (d) AF
Some hens are fish. (e) BD
Conclusion: 23. Which play is staged immediately before the day play E is
I. Some hens are birds. staged ?
II. No birds are hens; (a) B (b) A
17. Statements: (c) F (d) D
Some shoes are coats. (e) There is no such play staged
Some coats are buttons 24. If play D was staged on a Monday, which of the following
Conclusions: plays would definitely be staged on a Saturday (all the other
I. No button is shoe. conditions given above remain the same) ?
II. Some shoes are buttons. (a) B (b) A
18. Statements: (c) E (d) E or B
All bats are boys. (e) Cannot be determined
All boys are gloves. 25. Which play is staged on Thursday ?
Conclusions: (a) B (b) E
I. Some gloves are bats. (c) D (d) F
II. All bats are gloves.. (e) Cannot be determined

13. a b c d e 14. a b c d e 15. a b c d e 16. a b c d e 17. a b c d e


RESPONSE 18. a c d e 19. a c d e 20. a c d e 21. a c d e 22. a c d e
b b b b b
GRID
23. a b c d e 24. a b c d e 25. a b c d e
y
o
u
rs
m
a
h
SPEED TEST 28 57

b
o
o
DIRECTIONS (Qs.26-28) : In each question a group of letters is 30. Which of the following is an Assumption which is implicit in

b
.w
given followed by four combinations of number/symbol numbered the facts stated in the above paragraph

o
(a), (b), (c) and (d). Letters are to be coded as per the scheme and (a) Whatever you do you cannot protect yourself from

rd
conditions given below. You have to find out the serial number of fashion.

p
re
the combination, which represents the letter group. Serial number (b) Fashion changes the outward appearance and makes

s
of that combination is your answer. If none of the combinations is one feel better.

s
.c
correct, your answer is (e) i.e. None of these. (c) Today's youth is highly influenced by fashion.

o
m
(d) Normal clothing is good as it keeps a person grounded.
Letters Q M S I N G D K A L P R B J E (e) Fashion is spoiling the minds of the youngsters.
Number/ 7 @ 4 # % $ 6 1 2 £ 5 * 9 8 3
Symbol
31. Which of the following is an Inference which can be made
from the facts stated in the above paragraph?
Conditions : (a) Moderate investment of time and money by youth is
(i) If the first letter is a consonant and the last a vowel, both are acceptable.
to be coded as the code of the vowel.
(b) Fashion brings with itself feelings of materialism.
(ii) If the first letter is vowel and the last a consonant, the codes
(c) All of us have to adopt ourselves to latest fashion to
for the first and the last are to be interchanged. some bit.
(iii) If no vowel is present in the group of letters, the second and (d) Fashion helps in projecting a better image of one's
the fifth letters are to be coded as ©. personality.
26. BARNIS (e) People who will avoid fashion completely will be
(a) 9 2 * % # 4 (b) 9 2 4 # * %
outdated.
(c) 9 2 * # % 9 (d) 4 2 * # % 4
DIRECTIONS (Qs. 32-35) % In the following questions, the symbols
(e) None of these
27. DMBNIA @, #, $, © and % are used with the following meaning as illustrated
(a) 6 @ 9 % # 2 (b) 2 @ 9 % # 6 below :
(c) 2 @ 9 % # 6 (d) 2 © 9 % # 2 'P@ Q' means ‘P is not smaller than Q’,
(e) None of these 'P# Q' means ‘P is not greater than Q’.
28. IJBRLG
'P$ Q' means ‘P is neither greater than nor equal to Q’.
(a) # 8 9 * £ $ (b) # 8 9 * £ #
(c) $ 8 9 * £ # (d) $ 8 9 * £ $ 'P© Q' means ‘P is neither smaller than nor equal to Q’.
(e) None of these 'P% Q' means ‘P is neither smaller than nor greater than Q’.
DIRECTIONS (Qs. 29-31) : Study the given information carefully Now in each of the following questions, assuming the given
and answer the questions that follow: statements to be true, find which of the two conclusions I
Though the waste of time or the expenditure on fashions is very and II given below them is/are definitely true.
large, yet fashions have come to stay. They will not go, come what Give answer (a) if only conclusion I is true.
may. However, what is now required is that strong efforts should Give answer (b) if onlyconclusion II is true.
be made to displace the excessive craze for fashion from the minds Give answer (c) if either conclusion I or conclusion II is true.
of these youngsters. Give answer (d) if neither conclusion I nor conclusion II is
29. Which of the following is the conclusion which can be drawn true.
from the facts stated in the above paragraph? Give answer (e) if both conclusions I and II are true.
(a) Fashion is the need of the day. 32. Statements: % V $ W, W@T, T#H
(b) The excessive craze for fashion is detrimental to one's Conclusions: I. V©T
personality. II. H%W
(c) The hoard for fashion should be done away with so as 33. Statements: H © M, M @ E, E $ C
not to let down the constructive development.
Conclusions: I. C@M
(d) Work and other activities should be valued more than
the outward appearance. II. H©E
(e) None of these

26. a b c d e 27. a b c d e 28. a b c d e 29. a b c d e 30. a b c d e


RESPONSE
31. a b c d e 32. a b c d e 33. a b c d e
GRID
y
o
u
rs
m
a
h
58 SPEED TEST 28

b
o
o
34. Statements: N@J, J%R, R©H 38. How many such symbols are there in the above arrangement

b
.w
Conclusions: I. R#N each of which is immediately preceded by a number and

o
immediately followed by a letter?
II. N©H

rd
(a) None (b) One

p
35. Statements: L@K, K©A, A$W

re
(c) Two (d) Three
Conclusions: I . W$L

s
(e) More than three

s
.c
II. L#W 39. If all the numbers are dropped from the above arrangement,

o
m
DIRECTIONS (Qs. 36-40) : Study the following arrangement which of the following will be the eleventh from the left end?
carefully and answer the questions given below (a) B (b) H
D 5 d R @A K © 3 9 B J E* F $ M P I 4 H 1 W 6 (c) $ (d) *
2 # UQ 8 T N (e) None of these
36. How many such numbers are there in the above arrangement 40. Four of the following five are alike in a certain way based on
each of which is immediately preceded by a symbol and their positions in the above arrangement and so form a group.
immediately followed by a letter? Which is the one that does not belong to that group?
(a) None (b) One (a) 41I (b) 6#W
(c) Two (d) Three (c) QTU (d) RAd
(e) More than three (e) J9E
37. Which of the following is the ninth to the right of the twenty
second from the right end of the above arrangement ?
(a) E (b) I
(c) D (d) N
(e) None of these

34. a b c d e 35. a b c d e 36. a b c d e 37. a b c d e 38. a b c d e


RESPONSE
GRID 39. a b c d e 40. a b c d e
y
o
u
rs
m
a
h
b
29

o
o
13

b
.w
Number System - I

o
rd
p
re
s
s
.c
o
m
Max. Marks : 30 No. of Qs. 30 Time : 20 min. Date : ........./......../................
1. Which is the smallest of the following numbers ? 8. How many of the following numbers are divisible by 132 ?
264, 396, 4, 762, 792, 968, 2178, 5184, 6336
2 5 (a) 4 (b) 5 (c) 6
(a) (b) – 1.5 (c)
3 3 (d) 7 (e) None of these
(d) – 1.375 (e) None of these 9. Find the number which, when added to itself 13 times, gives
2. Three-fourth of 68 is less than two-thirds of 114 by 112.
(a) 12 (b) 25 (c) 35 (a) 9 (b) 7 (c) 8
(d) 48 (e) None of these (d) 11 (e) None of these
3. If one-third of a number is 3 more than one -fourth of the 10. The smaller number by which 3600 can be divided to make it
number, then the number is : a perfect cube is :
(a) 18 (b) 24 (c) 30 (a) 9 (b) 50 (c) 300
(d) 36 (e) None of these (d) 450 (e) None of these
4. Which is the smallest of the following numbers ? 11. The least number having four digits which is a perfect square
is :
1 7
(a) 7 (b) (c) (a) 1004 (b) 1016 (c) 1036
7 7 (d) 1226 (e) None of these
1 12. A positive integer, which added to 1000, gives a sum which is
(d) (e) None of these greater than the product obtained when it is multiplied by
7
1000. The positive integer is :
5. If one-eighth of a pencil is black, half of the remaining is (a) 1 (b) 5 (c) 2
1 (d) 3 (e) None of these
yellow and the remaining 3 cm is blue, then the length of
2 3 2
13. Of the 120 people in the room, are women. If of the
the pencil is : 5 3
(a) 6 cm (b) 7 cm (c) 8 cm people are married, then what is the maximum number of
(d) 11 cm (e) None of these women in the room who are unmarried?
6. A boy was asked to write 25 ´ 9 2 but he wrote 2592. The (a) 40 (b) 50 (c) 60
numerical difference between the two is: (d) 80 (e) None of these
(a) 0 (b) 3 (c) 2 14. How many odd numbered pages are there in a book of 1089
(d) 9 (e) None of these pages ?
7. In a group of buffaloes and ducks, the number of legs are (a) 545 (b) 544 (c) 543
24 more than twice the number of heads. What is the number (d) 547 (e) None of these
of buffaloes in the group ? 15. If p is a prime number greater than 3, then (p 2 - 1) is always
(a) 6 (b) 12 (c) 8
divisible by :
(d) 15 (e) None of these
(a) 6 but not 12 (b) 24 (c) 12 but not 24
(d) 16 (e) None of these

1. a b c d e 2. a b c d e 3. a b c d e 4. a b c d e 5. a b c d e
RESPONSE 6. a b c d e 7. a b c d e 8. a b c d e 9. a b c d e 10. a b c d e
GRID 11. a b c d e 12. a b c d e 13. a b c d e 14. a b c d e 15. a b c d e
y
o
u
rs
m
a
h
60 SPEED TEST 29

b
o
o
16. A man has 1044 candles. After burning all of them he can 24. What is the least number to be added to 1500 to make it a

b
make a new candle from 9 stubs left behind. Find the maximum

.w
perfect square?
number of candles that can be made :

o
(a) 20 (b) 21 (c) 22

rd
(a) 116 (b) 120 (c) 130
(d) 23 (e) None of these

p
(d) 136 (e) None of these

re
17. Find the whole number which when increased by 20 is equal 25. How many pieces of 8.6 metres length cloth can be cut out

s
s
to 69 times the reciprocal of the number: of a length of 455.8 metres cloth?

.c
(a) 7 (b) 5 (c) 3 (a) 43 (b) 48 (c) 55

o
m
(d) 2.5 (e) None of these (d) 53 (e) 62
18. A clock strikes 4 taking 9 seconds. In order to strike 12 at the 26. If an amount of ` 15,487 is divided equally among 76 students,
same rate, the time taken is approximately how much amount will each student get?
(a) 36 seconds (b) 27 seconds (c) 30 seconds (a) `206 (b) `210 (c) `204
(d) 33 seconds (e) None of these (d) `218 (e) `212
19. The sum of the place values of 3 in the number 50, 35, 35 is 27. The difference between a number and one fifth of it is 84.
(a) 3300 (b) 6 (c) 60 What is the number?
(d) 3030 (e) None of these (a) 95 (b) 100 (c) 105
20. How many numbers are there between 500 and 600 in which (d) 108 (e) 112
9 occurs only once? 28. A, B, C, D and E are five consecutive odd numbers The sum
(a) 19 (b) 18 (c) 20 of A and C is 146. What is the value of E?
(d) 21 (e) None of these (a) 75 (b) 81 (c) 71
21. The product of two consecutive even numbers is 12768. (d) 79 (e) None of these
What is the greater number ? 29. The sum of five consecutive even numbers of set A is 280.
(a) 110 (b) 108 (c) 114 What is the sum of a different set B of five consecutive
numbers whose lowest number is 71 less than double the
(d) 112 (e) None of these
lowest number of set A ?
22. The product of two consecutive odd numbers is 19043. (a) 182 (b) 165 (c) 172
Which is the smaller number? (d) 175 (e) None of these
(a) 137 (b) 131 (c) 133 30. Deepak has some hens and some goats. If the total number
(d) 129 (e) None of these of animal heads is 90 and the total number of animal feet is
23. The product of two successive numbers is 8556. What is the 248, what is the total number of goats Deepak has ?
smaller number? (a) 32 (b) 36 (c) 34
(a) 89 (b) 94 (c) 90 (d) Cannot be determined
(d) 92 (e) None of these (e) None of these

16. a b c d e 17. a b c d e 18. a b c d e 19. a b c d e 20. a b c d e


RESPONSE 21. a b c d e 22. a b c d e 23. a b c d e 24. a b c d e 25. a b c d e
GRID 26. a b c d e 27. a b c d e 28. a b c d e 29. a b c d e 30. a b c d e
y
o
u
rs
m
a
h
b
30

o
o
13

b
.w
o
Number System - II

rd
p
re
s
s
.c
o
m
Max. Marks : 30 No. of Qs. 30 Time : 20 min. Date : ........./......../................

2 8. Given, n = 1 + x, where x is the product of four consecutive


1. Farah got married 8 years ago. Today her age is 1 times
7 integers. Then which of the following is true ?
her age at the time of her marriage. At present her daughter's A. n is an odd integer
age is one-sixth of her age. What was her daughter's age 3 B. n is prime.
years ago? C. n is a perfect square
(a) 6 years (b) 2 years (a) Both A and C are correct.
(c) 3 years (d) Cannot be determined (b) Both A and B are correct.
(e) None of these (c) Only A is correct
2. If (12)3 is subtracted from the square of a number the answer (d) Only C is correct
so obtained is 976. What is the number? (e) None of these
(a) 58 (b) 56 (c) 54
9. 9 6 + 7, when divided by 8, would have a remainder :
(d) 52 (e) None of these
(a) 0 (b) 6 (c) 5
3. The cost of 5 chairs and 8 tables is ` 6,574. What is the cost
(d) 4 (e) None of these
of 10 chairs and 16 tables?
(a) ` 15674 (b) ` 16435 (c) ` 13148 10. The remainder when 7 84 is divided by 342 is :
(d) Cannot be determined (a) 0 (b) 1 (c) 21
(e) None of these (d) 340 (e) None of these
4. If (56)2 is added to the square of a number, the answer so 11. Students of a class are preparing for a drill and are made to
obtained is 4985. What is the number? stands in a row. If 4 students are extra in a row, then there
(a) 52 (b) 43 (c) 65 would be 2 rows less. But there would be 4 more rows if 4
(d) 39 (e) None of these students are less in a row. The number of students in the
5. The unit’s digit in the product 7 35 ´ 371 ´ 1155 is : class is :
(a) 96 (b) 56 (c) 69
(a) 1 (b) 3 (c) 7
(d) 9 (e) None of these (d) 65 (e) None of these
6. If p be a number between 0 and 1, which one of the following 12. Three friends divided some bullets equally. After each of them
shot 4 bullets, the total number of bullets remaining is equal
will be true ?
to the bullets each had after division . Find the original number
1 of bullets:
(a) p> p (b) > p
p2 (a) 15 (b) 17 (c) 20
(d) 18 (e) None of these
1 13. If x and y are negative, then which of the following statements
(c) p < (d) p3 > p 2 is/are always true ?
p
I. x + y is positive
(e) None of these
II. xy is positive
7. In a certain shop, 9 oranges cost as much as 5 apples, 5
III. x – y is positive
apples cost as much as 3 mangoes, 4 mangoes cost as
(a) I only (b) II only (c) III only
much as 9 lemons. If 3 lemons cost 48 paise, the price of an
(d) I and II only (e) None of these
orange is :
14. A student was asked to divide a number by 6 and add 12 to
(a) 12 paise (b) 14 paise (c) 13 paise
the quotient. He, however, first added 12 to the number and
(d) 15 paise (e) None of these
then divided it by 6, getting 112 as the answer. The correct
answer should have been :
(a) 122 (b) 118 (c) 114
(d) 124 (e) None of these
1. a b c d e 2. a b c d e 3. a b c d e 4. a b c d e 5. a b c d e
RESPONSE 6. a b c d e 7. a b c d e 8. a b c d e 9. a b c d e 10. a b c d e
GRID 11. a b c d e 12. a b c d e 13. a b c d e 14. a b c d e
y
o
u
rs
m
a
h
62 SPEED TEST 30

b
o
o
22. The letters L, M, N, O, P, Q, R, S and T in their order are
1

b
15. If Harry is rd the age of his father, George now, and was substituted by nine integers 1 to 9 but not in that order. 4 is

.w
3 assigned to P. The difference between P and T is 5. The

o
rd
1 difference between N and T is 3. What is the integer
th the age of his father 5 years ago, then how old will

p
assigned to N?
4

re
(a) 4 (b) 5 (c) 6

s
George be 5 years from now ?

s
(d) 7 (e) None of these

.c
(a) 20 years (b) 45 years (c) 40 years
23. A number when divided by 765 leaves a remainder 42. What

o
(d) 50 years (e) None of these

m
will be the remainder if the number is divided by 17 ?
16. On Monday, a certain animal shelter housed 55 cats and
(a) 8 (b) 7 (c) 6
dogs. By Friday, exactly 1/5 of the cats and 1/4 of the dogs
(d) 5 (e) None of these
had been adopted; no new cats or dogs were brought to the
24. If m and n are natural numbers such that 2m – 2n = 960, what
shelter during this period. What is the greatest possible
is the value of m ?
number of pets that could have been adopted from the animal
(a) 10 (b) 12 (c) 16
shelter between Monday and Friday ?
(d) Cannot be determined
(a) 11 (b) 12 (c) 13
(e) None of these
(d) 14 (e) None of these
25. At the first stop on his route, a driver uploaded 2/5 of the
17. The number 6n2 + 6n, for any natural number n, is always
packages in his van. After he uploaded another three
divisible by :
packages at his next stop, 1/2 of the original number of
(a) 6 only (b) 18 only (c) 12 only
packages remained. How many packages were in the van
(d) 6 and 12 (e) None of these
before the first delivery?
18. A bus starts from city X. The number of women in the bus is
(a) 25 (b) 10 (c) 30
half of the number of men. In city Y, 10 men leave the bus and
(d) 36 (e) None of these
five women enter. Now, the number of men and women is
equal. In the beginning, how many passengers entered the 2
bus? 26. A divisor is of the dividend and 2 times the remainder If
3
(a) 15 (b) 30 (c) 36 the remainder is 75, then find the dividend.
(d) 45 (e) None of these (a) 85 (b) 145 (c) 225
19. In a class, there are 18 boys who are over 160 cm tall. If these (d) 65 (e) None of these
constitute three-fourths of the boys and the total number of 27. The quotient arising from the division of 24162 by a certain
boys is two-thirds of the total number of students in the number x is 89 and the remainder is 43. Find x.
class, then what is the number of girls in the class? (a) 271 (b) 281 (c) 261
(a) 6 (b) 12 (c) 18 (d) 251 (e) None of these
(d) 24 (e) None of these 28. A boy multiplied a certain number x by 13. He found that the
20. In a family, a couple has a son and a daughter. The age of the resulting product consisted of all nines entirely. Find the
father is three times of his daughter and the age of the son is smallest value of x.
half of his mother. The wife is nine years younger to her (a) 76913 (b) 76933 (c) 76923
husband and the brother is seven years older than his sister. (d) 75933 (e) None of these
What is the age of the mother? 29. Find the unit’s digit in the product (2467)153 × (341)72.
(a) 40 years (b) 50 years (c) 45 years (a) 6 (b) 7 (c) 8
(d) 60 years (e) None of these (d) 9 (e) None of these
21. Soma purchases National Savings Certificates every year 30. What is the digit in the unit place of 251 ?
whose value exceeds the previous year’s purchase by Rs. (a) 2 (b) 8 (c) 1
400. After 8 years, she finds that she has purchased certificates (d) 4 (e) None of these
whose total face value is Rs. 48,000. What is the face value of
the certificates purchased by her in the first year ?
(a) Rs. 4,300 (b) Rs. 4,500 (c) Rs. 4,400
(d) Rs. 4,600 (e) None of these

15. a b c d e 16. a b c d e 17. a b c d e 18. a b c d e 19. a b c d e


RESPONSE 20. a b c d e 21. a b c d e 22. a b c d e 23. a b c d e 24. a b c d e
GRID 25. a b c d e 26. a b c d e 27. a b c d e 28. a b c d e 29. a b c d e
30. a b c d e
y
o
u
rs
m
a
h
b
31

o
31

o
13

b
.w
LCM, HCF and

o
rd
p
re
Fraction

s
s
.c
o
m
Max. Marks : 30 No. of Qs. 30 Time : 20 min. Date : ........./......../................
1. If the L.C.M and H.C.F. of two numbers are 2400 and 16, 8. Four runners started running simultaneously from a point on
one number is 480; find the second number. a circular track they took 200 sec, 300 sec, 360 sec and 450 sec
(a) 40 (b) 80 (c) 60 to complete one round, after how much time do they meet at
(d) 50 (e) None of these the starting point for the first time?
2. The L.C.M. of two number is 630 and their H.C.F. is 9. If the (a) 1800 sec (b) 3600 sec (c) 2400 sec
sum of numbers is 153, their difference is (d) 4800 sec (e) None of these
(a) 17 (b) 23 (c) 27 9. Philip, Tom and Brad start jogging around a circular field and
(d) 33 (e) None of these complete a single round in 18, 22 and 30 seconds respectively,
3. The H.C.F and L.C.M of two numbers are 21 and 4641 In how much time, will they meet again at the starting point ?
respectively. If one of the numbers lies between 200 and (a) 3 min 15 sec (b) 21 min
300, then the two numbers are (c) 16 min 30 sec (d) 12 min
(a) 273, 357 (b) 273, 361 (c) 273, 359 (e) None of these
(d) 273, 363 (e) None of these 10. The numbers 11284 and 7655, when divided by a certain
4. Suppose you have 108 green marbles and 144 red marbles. number of three digits, leave the same remainder. Find that
You decide to separate them into packages of equal number number of three digits.
of marbles. Find the maximum possible number of marbles (a) 161 (b) 171 (c) 181
in each package. (d) 191 (e) None of these
(a) 4 (b) 36 (c) 9 11. The LCM of two numbers is 2079 and their HCF is 27. if one of
(d) 12 (e) None of these the numbers is189, find the other.
5. One pendulum ticks 57 times in 58 seconds and another (a) 273 (b) 279 (c) 297
608 times in 609 seconds. If they started simultaneously, (d) 307 (e) None of these
find the time after which they will tick together. 12. Find the least number which, when divided by 18, 24, 30 and
42, will leave in each case the same remainder 1.
211 1217 1218 (a) 2521 (b) 2556 (c) 2456
(a) s (b) s (c) s
19 19 19 (d) 2473 (e) None of these
13. The HCF and LCM of two numbers are 44 and 264 respectively.
1018
(d) s (e) None of these If the first number is devided by 2, the quotient is 44. What is
19
the other number?
6. From 3 drums of milk, 271, 331 and 451 respectively are to (a) 108 (b) 44 (c) 124
be drawn out. To do it in a minimum time, the capacity of (d) 132 (e) None of these
the measuring can be 14. The product of two number si 2160 and their HCF is 12. Find
(a) 271 (b) 61 (c) 111 the possible pairs of numbers.
(d) 31 (e) None of these (a) 1 (b) 2 (c) 3
7. Product of two co-prime numbers is 117. Their L.C.M. should (d) 4 (e) None of these
be: 15. Three bells toll at intervals of 9, 12 and 15 minutes respectively.
(a) 1 (b) 117 All the three begin to toll at 8 a.m. At what time will they toll
(c) equal to their H.C.F. together again?
(d) cannot be calculated (a) 8.45 a.m. (b) 10. 30 a.m. (c) 11.00 a.m.
(e) None of these (d) 1. 30 p.m. (e) None of these

1. a b c d e 2. a b c d e 3. a b c d e 4. a b c d e 5. a b c d e
RESPONSE 6. a b c d e 7. a b c d e 8. a b c d e 9. a b c d e 10. a b c d e
GRID 11. a b c d e 12. a b c d e 13. a b c d e 14. a b c d e 15. a b c d e
y
o
u
rs
m
a
h
64 SPEED TEST 31

b
o
o
16. Four metal rods of lengths 78 cm, 104 cm, 117 cm and 169 cm 31

b
21 31
are to be cut into parts of equal length. Each part must be as (a) (b) (c)

.w
long as possible. What is the maximum number of pieces 38 38 48

o
rd
that can be cut? 17
(d) (e) None of these

p
(a) 27 (b) 36 (c) 43
48

re
(d) 400 (e) 402

s
s
17. The LCM and HCF of two numbers are 84 and 21, respectively. 9 2 8 5

.c
24. If the fractions , , , are arranged in ascending
If the ratio of two numbers be 1 : 4, then the larger of the two 13 3 11 7

o
m
numbers is : order, then the correct sequence is :
(a) 21 (b) 48 (c) 84
(d) 108 (e) None of these 9 2 8 5 2 9 5 8
(a) , , , (b) , , ,
18. Which of the following pair of fractions adds up to a number 13 3 11 7 3 13 7 11
greater than 5 ?
2 8 5 9 5 8 2 9
13 11 11 8 7 11 (c) , , , (d) , , ,
(a) , (b) , (c) , 3 11 7 13 7 11 3 13
5 6 4 3 3 5 (e) None of these
5 3 25. If the numerator and the denominator of a proper fraction are
(d) , (e) None of these increased by the same quantity, then the resulting fraction is:
3 4
(a) always greater than the original fraction
19. Which of the following fractions is the least ? (b) always less than the original fraction
12 1 4 (c) always equal to the original fraction
(a) (b) (c) (d) Can’t be determined
119 10 39
(e) None of these
7 26. If x is a positive number, then which of the following fraction
(d) (e) None of these
69 has the greatest value ?
4 x ( x + 1) x
æ 2ö (a) (b) (c)
20. Which of the following is not the reciprocal of ç ÷ ? x x ( x + 1)
è 3ø
( x + 2)
4 -4 -4 (d) (e) None of these
æ 3ö æ 2ö æ 3ö (x + 3)
(a) ç ÷ (b) ç ÷ (c) ç ÷
è 2ø è 3ø è 2ø 27. Four bells begin to toll together and toll respectively at in-
4 tervals of 6, 5, 7, 10 and 12 seconds. How many times they
3 will toll together in one hour excluding the one at the start ?
(d) (e) None of these
24 (a) 7 times (b) 8 times (c) 9 times
21. A certain type of wooden board is sold only in lengths of (d) 11 times (e) None of these
multiples of 25 cm from 2 to 10 metres. A carpenter needs a 28. HCF of 3240, 3600 and a third number is 36 and their LCM is
large quantity of this type of boards in 1.65 meter length. For
the minimum waste, the lengths to be purchased should be: 2 4 ´ 35 ´ 5 2 ´ 7 2 . The third number is :
(a) 3.30 m (b) 6.60 m (c) 8.25 m (a) 2 4 ´ 53 ´ 7 2 (b) 2 2 ´ 35 ´ 7 2
(d) 9.95 m (e) None of these
22. Which of the following group of fractions is arranged in (c) 2 3 ´ 35 ´ 7 2 (d) 25 ´ 52 ´ 7 2
ascending order ? (e) None of these
29. The LCM of two numbers is 4800 and their HCF is 160. If one
5 7 6 7 6 5
(a) , , (b) , , of the numbers is 480, then the other number is :
16 18 17 18 17 16 (a) 16 (b) 16000 (c) 160
5 6 7 6 7 5 (d) 1600 (e) None of these
(c) , , (d) , , 30. The traffic lights at three different road crossings change
16 17 18 17 18 16
after every 48 seconds, 72 seconds and 108 seconds
(e) None of these respectively. If they all change simultaneously at 8 : 20 hours,
23. What is the least fraction which, when added to or subtracted then at what time will they again change simultaneously ?
29 15 (a) 8 : 20 : 08 hrs (b) 8 : 24 : 10 hrs
from + , will make the result a whole number ? (c) 8 : 27 : 12 hrs (d) 8 : 30 : 15 hrs
12 16
(e) None of these

16. a b c d e 17. a b c d e 18. a b c d e 19. a b c d e 20. a b c d e


RESPONSE 21. a b c d e 22. a b c d e 23. a b c d e 24. a b c d e 25. a b c d e
GRID 26. a b c d e 27. a b c d e 28. a b c d e 29. a b c d e 30. a b c d e
y
o
u
rs
m
a
h
b
32

o
o
b
Square, Cube,

.w
o
rd
p
Indices & Surds

re
s
s
.c
o
m
Max. Marks : 40 No. of Qs. 40 Time : 25 min. Date : ........./......../................

DIRECTIONS (Qs. 1-25): What should come in place of the 10. 7 ´ 447 ¸ 21 + 73 - 26 = ?
question mark (?) in the following questions? (a) 196 (b) 16 (c) 13
(d) 169 (e) None of these
1. 1190 ¸ 7225 ´ ? = 3094
(a) 221 (b) 121 (c) 214 ? 15 ´ 4 - 40
11. =
(d) 241 (e) None of these 25 2
(a) 20 (b) 45 (c) 25
( )
2
2. 5 -1 = ?- 2 5 (d) 50 (e) None of these
(a) 6 (b) 6+2 5 (c) 6 5 12. ( 656 ¸ 164 )2 = ?
(d) 6-2 5 (e) None of these (a) 14 (b) 16 (c) 64
(d) 256 (e) None of these
? 25
3. = 13. 255 ¸ 17 ¸ 5 = ( ? )
2
36 (11´ 3 - 18 )
(a) 2 (b) 4 (c) 5 (a) 9 (b) 3 (c) 3
(d) 6 (e) None of these (d) 27 (e) 81
4. 12 ´ 145 ¸ 6 + 34 = ? 14. (32 × 42 × 5) ¸ 36 = (?)2 – 80
(a) – 18 (b) (324)2 (c) 18 (a) (100)2 (b) 10 (c) 100
(d) 18 (e) None of these (d) 10 (e) None of these

( )
2
5. (13)2 - ( 5)2 - 676 + 7 = ( ? )
2 15. 6 +1 = ?+ 2 6

(a) 10 (b) 20 (c) 5 (a) 7 (b) 6 (c) 4 6+7


(d) (25)2 (e) 5 (d) (e) None of these
4 6
2 2
6. (16 ) - 53 + 169 = ( ? ) 16. 12 ´184 ¸ 23 + 26 - 73 = ?
(a) (12)2 (b) 144 (c) 12 (a) 7 (b) ( 7 )2 (c) 7
(d) (144)2 (e) None of these
(d) (49)2 (e) None of these
2
7. 225 + 2304 = ?- (12 )
17. (13)2 - ( 4 )3 - 676 + 2 = ( ? )
2
(a) 205 (b) 207 (c) 206
(a) 3 (b) 9 (c) 81
(d) 208 (e) None of these
(d) 27 (e) 18
8. 450 + 890 + 685 = ?
(a) 43 (b) 45 (c) 55 18. ( 74 ´ ) (
676 - 42 ´ ? = 496 )
(d) 53 (e) None of these (a) 1024 (b) 1296 (c) 1156
9. 3 6859 = ?- 4 (d) 1089 (e) None of these
(a) 26 (b) 25 (c) 23
(d) 22 (e) None of these

1. a b c d e 2. a b c d e 3. a b c d e 4. a b c d e 5. a b c d e
RESPONSE 6. a b c d e 7. a b c d e 8. a b c d e 9. a b c d e 10. a b c d e

GRID 11. a b c d e 12. a b c d e 13. a b c d e 14. a b c d e 15. a b c d e


16. a b c d e 17. a b c d e 18. a b c d e
y
o
u
rs
m
a
h
66 SPEED TEST 32

b
o
o
19. 30. A gardener plants 34969 mango trees in his garden and
97344 = ?

b
arranges them so that there are so many rows as there are

.w
(a) 302 (b) 322 (c) 292
mango trees in each row. The number of rows is

o
(d) 342 (e) None of these

rd
(a) 187 (b) 176 (c) 169

p
(2 ) (
392 - 21 + 8 -7 )2 = ( ? )2 (d) 158 (e) None of these

re
20.

s
DIRECTIONS (Q. 31-40): What will come in place of question

s
(a) 4 (b) – 4 (c) 12

.c
mark (?) in the following questions?

o
(d) 2 (e) 6

m
31. 93 × 812 ¸ 273 = (3)?
21. 7365 + (5.4)2 + ? = 7437.16
(a) 3 (b) 4 (c) 5
(a) 1894 (b) 1681 (c) 1764 (d) 6 (e) None of these
(d) 2025 (e) None of these
22. ( )
3 ? = 756 ´ 67 ¸ 804 32. ( 9 )3 ´ ( 81)5 ¸ (27)2 = (3)(?)
(a) 195112 (b) 250047 (c) 226981 (a) 5 (b) 4 (c) 7
(d) 274625 (e) None of these (d) 6 (e) None of these
33. 81.1 × 42.7 × 23.3 = 2?
23. 17 + 51 + 152 + 289 = ? (a) 7.1 (b) 14 (c) 0.5
(a) 3 (b) 5 (c) 8 (d) 9 (e) None of these
(d) 11 (e) None of these 34. (49)3 ¸ (7)2 = ?
(a) 2401 (b) 49 (c) 343
24. 217 + 52 + 12 = ? (d) 7 (e) None of these
(a) 18 (b) 16 (c) 12 35. 643.1 × 84.3 = 8?
(d) 15 (e) 10 (a) 10.5 (b) 7.4 (c) 1.2
25. (d) 13.3 (e) None of these
10201 - 3136 = ?
36. 87 × 26 ¸ 82.4 = 8?
(a) 45 (b) 42 (c) 46
(a) 10.6 (b) 9.6 (c) 8.6
(d) 49 (e) None of these
(d) 6.6 (e) None of these
DIRECTIONS (Qs. 26-30): What approximate value will come in 37. (31)31 × (31)– 27 = ?
place of the question mark (?) in the following questions? (You are (a) (961)2 (b) 4 (c) (31)2
not expected to calculate the exact value.) (d) 29791 (e) None of these

{(12) }
26. If 21025 = 145 , then the value of 210.25 + 2.1025 = ?
-2 2
(a) 0.1595 (b) 1.595 (c) 159.5
38. =?
{(12) }
(d) 15.95 (e) None of these 2 -2
27. What is the least number to be added to 2000 to make it a
perfect square? 12
(a) 25 (b) 64 (c) 36 (a) 12 (b) 4.8 (c)
144
(d) 49 (e) None of these (d) 144 (e) None of these
28. If (22)3 is subtracted from the square of a number, the answer 39. 64 × 362 ¸ 216 = 6(?)
so obtained is 9516. What is the number? (a) 3 (b) 4 (c) 5
(a) 144 (b) 142 (c) 138 (d) 6 (e) None of these
(d) 136 (e) None of these
( 8)
6 3 ?
29. If the square of a number is subtracted from 4052 and the 40. ´ ( 64 ) ¸ 84 = ( 8 )
difference is multiplied by 15, the answer so obtained is
41340. What is the number? (a) 3 (b) 4 (c) 5
(a) 36 (b) 1024 (c) 32 (d) 7 (e) None of these
(d) 1296 (e) None of these

19. a b c d e 20. a b c d e 21. a b c d e 22. a b c d e 23. a b c d e


24. a b c d e 25. a b c d e 26. a b c d e 27. a b c d e 28. a b c d e
29. a b c d e 30. a b c d e 31. a b c d e 32. a b c d e 33. a b c d e
RESPONSE
34. a b c d e 35. a b c d e 36. a b c d e 37. a b c d e 38. a b c d e
GRID 39. a c d e 40. a c d e
b b
y
o
u
rs
m
a
h
b
33

o
o
b
.w
Simplification

o
rd
p
re
s
s
.c
o
m
Max. Marks : 30 No. of Qs. 30 Time : 20 min. Date : ........./......../................
8. (78700 ÷ 1748) + (3.79 × 121.24) = ?
DIRECTIONS (Qs. 1 to 12) : What will come in place of qustions
mark (?) in the following questions? (a) 305 (b) 415
1. 19.99 × 9.9 + 99.9 = ? (c) 525 (d) 635
(a) 129. 79 (b) 297. 801 (e) 745
(c) 1009 (d) 296.91 9. 6080 ¸ 3 74000 + 4 6560 = ?
(e) None of these
2. 456.675 + 35.7683 × 67.909 – 58.876 = ? (a) 30 (b) 80
(a) 33382 (b) 3587 (c) 130 (d) 170
(c) 1540 (d) 2756 (e) 210
(e) 2830
1 2 3
3. {(52)2 + (45)2} ¸ ? = 8 10. of of of 1715 = ?
(a) 611.345 (b) 487.225 8 3 5
(c) 591.125 (d) 372.425 (a) 80 (b) 85
(e) None of these (c) 90 (d) 95
4. (12.25)2 – 625 = ? (e) 75
11. 561 ÷ 35.05 × 19.99 = ?
(a) 235.1625 (b) 125.0625
(a) 320 (b) 330
(c) 375.2625 (d) 465.3625 (c) 315 (d) 325
(e) None of these (e) 335
5. 572 + 38 × 0.50 – 16 = ? 12. (15.01)2 × 730 = ?
(a) 289 (b) 305 (a) 6125 (b) 6225
(c) 448 (d) 565 (c) 6200 (d) 6075
(e) None of these (e) 6250
DIRECTIONS (13-20): What will come in place of the question
3 2 11 mark (?) in the following questions?
6. ´ ´ ´ ? = 1056
7 5 13 13. (3325 ¸ 25) × (152 ¸ 16) = ?
(a) 1269.4 (b) 1264.9
(a) 7280 (b) 7296
(c) 1265.3 (d) 1263.5
(c) 7308 (d) 7316 (e) None of these
(e) 7324
1 2 2
14. 5 +2 +3 = ?
7. 6138 + 3 17576 = ? ×9 5 15 3
(a) 15 (b) 13
(a) 676 (b) 729
11
(c) 784 (d) 841 (c) (d) 12
15
(e) 961
(e) None of these

1. a b c d e 2. a b c d e 3. a b c d e 4. a b c d e 5. a b c d e
RESPONSE 6. a b c d e 7. a b c d e 8. a b c d e 9. a b c d e 10. a b c d e

GRID 11. a b c d e 12. a b c d e 13. a b c d e 14. a b c d e


y
o
u
rs
m
a
h
68 SPEED TEST 33

b
o
o
15. – 15 – 27– 88 – 63 + 255 = ?
? - 11 = 1521

b
23.
(a) 55 (b) 74

.w
(c) 62 (d) 59 (b) (28)2

o
(a) 2500

rd
(e) None of these

p
16. (2525 × 0.25 ¸ 5) × 7 = ? (c) 28 (d) 50

re
(a) 889.43 (b) 883.75

s
(e) None of these

s
(c) 886.45 (d) 881.75

.c
(e) None of these 24. 8059 - 7263 = ? ´ 40

o
m
14 57 20 (a) 19.9 (b) 18.7
17. ´ ´ =? (c) 15.9 (d) 17.7
19 70 21
(e) None of these
2 4
(a) (b) 25. 4 ´ ? = 4062 ¸ 5
7 7
(a) 203.1 (b) 213.1
2 3 (c) 205.1 (d) 215.1
(c) (d)
9 7 (e) None of these
(e) None of these
18. 32% of 500 + 162% of 50 = ? 26. 3.5 ´ (80 ¸ 2.5) = ?
(a) 231 (b) 245 (a) 122 (b) 111
(c) 237 (d) 247
(c) 222 (d) 212
(e) None of these
19. 45316 + 52131 – 65229 = ? + 15151 (e) None of these
(a) 17063 (b) 17073 27. 13% of 258 – ? = 10
(c) 17076 (d) 17067 (a) 23.45 (b) 24.53
(e) None of these (c) 23.54 (d) 24.35
184 ´ 4 (e) None of these
20. =?
23 of 400 4 3 5
28. ´2 ¸ = ?
(a) 7 (b) 9 5 4 8
(c) 8 (d) 5
(e) None of these 12 12
(a) 4 (b) 1
21. What will come in place of both the question marks (?) in 35 35
the following question ?
11 13
(?) 4 / 3 128 (c) 2 (d) 3
= 35 25
32 ?5 / 3 (e) None of these
(a) 16 (b) 12 29. 623.15 – 218.82 – 321.43 = ?
(c) 18 (d) 14
(a) 89.2 (b) 82.2
(e) None of these
(c) 89.9 (d) 79.2
22. 16% of 450 ÷ ? % of 250 = 4.8 (e) None of these
(a) 12 (b) 6 30. 5437 – 3153 + 2284 = ? × 50
(c) 4 (d) 10 (a) 96.66 (b) 91.36
(e) None of these (c) 96.13 (d) 93. 16
(e) None of these

15. a b c d e 16. a b c d e 17. a b c d e 18. a b c d e 19. a b c d e


20. a b c d e 21. a b c d e 22. a b c d e 23. a b c d e 24. a b c d e
RESPONSE
25. a b c d e 26. a b c d e 27. a b c d e 28. a b c d e 29. a b c d e
GRID
30. a b c d e
y
o
u
rs
m
a
h
b
34

o
o
b
.w
o
Average

rd
p
re
s
s
.c
o
m
Max. Marks : 30 No. of Qs. 30 Time : 20 min. Date : ........./......../................
1. The average of five numbers is 281. The average of the first 8. The average of 5 consecutive numbers A, B, C, D and E is 48.
two numbers is 280 and the average of the last two numbers What is the product of A and E?
is 178.5. What is the third number ? (a) 2162 (b) 2208
(a) 488 (b) 336 (c) 2024 (d) 2800
(c) 228 (d) 464 (e) None of these
(e) None of these 9. If the value of 16a + 16b = 672, what is the average of a and b?
2. The average age of 3 friends is 32 years. If the age of a (a) 44 (b) 21
fourth friend is added, their average age comes to 31 years. (c) 24 (d) 42
What is the age of the fourth friend ? (e) None of these
(a) 32 years (b) 28 years
10. The sum of five numbers is 290. The average of the first two
(c) 24 years (d) 26 years
numbers is 48.5 and the average of last two numbers is 53.5.
(e) None of these
What is the third number?
3. Find the average of the following set of scores.
(a) 72 (b) 84
965, 362, 189, 248, 461, 825, 524, 234
(c) 96 (d) 108
(a) 476 (b) 504 (e) None of these
(c) 461 (d) 524 11. The average age of the family of five members is 24. If the
(e) None of these present age of youngest member is 8 yr, then what was the
4. If the value of 21a + 21b=1134, what is the average of a+b? average age of the family at the time of the birth of the youngest
(a) 29 (b) 27 member ?
(c) 58 (d) 54 (a) 20 yr (b) 16 yr (c) 12 yr
(e) None of these (d) 18 yr (e) 21 yr
5. Out of three given numbers, the first number is twice the 12. The average of four consecutive odd numbers is 36. What is
second and thrice the third. If the average of the three num- the smallest of these numbers ?
bers is 154, what is the difference between the first and the (a) 31 (b) 35 (c) 43
third number? (d) 47 (e) None of these
(a) 126 (b) 42 13. Average score of Rahul, Manish and Suresh is 63. Rahul’s
(c) 166 (d) 52 score is 15 less than Ajay and 10 more than Manish. If Ajay
(e) None of these scored 30 marks more than the average scores of Rahul,
1 Manish and Suresh, what is the sum of Manish’s and Suresh’s
6. Average weight of three boys P, T and R is 54 kgs while scores ?
3
the average weight of three boys, T, F and G is 53 kgs. What (a) 120 (b) 111
is the average weight of P, T, R, F and G? (c) 117 (d) Cannot be determined
(a) 53.8kgs (b) 52.4kgs (e) None of these
(c) 53.2kgs (d) Cannot be determined 14. The total marks obtained by a student in physics, Chemistry
(e) None of these and Mathematics together is 120 more than the marks obtained
7. Find the average of the following set of scores by him in Chemistry. What are the average marks obtained by
221, 231, 441, 359, 665, 525 him in Physics and Mathematics together ?
(a) 399 (b) 428 (a) 60 (b) 120
(c) 40 (d) Cannot be determined
(c) 407 (d) 415 (e) None of these

(e) None of these


1. a b c d e 2. a b c d e 3. a b c d e 4. a b c d e 5. a b c d e
RESPONSE 6. a b c d e 7. a b c d e 8. a b c d e 9. a b c d e 10. a b c d e

GRID 11. a b c d e 12. a b c d e 13. a b c d e 14. a b c d e


y
o
u
rs
m
a
h
70 SPEED TEST 34

b
o
o
15. A man drives to his office at 60 km/hr and returns home 23. The average of three numbers is 135. The largest number is

b
along the same route 30 km/hr. Find the average speed. 180 and the difference between the others is 25. The small-

.w
(a) 50 km/hr (b) 45 km/hr (c) 40 km/hr est number is :

o
rd
(d) 55 km/hr (e) None of these (a) 130 (b) 125 (c) 120

p
16. A cricketer has completed 10 innings and his average is 21.5 (d) 100 (e) None of these

re
runs. How many runs must he make in his next innings so as 24. There are 30 student in a class. The average age of the first

s
s
to raise his average to 24? 10 students is 12.5 years. The average age of the next 20

.c
(a) 69 (b) 59 (c) 49 students is 13.1 years. The average age of the whole class

o
m
(d) 39 (e) None of these is:
17. One-third of a certain journey is covered at th rate of 25 km / (a) 12.5 years (b) 12.7 years (c) 12.8 years
hr, one-fourth at the rate of 30 km / hr and the rest at 50 km / (d) 12.9 years (e) None of these
hr. Find the average speed for the whole journey. 25. The weight in kilograms of 10 students are 52, 45, 31, 35, 40,
1 1 1 55, 60, 38, 44, 36. If 44 is replaced by 46 and 40 is replaced by
(a) 33 km / hr (b) 44 km / hr (c) 33 km/ hr 35, then the new median will be :
3 4 5 (a) 42 (b) 40.5 (c) 40
1 (d) 41.5 (e) None of these
(d) 44 km / hr (e) None of these 26. The average (arithmetic mean) of x and y is 40. If z = 10, then
5
18. The average salary of all the workers in a workshop is what is the average of x, y and z ?
` 8,000. The average salary of 7 technicians is `12,000 and (a) 16
2
(b) 30 (c) 25
the average salary of the rest is Rs 6,000. The total number 3
of workers in the workshop is : 1
(a) 21 (b) 20 (c) 23 (d) 17 (e) None of these
2
(d) 22 (e) None of these
27. The average of 11 numbers is 10.9. If the average of the first
19. A batsman makes a scores of 98 runs in his 19th inning and
six numbers is 10.5 and that of the last six numbers is 11.4,
thus increases his average by 4. What is his average after
then the middle number is :
19th inning ?
(a) 11.5 (b) 11.4 (c) 11.3
(a) 22 (b) 24 (c) 28
(d) 11.0 (e) None of these
(d) 26 (e) None of these
28. The average of 10 numbers is 40.2. Later it is found that two
20. The average weight of 45 students in a class is 52 kg. 5 of
numbers have been wrongly copied. The first is 18 greater
them whose average weight is 48 kg leave the class and
than the actual number and the second number added is 13
other 5 students whose average weight is 54 kg join the
class. What is the new average weight (in kg) of the class ? instead of 31. Find the correct average.
(a) 40.2 (b) 40.4 (c) 40.6
2 1 (d) 40.8 (e) None of these
(a) 52.6 (b) 52 (c) 52
3 3 29. Last year, a Home Appliance Store sold an average (arithmetic
(d) 62.5 (e) None of these mean) of 42 microwave ovens per month. In the first 10
21. The batting average of 40 innings of a cricket player is 50 months of this year, the store has sold an average (arithmetic
runs. His highest score exceeds his lowest score by 172 mean) of only 20 microwave ovens per month. What was the
runs. If these two innings are excluded, the average of the average number of microwave ovens sold per month during
remaining 38 innings is 48. His highest score was : the entire 22 months period ?
(a) 172 (b) 173 (c) 174 (a) 21 (b) 30 (c) 31
(d) 176 (e) None of these (d) 32 (e) None of these
22. A car owner buys petrol at Rs 7.50, Rs 8.00 and Rs 8.50 per 30. The average temperature for the first four days of a week is
litre for three successive years. What approximately is his 40.2°C and that of the last four days is 41.3°C. If the average
average cost per litre of petrol if he spends Rs 4000 each temperature for the whole week is 40.6°C, then the tempera-
year ? ture on the fourth day is :
(a) Rs 8 (b) Rs 9 (c) Rs 7.98 (a) 40.8°C (b) 38.5°C (c) 41.3°C
(d) Rs 8.50 (e) None of these (d) 41.8°C (e) None of these

15. a b c d e 16. a b c d e 17. a b c d e 18. a b c d e 19. a b c d e

20. a b c d e 21. a b c d e 22. a b c d e 23. a b c d e 24. a b c d e

RESPONSE 25. a b c d e 26. a b c d e 27. a b c d e 28. a b c d e 29. a b c d e

GRID 30. a b c d e
y
o
u
rs
m
a
h
b
35

o
o
b
.w
Percentage

o
rd
p
re
s
s
.c
o
m
Max. Marks : 30 No. of Qs. 30 Time : 20 min. Date : ........./......../................
9. 36% of 245 – 40% of 210 = 10 – ?
DIRECTIONS (Qs. 1-18): What should come in place of the
question mark (?) in the following questions? (a) 4.2 (b) 6.8 (c) 4.9
(d) 5.6 (e) None of these
1. 76% of 1285 = 35% of 1256 + ?
(a) 543 (b) 537 (c) 547 1
10. of 3842 + 15% of ? = 24499
(d) 533 (e) None of these 2
2. (21.5% of 999)1/3 + (43% of 601)1/2 = ? (a) 3520 (b) 3250 (c) 3350
(d) 3540 (e) None of these
(a) 18 (b) 22 (c) 26
(d) 30 (e) 33 11. 57% of 394 – 2.5% of 996 = ?
3. 64.5% of 800 + 36.4% of 1500 = (?)2 + 38 (a) 215 (b) 175 (c) 200
(d) 180 (e) 205
(a) 32 (b) 38 (c) 42
(d) 48 (e) 34 12. 40% of 265 + 35% of 180 = 50% of ?
4. 41% of 601 – 250.17 = ? – 77% of 910 (a) 338 (b) 84.5 (c) 253.5
(d) 169 (e) None of these
(a) 800 (b) 500 (c) 700
(d) 650 (e) 550 1 1
13. 4 ´ 3 + ? = 20% of 120
5. 40.005% of 439.998 + ?% of 655.011 = 228.5 5 3
(a) 8 (b) 17 (c) 12
(d) 20 (e) 5 1
(a) 10 (b) 10 (c) 5
15
6. 25% of 84 × 24% of 85 = ?
(d) 15 (e) None of these
(a) 424.2 (b) 488.4 (c) 482.8
14. 14% of 250 × ? % of 150 = 840
(d) 428.4 (e) None of these
(a) 15 (b) 18 (c) 16
7. 20.06% of 599 + 10.01% of 901 = ?
(d) 12 (e) None of these
(a) 150 (b) 210 (c) 250
15. 18% of 609 + 27.5% of 450 = ?
(d) 280 (e) 300
(a) 220 (b) 233 (c) 267
8. 14.2% of 5500 + 15.6% of ? = 1795
(d) 248 (e) 274
(a) 6500 (b) 6200 (c) 5600
(d) 5800 (e) None of these

1. a b c d e 2. a b c d e 3. a b c d e 4. a b c d e 5. a b c d e
RESPONSE 6. a b c d e 7. a b c d e 8. a b c d e 9. a b c d e 10. a b c d e

GRID 11. a b c d e 12. a b c d e 13. a b c d e 14. a b c d e 15. a b c d e


y
o
u
rs
m
a
h
72 SPEED TEST 35

b
o
o
16. 125% of 3060 – 85% of ? = 408 (a) 700 (b) 730

b
.w
(a) 3890 (b) 3940 (c) 4020 (c) 745 (d) 765
(d) 4015 (e) None of these

o
(e) None of these

rd
17. If x% of 500 = y% of 300 and x% of y% of 200 = 60, then x = ?
24. The population of a town is 198000. It increases by 7% in the

p
re
(a) 10 2 (b) 20 2 (c) 15 2 1st year and decreases by 5% in the 2nd year. What is the

s
population of the town at the end of 2 years?

s
(d) 30 2 (e) None of these

.c
(a) 211860 (b) 201267

o
18. 185% of 400 + 35% of 240 = ? % of 1648

m
(a) 85 (b) 75 (c) 125 (c) 222453 (d) 198900
(d) 50 (e) None of these (e) None of these
2 25. Bovina spent ` 44,668 on her air tickets, ` 56,732 on buying
19. What is 25% of 50% of rd of 630 ? gifts for the family members and the remaining 22% of the
3
total amount she had as cash with her. What was the total
(a) 36.5 (b) 52.5 (c) 45.5
amount?
(d) 68.5 (e) None of these
(a) ` 28]600 (b) ` 1]30]000
20. There are 1225 employees in an organization, out of which
40% got transferred to different places. How many such (c) ` 1]01]400 (d) ` 33]800
employees got transferred ? (e) None of these
(a) 540 (b) 490 26. Rubina decided to donate 16% of her monthly salary to an
(c) 630 (d) 710 NGO. On the day of donation she changed her mind and
(e) None of these donated ` 6,567 which was 75% of what she had decided
21. If the numerator of a fraction is increased by 500% and the earlier. How much is Rubina’s monthly salary?
denominator is increased by 300%, the resultant fraction (a) ` 8]756 (b) ` 54]725
4 (c) ` 6]56]700 (d) ` 45]696
is 2 . What was the original fraction?
7 (e) None of these
27. In a college election between two candidates, one candidate
4 12
(a) (b) got 55% of the total valid votes. 15% of the votes were
7 7 invalid. If the total votes were 15,200, what is the number of
15 6 valid votes the other candidate got ?
(c) (d) (a) 7106 (b) 6840
4 5
(e) None of these (c) 8360 (d) 5814
22. If the numerator of a fraction is increased by 250% and the (e) None of these
28. If the radius of a circle is diminished by 10%, the area is
7
denominator is increased by 300%, the resultant fraction is . diminished by
9 (a) 36% (b) 20% (c) 19% (d) 10%
What is the original fraction ? (e) None of these
8 7 29. If A’s salary is 25% higher than B’s salary, then how much
(a) (b) per cent is B’s salary lower than A’s ?
11 8
1 1
8 7 (a) 16 % (b) 20% (c) 25% (d) 33 %
(c) (d) 3 3
9 11
(e) None of these
(e) None of these
30. A reduction of 20% in the price of an apple enable a man to
23. In an examination it is required to get 40% of the aggregate buy 10 apple more for ` 54. The reduced price of apple per
marks to pass. A student gets 261 marks and is declared failed dozen is
by 4% marks. What are the maximum aggregate marks a
(a) ` 4.32 (b) ` 12.96
student can get?
(c) ` 10.80 (d) ` 14.40
(e) None of these

16. a b c d e 17. a b c d e 18. a b c d e 19. a b c d e 20. a b c d e

21. a b c d e 22. a b c d e 23. a b c d e 24. a b c d e 25. a b c d e


RESPONSE
26. a b c d e 27. a b c d e 28. a b c d e 29. a b c d e 30. a b c d e
GRID
y
o
u
rs
m
a
h
b
36

o
o
b
.w
o
Ratio & Proportion

rd
p
re
s
s
.c
o
m
Max. Marks : 30 No. of Qs. 30 Time : 20 min. Date : ........./......../................
1. The total number of students in a school is 819. If the number (a) 10 years (b) 15 years
of girls in the school is 364, then what is the respective ratio (c) 24 years (d) Cannot be determined
of the total number of boys to the total number of girls in the
(e) None of these
school ?
8. In a college the students in Arts and Commerce faculties were
(a) 26 : 25 (b) 21 : 17
in the ratio of 4 : 5 respectively. When 65 more students joined
(c) 18 : 13 (d) 5 : 4 Commerce faculty the ratio became 8 : 11. How many students
(e) None of these are there in Arts faculty?
2. If a dividend of ` 57,834 is to be divided among Meena, (a) 520 (b) 650
Urmila and Vaishali in the proportion of 3:2:1, find Urmila's (c) 715 (d) Cannot be determined
share.
(e) None of these
(a) ` 19,281 (b) ` 17,350
9- The ratio between the boys and girls in a class is 6 : 5. If 8 more
(c) ` 23,133 (d) ` 19,278 boys join the class and two girls leave the class then the ratio
(e) None of these becomes 11 : 7. What is the number of boys in the class now?
3. The ratio of the ages of Richa and Shelly is 5 : 8. The ratio of (a) 28 (b) 38
their ages 10 years hence would be 7:10. What is the present
(c) 44 (d) 36
age of Shelly?
(e) None of these
(a) 45years (b) 40years
10. At present Meena is eight times her daughter's age. Eight
(c) 35years (d) 30years
years from now, the ratio of the ages of Meena and her
(e) 25years daughter will be 10 : 3. What is Meena's present age ?
4. A sum of money is to be divided equally amongst A, B, and (a) 32 years (b) 40 years
C in the respective ratio of 3:4:5 and another sum of money
(c) 36 years (d) Cannot be determined
is to be divided between E and F equally. If F got `1050 less
than A, how much amount did B receive? (e) None of these
(a) ` 750 (b) ` 2000 11. The ratio of the ages of a father and son is 17 : 7. 6 years ago
the ratio of their ages was 3 : 1 . What is the father’s present
(c) ` 1500 (d) Cannot be determined
age?
(e) None of these
(a) 64 (b) 51
5. The average age of a woman and her daughter is 42 years.
The ratio of their ages is 2:1. What is the daughter's age? (c) 48 (d) Cannot be determined
(a) 28 years (b) 48 years (e) None of these
(c) 52 years (d) 32 years 12. The ratio of the money with Rita an Sita is 7 : 15 and that with
Sita and Kavita is 7 : 16. If Rita has ` 490, how much money
(e) None of these
does Kavita have?
6. The difference between the present ages of Arun and Deepak
(a) 1050 (b) 2200 (c) 2400
is 14 years. Seven years ago the ratio of their ages was 5 : 7
(d) 2800 (e) None of these
respectively. What is Deepak’s present age?
13. In two alloys, the ratio of iron and copper is 4 : 3 and 6 : 1,
(a) 49 years (b) 42 years
respectively. If 14 kg of the first alloy and 42 kg of the second
(c) 63 years (d) 35 years alloy is mixed together to form a new alloy, then what will be
(e) None of these the ratio of iron to copper in the new alloy ?
7. Ratio of Rani’s and Komal’s age is 3 : 5. Ratio of Komal’s and (a) 11 : 3 (b) 11 : 8 (c) 8 : 1
Pooja’s age is 2 : 3. If Rani is two-fifth Pooja’s age, what is (d) 3 : 11 (e) None of these
Rani’s age?

1. a b c d e 2. a b c d e 3. a b c d e 4. a b c d e 5. a b c d e
RESPONSE 6. a b c d e 7. a b c d e 8. a b c d e 9. a b c d e 10. a b c d e

GRID 11. a b c d e 12. a b c d e 13. a b c d e


y
o
u
rs
m
a
h
74 SPEED TEST 36

b
o
o
23. An amount of money is to be distributed among P, Q and R
1 1 1

b
: : and its in the ratio 3 : 1 : 5. The difference between Q’s and R’s share

.w
14. The sides of a triangle are in the ratio
2 3 4 is Rs 3600. What is the total of P’s and Q’s share ?

o
perimeter is 104 cm. The length of the longest side is:

rd
(a) Rs 5400 (b) Rs 3600 (c) Rs 2700
(a) 52 cm (b) 48 cm (c) 32 cm

p
(d) Rs 1800 (e) None of these

re
(d) 26 cm (e) None of these 24. Mrs. X spends Rs 535 in purchasing some shirts and ties for

s
s
15. The ratio of two numbers is 4 : 7. If each of these numbers her husband. If shirts cost Rs 43 each and the ties cost Rs 21

.c
increases by 30, their ratio will become 5 : 8 . What is the each, then what is the ratio of the shirts to the ties, that are

o
m
average of these two numbers? purchased ?
(a) 135 (b) 145 (c) 155 (a) 1 : 2 (b) 2 : 1 (c) 2 : 3
(d) 165 (e) 175 (d) 3 : 4 (e) None of these
1 2 3 25. If a : b = 2 : 5, then the value of (2a + 3b) : (7a + 5b) is :
16. If ` 782 be divided into three part, proportional to : : ,
2 3 4
then the first part is : 19 99 31
(a) (b) (c)
(a) ` 182 (b) ` 190 (c) ` 196 39 13 19
(d) ` 204 (e) None of these
19
3 (d) (e) None of these
17. The scale of map is of cm = 1 km. If the distance on the 31
4
map between two towns is 60 cm, then the actual distance is 26. If two numbers are in the ratio 6 : 13 and their least common
multiple is 312, then the sum of the numbers is :
(a) 60 km (b) 80 km (c) 75 km
(d) 50 km (e) None of these (a) 75 (b) 57 (c) 76
(d) 67 (e) None of these
A B C
18. If A : B : C = 2 : 3 : 4, then : : is equal to : 5a + 3b 23
B C A 27. If = , then the value of a : b is :
2a - 3b 5
(a) 4 : 9 : 16 (b) 8 : 9 : 12 (c) 8 : 9 : 16
(d) 8 : 9 : 24 (e) None of these (a) 2 : 1 (b) 1 : 4 (c) 1 : 2
(d) 4 : 1 (e) None of these
19. The average age of three boys is 25 years and their ages are
in the proportion 3: 5 : 7. The age of the youngest boy is: 28. A bag contains Rs 216 in the form of one rupee, 50 paise and
25 paise coins in the ratio of 2 : 3 : 4. The number of 50 paise
(a) 21 years (b) 18 years (c) 15 years
coins is :
(d) 9 years (e) None of these
(a) 96 (b) 144 (c) 114
20. The income of A and B are in the ratio 3 : 2 and expenses are
(d) 141 (e) None of these
in the ratio 5 : 3. If both save ` 200, what is the income of A?
29. The ages of two persons are in the ratio of 5 : 7. Sixteen years
(a) ` 1000 (b) ` 1200 (c) ` 1500
ago, their ages were in the ratio of 3: 5. Their present ages
(d) ` 1800 (e) None of these
are:
21. ` 750 is distributed among A, B and C such that
(a) 30 years and 44 years
A’s share : B’s share = 2 : 3 and B’s share: C’s share = 6 : 5.
(b) 35 years and 52 years
The share of A is,
(c) 40 years and 56 years
(a) ` 150 (b) ` 175 (c) ` 200
(d) ` 250 (e) None of these (d) 45 years and 60 years
22. Two number are in the ratio 5 : 4 and their difference is 10. (e) None of these
What is the larger number ? 30. In a mixture of 28 litres, the ratio of milk and water is 5 : 2.
(a) 30 (b) 40 (c) 50 Another 2 litres of water is added to the mixture. The ratio of
(d) 60 (e) None of these milk and water in the new mixture is:
(a) 1: 1 (b) 2: 1 (c) 3: 1
(d) 4: 1 (e) None of these

14. a b c d e 15. a b c d e 16. a b c d e 17. a b c d e 18. a b c d e


19. a b c d e 20. a b c d e 21. a b c d e 22. a b c d e 23. a b c d e
RESPONSE 24. a b c d e 25. a b c d e 26. a b c d e 27. a b c d e 28. a b c d e
GRID 29. a b c d e 30. a b c d e
y
o
u
rs
m
a
h
b
37

o
o
b
Alligation and

.w
o
rd
p
Mixture

re
s
s
.c
o
m
Max. Marks : 25 No. of Qs. 25 Time : 20 min. Date : ........./......../................
1. One litre of water was mixed to 3 litres of sugar. Solution 8. An alloy contains copper and zinc in the ratio 5 : 3 and another
containing 4% of sugar. What is the percentage of sugar in alloy contains copper and tin in the ratio 8 : 5. If equal weights
the solution? of both the alloys are melted together, then the weight of tin
(a) 3 (b) 4 in the resulting alloy per kg will be:
(c) 6 (d) Insuffficient data
26 5 7
(e) None of these (a) (b) (c)
5 26 40
2. A trader mixes 80 kg of tea at ` 15 per kg with 20 kg of tea at
cost price of ` 20 per kg. In order to earn a profit of 25%, 40
what should be the sale price of the mixed tea? (d) (e) None of these
7
(a) ` 23.75 (b) ` 22 (c) ` 20
9. The ratio of milk and water in 55 litres of adulterated milk is
(d) ` 19.20 (e) None of these
7 : 4. How much water must be added to make the mixture’s
3. Alcohol cost ` 3.50 per litre and kerosene oil cost ` 2.50 per ratio 7 : 6?
litre. In what proportion these should be mixed so that the
(a) 5 lt (b) 10 lt (c) 15 lt
resulting mixture may be ` 2.75 per litre?
(d) 25 lt (e) None of these
(a) 2 : 5 (b) 1 : 3 (c) 4 : 7
10. A and B are two alloys of gold and copper prepared by mixing
(d) 2 : 3 (e) None of these
metals in the ratio 7 : 2 and 7 : 11 respectively. If equal
4. Pure milk costs ` 3.60 per litre. A milkman adds water to 25 quantities of the alloys are melted to form a third alloy C, then
litres of pure milk and sells the mixture at ` 3 per litre. How the ratio of gold and copper in alloy C will be
many litres of water does he add?
(a) 5 : 7 (b) 5 : 9 (c) 7 : 5
(a) 2 litres (b) 5 litres (c) 7 litres (d) 9 : 5 (e) None of these
(d) 11 litres (e) None of these
11. Sameer bought 10 kg of tea at ` 45 per kg and 8 kg at ` 50 per
5. In what ratio must water be mixed with milk to gain 20% by kg. He mixed both the brands and sold it at a total profit of `
selling the mixture at cost price? 32. What was the selling price per kg of the mixture?
(a) 1 : 3 (b) 1 : 5 (c) 1 : 7 (a) ` 48 (b) ` 50 (c) ` 49
(d) 1 : 10 (e) None of these (d) ` 47 (e) None of these
6. A chemist has 10 litres of a solution that is 10 per cent nitric 12. How many litres of pure alcohol must be added to 10 litres of
acid by volume. He wants to dilute the solution to 4 per mixture which is 15% alcohol to make a mixture which will be
cent strength by adding water. How many litres of water 25% alcohol?
must he add ?
(a) 15 (b) 20 (c) 18 5 5 3
(a) (b) (c)
(d) 25 (e) None of these 4 2 4

2 4
7. In what ratio must water be mixed with milk to gain 16 % (d) (e) None of these
3 3
on selling the mixture at cost price?
(a) 1 : 6 (b) 6 : 1 (c) 2 : 3
(d) 4 : 3 (e) None of these

1. a b c d e 2. a b c d e 3. a b c d e 4. a b c d e 5. a b c d e
RESPONSE 6. a b c d e 7. a b c d e 8. a b c d e 9. a b c d e 10. a b c d e

GRID 11. a b c d e 12. a b c d e


y
o
u
rs
m
a
h
76 SPEED TEST 37

b
o
o
13. How many kg of custard powder costing ` 40 kg must be 20. If 50% of a 2 : 3 solution of milk and water is replaced with

b
mixed with 16 kg of custard powder costing ` 55 kg so that water, then the concentration of the solution is reduced by:

.w
25% may be gained by selling the mixture at ` 60 kg? (a) 25% (b) 33.33% (c) 50%

o
rd
(a) 11 kg (b) 14 kg (c) 12 kg (d) 75% (e) None of these

p
(d) 20 kg (e) None of these 21. In a mixture of 45 litres, the ratio of milk and water is 3 : 2.

re
14. 300 gm of sugar solution has 40% sugar in it. How much How much water must be added to make the ratio 9 : 11?

s
s
.c
sugar should be added to make it 50% in the solution? (a) 10 litres (b) 15 litres (c) 17 litres

o
(a) 40 gm (b) 50 gm (c) 60 gm (d) 20 litres (e) None of these

m
(d) 70 gm (e) None of these 22. Three containers A, B and C are having mixtures of milk and
15. Gold is 19 times as heavy as water and copper 9 times. In water in the ratio 1 : 5, 3 : 5 and 5 : 7, respectively. If the
what ratio should these metals be mixed so that the mixture capacities of the containers are in the ratio 5 : 4 : 5, then find
may be 15 times as beavy as water? the ratio of the milk to the water if the mixtures of all the three
(a) 1 : 2 (b) 3 : 2 (c) 2 : 3 containers are mixed together.
(d) 4 : 5 (e) None of these (a) 51 : 115 (b) 52 : 115 (c) 53 : 115
16. In a mixture of 60 litres, the ratio of milk to water is 2 : 1. If the (d) 54 : 115 (e) None of these
ratio of milk to water is to be 1 : 2, then amount of water to be 23. Five litres of water is added to a certain quantity of pure milk
further added is ___________. costing Rs. 3 per litre. If by selling the mixture at the same
(a) 20 (b) 40 (c) 60 price as before, a profit of 20% is made, then what is the
(d) 80 (e) None of these amount of pure milk in the mixture ?
17. In a mixture of milk and water the proportion of water by (a) 20 litres (b) 30 litres (c) 25 litres
weight was 75%. If in 60 gm of mixture 15 gm water was (d) 35 litres (e) None of these
added, what would be the percentage of water? (Weight in 24. How many kg of tea worth Rs 25 per kg must be blended
gm) with 30 kg of tea worth Rs 30 per kg so that by selling the
(a) 75% (b) 88% (c) 90% blended variety at Rs 30 per kg there should be a gain of
(d) 100% (e) None of these 10%?
18. In what ratio must tea at Rs. 62 per kg be mixed with tea at (a) 32 kg (b) 40 kg (c) 36 kg
Rs. 72 per kg so that the mixture must be worth Rs. 64.50 per (d) 42 kg (e) None of these
kg? 25. How many kg of sugar costing Rs 5.75 per kg should be
(a) 3 : 1 (b) 3 : 2 (c) 4 : 3 mixed with 75 kg of cheaper sugar costing Rs 4.50 per kg so
(d) 5 : 3 (e) None of these that the mixture is worth Rs 5.50 per kg ?
19. Two vessels A and B contain spirit and water mixed in the (a) 350 kg (b) 300 kg (c) 250 kg
ratio 5 : 2 and 7 : 6 respectively. Find the ratio in which these (d) 325 kg (e) None of these
mixture be mixed to obtain a new mixture in vessel C contain-
ing spirit and water in the ratio 8 : 5 ?
(a) 4 : 3 (b) 3 : 4 (c) 5 : 6
(d) 7 : 9 (e) None of these

13. a b c d e 14. a b c d e 15. a b c d e 16. a b c d e 17. a b c d e

18. a b c d e 19. a b c d e 20. a b c d e 21. a b c d e 22. a b c d e


RESPONSE 23. a c d e 24. a c d e 25. a c d e
b b b
GRID
y
o
u
rs
m
a
h
b
38

o
o
Profit, Loss &

b
.w
o
rd
p
Discount

re
s
s
.c
o
m
Max. Marks : 30 No. of Qs. 30 Time : 20 min. Date : ........./......../................
1. If the cost price is 96% of the selling price, then what is the 9. A man buys milk at Rs 6 per litre and adds one third of water
profit percent? to it and sells mixture at Rs 7.20 per litre. The gain is
(a) 4.5% (b) 4.2% (c) 4% (a) 40% (b) 80% (c) 60%
(d) 3.8% (e) None of these (d) 25% (e) None of these
2. If the manufacturer gains 10%, the wholesale dealer 15% 10. A dishonest fruit seller professes to sell his goods at the cost
and the retailer 25%, then find the cost of production of a price but weighs 800 grams for a kg weight. Find his gain
table, the retail price of which is ` 1265? percent.
(a) ` 800 (b) ` 1000 (c) ` 900 (a) 100% (b) 150% (c) 50%
(d) ` 600 (e) None of these (d) 200% (e) None of these
1 11. Two electronic musical instruments were purchased for
3. A man sold his book for Rs 891, thereby gaining of its ` 8000. The first was sold at a profit of 40% and the second at
10
loss of 40%. If the sale price was the same in both the cases,
cost price. Find his cost price.
what was the cost price of two electronic musical instruments?
(a) ` 850 (b) ` 810 (c) ` 851
(a) ` 2000, ` 5000 (b) ` 2200, ` 5500
(d) ` 840 (e) None of these
(c) ` 2400, ` 5000 (d) ` 2400, ` 5600
4. A man buys 50 pencils for Rs 100 and sells 45 pencils for
` 90. Find his gain or loss %. (e) None of these
12. A tradesman is marketing his goods 20% above the cost price
(a) 20% (b) 35% (c) 25%
of the goods. He gives 10% discount on cash payment, find
(d) No gain or loss (e) None of these
his gain percent.
5. A dealer sold a mixer for ` 420 at a loss of 12.5%. At what
(a) 12% (b) 8% (c) 15%
price should he have sold it to gain 12.5%.
(d) 18% (e) None of these
(a) ` 620 (b) ` 540 (c) ` 650
13. A man sells his car for ` 5000 and loses something. Had he
(d) ` 750 (e) None of these
sold it for ` 5600, his gain would have been double the former
6. A man sold 10 eggs for 5 rupees and gained 20%.How loss. Find the cost price.
many eggs did he buy for 5 rupees?
(a) ` 5500 (b) ` 5100 (c) ` 5400
(a) 10 eggs (b) 12 eggs (c) 14 eggs
(d) ` 5200 (e) None of these
(d) 16 eggs (e) None of these
14. John sold a fan at a loss of 7%. If he had sold it for ` 48 more,
7. Coconuts were purchased at ` 150 per hundred and sold at he would have gained 5%. Find the cost price of the fan.
` 2 per coconut. If 2000 coconuts were sold, what was the
(a) ` 350 (b) ` 480 (c) ` 240
total profit made?
(d) ` 400 (e) None of these
(a) ` 500 (b) ` 1000 (c) ` 1500
15. The owner of an electronics shop charges his customer 22%
(d) ` 2000 (e) None of these
more than the cost price. If a customer paid ` 10,980 for a
8. A shopkeeper’s price is 50% above the cost price. If he
DVD player, then what was the cost price of the DVD player?
allows his customer a discount of 30% what profit does he
make? (a) ` 8000 (b) ` 8800 (c) ` 9500
(a) 5% (b) 10% (c) 15% (d) ` 9200 (e) None of these
(d) 20% (e) None of these

1. a b c d e 2. a b c d e 3. a b c d e 4. a b c d e 5. a b c d e
RESPONSE 6. a b c d e 7. a b c d e 8. a b c d e 9. a b c d e 10. a b c d e

GRID 11. a b c d e 12. a b c d e 13. a b c d e 14. a b c d e 15. a b c d e


y
o
u
rs
m
a
h
78 SPEED TEST 38

b
o
o
16- Cost of 24 bats and 32 sticks is `5,600. What is the price of 24. If the cost of 12 pencils is equal to the selling price of 10

b
pencils, the profit percent in the transaction is :

.w
3 bats and 4 sticks?
(a) ` 1,400 (b) ` 2,800

o
1 1

rd
(a) 16 % (b) 22 % (c) 20%
(c) ` 700 (d) Cannot be determined 3 2

p
re
(e) None of these (d) 25% (e) None of these

s
17. The profit earned after selling an article for ` 1,754 is the

s
25. Two motor cars were sold for Rs 9,900 each, gaining 10% on

.c
same as loss incurr ed after selling the article for one and losing 10% on the other. The gain or loss percent in

o
m
` 1,492. What is the cost price of the article? the whole transaction is :
(a) ` 1,623 (b) ` 1,523 (c) ` 1,689 (a) Neither loss no gain
(d) ` 1,589 (e) None of these 1 100
18. Prathik sold a music system to Karthik at 20% gain and Karthik (b) % gain (c) % profit
99 99
sold it to Swasthik at 40% gain. If Swasthik paid `10,500 for
(d) 1% loss (e) None of these
the music system, what amount did Prathik pay for the same?
26. The retail price of a water geyser is Rs 1265. If the
(a) ` 8,240 (b) ` 7,500 (c) ` 6,250 manufacturer gains 10%, the wholesale dealer gains 15%
(d) Cannot be determined and the retailer gains 25%, then the cost of the geyser is :
(e) None of these (a) Rs 800 (b) Rs 900 (c) Rs 550
19. The cost of 5 pendants and 8 chains is ` 1,45,785. What (d) Rs 650 (e) None of these
would be the cost of 15 pendants and 24 chains? 27. A man buys a watch for Rs. 1950 in cash and sells it for Rs.
(a) ` 3,25,285 (b) ` 4,39,355 2200 on credit of 1 year. If the rate of interest is 10% per
(c) ` 5,50,000 (d) Cannot be determined annum, then the man
(e) None of these (a) Gains Rs. 55 (b) Gains Rs. 50
20. What price should a shopkeeper mark on an article costing (c) Loses Rs. 30 (d) Gains Rs. 30
him Rs 153 to gain 20% after allowing a discount of 15% ? (e) None of these
(a) Rs 162 (b) Rs 621 (c) Rs 216 28. A shopkeeper marks up his goods to gain 35%. But he allows
(d) Rs 226 (e) None of these 10% discount for cash payment. His profit on the cash
21. Three successive discounts of 10%, 12% and 15% amount transaction in percentage, is
to a single discount of :
1 1
(a) 36.28 % (b) 34.68% (c) 37 % (a) 13 (b) 25 (c) 21
(d) 38% (e) None of these 2 2
22. A machine is sold at a profit of 10%. Had it been sold for Rs 1
80 less, there would have been a loss of 10%. The cost price (d) 31 (e) None of these
2
of the machine is : 29. A man sold two steel chairs for Rs. 500 each. On one he
(a) Rs 350 (b) Rs 400 (c) Rs 450 gains 20% and on other, he loses 12%. How much does he
(d) Rs 520 (e) None of these gain or lose in the whole transaction?
23. If selling price is doubled then, the profit triples. What is (a) 1.5% gain (b) 2% gain (c) 1.5% loss
profit per cent : (d) 2% loss (e) None of these
2 1 30. By selling 12 notebooks, the seller earns a profit equal to the
(a) 66 (b) 100 (c) 33 selling price of two notebooks. What is his percentage
3 3
profit?
(d) 125 (e) None of these
2
(a) 25% (b) 20% (c) 16 %
3
(d) Data inadequate
(e) None of these

16. a b c d e 17. a b c d e 18. a b c d e 19. a b c d e 20. a b c d e

21. a b c d e 22. a b c d e 23. a b c d e 24. a b c d e 25. a b c d e


RESPONSE 26. a c d e 27. a c d e 28. a c d e 29. a c d e 30. a c d e
b b b b b
GRID
y
o
u
rs
m
a
h
b
39

o
o
b
.w
Simple Interest

o
rd
p
re
s
s
.c
o
m
Max. Marks : 30 No. of Qs. 30 Time : 20 min. Date : ........./......../................
1. Ms. Sandhya deposits an amount of ` 31,400 to obtain a 8. What total amount would Mithilesh get at the end of three
simple interest at the rate of 12 per cent per annum for 8 years if he invests an amount of ` 11200 in a scheme, which
years. What total amount will Ms. Sandhya get at the end offers simple interest 8.5% per annum for three years ?
of 8 years? (a) ` 14056 (b) ` 14348 (c) ` 13852
(a) ` 31,444 (b) ` 61,544 (c) ` 41,544 (d) ` 15064 (e) None of these
(d) ` 31,144 (e) None of these 9. A sum of Rs. 2600 is lent out in two parts in such a way that
2. Mr. Deepak invested an amount of ` 21,250 for 6 years. At the interest on one part at 10% for 5 years is equal to that on
what rate of simple interest will he obtain the total amount the other part at 9% for 6 years. The sum lent out at 10% is
of ` 26,350 at the end of 6 years? ________ .
(a) 6 % p.a (b) 5 % p.a (c) 8 % p.a (a) 1250 (b) 1350 (c) 1450
(d) 12 % p.a (e) None of these (d) 1550 (e) 1650
3. Asmita invests an amount of ` 9535 at the rate of 4 per cent 10. At a certain rate of simple interest, a certain sum doubles
per annum to obtain a total amount of ` 11442 on simple itself in 10 years. It will treble itself in years ________ .
interest after a certain period. For how many year did she (a) 10 (b) 20 (c) 25
invest the amount to obtain the total sum? (d) 30 (e) 45
(a) 10 years (b) 2 years (c) 5 years 11. A sum of money at simple interest amounts to ` 600 in 4 years
(d) 4 years (e) None of these and ` 650 in 6 years. Find the rate of interest per annum.
4. Girish invested a certain amount at the rate of 8% p.a. for 6 (a) 3% (b) 5% (c) 9%
year to obtain an amount of ` 28,046. How much amount (d) 10% (e) 15%
did Girish obtain as simple interest? 12. A person lent at certain sum of money at 4% simple interest;
(a) `12,550 (b) `9,096 (c) `18,950 and in 8 years the interest amounted to ` 340 less than the
(d) Cannot be determined sum lent. Find the sum lent.
(e) None of these (a) 500 (b) 600 (c) 1000
5. Mr. Anuraag Awasthi deposits an amount of ` 56500 to (d) 1500 (e) 1700
obtain a simple interest at the rate of 12% p.a. for 3 years. 1
What total amount will Mr. Anuraag Awasthi get at the end 13. In what time will ` 72 become ` 81 at 6 % p.a. simple interest?
4
of 3 year ?
(a) 1 year 6 months (b) 2 years (c) 1 years
(a) `75680 (b) `77540 (c) `76840
1
(d) `73420 (e) None of these (d) 2
years (e) None of these
6. Veena obtained an amount of ` 8, 376/- as simple interest 2
on a certain amount at 8% p.a. after 6 years. What is the 14. A sum of money lent out at simple interest amounts to
amount invested by Veena? ` 1008 in 2 years and ` 1164 in 3½ years. Find the rate % p.a.
(a) ` 17,180 (b) ` 18,110 (c) ` 16,660 1
(d) ` 17,450 (e) None of these (a) 13% (b) 14% (c) 12 %
2
7. The simple interest accrued on a sum of certain principal is
(d) 15% (e) None of these
` 2000 in five years at the rate of 4% per annum. What
would be the compound interest accrued on same principal 15. A sum of money lent out at simple interest amounts to ` 720
at same rate in two years ? after 2 years and to ` 1,020 after a further period of 5 years.
(a) ` 716 (b) ` 724 (c) ` 824 Find the sum and the rate %.
(d) ` 816 (e) None of these (a) ` 500, 10% (b) ` 600, 10% (c) ` 500, 12%
(d) ` 600, 12% (e) None of these

1. a b c d e 2. a b c d e 3. a b c d e 4. a b c d e 5. a b c d e
RESPONSE 6. a b c d e 7. a b c d e 8. a b c d e 9. a b c d e 10. a b c d e

GRID 11. a b c d e 12. a b c d e 13. a b c d e 14. a b c d e 15. a b c d e


y
o
u
rs
m
a
h
80 SPEED TEST 39

b
o
o
16. On retirement, a person gets 1.53 lakhs of his provident fund 24. Nitin borrowed some money at the rate of 6% p.a. for the

b
which he invests in a scheme at 20% p.a. His monthly income first three years, 9% p.a. for the next five years and 13% p.a.

.w
from this scheme will be for the period beyond eight years If the total interest paid by

o
rd
(a) ` 2, 450 (b) ` 2,500 (c) ` 2, 550 him at the end of eleven years is ` 8160, how much money

p
(d) ` 2, 600 (e) None of these did he borrow?

re
17. A sum was put at simple interest at a certain rate for 4 years (a) ` 8000 (b) ` 10,000 (c) ` 12,000

s
s
Had it been put at 2% higher rate, it would have fetched ` 56 (d) Data inadequate (e) None of these

.c
more. Find the sum. 25. An automobile financier claims to be lending money at simple

o
m
(a) ` 500 (b) ` 600 (c) ` 700 interest, but he includes the interest every six months for
(d) ` 800 (e) None of these calculating the principal. If he is charging an interest of 10%,
18. The simple interest on ` 200 for 7 months at 5 paise per the effective rate of interest becomes :
rupee per month is (a) 10% (b) 10.25% (c) 10.5%
(a) ` 70 (b) ` 7 (c) ` 35 (d) Data inadequate (e) None of these
(d) ` 30.50 (e) None of these 26. A lent ` 5000 to B for 2 years and ` 3000 to C for 4 years on
19. A father left a will of ` 68,000 to be divided between his two simple interest at the same rate of interest and received
sons aged 10 years and 12 years such that they may get ` 2200 in all from both of them as interest. The rate of interest
equal amount when each attains the age of 18 years If the per annum is:
money is reckoned at 10% p.a., find how much each gets at 1
the time of the will. (a) 5% (b) 7% (c) 7 %
8
(a) ` 30,000, ` 38,000 (b) ` 28,000, ` 40,000
(c) ` 32,000, ` 36,000 (d) cannot be determined. (d) 10% (e) None of these
(e) None of these 27. The rates of simple interest in two banks A and B are in the
20. In how many minimum number of complete years, the interest ratio 5 : 4. A person wants to deposit his total savings in two
on ` 212.50 P at 3% per annum will be in exact number of banks in such a way that he received equal half-yearly
rupees? interest from both. He should deposit the savings in banks
(a) 6 (b) 8 (c) 9 A and B in the ratio.
(a) 2 : 5 (b) 4 : 5 (c) 5 : 2
(d) 7 (e) None of these
(d) 5 : 4 (e) None of these
21. What annual instalment will discharge a debt of ` 4,200 due
in 5 years at 10% simple interest? 28. If a certain sum of money becomes double at simple interest
(a) ` 500 per year (b) ` 600 per year in 12 years, what would be the rate of interest per annum ?
(c) ` 700 per year (d) ` 800 per year 1
(a) 8 (b) 10 (c) 12
(e) None of these 3
22. A certain amount earns simple interest of ` 1750 after 7 years (d) 14 (e) None of these
Had the interest been 2% more, how much more interest 29. Two equal sums were borrowed at 8% simple interest per
would it have earned? annum for 2 years and 3 years, respectively. The difference
(a) ` 35 (b) ` 245 (c) ` 350 in the interests was Rs 56. The difference in the interests
(d) Cannot be determined was Rs 56. The sum borrowed were :
(e) None of these (a) Rs 690 (b) Rs 700 (c) Rs 740
23. What will be the ratio of simple interest earned by certain (d) Rs 780 (e) None of these
amount at the same rate of interest for 6 years and that for 30. How much interest will Rs 10,000 earn in 9 months at an
9 years? annual rate of 6 percent ?
(a) 1 : 3 (b) 1 : 4 (c) 2 : 3 (a) Rs 450 (b) Rs 460 (c) Rs 475
(d) Data inadequate (e) None of these (d) Rs 600 (e) None of these

16. a b c d e 17. a b c d e 18. a b c d e 19. a b c d e 20. a b c d e


RESPONSE 21. a b c d e 22. a b c d e 23. a b c d e 24. a b c d e 25. a b c d e
GRID 26. a b c d e 27. a b c d e 28. a b c d e 29. a b c d e 30. a b c d e
y
o
u
rs
m
a
h
b
40

o
o
b
.w
Compound Interest

o
rd
p
re
s
s
.c
o
m
Max. Marks : 25 No. of Qs. 25 Time : 20 min. Date : ........./......../................
1. Sudhanshu invested ` 15,000 at interest @ 10% p.a for one 7. Rohit invested some amount at the rate of 6 pcpa and at the
year. If the interest is compounded every six months what end of 2 yr he got ` 8730 simple interest. How much compound
amount will Sudhanshu get at the end of the year? interest he will get on same amount and same rate of interest
(a) ` 16,537.50 (b) ` 16,5000 after 2 yr.
(c) ` 16,525.50 (d) ` 18,150 (a) ` 5820 (b) ` 5949.60 (c) ` 5900
(e) None of these (d) ` 5994.60 (e) None of these
2. What would be the compound interest obtained on an 8. Sonika invested an amount of ` 5800 for 2 years. At what rate
amount of ` 1,250 at the rate of 8% p.a. after 2 year ? of compound interest will she get an amount of ` 594.5 at the
end of two years ?
(a) ` 200 (b) ` 208 (c) `212
(a) 5 p.c.p.a. (b) 4 p.c.p.a. (c) 6 p.c.p.a.
(d) `220 (e) None of these (d) 8 p.c.p.a. (e) None of these
3. Shyam invests an amount of ` 5,690 at the rate of 5 per cent 9. What would be the compound interest accrued on an amount
per annum for 3 years. What approximate amount of of ` 7400 @ 13.5 p.c.p.a. at the end of two years ?
compound interest will he obtain at the end of 3 years? (Rounded off to two digits after decimal)
(a) ` 854 (b) ` 799 (c) ` 843 (a) ` 2136.87 (b) ` 2306.81 (c) ` 2032.18
(d) ` 787 (e) ` 897 (d) ` 2132.87 (e) None of these
4. The simple interest accrued on an amount of ` 84,000 at the 10. If the compound interest accrued on an amount of ` 14500 in
end of three year is ` 30,240. What would be the compound two years is ` 4676.25, what is the rate of interest p.c.p.a ?
interest accrued on the same amount at the same rate in the (a) 11 (b) 9 (c) 15
same period? (d) 18 (e) None of these
(a) ` 30,013.95 (b) ` 31,013.95 11. What will be the compound interest accrued on an amount of
(c) ` 32,013.95 (d) ` 33,013.95 ` 10000 @ per annum in two years if the interest is
(e) ` 34,013-95 compounded half-yearly ?
5. What will be the difference between the compound interest (a) ` 4400 (b) ` 4600 (c) ` 4641
and simple interest at the rate of 5% p.a. on an amount of (d) ` 4680 (e) None of these
` 4,000 at the end of two years? 12. What will be the difference between the simple interest and
(a) ` 10 (b) ` 20 compound interest earned on a sum of ` 985.00 at the rate of
(c) ` 30 (d) Data inadequate 14% per annum at the end of two years ?
(e) None of these (a) ` 16.408 (b) ` 14.214 (c) ` 19.218
6. Pamela invested an amount of ` 35,000 for two year at the (d) ` 17.405 (e) None of these
rate of 5% p.a. What amount of compound interest would 13. A sum of money at compound interest amounts in two years
she receive at the end of two year ? to ` 2809, and in three years to ` 2977.54. Find the rate of
(a) ` 3587.50 (b) ` 3500 (c) ` 3580.50 interest and the original sum.
(d) ` 3565.50 (e) None of these (a) 2000 (b) 2100 (c) 2200
(d) 2500 (e) 3000

1. a b c d e 2. a b c d e 3. a b c d e 4. a b c d e 5. a b c d e
RESPONSE 6. a b c d e 7. a b c d e 8. a b c d e 9. a b c d e 10. a b c d e

GRID 11. a b c d e 12. a b c d e 13. a b c d e


y
o
u
rs
m
a
h
82 SPEED TEST 40

b
o
o
14. In what time will 6250 amount to ` 6632.55 at 4 p.c. compound 20. A sum of money invested at compound interest amounts in

b
interest payable half-yearly? 3 years to Rs 2,400 and in 4 years to Rs. 2,520. The interest

.w
rate per annum is :

o
3 3 3

rd
(a) (b) (c) (a) 6% (b) 5% (c) 10%
5 2 4

p
(d) 12% (e) None of these

re
5 9 21. A computer is available for Rs. 39,000 cash or Rs. 17,000 as

s
s
(d) (e) cash down payment followed by five monthly installments

.c
2 11
of Rs. 4,800 each. What is the rate of interest under the

o
15. If the simple interest on a certain sum of money for 3 years at

m
instalment plan ?
5% is ` 150, find the corresponding CI.
(a) 35.71% p.a. (b) 37.71% p.a.
(a) 197 (b) 157.62 (c) 137.36
(c) 36.71% p.a. (d) 38.71% p.a.
(d) 117.17 (e) 127.34
16. A sum of money becomes eight times in 3 years if the rate is (e) None of these
compounded annually. In how much time, the same amount 22. Under the Rural Housing Scheme, the Delhi Development
at the same compound interest rate will become sixteen times? Authority (DDA) allotted a house to Kamal Raj for Rs.
(a) 6 years (b) 4 years (c) 8 years 1,26,100. This payment is to be made in three equal annual
(d) 5 years (e) None of these instalments. If the money is reckoned at 5% per annum
17. The difference between the simple interest and the compound compound interest, then how much is to be paid by Kamal
interest compounded annually at the rate of 12% per annum Raj in each instalment ?
on Rs 5000 for two years will be : (a) Rs. 45,205 (b) Rs. 47,405 (c) Rs. 46,305
(a) Rs 47.50 (b) Rs 63 (c) Rs 45 (d) Rs. 48,505 (e) None of these
(d) Rs 72 (e) None of these 23. A finance company declares that, at a certain compound
18. Rahul borrowed a certain sum from Dhawan at a certain rate interest rate, a sum of money deposited by anyone will
of simple interest for 2 years. He lent this sum to Ramesh at become 8 times in three years. If the same amount is deposited
the same rate of interest compounded annually for the same at the same compound rate of interest, then in how many
period. At the end of two years, he received Rs 4200 as year will it become 16 times ?
compound interest but paid Rs 4000 only as simple interest. (a) 5 years (b) 4 years (c) 6 years
Find the rate of interest. (d) 7 years (e) None of these
(a) 12% (b) 25% (c) 35% 24. Seema invested an amount of Rs. 16,000 for two years on
(d) 10% (e) None of these compound interest and received an amount of Rs. 17,640 on
19. What will Rs 1000 be worth after three years if it earns interest maturity. What is the rate of interest ?
at the rate of 5% compounded annually ? (a) 5 % pa (b) 8 % pa
(a) Rs 1075 (b) Rs 1257 (c) Rs 1157 (c) 4 % pa (d) Data inadequate
(d) Rs 1300 (e) None of these (e) None of these
25. Two friends A and B jointly lent out Rs. 81,600 at 4% per
annum compound interest. After 2 years A gets the same
amount as B gets after 3 years. The investment made by B
was
(a) Rs. 40,000 (b) Rs. 30,000 (c) Rs. 45,000
(d) Rs. 38,000 (e) None of these

14. a b c d e 15. a b c d e 16. a b c d e 17. a b c d e 18. a b c d e


RESPONSE 19. a b c d e 20. a b c d e 21. a b c d e 22. a b c d e 23. a b c d e
GRID 24. a b c d e 25. a b c d e
y
o
u
rs
m
a
h
b
41

o
o
b
.w
Distance, Speed

o
rd
p
re
and Time

s
s
.c
o
m
Max. Marks : 30 No. of Qs. 30 Time : 20 min. Date : ........./......../................
1. How many seconds will a train 60 m in length, travelling at 8. A 200 meter long train crosses a platform double its length in
the rate of 42 km an hour, rate to pass another train 84 m 36 seconds. What is the speed of the train in km/hr ?
long, proceeding in the same direction at the rate of 30 km (a) 60 (b) 48 (c) 64
an hour?
(d) 66 (e) None of these
(a) 42 (b) 43.2 (c) 45
9. A 160 meter long train running at a speed of 90 km/h crosses
a platform in 18 seconds. What is the length of the platform in
(d) 50 (e) None of these
meters?
2. A train takes 5 seconds to pass an electric pole. If the length
of the train is 120 metres, the time taken by it to cross a (a) 210 (b) 240 (c) 290
railway platform 180 metres long is _______ seconds. (d) 310 (e) None of these
(a) 12.5 (b) 13.5 (c) 14.5 10. Excluding the stoppages, the speed of a bus is 64 km/hr and
(d) 15.5 (e) None of these including the stoppages the speed of the bus is 48 km/hr. For
how many minutes does the bus stop per hour?
3. A train is running at the rate of 40 kmph. A man is also
(a) 12.5 minutes (b) 15 minutes (c) 10 minutes
going in the same direction parallel to the train at the speed (d) 18 minutes (e) None of these
of 25 kmph. If the train crosses the man in 48 seconds, the 11. The ratio between the speed of a train and a car is 18 : 13 .
length of the train is _______ metres. Also, a bus covered a distance of 480 kms. in 12 hours. The
(a) 100 (b) 150 (c) 200 speed of the bus is five-ninth the speed of the train. How
(d) 250 (e) None of these much distance will the car cover in 5 hours ?
4. A train speeds past a pole in 15 seconds and speeds past a (a) 250 km. (b) 280 km.
platform 100 metres long in 25 seconds. Its length in metres (c) 260 km. (d) Cannot be determined
is ________ . (e) None of these
(a) 50 (b) 100 (c) 150 12. A 300 meter long train moving with an average speed of 126
(d) 200 (e) None of these km/hr crosses a platform in 24 seconds. A man crosses the
5. A bus covers a distance of 2,924 kms in 43 hours. What is same platform in 5 minutes. What is the speed of the man in
the speed of the bus? meters/second
(a) 72 kmph (b) 60kmph (a) 1.8 m/s (b) 1.2 m/s
(c) 68kmph (d) Cannot be determined (c) 1.5 m/s (d) Cannot be determined
(e) None of these (e) None of these
6. A bus travels at the speed of 49 kmph and reaches its 13. A bike covers a certain distance at the speed of 64 km/hr in 8
destination in 7 hours. What is the distance covered by the hours. If a bike was to cover the same distance in
bus? approximately 6 hours, at what approximate speed should the
(a) 343 km (b) 283 km (c) 353 km bike travel?
(d) 245 km (e) 340 km (a) 80 km./hr. (b) 85 km/hr. (c) 90 km./hr.
7. Nilesh goes to school from his village & returns at the (d) 75 km/hr. (e) 70 km./hr
speed of 4 km/hr. If he takes 6 hours in all, then what is the
distance between the village and the school?
(a) 6 km (b) 5 km
(c) 4 km (d) Cannot be determined
(e) None of these

1. a b c d e 2. a b c d e 3. a b c d e 4. a b c d e 5. a b c d e
RESPONSE 6. a b c d e 7. a b c d e 8. a b c d e 9. a b c d e 10. a b c d e

GRID 11. a b c d e 12. a b c d e 13. a b c d e


y
o
u
rs
m
a
h
84 SPEED TEST 41

b
o
o
14. The driver of a car driving @ 36 kmph locates a bus 40 23. A train does a journey without stoppage in 8 hours, if it had

b
meters ahead of him. After 20 seconds the bus is 60 meters travelled 5 km/h faster, it would have done the journey in 6

.w
behind. The speed of the bus is : hours 40 minutes. Find its original speed.

o
rd
(a) 36 kmph (b) 20 m/sec. (c) 72 m/sec. (a) 25 km/h (b) 40 km/h (c) 45 km/h

p
(d) 18 kmph (e) None of these (d) 36.5 km/h (e) None of these

re
15. Two trains 100 meters and 120 meters long are running in the 24. A car travels 25 km an hour faster than a bus for a journey of

s
s
same direction with speeds of 72 km/h and 54 km/h. In how 500 km. If the bus takes 10 hours more than the car, then the

.c
much time will the first train cross the second? speeds of the bus and the car are

o
m
(a) 50 sec (b) 44 sec (c) 38 sec (a) 25 km/h and 40 km/h respectively
(d) 42 sec (e) None of these (b) 25 km/h and 60 km/h respectively
16. A man covers a certain distance on a scooter. If the scooter (c) 25 km/h and 50 km/h respectively
moved 4 km/h faster, it would take 30 minutes less. If it moved (d) Cannot be determined
2 km/h slower, it would have taken 20 minutes more. Find the (e) None of these
distance. 25. A thief goes away with a Maruti car at a speed of 40 km/h.
(a) 60 km (b) 58 km (c) 55 km The theft has been discovered after half an hour and the
(d) 50 km (e) None of these owner sets off in another car at 50 km/h. When will the owner
17. A boat running downstream covers a distance of 16 km in 2 overtake the thief from the start.
hours while for covering the same distance upstream, it takes 1
4 hours. What is the speed of the boat in still water? (a) 2 hours (b) 2 hr 20 min
2
(a) 4 km/h (b) 6 km/h (c) 1 hr 45 min (d) Cannot be determined
(c) 8 km/h (d) Data inadequate (e) None of these
(e) None of these 26. In a flight of 600 km, an aircraft was slowed down due to bad
18. In a 800 m race around a stadium having the circumference weather. Its average speed for the trip was reduced by 200
of 200 m, the top runner meets the last runner on the 5th km/ hr and the time of flight increased by 30 minutes. The
minute of the race. If the top runner runs at twice the speed duration of the flight is:
of the last runner, what is the time taken by the top runner to (a) 1 hours (b) 2 hours (c) 3 hours
finish the race ?
(d) 4 hours (e) None of these
(a) 20 min (b) 15 min (c) 10 min
27. A train covers 180 km distance in 4 hours. Another train
(d) 5 min (e) None of these
covers the same distance in 1 hour less. What is the difference
19. A long distance runner runs 9 laps of a 400 meters track
in the distances covered by these trains in one hour ?
everyday. His timings (in minutes) for four consecutive days
(a) 45 km (b) 9 km (c) 40 km
are 88, 96, 89 and 87 resplectively. On an average, how many
(d) 25 km (e) None of these
meters/minute does the runner cover ?
28. The jogging track in a sports complex is 726 metres in
(a) 40 m/min (b) 45 m/min (c) 38 m/min
circumference. Pradeep and his wife start from the same point
(d) 49 m/min (e) None of these
and walk in opposite directions at 4.5 km/h and 3.75 km/h,
20. A cyclist covers a distance of 750 m in 2 min 30 sec. What is respectively. They will meet for the first time in :
the speed in km/h of the cyclist ? (a) 5.5 min (b) 6.0 min (c) 5.28 min
(a) 18 km/h (b) 15 km/h (c) 20 km/h (d) 4.9 min (e) None of these
(d) 22 km/h (e) None of these 29. The speed of a boat in still water is 15 km/h and the rate of
21. An aeroplane flies along the four sides of a square at the stream is 5 km/h. The distance travelled downstream in 24
speeds of 200, 400, 600 and 800 km/h. Find the average speed minutes is
of the plane around the field. (a) 4 km (b) 8 km (c) 6 km
(a) 384 km/h (b) 370 km/h (c) 368 km/h (d) 16 km (e) None of these
(d) 378 km/h (e) None of these 30. A man makes his upward journey at 16 km/h and downward
22. R and S start walking each other at 10 AM at the speeds of journey at 28 km/h. What is his average speed ?
3 km/h and 4 km/h respectively. They were initially 17.5 km (a) 32 km/h (b) 56 km/h (c) 20.36 km/h
apart. At what time do they meet? (d) 22 km/h (e) None of these
(a) 2 : 30 PM (b) 11 : 30 AM (c) 1 : 30 PM
(d) 12 : 30 PM (e) None of these

14. a b c d e 15. a b c d e 16. a b c d e 17. a b c d e 18. a b c d e

RESPONSE 19. a b c d e 20. a b c d e 21. a b c d e 22. a b c d e 23. a b c d e

GRID 24. a b c d e 25. a b c d e 26. a b c d e 27. a b c d e 28. a b c d e

29. a b c d e 30. a b c d e
y
o
u
rs
m
a
h
b
42

o
o
b
Time and Work /

.w
o
rd
p
Pipe and Cistern

re
s
s
.c
o
m
Max. Marks : 30 No. of Qs. 30 Time : 20 min. Date : ........./......../................
1. ‘A’ can complete a piece of work in 12 days. ‘A’ and ‘B’ together 7. Sunil and Pradeep can complete a work in 5 days and 15 days
can complete the same piece of work in 8 days. In how many respectively. They both work for one day and then Sunil
days can ‘B’ alone complete the same piece of work? leaves. In how many days will the remaining work be completed
by Pradeep ?
(a) 15 days (b) 18 days (c) 24 days
(a) 11 days (b) 12 days (c) 15 days
(d) 28 days (e) None of these (d) 8 days (e) None of these
2. George takes 8 hours to copy a 50 page manuscript while 8. If 6 men and 8 boys can do a piece of work in 10 days while 26
Sonia can copy the same manuscript in 6 hours. How many men and 48 boys can do the same in 2 days, the time taken by
hours would it take them to copy a 100 page manuscript, if 15 men and 20 boys in doing the same work will be:
they work together ? (a) 4 days (b) 5 days (c) 6 days
6 5 (d) 7 days (e) None of these
(a) 6 (b) 9 (c) 9 9. 12 men complete a work in 9 days. After they have worked for
7 7
6 days, 6 more men join them. How many days will they take to
(d) 14 (e) None of these
complete the remaining work?
3. A can finish a work in 18 days and B can do the same work
(a) 2 days (b) 3 days (c) 4 days
in half the time taken by A. Then, working together, what
(d) 5 days (e) None of these
part of the same work can they finish in a day?
10. A and B can do a job in 16 days and 12 days respectively. B
1 1 2 has started the work alone 4 days before finishing the job, A
(a) (b) (c) joins B. How many days has B worked alone?
6 9 5
(a) 6 days (b) 4 days (c) 5 days
2
(d) (e) None of these (d) 7 days (e) None of these
7 11. Three taps A, B and C can fill a tank in 12, 15 and 20 hours
4. A and B can finish a work in 10 days while B and C can do it respectively. If A is open all the time and B and C are open for
in 18 days. A started the work, worked for 5 days, then B one hour each alternately, then the tank will be full in :
worked for 10 days and the remaining work was finished by
2
C in 15 days. In how many days could C alone have finished (a) 6 hrs. (b) 6 hrs. (c) 7 hrs.
the whole work ? 3
(a) 30 days (b) 15 days (c) 45 days 1
(d) 7 hrs. (e) None of these
(d) 24 days (e) None of these 2
5. Two pipes A and B can fill a cistern in 10 and 15 minutes 12. Two pipes A and B when working alone can fill a tank in 36
respectively. Both fill pipes are opened together, but at the min. and 45 min. respectively. A waste pipe C can empty the
end of 3 minutes, ‘B’ is turned off. How much time will the tank in 30 min. First A and B are opened. After 7 min., C is also
cistern take to fill ? opened. In how much time will the tank be full ?
(a) 6 min (b) 8 min (c) 10 min (a) 1/60 (b) 1/30 (c) 7/20
(d) 12 min (e) None of these (d) 13/20 (e) None of these
6. A is 30% more efficient than B. How much time will they, 13. A man can do a piece of work in 10 days but with the assistance
working together, take to complete a job which A alone could of his son, the work is done in 8 days. In how many days, his
have done in 23 days? son alone can do the same piece of work?
(a) 11 days (b) 13 days (a) 15 days (b) 22 days (c) 30 days
3 (d) 40 days (e) None of these
(c) 20 days (d) Cannot be determined
17
(e) None of these

1. a b c d e 2. a b c d e 3. a b c d e 4. a b c d e 5. a b c d e
RESPONSE 6. a b c d e 7. a b c d e 8. a b c d e 9. a b c d e 10. a b c d e

GRID 11. a b c d e 12. a b c d e 13. a b c d e


y
o
u
rs
m
a
h
86 SPEED TEST 42

b
o
o
3 22. A contractor undertook to do a piece of work in 9 days. He

b
14. A can do of a work in 12 days. In how many days can he employed certain number of laboures but 6 of them were

.w
4 absent from the very first day and the rest could finish the

o
1 work in only 15 days. Find the number of men originally

rd
finish of the work?
employed .

p
8

re
(a) 6 days (b) 5 days (c) 3 days (a) 15 (b) 6 (c) 13

s
(d) 2 days (e) None of these (d) 9 (e) None of these

s
.c
15. A can do a piece of work in 25 days and B in 20 days. They 1
23. After working for 8 days, Anil finds that only of the work

o
3

m
work together for 5 days and then A goes away. In how
has been done. He employs Rakesh who is 60% efficient as
many days will B finish the remaining work ?
Anil. How many more days will Anil take to complete the job?
(a) 17 days (b) 11 days (c) 10 days (a) 15 days (b) 12 days (c) 10 days
(d) 15 days (e) None of these (d) 8 days (e) None of these
16. A contractor undertakes to built a walls in 50 days. He 24. 2 men and 3 boys can do a piece of work in 10 days while
employs 50 peoples for the same. However after 25 days he 3 men and 2 boys can do the same work in 8 days. In how
finds that only 40% of the work is complete. How many more many days can 2 men and 1 boy to the work ?
man need to be employed to complete the work in time? 1 1 1
(a) 25 (b) 30 (c) 35 (a) 12days (b) 11 days (c) 15 days
2 2 2
(d) 20 (e) None of these
17. A and B can finish a work in 10 days while B and C can do it 1
(d) 13 days (e) None of these
in 18 days. A started the work, worked for 5 days, then B 2
worked for 10 days and the remaining work was finished by C 25. A can do a certain job in 12 days. B is 60% more efficient than
in 15 days. In how many days could C alone have finished A. How many days B alone take to do the same job?
the whole work ? 1 1
(a) 30 days (b) 15 days (c) 45 days (a) 7 (b) 11 (c) 8
2 2
(d) 24 days (e) None of these (d) 8 (e) None of these
18. 12 men complete a work in 18 days. Six days after they had 26. A man can do a piece of work in 5 days but with the help of
started working, 4 men joined them. How many days will all his son he can do it in 3 days. In what time can the son do it
of them take to complete the remaining work ? alone?
(a) 10 days (b) 12 days (c) 15 days 1 1
(d) 9 days (e) None of these (a) 6days (b) 7 days (c) 7
2 2
19. 10 men can complete a piece of work in 15 days and 15 women (d) 8 days (e) None of these
can complete the same work in 12 days. If all the 10 men and 27. x is 3 times as faster as y and is able to complete the work in
15 women work together, in how many days will the work get 40 days less than y. Then the time in which they can complete
completed? the work together?
1 2 1
(a) 6 (b) 6 (c) 6 (a) 15 days (b) 10 days (c) 7 days
3 3 2
2 (d) 5 days (e) None of these
(d) 7 (e) None of these 28. Pipe A can fill a tank in 5 hours, pipe B in 10 hours and pipe C
3 in 30 hours. If all the pipes are open, in how many hours will
20. A can do a piece of work in 10 days, while B alone can do it in
the tank be filled ?
15 days. They work together for 5 days and the rest of the (a) 2 (b) 2.5 (c) 3
work is done by C in 2 days. If they get ` 450 for the whole (d) 3.5 (e) None of these
work, how should they divide the money ? 29. Two taps can fill a tank in 12 and 18 minutes respectively.
(a) ` 225, ` 150, ` 75 (b) ` 250, ` 100, ` 100 Both are kept open for 2 minutes and the first is turned off. In
(c) ` 200, ` 150, ` 100 (d) ` 175, ` 175, ` 100 how many minutes more will the tank be filled ?
(e) None of these (a) 15 min. (b) 20 min. (c) 11 min.
21. 10 men and 15 women together can complete a work in 6 (d) 13 min. (e) None of these
days. It takes 100 days for one man alone to complete the 30. A cistern has three pipes, A, B and C. The pipes A and B can
same work. How many days will be required for one woman fill it in 4 and 5 hours respectively and C can empty it in 2
hours. If the pipes are opened in order at 1, 2 and 3 a.m.
alone to complete the same work?
respectively, when will the cistern be empty ?
(a) 90 (b) 125 (c) 145 (a) 3 p.m. (b) 4 p.m. (c) 5 p.m.
(d) 105 (e) None of these (d) 6 p.m. (e) None of these

14. a b c d e 15. a b c d e 16. a b c d e 17. a b c d e 18. a b c d e

RESPONSE 19. a b c d e 20. a b c d e 21. a b c d e 22. a b c d e 23. a b c d e

GRID 24. a b c d e 25. a b c d e 26. a b c d e 27. a b c d e 28. a b c d e

29. a b c d e 30. a b c d e
y
o
u
rs
m
a
h
b
43

o
o
Problem Based

b
.w
o
rd
on Ages

p
re
s
s
.c
o
m
Max. Marks : 30 No. of Qs. 30 Time : 20 min. Date : ........./......../................
1. The average age of a man and his son is 16 years. The ratio (a) 10 yrs (b) 30 yrs (c) 20 yrs
of their ages is 15 : 1 respectively. What is the son’s age? (d) 40 yrs (e) None of these
(a) 30 years (b) 32 years (c) 2 years 9. Ten yrs ago, A was half of B in age. If the ratio of their present
(d) 4 years (e) None of these ages is 3 : 4, what will be the total of their present ages?
2. The average age of a lady and her daughter is 28.5. The (a) 25 (b) 35 (c) 45
ratio of their ages is 14 : 5 respectively. What is the daugh- (d) 50 (e) None of these
ters age? 10. The sum of the ages of a mother and her daughter is 50 yrs.
(a) 12 years (b) 15 years
Also 5 yrs ago, the mother’s age was 7 times the age of the
(c) 18 years (d) Cannot be determined
daughter. What are the present ages of the mother and the
(e) None of these
daughter?
3. Present age of Sudha and Neeta are in the ratio of 6 : 7
respectively. Five years ago their ages were in the ratio of (a) 35, 5 (b) 40, 10 (c) 30, 20
5 : 6 respectively. What is Sudha’s present age? (d) 25, 15 (e) None of these
(a) 30 years (b) 35 years 11. The ratio of the father’s age to the son’s age is 4 : 1. The
(c) 40 years (d) Cannot be determined product of their ages is 196. What will be the ratio of their
(e) None of these ages after 5 years?
4. Average age of 36 children of the class is 15 years. 12 more (a) 7 : 3 (b) 14 : 9 (c) 11 : 4
children joined whose average age is 16 years. What is the (d) 17 : 3 (e) None of these
average age of all the 48 children together? 12. The ratio of Rita’s age to the age of her mother is 3 : 11. The
(a) 15.25 years (b) 15.5 years difference of their ages is 24 yrs. What will be the ratio of their
(c) 15.3 years (d) 15.4 years
ages after 3 yrs?
(e) None of these
5. Two years ago the ratio of the ages of Swati and Khyati (a) 1 : 2 (b) 1 : 3 (c) 3 : 7
was 5 : 7 respectively. Two years hence the ratio of their (d) 2 : 5 (e) None of these
ages will be 7 : 9 respectively. What is the present age of 1
Khyati? 13. A man’s age is 125% of what it was 10 years ago, but 83 %
3
(a) 16 years (b) 14.5 years
(c) 12 years (d) Cannot be determined of what it will be after 10 years. What is his present age?
(e) None of these (a) 30 yrs (b) 40 yrs (c) 50 yrs
6. The age of a man is 4 times that of his son. 5 yrs ago, the (d) 60 yrs (e) None of these
man was nine times as old as his son was at that time. What 14. The age of a man is three times the sum of the ages of his two
is the present age of the man? sons. Five years hence, his age will be double of the sum of
(a) 28 yrs (b) 32 yrs (c) 40 yrs the ages of his sons. The father's present age is
(d) 42 yrs (e) None of these (a) 40 years (b) 45 years (c) 50 years
7. After 5 yrs, the age of a father will be thrice the age of his (d) 55 years (e) 65 years
son, whereas five years ago, he was 7 times as old as his
15. The ratio between the present ages of P and Q is 3 : 4
son was. What are their present ages?
respectively. Four years hence Q will be 5 years older than P.
(a) 30 yrs (b) 40 yrs (c) 50 yrs
What is P’s present age?
(d) 60 yrs (e) None of these
8. 10 Yrs ago, Sita’s mother was 4 times older than her (a) 15 years (b) 20 years
daughter. After 10 yrs, the mother will be two times older (c) 25 years (d) Cannot be determined
than the daughter. What is the present age of Sita? (e) None of these

1. a b c d e 2. a b c d e 3. a b c d e 4. a b c d e 5. a b c d e
RESPONSE 6. a b c d e 7. a b c d e 8. a b c d e 9. a b c d e 10. a b c d e

GRID 11. a b c d e 12. a b c d e 13. a b c d e 14. a b c d e 15. a b c d e


y
o
u
rs
m
a
h
88 SPEED TEST 43

b
o
o
16. Present ages of Rama and Shyama are in the ratio of 4 : 5 24. The sum of the ages of 5 children born at the intervals of 3

b
.w
respectively. Five years hence the ratio of their ages be- years each is 50 years. What is the age of the youngest
comes 5 : 6 respectively. What is Rama’s present age?

o
child?

rd
(a) 25 years (b) 22 years (c) 20 years (a) 4 years (b) 8 years (c) 10 years

p
re
(d) 30 years (e) None of these
(d) 12 years (e) None of these

s
17. In a family, a couple has a son and daughter. The age of the

s
.c
father is three times that of his daughter and the age of the 25. If 6 years are subtracted from the present age of Gagan and

o
the remainder is divided by 18, then the present age of his

m
son is half of his mother. The wife is nine years younger to
her husband and the brother is seven years older than his grandson Anup is obtained. If Anup is 2 years younger to
sister. What is the age of the mother? Madan whose age is 5 years, then what is Gagan’s present
(a) 40 years (b) 45 years (c) 50 years age?
(d) 60 years (e) 65 years (a) 48 years (b) 60 years (c) 84 years
18. Ram’s present age is three times his son’s present age and (d) 96 years (e) 100 years
two-fifth of his father’s present age. The average of the
26. The ratio between the school ages of Neelam and Shaan is
present ages of all of them is 46 years. What is the difference
5 : 6 respectively. If the ratio between the one-third age of
between the Ram’s son’s present age and Ram’s father’s
Neelam and half of Shaan’s age is 5 : 9, then what is the
present age?
school age of Shaan?
(a) 68 years (b) 88 years
(c) 58 years (d) Cannot be determined (a) 25 years
(e) None of these (b) 30 years
19. Abhay’s age after six years will be three-seventh of his (c) Cannot be determined
father’s age. Ten years ago, the ratio of their ages was 1 : 5.
(d) 35 years
What is Abhay’s father’s age at present?
(a) 30 yrs. (b) 40 yrs. (c) 50 yrs. (e) None of these
(d) 60 yrs. (e) 70 years 27. A is two years older than his son. In two years, his age will
20. The present ages of three persons are in proportions be twice the age of his son. The present age of the son is:
4 : 7 : 9. Eight years ago, the sum of their ages was 56. Find (a) 7 (b) 8 (c) 9
their present ages (in years).
(d) 10 (e) 11
(a) 8, 20, 28 (b) 16, 28, 36 (c) 20, 35, 45
(d) 25, 30, 40 (e) None of these 28. Eighteen years ago, a father was three times as old as his
21. Tanya’s grandfather was 8 times older to her 16 years ago. son. Now the father is only twice as old as his son. Then the
He would be 3 times of her age 8 years from now. Eight years sum of the present ages of the son and the father is:
ago, what was the ratio of Tanya’s age to that of her grand- (a) 54 (b) 72 (c) 105
father? (d) 108 (e) 116
(a) 1 : 2 (b) 1 : 5 (c) 3 : 8 29. One year ago, Preeti was four times as old as her daughter
(d) 11 : 53 (e) None of these Sonal. Six years hence, Preeti’s age will exceed her daughter’s
22. Q is as much younger than R as he is older than T. If the sum age by 9 years. The ratio of the present ages of Preeti and
of the ages of R and T is 50 years, what is definitely the
her daughter is :
difference between R and Q’s age?
(a) 1 year (b) 2 years (c) 25 years (a) 9 : 2 (b) 11 : 3 (c) 12 : 5
(d) Data inadequate (e) None of these (d) 13 : 4 (e) 17 : 7
23. The sum of the ages of a father and his son is 45 years. Five 30. The present age of the father and the son are in the ratio of
years ago, the product of their ages is 34. Find the present 8:3. After 12 years the ratio of their ages will be 2:1. What is
age of father. the sum of the present age of the father and the son?
(a) 32 years (b) 36 years (c) 38 years (a) 66yrs (b) 70yrs (c) 74yrs
(d) 40 years (e) 39 years (d) 78yrs (e) 80yrs.

16. a b c d e 17. a b c d e 18. a b c d e 19. a b c d e 20. a b c d e


21. a b c d e 22. a b c d e 23. a b c d e 24. a b c d e 25. a b c d e
RESPONSE
26. a b c d e 27. a b c d e 28. a b c d e 29. a b c d e 30. a b c d e
GRID
y
o
u
rs
m
a
h
b
44

o
o
b
Permutation and

.w
o
rd
p
Combination

re
s
s
.c
o
m
Max. Marks : 30 No. of Qs. 30 Time : 20 min. Date : ........./......../................
1. In how many ways can six different rings be worn on four 8. In how many different ways can it be done so that the
fingers of one hand ? committee has at least one woman?
(a) 10 (b) 12 (c) 15 (a) 210 (b) 225 (c) 195
(d) 185 (e) None of these
(d) 16 (e) None of these
9. In how many different ways can it be done, so that the
2. In how many ways can 7 persons be seated at a round table
committee has at least 2 men?
if 2 particular persons must not sit next to each other ? (a) 210 (b) 225 (c) 195
(a) 5040 (b) 240 (c) 480 (d) 185 (e) None of these
(d) 720 (e) None of these 10. In how many ways can 5 boys be chosen from 6 boys and 4
3. In how many different ways can the letters of the word girls so as to include exactly one girl?
‘MATHEMATICS’ be arranged so that the vowels always (a) 252 (b) 210 (c) 126
come together ? (d) 90 (e) 60
(a) 10080 (b) 4989600 (c) 120960 11. In how many different ways can the letters of the word
(d) 12960 (e) None of these CORPORATION be arranged?
4. The number of ways in which four letters of the word (a) 3326400 (b) 1663200 (c) 831600
‘MATHEMATICS’ can be arranged is (d) 415800 (e) 207900
(a) 136 (b) 2454 (c) 1680 12. In how many different ways can the letters of the word
(d) 192 (e) None of these "COUNTRY" be arranged in such a way that the vowels
5. In how many different ways can hte letters of the word always come together?
(a) 720 (b) 1440 (c) 2880
‘PRETTY’ be arranged?
(d) 5040 (e) None of these
(a) 120 (b) 36 (c) 360
13. In how many different ways can the letters of the word
(d) 720 (e) None of these ‘PROBLEM’ be arranged ?
6. In how many different ways can be letters of the word (a) 5060 (b) 720 (c) 5040
‘CYCLE’ be arranged? (d) 980 (e) None of these
(a) 120 (b) 4240 (c) 30 14. How many different ways can the letters in the word ATTEND
(d) 80 (e) None of these
be arranged?
7. In how many different ways can the letters of the word
TRUST be arranged? (a) 60 (b) 120 (c) 240
(a) 240 (b) 120 (c) 80 (d) 80 (e) None of these
(d) 25 (e) None of these
15. In how many different ways can the letters of the word
DIRECTIONS (Qs.8 & 9): Answer these questions on the basis ‘OFFICES’ be arranged?
of the information given below :
From a group of 6 men and 4 women a Committee of 4 persons is (a) 2520 (b) 5040 (c) 1850
to be formed. (d) 1680 (e) None of these

1. a b c d e 2. a b c d e 3. a b c d e 4. a b c d e 5. a b c d e
RESPONSE 6. a b c d e 7. a b c d e 8. a b c d e 9. a b c d e 10. a b c d e

GRID 11. a b c d e 12. a b c d e 13. a b c d e 14. a b c d e 15. a b c d e


y
o
u
rs
m
a
h
90 SPEED TEST 44

b
o
o
16. In how many different ways can the letters of the word 22. Letters of the word DIRECTOR are arranged in such a way

b
.w
‘ARMOUR’ be arranged? that all the vowels come together. Find out the total number
of ways for making such arrangement.

o
(a) 720 (b) 300 (c) 640

rd
(a) 4320 (b) 2720

p
(d) 350 (e) None of these (c) 2160 (d) 1120

re
(e) None of these

s
17. In how many different ways can 4 boys and 3 girls be

s
23. How many three digit numbers can having only two

.c
arranged in a row such that all boys stand together and all

o
consecutive digits identical is

m
the girls stand together?
(a) 153 (b) 162
(a) 75 (b) 576 (c) 288 (c) 168 (d) 163
(d) 24 (e) None of these (e) None of these
18. In how many different ways can be letters of the word 24. In how many ways can the letters of the word 'PRAISE' be
SOFTWARE be arranged in such a way that the vowels arranged. So that vowels do not come together?
always come together? (a) 720 (b) 576
(c) 440 (d) 144
(a) 13440 (b) 1440
(e) None of these
(c) 360 (d) 120
25. The number of ways in which one or more balls can be
(e) None of these
selected out of 10 white, 9 green and 7 blue balls is
19. A bag contains 2 red, 3 green and 2 blue balls. 2 balls are to
(a) 892 (b) 881
be drawn randomly. What is the probability that the balls (c) 891 (d) 879
drawn contain no blue ball? (e) None of these
5 10 26. How many 3-digit numbers, each less than 600, can be formed
(a) (b) from {1, 2, 3, 4, 7, 9} if repetition of digits is allowed?
7 21
(a) 216 (b) 180
2 11 (c) 144 (d) 120
(c) (d)
7 21 (e) None of these
(e) None of these 27. If a secretary and a joint secretary are to be selected from a
20. In how many different ways can the letters of the word committee of 11 members, then in how many ways can they
BOOKLET be arranged such that B and T always come be selected ?
together? (a) 110 (b) 55
(a) 360 (b) 720 (c) 22 (d) 11
(c) 480 (d) 5040 (e) None of these
(e) None of these 28. On a railway route there are 20 stations. What is the number
21. In a box there are 8 red, 7 blue and 6 green balls. One ball is of different tickets required in order that it may be possible
picked up randomly. What is the probability that it is neither to travel from every station to every other station?
red nor green? (a) 40 (b) 380
7 2 (c) 400 (d) 420
(a) (b) (e) None of these
19 3
29. What is the number of five-digit numbers formed with 0, 1, 2,
3 9 3, 4 without any repetition of digits?
(c) (d)
4 21 (a) 24 (b) 48
(e) None of these (c) 96 (d) 120
(e) None of these
30. What is the number of three-digit odd numbers formed by
using the digits 1, 2, 3, 4, 5, 6 if repetition of digits is allowed?
(a) 60 (b) 108
(c) 120 (d) 216
(e) None of these

16. a b c d e 17. a b c d e 18. a b c d e 19. a b c d e 20. a b c d e


RESPONSE 21. a b c d e 22. a b c d e 23. a b c d e 24. a b c d e 25. a b c d e
GRID 26. a b c d e 27. a b c d e 28. a b c d e 29. a b c d e 30. a b c d e
y
o
u
rs
m
a
h
b
o
45

o
b
.w
o
Probability

rd
p
re
s
s
.c
o
m
Max. Marks : 25 No. of Qs. 25 Time : 20 min. Date : ........./......../................
1. Tickets numbered 1 to 20 are mixed up and then a ticket is 6. In a box, there are 8 red, 7 blue and 6 green balls. One ball is
drawn at random. What is the probability that the ticker picked up randomly. What is the probability that it is neither
drawn bears a number which is a multiple of 3? red nor green?
3 3 2 2 3 7
(a) (b) (c) (a) (b) (c)
10 20 5 3 4 19
1 8 9
(d) (e) None of these (d) (e)
2 21 21
7. Two dice are tossed. The probability that the total score is a
2. In a lottery, there are 10 prizes and 25 blanks. A lottery is
prime number is :
drawn at random. What is the probability of getting a prize?
1 5 1
1 2 2 (a) (b) (c)
(a) (b) (c) 6 12 2
10 5 7
7
5 (d) (e) None of these
(d) (e) None of these 9
7 8. In a simultaneous throw of two coins, the probability of getting
3. One card is drawn at random from a pack of 52 cards. What at least one head is
is the probability that the card drawn is a face card? 1 1 2
(a) (b) (c)
1 4 1 2 3 3
(a) (b) (c)
13 13 4 3
(d) (e) None of these
9 4
(d) (e) None of these 9. Three unbiased coins are tossed. What is the probability of
52
getting at least 2 heads?
4. One card is drawn from a pack of 52 cards. What is the
probability that the card drawn is either a red card or a 1 1 1
(a) (b) (c)
king? 4 2 3
1 6 7 1
(a) (b) (c) (d) (e) None of these
2 13 13 8
10. In a single throw of a die, what is the probability of getting a
27 number greater than 4?
(c) (e) None of these
52
1 1 2
5. A bag contains 6 black and 8 white balls. One ball is drawn (a) (b) (c)
at random. What is the probability that the ball drawn is 2 3 3
white? 1
(d) (e) None of these
3 4 1 4
(a) (b) (c) 11. In a simultaneous throw of two dice, what is the probability
4 7 8 of getting a total of 7?
3 1 1 2
(d) (e) None of these (a) (b) (c)
7 6 4 3
3
(d) (e) None of these
4

1. a b c d e 2. a b c d e 3. a b c d e 4. a b c d e 5. a b c d e
RESPONSE 6. a b c d e 7. a b c d e 8. a b c d e 9. a b c d e 10. a b c d e

GRID 11. a b c d e
y
o
u
rs
m
a
h
92 SPEED TEST 45

b
o
o
12. What is the probability of getting a sum 9 from two throws 19. What is the probability that a leap year selected at random

b
of a date? contains 53 Mondays?

.w
o
1 1 1 1 2 7

rd
(a) (b) (c) (a) (b) (c)
6 8 9 7 7 366

p
re
1 26

s
(d) (e) None of these (d) (e) None of these

s
12 183

.c
o
13. In a simultaneous throw of two dice, what is the probability 20. What is the probability of getting a sum of 7 with two dice?

m
of getting a doublet?
1 1 1
1 1 2 (a) (b) (c)
(a) (b) (c) 6 3 12
6 4 3
5
3 (d) (e) None of these
(d) (e) None of these 36
7 21. A bag contains 5 white, 7 red and 8 black balls. If 4 balls are
14. In a simultaneous throw of two dice, what is the probability drawn one by one with replacement, what is the probability
of getting a total of 10 or 11? that all are white ?
1 1 7 1 1 4
(a) (b) (c) (a) (b) (c)
4 6 12 256 16 20
5 4
(d) (e) None of these (d) (e) None of these
36 8
15. A coin is tossed three times. What is the probability of 22. Two dice are tossed. The probability that the total score is a
getting head and tail (HTH) or tail and head (THT) prime number is :
alternatively ? 1 5 1
(a) 1/4 (b) 1/5 (c) 1/6 (a) (b) (c)
6 12 2
(d) 1/8 (e) None of these
16. In a lottery, 16 tickets are sold and 4 prizes are awarded. If a 7
(d) (e) None of these
person buys 4 tickets,what is the probability of his winning 9
a prize? 23. A bag contains 3 white balls and 2 black balls. Another bag
contains 2 white balls and 4 black balls. A bag is taken and a
4 175 1 ball is picked at random from it. The probability that the ball
(a) 4 (b) (c)
16 256 4 will be white is:
81 7 7 5
(d) (e) None of these (a) (b) (c)
256 11 30 11
17. Two letters are drawn at random from the word ‘HOME’. 7
(d) (e) None of these
What is the probability that both the letters are vowels? 15
(a) 1/ 6 (b) 5/ 6 (c) 1/ 2 24. Out of 20 consecutive positive integers, two are chosen at
(d) 1/ 3 (e) None of these random. The probability that their sum is odd is
18. Three dice are thrown. What is the probability that the same (a) 19/20 (b) 10/19 (c) 1/20
number will appear on each of them? (d) 9/19 (e) None of these
1 1 1 25. An unbiased die is tossed. What is the probability of getting
(a) (b) (c) a multiple of 3.
6 18 24
1 1 2
1 (a) (b) (c)
(d) (e) None of these 2 3 5
36
1
(d) (e) None of these
6

12. a b c d e 13. a b c d e 14. a b c d e 15. a b c d e 16. a b c d e


RESPONSE 17. a b c d e 18. a b c d e 19. a b c d e 20. a b c d e 21. a b c d e
GRID 22. a b c d e 23. a b c d e 24. a b c d e 25. a b c d e
y
o
u
rs
m
a
h
b
46

o
o
b
.w
Area and Perimeter

o
rd
p
re
s
s
.c
o
m
Max. Marks : 35 No. of Qs. 35 Time : 25 min. Date : ........./......../................
1. The circumference of a circle is 44 metres. Find the area of 10. How many plants will be there in a circular bed whose outer
the circle. edge measure 30 cms, allowing 4 cm2 for each plant ?
(a) 154 m2 (b) 160 m2 (c) 175 m2 (a) 18 (b) 750 (c) 24
(d) 168 m 2 (e) None of these (d) 120 (e) None of these
2. The length and breadth of a rectangle are in the ratio 9 : 5. 11. A rectangular plot 15 m ×10 m, has a path of grass outside it.
If its area is 720 m2, find its perimeter. If the area of grassy pathway is 54 m 2, find the width of the
(a) 112 metre (b) 115 metre path.
(c) 110 metre (d) 118 metre (a) 4 m (b) 3 m (c) 2 m
(e) None of these (d) 1 m (e) None of these
3. A circle and a rectangle have the same perimeter. The sides 12. If the area of a circle decreases by 36%, then the radius of a
of the rectangle are 18 cm and 26 cm. What is the area of the circle decreases by
circle ? (a) 20% (b) 18% (c) 36%
(a) 88 cm2 (b) 154 cm2 (c) 1250 cm2 (d) 64% (e) None of these
(d) 616 cm2 (e) None of these 13. The floor of a rectangular room is 15 m long and 12 m wide.
4. The cost of carpeting a room 18m long with a carpet 75 cm The room is surrounded by a verandah of width 2 m on all
wide at ` 4.50 per metre is ` 810. The breadth of the room is: its sides. The area of the verandah is :
(a) 7 m (b) 7.5 m (c) 8 m (a) 124 m2 (b) 120 m2 (c) 108 m2
(d) 8.5 m (e) None of these (d) 58 m 2 (e) None of these
5. If the perimeter and diagonal of a rectangle are 14 and 5 cms 14. A circular grass lawn of 35 metres in radius has a path 7
respectively, find its area. metres wide running around it on the outside. Find the area of
(a) 12 cm2 (b) 16 cm2 (c) 20 cm2 path.
(d) 24 cm2 (e) None of these (a) 1694 m2 (b) 1700 m2 (c) 1598 m2
6. In an isoscele right angled triangle, the perimeter is 20 metre. (d) 1900 m2 (e) None of these
Find its area. 15. The radius of the wheel of a bus is 70 cms and the speed of
(a) 9,320 m2 (b) 8,750 m2 (c) 7,980 m2 the bus is 66 km/h, then the r.p.m. (revolutions per minutes) of
(d) 8,150 m 2 (e) None of these the wheel is
7. The diameter of a garden roller is 1.4 m and it is 2 m long. (a) 200 (b) 250 (c) 300
(d) 330 (e) None of these
æ 22 ö
How much area will it cover in 5 revolutions ? ç use p = ÷ 16. The altitude drawn to the base of an isosceles triangle is 8 cm
è 7 ø
and the perimeter is 32 cm. The area of the triangle is
(a) 40 m2 (b) 44 m2 (c) 48 m2 (a) 72 cm2 (b) 60 cm2 (c) 66 cm2
(d) 36 m2 (e) None of these (d) 67 cm 2 (e) None of these
8. The area of a triangle is 615 m2. If one of its sides is 123 17. The area of a square field is 576 km2. How long will it take for
metre, find the length of the perpendicular dropped on that a horse to run around at the speed of 12 km/h ?
side from opposite vertex. (a) 12 h (b) 10 h (c) 8 h
(a) 15 metres (b) 12 metres (d) 6 h (e) None of these
(c) 10 metres (d) 9 metres 18. The area of a rectangular field is 144 m2. If the length had
(e) None of these been 6 metres more, the area would have been 54 m2 more.
9. A horse is tethered to one corner of a rectangular grassy The original length of the field is
field 40 m by 24 m with a rope 14 m long. Over how much (a) 22 metres (b) 18 metres (c) 16 metres
area of the field can it graze? (d) 24 metres (e) None of these
(a) 154 cm2 (b) 308 m2 (c) 150 m2
(d) 145 cm 2 (e) None of these

1. a b c d e 2. a b c d e 3. a b c d e 4. a b c d e 5. a b c d e

RESPONSE 6. a b c d e 7. a b c d e 8. a b c d e 9. a b c d e 10. a b c d e

GRID 11. a b c d e 12. a b c d e 13. a b c d e 14. a b c d e 15. a b c d e


16. a b c d e 17. a b c d e 18. a b c d e
y
o
u
rs
m
a
h
94 SPEED TEST 46

b
o
o
19. The ratio between the length and the breadth of a rectangular 28. The perimeter of a square is double the perimeter of a

b
park is 3 : 2. If a man cycling along the boundary of the park rectangle. The area of the rectangle is 240 sq cm. What is the

.w
at the speed of 12km / hr completes one round in 8 minutes, area of the square ?

o
rd
then the area of the park (in sq. m) is: (a) 100 sq cm (b) 36 cq cm

p
(a) 15360 (b) 153600 (c) 30720 (c) 81 sq cm (d) Cannot be determined

re
(d) 307200 (e) None of these (e) None of these

s
s
20. A wire can be bent in the form of a circle of radius 56 cm. If it 29. If the perimeter of a square is equal to the radius of a circle

.c
is bent in the form of a square, then its area will be: whose area is 39424 sq. cm, what is the area of the square ?

o
(a) 3520 cm2 (b) 6400 cm2 (c) 7744 cm2

m
(a) 1225 sq. cm (b) 441 sq. cm
(d) 8800 cm 2 (e) None of these (c) 784 sq. cm (d) Cannot be determined
21. The length of a room is double its breadth. The cost of (e) None of these
colouring the ceiling at ` 25 per sq. m is ` 5,000 and the cost 30. Two poles, 15 m and 30 m high, stand upright in a playground.
of painting the four walls at ` 240 per sq. m is ` 64,800. Find If their feet be 36 m apart, find the distance between their
the height of the room. tops.
(a) 4.5 m (b) 4 m (c) 3.5 m (a) 35 cm (b) 39 cm (c) 45 cm
(d) 5 m (e) None of these (d) 50 cm (e) None of these
22. The surface area of a cube is 150 m2. The length of its 31. A semi-circle is constructed on each side of a square of
diagonal is length 2m. Find the area of the whole figure.
(a) (5 + 3p) m2 (b) (4 + 3p) m2 (c) (4 + p) m2
10
(a) 5 3m (b) 5 m (c) m (d) (4 + 2p) m2 (e) None of these
3 32. In a quadrilateral, the length of one of its diagonal is 23 cm
(d) 15 m (e) None of these and the perpendiculars drawn on this diagonal from other
23. The length and breadth of a playground are 36m and 21 m two vertices measure 17 cm and 7 cm respectively. Find the
respectively. Poles are required to be fixed all along the area of the quadrilateral.
boundary at a distance 3m apart. The number of poles (a) 250 cm2 (b) 276 cm2 (c) 300 cm2
required will be (d) 325 cm 2 (e) None of these
(a) 39 (b) 38 (c) 37 33. A circular wire of radius 42 cm is cut and bent in the form of
(d) 40 (e) None of these a rectangle whose sides are in the ratio of 6 : 5. Find the
24. What would be the area of a square whose diagonal measures smaller side of the rectangle.
28 cm? (a) 50 cm (b) 60 cm (c) 70 cm
(a) 288 sq cm (b) 514 sq cm (c) 428 sq cm (d) 80 cm (e) None of these
(d) 392 sq cm (e) None of these 34. The following figure contains three squares with areas of
25. The perimeter of a square is one-fourth the perimeter of a 100, 16 and 49 sq. units respectively laying side by side as
rectangle. If the perimeter of the square is 44 cm and the shown. By how much should the area of the middle square
length of the rectangle is 51 cm, what is the difference be reduced in order that the total length PQ of the resulting
between the breadth of the rectangle and the side of the three squares is 19?
square?
(a) 30 cm (b) 18 cm (c) 26 cm
(d) 32 cm (e) None of these
26. The area of a rectangle is equal to the area of a circle with 100
circumference equal to 220 metres. What is the length of the 49
rectangle if its breadth is 50 metres? 16
(a) 56 metres (b) 83 metres (c) 77 metres
P Q
(d) 69 metres (e) None of these
27. A man riding a bicycle, completes one lap of a circular field (a) 12 (b) 4 (c) 3
along its circumference at the speed of 79.2 km/hr in 2 minutes (d) 2 (e) 6
40 seconds. What is the area of the field? 35. A rectangle has perimeter of 50 metres. If its length is 13
(a) 985600 sq metre (b) 848500 sq metre metres more than its breadth, then its area is:
(c) 795600 sq metre (d) Cannot be determined (a) 124 m2 (b) 144 m2 (c) 114 m2
(d) 104 m 2 (e) 117m 2
(e) None of these

19. a b c d e 20. a b c d e 21. a b c d e 22. a b c d e 23. a b c d e

RESPONSE 24. a b c d e 25. a b c d e 26. a b c d e 27. a b c d e 28. a b c d e

GRID 29. a b c d e 30. a b c d e 31. a b c d e 32. a b c d e 33. a b c d e

34. a b c d e 35. a b c d e
y
o
u
rs
m
a
h
b
47

o
o
b
Volume and

.w
o
rd
p
Surface Area

re
s
s
.c
o
m
Max. Marks : 25 No. of Qs. 25 Time : 20 min. Date : ........./......../................
1. A cylindrical bucket of height 36 cm and radius 21 cm is 7. The length of a cold storage is double its breadth. Its height
filled with sand. The bucket is emptied on the ground and is 3 metres. The area of its four walls (including the doors) is
a conical heap of sand is formed, the height of the heap 108 m2. Find its volume.
being 12 cm. The radius of the heap at the base is : (a) 215 m3 (b) 216 m3
(a) 63 cm (b) 53 cm (c) 217 m 3 (d) 218 m3
(c) 56 cm (d) 66 cm (e) None of these
(e) None of these 8. A cube of 384 cm2 surface area is melt to make x number of
2. A metal cube of edge 12 cm is melted and formed into three small cubes each of 96 mm2 surface area. The value of x is
smaller cubes. If the edges of two smaller cubes are 6 cm (a) 80,000 (b) 8
and 8 cm, then find the edge of the third smaller cube. (c) 8,000 (d) 800
(a) 10 cm (b) 14 cm (e) None of these
(c) 12 cm (d) 16 cm 9. A conical vessel, whose internal radius is 12 cm and height 50
(e) None of these cm, is full of liquid. The contents are emptied into a cylindrical
3. A well 22.5 deep and of diameter 7 m has to be dug out. Find vessel with internal radius 10 cm. Find the height to which the
the cost of plastering its inner curved surface at ` 3 per sq. liquid rises in the cylindrical vessel.
metre. (a) 18 cm (b) 22 cm
(a) ` 1465 (b) ` 1485 (c) 24 cm (d) 20 cm
(c) ` 1475 (d) ` 1495 (e) None of these
(e) None of these 10. A hollow sphere of internal and external diameters 4 cm and 8
4. A copper sphere of radius 3 cm is beaten and drawn into a cm respectively is melted into a cone of base diamater 8 cm.
wire of diametre 0.2 cm. The length of the wire is The height of the cone is:
(a) 9 m (b) 12 m (a) 12 cm (b) 14 cm
(c) 18 m (d) 36 m (c) 15 cm (d) 18 cm
(e) None of these (e) None of these
5. If the volume of a sphere is divided by its surface area, the 11. A cone of height 9 cm with diameter of its base 18 cm is
result is 27 cms. The radius of the sphere is carved out from a wooden solid sphere of radius 9 cm. The
(a) 9 cms (b) 27 cms percentage of the wood wasted is:
(c) 81 cms (d) 243 cms (a) 25% (b) 30%
(e) None of these (c) 50% (d) 75%
6. A cistern 6 m long and 4 m wide contains water up to a (e) None of these
depth of 1 m 25 cm. The total area of the wet surface is: 12. A monument has 50 cylindrical pillars each of diameter 50 cm
(a) 49 m2 (b) 50 m2 and height 4 m. What will be the labour charges for getting
(c) 53.5 m 2 (d) 55 m2 these pillars cleaned at the rate of 50 paise per sq. m?
(e) None of these (use p = 3.14)
(a) ` 237 (b) ` 157
(c) ` 257 (d) ` 353
(e) None of these

1. a b c d e 2. a b c d e 3. a b c d e 4. a b c d e 5. a b c d e
RESPONSE 6. a b c d e 7. a b c d e 8. a b c d e 9. a b c d e 10. a b c d e
GRID 11. a c d e 12. a c d e
b b
y
o
u
rs
m
a
h
96 SPEED TEST 47

b
o
o
13. A conical vessel of base radius 2 cm and height 3 cm is filled 20. The curved surface of a right circular cone of height 15 cm

b
with kerosene. This liquid leaks through a hole in the bottom and base diameter 16 cm is :

.w
and collects in a cylindrical jar of radius 2 cm. The kerosene (a) 120p cm2 (b) 60 p cm2

o
rd
level in the jar is (c) 136 p cm 2 (d) 68p cm2

p
(a) p cm (b) 1.5 cm (e) None of these

re
(c) 1 cm (d) 3 cm 21. The weight of a solid cone having diameter 14 cm and vertical

s
s
(e) None of these height 51 cm is ....., if the material of solid cone weighs 10

.c
14. In a swimming pool measuring 90 m by 40 m, 150 men take a grams per cubic cm :

o
m
dip. If the average displacement of water by a man is 8 cubic (a) 16.18 kg (b) 17.25 kg
metres, what will be the rise in water level? (c) 26.18 kg (d) 71.40 kg
(a) 33.33 cm (b) 30 cm (e) None of these
(c) 20 cm (d) 25 cm 22. A hemispherical bowl is made of steel 0.5 cm thick. The inside
(e) None of these radius of bowl being 4 cm. The volume of the steel used in
15. Three cubes of a metal are of edges 3 cm, 4 cm and 5 cm. making the bowl is :
These are melted together and from the melted material, (a) 55.83 cm2 (b) 56.83 cm2
another cube is formed. The edge of this cube is : (c) 57.83 cm 2 (d) 58.83 cm2
(a) 8 cm (b) 10 cm (e) None of these
(c) 9 cm (d) 6 cm 23. Consider the volumes of the following :
(e) None of these A. A parallelopiped of length 5 cm, breadth 3 cm and height
16. A cylindrical bath tub of radius 12 cm contains water to a 4 cm.
depth of 20 cm. A spherical iron ball is dropped into the tub B. A cube having each side 4 cm.
and thus the level of water is raised by 6.75 cm. What is the C. A cylinder of radius 3 cm and length 3 cm.
radius of the ball? D. A sphere of radius 3 cm.
(a) 8 cm (b) 9 cm The volumes of these in the decreasing order is :
(c) 12 cm (d) 7 cm (a) A, B, C and D (b) A, C, B and D
(e) None of these (c) D, B, C and A (d) D, C, B and A
17. The volume of a cube is numerically equal to the sum of the (e) None of these
lengths of its edges. What is its total surface area in square 24. Two solid spheres of radii 1 cm and 2 cm were melted and
units ? combined to form a bigger sphere. The radius of the bigger
(a) 64 (b) 144 sphere is:
(c) 36 (d) 72 1 1
(e) None of these (a) (b)
23 33
18. The volume of a cube whose surface area is 726m2, is :
(a) 1300 m3 (b) 1331 m3 1 1
(c) (d)
(c) 1452 m 3 (d) 1542 m3 83 93
(e) None of these (e) None of these
19. If a cistern is 3 m long, 2 m wide and 1 m deep, its capacity in 25. The inner diameter of a circular building is 54 cm and the
litres is : base of the wall occupies a space of 352 cm 2. The thickness
(a) 6 (b) 600 of the wall is :
(c) 6,000 (d) 60,000 (a) 29 cm (b) 2 cm
(e) None of these (c) 4 cm (d) 58 cm
(e) None of these

13. a b c d e 14. a b c d e 15. a b c d e 16. a b c d e 17. a b c d e


RESPONSE 18. a b c d e 19. a b c d e 20. a b c d e 21. a b c d e 22. a b c d e
GRID 23. a c d e 24. a c d e 25. a c d e
b b b
y
o
u
rs
m
a
h
b
48

o
o
b
.w
Geometry

o
rd
p
re
s
s
.c
o
m
Max. Marks : 25 No. of Qs. 25 Time : 20 min. Date : ........./......../................
1. In triangle ABC, angle B is a right angle. If (AC) is 6 cm, and (a) 8 cm (b) 4 cm
D is the mid-point of side AC. The length of BD is (c) 5 cm (d) 7 cm
(e) None of these
A
5. If the angles of a triangle are in the ratio 5 : 3 : 2, then the
triangle could be :
D (a) obtuse (b) acute
(c) right (d) isosceles
(e) None of these
B C 6. In the figure, AB = 8, BC = 7 m, ÐABC = 1200. Find AC.

(a) 4 cm (b) A
6cm
(c) 3 cm (d) 3.5 cm
(e) None of these 8
1200
2. AB is diameter of the circle and the points C and D are on M B C
the circumference such that ÐCAD = 30°. What is the
(a) 11 (b) 12
measure of ÐACD ? (c) 13 (d) 14
C (e) None of these
D 7. The perimeters of two similar triangles ABC and PQR are
36 cm, and 24 cm, respectively. If PQ = 10 cm, then the
70° length of AB is :
A B
(a) 16 cm (b) 12 cm
(c) 14 cm (d) 15 cm
(e) None of these
(a) 40° (b) 50°
8. Two isosceles triangles have equal vertical angles and their
(c) 30° (d) 90°
areas are in the ratio 9 : 16. The ratio of their corresponding
(e) None of these
heights is :
3. The sum of the interior angles of a polygon is 1620°. The
(a) 3 : 4 (b) 4 : 3
number of sides of the polygon are :
(c) 2 : 1 (d) 1 : 2
(a) 9 (b) 11
(e) None of these
(c) 15 (d) 12
9. The circumcentre of a triangle is always the point of
(e) None of these
intersection of the :
4. A point P is 13 cm. from the centre of a circle. The length of
(a) medians
the tangent drawn from P to the circle is 12cm. Find the
(b) angle bisectors
radius of the circle.
(c) perpendicular bisectors of sides
T (d) perpendiculars dropped from the vertices on the
12c opposite sides of the triangle.
r m.
(e) None of these
O 13cm. P

RESPONSE 1. a b c d e 2. a b c d e 3. a b c d e 4. a b c d e 5. a b c d e

GRID 6. a b c d e 7. a b c d e 8. a b c d e 9. a b c d e
y
o
u
rs
m
a
h
98 SPEED TEST 48

b
o
o
10. In a triangle ABC, Ð A = x , ÐB = y and ÐC = y + 20 . 18. Number of degrees in two and half right angles is

b
__________ .

.w
If 4x – y = 10, then the triangle is :
(a) 245 (b) 225

o
(a) Right-angled (b) Obtuse-angled

rd
(c) 200 (d) 180
(c) Equilateral (d) Cannot be determined

p
(e) None of these

re
(e) None of these
19. Number of degrees in five and two-third of a right angle is

s
11. If the sides of a right triangle are x, x + 1 and x – 1, then its

s
(a) 510 (b) 490

.c
hypotenuse is :
(c) 486 (d) 480

o
(a) 5 (b) 4

m
(e) None of these
(c) 1 (d) 0
20. An angle is two-third of its complement and one-fourth of
(e) None of these
its supplement, then the angle is
12. Two circles touch each other internally. Their radii are 2 cm
(a) 46º (b) 56º
and 3 cm. The biggest chord of the outer circle which is
(c) 36º (d) 40º
outside the inner circle is of length
(e) None of these
(a) 2 2 cm (b) 3 2 cm 21. The measures of the four angles of a quadrilateral are in the
(c) 2 3 cm (d) ratio of 1 : 2 : 3 : 4.
4 2 cm
What is the measure of fourth angle?
(e) None of these
(a) 144º (b) 135º
1 (c) 125º (d) 150º
13. An angle is equal to rd of its supplement. Find its measure. (e) None of these
3
(a) 60° (b) 80° 22. Find the length of a chord which is at a distance of 3 cm from
(c) 90° (d) 45° the centre of a circle of radius 5 cm
(e) None of these (a) 8 cm (b) 10 cm
14. An angle measuring 270° is an example of (c) 12 cm (d) 6 cm
(a) acute angle (d) obtuse angle (e) None of these
(c) right angle (d) reflex angle 23. Two adjacent angles of a parallelogram are in the ratio 2 : 3.
(e) None of these Find the measures of all angles
15. Find the angle which is equal to its supplementary angle (a) 72o, 108o, 72o, 108o
(a) 45° (b) 180° (b) 72o, 108o, 70o, 110o
(c) 90° (d) 360° (c) 80o, 100o, 80o, 100o
(e) None of these (d) Cannot be determined
16. Three angles of a quadrilaterals are in the ratio (e) None of these
1 : 2 : 3. The sum of the least and the greatest of these angles 24. If the angle of triangle are in the ratio of 4 : 3 : 2, then the
is equal to 180°. All the angles of the quadrilateral will be : triangle
(a) 30°, 90°, 45°, 60° (a) is obtuse angled triangle
(b) 45°, 90°, 135°, 90° (b) has one angle greater than 80°
(c) 120°, 150°, 210°, 360° (c) is a right triangle
(d) Cannot be determined (d) is acute angled triangle
(e) None of these (e) None of these
17. The number of degrees in four and one-third right angles is 25. One of the angles of a parallelogram is 45°. What will be the
(a) 405 (b) 390 sum of the larger angle and twice the smaller angle of the
(c) 395 (d) 400 parallelogram ?
(e) None of these (a) 228° (b) 224°
(c) 225° (d) 222°
(e) None of these

10. a b c d e 11. a b c d e 12. a b c d e 13. a b c d e 14. a b c d e

RESPONSE 15. a b c d e 16. a b c d e 17. a b c d e 18. a b c d e 19. a b c d e

GRID 20. a b c d e 21. a b c d e 22. a b c d e 23. a b c d e 24. a b c d e

25. a b c d e
y
o
u
rs
m
a
h
b
49

o
o
b
.w
Number Series - I

o
rd
p
re
s
s
.c
o
m
Max. Marks : 30 No. of Qs. 30 Time : 20 min. Date : ........./......../................

DIRECTIONS (Qs. 1-5): What should come in place of question 9. 9 49 201 1009 ? 20209 80841
mark (? ) in the following number series ? (a) 4054 (b) 4049
1. 121 117 108 92 67 ? (c) 4050 (d) 4041
(a) 31 (b) 29
(c) 41 (d) 37 (e) None of these
(e) None of these 10. 31 35 44 60 85 ?
2. 50 26 14 ? 5 3.5 (a) 121 (b) 111
(a) 6 (b) 8
(c) 109 (d) 97
(c) 10 (d) 12
(e) None of these (e) None of these
3. 3 23 43 ? 83 103 DIRECTIONS (Q.11-15) : What will come in place of the question
(a) 33 (b) 53 mark (?) in the following number series?
(c) 63 (d) 73
11. 7 9 12 16 ?
(e) None of these
(a) 2 2 (b) 19
4. 748 737 715 682 638 ?
(c) 20 (d) 21
(a) 594 (b) 572
(e) None of these
(c) 581 (d) 563
12. 384 192 96 48 ?
(e) None of these
(a) 36 (b) 28
5. 1 9 25 49 81 ? 169
(a) 100 (b) 64 (c) 24 (d) 32
(c) 81 (d) 121 (e) None of these
(e) None of these 13. 5 6 14 45 ?
DIRECTIONS (Qs. 6-10) : What should come in place of question (a) 183 (b) 185
mark (?) in the following number series? (c) 138 (d) 139
6. 36 20 ? 8 6 5 (e) None of these
(a) 10 (b) 12 14. 8 9 13 22 ?
(c) 14 (d) 16 (a) 30 (b) 31
(e) None of these (c) 34 (d) 36
7. 668 656 632 584 ? 296 (e) None of these
(a) 392 (b) 438 15. 6 11 21 41 ?
(c) 488 (d) 536 (a) 81 (b) 61
(e) None of these (c) 71 (d) 91
8. 1 121 441 961 1681 ? (e) None of these
(a) 2701 (b) 2511
(c) 2611 (d) 2801
(e) None of these

1. a b c d e 2. a b c d e 3. a b c d e 4. a b c d e 5. a b c d e
RESPONSE 6. a b c d e 7. a b c d e 8. a b c d e 9. a b c d e 10. a b c d e
GRID 11. a c d e 12. a c d e 13. a c d e 14. a c d e 15. a c d e
b b b b b
y
o
u
rs
m
a
h
100 SPEED TEST 49

b
o
o
DIRECTIONS (Qs. 16-20): What will come in place of question 24. 4 2 3.5 7.5 26.25 118.125

b
.w
mark (?) in the following number series? (a) 118.125 (b) 26.25

o
16. 7 13 25 49 ? (c) 3.5 (d) 2

rd
(a) 99 (b) 97 (e) 7.5

p
re
(c) 89 (d) 87 25. 16 4 2 1.5 1.75 1.875

s
(e) None of these

s
(a) 1.875 (b) 1.75

.c
17. 5 6 10 19 ?

o
(c) 1.5 (d) 2
(a) 28 (b) 37

m
(c) 36 (d) 35 (e) 4
(e) None of these DIRECTIONS (Q. 26-30) : In the following number series only
18. 8 9 20 63 ? one number is wrong. Find out the wrong number.
(a) 256 (b) 252 26. 4 6 18 49 201 1011
(c) 246 (d) 242 (a) 1011 (b) 201
(e) None of these
(c) 18 (d) 49
19. 11 13 16 20 ?
(a) 24 (b) 26 (e) None of these
(c) 28 (d) 27 27. 48 72 108 162 243 366
(e) None of these (a) 72 (b) 108
20. 608 304 152 76 ?
(c) 162 (d) 243
(a) 39 (b) 36
(c) 38 (d) 37 (e) None of these
(e) None of these 28. 2 54 300 1220 3674 7350
DIRECTIONS (Qs. 21-25) % In the following number series, a wrong (a) 3674 (b) 1220
number is given. Find out that wrong number.
(c) 300 (d) 54
21. 2 11 38 197 1172 8227 65806
(e) None of these
(a) 11 (b) 38
29. 8 27 64 125 218 343
(c) 197 (d) 1172
(e) 8227 (a) 27 (b) 218

22. 16 19 21 30 46 71 107 (c) 125 (d) 343


(a) 19 (b) 21 (e) None of these
(c) 30 (d) 46 30. 19 68 102 129 145 154
(e) 71 (a) 154 (b) 129
23. 7 9 16 25 41 68 107 173 (c) 145 (d) 102
(a) 107 (b) 16
(e) None of these
(c) 41 (d) 68
(e) 25

16. a b c d e 17. a b c d e 18. a b c d e 19. a b c d e 20. a b c d e


RESPONSE 21. a b c d e 22. a b c d e 23. a b c d e 24. a b c d e 25. a b c d e
GRID 26. a c d e 27. a c d e 28. a c d e 29. a c d e 30. a c d e
b b b b b
y
o
u
rs
m
a
h
b
50

o
o
b
.w
Number Series - II

o
rd
p
re
s
s
.c
o
m
Max. Marks : 30 No. of Qs. 30 Time : 20 min. Date : ........./......../................

DIRECTIONS (Qs.1-5) : What should come in place of the 8. 101 103 99 105 97 ? 95
question mark (?) in the following number series ? (a) 93 (b) 104
(c) 108 (d) 107
1. 353 354 351 356 349 ?
(a) 348 (b) 358 (e) None of these
(c) 338 (d) 385 9. 3 219 344 408 ? 443 444
(e) 340
2. 1 5 13 29 ? 125 253 (a) 416 (b) 435
(a) 83 (b) 69 (c) 423 (d) 428
(c) 61 (d) 65
(e) None of these
(e) 81
3. 45 57 81 117 165 ? 10. 7 10 16 28 52 ? 196
(a) 235 (b) 215 (a) 100 (b) 90
(c) 205 (d) 245 (c) 160 (d) 150
(e) 225
(e) None of these
4. 17 18 26 53 117 ? 458
(a) 342 (b) 142 DIRECTIONS (Qs. 11-15) : What should come in place of the
(c) 257 (d) 262 question mark (?) in the following number series ?
(e) 242 11. 1 1 2 6 ? 120
1 1 3 1 1 3 (a) 24 (b) 60
5. 11 1 1 ?
4 2 4 4 2 4 (c) 100 (d) 30
(a) 2 (b) 4 (e) None of these
1 12. 7 8 16 43 ? 232
(c) 6 (d) 1 (a) 204 (b) 107
5
(c) 119 (d) 89
2 (e) None of these
(e) 1
3 13. 4 13 17 ? 30 39
DIRECTIONS (Qs.6-10) : What should come in place of the (a) 29 (b) 21
question mark (?) in the following number series? (c) 26 (d) 19
6. 4 19 49 94 154 ? (e) None of these
(a) 223 (b) 225 14. 982 977 952 827 822 ?
(c) 229 (d) 239
(a) 779 (b) 817
(e) None of these
(c) 789 (d) 697
1 1 1
7. 1 1 2 2 3 ? (e) None of these
2 2 2 15. 41472 5184 576 72 8 ?
1 1
(a) 3 (b) 2 (a) 0 (b) 9
2 3
(c) 1 (d) 8
1
(c) 4 (d) 3 (e) None of these
4
(e) None of these

1. a b c d e 2. a b c d e 3. a b c d e 4. a b c d e 5. a b c d e
RESPONSE 6. a b c d e 7. a b c d e 8. a b c d e 9. a b c d e 10. a b c d e
GRID 11. a c d e 12. a c d e 13. a c d e 14. a c d e 15. a c d e
b b b b b
y
o
u
rs
m
a
h
102 SPEED TEST 50

b
o
o
DIRECTIONS (Qs. 16-20) : What should come in place of the 23. 2 8 12 20 30 42 56

b
.w
question mark (?) in the following number series? (a) 8 (b) 42 (c) 30

o
16. 64 54 69 49 74 44 ? (d) 20 (e) 12

rd
24. 32 16 24 65 210 945 5197.5

p
(a) 89 (b) 69

re
(c) 59 (d) 99 (a) 945 (b) 16 (c) 24

s
s
(d) 210 (e) 65

.c
(e) None of these

o
25. 7 13 25 49 97 194 385
17. 4000 2008 1012 ? 265 140.5 78.25

m
(a) 13 (b) 49 (c) 97
(a) 506 (b) 514
(d) 194 (e) 25
(c) 520 (d) 512
(e) None of these DIRECTIONS (Qs. 26 - 30) :In each of these questions, a number
series is given. In each series, only one number is wrong number.
18. 5 5 15 75 ? 4725 51975
Find out the wrong number.
(a) 520 (b) 450
26. 16 19 21 30 46 71 107
(c) 525 (d) 300
(a) 19 (b) 21 (c) 30
(e) None of these
(d) 46 (e) 71
19. 52 26 26 39 78 ? 585
27. 7 9 16 25 41 68 107 173
(a) 195 (b) 156
(a) 107 (b) 16 (c) 41
(c) 234 (d) 117
(d) 68 (e) 25
(e) None of these 28. 32 16 24 65 210 945 5197.5
20. 29, 23, ?, 17, 13, 11, 7 (a) 945 (b) 16 (c) 24
(a) 19 (b) 21 (d) 210 (e) 65
(c) 23 (d) 27 29. 850 600 550 500 475 462.5 456.25
(e) None of these (a) 600 (b) 550 (c) 500
DIRECTIONS (Qs. 21 to 25): In each of these questions, a number (d) 4625 (e) None of these
series is given. In each series, only one number is wrong. Find out 30. 8 12 24 46 72 108 216
the wrong number. (a) 12 (b) 24 (c) 46
21. 3601 3602 1803 604 154 36 12 (d) 72 (e) None of these
(a) 3602 (b) 1803 (c) 604
(d) 154 (e) 36
22. 4 12 42 196 1005 6066 42511
(a) 12 (b) 42 (c) 1005
(d) 196 (e) 6066

16. a b c d e 17. a b c d e 18. a b c d e 19. a b c d e 20. a b c d e


RESPONSE 21. a b c d e 22. a b c d e 23. a b c d e 24. a b c d e 25. a b c d e
GRID 26. a c d e 27. a c d e 28. a c d e 29. a c d e 30. a c d e
b b b b b
y
o
u
rs
m
a
h
b
51

o
o
b
.w
Data Interpretation

o
rd
p
re
s
s
.c
o
m
Max. Marks : 20 No. of Qs. 20 Time : 20 min. Date : ........./......../................

DIRECTIONS (Q. 1-5): Study the following graph carefully to answer these questions.
Quantity of Various Items Sold and Price per kg

30 60

25 50

20 40
Price

Price in ` per kg
15 30 Quantity sold in quintals
10 20 Quantity

5 10

0 0
A B C D E F
Items

1. If the quantity sold of item D increased by 50% and the DIRECTIONS (Qs. 6 - 10): Study the following graph carefully
price reduced by 10%. What was the total value of the and answer the questions given below it.
quantity sold for item D ?
Number of Students Studying in Various Colleges from
(a) ` 675 (b) ` 6750 (c) `67550
Various Faculties (Number in Thousands)
(d) ` 67500 (e) None of these
2. Approximately, what is the average price per kg of items A, 80
B& C?
70 65
(a) ` 9.50 (b) ` 8 (c) ` 7.50 60 56
60
Number of Students

(d) ` 9 (e) ` 10.50 51.2 50


50 Arts
3. What is the ratio between the total values of quantity sold 40
44
for items E & F respectively ? 40 36.5 Science
33
30 30 Commerce
(a) 15 : 14 (b) 3 : 2 (c) 5 : 7 30 25
(d) 7 : 5 (e) None of these 20
4. Total value of the quantity sold for item C is what per cent 10
of the total value of the quantity sold for item E ? 0
H I J K
(a) 111 (b) 85 (c) 90 Colleges
(d) 87.5 (e) None of these
5. If the price as well as the quantity sold is increased by 20% 6. What is the difference between the total number of students
for item A, what is the total value of quantity sold for item studying in college H and those studying in college K ?
A? (a) 16100 (b) 15800 (c) 16300
(a) ` 48500 (b) ` 49000 (c) ` 42000 (d) 16700 (e) None of these
(d) ` 50400 (e) None of these 7. What is the total number of students studying in all the
colleges together ?
(a) 520900 (b) 520700 (c) 610200
(d) 510800 (e) None of these

1. a b c d e 2. a b c d e 3. a b c d e 4. a b c d e 5. a b c d e
RESPONSE 6. a b c d e 7. a b c d e
GRID
y
o
u
rs
m
a
h
104 SPEED TEST 51

b
o
o
8. What is the respective ratio of the students from the faculty (a) 634 (b) 654 (c) 658

b
of Science from colleges H and I together to the students (d) 778 (e) None of these

.w
from the same faculty from colleges J and K together ? 14. What is the approximate average number of teachers teaching

o
rd
(a) 43 : 45 (b) 41 : 43 (c) 45 : 43 Economics, History and Biology together ?

p
(d) 43 : 41 (e) None of these (a) 400 (b) 420 (c) 450

re
9. The number of students from the faculty of Science from (d) 480 (e) 470

s
s
college I are approximately what per cent of the total number 15. What is the respective ratio of the number of teachers who

.c
of students studying in that college ? teach Biology and the number of teachers who teach Physics?

o
m
(a) 34 (b) 37 (c) 29 (a) 6 : 7 (b) 4 : 7 (c) 3 : 5
(d) 31 (e) 39 (d) 4 : 5 (e) None of these
10. What is the average number of students from the faculty of
Commerce from all the colleges together ? DIRECTIONS (Q. 16-20) : Study the following table carefully to
(a) 36825 (b) 38655 (c) 35625 answer the questions that follow.
(d) 36585 (e) None of these Number of flights cancelled by five different
DIRECTIONS (Qs. 11-15) : Study the following Pie-chart carefully airlines in six different years
to answer these questions.
Percentagewise Distribution of Teachers who Teach six Different Airline P Q R S T
Subjects Year
Total Number of Teachers = 2000 2005 240 450 305 365 640
Percentage of Teachers 2006 420 600 470 446 258
2007 600 680 546 430 610
Biology English 2008 160 208 708 550 586
12% 7% 2009 140 640 656 250 654
2010 290 363 880 195 483

16. What was the difference between the highest number of


History flights cancelled by airline - Q and the lowest number of
Economics
27%
25% flights cancelled by airline-T out of all the six years ?
(a) 446 (b) 456 (c) 432
(d) 442 (e) None of these
17. What was the approximate percentage increase in number
of flights cancelled by airline-S in the year 2008 as compared
Physics Mathematics to previous year ?
15% 14% (a) 127 (b) 27 (c) 150
(d) 45 (e) 117
11. If five-seventh of the teachers who teach Mathematics are 18. What was the average number of flights cancelled by the
female, then number of male Mathematics teachers is airlines P, R, S and T in the year 2008 ?
approximately what percentage of the total number of (a) 551.5 (b) 501 (c) 405
teachers who teach English ? (d) 442.4 (e) None of these
(a) 57 (b) 42 (c) 63 19. In 2010, 40% flights are cancelled by airline-R due to bad
(d) 69 (e) 51 weather and technical fault. How many flights are cancelled
12. What is the difference between the total number of teachers by airline-R due to technical fault ?
who teach English and History together and the total number
(a) 528 (b) 568 (c) 468
of teachers who teach Mathematics and Biology together ?
(d) 548 (e) None of these
(a) 146 (b) 156 (c) 180
20. What is the approximate percentage of cancelled flights by
(d) 160 (e) None of these
13. If the percentage of Biology teachers is increased by 40 per airline’s-P and R in 2007 compared to cancelled flights by
cent and percentage of History teachers decreased by 20 airline-S in 2005 ?
per cent then what will be the total number of Biology and (a) 356 (b) 280 (c) 265
History teachers together ? (d) 340 (e) 314

8. a b c d e 9. a b c d e 10. a b c d e 11. a b c d e 12. a b c d e


RESPONSE 13. a b c d e 14. a b c d e 15. a b c d e 16. a b c d e 17. a b c d e
GRID 18. a b c d e 19. a b c d e 20. a b c d e
y
o
u
rs
m
a
h
b
52

o
o
b
Section Test :

.w
o
rd
p
Quantitative Aptitude

re
s
s
.c
o
m
Max. Marks : 40 No. of Qs. 40 Time : 25 min. Date : ........./......../................
DIRECTIONS (Q.1-10) : What will come in place of question 11. Cost of 12 belts and 30 wallets is Rs 8940. What is the cost of
mark (?) in the following questions ? 4 belts and 10 wallets?
(a) Rs 2890 (b) Rs 2980 (c) Rs 2780
1. 48% of 525 + ? % of 350 = 399 (d) Rs 2870 (e) None of these
(a) 42 (b) 46 (c) 28 12. Ghanshyam purchased an article for Rs 1850. At what price
(d) 26 (e) None of these should he sell it so that 30% profit is earned?
3 4 5 (a) Rs 2450 (b) Rs 2245 (c) Rs 2405
2. of of of 490 = ? (d) Rs 2425 (e) None of the above
7 5 8
(a) 115 (b) 105 (c) 108 13. What is the compound interest accrued on an amount of Rs
(d) 116 (e) None of these 8500 in two years @ interest 10% per annum?
2
(a) Rs 1875 (b) Rs 1885 (c) Rs 1775
3. ? + 17 = 335 (d) Rs 1765 (e) None of these
(a) 46 (b) 42 (c) 1764 14. A train running at the speed of 60 kmph crosses a 200 m long
(d) 2116 (e) None of these platform in 27 s. What is the length of the train ?
4. 125% of 560 + 22% of 450 = ? (a) 250 m (b) 200 m (c) 240 m
(a) 799 (b) 700 (c) 782 (d) 450 m (e) None of these
(d) 749 (e) None of these 15. 10 men can complete a piece of work in 8 days. In how many
days can 16 men complete that work?
28 ´ 5 - 15 ´ 6
5. =? (a) 4 days (b) 5 days (c) 6 days
7 2 + 256 + (13)2 (d) 3 days (e) None of these
16. Find the average of following set of numbers.
27 22 25
(a) (b) (c) 76, 48, 84, 66, 70, 64
115 117 117 (a) 72 (b) 66 (c) 68
22 (d) 64 (e) None of these
(d) (e) None of these 17. If the numerator of a certain fractions increased by 100% and
115
the denominator is increased by 200%; the new fraction thus
6. 18.76 + 222.24 + 3242.15 = ?
(a) 3384.15 (b) 3483.15 (c) 3283.25 4
formed is . What is the original fraction?
(d) 3383.25 (e) None of these 21
7. 784 ÷ 16 ÷ 7 = ?
2 3 2
(a) 49 (b) 14 (c) 21 (a) (b) (c)
(d) 7 (e) None of these 7 7 5
3 5 4
8. of 455 + of 456 = ? (d) (e) None of these
2 8 7
(a) 448 (b) 476 (c) 480 18. In how many different ways can the letters of the word
(d) 464 (e) None of these 'SIMPLE' be arranged?
9. 1.05% of 2500 + 2.5% of 440 = ? (a) 520 (b) 120 (c) 5040
(a) 37.50 (b) 37.25 (c) 370.25 (d) 270 (e) None of these
(d) 372.50 (e) None of these 19. The ratio of the ages of A and B seven years ago was 3 : 4
10. 4900 ÷ 28 × 444 ÷ 12 = ? respectively. The ratio of their ages nine years from now
(a) 6575 (b) 6475 (c) 6455 will be 7 : 8 respectively. What is B’s age at present ?
(d) 6745 (e) None of these (a) 16 years (b) 19 years (c) 28 years
(d) 23 years (e) None of these

1. a b c d e 2. a b c d e 3. a b c d e 4. a b c d e 5. a b c d e
RESPONSE 6. a b c d e 7. a b c d e 8. a b c d e 9. a b c d e 10. a b c d e

GRID 11. a b c d e 12. a b c d e 13. a b c d e 14. a b c d e 15. a b c d e


16. a b c d e 17. a b c d e 18. a b c d e 19. a b c d e
y
o
u
rs
m
a
h
106 SPEED TEST 52

b
o
o
20. The sum of three consecutive odd numbers is 1383. What 31. 4 women and 12 children together take four days to complete

b
is the largest number ? a piece of work. How many days will four children alone take

.w
(a) 463 (b) 49 (c) 457 to complete the piece of work if two women alone can

o
rd
(d) 461 (e) None of these complete the piece of work in 16 days?
(a) 32 (b) 24 (c) 16

p
DIRECTIONS (Q. 21-25) : What should come in place of ques-

re
tion mark (?) in the following number series? (d) 12 (e) None of these

s
32. The average of four consecutive odd number A, B, C and D

s
21. 8 52 ? 1287 4504.5 11261.25 16891.875

.c
respectively is 40. What is the product of B and D?

o
(a) 462 (b) 286 (c) 194 (a) 1599 (b) 1591 (c) 1763

m
(d) 328 (e) None of these (d) 1677 (e) None of these
22. 3 42 504 ? 40320 241920 967680 33. Anu walks 2.31 km in three weeks by walking an equal
(a) 6048 (b) 5544 (c) 4536 distance each day. How many metres does she walk each
(d) 5040 (e) None of these day?
23 403 400 394 382 358 310 ? (a) 110 m (b) 90 m (c) 140 m
(a) 244 (b) 210 (c) 214 (d) 120 m (e) None of these
(d) 256 (e) None of these 34. A man riding a bicycle completes one lap of a square field
24. 7 8 4 13 –3 22 ? along its perimeter at the speed of 43.2 km/hr in 1 minute 20
(a) –7 (b) –10 (c) –12 seconds. What is the area of the field?
(d) –14 (e) None of these (a) 52900 sq m (b) 57600 sq m (c) 48400 sq m
25. 250000 62500 12500 3125 625 ? 31.25 (d) Can’t be determined
(a) 156.25 (b) 172.25 (c) 125 (e) None of these
(d) 150 (e) None of these 35. On Teacher’s Day, 4800 sweets were to be equally distributed
1 7 3 5 6 among a certain number of children. But on that particular
26. Out of the fractions , , , and , what is the differ- day 100 children were absent. Hence, each child got four
2 8 4 6 7 sweets extra. How many children were originally supposed
ence between the largest and the smallest fraction? to be there?
7 3 4 (a) 300 (b) 400 (c) 540
(a) (b) (c) (d) 500 (e) Can’t be determined.
13 8 7
36. The ratio of the monthly oncomes of Sneha, Tina and Akruti
1 is 95:110:116. If Sneha’s annual income is `3,42,000, what is
(d) (e) None of these
6 Akruit’s annual income?
27. What will come in place of both questions marks(?) in the (a) `3,96,900 (b) `5,63,500 (c) `4,17,600
following equation? (d) `3,88,000 (e) None of these
37. A truck covers a distance of 256 km at the speed of 32 km/hr.
(?)0.6 26 What is the average speed of a car which travels a distance
=
104 (?)1.4 of 160 km more than the truck in the same time?
(a) 46 kmh–1 (b) 52 kmh–1 (c) 49 kmh–1
(a) 58 (b) –48 (c) –56 (d) 64 kmh –1 (e) None of these
(d) 42 (e) –52 38. In an examination, the maximum aggregate marks is 1020. In
28. The perimeter of a square is thrice the perimeter of a rectange. order to pass the exam a student is required to obtain 663
If the perimeter of the square is 84 cm and the length of the marks out of the aggregate marks. Shreya obtained 612
rectangel is 8 cm, what is the difference between the breadth marks. By what per cent did Shreya fail the exam?
of the rectangle and the sidce of the square? (a) 5% (b) 8% (c) 7%
(a) 15 cm (b) 19 cm (c) 10 cm (d) Can’t be determined
(d) 8 cm (e) None of these (e) None of these
29. The area of a circle is equal to the area of a rectangel with 39. The average height of 21 girls was recorded as 148 cm. When
perimeter equal to 42 m and breadth equal to 8.5 m. What is the teacher’s height was included, the average of their
the area of the circle? heights increased by 1 cm. What was the height of the
(a) 116.25 sq m (b) 104.25 sq m (c) 146.25 sq m teacher?
(d) 128.25 sq m (e) None of these (a) 156 cm (b) 168 cm (c) 170 cm
30. The product of 5% of a positive number and 3% of the same (d) 162 cm (e) None of these
number is 504.6 What is half of that number? 40. What would be the area of a circle whose diameter is 35 cm?
(a) 290 (b) 340 (c) 680 (a) 962.5 sq cm (b) 875.5 sq cm (c) 981.5 sq cm
(d) 580 (e) None of these (d) 886.5 sq cm (e) None of these
20. a b c d e 21. a b c d e 22. a b c d e 23. a b c d e 24. a b c d e
25. a b c d e 26. a b c d e 27. a b c d e 28. a b c d e 29. a b c d e
RESPONSE 30. a b c d e 31. a b c d e 32. a b c d e 33. a b c d e 34. a b c d e
GRID 35. a b c d e 36. a b c d e 37. a b c d e 38. a b c d e 39. a b c d e
40. a b c d e
y
o
u
rs
m
a
h
b
53

o
o
b
Reading

.w
o
rd
p
Comprehension - I

re
s
s
.c
o
m
Max. Marks : 30 No. of Qs. 30 Time : 20 min. Date : ........./......../................
Gandhi. He has rightly been called the Father of the Nation because
DIRECTIONS : Read the following passages carefully and answer
it was he who awakened in the people of this country a sense of
the questions given below it. Certain words are printed in bold to
national consciousness and instilled in them a high sense of
help you to locate them while answering some of the questions.
patriotism without which it is not possible to build a country into
Passage 1 nationhood. By the time the Constitution of India came to be enacted,
We find that today the unity and integrity of the nation is insurgent India, breaking a new path of non-violent revolution and
threatened by the divisive forces of regionalism, linguism and fighting to free itself from the shackles of foreign domination, had
communal loyalties which are gaining ascendancy in national emerged into nationhood and “the people of India” were inspired by
life and seeking to tear apart and destroy national integrity. We a new enthusiasm, a high and noble spirit of sacrifice and above all,
tend to forget that India is one nation and we are all Indians first a strong sense of nationalism and in the Constitution which they
and Indians last. It is time we remind ourselves what the great framed. They set about the task of a strong nation based on certain
visionary and builder of modern India Jawaharlal Nehru said, cherished values for which they had fought.
“Who dies if India lives, who lives if India dies?” We must realise, 1. The author has quoted Jawaharlal Nehru to emphasise the
and this is unfortunately what many in public life tend to overlook, point that
sometimes out of ignorance of the forces of history and (a) national interest must enjoy supreme importance
sometimes deliberately with a view to promoting their self- (b) India is going to survive even if the world is under the
interest, that national interest must inevitably and forever prevail spell of destruction
over any other considerations proceeding from regional, linguistic (c) the world will be destroyed if India is on the threshold of
or communal attachments. The history of India over the past destruction
centuries bears witness to the fact that India was at no time a (d) the survival of the world depends only upon the well
single political unit. Even during the reign of the Maurya dynasty, being of India
though a large part of the country was under the sovereignty of (e) None of these
the Mauryan kings, there were considerable portions of the 2. What, according to the author, is the impact of the divisive
territory which were under the rule of independent kingdoms. So forces on our nation?
also during the Mughal rule which extended over large parts of (a) They promote a sense of regional pride.
the territory of India, there were independent rulers who enjoyed (b) They help people to form linguistic groups.
political sovereignty over the territories of their respective (c) They separate groups of people and create enmity among
kingdoms. It is an interesting fact of history that India was forged them.
into a nation, neither on account of a common language nor on
(d) They encourage among people the sense of loyalty to
account of the continued existence of a single political regime
their community.
over its territories but on account of a common culture evolved
(e) They remind us of our national pride.
over the centuries. It is cultural unity—something more
3. “Communal loyalties” have been considered by the author as
fundamental and enduring than any other bond which may unite
the people of a country together which has welded this country (a) a good quality to be cherished
into a nation. But until the advent of the British rule, it was not (b) of no consequence to the nation
constituted into a single political unit. There were, throughout (c) a very important aspect for nation-building
the period of history for which we have fairly authenticated (d) a threat to the solidarity of the nation
accounts, various kingdoms and principalities which were (e) None of these
occasionally engaged in conflict with one another. During the 4. Which of the following was instrumental in holding the
British rule, India became a compact political unit having one different people of India together?
single political regime over its entire territories and this led to the (a) A common national language
evolution of the concept of a nation. This concept of one nation (b) A common cultural heritage
took firm roots in the minds and hearts of the people during the (c) The endurance level of the people
struggle for independence under the leadership of Mahatma (d) Fundamentalist bent of mind of the people
(e) None of these

RESPONSE 1. a b c d e 2. a b c d e 3. a b c d e 4. a b c d e
GRID
y
o
u
rs
m
a
h
108 SPEED TEST 53

b
o
o
5. The passage appears to have been written with the purpose of DIRECTIONS (Qs. 11-13): Choose the word/group of words which

b
(a) giving a piece of advice to politicians of free India

.w
is most nearly the SAME in meaning as the word given in bold as
(b) assessing the patriotic values and sacrifices made by used in the passage.

o
rd
people for India’s freedom 11. awakened

p
(c) justifying the teaching of Mahatma Gandhi and its (a) moved (b) segregated

re
impact on the people (c) extracted (d) kindled

s
s
(d) giving a historical account of how India evolved as a (e) supported

.c
o
nation 12. cherished

m
(e) None of these (a) maintained carefully
6. History shows that India, which was not a political unit (b) available abundantly
earlier, became so (c) managed tactfully
(a) during the reign of Maurya dynasty (d) accepted happily
(b) during the Mughal rule (e) protected lovingly
(c) after one-national-language policy was adopted 13. authenticated
(d) during the regime of independent rulers (a) established (b) documented
(e) during the British rule (c) hea rsay (d) audited
7. Why do people tend to overlook the paramount importance (e) maintained
of national interest? DIRECTIONS (Qs. 14 & 15) : Choose the word which is most
(A) Because they are unaware of the imperative need of OPPOSITE in meaning of the word given in bold as used in the
the day passage.
(B) Because they give undue importance to their selfish 14. considerable
motives (a) inconsiderate (b) uncountable
(C) Because historical events force them to do so (c) unfathomable (d) irresolute
(e) negligible
(a) Only A (b) Only B
15. deliberately
(c) Only C (d) A and B only
(a) reluctantly (b) unintentionally
(e) B and C only
(c) unauthorisedly (d) wrongly
8. The “people of India”, as highlighted by the author in the last
(d) notoriously
sentence of the passage, refer to
(a) the people of one unified nation Passage - 2
(b) the subjects of several independent rulers In a reversal of the norm elsewhere, in India policymakers and
(c) the patriots who sacrificed themselves in the freedom economists have become optimists while bosses do the worrying.
struggle The country’s Central Bank has predicted that the country’s
economy is likely to grow at a double digit rate during the next 20-
(d) the people who were instrumental in writing the
30 years. India has the capability with its vast labour and lauded
Constitution entrepreneurial spirit. But the private sector which is supposed to
(e) None of these do the heavy lifting that turns India from the world’s tenth largest
9. India’s insurgence was for economy to its third largest by 2030 has become fed up. Business
(a) breaking the path of non-violence people often carp about India’s problems but their irritation this
(b) having one common national language time has a nervous edge. In the first quarter of 2011, GDP grew at
(c) insisting on a unique cultural identity an annual rate of 7.8 percent; in 2005-07 it managed 9-10 percent.
(d) several independent sovereign rulers The economy may be slowing naturally as the low interest rates
(e) None of these and public spending that got India through the global crisis are
10. Transformation of our country into nationhood was possible belatedly withdrawn. At the same time the surge in inflation caused
by exorbitant food prices has spread more widely, casting doubt
because of
over whether India can grow at 8-10 percent in the medium term
(A) People’s spontaneously referring to Mahatma Gandhi without overheating.
as the Father of the Nation In India, as in many fast growing nations, the confidence to invest
(B) People’s sense of national consciousness depends on the conviction that the long term trajectory is intact
(C) Generation of a high sense of dedication to the nation and it is that which is in doubt. Big Indian firms too sometimes
among the people seem happier to invest abroad than at home, in deals that are
(a) A and B only (b) A and C only often hailed as symbols of the country’s growing clout but
(c) B and C only (d) All the three sometimes speak to its weaknesses – purchases of natural
(e) None of these resources that India has in abundance but struggles to get out of
the ground. In fact a further dip in investment could be self

5. a b c d e 6. a b c d e 7. a b c d e 8. a b c d e 9. a b c d e
RESPONSE 10. a b c d e 11. a b c d e 12. a b c d e 13. a b c d e 14. a b c d e
GRID 15. a b c d e
y
o
u
rs
m
a
h
SPEED TEST 53 109

b
o
o
fulfilling: if fewer roads, ports and factories are built, this will hurt 19. What is the author’s main objective in writing the passage?

b
both short term growth figures and reduce the economy’s long (a) Showcasing the potential of India’s growth potential to

.w
term capacity. entice foreign investors .

o
rd
There is a view that because a fair amount of growth is assured (b) Exhorting India to implement measures to live up to its

p
the government need not try very hard. The liberalization reforms potential.

re
that began in 1991 freed markets for products and gave rise to (c) Recommending India’s model of development to other

s
s
vibrant competition, at the same time what economists call factor developing countries

.c
markets, those for basic inputs like land, power, labour etc remain (d) Berating the private sector for not bidding for

o
m
unreformed and largely under state control, which creates infrastructure development projects.
difficulties. Clearances today can take three to four years and (e) Criticising the measures taken by India during the global
many employers are keen to replace workers with machines economic crisis.
20. What impact has the GDP growth of 7.8 percent had?
despite an abundance of labor force. This can be attributed to
(1) Indian Industry is anxious about India’s economic
labor laws which are inimical to employee creation and an
growth.
education system that means finding quality manpower a major
(2) India has achieved status as the world’s third largest
problem. In fact the Planning Commission concluded that even economy at present.
achieving 9 percent growth will need marked policy action in (3) Foreign investment in India has drastically increased.
unreformed sectors. Twenty years age it was said that yardstick (a) Only (1)
against which India should be measured was its potential and it (b) All (1), (2) and (3)
is clear that there remains much to do. (c) Only (1) and (3)
16. Which of the following can be said about the Indian (d) Only (1) and (2)
economy at present? (e) None of these
(a) It can comfortably achieve double digit growth rare at 21. Which of the following is most similar in meaning to the
present. word CLOUT given in bold as used in the passage?
(b) High food prices have led to overheating of the (a) Strike
economy. (b) Standing
(c) Citizens are affluent owing to laxity in regulation. (c) Force
(d) Private sector confidence in India’s growth potential (d) Launch
is high. (e) Achieve
22. Which of the following is most opposite in meaning to the
(e) Unreformed sectors are a drag on economic growth.
word MARKED given in bold as used in the passage?
17. Why are employers reluctant to hire Indian labour force?
(a) Decreased
(1) India’s labour force is overqualified for the (b) Ignored
employment opportunities available. (c) Clear
(2) High attrition rate among employees stemming from (d) Assessed
their entrepreneurial spirit. (e) Imperceptible
(3) Labour laws are not conducive to generating 23. What measures do experts suggest be taken to ensure targeted
emploment. economic growth?
(a) Only (3) (a) Loweing of interest rates to help industries hit by
(b) All (1), (2) and (3) recession.
(c) Only (1) and (3) (b) Prolonged financial support for basic input industries.
(d) Only (1) and (2) (c) Incentives to Indian companies to invest in infrastucture.
(e) None of these (d) Formulation of policies and their implementation in factor
18. What is the state of India’s basic input sectors at present? markets
(a) These sectors attract Foreign Direct Investment (e) Stringent implementation of licensing system.
because of their vast potential. Passage - 3
(b) These sectors are lagging as projects are usually In many countries, a combustible mixture of authoritarianism,
awarded to foreign companies. unemployment and youth has given rise to disaffection with
(c) These sectors are stagnating and badly in need of strongmen rulers which has in turn spill over into uprising. Young
reforms. people in these countries are far better educated than their parents
(d) These sectors are well regulated as these are governed were. In 1990 the average Egyptian had 4.4 years of schooling; by
by the State. 2010 the figure had risen to 7.1 years. Could it be that education, by
(e) None of these making people less willing to put up with restrictions on freedom

RESPONSE 16. a b c d e 17. a b c d e 18. a b c d e 19. a b c d e 20. a b c d e

GRID 21. a b c d e 22. a b c d e 23. a b c d e


y
o
u
rs
m
a
h
110 SPEED TEST 53

b
o
o
and more willing to question authority, promotes democratization. 25. Which of the following is most similar in meaning to the

b
Ideas about the links between education, Income and democracy word PROMOTES given in bold as used in the passage?

.w
are at the heart of what social scientists have long studied. Since (a) Up grades (b) Prefers

o
(c) Recommends (d) Advocates

rd
then plenty of economists and political scientists have looked for
(e) Publicizes

p
statistical evidence of a causal link between education and

re
democratization. Many have pointed to the strong correlation that 26. What conclusion can be drawn from the statistics cited about

s
Egypt’s education system?

s
exists between levels of education and measures like the pluralism

.c
of party politics and the existence of civil liberties. The patterns (a) Job prospects have been on the rise in Egypt in recent

o
times.

m
are similar when income and democracy are considered. There are
outliers, of course – until recently, many Arab countries managed to (b) Authoritarian leaders have played a vital role in
reforming Egypt’s education system.
combine energy-based wealth and decent education with
(c) Egypt has one of the youngest and best educated
undemocratic political systems. But some deduce from the overall
demographies in the world.
picture that as China and other authoritarian states get more educated (d) Egypt is likely to be successful vibrant democracy.
and richer, their people will agitate for greater political freedom, (e) There has been a rise in education levels in Egypt in
culminating in a shift to a more democratic form of government. recent times.
This apparently reasonable intuition is shakier than it seems. Critics 27. In the context of the passage which of the following
of the hypothesis point out that correlation is hardly causation. characterize (s) democracies?
The general trend over the past half century may have been towards (1) Active participation of majority of educated citizens in
rising living standards, a wider spread of basic education and more electoral process.
democracy, but it is entirely possible that this is being by another (2) Fast paced economic growth and accountability of those
variable. Even if the correlation were not spurious, it would be difficult in power.
to know which way causation ran. Does more education lead to (3) Better standards of living and access to higher education.
greater democracy? Or are more democratic countries better at (a) All (1), (2) and (3) (b) Only (2) and (3)
educating their citizens? A recent NBER paper compared a group of (c) Only (3) (d) Only (1) and (2)
Kenyan girls in 69 primary school whose students were randomly (e) None of these
selected to receive a scholarship with similar students in schools 28. What according to the author has led to uprisings in
which received no such financial aid. Previous studies has shown authoritarian countries?
that the scholarship programme led to higher test scores and (a) Lack of access to education.
increased the likelihood that girls enrolled in secondary school. (b) Vast numbers of uneducated and unemployable youth.
Overall, it significantly increased the amount of education obtained. (c) Frustration with the existing system of governance.
For the new study the authors tried to see how the extra schooling (d) Unavailability of natural energy resources like coal and
had affected the political and social attitudes of the women in oil.
(e) Government’s overambitious plans for development.
question. Findings suggested that education may make people more
29. Which of the following is/are true about China in the context
interested in improving their own lives but they may not necessarily
of the passage?
see democracy as the way to do it. Even in established democracies, (1) China’s citizens are in favor of a more representative
more education does not always mean either more active political form of government.
participation or greater faith in democracy. Poorer and less educated (2) China has made huge strides in infrastructure
people often vote in larger numbers than their more educated developments.
compatriots, who often express disdain for the messiness of (3) China is in the midst of a political revolution.
democracy, yearning for the kind of government that would deal (a) None (b) Only (1)
strongly with the corrupt and build highways, railway lines and (c) Only (1) and (3) (d) Only (2)
bridges at a dizzying pace of authoritarian China. (e) All (1), (2) and (3)
24. Which of the following most aptly describes the central 30. What does the phrase “messiness of democracy” convey
theme of the passage? in the context of the passage?
(a) Democratic nations are richer and have a better track (a) Democratic nations are chaotic on account of individual
record of educating their citizens. freedoms.
(b) Education does not necessarily lead to greater (b) Most democratic countries frequently have violent
enthusiasm for a democratic form of government revolts among their citizens.
(c) Educated societies with autocratic form of government (c) The divide between the poor and educated is growing
enjoy a better quality of life than democracies. wider in democracies.
(d) Citizens can fulfill their personal aspirations only under (d) High levels of pollution on account of frantic pace of
a democratic form of government. infrastructure development.
(e) Democracy makes citizens more intolerant as it does (e) Resigned acceptance of intrinsic corruption in the
not restrict personal freedoms education system.

RESPONSE 24. a b c d e 25. a b c d e 26. a b c d e 27. a b c d e 28. a b c d e


GRID 29. a b c d e 30. a b c d e
y
o
u
rs
m
a
h
b
54

o
o
Reading

b
.w
o
rd
Comprehension - II

p
re
s
s
.c
o
m
Max. Marks : 30 No. of Qs. 30 Time : 20 min. Date : ........./......../................

DIRECTIONS (Qs. 1-9): Read the following passage carefully 1. Why is the consumer likely to be swept off his feet?
and answer the questions given below it. Certain words/phrases (a) He is easily taken in by the deceptive publicity.
in the passage are printed in bold to help you locate them while (b) He is wooed by the charm of foreign brands readily
answering some of the questions.
available in the market.
In a country where consumers have traditionally had a raw
(c) He is not aware of the Law of Torts as practised abroad.
deal, the Consumer Protection Act was one of the most
progressive acts of legislation introduced in 1986. Before this, a (d) He is not aware of the benefits of the consumer rights.
shop could get away easily with the line “goods once sold will (e) The Consumer Protection Act has been implemented and
not be taken back or exchanged” or a car parking contractor with he can seek redressal.
“park at your own risk”. It is not that things have changed now
but at least a legislation is in place and a forum is available to 2. What does ‘lack of... verdicts’ imply?
seek redressal . One of the basic limitations of this act is its (a) A lack of the basis of the system, trained staff and
mystification and general ignorance. No consumer agency or decisions based on fact
group has made its provisions general, nor has any redressal (b) A paucity of funds, jury and judgement
commission or forum. Restricted as it is by a lack of in frastructure
and personnel and great verdicts to encourage consumers. The (c) A lack of resources, employees and final decision based
legislation is comprehensive. It gives consumers the right to on facts
redress against defective goods, deficient services and unfair (d) Not having the required manpower, economy and
trade practices. Consumer courts must deliver their judgements decisive ruling
within 40 days, but rarely is this deadline adhered to. This
reviewer had a first-hand experience of the chairman of a consumer (e) None of these
court in Delhi who adjourned a case against a foreign airline for 3. Which of the following statements is/are true?
two years on the grounds that he did not have staff to type the A. Girimaji’s attempt is comprehensive but could have done
orders. His replacement found the backlog so shocking that he with an angle or two more.
dismissed several cases without applying his mind, in the process
working against the interests of consumers. But what is more B. Though the Act allows the consumer to approach the
important is that the law has it that a consumer can approach court on his own, yet a lawyer to represent him is insisted
court on his own without having to pay legal fees. In practice, upon.
this does not happen. The chairperson of the National C. Despite the Act, much remains the same.
Commission, who is a sitting judge, is so attuned to delivering
judgments which can stand scrutiny in a civil court of law that it (a) Only A and C (b) Only A and B
is insisted upon that a consumer must be represented by a lawyer. (c) Only B and C (d) Only B and C
If not, cases are adjourned with impunity and set for another (e) None of these
day. Girimaji’s attempt is creditable in that it is the first of its kind
and has addressed almost all possible angles. She has discussed 4. What does the author mean by ‘mystification of the Act’?
redressals in complaints about housing, basic telephony, rail (a) The mysterious Act is yet to be resolved.
transportation, power supply, life insurance and medical (b) The consumer is wary of the Act.
negligence. There are even tips on how to file a complaint. But it
(c) The Act is not easily accessible.
is mired in the case files of the National/ State Commissions of
the Consumer Forum. A useful dimension would have been a (d) The consumer remains unaware of his rights and
comparison with the Law of Torts practised abroad. It is necessary privileges.
here also, especially in an era of economic liberalisation, when the (e) The plight of the consumer is yet to end.
consumer is like ly to be swept off his feet by free-market forces.

RESPONSE 1. a b c d e 2. a b c d e 3. a b c d e 4. a b c d e
GRID
y
o
u
rs
m
a
h
112 SPEED TEST 54

b
o
o
5. Which of the following best describes the judge’s 11. Attuned

b
.w
replacement? (a) Brought into harmony
(a) He was partial towards the airline as it was a foreign

o
(b) Adjusted

rd
one. (c) Hazardous

p
(b) He never bothered to safeguard the interests of the

re
(d) Out of tune

s
reviewer. (e) Malpractice

s
.c
(c) He dismissed cases without even giving a second 12. Adjourned

o
thought to what cases came to him. (a) Stopped (b) Postponed (c) Decided

m
(d) He was apathetic and uninterested about the direction (d) Cleared (e) Pended
the case might head in. DIRECTIONS (Qs. 13-15): Select the word which is most OPPOSITE
(e) He passed irrelevant verdicts indifferently. in meaning of the word printed in bold as used in the passage.
6. What does the Act broadly cover? 13. Impunity
(a) It protects the right to redress. (a) Penalised
(b) It is a forum that protects the redresser. (b) Fine
(c) It shields the consumer from deceptive and unfair trade (c) Sentence
practices. (d) Freedom from punishment
(d) It enables the plaintiff to fight his case free of cost. (e) None of these
14. Mired
(e) None of these
(a) Buried (b) Muddy (c) Steeped
7. Which of the following is a limitation of the Act? (d) Free (e) None of these
(a) It does not cover the international law of torts. 15. Redressal
(b) It is not comprehensive with regard to liberal economy. (l) Plea
(c) No forum or commission has come forward to bring it (b) Justice
to light. (c) Sue for compensation
(d) Its red-tapism (d) Not to compensate
(e) Put right
(e) None of these
DIRECTIONS (Qs. 16-23): Read the following passage carefully
8. How has Girimaji’s attempt been creditable?
and answer the questions given below it. Certain words / phrases
(a) It has given the Act a new dimension.
have been printed in bold to help you locate them while answering
(b) She has brought all the loopholes in the Act to the some of the questions.
consumer’s notice. Amartya Sen wrote about the Indian tradition of skepticism
(c) She has looked at the Act in a very disinterested and and heterodoxy of opinion that led to high levels of intellectual
impersonal manner. argument. The power sector in India is a victim of this tradition at
(d) She has discussed the law in the most explicit manner. its worst. Instead of forcefully communicating, supporting and
(e) Her implicit dialogue with the consumer has made him honestly and firmly implementing policies, people just debate
aware of his rights. them. It is argued that central undertakings produce power at
lower tariffs and must therefore build most of the required extra
9. What is the functionary role of the chairman of the National
capacities. This is a delusion. They no longer have access to
Commission?
low-cost government funds.
(a) To be the titular head of the commission
Uncertainty about payment remains a reason for the
(b) To be accountable to the public
hesitation of private investment. They had to sell only to SEBs
(c) To prevent any dissent arising out of his verdicts and (State Electricity Boards). SEB balance sheets are cleaner after the
Acts “securitisation” of the Rs 40,000 crore or so owed by SEBs to
(d) To adjourn the cases with impunity central government undertakings, now shown as debt instruments.
(e) None of these But state governments have not implemented agreed plans to
ensure repayment when due. The current annual losses of around
DIRECTIONS (Qs. 10-12): Choose the word which is most SIMILAR
Rs 28,000 crore make repayment highly uncertain. The central
in meaning to the word printed in bold as used in the passage. undertakings that are their main suppliers have payment security
10. Forum because the government will come to their help. Private enterprises
(a) Dias (b) Podium (c) Platform do not have such assurance and are concerned about payment
(d) Stage (e) None of these security, that must be resolved.

5. a b c d e 6. a b c d e 7. a b c d e 8. a b c d e 9. a b c d e
RESPONSE 10. a b c d e 11. a b c d e 12. a b c d e 13. a b c d e 14. a b c d e
GRID 15. a b c d e
y
o
u
rs
m
a
h
SPEED TEST 54 113

b
o
o
By the late 1990s, improving the SEB finances was 18. Which of the following is the reason for apathy of private

b
.w
recognised as fundamental to power reform. Unbundling SEBs, investors in power sector?
(a) Their hesitation

o
working under corporate discipline and even privatisation and

rd
not vertically integrated state enterprises, are necessary for (b) Uncertainly of their survival

p
efficient and financially viable electricity enterprises. Since (c) Cut-throat competition

re
(d) Lack of guarantee of timely returns

s
government will not distance itself from managing them,

s
(e) None of these

.c
privatising is an option. The Delhi model has worked. But it

o
receives no public support. 19. What was the serious omission on the part of the State

m
Government?
The Electricity Act 2003, the APRDP (Accelerated Power
(a) Agreement for late recovery of dues
Reform and Development Programme) with its incentives and
(b) Reluctance to repay to private investors as per agreed
penalties, and the creation of independent r egulatory
plan
commissions, were the means to bring about reforms to improve
(c) Non-implementation of recovery due to unplanned and
financial viability of power sector. Implementation has been half-
haphazard polices
hearted and results disappointing. The concurrent nature of
(d) Lack of assurance from private enterprises
electricity in the Constitution impedes power sector improvement.
(e) None of these
States are more responsive to populist pressures than the central
20. Which of the following is/are considered necessary for
government, and less inclined to take drastic action against
electricity thieves. improving performance of electricity enterprises?
(A) Corporate work culture
Captive power would add significantly to capacity. However, (B) Privatisation
captive generation, three years after the Act enabled it, has added (C) Properly integrated state enterprises
little to capacity because rules for open access were delayed.
(a) All the three (b) (a) and (b) only
Redefined captive generation avoids state vetoes on purchase
(c) (a) and (c) only (d) (b) and (c) only
or sale of electricity except to state electricity enterprises.
(e) None of these
Mandating open access on state-owned wires to power
21. The example of “Delhi Model” quoted by the author
regardless of ownership and customer would encourage
underlines his feelings of
electricity trading. The Act recognised electricity trading as a
A. happiness about its success.
separate activity. A surcharge on transmission charges will pay
B. unhappiness for lack of public support
for cross-subsidies. These were to be eliminated in time. Rules
C. disgust towards privatisation.
for open access and the quantum of surcharge by each state
(a) (a) and (b) only (b) (b) and (c) only
commission (under broad principles defined by the central
(c) (a) and (c) only (d) All the three
commission) have yet to be announced by some. The few who
have announced the surcharge have kept it so high that no trading (e) None of these
can take place. 22. Which of the following was/were not considered as the
instrument(s) to accomplish financial well-being of power
16. The author thinks it appropriate to sector?
(a) discuss any policy in details and make it fool proof (a) The Electricity Act 2003
instead of implementing it hastily.
(b) The APRDP with its incentives and penalties
(b) follow Indian tradition meticulously as skepticism is
(c) Setting up of independent regulatory commissions
essential for major decisions.
(d) States vulnerability to populist pressures
(c) divert our energies from fruitlessly contracting policies
(e) Taking drastic action against electricity thieves.
to supporting its implementation whole-heartedly.
23. Why were the results of the power sector reforms NOT as
(d) intellectual arguments and conceptualisation of every
had been anticipated?
policy is definitely better than its enforcement.
(a) The means to bring about reforms were illconceived.
(e) none of these
(b) The enforcement of the reform means was inadequate
17. Why are the central undertakings not capable of generating
and apathetic.
power at low cost?
(a) Due to paucity of low-cost funds (c) The Act and the reform measures were contradicting
(b) Due to their access to Government funds with each other.
(c) Due to their delusion about government funds (d) The incentives on the one hand and penalties on the
(d) Because of their extra capacities other created dissatisfaction.
(e) None of these (e) None of these

RESPONSE 16. a b c d e 17. a b c d e 18. a b c d e 19. a b c d e 20. a b c d e


21. a b c d e 22. a b c d e 23. a b c d e
GRID
y
o
u
rs
m
a
h
114 SPEED TEST 54

b
o
o
DIRECTIONS (Qs. 24-30) : Read the following passage and answer 25. The author thinks that openness in budget is essential as it

b
leads to

.w
the questions given below it. Certain words/phrases are given in
(a) prevention of tax implications

o
hold to bold you to locate them while answering some of the

rd
questions. (b) people’s reluctance to accept their moral duties

p
(c) exaggerated revelation of the strengths and

re
We have inherited the tradition of secrecy about the budget
weaknesses of economy

s
from Britain where also the system has been strongly attacked by

s
.c
eminent economists and political scientists including Peter Jay. (d) making our country on par with Finland

o
(e) None of these

m
Sir Richard Clarke, who was the originating genius of nearly every
important development in the British budgeting techniques during 26. The author seems to be in favour of
the last two decades, has spoken out about the abuse of budget (a) maintaining secrecy of budget
secrecy: “The problems of long-term tax policy should surely be (b) judicious blend of secrecy and openness
debated openly with the facts on the table. In my opinion, all (c) transparency in budget proposals
governments should have just the same duty to publish their (d) replacement of public constitution by secrecy
expenditure policy. Indeed, this obligation to publish taxation (e) None of these
policy is really essential for the control of public expenditure in 27. The secrecy of the budget is maintained by all of the
order to get realistic taxation implications.” Realising that following countries except
democracy flourishes best on the principles of open government, A. Finland
more and more democracies are having an open public debate on B. India
budget proposals before introducing the appropriate Bill in the C. United States
legislature. In the United States the budget is conveyed in a (a) Only A (b) Only B (c) Only C
message by the President to the Congress, which comes well in (d) A and C (e) B and C
advance of the date when the Bill is introduced in the Congress. 28. Which of the following statements is definitely TRUE in the
In Finland the Parliament and the people are already discussing in context of the passage?
June the tentative budget proposals which are to be introduced in (a) The British Government has been religiously
the Finnish Parliament in September. Every budget contains a maintaining budget secrecy.
cartload of figures in black and white - but the dark figures represent (b) Budget secrecy is likely to lead to corrupt practices.
the myriad lights and shades of India’s life, the contrasting tones (c) Consulting unjustifiable taxes with public helps make
of poverty and wealth, and of bread so dear and flesh and blood them accept those taxes.
so cheap, the deep tints of adventure and enterprise and man’s (d) There should be no control on public expenditure in
ageless struggle for a brighter morning. The Union budget should democratic condition.
not be an annual scourge but a part of presentation of annual (e) None of these
accounts of a partnership between the Government and the people. 29. Sir Richard Clarke seems to deserve the credit for
That partnership would work much better when the nonsensical (a) transformation in the British budgetary techniques.
secrecy is replaced by openness and public consultations, resulting (b) maintenance of secrecy of the British budget.
in fair laws and the people’s acceptance of their moral duty to pay. (c) detection of abuse of transparency in budget.
24. How do the British economists and political scientists react (d) bringing down the tax load on British people.
to budget secrecy? They are (e) None of these
(a) in favour of having a mix of secrecy and openness. 30. From the contents of the passage, it can be inferred that the
(b) indifferent to the budgeting techniques and taxation author is
policies. (a) authoritarian in his approach.
(c) very critical about maintenance of budget secrecy. (b) a democratic person.
(d) advocates of not disclosing in advance the budget (c) unaware of India’s recent economic developments.
contents. (d) a conservative person.
(e) None of these (e) None of these

RESPONSE 24. a b c d e 25. a b c d e 26. a b c d e 27. a b c d e 28. a b c d e

GRID 29. a b c d e 30. a b c d e


y
o
u
rs
m
a
h
55

b
o
o
Reading

b
.w
o
rd
Comprehension - III

p
re
s
s
.c
o
m
Max. Marks : 30 No. of Qs. 30 Time : 20 min. Date : ........./......../................

DIRECTION : Read the following passages to answer the given The political benefits of the loan waiver have also been
question bused on it. Some words/phrases are printed in bold to exaggerated since if only a small fraction of farm families benefit,
and many of these have to pay bribes to get the actual benefit, will
help you locate them while answering some of the questions.
the waiver really be a massive vote-winner? Members of joint families
Political ploys initially hailed as master-strokes often end will feel aggrieved that, despite having less than one hectare per
up as flops. The ` 60,000 crore farm loan waiver announced in head, their family holding is too large to qualify for the 100% waiver.
the budget writes off 100% of overdues of small and marginal All finance ministers, of central or state governments, give away
farmers holding upto two hectares, and 25% of overdues of larger freebies in their last budgets, hoping to win electoral regards. yet,
farmers. While India has enjoyed 8%-9% GDP growth for the four-fifth of all incumbent government are voted out. This shows
past few years, the boom has bypassed many rural areas and that beneficiaries of favours are not notably grateful, while those
farmer distress and suicides have made newspaper headlines. not so favoured may feel aggrieved, and vote for the opposition.
Various attempts to provide relief (employment guarantee scheme, That seems to be why election budgets constantly fail to win
public distribution system) have made little impact, thanks to elections in India and the loan waiver will not change that pattern.
huge leakages from the government’s lousy delivery systems. 1. Why do economists feel that loan waivers will benefit farmers
in distress?
So, many economists think the loan waiver is a worthwhile
(a) It will improve the standard of living of those farmers
alternative to provide relief. who can afford to repay their loans but are exempted.
However the poorest rural folk are landless labourers who (b) Other government relief measures have proved
get neither farm loans nor waivers. Half of the small and marginal ineffective,
farmers get no loans from banks, and depend entirely on (c) Suicide rates of farmers have declined after the
moneylenders, and will not benefit. Besides, rural India is full of announcement of the waiver.
the family holdings rather than individual holdings and family (d) Farmers will be motivated to increase the size of their
holdings will typically be much larger than two hectares even for family holdings not individual holdings.
dirt-poor farmers, who will, therefore, be denied the 100% waiver. It (e) The government will be forced to re-examine and improve
will thus fail in both economic and political objectives. IRDP loans the public distribution system.
to the rural poor in the 1980s demonstrated that crooked bank 2. What message will the loan waiver send to farmers who have
officials demand bribes amounting to one third the intended repaid loans?
(a) The Government will readily provide them with loans in
benefits. Very few of the intended beneficiaries who merited relief
the future.
received it. After the last farm loan waiver will similarly slow down (b) As opposed to money lenders banks are a safer and
fresh loans to deserving farmers. While overdues to co-operatives more reliable source of credit.
may be higher, economist Snrjit Bhalla says less then 5% of farmer (c) Honesty is the best policy.
loans to banks are overdue ie overdues exist for only 2.25 million (d) It is beneficial to take loans from co-operatives since
out of 90 million farmers. If so, then the 95% who have repaid loans their rates of interest are lower.
will not benefit. They will be angry at being penalised for honesty. (e) They will be angry at being penalised for honesty.
The budget thus grossly overestimates the number of 3. What is the author's suggestion to provide aid to farmers?
beneficiaries. It also underestimates the negative effects of the (a) Families should split their joint holding to take advantage
waiver encouraging wilful default in the future and discouraging of the loan waiver.
fresh bank lending for some years. Instead of trying to reach the (b) The government should increase the reach of the
needy, through a plethora of leaky schemes we should transfer employment guarantee scheme.
cash directly to the needy using new technology like biometric (c) Loans should be disbursed directly into bank accounts
of the farmers using the latest technology.
smart cards, which are now being used in many countries, and
(d) Government should ensure that loans waivers can be
mobile phones bank accounts. Then benefits can go directly to implemented over the number of years.
phone accounts operable only by those with biometric cards, (e) Rural infrastructure can be improved using schemes
ending the massive leakages of current schemes. which were successful abroad.

RESPONSE 1. a b c d e 2. a b c d e 3. a b c d e
GRID
y
o
u
rs
m
a
h
116 SPEED TEST 55

b
o
o
4. What was the outcome of IRDP loans to the rural poor? 9. What impact will the loan waiver have on banks?

b
.w
(a) The percentage of bank loan sanctioned to family (a) Banks have to bear the entire brunt of the write off.

o
owned farms increased. (b) Loss of trust in banks by big farmers.

rd
(b) The loans· benefited dishonest moneylenders not (c) Corruption among bank staff will increase.

p
re
landless labourers. (d) Farmers will make it a habit to default on loans

s
(c) Corrupt bank officials were the unintended beneficiaries (e) None of these

s
.c
of the loans. 10. According to the author what is the government's motive in

o
m
(d) It resulted in tne Government sanctioning thrice the sanctioning the loan waiver?
amount for the current loan waiver. (a) To encourage farmers to opt for bank loans from money
(e) None of these. lenders.
5. What are the terms of the loan waiver? (b) To raise 90 million farmers out of indebtedness.
(1) One-fourth of the overdue loans of landless labourers (c) To provide relief to those marginal farmers who have
will be written off. the means to but have not repaid their loans
(2) The ` 60,000 crore loan waiver has been sanctioned for (d) To ensure they will be re-elected
2.25 million marginal farmers. (e) None of these
(3) Any farmer with between 26 per cent to 100 per cent of DIRECTION (Q. 11-13) : Choose the word which is most nearly
their loan repayments overdue will be penalised. the SAME in meaning to the word printed in bold as used in the
(a) Only (1) (b) On1y (2) passage.
(c) Both (2) and (3) (d) All (1), (2) and (3) 11. incumbent
(e) None of these (a) mandatory (b) present
6. What is the author's view of the loan waiver? (c) incapable (d) tazy
(a) It will have an adverse psychological impact on those (e) officious
who cannot avail of the waiver. 12. ploys
(b) It is a justified measure in view of the high suicide rate (a) surveys (b) entreaties
among landless labourers. (c) ruses (d) sliders
(c) It makes sound economic and political sense in the (d) assurances
existing scenario. 13. aggrieved
(d) It will ensure that the benefits of India's high GDP are (a) vindicated (b) intimidated
felt by the rural poor. (c) offensive (d) wronged
(e) None of these (d) disputed
7. Which of the following cannot be said about loan waiver? DIRECTIONS (Q. 14 & 15) : Choose the word which is most OPPOSITE
(1) Small and marginal farmers will benefit the most. in meaning to the word printed in bold as used in the passage.
(2) The loan waiver penalises deserving farmers. 14. plethora
(3) A large percentage ie ninety five per cent of distressed (a) dearth (b) missing
farmers will benefit. (c) superfluous (d) sufficient
(a) Only (3) (b) Both (1) and (3) (e) least
(c) Only (1) (d) Both (2) and (3) 15. merited
(e) None of these (a) ranked· (b) unqualified for
8. Which of the following will definitely be an impact of loan (c) lacked (d) inept at
waivers? (e) unworthy of
(1) Family holdings will be split into individual holdings DIRECTIONS (Q. 16-25): Read the following passage to answer
not exceeding one hectare. the given questions based on it, Some words/phrases are printed in
(2) The public distributipn system will be revamped. bold to help you locate them while answermg some of the questions.
(3) Opposition will definitely win the election. The e-waste (Management and Handling) Rules, 2011,
(a) None (b) Only (1) notified by the Ministry of Environmynt and Forests, have the
(c) Both (1) and (2) (d) Only (3) potential to turn a growing problem into a developmental
opportunity. With almost half-a-year to go before the rules take
(e) All (1), (2) and (3)

4. a b c d e 5. a b c d e 6. a b c d e 7. a b c d e 8. a b c d e
RESPONSE 9. a b c d e 10. a b c d e 11. a b c d e 12. a b c d e 13. a b c d e
GRID 14. a b c d e 15. a b c d e
y
o
u
rs
m
a
h
SPEED TEST 55 117

b
o
o
effect, there is enough time to create the necessary infrastructure 17. Which of the following can be one ofthe by-products of

b
.w
for collection, dismantling and recycling of electronic waste. The effective e-waste management? .
focus must be on sincere and efficient implementation. Only (a) India can guide other countries in doing so.

o
rd
decisive action can reduce the pollution and health costs (b) It will promote international understanding.

p
associated with India's hazardous waste recycling industry. If (c) It will promote national integration.

re
s
India can achieve a transformation, it will be creating a whole (d) It will create a new employment sector.

s
.c
new-employment sector that provides good wages and working (e) It will further empowet judiciary.

o
conditions for tens of thousands. The legacy response of the 18. Which of the following rules has not been indicated in the

m
States to even the basic law on urban waste, the Municipal Solid passage?
Wastes (Management and Handling) Rules has been one of (a) e-waste Rules 20 11
indifference; many cities continue to simply bum the garbage or (b) Pollution Check Rules
dump it in lakes. With the emphasis now on segregation of waste (c) Hazardous Wastes Rules, 2008
at source and recovery of materials. it should be feasible to (d) Municipal Solid Wastes Rules
implement both sets of rules efficiently. A welcome feature of the (e) All these have been indicated
new e-waste rules is the emphasis on extended producer 19. “Both sets of rules” is being referred to which of the following?
responsibility. In other words, producers must take responsibility (a) Solid wastes and hazardous wastes
for the disposal of end-of-life products. For this provision to (b) e-waste and hazardous waste
work, they must ensure that consumers who sell scrap get some (c) Solid waste and e-waste
form of financial incentive. (d) e-waste and e-production
The e-waste rules, which derive from those pertaining to (e) Solid waste and recycling waste
hazardous waste, are scheduled to come into force on May 1, 20. e-waste rules have been derived from those pertaining to
2012. Sound as they are, the task of scientifically disposing a few (a) Hazardous waste (b) PC waste
hundred thousand tonnes of trash electronics annually depends (c) Computer waste (d) Municipal solid waste
heavily on a system of oversight by State Pollutions Control (d) National waste
Boards (PCBs). Unfortunately, most PCBs remain unaccountable 21. Which of the following will help implement “both sets of
and often lack the resources for active enforcement. It must be rules”?
pointed out that, although agencies handling e-waste must obtain (a) Employment opportunities
environmental clearances and be authorised and registered by (b) International collaboration
the PCBs even under the Hazardous Wastes (Management, (c) Financial incentive
Handling and Transboundary Movement) Rules 2008, there has (d) Segregation of waste at source
been little practical impact. Over 95 per cent of electronic waste is (e) Health costs
collected and recycled by the informal sector. The way forward is 22. e-waste Rules came/come into force from
for the PCBs to be made accountable for enforcement of the e- (a) 2008 (b) 2009
waste rules and the levy of penalties under environmental laws. (c) 2010 (d) 2011
Clearly, the first order priority is to create a system that will absorb
(e) 2012
the 80,000-strong workforce in the informal sector into the
23. Which of the following best explains the meaning of the
proposed scheme for scientific recycling. Facilities must be created phrase “which could only be imagined few years back” as
to upgrade the skills of these workers through training and their used in the passage?
occupational health must be ensured. (a) It was doomed.
Recycling of e-waste is one of the biggest challenges today. In (b) It took us few years.
such a time, when globalisation and information technology are (c) It took us back by few years
growing at a pace which could only be imagined few years back, (d) Imagination is better than IT.
e-waste and its hazards have become more prominent over a (e) None of these
period of time and should be given immediate attention. 24. Which of the following is true in the context of the passage?
16. What, according to the passage, is important now for (a) No city dumps its waste in lakes.
e-waste management? (b) Some cities burn garbage.
(a) Making rules (b) Reviewing rules (c) PCBs have adequate resources for'active enforcement.
(c) Implementing rules (d) Notifying rules (d) e-waste was a much bigger chajlenge in the past.
(e) Amending rules (e) None of these

RESPONSE 16. a b c d e 17. a b c d e 18. a b c d e 19. a b c d e 20. a b c d e


21. a b c d e 22. a b c d e 23. a b c d e 24. a b c d e
GRID
y
o
u
rs
m
a
h
118 SPEED TEST 55

b
o
o
25. Which of the following is not true 10 the context of the 27. Turn

b
.w
passage? (a) Throw (b) Chance
(c) Send (d) Transform

o
(a) Some fonn offmancial incentive is recommended for

rd
the producers. (e) Rotate

p
28. Potential

re
(b) Some financial incentive is recommended for the

s
consumers. (a) Intelligence (b) Aptitude

s
.c
(c) e-waste will be a few hundred thousand tonnes. (c) Possibility (d) Portion

o
(e) Will

m
(d) The agencies handling e-waste have to obtain
environmental clearances. DIRECTIONS (Q. 29 & 30): Choose the word which is most
(e) Those involved in e-waste management would need to opposite in meaning of the word printed in bold as used in the
upgrade their skills. passage.
29. Feasible
DIRECTIONS (Q. 26-28): Choose the word which is most nearly (a) Unattended (b) Physical
the same in meaning of the word printed in bold, as used in the (c) Practical (d) Unviabie
passage. (e) Wasteful
26. Clearance 30. Indifference
(a) Cleaning (b) Permisssion (a) Interest (b) Difference
(c) Sale (d) Remedy (c) Ignorance (d) Rule-bound
(e) Clarity (e) Insignificance

RESPONSE 25. a b c d e 26. a b c d e 27. a b c d e 28. a b c d e 29. a b c d e

GRID 30. a b c d e
y
o
u
rs
m
a
h
b
56

o
o
b
.w
Synonyms

o
rd
p
re
s
s
.c
o
m
Max. Marks : 40 No. of Qs. 40 Time : 25 min. Date : ........./......../................

DIRECTIONS (Qs. 1-25): Select the synonyms of the word given 9. PROTAGONIST
in CAPITAL letters. (a) Talented child (b) Reserved person
(c) Leading character (d) Fearless
1. VICARIOUS
(e) None of these
(a) Ambitious 10. FACTITIOUS
(b) Not experienced personally (a) Humorous (b) Truthful
(c) Nostalgic (c) Artificial (d) Causing fatigue
(d) Vindictive (e) None of these
(e) None of these 11. HOSPITABLE
2. CRAVEN (a) Convivial (b) Liberal
(a) Greedy (b) Cowardly (c) Congenital (d) Welcoming
(c) Flattering (d) Restless (e) None of these
(e) None of these 12. SCARCELY
3. TEPID (a) Hardly (b) Always
(c) Sometimes (d) Frequently
(a) Irreversible (b) Causing fatigue
(e) None of these
(c) Fast moving (d) Lukewarm
13. DISDAIN
(e) None of these (a) Disown (b) Condemn
4. TENUOUS (c) Hate (d) Criticise
(a) Contentious (b) Dark (e) None of these
(c) Slender (d) Malfunctioning 14. ABSURD
(e) None of these (a) Senseless (b) Clean
5. PROBITY (c) Abrupt (d) Candid
(a) Integrity (b) Impudence (e) None of these
(c) Profane (d) Preface 15. PHILANTHROPY
(a) Generosity (b) Perversity
(e) None of these
(c) Perjury (d) Flaunting
6. MUSTY (e) None of these
(a) Certainty (b) Stale 16. MUTUAL
(c) Modern (d) Mysterious (a) Reciprocal (b) Agreed
(e) None of these (c) Common (d) Conjugal
7. ALLEVIATE (e) None of these
(a) To release (b) To lessen 17. WEIRD
(c) To deprive (d) To deceive (a) Beastly (b) Unpleasant
(e) None of these (c) Frightening (d) Unnatural
8. MOROSE (e) None of these
(a) Humble (b) Morsel
(c) Sullen (d) Repugnant
(e) None of these

1. a b c d e 2. a b c d e 3. a b c d e 4. a b c d e 5. a b c d e
RESPONSE 6. a b c d e 7. a b c d e 8. a b c d e 9. a b c d e 10. a b c d e
GRID 11. a b c d e 12. a b c d e 13. a b c d e 14. a b c d e 15. a b c d e
16. a b c d e 17. a b c d e
y
o
u
rs
m
a
h
120 SPEED TEST 56

b
o
o
18. PESSIMISTIC (c) Clear (d) Ambiguous

b
(a) Indifferent (b) Ascetic (e) None of these

.w
(c) Unsettle (d) Not hopeful 30. Friends have always DEPLORED my unsociable nature.

o
rd
(e) None of these (a) Deprived (b) Implored

p
19. ANALOGOUS (c) Denied (d) Regretted

re
(a) Unsuitable (b) Uncritical (e) None of these

s
s
(c) Similar (d) Disproportionate

.c
31. People fear him because of his VINDICTIVE nature.
(e) None of these

o
(a) Violent (b) Cruel

m
20. EXAGGERATE
(a) Bluff (b) Overstate (c) Revengeful (d) Irritable
(c) Explain (d) Underestimate (e) None of these
(e) None of these 32. He always has a very PRAGMATIC approach to life.
21. EVIDENT (a) Practical (b) Proficient
(a) Prominent (b) Seen (c) Potent (d) Patronizing
(c) Observed (d) Quite clear (e) None of these
(e) None of these 33. The song had a SOPORIFIC effect on the child.
22. PENALIZE
(a) Soothing (b) Terrific
(a) Persecute (b) Punish
(c) Torture (d) Ruin (c) Supreme (d) Sleep-inducing
(e) None of these (e) None of these
23. REMEDIAL 34. His bad behaviour EVOKED punishment.
(a) Punitive (b) Stringent (a) Escaped (b) Called for
(c) Corrective (d) Strict (c) Produced (d) Summoned
(e) None of these (e) None of these
24. TRIVIAL 35. The boy said that pain had ABATED.
(a) Unimportant (b) Transparent (a) Reduced (b) Vanished
(c) Important (d) Unexpected (c) Increased (d) Stabilised
(e) None of these (e) None of these
25. INCREDIBLE 36. The queen was aware of the INSOLENT behaviour of the
(a) Hard to believe (b) Considerable lords.
(c) Inconsistent (d) Unsatisfactory
(a) Violent (b) Polite
(e) None of these
(e) None of these
DIRECTIONS (Qs. 26-40): Select the synonym of the word (c) Insulting (d) Frivolous
occuring in the sentence in CAPITAL letters as per the context. 37. The newspaper reports were MENDACIOUS
26. After weeks of FRENETIC activity, the ground was ready (a) mischievous (b) truthful
for the big match. (c) provocative (d) false
(a) Strenuous (b) Hurried (e) None of these
(c) Excited (d) Hectic
38. He INDUCES human beings to want things they don’t want.
(e) None of these
(a) Influences (b) Dictates
27. The policemen kept on questioning the criminal but failed to
ELICIT the truth. (c) Persuades (d) Appreciates
(a) Evoke (b) Wrest (e) None of these
(c) Obtain (d) Extort 39. His information is not AUTHENTIC.
(e) None of these (a) Real (b) Reliable
28. Teaching in universities is now considered a LUCRATIVE (c) Believable (d) Genuine
job. (e) None of these
(a) Risky (b) Profitable 40. It is the difficulty or SCARCITY of a thing that it makes it
(c) Honourable (d) Undignified precious.
(e) None of these (a) poverty (b) absence
29. His descriptions are VIVID. (c) insufficiency (d) disappearance
(a) Detailed (b) Categorical (e) None of these

18. a b c d e 19. a b c d e 20. a b c d e 21. a b c d e 22. a b c d e


23. a b c d e 24. a b c d e 25. a b c d e 26. a b c d e 27. a b c d e
RESPONSE 28. a b c d e 29. a b c d e 30. a b c d e 31. a b c d e 32. a b c d e
GRID 33. a b c d e 34. a b c d e 35. a b c d e 36. a b c d e 37. a b c d e
38. a b c d e 39. a b c d e 40. a b c d e
y
o
u
rs
m
a
h
b
57

o
o
b
.w
Antonyms

o
rd
p
re
s
s
.c
o
m
Max. Marks : 40 No. of Qs. 40 Time : 25 min. Date : ........./......../................

DIRECTIONS (Qs. 1-20): Select the antonyms of the word given 11. EXPLOIT
in CAPITAL letters. (a) Utilize (b) Alert
(c) Support (d) Neglect
1. REINFORCING
(e) None of these
(a) contradicting (b) wishing
12. SHARP
(c) jolting (d) forcing
(e) re-inventing (a) Bleak (b) Blunt
2. BEFRIENDED (c) Bright (d) Blond
(a) recoiled (b) killed (e) None of these
(c) accepted (d) mistrusted 13. CONDEMN
(e) ignored (a) Censure (b) Approve
3. FRAIL (c) Recommend (d) Praise
(a) Unhealthy (b) Massive (e) None of these
(c) Rich (d) Robust 14. RELUCTANT
(e) Civilised (a) Avoiding (b) Anxious
4. SPLENDIDLY (c) Refuse (d) Eager
(a) Wisely (b) Unfairly (e) None of these
(c) Rudely (d) Reluctantly 15. SCARCITY
(e) Unimpressively (a) Plenty (b) Prosperity
5. CHARMED (c) Facility (d) Simplicity
(a) Offended (b) Stunned (e) None of these
(c) Repulsed (d) Jealous 16. BLEAK
(e) Outraged (a) Bright (b) Confusing
6. OBLIGATORY (c) Uncertain (d) Great
(a) Doubtful (b) Voluntary (e) None of these
(c) Sincerely (d) Faithfully 17. STERN
(e) None of these (a) Violent (b) Generous
7. OBSCURE (c) Mild (d) Forgiving
(a) Suitable (b) Apt (e) None of these
(c) Thalamus (d) Clear 18. SUPERFICIAL
(e) None of these (a) Profound (b) Difficult
8. REPULSIVE (c) Secretive (d) Mystical
(a) Attractive (b) Colourful (e) None of these
(c) Unattractive (d) Striking 19. ELEGANCE
(e) None of these (a) Balance (b) Indelicacy
9. VITAL (c) Clumsiness (d) Savagery
(a) Trivial (b) Peripheral (e) None of these
(c) Optional (d) Superficial 20. COARSE
(e) None of these (a) Pleasing (b) Rude
10. INHIBIT (c) Polished (d) Soft
(a) Pamper (b) Breed
(e) None of these
(c) Accept (d) Promote
(e) None of these

1. a b c d e 2. a b c d e 3. a b c d e 4. a b c d e 5. a b c d e
RESPONSE 6. a b c d e 7. a b c d e 8. a b c d e 9. a b c d e 10. a b c d e
GRID 11. a b c d e 12. a b c d e 13. a b c d e 14. a b c d e 15. a b c d e
16. a b c d e 17. a b c d e 18. a b c d e 19. a b c d e 20. a b c d e
y
o
u
rs
m
a
h
122 SPEED TEST 57

b
o
o
DIRECTIONS (Qs. 21-40): Select the antonym of the word (a) Active (b) Lazy

b
.w
occuring in the sentence in CAPITAL letters as per the context. (c) Strong (d) Resolute
(e) None of these

o
21. He is a man of EXTRAVAGANT habits.

rd
(a) Sensible (b) Careful 31. The boy comes of an AFFLUENT family.

p
(a) Poor (b) Ordinary

re
(c) Economical (d) Balanced
(c) Infamous (d) Backward

s
(e) None of these

s
(e) None of these

.c
22. They employ only DILIGENT workers.

o
32. The young athlete is ENERGETIC enough to run ten

m
(a) Unskilled (b) Lazy thousand meters at a stretch.
(c) Careless (d) Idle (a) inactive (b) dull
(e) None of these (c) gloomy (d) lethargic
23. The characters in this story are not all FICTITIOUS. (e) None of these
(a) Common (b) Factual 33. He has a SECURE position in the entrance examination.
(c) Real (d) Genuine (a) rigid (b) precarious
(e) None of these (c) static (d) secondary
24. He is a GENEROUS man. (e) None of these
(a) Stingy (b) Uncharitable 34. The criminal was known to the police by VARIOUS names.
(c) Selfish (d) Ignoble (a) separate (b) distinct
(e) None of these (c) identical (d) similar
25. The issue raised in the form can be IGNORED. (e) None of these
(a) Removed (b) Considered 35. RECESSION is a major cause of unemployment.
(c) Set aside (d) Debated (a) Education (b) Inflation
(e) None of these (c) Poverty (d) Computerization
26. After swallowing the frog had become LETHARGIC. (e) None of these
(a) Aggressive (b) Dull 36. The accused emphatically DENIED the charge in the court.
(a) Accepted (b) Agreed
(c) Active (d) Hungry
(c) Asserted (d) Affirmed
(e) None of these
(e) None of these
27. Dust storms and polluted rivers have made it HAZARDOUS
37. The three states signed a pact to have COLLECTIVE
to breathe the air and drink the water. economy.
(a) Convenient (b) Risky (a) distributive (b) disintegrative
(c) Wrong (d) Safe (c) individual (d) divided
(e) None of these (e) None of these
28. Only hard work can ENRICH our country. 38. The child was ABDUCTED when he was going to school.
(a) Impoverish (b) Improve (a) seized (b) set free
(c) Increase (d) Involve (c) kidnapped (d) ransomed
(e) None of these (e) None of these
29. The story you have just told is INCREDIBLE. 39. The government has ENHANCED the tuition fees in schools
(a) Credible (b) Fantastic and colleges.
(c) Probable (d) Believable (a) magnified (b) aggravated
(e) None of these (c) decreased (d) augmented
30. The doctor found the patient INERT. (e) None of these
40. Children ADORE the cricket players.
(a) Discuss (b) Condemn
(c) Benefit (d) Check
(e) None of these

21. a b c d e 22. a b c d e 23. a b c d e 24. a b c d e 25. a b c d e


RESPONSE 26. a b c d e 27. a b c d e 28. a b c d e 29. a b c d e 30. a b c d e

GRID 31. a b c d e 32. a b c d e 33. a b c d e 34. a b c d e 35. a b c d e


36. a b c d e 37. a b c d e 38. a b c d e 39. a b c d e 40. a b c d e
y
o
u
rs
m
a
h
b
58

o
o
Sentence

b
.w
o
rd
Completion - I

p
re
s
s
.c
o
m
Max. Marks : 30 No. of Qs. 30 Time : 20 min. Date : ........./......../................

DIRECTIONS: Select the correct word or phrase to complete a 8. Of the two assistants we employed last month, I find
grammatical sentence. In case of more than one blank, the different Raman .............. hard working.
(a) most (b) more
words given in the options shall fill in the corrosponding order.
(c) least (d) only
1. As soon as the visitor’s dishonest purpose was discovered (e) None of these
he was ........ the door. 9. It is earth’s gravity which .............. people their weight.
(a) show with (b) shown to (a) gives (b) give
(c) shown (d) shown out of (c) giving (d) given
(e) None of these (e) None of these
2. As soon as my attention was ........ the dangerous state of 10. Total weight of all the ants in the world is much greater
the staircase, I got it repaired. than ............... .
(a) drawn for (b) drawn upon (a) to all human beings
(c) drawn near (d) drawn to (b) that of all human beings
(e) None of these (c) is of all human beings
3. I wish my brother -........ here to listen to this entertaining (d) that of the all human beings
lecture. (e) None of these
(a) would be (b) has been 11. It is good form to use the name of the person ............... .
(c) is (d) were (a) who are greeting
(e) None of these (b) you are greeting
4. Did you think you ........ somewhere before? (c) which you are greeting
(a) have seen me (b) saw me (d) greeting for you
(e) None of these
(c) had seen me (d) would see me
12. .............. that increasing numbers of compact disc players will
(e) None of these
be bought by consumers in the years to come.
5. Do not force me to ........ you on this issue; I am not at all
(a) They are anticipated
convinced. (b) In anticipation
(a) agree upon (b) concur with (c) Anticipating
(c) join over (d) equate with (d) It is anticipated
(e) None of these (e) None of these
6. Having ........ only in salt water before, I found it a little 13. He was frightened ...............
difficult to swim in fresh water. (a) to be killed (b) to being killed
(a) swam (b) swum (c) for being killed (d) of being killed
(c) had swam (d) swimming (e) None of these
(e) None of these 14. Capitalist society .............. profit as a valued goal.
7. If I were you, I ............... be careful with my words. (a) which regards (b) regarding
(a) will (b) would (c) regards (d) was regarded
(c) shall (d) should (e) None of these
(e) None of these 15. The impact of two vehicles can cause a lot of .............. to both.
(a) damage (b) damages
(c) damaging (d) damagings
(e) None of these

1. a b c d e 2. a b c d e 3. a b c d e 4. a b c d e 5. a b c d e
RESPONSE
6. a b c d e 7. a b c d e 8. a b c d e 9. a b c d e 10. a b c d e
GRID
11. a b c d e 12. a b c d e 13. a b c d e 14. a b c d e 15. a b c d e
y
o
u
rs
m
a
h
124 SPEED TEST 58

b
o
o
16. The conditions necessary .............. this project have not been DIRECTIONS : Select the correct word or phrase to complete a

b
met.

.w
grammatical and idiomatic sentence.
(a) of completion (b) for the complete of

o
24. If you are really not feeling well. you .......... a doctor.

rd
(c) of complete (d) for the completion of

p
(e) None of these (a) should better see (b) may Sec

re
17. The weather in the far north is not .............. it is down south. (c) had better see (d) would rather see

s
s
(a) like humid as (b) as humid as (e) None of these

.c
(c) humid as (d) so humid that

o
25. If only I.......... his address, I would most certainly have told

m
(e) None of these you.
18. .............. the reactions of people with amnesia, scientists are (a) know (b) knew
learning about the process of memory of the brain.
(c) had known (d) off
(a) By studying (b) To study
(e) None of these
(c) They study (d) They are studying
(e) None of these 26. The marathon race is intended to test one’s endurance
19. After the election .............. a new stage. more...........
(a) the entering nation (a) than his speed (b) than how fast one runs
(b) the nation will enter (c) than one’s speed (d) lain off
(c) to enter the nation (e) None of these
(d) will the nation enter 27. She expects me to type the letter in five minutes .......... is
(e) None of these impossible.
20. I hope she ............... . (a) that (b) which
(a) must come (b) should come (c) what (d) but
(c) will come (d) must be coming (e) None of these
(e) None of these
28. If only you had spoken clearly, you
21. An increase in population, without an increase in
(a) would not be misunderstood
economic level, .............. result in a lower standard of
living. (b) would not have been misunderstanding.
(a) tends to (b) tending to (c) would not have been misunderstood.
(c) will tend (d) tends (d) would not have misunderstood.
(e) None of these (e) None of these
22. .............. as President, a candidate must win a majority of 29. It is high time that he .......... himself.
votes. (a) had reformed (b) will reform
(a) Elected (b) To be elected (c) has to reform (d) reformed
(c) Having elected (d) Electing (e) None of these
(e) None of these 30. .........., a bus almost ran over him.
23. Encounters between people from different countries can (a) Running across the road
result in misunderstandings .............. different conceptions
(b) Running on the road
about space.
(c) When he ran across the road
(a) because they (b) is because they
(c) is because their (d) of their (d) When he was running through the road.
(e) None of these (e) None of these

16. a b c d e 17. a b c d e 18. a b c d e 19. a b c d e 20. a b c d e


RESPONSE 21. a b c d e 22. a b c d e 23. a b c d e 24. a b c d e 25. a b c d e
GRID 26. a c d e 27. a c d e 28. a c d e 29. a c d e 30. a c d e
b b b b b
y
o
u
rs
m
a
h
b
59

o
o
Sentence

b
.w
o
rd
Completion - II

p
re
s
s
.c
o
m
Max. Marks : 35 No. of Qs. 35 Time : 20 min. Date : ........./......../................
DIRECTIONS: Select the correct word or phrase to complete a 8. His interest in the study of human behaviour is indeed very
grammatical and idiomatic sentence. .........
(a) strong (b) large
1. These essays are intellectually .............. and represent (c) broad (d) vast
various levels of complexity. (e) deep
(a) revealing (b) modern 9. The improvement made by changes in the system was .... and
(c) superior (d) demanding id not warrant the large expenses.
(e) persistent (a) large (b) small
(c) minute (d) marginal
2. The soldiers are instructed to .......... restraint and handle
(e) uncertain
the situation peacefully. 10. He is too ..... to be deceived easily.
(a) exercise (b) control (a) strong (b) modern
(c) enforce (d) remain (c) kind (d) honest
(e) None of these (e) intelligent
3. Since one connot read every book, one should be content 11. There has been a ...... lack of efficiency in all the crucial areas
with making a ........ selection. of the working of Public Sector Undertakings.
(a) positive (b) surprising
(a) normal (b) standard
(c) conspicuous (d) stimulative
(c) sample (d) moderate (e) insignificant
(e) judicious 12. I will be leaving for Delhi tonight and ........... to return by this
4. Some people ........ themselves into believing that they are week end.
indispensable to the organisation they work for. (a) waiting (b) plan
(a) keep (b) fool (c) going (d) likely
(c) force (d) denigrate (e) making
13. Ravi’s behaviour is worthy of ....... by all the youngsters.
(e) delude (a) trial (b) emulation
5. How do you expect that country to progress when her (c) following (d) exploration
government is corrupt, ..... and still lergely feudal? (e) experiment
(a) devalued (b) dwinding 14. The only way to ...... the country from the evils of
(c) demobilised (d) demeaning communalism is to enforce the rule of law.
(e) None of these (a) eradicate (b) mobilise
(c) extricate (d) purge
6. The truck was ...... the trafic and the policeman asked the
(e) strengthen
driver to move off. 15. Even at the risk of economic loss, he ...... refused to take the
(a) failing (b) obstructing beaten track.
(c) obviating (d) hiding (a) repeatedly (b) stead fastly
(e) disturbing (c) regularly (d) continuously
7. The paternalistic attitude is so ingrained to the (e) None of these
managements that they will have to ........ try to change it. 16. On his sudden demise, may emotions were so complicated
that it was ......... how I felt.
(a) casually (b) slowly
(a) unreasonable (b) impossible
(c) subtly (d) inadvertently (c) inexplicable (d) unimaginable
(e) None of these (e) None of these

1. a b c d e 2. a b c d e 3. a b c d e 4. a b c d e 5. a b c d e
RESPONSE 6. a b c d e 7. a b c d e 8. a b c d e 9. a b c d e 10. a b c d e
GRID 11. a b c d e 12. a b c d e 13. a b c d e 14. a b c d e 15. a b c d e
16. a b c d e
y
o
u
rs
m
a
h
126 SPEED TEST 59

b
o
o
17. Two of the fugitives managed to remain free by adeptly 27. I will write a letter to you tentatively ....... the dates of the

b
avoiding the ....... of the police.

.w
programme.
(a) torture (b) pursuit (a) involving (b) urging

o
rd
(c) discovery (d) following (c) guiding (d) indicating

p
(e) None of these (e) propagating

re
18. Experts fail to understand the ..... behind the decision to

s
28. Contemporary economic development differs ....... form the

s
move coal by road when there is enough rail capacity in this

.c
Industrial Revolution of the 19th century.
sector

o
(a) naturally (b) usually

m
(a) ideology (b) judgement
(c) rationale (d) politics (c) literally (d) specially
(e) logistics (e) markedly
19. Automobile manufacturers are reviving up to launch a 29. The word gharana points to the ...... concepts of stylistic
compaign designed to increase consumer ........ about the individuality and handing down of tradition within family
new emmission control. confines.
(a) production (b) education (a) joint (b) conflicting
(c) capacity (d) knowledge (c) dual (d) contradictory
(e) awareness (e) extraordinary
20. His logic ......... everyone, including the expects. 30. It ws the help he got from his friends which ....... him through
(a) teased (b) defied
the tragedy.
(c) surprised (d) confounded
(e) overwhelmed (a) helped (b) boosted
21. The factory went into a state of suspended ....... today with (c) perked (d) supported
all its workers on strike. (e) sustained
(a) symbiosis (b) animation 31. The criminals managed to escape from the prison even
(c) ways (d) condition through two armed policemen were ....... vigil over them.
(e) mortification (a) taking (b) putting
22. It is not fair to cast ....... on honest and innocent persons. (c) guarding (d) keeping
(a) aspiration (b) aspersions (e) looking
(c) inspiration (d) adulation 32. The speaker did not properly use the time as he went on .....
(e) None of these on one point alone.
23. You must ....... your career with all seriousness. (a) dilating (b) devoting
(a) direct (b) complete
(c) deliberating (d) diluting
(c) follow (d) manage
(e) pursue (e) distributing
24. The villagers .......... the death of their leader by keeping all 33. Ravi had to drop his plan of going to picnic as he had certain
the shops closed. ..... to meet during that period.
(a) announced (b) protested (a) preparations (b) observations
(c) mourned (d) consoled (c) urgencies (d) transactions
(e) None of these (e) commitments
25. These medicines are ........ for curing cold. 34. Even in today’s modern society, people ....... god to bring
(a) proper (b) real rains.
(c) effective (d) capable (a) provoke (b) evoke
(e) powerful (c) propitiate (d) superimpose
26. The poor ones continue to ........... out a living inspite of
(e) None of these
economic liberalisation in that country.
35. The good is often ....... with their bones.
(a) find (b) go
(c) bring (d) manage (a) buried (b) covered
(e) None of these (c) exhumed (d) interred
(e) fleshed

17. a b c d e 18. a b c d e 19. a b c d e 20. a b c d e 21. a b c d e


RESPONSE 22. a b c d e 23. a b c d e 24. a b c d e 25. a b c d e 26. a b c d e
GRID 27. a b c d e 28. a b c d e 29. a b c d e 30. a b c d e 31. a b c d e
32. a b c d e 33. a b c d e 34. a b c d e 35. a b c d e
y
o
u
rs
m
a
h
b
60

o
o
b
Similar

.w
o
rd
p
Substitution

re
s
s
.c
o
m
Max. Marks : 25 No. of Qs. 25 Time : 20 min. Date : ........./......../................

DIRECTIONS : In each of these questions, two sentences (I) 6. I. The truck stopped ___________.
and (II) are given. Each sentence has a blank in it. Five words (a), II. We take a ___________ walk every day.
(b), (c), (d) and (e) are suggested. Out of these, only one fits at (a) suddenly (b) long
both the places in the context of each sentence. Option of that (c) short (d) distant
word is the answer. (e) near
1. I. He is__________ with whatever little he has. 7. I. I got the grains ___________ in the machine.
II. I do not have any ___________ for doubting him.
II. They kept the ____________ of the communication
(a) done (b) basis
a secret.
(c) ground (d) crushed
(a) happy (b) matter
(e) tune
(c) gist (d) content
8. I. We were asked to design a ___________ of the dam.
(e) sense
II. This Institute is a ___________ of modern thinking.
2. I. It is hard to believe the ___________ of operations
(a) picture (b) type
involved in this activity. (c) function (d) fabric
II. The map is drawn to a ___________ of 1 inch to 50 (e) model
km. 9. I. Keep a ___________ grip on the railing.
(a) magnitude (b) size II. He was ___________ asleep.
(c) scale (d) proportion (a) fast (b) firm
(e) significance (c) deep (d) strong
3. I. Heavy snow did ___________ the rescue efforts. (e) sure
II. The food was kept in a ___________. 10. I. He asked me to ___________ over the fence.
(a) delay (b) bundle II. We should keep the valuables in the ___________.
(c) basket (d) hamper (a) vault (b) cross
(e) holder (c) safe (d) tie
4. I. They left ___________ after breakfast. (e) locker
II. It is difficult to find a ___________ person for this job. 11. I. He has now become a ___________ to reckon with.
(a) right (b) immediately II. It is better not to use ___________ to prove one’s point.
(c) suitable (d) best (a) model (b) force
(e) soon (c) coercion (d) name
5. I. He would always do ___________ was told by his (e) influence
superiors. 12. I. He tried his ___________ best to score distinction in
II. He appeared on stage ___________ a narrator of the this exam.
drama. II. It pays to keep ___________ head in an emergency.
(a) cool (b) utmost
(a) as (b) what
(c) very (d) possible
(c) about (d) whatever
(e) level
(e) always

1. a b c d e 2. a b c d e 3. a b c d e 4. a b c d e 5. a b c d e
RESPONSE 6. a b c d e 7. a b c d e 8. a b c d e 9. a b c d e 10. a b c d e
GRID
11. a b c d e 12. a b c d e
y
o
u
rs
m
a
h
128 SPEED TEST 60

b
o
o
13. I. The system is working with ___________ to getting (a) edition (b) volume

b
.w
things done. (c) channel (d) frequency

o
II. ___________ must be commanded and not (e) pitch

rd
demanded. 20. I. It helps to rinse one’s mouth early morning with a

p
re
(a) status (b) relation
___________ of salt and water.

s
(c) attitude (d) respect

s
II. You can always refer to this reference material to find

.c
(e) honour

o
the ___________ to these problems.

m
14. I. There is a complaint against him that he ___________
the mistakes of his juniors. (a) mixture (b) answers
II. A good thing about this house is that it ___________ (c) liquid (d) fix
the sea. (e) solution
(a) ignores (b) promotes 21. I. Our office decided to organize a party for the
(c) examines (d) overlooks ……………….. couple.
(e) faces II. She …………… him in conversation to while away
15. I. Out of the total loans ___________ by the bank, the some time.
largest share was for infrastructure. (a) new (b) engaged
II. The trees are ___________ throughout the area.
(c) pledged (d) held
(a) disbursed (b) covered
(e) encountered
(c) distributed (d) spanned
22. I. It is required that you fill out these two ………………..
(e) extended
16. I. Boats take more time going against the ___________ to register for the job.
of the river. II. This subject has many practical ……………......... in day
II. She keeps herself abreast of ___________ events. to day life.
(a) low (b) latest (a) forms (b) applications
(c) water (d) all (c) relevance (d) statements (e) views
(e) current 23. I. The next ……………….. of this case is after two months.
17. I. While trying to open the door, the ___________ broke. II. After the accident her ……………….. has been affected.
II. It is not difficult to ___________ tricky situations. (a) dates (b) balance
(a) handle (b) knob
(c) evidence (d) health
(c) bracket (d) overcome
(e) hearing
(e) win
18. I. This course teaches you not to ___________ to 24. I. As a last ………………. we had to accept these terms
temptations. and conditions.
II. We hope to increase our ___________ of rice this II. This place has become a good tourist ………… now.
year. (a) resort (b) step
(a) succumb (b) produce (c) attraction (d) spot (e) means
(c) yield (d) share 25. I. The organization decided on an hourly minimum
(e) submit …………………….. of ` 35.
19. I. When you play your radio at high ___________ it II. Will he ……………a war on these fronts to improve
disturbs others. things?
II. We have just received a latest ___________ of this
(a) pay (b) declare (c) pose
encyclopedia.
(d) wage (e) campaign

13. a b c d e 14. a b c d e 15. a b c d e 16. a b c d e 17. a b c d e


RESPONSE 18. a b c d e 19. a b c d e 20. a b c d e 21. a b c d e 22. a b c d e
GRID 23. a b c d e 24. a b c d e 25. a b c d e
y
o
u
rs
m
a
h
b
61

o
o
Correct Usage of

b
.w
o
rd
Preposition

p
re
s
s
.c
o
m
Max. Marks : 35 No. of Qs. 35 Time : 20 min. Date : ........./......../................

DIRECTIONS : Select the correct word or phrase to complete a (a) with, than (b) within, to
grammatical sentence. In case of more than one blank, the different (c) within, against (d) on, from
words given in the options shall fill in the corresponding order. (e) None of these
1. The patient was cheered ........ by the news that she was 9. I doubt whether the accommodation in that place is
likely to be discharged in a day or two. adequate........our needs.
(a) on (b) up (a) according to (b) with
(c) out (d) down (c) for (d) against
(e) None of these (e) None of these
2. The thieves had driven ten miles before the police caught 10. The teacher warned the students to desist ........making noise.
........them. (a) from (b) about
(a) on with (b) upon (c) with (d) by
(c) up to (d) up with (e) None of these
(e) None of these 11. It was customary ........ devotees going to that temple to take
3. The way he is currying favour ........ his rich neighbour is a bath in the nearby tank first.
sickening. (a) with (b) on
(a) of (b) with
(c) for (d) about
(c) for (d) to (e) None of these
(e) None of these
12. Absorbed ........ his own thought, he paid scant attention to
4. Because of his distrust........ every one, he could never gain
what was happening ........ him.
any good friend.
(a) with, about (b) in, around
(a) by (b) of
(c) by, to (d) of, besides
(c) with (d) in
(e) None of these
(e) None of these
13. Though young, he proved a worthy rival ........ his opponent
5. The new Twenty Point Programme is designed to be a
renewed assault ........ poverty. who had much more experience.
(a) for (b) into (a) to (b) against
(c) on (d) upon (c) for (d) with
(e) None of these (e) None of these
6. If you have a good project but are lacking ........finance, you 14. He was of a charitable disposition, but did not like a number
should enter into partnership ........an affluent person. of his relatives trying to live ........ him without trying to earn
their living.
(a) of, with (b) for, with
(a) with (b) near
(c) in, with (d) in, of
(c) off (d) through
(e) None of these
(e) None of these
7. He was operated ........ an abscess in his leg.
15. The by-election, ........ closely fought, resulted ........ a surprise
(a) for (b) against
landslide for the ruling party.
(c) on for (d) on against
(a) if, at (b) since, with
(e) None of these
(c) whether, in (d) though, in
8. Living a simple life ........ one’s won resources is preferable
(e) None of these
........ leading a luxurious life on borrowed funds.

1. a b c d e 2. a b c d e 3. a b c d e 4. a b c d e 5. a b c d e
RESPONSE
6. a b c d e 7. a b c d e 8. a b c d e 9. a b c d e 10. a b c d e
GRID 11. a b c d e 12. a b c d e 13. a b c d e 14. a b c d e 15. a b c d e
y
o
u
rs
m
a
h
130 SPEED TEST 61

b
o
o
16. You cannot be too sensitive........ criticism, if you have chosen 26. His claims of close acquaintance........ the high and the mighty

b
a political career.

.w
are unbelievable.
(a) about (b) with

o
(a) about (b) towards

rd
(c) for (d) to (c) with (d) from

p
(e) None of these (e) None of these

re
17. ........ we are good friends his views differ ........mine on many

s
27. Compared........China, India’s progress in the agricultural field

s
issues.

.c
has been commendable.

o
(a) Despite, with (b) Because, off (a) against (b) over

m
(c) Although, from (d) Nevertheless, form
(c) towards (d) to
(e) None of these
(e) None of these
18. A peculiar custom prevailing........ Toads is the sacrifice of
28. The court found him guilty and imposed ........ him a fine of
buffaloes on ceremonial occasions.
Rs.500.
(a) over (b) with
(c) by (d) among (a) to (b) for
(e) None of these (c) upon (d) against
19. However poor one may be, one can be happy only if one has (e) None of these
the right attitude to life; happiness lies........ contentment. 29. It is believed that hypertension is most often the cause ........
(a) through (b) in heart attack.
(c) with (d) over (a) towards (b) with
(e) None of these (c) in (d) of
20. Being himself very quick ........ arithmetical calculations, he (e) None of these
did not need a calculator. 30. Ravi was ........ Nagpur ........ 2nd Jan, 67 ........4 in the morning.
(a) ahout (b) for (a) at, on, at (b) in, on, at
(c) over (d) in (c) in, in, about (d) at, at, at
(e) None of these (e) None of these
21. Though accused of partiality........ his home team, the umpire 31. .............. a very long time this city has been prosperous.
had a clear conscience and believed in the correctness of (a) Since (b) For
his decisions. (c) From (d) Till
(a) with (b) towards (e) None of these
(c) for (d) against 32. The mother of the dead child was overwhelmed .............
(e) None of these
grief.
22. Though I would not recommend it, I have no objection........
(a) by (b) with
your going to that movie.
(c) from (d) for
(a) for (b) against
(c) to (d) upon (e) None of these
(e) None of these 33. There is no use discussing .............. prohibition.
23. Do not force me to ........ you on this issue; I am not at all (a) on (b) about
convinced. (c) of (d) for
(a) agree upon (b) concur with (e) None of these
(c) join over (d) equate with 34. .............. business, a merger is a combination of two or more
(e) None of these corporations under one management.
24. The court acquited him ........ all the charges. (a) At (b) In
(a) from (b) against (c) The (d) On
(c) for (d) of (e) None of these
(e) None of these 35. How is life enhanced? A beautiful passage from Tagore
25. There is no point in counting........ his support he is comes .............. mind.
notoriously undependable. (a) in (b) to
(a) for (b) on (c) from (d) out
(c) with (d) through
(e) None of these
(e) None of these

16. a b c d e 17. a b c d e 18. a b c d e 19. a b c d e 20. a b c d e


RESPONSE 21. a b c d e 22. a b c d e 23. a b c d e 24. a b c d e 25. a b c d e
GRID 26. a b c d e 27. a b c d e 28. a b c d e 29. a b c d e 30. a b c d e
31. a b c d e 32. a b c d e 33. a b c d e 34. a b c d e 35. a b c d e
y
o
u
rs
m
a
h
b
62

o
Sentence

o
b
.w
o
rd
Improvement

p
re
s
s
.c
o
m
Max. Marks : 30 No. of Qs. 30 Time : 20 min. Date : ........./......../................

DIRECTIONS (Qs.1-20) : Which of the following phrases (a), 8. Belonged to this cadre, you are eligible for facilities such as
(b), (c) and (d) given below each sentences should replace the free air travel and accommodation.
phrase printed in bold in the sentence to make it grammatically (a) Since you belong to (b) Whoever belongs
correct? If the sentence is correct as it is given and ‘No correction (c) For belonging to (d) To belong in
is required’, mark (e) as the answer. (e) No correction required
1. Starting out my own business at this time would affect the 9. The bank has hired a consultant who will look into any issues
financial stability of my family. which arise during the merger.
(a) is looking over (b) will be looked after
(a) Starting up my (b) For starting with
(c) will look out (d) looks down on
(c) To start out mine (d) By starting my
(e) No correction required
(e) No correction required
10. I had severe doubts about if I successfully run a company,
2. Use a tactic for mixing the inferior with good quality rice is but my father encouraged me.
dishonest and you will lose your license. (a) if I am successful in (b) how should I successfully
(a) Using tacti as (b) Using a tactic like (c) whether I would successfully
(c) To use tactics (d) Used to tactics like (d) that I would succeed to
(e) No correction required (e) No correction required
3. The company will invest more six hundred crores in the 11. As it was a dark and stormy night, Lata was too scared to go
next five years to expand its operations in Britain. home alone.
(a) will future invest (b) has invested more than (a) very scary to (b) much scared to
(c) have invested over (d) will be invested above (c) as scared to (d) to scared too
(e) No correction required (e) No correction required
4. Several of our projects have delayed because the equipment 12. Since it was her engagement party, Riya was dress to kill.
we ordered was delivered late. (a) dresses to kill (b) dressed to kill
(a) have been delayed when (c) dressed to killings (d) dressing to killed
(b) delayed because of (e) No correction required
(c) are delayed since (d) were delayed with 13. Ramesh worries endlessly about his son’s future as he was
(e) No correction required so poor in studies.
5. The committee has ruled out the possible raising taxes for (a) worry endless (b) worried endless
this financial year. (c) worried endlessly (d) worries endless
(a) possibly raised (b) possible rise of (e) No correction required
14. Now that the actual criminal had been caught, Kunal was
(c) possibility to raise (d) possibility of raising
happy that he was finally let of the hook.
(e) No correction required
(a) off the hook (b) of the hookings
6. The company has set up a foundation which helps students
(c) off the hooks (d) of the hooks
who do not have the necessary funds to study ahead. (e) No correction required
(a) further to study (b) of studying more 15. The little boy appeared all of a sudden out of nowhere and
(c) to study onward (d) for higher studies take everyone by surprise.
(e) No correction required (a) took everyone as surprised
7. If this land is used to cultivate crops it will be additionally (b) take everyone with surprised
source of income for the villagers. (c) took everyone by surprises
(a) a source of additional (b) an additionally source (d) took everyone by surprise
(c) an additional source (d) additionally the source (e) No correction required
(e) No correction required

1. a b c d e 2. a b c d e 3. a b c d e 4. a b c d e 5. a b c d e
RESPONSE 6. a b c d e 7. a b c d e 8. a b c d e 9. a b c d e 10. a b c d e
GRID
11. a b c d e 12. a b c d e 13. a b c d e 14. a b c d e 15. a b c d e
y
o
u
rs
m
a
h
132 SPEED TEST 62

b
o
o
16. A young and successful executive was travelling down a 23. The course of events made it necessary for Joseph to start working.

b
neighbourhood street, going a bit to fast in his new car. (a) events that were planned.

.w
(a) a bit too fastly (b) a bit as fast (b) long list of future events.

o
rd
(c) a bit to fastly (d) a bit too fast (c) A succession of unexpected events.

p
(e) No correction required (d) nature of events that followed after Joseph joined work.

re
17. All she could think about was the beautiful dress and how (e) None of these

s
s
she could earn enough money to buy it. 24. The new law on “Right to Food Safety” will come into force

.c
(a) All she can think (b) All she could thought next month.

o
m
(c) All she can thought (d) All she can thinking (a) be forced upon the people.
(e) No correction required (b) be associated from next month onwards.
18. He told his employer of his plans to leave the business to (c) be implemented next month.
lead a more leisure life. (d) be withdrawn next month.
(a) more leisurely life (b) many leisurely life (e) be widely rejected next month.
(c) many leisured life (d) more leisurely live 25. When the girl wanted to stay out past midnight, her father
(e) No correction required put his foot down.
19. Padma could convince anyone with her talks as she had the (a) gave in to her request
gift of the gabbing. (b) walked away disapprovingly.
(a) gifting of the gabbing (b) gift of the gab (c) obstructed her from leaving the house.
(c) gifting of the gab (d) gift of the gab (d) requested her to be home on time.
(e) No correction required (e) None of these
20. For countries undergoing a recession, large cuts in public 26. In all likelihood the missing boy has run away to the forest
spending seem to be the ordering of the day. (a) with good intentions (b) there's no chance
(a) be the ordering of days (c) without doubt (d) in most probability
(b) being the order of the day (e) None of these
(c) be the order of the day 27. The parents were completely in the dark concerning their
(d) being the ordering of days daughter's plans.
(e) No correction required (a) ignorant about (b) ashamed of
DIRECTIONS (Qs.21-30) : In each of the following sentences, (c) pretending to be unaware
an idiomatic expression or a proverb is highlighted. Select the (d) unhappy about (e) None of these
alternative which best describes its use in the sentence. 28. I am in touch with the police, and they will be here in ten minutes.
21. The team put their plan into execution the very next day. (a) in communication with
(a) proposed a plan. (b) in close proximity with
(b) discussed their plan. (c) in good terms with (d) familiar with
(c) started thinking about a plan (e) None of these
(d) started carrying out their plan. 29. I stumbled upon some interesting old letters in my
(e) None of these Grandfather's desk.
22. Mrs. Nayak opened the discussion on the “alarming rate of (a) deliberately went through
poverty in India”. (b) surveyed (c) tripped over
(a) started the discussion. (d) discovered by chance (e) None of these
(b) gave her opinion in the discussion. 30. The secretary made an entry of the arrangement.
(c) did not agree on the discussion. (a) initiated discussion (b) made a record
(d) welcomed the people to the discussion. (c) brought notice (d) showed approval
(e) None of these (e) None of these

16. a b c d e 17. a b c d e 18. a b c d e 19. a b c d e 20. a b c d e


RESPONSE
21. a b c d e 22. a b c d e 23. a b c d e 24. a b c d e 25. a b c d e
GRID 26. a c d e 27. a c d e 28. a c d e 29. a c d e 30. a c d e
b b b b b
y
o
u
rs
m
a
h
b
63

o
o
Spotting the

b
.w
o
rd
Errors - I

p
re
s
s
.c
o
m
Max. Marks : 41 No. of Qs. 41 Time : 25 min. Date : ........./......../................

DIRECTIONS (Qs.1-41) : Read each sentence to find out whether 10. The agreement on (a)/ which all of us have (b)/ worked so
there is any grammatical error or idiomatic error in it. The error, if hard will (c)/ be sign tomorrow (d). No error (e).
any, will be in one part of the sentence. The number of that part 11. Nuclear waste will still being (a)/ radioactive even after twenty
is the answer. If there is no error, the answer is (e). (Ignore errors thousand years, (b)/ so it must be disposed (c)/ of very
of punctuation, if any.) carefully. (d)/ No error (e)
1. His proposal had (a) / to be send to (b) / the President of 12. My friend lived at the top (a)/ of an old house (b)/ which attic
the company (c) / for her approval (d). No error (e). had been (c)/ converted into a flat. (d)/ No error (e)
2. Each tuesday evening we visited (a) / the farmers in the 13. A public safety advertising (a)/ campaign in Russia (b)/ hope
area (b) / and held a meeting (c) / to discuss the problems to draw attention (c)/ of pedestrians crossing the road.
they faced (c). No error (e). (d)/ No error (e)
3. Though our training facilities (a) / are limited only a (b) / 14. A cash prize was (a)/ award to the most (b)/ successful salesman
few employees have been (c) / selected for training (d). No of the year (c)/ by the President of the company. (d)/ No error
error (e). (e)
4. During the interview (a) / the panel asked me (b) / several 15. The Renaissance was (a)/ a time to ‘re-awakening’ (b)/ in both
technical questions (c) / and I answered all of it (d). No the arts (c)/ and the sciences. (4)/ No error (e)
error (e). 16. In times of crisis, (a) / the Bhagavad Gita gives light (b)/ and
5. He decided to work for (a) / an NGO, but most of his (b) / guide to the mind tortured by doubt (c)/ and torn by conflict
classmates opted for high paid (c) / jobs in multinational of duties. (d)/ No Error (e)
companies (d). No error (e). 17. It was not easy for late Raja Ram Mohan Roy (a)/ to root out
6. It is necessarily to maintain (a)/ a record of all transactions the custom of sati (b) / because a majority of (c)/ the educated
(b) / in case the auditors (c)/ want to see it. (d)/ No error (e). class does not support him. (d) / No Error (e)
7. Very few young trainees (a)/ willingly undertake (b)/ a 18. Deplete of the Ozone layer (a) / and the greenhouse effect (b)
posting to a branch (c)/ located in a rural area (d)/ No error / are two long-term effects (c)/of air pollution. (d)/ No Error (e)
(e). 19. Most of the people which (a)/ have been victims (b) / of extreme
8. He has travelled (a)/ all over the world (b)/ yet he speaks violence (c)/ are too frightened to report it to the police. (d)/
(c)/ several languages fluently (d)/. No error (e). No Error (e)
9. A successful company is (a)/ any that makes a good (b)/ 20. The doctor helps (a)/ to reducing human suffering (b)/by
profit and provides (c)/ high returns to its shareholders (d). curing diseases (c)/ and improving health. (d)/ No Error (e)
No error (e).

1. a b c d e 2. a b c d e 3. a b c d e 4. a b c d e 5. a b c d e
RESPONSE 6. a b c d e 7. a b c d e 8. a b c d e 9. a b c d e 10. a b c d e
GRID 11. a b c d e 12. a b c d e 13. a b c d e 14. a b c d e 15. a b c d e
16. a b c d e 17. a b c d e 18. a b c d e 19. a b c d e 20. a b c d e
y
o
u
rs
m
a
h
134 SPEED TEST 63

b
o
o
21. The shepherd counted (a) / his sheep and found (b) / that 32. We may have to await for (a) / a new political revival (b) / to

b
.w
one of (c) / them is missing. (d) / No Error (e) eradicate the (c) / corruption from our economy. (d) / No

o
22. The teacher were (a) / impressed by her performance (b) / Error (e)

rd
and asked her to (c) / participate in the competition. (d) No 33. When she was (a) / in jail (b) / she was debarred to send

p
re
Error (e) (c) / a letter even to her son. (d) / No Error (e)

s
s
23. She asked her (a) / son for help her (b) / find a place to bury 34. Despite of the best efforts (a) / put by the doctors (b) / the

.c
o
(c) / the gold ornaments (d) No Error (e) condition of the patient (c) / is deteriorating from bad to

m
24. The painter was (a) / ask to paint a (b) / picture of the king, worse. (d) / No Error (e)
(c) / sitting on his throne (d) No Error (e) 35. The militant yielded for (a) / the temptation and fell (b) / into
25. The story was (a) / about how an (b) / intelligent man had the trap (c) / of police. (d) / No Error (e)
saving (c) / himself from being robbed (d) No Error (e) 36. Many people in India (a) / are dying from hunger (b) / but
26. The decline of her moral (a) / was caused by a lot of (b) / government seems (c) / to be ignorant of such crude fact. (d)
factors that were once (c) / fascinating to her. (d) / No Error / No Error (e)
(e) 37. In difficult time (a) she prefers keeping her counsel (b) rather
27. He took me to a restaurant (a) / and ordered for two cups than wandering (c) / here and there for relief. (d) / No Error
(b) / of cold coffee (c) / which the waiter brought in an hour. (e)
(d) / No Error (e) 38. The persons who are (a) / suffering from diabetes are (b) /
28. There are some animals (a) / that can live (b) / both in water advised to substitute (c) / saccharine by sugar. (d) / No Error
an land (c) / without any difficulty. (d) / No Error (e) (e)
29. During his tour (a) / to the south (b) he visited not only to 39. He always says (a) / that he prefers to go (b) / home to stay
Chennai (c) / but also Karnataka. (d) / No Error (e) in (c) / a hotel at night. (d) / No Error (e)
30. The President Mr. Kalam (a) / is much sought after (b) / by 40. Hardly had we settled down (a) / for the rest (b) / when we
school students and (c) / is invited for many functions. were startled by the (c) / strange sound of trumpets. (d) / No
(d) / No Error (e) Error (e)
31. His mother is not well (a) / but he (b) / does not look for her 41. He was able to (a) / free himself with (b) / the debts by (c) /
(c) / properly. (d) / No Error (e) working day and night. (d) / No Error (e)

21. a b c d e 22. a b c d e 23. a b c d e 24. a b c d e 25. a b c d e


26. a b c d e 27. a b c d e 28. a b c d e 29. a b c d e 30. a b c d e
RESPONSE 31. a c d e 32. a c d e 33. a c d e 34. a c d e 35. a c d e
b b b b b
GRID 36. a b c d e 37. a b c d e 38. a b c d e 39. a b c d e 40. a b c d e
41. a b c d e
y
o
u
rs
m
a
h
b
64

o
o
b
Spotting the

.w
o
rd
p
Errors - II

re
s
s
.c
o
m
Max. Marks : 40 No. of Qs. 40 Time : 25 min. Date : ........./......../................

DIRECTIONS : Read each sentence to find out whether there is 11. All three products help wean smokers (a) / from cigarettes by
any error in it. The error, if any, will be in one part of the sentence. providing small doses (b) / of nicotine designed to replace
The number of this part is the answer. If there is no error, the the basic level (c) / of average smoker's day's nicotine
answer is (e). consumption. (d) No error (e)

1. The banker’s association (a) / has submitted a memorandum 12. The main lesson is that (a) / the mosquito eradication
(b) / for the fulfilment of (c) / their demands. (d) / No Error campaigns (b) / such as the one the French conducted (c) /
(e) needs to be followed up. ( d) / No error (e)
13. By arresting the local criminals (a) / and encouraging good
2. Five quintals of wooden coal (a) / are (b) / his annual
people, (b) / we can end (c) / hostilities of that area (d) / No
requirement (c) / for the unit. (d) / No Error (e)
error (e)
3. Dickens have (a) / vehemently criticised (b) / the philosophy
14. We admired thre way(a) / he had completed all his work (b) /
(c) in ‘Hard Times’. (d) / No Error (e)
and appreciation the method (c) / adopted by him. (d) / No
4. All his money (a) / is spent (b) / and all his (c) / hopes error
ruined. (d) / No Error (e)
15. I was being astonished (a) / when I heard that (b) / he had left
5. This rule may (a) / and ought to be (b) / disregarded for (c) the country (c) / without informing anyone of us . (d) / No.
/ the time being. (d) / No Error (e) error (e)
6. Why come people don't get (a) / what they deserve (b) / 16. The Head of the Department, along with his colleagues (a) /
and why others get what they don't deserve (c) / is a matter are coming to attend (b) / the conference which is (c) /
decided by luck. (d)/No error(e) scheduled this afternoon. (d) / No error (e)
7. The committee is thankful to Mr. Roy (a) / for preparing not 17. Govind loved his Guru immensely (a) / and gave him fullest
only the main report (b) / but also for preparing (c) the loyalty, (b) / yet he had his own (c) / independent way of
agenda notes and minutes. (d) / No error (e) thinking (d) / No error (e)

8. In order to save petrol (a ) / motorists must have to (b) / be 18. In a very harsh tone, (a) / he shouted at his servants (b) / and
very cautious (c) / while driving along the highways. (d) / told them that (c) / he does not neEid their services (d) ./ No
No error (e) error (e)

9. No country can long endure (a) / if its foundations (b) / 19. This is an important difference in (a) / that it marks the first
were not laid deep (c) / in the material prosperity. (d) / No move towards (b) / an institution for money - earning
error (e) proposition. (d) / No error (e)

10. Due to certain inevitable circumstances (a) / the scheduled 20. The ultimate problem of physics (a)/ is to reduce matter by
programme had to be (b) / post poned indefinite (c) / but analysis (b) / to its lowest condition of divisibility (c) / No.error
(d)
the members could not be informed. (d) / No error (e)

1. a b c d e 2. a b c d e 3. a b c d e 4. a b c d e 5. a b c d e
RESPONSE 6. a b c d e 7. a b c d e 8. a b c d e 9. a b c d e 10. a b c d e
GRID 11. a b c d e 12. a b c d e 13. a b c d e 14. a b c d e 15. a b c d e
16. a b c d e 17. a b c d e 18. a b c d e 19. a b c d e 20. a b c d e
y
o
u
rs
m
a
h
136 SPEED TEST 64

b
o
o
21. It was astonished (a) / to find that I scarcely had (b) / enough 32. Whether this happens, and whether the BIFR will (a) / once

b
money (c) / to pay the bills. (d) / No error (e) again reconsiger the Sirmour package (b) / are questions

.w
those will be (c) / answered in the comming months. (d) / No

o
22. We can not handle (a)./ this complicated case to day (b) /

rd
error (e)
unless full details are not given (c) / to us by now. (d) / No

p
re
error (e) 33. We now look forward for (a) / some great achievements (b) /

s
which to some extent (c) / can restore the country's prestige

s
23. Even after worked in the office (a) / for as many as fifteen

.c
once again (d) / No error (e)
years, (b) / he still does not understand (c) / the basic

o
m
objectives of the work. (d) / No error (e) 34. Honesty and integrity are (a) / the qualityes which cannot
be (b) / done away with (c) / and hence assume a lot of
24. Neither the earthquake (a) / nor the subsequent fire ( b) /
importance (d) / No error (e)
was able to dampen (c) / the spirit of the residents, (d) / No
error (e) 35. The foreign funds are cheaper than those (a) / available the
domestic market and (b) / the company is competent that (c)
25. Our school is making (a) / every possible effort (b) / to provide
/ it will soon touch its earlier annual turnover, (d) / No error
best facilities (c) / land person attention for each child (d) /
(e)
No error (e)
36. Honesty, integrity and being intelligent (a) / are the qualities
26. Our neighbours had repeated (a) / the same illogical sequence
which (b) / we look for when (c) / we interview applicants (d)
of activites (b) / if we had not brought the (c) /facts to their
/ No error (e)
notice. ( d) / No error (e)
37. Not only the judges acquitted (a) / him of all the charges (b)
27. This has forced them to focus at (a) I how to reach and serve
/ levelled against him, but (c) / also commended all his actions.
their customers, (b) / rather than, say, pumping money (c) /
(d) /No error (e)
into fancy graphics that look good in management meetings.
(d) / No error (e) 38. One of the most effective (a) / solutions is that (b) / she
should work on Sunday (c) / and complete the assignment.
28. The loss of forests thus entail (a) / large social and economic
(d) / No error (e)
costs; (b) the lives of more than one billion people (c) / are
already affected by loss of this forest cover (d) / No error (e) 39. Our system of assigning (a) I different jobs to different people
(b) / should be based on (c) / their strengths and weaknesses
29. Not one of the children (a) / has ever sang (b) / on any
(d) / No error (e)
occasion (c) / in public before. (d) / No error (e)
40. There he stood on the dais, (a) / debunked the manner in
30. Reasonable ambition, if supported (a) / at persistent efforts,
which the company was run ( b) / by the former chairman
(b) / is likely to yield (c) / the desied results. (d) / No error (e)
and managing director, (c) / and promising higher growth
31. I would have lost (a) / my luggage and other belonging (b) / and industry. (d) No error (e)
if I would have left the compartment (c) / and gone out to
fetch drinking water. (d) / No errro (e)

21. a b c d e 22. a b c d e 23. a b c d e 24. a b c d e 25. a b c d e


RESPONSE 26. a b c d e 27. a b c d e 28. a b c d e 29. a b c d e 30. a b c d e
GRID 31. a b c d e 32. a b c d e 33. a b c d e 34. a b c d e 35. a b c d e
36. a b c d e 37. a b c d e 38. a b c d e 39. a b c d e 40. a b c d e
y
o
u
rs
m
a
h
b
65

o
o
b
.w
Spelling Test

o
rd
p
re
s
s
.c
o
m
Max. Marks : 40 No. of Qs. 40 Time : 25 min. Date : ........./......../................
DIRECTIONS: Choose the correct spelling of the given word. 11. (a) Entreprenuer (b) Entrepraneur
1. (a) Coimmission (b) Comision (c) Entrapreneur (d) Entrepreneur
(c) Comission (d) Commision (e) None of these
(e) None of these 12. (a) Skillful (b) Skillfull
2. (a) Jewelery (b) Jewellry (c) Skilful (d) Skilfull
(c) Jwellry (d) Jewellery (e) None of these
(e) None of these 13. (a) Varstile (b) Verstile
3. (a) Sattellite (b) Satellite (c) Versatile (d) Vorstyle
(c) Sattelite (d) Satelite (e) None of these
(e) None of these 14. (a) Correspondant (b) Corraspondent
4. (a) Ocasion (b) Ocassion (c) Corraspondant (d) Correspondent
(c) Occasion (d) Occassion (e) None of these
(e) None of these 15. (a) Etiquete (b) Etiquette
5. (a) Comettee (b) Committe (c) Ettiquete (d) Ettiquette
(c) Comittee (d) Committee (e) None of these
(e) None of these 16. (a) Necessary (b) Necesarry
6. (a) Achievment (b) Acheivment (c) Necesary (d) Neccessary
(c) Achievement (d) Achevement (e) None of these
(e) None of these 17. (a) Homeopathy (b) Homoepathy
7. (a) Hetrogenous (b) Hetrogeneous (c) Homiopathy (d) Homoeopathy
(c) Heterogenous (d) Heterogeneous (e) None of these
(e) None of these 18. (a) Lieutinant (b) Lieutenant
8. (a) Foreigner (b) Forienor (c) Leutenant (d) Liutenant
(c) Foriegnor (d) Foreiner (e) None of these
(e) None of these 19. (a) Paralelogram (b) Paralellogram
9. (a) Colaboration (b) Collaberation (c) Parallelogram (d) Parallellogram
(c) Colaberation (d) Collaboration (e) None of these
(e) None of these 20. (a) Milennium (b) Millenium
10. (a) Acurrate (b) Accurate (c) Millennium (d) Milenium
(c) Acurate (d) Accuratte (e) None of these
(e) None of these

1. a b c d e 2. a b c d e 3. a b c d e 4. a b c d e 5. a b c d e
RESPONSE 6. a b c d e 7. a b c d e 8. a b c d e 9. a b c d e 10. a b c d e
GRID 11. a b c d e 12. a b c d e 13. a b c d e 14. a b c d e 15. a b c d e
16. a b c d e 17. a b c d e 18. a b c d e 19. a b c d e 20. a b c d e
y
o
u
rs
m
a
h
138 SPEED TEST 65

b
o
o
DIRECTIONS (Qs. 21-40) : In each question below a sentence 30. Though the government initiated (a)/ a large sum (b)/ of

b
.w
with four words printed in bold type is given. These are numbered money in the scheme (c)/ it was a failure (d)/. All correct (e).

o
as (a), (b), (c) and (d). One of these four words printed in bold may 31. It is not unusual (a)/for guests of the hotel to carry(b)/

rd
be either wrongly spelt or inappropriate in the context of the

p
souveniers(c)/ back with them when they return (d)/to their

re
sentence. Find out the word which is wrongly spelt or inappropriate homes. All correct (e)

s
s
if any. The number of that word is your answer. If all the words

.c
32. She vested(a)/ her time in chatting(b)/ over the phone and

o
printed in bold are correctly spelt and also appropriate in the ultimately(c)/ ended up not finishing (d)/ her work. All

m
context of the sentence, mark (e) i.e. ‘All correct’ as your answer. correct (e)
21. Under existing (a) / regulations we are not permitted (b) / to 33. She had not eaten(a)/ anything(b)/ for a very long time now
owe (c) / more than a forty percent share (d) / of the family and her stomach(c)/ was groling(d)/ All correct(e)
business. All correct (e). 34. Half of the harm(a)/ that is done in this world(b)/ is due to
22. In case of any land dispute (a) / panchayat officials (b) / will people(c)/ who want to feel important(d)/ All correct(e)
determine (c) / how the property is to be dividend (d) / All 35. Life is like a mirror(a)/ smile at it and its charmeng(b)/
correct (e). frown(c)/ at it and it becomes sinister(d)/ All correct(e)
23. The World Bank has consented (a) / to sanction (b) / the 36. The Whole (a)/ time she walked with her child in her arms
necessary (c) / finance (d) / for the project. All correct (e). the only thing (b)/ that worried (c)/ her was her son’s feature.
24. To obtain (a) / a refund you will have to fill (b) / a claim (c) (d)/ All correct (e)
/ with the appropriate (d) / authority. All correct (e). 37. When the young artist returned (a)/ to his village, his family
held a festive (b)/ dinner on it’s lawn to celebrate his triumpant
25. Experts predict (a) / there will be shortage (b) / of investment
(c)/ homecoming. (d)/ All correct (e)
(c) / in the infrastructure (d) / sector. All correct (c).
38. Had she not suppressed (a)/ all the details of her Company’s
26. In order to succeed (a)/ it is crucial (b)/ for an organisation
project (b)/ her Company would have bagged (c)/ the
to constantly (c)/ improve (d)/. All correct (e). contract. (d)/ All correct (e)
27. With some assistance (a)/ from her son she was enable (b)/ 39. She trusted Mira with all her heart (a)/ and thus handled (b)/
to settle (c)/ her debts (d)/ on time. All correct (e). over her life’s (c)/ savings to her instantly. (d)/ All correct (e).
28. We have prepared a detailed (a)/ report giving various (b)/ 40. It is difficullt (a)/ to see the picture (b)/ when you are inside
solutions (c)/ to resort (d)/ the problem. All correct (e). (c)/ the frame. (d)/ All correct (e)
29. RBI has attempted (a)/ to spend (b)/ financial (c)/ awareness
(d) / through this programme. All correct (e).

21. a b c d e 22. a b c d e 23. a b c d e 24. a b c d e 25. a b c d e


RESPONSE 26. a b c d e 27. a b c d e 28. a b c d e 29. a b c d e 30. a b c d e
GRID 31. a b c d e 32. a b c d e 33. a b c d e 34. a b c d e 35. a b c d e
36. a b c d e 37. a b c d e 38. a b c d e 39. a b c d e 40. a b c d e
y
o
u
rs
m
a
h
b
66

o
o
b
One Word

.w
o
rd
p
Substitution

re
s
s
.c
o
m
Max. Marks : 30 No. of Qs. 30 Time : 20 min. Date : ........./......../................

DIRECTIONS (Qs.1-25) : In each of the following questions, 8. The original inhabitants of a country
out of the four alternatives, choose the one which can be (a) Aborigines (b) Citizens
substituted for the given words/sentence. (c) Natives (d) Primitive
(e) None of these
1. List of headings of the business to be transacted at a
9. One desirous of getting money
meeting
(a) Avaracious (b) Voracious
(a) Schedule (b) Agenda (c) Garrulous (d) Greedy
(c) Proceedings (d) Excerpts (e) None of these
(e) None of these 10. Be the embodiment or perfect example of
2. Through which light cannot pass (a) Signify (b) Characterise
(a) Dull (b) Dark (c) Personify (d) Masquerade
(c) Obscure (d) Opaque (e) None of these
(e) None of these 11. Cutting for stone in the bladder
3. Stealing from the writings of others (a) Dichotomy (b) Tubectomy
(a) Copying (b) Reframing (c) Vasectomy (d) Lithotomy
(c) Reproducing (d) Plagiarism (e) None of these
(e) None of these 12. That which makes it difficult to recognise the presence of real
4. Constant effort to achieve something nature of somebody or something
(a) Perseverance (b) Attempt (a) Cover (b) Mask
(c) Enthusiasm (d) Vigour (c) Pretence (d) Camouflage
(e) None of these (e) None of these
5. A person not sure of the existence of God 13. Yearly celebration of a date or an event
(a) Theist (b) Atheist (a) Centenary (b) Jubilee
(c) Agnostic (d) Cynic (c) Anniversary (d) Birthday
(e) None of these (e) None of these
6. One who deserts his religion 14. One who has suddenly gained new wealth, Power or prestige
(a) Aristocrat (b) Affluent
(a) Deserter (b) Turn-coat
(c) Maverick (d) Parvenu
(c) Fanatic (d) Apostate
(e) None of these
(e) None of these
15. Code of diplomatic etiquette and precedence
7. One who uses fear as a weapon of power
(a) Formalism (b) Statesmanship
(a) Terrorist (b) Militant
(c) Protocol (d) Hierarchy
(c) Extremist (d) Anarchist
(e) None of these
(e) None of these

1. a b c d e 2. a b c d e 3. a b c d e 4. a b c d e 5. a b c d e
RESPONSE 6. a b c d e 7. a b c d e 8. a b c d e 9. a b c d e 10. a b c d e
GRID
11. a b c d e 12. a b c d e 13. a b c d e 14. a b c d e 15. a b c d e
y
o
u
rs
m
a
h
140 SPEED TEST 66

b
o
o
16. Of outstanding significance 24. One who studies mankind

b
.w
(a) Meaningful (b) Ominous (a) Anthropologist (b) Physicist

o
(c) Evident (d) Monumental (c) Pathologist (d) Philanthropist

rd
(e) Rational (e) None of these

p
re
17. One who promotes the idea of absence of government of DIRECTIONS (Qs.26-30) : Choose the one word for the italic

s
any kind, when every man should be a law into himself

s
sentences.

.c
(a) Anarchist (b) Belligerent

o
25. An opinion contrary to popular belief

m
(c) Iconoclast (d) Agnostic
(a) Paradox (b) Orthodoxy
(e) None of these
(c) Hearsay (d) Heresy
18. Land so surrounded by water as to be almost an island
(e) None of these
(a) Archipelago (b) Isthmus
26. Rajesh was a hater of learning and knowledge.
(c) Peninsula (d) Lagoon
(a) misogynist (b) misologist
(e) None of these
19. That which cannot be done without (c) misanthropist (d) bibliophile
(a) Irrevocable (b) Impracticable (e) None of these
(c) Indispensable (d) Impossible 27. The bus has to go back and forth every six hours.
(e) None of these (a) travel (b) run
20. One who travels from place to place (c) cross (d) shuttle
(a) Itinerant (b) Mendicant (e) commute
(c) Journeyman (d) Tramp 28. A man can be sentenced to death for killing another human
(e) None of these being.
21. An act or notion to look back in the past (a) fratricide (b) regicide
(a) Retrospective (b) Postnatal (c) homicide (d) genocide
(c) Retrogressive (d) Primitive (e) None of these
(e) None of these 29. A careful preservation and protection of wildlife is the need
22. Medicine to counteract the effect of a poison of the hour.
(a) Emetic (b) Antidote (a) Embarkment (b) Promotion
(c) Anti-venom (d) Antiseptic (c) Conservation (d) Management
(e) None of these (e) Enhancement
23. A collection of poems 30. The officer was not willing to take a definite stand on that
(a) Pathology (b) Anthology point.
(c) Oncology (d) Pedology (a) vague (b) evasive
(e) None of these (c) ambiguous (d) complex
(e) None of these

16. a b c d e 17. a b c d e 18. a b c d e 19. a b c d e 20. a b c d e


RESPONSE 21. a b c d e 22. a b c d e 23. a b c d e 24. a b c d e 25. a b c d e
GRID 26. a b c d e 27. a b c d e 28. a b c d e 29. a b c d e 30. a b c d e
y
o
u
rs
m
a
h
b
67

o
o
b
.w
o
Para Jumbles

rd
p
re
s
s
.c
o
m
Max. Marks : 30 No. of Qs. 30 Time : 20 min. Date : ........./......../................

DIRECTIONS (Qs. 1 to 5): Rearrange the following five sentences C. The broad–based agitation against SEZs has demonstrated
(A), (B), (C), (D) and (E) in the proper sequence to form a meaningful the power of popular protest in the State.
paragraph, then answer the questions given below them. D. Those opposed to the projects had questioned the propriety
of the government acquiring large tracts of land and then
A. It will take extraordinary political commitment and liberal selling them to promoters at low prices.
public funding during the 11th Plan for affordable housing E. A coastal State with an area of 3,700 square kilometers and a
to become a credible goal. population of about 1.4 million, Goa has always been extremely
B. The National Urban Housing and Habitat Policy of the sensitive to the impact of unrestrained economic development.
United Progressive Alliance Government seeks to make 6. Which of the following should be the FIRST sentence?
access to housing, long acknowledged as a fundamental
(a) A (b) B (c) C
right, a reality for all. (d) D (e) E
C. The task is staggering even if we go by conservative estimates. 7. Which of the following should be the SECOND sentence?
D. The housing shortage to be met during the Plan is 26.53 (a) A (b) B (c) C
million units, which include the backlog from the 10th Plan. (d) D (e) E
E. If the existing stock of poor quality dwellings and the 8. Which of the following should be the THIRD sentence?
growing urbanization–driven demand are taken into (a) A (b) B (c) C
account, the real deficit will be even higher. (d) D (e) E
1. Which of the following should be the FIRST sentence? 9. Which of the following should be the FOURTH sentence?
(a) A (b) B (c) C (a) A (b) B (c) C
(d) D (e) E (d) D (e) E
2. Which of the following should be the SECOND sentence? 10. Which of the following should be the FIFTH (LAST) sentence?
(a) A (b) B (c) C (a) A (b) B (c) C
(d) D (e) E (d) D (e) E
3. Which of the following should be the THIRD sentence?
DIRECTIONS (Qs. 11 to 15): Rearrange the following five sentences
(a) A (b) B (c) C
(A), (B), (C), (D) and (E) in the proper sequence to form a meaningful
(d) D (e) E
paragraph, then answer the questions given below them.
4. Which of the following should be the FOURTH sentence?
(a) A (b) B (c) C A. The British government plans to insist that spouses should
(d) D (e) E have to learn English before they are allowed into Britain to
5. Which of the following should be the FIFTH (LAST) join their husbands or wives have run into a barrage of
sentence? opposition and warnings that the idea could breach human
(a) A (b) B (c) C rights laws.
(d) D (e) E B. The responses to an official consultation on the proposal
published on Thursday was more than two to one against the
DIRECTIONS (Qs. 6 to10): Rearrange the following five sentences proposal, with many warning it could break up marriages
(A), (B), (C), (D) and (E) and in the proper sequence to form a because many cannot afford or access English lessons.
meaningful paragraph, then answer the questions given below them. C. Immigration lawyers have told ministers that spouses and
A. The upsurge of public activism against the setting up of fiances should not be barred from joining a partner in the
Special Economic Zones, which eventually forced the State U.K. for language reasons and that the plan could breach the
Government to announce the scrapping of all 15 such human rights convention's guarantees to the right to marry
projects, is an impressive case in point. and have a family life.
B. Early last year, a similar agitation coerced the government into D. The anonymised responses were 68 to 31 against the pre-
calling for a revision of the Goa Regional Plan 2011, a entry english test for spouses.
controversial document that opened up large swathes of land, E. Other immigration organizations said the measure would
including green belts and coastal stretches, for construction. discriminate against those from rural areas in South Asia,
where the opportunities to learn English are limited.

RESPONSE 1. a b c d e 2. a b c d e 3. a b c d e 4. a b c d e 5. a b c d e
GRID 6. a b c d e 7. a b c d e 8. a b c d e 9. a b c d e 10. a b c d e
y
o
u
rs
m
a
h
142 SPEED TEST 67

b
o
o
11. Which of the following should be the FIRST sentence? (C) He rushed to his village and placed his humble offering of

b
(a) A (b) B (c) C milk in a bowl before the snake.

.w
(d) D (e) E (D) Vishnu Raman was a poor Brahmin and a farmer by profession.

o
12. Which of the following should be the SECOND sentence? (E) The next day when he returned, he was rewarded with a gold

rd
(a) A (b) B (c) C coin in the bowl he left behind.

p
(d) D (e) E (F) Just as he was preparing to lie down he saw a huge cobra

re
13. Which of the following should be the THIRD sentence? swaying with his hood open.

s
21. Which of the following should be the SECOND sentence

s
(a) A (b) B (c) C

.c
(d) D (e) E after rearrangement?

o
14. Which of the following should be the FOURTH sentence? (a) B (b) C (c) E

m
(a) A (b) B (c) C (d) D (e) F
(d) D (e) E 22. Which of the following should be the FIRST sentence after
15. Which of the following should be the FIFTH (LAST) rearrangement?
sentence? (a) A (b) D (c) F
(a) A (b) B (c) C (d) C (e) E
(d) D (e) E 23. Which of the following should be the FIFTH sentence after
DIRECTIONS (Qs. 16 to 20) : Rearrange the following six sentences rearrangement?
(A), (B), (C), (D), (E) and (F) in the proper sequence to from a (a) F (b) D (c) C
meaningful paragraph; then answer the questions given below them. (d) B (e) E
24. Which of the following should be the SIXTH (LAST)
(A) He immediately acknowledged Mohan’s good work and sentence after rearrangement?
invited him to his home for dinner. (a) D (b) B (c) C
(B) One day a wealthy merchant sent his son’s bicycle to the (d) E (e) F
shop for repair. 25. Which of the following should be the FOURTH sentence
(C) The next day the merchant came to claim the bicycle and after rearrangement?
noticed that it was shiny. (a) E (b) F (c) B
(D) After repairing the bicycle, Mohan cleaned it up and made it (d) A (e) D
look new.
(E) Once upon a time, there was a boy named Mohan who DIRECTIONS (Qs. 26 to 30) : In each of the following items some
worked as an apprentice in a bicycle shop. parts have been jumbled up. You are required to rearrange these parts
(F) Other apprentices in the shop laughed at Mohan for doing which are labelled P, Q, R, S to produce the correct sentence. Choose
unnecessary work. the proper sequence and mark in your Answer Sheet accordingly.
16. Which of the following should be the SECOND sentence 26. Feeling flattered by praise of the fox to the piece of cheese
after rearrangement? (P) / the crow began to crow (Q) / it held on its beak (R) /
(a) A (b) B (c) C unmindful of what will / happen (S).
(d) D (e) F The proper sequence should be
17. Which of the following should be the THIRD sentence after (a) SPRQ (b) QSRP (c) RSPQ
rearrangement? (d) QSPR (e) PQSR
(a) A (b) B (c) C 27. There is that the woman is a kitchen-maid and (P) / the
(d) D (e) E traditional belief (Q) / an instrument of man’s pleasure (R) /
18. Which of the following should be the FIRST sentence after and a child-bearing machine (S).
rearrangement? The proper sequence should be
(a) A (b) B (c) C (a) QPSR (b) RQPS (c) QSPR
(d) D (e) E (d) RSPQ (e) PSRQ
19. Which of the following should be the LAST (SIXTH) 28. The rapid endangerment and death of many minority
sentence after rearrangement? languages not only among logistics and anthropologists
(a) A (b) B (c) D (P) / with issues of cultural identity (Q) / is a matter of
(d) E (e) F widespread concern (R) / but among all concerned (S).
20. Which of the following should be the FOURTH sentence The proper sequence should be
after rearrangement? (a) PSRQ (b) RQPS (c) RPSQ
(a) B (b) C (c) D (d) QRPS (e) SPQR
(d) E (e) F 29. Violence even before she is born (P) / and can happen
DIRECTIONS (Qs. 21 to 25) : Rearrange the following six throughout a woman’s life (Q) / against women (R) / takes
sentences (A), (B), (C), (D), (E) and (F) in the proper sequence to many forms (S).
from a meaningful paragraph; then answer the questions given The proper sequence should be
below them. (a) RPSQ (b) PQSR (c) RSQP
(d) SQRP (e) QPRS
(A) At first he got scared, but then he thought, “I have never 30. I saw two roads covered with the yellow (P) / directions in a
worshipped her; that is why I am not able to get anything forest (Q) / branching in two different (R) leaves of autumn (S).
from my land.” The proper sequence should be
(B) One day unable to tolerate the summer heat, he went to rest (a) PQRS (b) PQSR (c) PRSQ
under a big banyan tree. (d) PSRQ (e) QSRP

11. a b c d e 12. a b c d e 13. a b c d e 14. a b c d e 15. a b c d e


RESPONSE 16. a b c d e 17. a b c d e 18. a b c d e 19. a b c d e 20. a b c d e
GRID 21. a b c d e 22. a b c d e 23. a b c d e 24. a b c d e 25. a b c d e
26. a b c d e 27. a b c d e 28. a b c d e 29. a b c d e 30. a b c d e
y
o
u
rs
m
a
h
b
68

o
o
Idioms and

b
.w
o
rd
Phrases

p
re
s
s
.c
o
m
Max. Marks : 36 No. of Qs. 36 Time : 25 min. Date : ........./......../................

DIRECTIONS (Qs. 1 to 36): In each of the following sentences, 10. We should give a wide berth to bad characters.
an idiomatic expression or a proverb is highlighted Select the (a) give publicity to (b) publicly condemn
alternative which best describes its use in the sentence. (c) keep away from (d) not sympathise with
(e) none of these
1. I have a bone to pick with you in this matter. 11. The authorities took him to task for his negligence.
(a) Am in agreement (b) Am angry (a) gave him additional work (b) suspended his assignment
(c) Am indebted (d) Will join hands (c) reprimanded him (d) forced him to resign
(e) None of these (e) none of these
2. The new CM stuck his neck out today and promised 10kgs. 12. In spite of the immense pressure exerted by the militants, the
free wheat a month for all rural families. Government has decided not to give in.
(a) took an oath (b) took a risk (a) accede (b) yield
(c) extended help (d) caused embarrassment (c) oblige (d) confirm
(e) None of these (e) none of these
3. Harassed by repeated acts of injustice he decided to put 13. Their business is now on its last legs.
his foot down. (a) About to fructify (b) About to perish
(a) not to yield (b) resign (c) About to produce results
(c) to accept the proposal unconditionally (d) About to take off (e) none of these
(d) withdraw (e) none o fthese 14. He went back on his promise to vote for me.
4. The class could not keep a straight face on hearing the (a) withdrew (b) forgot
strange pronunciation of the new teacher. (c) reinforced (d) supported
(a) remain silent (b) remain serious (e) none of these
(c) remain mute (d) remain disturbed 15. The old beggar ran amuck & began to throw stones at the passerby.
(e) none of these (a) became desperate (b) ran about wildly
5. His speech went down well with the majority of the audience. (c) become annoyed (d) felt disgusted
(a) found acceptance with (e) none of these
(b) was attentively listened to by 16. Turban is in vogue in some communities.
(c) was appreciated by (d) was applauded by (a) in fashion (b) out of use
(e) none of these (c) vaguely used (d) never used
6. Rohit has bitten off more than he chew. (e) none of these
(a) Is trying to do much (b) Is very greedy 17. The old man was cut to the quick when his rich son refused
(c) Is always hungry (d) Has little regard for others to recognise him.
(e) none of these (a) surprised (b) hurt intensely
7. The detective left no stone unturned to trace the culprit. (c) annoyed (d) irritated
(a) took no pains (b) did very irrelevant things (e) none of these
(c) resorted to illegitimate practices 18. I requested him to put in a word for me.
(d) used all available means (a) introduce (b) assist
(e) none of these (c) support (d) recommend
8. He believes in the policy of making hay while the sun shines. (e) none of these
(a) giving bribes to get his work done 19. The dacoit murdered the man in cold blood.
(b) seeking advice from one and all (a) coldly (b) boldly
(c) helping those who help him (c) ruthlessly (d) deliberately
(d) making the best use of a favourable situation (e) none of these
(e) none of these 20. He is always picking holes in every project.
9. His friends advised him to be fair and square in his dealings. (a) creating problems in (b) finding fault with
(a) Careful (b) Considerate (c) suggesting improvement in
(c) Polite (d) Upright (d) asking irrelevant questions on
(e) none of these (e) None of these
1. a b c d e 2. a b c d e 3. a b c d e 4. a b c d e 5. a b c d e
6. a b c d e 7. a b c d e 8. a b c d e 9. a b c d e 10. a b c d e
RESPONSE 11. a b c d e 12. a b c d e 13. a b c d e 14. a b c d e 15. a b c d e
GRID 16. a b c d e 17. a b c d e 18. a b c d e 19. a b c d e 20. a b c d e
y
o
u
rs
m
a
h
144 SPEED TEST 68

b
o
o
21. Pt. Nehru was born with a silver spoon in his mouth. 31. The speaker gave a bird's eye view of the political

b
(a) born in a middle class family

.w
conditions in the country.
(b) born in a wealthy family (a) a personal view (b) a general view

o
(c) born in a royal family

rd
(c) a biased view (d) a detailed presentation
(d) born in a family of nationalists

p
(e) None of these

re
(e) none of these
32. The stunt that I recently attempted was a piece of cake

s
22. The arrival of the mother-in-law in the family proved a rift in

s
(a) The stunt that I recently attempted was enjoyable to watch

.c
the lute.

o
(a) caused unnecessary worries (b) The stunt that I recently attempted was very challenging

m
(b) brought about disharmony (c) The stunt that I recently attempted was celebrated by all
(c) caused a pleasant atmosphere (d) The stunt that I recently attempted turned out to be a failure
(d) brought about a disciplined atmosphere (e) The stunt that I recently attempted was a simple task
(e) none of these 33. The boy broke the window and took to his heels.
23. Having sold off his factory, he is now a gentleman at large.
(a) The boy broke the window and fell on his heels
(a) Has no serious occupation
(b) Is living comfortably (b) The boy broke the window and ran away
(c) Is respected by everybody (c) The boy broke the window with his heels
(d) Is held in high esteem (e) none of these (d) The boy ran into the window
24. Though he has lot of money, yet all his plans are built upon sand. (e) The boy broke the window and robbed a pair of heels
(a) established on insecure foundations 34. I pledged myself to serve the king faithfully.
(b) based on inexperience (c) resting on cheap material (a) I made a mistake by promising to serve the king
(d) resting on immature ideas faithfully
(e) none of these (b) I made a fool of myself in order to serve the king
25. There has been bad blood between the two communities even
before shouting. (c) I boasted about serving the king faithfully
(a) Impure blood (b) Ill feeling (d) I was forcibly made to serve the king
(c) Bloody fights (d) Quarrels (e) I made a solemn and formal promise to serve the king
(e) none of these faithfully
26. The curious neighbors were disappointed as the young 35. There is a crying need for improvements to our public
couple's quarrel was just a storm in a tea cup. transport system.
(a) violent quarrel (b) fuss about a trifle (a) There is an obvious need for improvements to our public
(c) brittle situation (d) quarrel about tea cups transport system
(e) none of these
(b) There is a well documented need for improvements to
27. My father strained every nerve to enable me to get settled in life.
(a) worked very hard (b) spent a huge amount our public transport system.
(c) tried all tricks (d) bribed several persons (c) There is a minor need for improvements to our public
(e) none of these transport system.
28. Madhuri might scream blue murder, but I feel Deepali should (d) There is a serious need for improvements to our public
get the promotion since she is better qualified for the job. transport system.
(a) Someone has been murdered with some blue liquid (e) There is no urgency for improvements to our public
(b) Someone is being murdered and has become blue transport system.
(c) Suffer from persecution complex 36. In an old bookshop I happened to light upon a volume that
(d) Make a great deal of noise and object vehemently
(e) none of these belonged to my grandfather.
29. Why do you wish to tread on the toes? (a) In an old bookshop I happened to discard a volume
(a) To give offence to them that belonged to my grandfather.
(b) To follow them grudgingly (b) In an old bookshop I happened to purchase a volume
(c) To treat them indifferently that belonged to my grandfather.
(d) To be kicked by them (e) None of these (c) In an old bookshop I happened to look for a volume
30. The autographed bat from the famous cricketer Sunil that belonged to my grandfather.
Gavaskar is worth a jew's eye. (d) In an old bookshop I happened to discover by chance
(a) Not a worthy possession
a volume that belonged to my grandfather
(b) unnecessary (c) A costly items
(d) A possession of high value (e) In an old bookshop I happened to reveal a volume that
(e) None of these belonged to my grandfather.

21. a b c d e 22. a b c d e 23. a b c d e 24. a b c d e 25. a b c d e


RESPONSE 26. a b c d e 27. a b c d e 28. a b c d e 29. a b c d e 30. a b c d e
GRID 31. a b c d e 32. a b c d e 33. a b c d e 34. a b c d e 35. a b c d e
36. a b c d e
y
o
u
rs
m
a
h
b
69

o
o
b
.w
o
Cloze Test - I

rd
p
re
s
s
.c
o
m
Max. Marks : 34 No. of Qs. 34 Time : 20 min. Date : ........./......../................

DIRECTIONS (Qs.1-34) : In the following passage there are Passage - 2


blanks each of which has been numbered. These numbers are Today, twenty-two years after the bank (9), it has over a
printed below the passage and against each, five words are thousand branches all over the country and the staff (10) about
suggested, one of which fits the blank appropriately. Find out twenty-three lakh borrowers. We decided to operate (11) from
the approptiate words in each case. conventional banks who would ask their clients to come to their
Passage - 1 office. Many people in rural areas found this (12). Our bank is
When we (1) started thirty years ago in 1977, we did not therefore based on the (13) that people should not come to the
know anything about how to run a bank for the poor. We therefore bank but that the bank should go to the people. Our loans are also
looked at how others ran their operations and (2) from their (14)- we give them for activities for candlemaking to tyre repair. We
mistakes. In Bangladesh, conventional banks and credit co- also keep (15) checks on the borrower through weekly visits. We
operatives always (3) lump sum repayments. This created (4) do this to make certain that the family of the borrower is (16) from
problems because repaying in a lump sum was a mental hurdle the loan.
for borrowers. They tended to delay repayment and get further 9. (a) inaugurated (b) origin
into debt in the (5). In the end they usually (6) totally on the loan, (c) commence (d) existed
which was a loss to the bank. In structuring our own loans, I (e) began
decided to ask for a daily payment, Monitoring repayment was 10. (a) handle (b) assemble
(7) and it filled people with (8) that they could repay their loans. (c) cope (d) interact
1. (a) firstly (b) freshly (e) deal
(c) foremost (d) initially 11. (a) identically (b) differently
(e) recently (c) similar (d) reverse
2. (a) copied (b) observed (e) opposite
(c) learned (d) understood 12. (a) threatening (b) worried
(e) improving (c) upset (d) panicking
3. (a) asked (b) insisted (e) anxious
(c) demanded (d) settled 13. (a) advantage (b) principle
(e) lend (c) discipline (d) opportunity
4. (a) severe (b) no (e) chance
(c) additionally (d) variety 14. (a) diverse (b) worth
(e) plenty (c) vary (d) disburse
5. (a) time (b) process (e) contrast
(c) return (d) event 15. (a) daily (b) consistently
(e) action (c) regular (d) often
6. (a) neglected (b) abandoned (e) frequently
(c) defaulted (d) depended 16. (a) progress (b) benefiting
(e) disappointed (c) serving (d) welfare
7. (a) benefit (b) easier (e) obliged
(c) reckless (d) disorganised Passage - 3
(e) secure The Government seems to be in right earnest to ensure more
8. (a) sense (b) confidence (17) in governance. The Prime Minister’s announcement that his
(c) challenge (d) doubt Government is (18) drafting legislation to establish the citizen’s
(e) believe right to information is indeed welcome. Though the talk on the
right to information is not new, we may (19) the bill to be brought

1. a b c d e 2. a b c d e 3. a b c d e 4. a b c d e 5. a b c d e
6. a b c d e 7. a b c d e 8. a b c d e 9. a b c d e 10. a b c d e
RESPONSE 11. a b c d e 12. a b c d e 13. a b c d e 14. a b c d e 15. a b c d e
GRID 16. a b c d e
y
o
u
rs
m
a
h
146 SPEED TEST 69

b
o
o
early this time. The previous Government had set up a high-level $ 86 billion. (32) countries have now begun to notice the (33)

b
committee to prepare a draft bill. But nothing has been heard about

.w
available in Africa. China’s attitude has (34) the way the world
the matter since, (20) the committee did quite some work. The deals with poor countries. “Trade not aid” is the new mantra of

o
rd
issue, however, has come to such a pass that a solution cannot be African nations.

p
(21) further. Sunlight is the best disinfectant, a foreign judge once 25. (a) belonging (b) similarly

re
said, while (22) the unwarranted secrecy in an administrative (c) compared (d) with

s
s
system. When those in authority know that people have the right

.c
(e) like
to ask questions and the government is under the (23) to provide

o
26. (a) efforts (b) practices

m
them with answers, (24) of authority, or of public finances, for (c) challenges (d) achievements
personal or party ends is less likely to happen.
(e) attempt
17. (a) strictness (b) rudeness (c) leniency
27. (a) given (b) approved
(d) economy (e) transparency
(c) regular (d) often
18. (a) personally (b) busy (c) not (e) being
(d) reluctantly (e) absolutely 28. (a) grant (b) sanctioned
19. (a) expect (b) wait (c) try
(c) took (d) hired
(d) frustrate (e) appeal
(e) apply
20. (a) even (b) as (c) because
29. (a) goal (b) fund
(d) until (e) though
(c) way (d) skill
21. (a) found (b) expected (c) delayed
(e) dream
(d) looked (e) longed
30. (a) countries (b) others
22. (a) nurturing (b) criticising (c) demanding
(c) abroad (d) neighbours
(d) appreciating (e) upholding
(e) poor
23. (a) pretention (b) affect (c) substance
31. (a) further (b) extra
(d) obligation (e) property
(c) more (d) less
24. (a) misuse (b) governance (c) dishonour (e) high
(d) curbing (e) breach 32. (a) Recently (b) Any
Passage - 4
(c) Friendly (d) Many
Today the economies of African countries are growing at
(e) While
5 per cent every year.
33. (a) differences (b) supply
Rich countries usually help poorer ones (25) African
(c) quantity (d) people
countries through donations and aid. Their (26) are not always
(e) opportunities
successful as loans are (27) not used for the projects for which
34. (a) substitute (b) changed
they are (28). China, however, has found a different (29) to help
(c) exchanged (d) transform
Africa - by trading more with the (30). In 2009 China’s trade with
(e) convert
African countries was $ 90 billion - (31) than the U.S., which was

17. a b c d e 18. a b c d e 19. a b c d e 20. a b c d e 21. a b c d e


RESPONSE 22. a b c d e 23. a b c d e 24. a b c d e 25. a b c d e 26. a b c d e
GRID 27. a b c d e 28. a b c d e 29. a b c d e 30. a b c d e 31. a b c d e
32. a b c d e 33. a b c d e 34. a b c d e
y
o
u
rs
m
a
h
b
70

o
o
b
.w
Cloze Test - II

o
rd
p
re
s
s
.c
o
m
Max. Marks : 35 No. of Qs. 35 Time : 20 min. Date : ........./......../................

DIRECTIONS : In the following passage there are blanks, each 6. (a) felt (b) said
of which has been numbered. These numbers are printed below (c) know (d) accept
the passage, against each, five words are suggested, one of which (e) saw
fits the blank appropriately. Find out the appropriate word in 7. (a) asks (b) chooses
each case. (c) look (d) find
(e) wish
Passage - 1 8. (a) sure (b) put
Emperor Akbar was fond of (1) tricky questions to Birbal. One (c) shown (d) seen
day he asked Birbal what he would (2) if he were given a choice (e) made
between justice and a gold coin. “The gold coin,” said Birbal. 9. (a) no (b) rich
Akbar was (3) aback. He had known Birbal for many years and (c) short (d) poor
he knew that Birbal was a just person. Then how could he choose (e) plenty
the gold coin. “You would prefer a gold coin to justice ?” He 10. (a) but (b) not
asked, incredulously. “Yes,” said Birbal. The other courtiers were (c) and (d) s o
amazed by Birbal’s (4) of idiocy. For years they had been trying (e) only
to discredit Birbal in the emperor’s eyes but without success and Passage - 2
now the man had gone and (5) it himself ! They could not Once upon a time, two friends were (11) through the desert. During
believe their good fortune. “I would have been dismayed if even some point of the (12) they had an argument, and one friend slapped
the lowliest of my servants had said this,” continued the em- the other one in the face. The one who got slapped was (13), but
peror. “But coming from you it’s shocking - and sad. I did not (6) without saying anything, he wrote in the sand, “Today my best
you were so debased ! I never expected this from you. How friend slapped me in the face.” They kept on walking (14) they
could you be so shallow ?” found an oasis, where they (15) to take a bath. The one, who had
One (7) for what one does not have, Your Majesty !” said Birbal, been slapped, got (16) in the quicksand and started drowning, but
quietly. “You have (8) to it that in our country justice is available the friend saved him. After the friend (17) from the near drowning
to everybody. So as justice is already available to me and as I’m he wrote on a stone, “Today my best friend saved my life.” The
always (9) of money I said I would choose the gold coin.” The friend who had slapped and saved his best friend asked him, “After
I hurt you, you wrote in the sand and (18) you write on a stone,
emperor laughed. He thought to himself, ‘I should have known
why?” The other friend (19), “When someone hurt us, we should
that Birbal would come up with a witty reply as always.’ He was
write it down in sand where wind of forgiveness can erase it away.
so pleased with Birbal’s reply that he gave him (10) one but a But, when someone does something good for us, we must (20) it in
thousand gold coins. stone where no wind can ever erase it.”
1. (a) showing (b) asking 11. (a) crawling (b) speaking
(c) naming (d) finding (c) swimming (d) walking
(e) telling (e) dancing
2. (a) look (b) said 12. (a) journey (b) sand
(c) think (d) choose (c) running (d) border
(e) find (e) hunt
3. (a) pushed (b) fallen 13. (a) dead (b) captured
(c) pulled (d) sent (c) presentable (d) missing
(e) taken (e) hurt
4. (a) idea (b) display 14. (a) as (b) until
(c) reply (d) place (c) from (d) with
(e) showing (e) through
5. (a) speak (b) thought 15. (a) decided (b) fell
(c) done (d) create (c) made (d) want
(e) told (e) left

1. a b c d e 2. a b c d e 3. a b c d e 4. a b c d e 5. a b c d e
RESPONSE
6. a b c d e 7. a b c d e 8. a b c d e 9. a b c d e 10. a b c d e
GRID 11. a b c d e 12. a b c d e 13. a b c d e 14. a b c d e 15. a b c d e
y
o
u
rs
m
a
h
148 SPEED TEST 70

b
o
o
16. (a) home (b) stuck 24. (a) affect (b) ideas (c) practice

b
(c) blended (d) mixed (d) concept (e) procedure

.w
(e) sitting 25. (a) benefit (b) merit (c) chance

o
rd
17. (a) separated (b) leaked (d) basis (e) method

p
(c) died (d) recovered 26. (a) unless (b) until (c) executed

re
(e) saved (d) provided (e) exercised

s
s
18. (a) so (b) how 27. (a) other (b) any (c) two

.c
(c) when (d) tomorrow (d) differ (e) after

o
m
(e) now 28. (a) on (b) of (c) often
19. (a) called (b) tell (d) taken (e) off
(c) replied (d) questioned 29. (a) soft (b) more (c) less
(e) asked (d) only (e) hard
20. (a) talk (b) push 30. (a) need (b) equilibrium
(c) engrave (d) add (c) expectation (d) attempt
(e) bury (e) aspects
Passage - 3 Passage - 4
The Right of Children to Free and Compulsory Education (RTE) The Bhagavad Gita is a poem of 700 verses which is a part of the
Act, 2009, which came (21) effect in April this year, is meant to Mahabharata. It is the only philosophical song existing in all
transform the education sector and take India closer to the goal of languages. Its popularity and influence have never waned. It (31)
universal schooling. But with admissions to the new academic light and guidance to the troubled mind in times of crisis. It is in
session just (22) the corner, it is fast becoming clear that (23) well the (32) of a dialogue between Arjuna and Krishna on the
intentioned ideas into (24) will take some doing. For a start, the battlefield. Arjuna’s mind is troubled at the thought of the killings
guidelines for admissions under the RTE prohibit schools from of his friends and relatives. He cannot conceive of any gain. Arjuna
conducting any sort of student profiling. The stress on a random is the (33) of the tortured spirit of man tom by conflicting
yet justifiable admission process means that schools will have to obligations and molalities.
resort to something as quirky as a lottery system. However, leaving The dialogue proceeds and takes upto the higher level of
admission to a good school to pure (25) will only incentivise individual duty and social behaviour, application of ethics to
manipulations, defeating the very essence of RTE. practical life and social outlook that should govern all. An attempt
The main problem facing the education sector is that of a is (34) to reconcile the three paths of human advancement - the
resource crunch. The provisions for ensuring universal access to path of knowledge, the path of action and the path of faith. But
education are all very well, (26) we have the infrastructure in place more (35) is laid on faith. There is a call of action to meet the
first. Brick and mortar schools need to precede open admission obligations of life, keeping in view the spiritual background and
and not the (27) way around. In that sense, legislators’ assessment the large purpose of the universe.
of ground realities is (28) target when they endorse the closure of 31. (a) provides (b) shines
tens of thousands of low-cost private schools for not meeting the (c) enforces (d) secures
minimum standards of land plot, building specifications and (e) seeks
playground area as laid out in the RTE Act. Instead of bearing 32. (a) programme (b) constitution
down (29) on private schools for failing to conform to abstract (c) part (d) formation
bureaucratic criteria, efforts to bring about universal education (e) form
should focus on upgrading and expanding the existing 33. (a) conceived (b) dream
government school infrastructure to accommodate all. Only then (c) source (d) figures
can we ensure the much needed supply-demand (30) in the (e) symbol
education sector. 34. (a) generated (b) made
21. (a) with (b) for (c) on (c) established (d) coined
(d) into (e) in (e) given
22. (a) around (b) near (c) into 35. (a) important (b) Significant
(d) about (e) reaching (c) declaration (d) emphasis
23. (a) forming (b) translating (c) having (e) blessings
(d) taking (e) framing

16. a b c d e 17. a b c d e 18. a b c d e 19. a b c d e 20. a b c d e


RESPONSE 21. a b c d e 22. a b c d e 23. a b c d e 24. a b c d e 25. a b c d e
GRID 26. a b c d e 27. a b c d e 28. a b c d e 29. a b c d e 30. a b c d e
31. a b c d e 32. a b c d e 33. a b c d e 34. a b c d e 35. a b c d e
y
o
u
rs
m
a
h
b
71

o
o
b
Section Test :

.w
o
rd
p
English Language

re
s
s
.c
o
m
Max. Marks : 40 No. of Qs. 40 Time : 20 min. Date : ........./......../................
DIRECTIONS : Read the following passage carefully and answer or start a manufacturing business. It can help those who have
the questions given below it. Certain words/group of words have found a job and are still nonetheless poor. It gives the victims of
been printed in bold to help you locate them while answering prejudice who would not be hired because of their colour or national
some of the questions. origin a chance to earn a living. The average cost of creating self-
Unemployment is the problem of every modern nation. Even employment is ten, twenty or hundred times lesser than creating
industrialised nations are not able to ensure a job for everyone. industry-based employment. It helps isolated poor people gain self-
Following the conventional strategy of creating employment, confidence step by step.
governments of many developing countries try to attract Obviously self-employment has limits, but in many cases it is
employers (business houses/industrialists) by offering tax the only solution to help those whom economies refuse to hire and
rebates and many other facilities so that they locate their taxpayers do not want to carry on their shoulders. The policy needed
upcoming plants on their soil, and thereby create industrial for the eradication of poverty must be much wider and deeper than
employment. But there is a limit to what industry can bring. the policy for the provision of mere employment. The real eradication
Also, industrial plants often create toxic waste which results in of poverty begins when people are able to control their own fate.
air and water pollution and environmental problems which can Poor people are like bonsai trees. When you plant the best seed of
outweigh whatever employment benefit industrial employment the tallest tree in a flower-pot, you get a replica of the tallest tree,
brings. In addition, they don't bring as substantial relief to the only inches tall. There is nothing wrong with the seed you planted;
dwindling economy of the host country as they seem to promise, only the soil-base that is too inadequate. Poor people are bonsai
as the profits of such foreign investments are carried back to the people. There is nothing wrong in their seeds. Simply, society never
parent company and foreign shareholders abroad. gave them the base to grow on. All it takes to get the poor people
Self-employment has none of these drawbacks. The problem out of poverty is for us to create an enabling environment for them.
is that self-employment is not as obviously glamorous as a shiny Once the poor can unleash their energy and creativity, poverty will
new factory. But profits from self-employment remain in the disappear very quickly.
country where they are produced. It is too small to create DIRECTIONS (Q. 1-3) : Choose the word/group of words which is
environmental hazards. It also puts the poor person in charge of MOST OPPOSITE in MEANING to the word/group of words printed
his or her own working hours and conditions. The hours are in bold as used in the passage.
flexible and can adapt to fit any family situation. It allows people
1. RIGID
to choose between running a business full-time, or part-time when
they face a crisis, or to put their business on hold and work full- (a) Unstructured (b) Flexible
time for a salary. Self-employment is tailor-made for anyone who (c) Soft (d) Gentle
is street-smart and has many acquired and inherited traditional (e) Calm
skills, rather than learning acquired from books and technical 2. STEP BY STEP
schools. This means the illiterate and the poor can exploit their (a) All at once (b) In quick succession
strengths, rather than be held back by their weaknesses. It allows (c) In slow motion (d) In a nutshell
a person to turn their hobbies into gainful employment. It allows (e) Once and for all
individuals who cannot work well in a rigid hierarchy to run their
3. OFFERING
own show.
(a) Stealing (b) Permitting
Financing the poor to start their own little ventures elevates
their sense of pride and self-respect. It offers a way out of welfare (c) Refusing (d) Protesting
dependency, not just to become wage slaves, but to open a store (e) Questioning

RESPONSE 1. a b c d e 2. a b c d e 3. a b c d e
GRID
y
o
u
rs
m
a
h
150 SPEED TEST 71

b
o
o
DIRECTIONS (Q. 4-15): Choose the word/group of words which 10. Which of the following may be inferred about self-

b
.w
is MOST SIMILAR in MEANING to the word printed in bold as employment?

o
used in the passage. (A) Self-employment slowly but steadily strengthens the

rd
4. REMAIN economy of the country.

p
re
(a) Left-over (b) Stay (B) Self-employment checks unemployment.

s
(C) As a str ategy of providing employment, self-

s
(c) Stagnate (d) Continue

.c
employment is still unexplored.

o
(e) Linger

m
5. HIRED (a) Only (B) (b) Only (B) and (C)
(a) Allowed (b) Rented (c) Only (A) (d) Only (A) and (B)
(c) Authorised (d) Employed (e) Only (A) and (C)
(e) Delegated 11. What does the author indicate by the example of a bonsai
tree?
6. LIMIT
(A) When provided the right kind of financial help, poor
(a) Maximum (b) Finish
people in can flourish.
(c) Cap (d) Decrease
(B) The poor people are as capable as the well-to-do class.
(e) Barrier
(C) Conventional (industrial) employment can help the
7. Which of the following is a reason foreign investments do
poor people create their own base.
not strengthen the economies of host nations?
(a) Only (A) (b) Only (B)
(a) The parent company pays all the profit as tax to its
(c) Only (A) and (B) (d) Only (A) and (C)
nation.
(e) Only (B) and (C)
(b) The profit of such enterprises does not remain in the
host nation; rather it goes back to the share holders 12. The author claims that self-employment is "tailor-made" for
and owners of the parent company. people with certain qualities. Which of the following are the
qualities of such people?
(c) The employees of the parent company demand extra
pay from profits that the companies earn from factories (A) They have an unconventional approach to all things.
in another nation. (B) They are street-smart.
(d) The profit earned by such enterprises is too less to (C) They possess many acquired and traditional skills.
provide for anything beyond the salaries of employees. (a) Only (A) (b) Only (C)
(e) None of these (c) Only (B) and (C) (d) Only (A) and (C)
8. What is the tone of the passage? (e) Only (B)
(a) Offensive (b) Satirical 13. Which of the following outweighs the employment benefits
(c) Analytical (d) Humorous that foreign industrialists bring?
(e) Speculative (a) Huge industries set up by them cause environmental
9. Which of the following is an advantage(s) that self- pollution.
employment has over industry-based employment? (b) They employ more number of people belonging to their
(A) The work timings are highly flexible. native nations than the host nations' unemployed.
(B) Starting one's own venture is an easy task and needs (c) They evade many taxes that could be a source of revenue
no investment as financiers are readily available. for the host nation.
(C) Self-employment makes one a master of other people (d) They manufacture products that have no market in the
and thus satisfies their need to control others. host nation.
(a) Only(C) (b) Only (A) (e) They practise discrimination on grounds of gender
when providing employment to host nations' residents.
(c) Only (B) (d) Only (A) and (B)
(e) All (A), (B) and (C)

RESPONSE 4. a b c d e 5. a b c d e 6. a b c d e 7. a b c d e 8. a b c d e

GRID 9. a b c d e 10. a b c d e 11. a b c d e 12. a b c d e 13. a b c d e


y
o
u
rs
m
a
h
SPEED TEST 71 151

b
o
o
14. Which of the following may be an appropriate title for the 20. Each business activity ___________ employment to people

b
.w
passage? who would otherwise be unemployed.

o
(a) Addressing conventional employment in developed (a) taking (b) finds

rd
nations (c) creates (d) provides

p
re
(b) Varied strategies and approaches to eradicating (e) given

s
poverty

s
DIRECTIONS (Q. 21-30) : In the following passage, there are

.c
(c) Limitations of industrial employment blanks, each of which has been numbered. These numbers are

o
m
(d) How is poverty linked to conventional (industrial) printed below the passage and against each five words are
employment? suggested, one of which fits the blank appropriately. Find out the
(e) Role of self-employment in battling unemployment and appropriate word in each case.
eradication of poverty
15. Which of the following is TRUE as per the passage? The latest technology (21) put to use or about to arrive in
(a) Self-employment is beneficial only for developing market must be (22) to all entrepreneurs. The reason is that it may
economies. have an (23) effect on business. Valve radios gave way to transistor
(b) Self-employment is not as glamorous as conventional radios and with micro chips, technology is giving way to digital
(industrial) employment. equipment. Business has (24) the same but the technology has
(c) Finance for poor is readily available in the developed kept changing. A notable feature is that the size of the receivers
nations of the world. decreased (25) so did the use of its material and consequently its
(d) Small-scale industries produce as much toxic waste price. The traditional flour mills are losing business (26) customers
as big industries. now buy flour (27) from the market. As a result of this, the business
is (28). Following the same lines as technology, the social trends
(e) None is true
also go on changing and influence the market. The Indian sarees
DIRECTIONS (Q. 16-20): Pick out the most effective word from are being taken (29) by readymade stitched clothes. Every
the given words to fill in the blank to make the sentence entrepreneur must note such changes in the environment and also
meaningfully complete. the technology and plan in (30)with these to ensure the success of
16. The government is planning to set ___________ family his endeavour.
welfare centres for slums in cities. 21. (a) to (b) needed
(a) another (b) with (c) decided (d) besides
(c) for (d) in (e) being
(e) up 22. (a) hoped (b) welcome
17. Economic independence and education have ___________ (c) released (d) known
women more assertive. (e) aware
(a) prepared (b) made 23. (a) approximate (b) huge
(c) marked (d) resulted (c) uniform (d) excellence
(e) adjusted (e) enormous
18. In the modern world, the ___________ of change and 24. (a) maintained (b) remained
scientific innovation is unusually rapid.
(c) often (d) mentioned
(a) supplies (b) context
(e) become
(c) pace (d) fantasy
25. (a) mainly (b) and
(e) requirement
(c) how (d) also
19. The unprecedented economic growth of China has
(e) some
___________ worldwide attention.
26. (a) reason (b) due
(a) perceived (b) proposed
(c) young (d) as
(c) neither (d) astonished
(e) old
(e) attracted

14. a b c d e 15. a b c d e 16. a b c d e 17. a b c d e 18. a b c d e


RESPONSE 19. a c d e 20. a c d e 21. a c d e 22. a c d e 23. a c d e
b b b b b
GRID
24. a b c d e 25. a b c d e 26. a b c d e
y
o
u
rs
m
a
h
152 SPEED TEST 71

b
o
o
27. (a) knowingly (b) ease 34. It is difficult to work with him because he is one of those

b
.w
(c) cheap (d) directly persons who think he is always right.

o
(e) forcefully (a) think they are always

rd
28. (a) shrinking (b) blooming

p
(b) always thinks he is (c) is always thinking they are

re
(c) returned (d) same
(d) always think his (e) No correction required

s
s
(e) small

.c
29. (a) against (b) to 35. Foreign businesses in developing countries have usually

o
m
(c) over (d) up problems with lack of infrastructure and rigid laws.
(e) for (a) usual problems as (b) usually problems on
30. (a) lines (b) relativity (c) as usual problems like (d) the usual problems of
(c) accordance (d) proper (e) No correction required
(e) toning
DIRECTIONS (Qs. 36-40) : Read each sentence to find out whether
DIRECTIONS (Q. 31-35): Which of the phrases (a), (b), (c) and
(d) given below should replace the phrase given in bold in the there is any grammatical mistake/error in it. The error if any, will be
following sentences to make the sentence grammatically correct? in one part of the sentence. Mark the number of the part with error
If the sentence is correct as it is and there is no correction required as your answer. If there is no error, mark (e).
mark (e) ie ‘No correction required’ as the answer. 36. The cost of constructing (a) / houses are increased (b) /
31. During the recession many companies will be forced to because of the high (c) / price of cement. (d) / No error (e).
lay off workers.
37. According to the Twelfth (a) / Five Year Plan, India should
(a) have the force to (b) be forced into
(b) / invest one trillion dollars (c) / in infrastructure projects.
(c) forcibly have (d) forcefully
(d) / No error (e)
(e) No correction required
32. He wanted nothing else expecting to sleep after a Stressful 38. To increase the selling (a) / of products in rural areas (b) / the
day at work. company will hire (c) / over five hundred trainees. (d) / No
(a) nothing better than (b) anything else unless error (e)
(c) nothing but having (d) nothing else than 39. We have spent (a) / most of the profits (b) / that we earn (c)
(e) No correction required / last year on purchasing new computers, (d) / No error (e).
33. Ramesh took charge of the project, within a few days of 40. The Government has (a) / promised to revise (b) / the
having appointed ?
pension scheme for bank (c) / staff since next year. (d) / No
(a) having an appointment(b) being appointed
error (e)
(c) after being appointed (d) appointing
(e) No correction required

27. a b c d e 28. a b c d e 29. a b c d e 30. a b c d e 31. a b c d e


RESPONSE
32. a b c d e 33. a b c d e 34. a b c d e 35. a b c d e 36. a b c d e
GRID
37. a b c d e 38. a b c d e 39. a b c d e 40. a b c d e
y
o
u
rs
m
a
h
b
72

o
o
Computer Fundamentals/

b
.w
o
rd
Binary System/

p
re
s
Operating System

s
.c
o
m
Max. Marks : 30 No. of Qs. 30 Time : 20 min. Date : ........./......../................

1. Most of the commonly used personal computers/laptops 9. How many megabytes make a gigabyte?
do not have a command key known as ________. (a) 1024 (b) 128
(a) Turnover (b) Shift (c) Alter (c) 256 (d) 512
(d) Delete (e) Insert
(e) 64
2. Which of the following is NOT a hardware of a computer?
10. Which of the following categories would include a keyboard?
(a) Monitor (b) Key Board
(a) Printing Device (b) Output Device
(c) Windows (d) Central Processing Unit
(c) Pointing Device (d) Storage Device
(e) Mouse
3. Most of the commonly available personal computers/laptops (e) Input Device
have a keyboard popularly known as ________. 11. Which of the following is hardware and not software ?
(a) QWERTY (b) QOLTY (c) ALTER (a) Excel (b) Printer driver
(d) UCLIF (e) None of these (c) Operating System (d) Power Point
4. Computers send and receive data in the form of ________ (e) CPU
signals.
12. A _________ is an electronic device that process data,
(a) Analog (b) Digital converting it into information.
(c) Modulated (d) Demodulated (a) computer (b) processor
(e) All of these
(c) case (d) stylus
5. The smallest unit of information a computer can understand
(e) None of these
and process is known as a_______.
13. A computer works on a _________ number system.
(a) digit (b) byte (c) megabyte
(d) kilobyte (e) bit (a) binary (b) octal
6. A byte can represent any number between 0 and_____ (c) decimal (d) hexadecimal
(a) 2 (b) 255 (c) 256 (e) None of these
(d) 1024 (e) 1025 14. The physical components of a computer system is _______
7. External devices such as printers, keyboards and modems (a) Software (b) Hardware
are known as
(c) ALU (d) Control Unit
(a) add-on devices.
(e) None of these
(b) peripherals.
15. Files deleted from the hard disk are sent to the ______.
(c) extra hardware devices.
(a) recycle bin (b) floppy disk
(d) PC expansion slot add-ons.
(c) clipboard (d) motherboard
(e) special-buys
8. The most common pointing input device is the (e) None of these
(a) trackball (b) touchpad 16. Which of the following is NOT a famous operating system?
(c) touchscreen (d) mouse (a) Windows Vista (b) Mac OS X (c) Linux
(d) Sun OS (e) Virtual Box
(e) scanner

1. a b c d e 2. a b c d e 3. a b c d e 4. a b c d e 5. a b c d e
RESPONSE 6. a b c d e 7. a b c d e 8. a b c d e 9. a b c d e 10. a b c d e

GRID 11. a b c d e 12. a b c d e 13. a b c d e 14. a b c d e 15. a b c d e


16. a b c d e
y
o
u
rs
m
a
h
154 SPEED TEST 72

b
o
o
17. Window 95, Windows 98 and Windows NT are known as 25. Executing more than one program concurrently by one user

b
.w
(a) processors (b) domain names (c) modems on one computer is known as

o
(d) operating systems (e) None of these (a) multi-programming (b) multi-processing

rd
p
18. Two different files can have the same name if (c) time sharing (d) multi-tasking

re
(a) they are in different folders (e) multi-action

s
s
.c
(b) they are on different drives 26. All computers must have

o
m
(c) Never (a) a word processing software
(d) the names are capitalised differently (b) an operating system
(e) None of these (c) an attached printer
19. Every computer has a(n) ____________; many also have (d) a virus checking program
__________.
(e) None of these
(a) operating system; a client system
27. When you instal a new program on your computer,it is typi-
(b) operating system; instruction sets
cally added to the _________ menu.
(c) application programs; an operating system
(a) All Programs (b) Select Programs
(d) application programs; a client system
(c) Start Programs (d) Desktop Programs
(e) operating system; application programs
(e) None of these
20. Applications are often referred to as
28. The operating system, that is self-contained in a device and
(a) data file (b) executable files
resident in the ROM .is
(c) system software (d) the operating system
(a) Batch Operating System
(e) None of these
(b) Real-time Operating System
21. The process of transferring files from a computer on the
Internet to your computer is called __________. (c) Embedded Operating System
(a) downloading (b) uploading (c) FTP (d) Mutli-Processor Operating System
(d) JPEG (e) downsizing (e) None of these
22. The operating system called UNIX is typically used for 29. If you change Windows 98 to Windows XP, you are actually
(a) Desktop computers performing
(b) Laptop computers (a) upstart (b) upgrade
(c) Super computers (c) update (d) patch
(d) Web servers (e) None of these
(e) All of these 30. What happens when we try to delete the files on the floppy?
23. When data changes in multiple lists and all lists are not (a) The files get moved to the Recycle Bin
updated, this causes (b) Files on a floppy cannot be deleted
(a) data redundancy (b) information overload (c) The files get deleted and can be restored again from
(c) duplicate data (d) data inconsistency Recycle Bin .
(e) data repetition (d) The files get deleted and cannot be restored again
24. Which process checks to ensure the components of the (e) The file gets copied on the Hard disk
computer are operating and connected properly?
(a) Booting (b) Processing (c) Saving
(d) Editing (e) Starting

17. a b c d e 18. a b c d e 19. a b c d e 20. a b c d e 21. a b c d e


RESPONSE 22. a b c d e 23. a b c d e 24. a b c d e 25. a b c d e 26. a b c d e
GRID 27. a b c d e 28. a b c d e 29. a b c d e 30. a b c d e
y
o
u
rs
m
a
h
b
73

o
o
b
MS Office/ Commands

.w
o
rd
and Shortcut Keys

p
re
s
s
.c
o
m
Max. Marks : 30 No. of Qs. 30 Time : 20 min. Date : ........./......../................

1. The name of a Microsoft Office Word document is displayed 8. The .xls extension is used for ________ files.
in both the ___________ and the taskbar. (a) Windows (b) Access (c) PowerPoint
(a) menu bar (b) taskbar (d) Word (e) Excel
(c) Formatting toolbar (d) Standard toolbar 9. In word, when you indent a paragraph, you
(e) title bar (a) push the text in with respect to the margin
2. Microsoft Office is an example of a _________. (b) change the margins on the page
(a) closed-source software (c) move the text up by one line
(b) open-source software (d) move the text down by one line
(c) horizontal-market software (e) None of these
(d) vertical-market software 10. Using Print Preview is useful when you want to-
(e) compiler (a) Colour the document
3. You cannot link Excel worksheet data to a Word document (b) Save the document
___________. (c) Delete the document
(a) with the right drag method (d) Copy the document
(e) View how trip document will appear when printed
(b) with the hyperlink
11. In word, you can change Page Margins by
(c) with the copy and paste special commands
(a) Dragging the scroll box on the scroll bars
(d) with the copy and paste buttons on the standard
(b) Deleting the margin boundaries on the Ruler
commands
(c) Dragging the margin boundaries on the Ruler
(e) All of these
(d) Clicking the right mouse button on the Ruler
4. In Excel, Charts are created using which option?
(e) None of these
(a) Chart Wizard (b) Pivot Table
12. In Excel, this is a prerecorded formula that provides a shortcut
(c) Pie Chart (d) Bar Chart
for complex calculations
(e) None of these (a) Value (b) Data Series
5. Each cell in a Microsoft Office Excel document is referred to (c) Function (d) Field
by its cell address, which is the (e) None of these
(a) cell’s column label 13. This is not a function category in Excel
(b) cell’s column label and worksheet tab name (a) Logical (b) Data Series
(c) cell’s row label (c) Financial (d) Text
(d) cell’s row and column labels (e) None of these
(e) cell’s contents 14. Text in a column is generally aligned _________
6. This Excel feature includes functions to calculate an Average, (a) justified (b) right
Minimum, Maximum and Count. (c) center (d) left
(a) Format (b) Number (c) AutoSum (e) None of the above
(d) Calculate (e) MIN 15. In Excel _________ contains one or more worksheets.
7. This is a set of values that you want to chart in Excel. (a) Template (b) Workbook
(a) Object (b) Numbers (c) Data Mart (c) Active cell (d) Label
(d) Formulas (e) Data series (e) None of these

1. a b c d e 2. a b c d e 3. a b c d e 4. a b c d e 5. a b c d e
RESPONSE 6. a b c d e 7. a b c d e 8. a b c d e 9. a b c d e 10. a b c d e
GRID 11. a b c d e 12. a b c d e 13. a b c d e 14. a b c d e 15. a b c d e
y
o
u
rs
m
a
h
156 SPEED TEST 73

b
o
o
16. You click at B to make the text _________. 23. ________ is the key to close a selected drop - down list;

b
.w
(a) Italics cancel a command and close a dialog box.

o
(b) Underlined (a) TAB (b) SHIFT

rd
(c) Italics and Underlined (c) ESC (d) F10

p
re
(d) Bold (e) None of these

s
24. ________ is the function key to display save-as box.

s
(e) None of these

.c
17. The shortcut key Ctrl + F in Word is used for (a) F5 (b) F6

o
m
(a) To view document in full view (c) F9 (d) F12
(b) To open the Formula dialog box (e) None of these
(c) To save the file 25. ________ is the Keyboard shortcut key to insert auto sum
(d) To open the Find and Replace dialog box (a) ALT (b) ALT=
(e) None of these (c) ALT+ (d) ALT–
18. To restart the computer ________ key is used. (e) ALT+CTRL
(a) Del + Ctrl (b) Backspace + Ctrl 26. To restart the computer the following combination of keys
(c) Ctrl + Alt + Del (d) Reset is used
(e) None of these (a) Del + Ctrl (b) Backspace + Ctrl
19. ctrl, shift and alt are called ________ keys. (c) Esc + Ctrl (d) Insert + Esc
(a) adjustment (b) function (e) Ctrl + Alt + Del
(c) modifier (d) alphanumeric 27. To make a copy of the current document to disk
(e) None of these (a) Use the “save” command
20. To select or unselect one word to the right (b) This cannot be done
(a) CTRL + SHIFT (c) Use the “duplicate” command
(b) CTRL + SHIFT + UP Arrow (d) Copy the document
(c) CTRL + SHIFT + Down Arrow (e) Use the “save as” command
(d) CTRL + SHIFT + Right Arrow 28. What type of keys are ‘ctrl’ and ‘shift’?
(e) None of these (a) adjustment (b) function
21. To maximize or restore a selected window (c) modifier (d) alphanumeric
(a) CTRL + F7 (b) CTRL + F10 (e) None of these
(c) CTRL + F8 (d) CTRL + F9 29. Which among the following key is used for checking
(e) None of these grammar and spelling?
22. To Copy a picture of the selected window to the clipboard (a) F3 (b) F5
(a) ALT + TAB (c) F7 (d) F2
(b) Ctrl + TAB (e) None of these
(c) ALT + Print Screen 30. While browsing the internet , which of the following key is
used for full screen view?
(d) Both (a) and (b)
(a) F3 (b) F5
(e) None of these
(c) F11 (d) F9
(e) None of these

RESPONSE 16. a b c d e 17. a b c d e 18. a b c d e 19. a b c d e 20. a b c d e


21. a b c d e 22. a b c d e 23. a b c d e 24. a b c d e 25. a b c d e
GRID 26. a b c d e 27. a b c d e 28. a b c d e 29. a b c d e 30. a b c d e
y
o
u
rs
m
a
h
b
74

o
o
b
Softwares/

.w
o
rd
p
Programming

re
s
s
.c
o
m
Max. Marks : 30 No. of Qs. 30 Time : 20 min. Date : ........./......../................
1. What kind of software would you most likely use to keep 8. A sales clerk at a checkout counter scanning a tag on an
track of a billing account? item rather than keying it into the system, is using _______.
(a) Word processing (b) Electronic publishing (a) input automation (b) item data automation
(c) Spreadsheet (d) Web authoring (c) scanning automation
(e) None of these (d) source data automation
2. The two major categories of software include (e) None of these
(a) operating system and utility 9. A document that explains how to use a software program is
called ______ manual
(b) Personal productivity and system
(a) User (b) System
(c) system and application
(c) Software (d) Program
(d) system and utility
(e) Technical
(e) None of these 10. The process that deals with the technical and management
3. A directory within a directory is called. issues of software development is ______
(a) Mini Directory (b) Junior Directory (a) Delivery process (b) Control process
(c) Part Directory (d) Sub Directory (c) Software process (d) Testing process
(e) None of these (e) Monitoring process
4. Which of the following is not a common feature of software 11. Linux is a type of ________ software.
applications? (a) Shareware (b) Commercial
(a) Menus (b) Windows (c) Proprietary (d) Open Source
(c) Help (d) Search (e) Hidden type
(e) None of these 12. The _________ of a system includes the programs or
5. Software applies __________, also called algorithms, to instructions.
process data. (a) hardware (b) icon
(a) arithmetic (b) procedures (c) information (d) software
(c) objects (d) rules (e) None of these
(e) None of these 13. The software that allows users to surf the Internet is called
6. Application software is designed to accomplish _________. a/ an _________
(a) real-world tasks (b) computer-centric tasks (a) Search engine (b) Internet Service Provider (ISP)
(c) gaming tasks (d) operating-system tasks (c) Multimedia application
(e) None of these (d) Browser (e) None of these
14. The ______ of software contains lists of commands and
7. Which is the best definition of a software package?
options.
(a) An add-on for your computer such as additional
(a) menu bar (b) tool bar
memory
(c) title bar (d) formula bar
(b) A set of computer programs used for a certain function
(e) None of these
such as word processing
15. ____ is a procedure that requires users to enter an
(c) A protection you can buy for a computer
identification code and a matching password.
(d) The box, manual and license agreement that accom-
(a) Paging (b) Logging on
pany commercial software.
(c) Time-sharing (d) Multitasking
(e) None of these
(e) None of these

1. a b c d e 2. a b c d e 3. a b c d e 4. a b c d e 5. a b c d e
RESPONSE 6. a b c d e 7. a b c d e 8. a b c d e 9. a b c d e 10. a b c d e
GRID 11. a b c d e 12. a b c d e 13. a b c d e 14. a b c d e 15. a b c d e
y
o
u
rs
m
a
h
158 SPEED TEST 74

b
o
o
16. A program that works like a calculator for keeping track of 23. A program that enables you to perform calculations

b
.w
money and making budgets __________. involving rows and columns of numbers is called a
_________.

o
(a) calculator (b) scholastic (c) keyboard

rd
(d) spreadsheet (e) None of these (a) spreadsheet program

p
re
17. Compiling creates a(n) ___________. (b) word processor

s
(a) program specification (c) graphics package

s
.c
(b) algorithm (c) executable program (d) window

o
m
(d) subroutine (e) None of these (e) None of the above
18. Multiprogramming systems: 24. ______ is a feature for scheduling and multiprogramming
(a) are easier to develop than single programming systems. to provide an economical interactive system of two or more
(b) execute each job faster. users
(c) execute more jobs in the same time period. (a) Time sharing (b) Multitasking
(d) use only one large mainframe computer. (c) Time tracing (d) Multiprocessing
(e) None of these (e) None of these
19. A ________ contains specific rules and words that express 25. A programming language having a ______ is slow in
the logical steps of an algorithm. execution
(a) programming language (a) Interpreter (b) Compiler
(b) programming structure (c) Assembler (d) Linker
(c) syntax (e) none of these
(d) logic chart 26. Assembly language is
(e) None of these (a) Machine Language
20. _________ is a set of keywords, symbols, and a system of (b) High-level programming language
rules for constructing statements by which humans can (c) A low-level programming language
communicate the instructions to be executed by a computer. (d) Language for assembling computers
(a) A computer program (e) None of these
(b) A programming language 27. What is correcting erros in a program called?
(c) An assembler (a) Compiling (b) Debugging
(d) Syntax (c) Grinding (d) Interpreting
(e) None of these (e) None of these
21. ________ is the process of finding errors in software code. 28. A (n)___is a program that makes the computer easier to use.
(a) Compiling (a) utility (b) application
(b) Assembling (c) operating system (d) network
(c) Interpreting (e) None of these
(d) Debugging 29. What is used in most programs that is a part of a program
(e) None of these and guides the user through certain steps?
22. Documentation of computer programs is important so that (a) Software (b) Wizard
(a) users can learn how to use the program (c) Wiki (d) Language
(b) other programmers can know how to maintain the (e) None of these
program 30. Which of the following was used in programming the first
(c) the programmer can see why the code is written that computers?
way while hunting for sources of error (a) Object code (b) Source Code
(d) All of the above (c) Machine Language (d) Assembly Language
(e) None of the above (e) None of these

RESPONSE 16. a b c d e 17. a b c d e 18. a b c d e 19. a b c d e 20. a b c d e


21. a b c d e 22. a b c d e 23. a b c d e 24. a b c d e 25. a b c d e
GRID 26. a b c d e 27. a b c d e 28. a b c d e 29. a b c d e 30. a b c d e
y
o
u
rs
m
a
h
b
75

o
o
b
.w
Internet, Networking and

o
rd
p
Computer Abbreviations

re
s
s
.c
o
m
Max. Marks : 30 No. of Qs. 30 Time : 20 min. Date : ........./......../................

1. Which of the following is a Web browser? 9. Which of the following is not a term pertaining to the
(a) Paint (b) Power Point Internet?
(c) Fire fox (d) Word (a) Keyboard (b) Link
(e) All are Web browsers (c) Browser (d) Search Engine
2. Junk e-mail is also called __________.
(e) Hyperlink
(a) spam (b) spoof
10. In a web site, the ‘home’ page refers to –
(c) sniffer script (d) spool
(e) None of these (a) the best page (b) the last page
3. Most World Wide Web pages contain commands in the (c) the first page (d) the most recent page
language _________. (e) the oldest page
(a) NIH (b) URL 11. An e-mail address typically consists of a user ID followed
(c) HTML (d) IRC by the _________ sign and the name of the e-mail server
(e) FTP that manages the user’s electronic post office box.
4. Computers connected to a LAN (local Area Network) (a) @ (b) #
can _________.
(c) & (d) «
(a) run faster
(e) None of these
(b) go on line
(c) share information and/or share peripheral equipment 12. A Web __________ consists of one or more Web pages
(d) E-mail located on a Web server.
(e) None of these (a) hub (b) site
5. What does a Web site address uniquely specify? (c) story (d) template
(a) Web browser (b) Web site (e) None of these
(c) PDA (d) Storage 13. Programs such as Internet Explorer that serve as navigable
(e) Hard-disk windows into the Web are called
6. Web pages are saved in __________ format. (a) Hypertext (b) Networks
(a) http:// (b) HTML
(c) Internet (d) Web browsers
(c) DOC (d) URL
(e) None of these
(e) None of these
7. What are the two parts of an E-mail address? 14. A word in a web page that, when clicked, opens another
(a) User name and street address document.
(b) Legal name and phone number (a) anchor (b) URL
(c) User name and domain name (c) hyperlink (d) reference
(d) Initials and password (e) None of these
(e) login name and password 15. WWW stands for ________.
8. An educational institution would generally have the (a) World Work Web (b) Wide Work Web
following in its domain name –
(c) Wide World Web (d) World Wide Web
(a) .org (b) .edu
(e) None of the above.
(c) .inst (d) .com
(e) .sch

1. a b c d e 2. a b c d e 3. a b c d e 4. a b c d e 5. a b c d e
RESPONSE 6. a b c d e 7. a b c d e 8. a b c d e 9. a b c d e 10. a b c d e
GRID 11. a b c d e 12. a b c d e 13. a b c d e 14. a b c d e 15. a b c d e
y
o
u
rs
m
a
h
160 SPEED TEST 75

b
o
o
16. The collection of links throughout the Internet creates an 23. FAT stands for

b
.w
interconnected network called the _________. (a) File Activity Table

o
(a) WWW (b) Web (b) File Allocation Table

rd
(c) World Wide Web (d) All of the above (c) File Access Tape

p
re
(e) Wide Area Web (d) File Accommodation Table

s
17. ASCII stands for (e) None of these

s
.c
(a) American Special Computer for Information Interaction 24. ‘IMAP’ in computer language stands for

o
m
(b) American Standard Computer for Information (a) Internet Message Access Protocol
Interchange (b) Internet Money Access Protocol
(c) American Special Code for Information Interchange (c) Intrusion Message Access Protocol
(d) Internal Message Access Protocol
(d) American Special Computer for Information
(e) International Marketing Authority Protocol
Interchange
25. The BIOS is the abbreviation of
(e) American Standard Code for Information Interchange
(a) Basic Input Output System
18. POST stands for
(b) Best Input Output System
(a) Power on Self Test
(c) Basic Input Output Symbol
(b) Program on Self Test (d) Base Input Output System
(c) Power on System Test (e) None of these
(d) Program on System Test 26. The computer abbreviation KB usually means
(e) Power Off System Test (a) Key Block (b) Kernal Boot
19. CPU stands for _________ (c) Key Byte (d) Kit Bit
(a) Computer Processing Unit (e) Kilo Byte
(b) Central Processing Unit 27. URL stands for
(c) Computer Protection Unit (a) Universal Resource List
(d) Central Processing Upload (b) Universal Research List
(e) None of the above (c) Uniform Resource List
20. OCR stands for _________ (d) Uniform Research Locater
(a) Optical Character Recognition (e) Uniform Resource Locater
28. The meaning of double-click is
(b) Optical CPU Recognition
(a) pushing and releasing the main mouse button twice in
(c) Optimal Character Rendering
rapid succession when the on-tree mouse pointer is
(d) Other Character Restoration positioned over the desired item
(e) None of these (b) appearance of an icon
21. Line printer speed is specified in terms of : (c) to take a selection from the document and move it to
(a) LPM (Line per minute) the clipboard
(b) CPM (Character per minute) (d) All of the above
(c) DPM (e) None of these
(d) Any of the above 29. BPS stands for
(e) None of these (a) Bits Per Second (b) Bits Per Season
22. What does ‘DOS’ stand for ? (c) Bytes Per Second (d) Bits Per System
(a) Disk Originating System (e) None of these
(b) Dynamic Operating System 30. Reusable optical storage will typically have the
acronym _________
(c) Disk Operating System
(a) CD (b) DVD
(d) Default Operating System
(c) ROM (d) RW
(e) None of these
(e) None of these

RESPONSE 16. a b c d e 17. a b c d e 18. a b c d e 19. a b c d e 20. a b c d e


21. a b c d e 22. a b c d e 23. a b c d e 24. a b c d e 25. a b c d e
GRID 26. a b c d e 27. a b c d e 28. a b c d e 29. a b c d e 30. a b c d e
y
o
u
rs
m
a
h
b
76

o
Fundamentals of

o
b
.w
o
Marketing,

rd
p
re
Product and Branding

s
s
.c
o
m
Max. Marks : 30 No. of Qs. 30 Time : 20 min. Date : ........./......../................

1. Marketing is: (c) Lifelong relationship with the buyer


(a) Only selling (d) All of these
(b) meeting human & social needs while earning profits (e) None of these
(c) focus on customer 9. A Market Plan is ________.
(d) focus on producing goods/ service (a) company's prospectus
(e) Both (b) and (c) (b) Memorandum of Association
2. Long term objective of marketing is (c) document for marketing strategies
(a) customer satisfaction (d) business goals
(b) profit maximisation (e) action plan for better production
(c) cost cutting 10. Marketing helps in _______.
(d) profit maximisation with customer (a) boosting production
(e) None of these (b) getting new clients
3. Market information means (c) interacting with strangers
(a) knowledge of companies (d) All of these
(b) cross-country information (e) None of these
(c) knowledge of related markets 11. Marketing is the art of ________.
(d) knowledge of current customers (a) buying more (b) paying more
(e) None of these
(c) selling more (d) talking more
4. Marketing and Selling are
(a) not required if profit is high (e) only (a) & (b)
(b) not required if sales are high 12. Selling is _______.
(c) not required in monopolistic conditions (a) different from Marketing
(d) All of the above (b) a sub-function of marketing
(e) None of these (c) same as Marketing
5. Direct Marketing is neccessary for (d) more than Marketing
(a) having a focussed approach (e) All of these
(b) boosting sales 13. Social Marketing is _______.
(c) better customer contacts (a) Share market prices
(d) All of the above (b) Marketing by the entire society
(e) None of these
(c) Internet Marketing
6. Marketing is required for
(a) boosting production (d) Marketing for a social cause
(b) reducing costs (e) Society bye-laws
(c) boosting profits 14. Marketing is the combined study of
(d) improving customer service (a) Buyer’s behaviour and consumer tasks
(e) All of the above (b) Product demand and Product supply
7. Marketing is successful when (c) Brand building and Publicity
(a) demand exceeds supply (d) Sales force abilities and customer response
(b) supply exceeds demand (e) All of the above
(c) exports are heavy and costly 15. The sales process begins with
(d) salesmen are effective (a) customer identification
(e) All the above (b) lead generation
8. In marketing the benefits of selling extend to (c) sales presentation
(a) Only products and services (d) sales closure
(b) Only after sales services (e) sales meet

1. a b c d e 2. a b c d e 3. a b c d e 4. a b c d e 5. a b c d e
RESPONSE 6. a b c d e 7. a b c d e 8. a b c d e 9. a b c d e 10. a b c d e
GRID 11. a b c d e 12. a b c d e 13. a b c d e 14. a b c d e 15. a b c d e
y
o
u
rs
m
a
h
162 SPEED TEST 76

b
o
o
16. XXX is selecting and analyzing a target market and 24. To investigate new markets _____ management function is

b
developing a marketing mix to gain long-run competitive important.

.w
advantages. XXX is creating a. (a) Finance functions. (b) Marketing.

o
rd
(a) Corporate strategy (b) Target design (c) Production. (d) HRM.

p
(c) Mix strategy (d) Marketing strategy (e) None of these

re
(e) None of these

s
25. _____ is a “category killer”.

s
17. Critical success factors for a firm includes ________.

.c
(a) Products which stock must have products that need to

o
(a) Changing lifestyles and attitudes be stocked by retailers due to consumer demand like

m
(b) Low-cost production efficiency Coca Cola & Kellogg's.
(c) Both (a) and (b) (b) These are speciality stores with a deep product line to
(d) Marketing strategy be sold in restricted shop space
(e) None of the above (c) These are retail outlets with a narrow product focus
18. A differentiated product may be unique by itself but it will but sell products at low prices by bulk buying, low
only be successful only ________ . margins and selling high volumes
(a) if it satisfies customers' needs (d) These are retail outlets with a wide product focus but
(b) if price differential is minimal with a wide width and depth to products
(c) if brand can be classed as aspirational (e) These are retail outlets with a narrow product focus
(d) differentiated products will always be successful but with wide width and depth
(e) None of these 26. Product life cycle theory maximizes profit at.
19. Establishing and maintaining a distinctive place in the market (a) Developed Stage (b) Early Stage
for an organization/product is ________. (c) Matured Stage (d) Declined Stage
(a) Profiling (b) Profiling segmentation (e) Cannot be predicted
(c) Segmentation (d) Positioning 27. Marketing Plans are used for
(e) None of these (a) doing Research by Marketing students
20. _____ of ad means how frequently you should expose your (b) planning Departments
target group to your message. (c) purchase of consumable items from retail outlets
(a) Frequency (b) Copy (d) All of the above
(c) Copy strategy (d) Media (e) None of these
(e) None of these 28. Product mix means
21. Market is divided into groups on the basis of age, family (a) distributing mix products
size, gender, income, occupation, education, religion, race, (b) collecting ideas to sell better
generation, nationality, or social class is the best description
(c) satisfying the customer
of _____ .
(d) bundle of products required by the customer
(a) Demographics (b) Psychographics
(e) products designed by the Company
(c) Behavioral (d) Geographic
(e) None of these 29. Buyer Resistance' means _____.
22. What approach should a brand manager adopt to know the (a) Buyer's interest in the product
status of a brand in terms of consumer perceptions? (b) Buyer being aggressive with the seller
(a) Compare two or three brands (c) Buyer's hesitation in buying the product
(b) Analyze market segmentations (d) Buyer becoming a seller
(b) Select potential target markets (e) Buyer purchasing the product
(c) Understand customer's needs
30. Direct Marketing is useful for _____ .
(e) None of these
(a) Designing Products
23. Introducing additional items in the same product category
by adding new flavors, forms, colors, ingredients or package (b) Sending e-mails
sizes, under the same brand name, is _____. (c) Increased production
(a) Line extensions (b) Product mix (d) Bigger job opportunities
(c) Interactive marketing (d) Service intangibility (e) None of these
(e) None of these

16. a b c d e 17. a b c d e 18. a b c d e 19. a b c d e 20. a b c d e


RESPONSE 21. a b c d e 22. a b c d e 23. a b c d e 24. a b c d e 25. a b c d e
GRID 26. a b c d e 27. a b c d e 28. a b c d e 29. a b c d e 30. a b c d e
y
o
u
rs
m
a
h
b
77

o
o
b
Market Situations Based on

.w
o
rd
Price, Distribution,

p
re
Promotion and Advertising

s
s
.c
o
m
Max. Marks : 30 No. of Qs. 30 Time : 20 min. Date : ........./......../................
1. A company's own retail outlets are meant _____. 8. Advertising copy _____.
(a) To avoid the threat of distributors' power (a) Provides continuity in a brand's advertising
(b) To own and control the distribution channel (b) Help a brand achieve distinctiveness
(c) Distribution is profitable (c) Provides a common benchmark on which all concerned
(d) All of the above in the company and the agency can evaluate merits of
(e) None of the above advertising campaign
2. Advertising _____ can attract consumers only if it is based (d) All of the above
on their needs. (e) None of these
(a) Reach (b) Copy 9. Duration of _____ should be short and should not be
(c) Frequency (d) Media repeated.
(e) None of these (a) Sales promos (b) Market promos
(c) Brand promos (d) Product promos
3. Major source of power in a distribution channel is the _____.
(e) None of these
(a) Company (b) Brand 10. An effective advertising campaign _____
(c) Distributor (d) Customer (a) Revolves around a strong central idea
(e) None of these (b) Should appeal to consumers self interest
4. Factors affecting choice of distribution channel include (c) Must not be generalised
_____. (d) All of the above
(a) Customer value (b) Sales revenues (e) None of these
11. Prices of luxury product are explained by _____ .
(c) Both [a] and [b] (d) Customer services
(a) Plus-one pricing
(e) None of the above (b) Skim pricing
5. Air India runs TV commercials that show its staff going out (c) Strategic account pricing
of their way to help customers. An important secondary (d) Segment pricing
audience for these ads is: (e) None of these
(a) Civil aviation authority 12. _____ about Place/Distribution decisions is CORRECT?
(b) Competitors (a) Product classes are not related to Place objectives
(c) Air India employees (b) The product life cycle is not related to Place objectives
(c) Place decisions are short-term decisions that are easy
(d) Air travelers to change
(e) None of these (d) Different market segments may require separate Place
6. Pricing models offers opportunity to set different prices for arrangements.
different needs is _____. (e) None of these
(a) Segment pricing 13. Addition of 2.25 liter bottle by Coca Cola will ____ .
(b) Skim pricing (a) Increase customer base and usage
(c) Value-in-use pricing (b) Enhance customer loyalty
(c) Generate more profit
(d) Strategic account pricing
(d) Develop brand image
(e) None of these (e) All of the above
7. Advertising helps sales promotion by creating awareness 14. Communication through a news story about an organization
and comprehension that creates _____. and its products that is transmitted through a mass medium
(a) Customer pull (b) Customer push at no charge is ____.
(c) Customer loyal (d) Customer image (a) Advertising (b) Sales promotion
(e) None of these (c) Personal selling (d) Publicity
(e) None of these

1. a b c d e 2. a b c d e 3. a b c d e 4. a b c d e 5. a b c d e
RESPONSE 6. a b c d e 7. a b c d e 8. a b c d e 9. a b c d e 10. a b c d e
GRID 11. a b c d e 12. a b c d e 13. a b c d e 14. a b c d e
y
o
u
rs
m
a
h
164 SPEED TEST 77

b
o
o
15. _____ is a disadvantage of using an agent in a channel of 22. If Marketing is done effectively, _____ is not required

b
distribution?

.w
(a) Advertisment (b) Pubilicity
(a) Length of channel (b) Cost factor (c) Market Research (d) Market Segmentation

o
(c) Lack of control (d) Speed of distribution

rd
(e) None of these
(e) Absence of market knowledge

p
23. Sales forecasting involves

re
16. _____ enables a company to contr ol channels of
(a) sales planning (b) sales pricing

s
s
distribution?

.c
(c) distribution channels (d) consumer tastes
(a) Vertical marketing system

o
(e) All of these

m
(b) Franchising
24. Proper pricing is essential for ____.
(c) Exclusive dealing arrangements
(a) extra charges for extra services
(d) Vertical integration
(b) levy of VAT
(e) All of the above
(c) good customer service
17. Distribution where a limited number of outlets in a
(d) putting burden on the customer
geographical area to sell its products are used is called
(e) service with extra facilities
_____?
25. Selling skills are measured by ____.
(a) Exhaustive distribution
(a) number of goods sold
(b) Exclusive distribution
(b) amount of profit earned
(c) Intensive distribution
(c) number of customers converted
(d) Selective distribution
(d) All of the above
(e) Segmented distribution
(e) None of these
18. Power in marketing channels is now with _____ ?
26. Promotion means
(a) Distributor (b) Manufacturer
(a) additional responsibility
(c) Retailer (d) Wholesaler
(b) undertaking research in marketing
(e) Consumer
(c) advertisement and publicity for marketing
19. ______ is a strength associated with franchising?
(d) going up the promotional ladder
(a) Reduces marketing promotional and administration
(e) All of the above
costs
27. Negotiation skills help in ____.
(b) Goal conflict does not arise
(a) evolving a consensus
(c) Reduces levels of channel conflict within the channel
(b) breaking the ice
(d) Combine the strengths of a large sophisticated
(c) carry marketing further
marketing-oriented organization with the energy and
(d) Mutual win-win situation
motivation of a locally owned outlet
(e) All of these
(e) All of the above
28. Relationship Marketing is useful for
20. Best method of advertisement is
(a) trade between relatives
(a) glow sign boards (b) internet
(b) trade between sister concerns
(c) Television (d) Consumer awareness
(c) cross-selling of products
(e) customer satisfaction
(d) preparing a list of relatives
21. A form of distribution in which manufacturer makes an agreement
(e) There is no such term as Relationship Marketing
with a middleman in each market stipulating that the distribution
29. Good Public Relations means improved _____.
of the product will be confined in that area is:
(a) Marketing skills (b) Brand Image
(a) mass distribution
(c) Customer Service (d) All of these
(b) exclusive agency distribution
(e) None of these
(c) selective distribution
30. One method of Market Monitoring is ____.
(d) price based distribution
(a) Monitor Performance of sales staff.
(e) None of these
(b) Monitor the SENSEX
(c) Monitor Media Outlets
(d) Monitor profits
(e) None of these

15. a b c d e 16. a b c d e 17. a b c d e 18. a b c d e 19. a b c d e


20. a b c d e 21. a b c d e 22. a b c d e 23. a b c d e 24. a b c d e
RESPONSE
25. a b c d e 26. a b c d e 27. a b c d e 28. a b c d e 29. a b c d e
GRID
30. a b c d e
y
o
u
rs
m
a
h
b
78

o
o
b
.w
Market Segmentation,

o
rd
p
Targeting and Positioning

re
s
s
.c
o
m
Max. Marks : 30 No. of Qs. 30 Time : 20 min. Date : ........./......../................
1. 4 P's of Marketing are ____. 9. Positioning has to stem from the point of view of _____.
(a) Primary Marketing Techniques (a) Customers' (b) Competitors'
(b) Person, Place, Product and Promotion (c) General Managers' (d) Brand Owners'
(c) Promoting Authority (e) None of these
(d) Purpose, Place, Passion, and Product 10. _____ is a marketer's major positioning tool that has a direct
(e) None of these impact on product performance and is linked to customer
2. Qualities essential in good marketing are ____. value.
(a) aggressiveness (b) pushy (a) Product quality (b) Social marketing
(c) perseverance (d) politeness (c) Specialty marketing (d) Production quality
(e) Only (c) and (d) (e) None of these
3. ______ is not a part of 7 P's of Marketing. 11. ______ determines why customers buy?
(a) Product (b) Price (a) Customer needs analysis
(c) Production (d) Promotion (b) Brand-based customer model
(e) People (c) Good brand promise
4. Market Share means _____. (d) Brand management process
(a) share market (b) share prices (e) None of these
(c) IPOs (d) Scope for marketing 12. Marketers need to position their brands clearly in target
(e) Share of business among pproducers customers' minds. The strongest brands go beyond attributes
5. If lots of customers like the brand and are inclined to be or benefit positioning. They are positioned on the basis of
bound into a contract, they would be known as _____. _____.
(a) Loyal customer (b) Difficult customer (a) Desirable benefit
(c) Potential customer (d) Finicky customer (b) Good packaging
(e) None of these (c) Strong beliefs and values
6. _____ evokes a hierarchical set of customer response effects (d) Service inseparability
i.e. building awareness, comprehension, intentions, and (e) None of these
actions.
13. Target marketing featuring customized marketing
(a) Distribution (b) Communication
programmes is _______.
(c) Merchandizing (d) Branding
(a) Individual marketing (b) Segment marketing
(e) None of these
(c) Family marketing (d) Local marketing
7. Apart from the 'four Ps' of marketing mix, the three additional
(e) Niche marketing
elements of service brands are people, process, and ____ .
(a) Physical evidence 14. _____ segmentation divides the market into different units
(b) Physiological evidence like nations, states, regions, cities or neighbourhoods.
(c) Psychological evidence (a) Geographic (b) Demographic
(d) Packaging (c) Psychographic (d) Behavioral
(e) None of these (e) Socio-economic
8. Unique Selling Proposition (USP) started in _____. 15. _____ segmentation is where the market is divided on the
basis of age, family size, life cycle, gender, income,
(a) Advertising era (b) Image era
occupation, education and religion .
(c) Product era (d) Positioning era
(a) Geographic (b) Demographic
(e) None of these (c) Psychographic (d) Behavioral
(e) Socio-economic
1. a b c d e 2. a b c d e 3. a b c d e 4. a b c d e 5. a b c d e
RESPONSE 6. a b c d e 7. a b c d e 8. a b c d e 9. a b c d e 10. a b c d e
GRID 11. a b c d e 12. a b c d e 13. a b c d e 14. a b c d e 15. a b c d e
y
o
u
rs
m
a
h
166 SPEED TEST 78

b
o
o
16. To be useful market segments must assess on _____ criteria. 24. Motivation is essential to effective marketing. What other

b
(a) five (b) two qualities are required for marketing?

.w
(c) three (d) four (a) Confidence

o
rd
(e) Six (b) Effective Communication skills

p
17. Market Segmentation means (c) Team work

re
(a) dividing the market into groups (d) Perseverance

s
s
(b) segmenting by age

.c
(e) All of the above
(c) segmenting by tastes

o
25. Market Penetration is possible through

m
(d) geographic segmenting
(a) more calls to the same buyers
(e) None of the above
(b) more calls to potential buyers
18. In Marketing, Market penetration means
(c) surrogate marketing
(a) entering customers houses
(b) covering stores and shops (d) alternate marketing
(c) covering a wide market (e) All of these
(d) All of the above 26. Market penetration connotes
(e) None of these (a) covering a wider area
19. Rural Marketing is more effective if arranged through. (b) entering sellers' houses
(a) melas (c) covering all shops and business houses
(b) village fairs (d) All of these
(c) door to door campaign (e) None of these
(d) All of these 27. Lead generation means ____.
(e) None of these (a) Tips for selling tactics
20. ‘Buyer Resistance’ means _____. (b) Tips for more efficient production
(a) Buyer’s interest in the product (c) Develop leaders
(b) Buyer fighting with the seller (d) sources for prospective clients
(c) Buyer’s hesitation in buying the product (e) None of these
(d) Buyer becoming a seller
28. Marketing orientation focuses on _____?
(e) Buyer buying the product
(a) Customers (b) Suppliers
21. Opportunities for growth and expansion are identified by
finding _____ (c) Competitors (d) Employees
(a) Customers' beliefs about the segment (e) All of the above
(b) Customers believe about competitors 29. ‘Push’ marketing style requires _____.
(c) Customer's perceptions about the brand (a) Proper planning
(d) Customer's response about the products (b) Good pushing strength
(e) None of these (c) Teamwork
22. What explains Maslow's Motivation Theory best? (d) Ability to identify products
(a) Importance of motivation of customer development (e) Aggressive marketing
(b) When people are driven by particular need at particular 30. CRM (Customer Relationship Management) is _____.
times (a) A pre-sales activity
(c) When human needs are arranged in a hierarchy (b) A tool for lead generation
(d) All of these (c) An on going daily activity
(e) Only (b) and (c) (d) Task of a DSA
23. Customer Database is used by
(e) Customer complaint cell
(a) individuals (b) institutions
(c) builders (d) marketing experts
(e) None of these

16. a b c d e 17. a b c d e 18. a b c d e 19. a b c d e 20. a b c d e


RESPONSE 21. a b c d e 22. a b c d e 23. a b c d e 24. a b c d e 25. a b c d e
GRID 26. a b c d e 27. a b c d e 28. a b c d e 29. a b c d e 30. a b c d e
y
o
u
rs
m
a
h
b
79

o
o
Modern Marketing /

b
.w
o
Marketing in

rd
p
re
Banking Industry

s
s
.c
o
m
Max. Marks : 30 No. of Qs. 30 Time : 20 min. Date : ........./......../................
1. Modern marketing includes (b) Tele-marketing
(a) publicity on internet (c) Door-to-door marketing
(b) advertisement on internet
(d) E-mail solicitation
(c) Bulk emails
(e) None of these
(d) telemarketing
(e) All of these 9. Web marketing involves
2. e-marketing is same as _______. (a) Selling web cameras (b) Web advertisements
(a) virtual marketing (b) digital marketing (c) e-mail chatting (d) Browsing the web
(c) real time marketing (d) All of these (e) Door-to-door canvassing
(e) None of these 10. Online Marketing is mostly useful for marketing of
3. Digital marketing is selling ______. (a) saving accounts (b) credit cards
(a) digital goods (b) calculators (c) home loans (d) NRI deposits
(c) through internet (d) All of these (e) business accounts
(e) None of these 11. The best promotional tool in any marketing is _______
4. Online Marketing is useful for ______. (a) Pamphlets
(a) Selling Old Products (b) Newsletters
(b) Sending e-mails (c) Word of mouth publicity
(c) Increasing production (d) Regional Advertisement
(d) Additional job opportunities (e) Viral marketing
(e) Higher expenses
12. Internet marketers are using ________ as a form of word of
5. Online value proposition should _______.
mouth, or word of mouse, to draw attention to their sites.
(a) Be communicated to site visitors and in all marketing
(a) event marketing (b) subliminal marketing
communications
(c) viral marketing (d) place marketing
(b) Be a clear differentiator from online competitors
(e) public relations
(c) Target market segment(s) that the proposition will
13. The major marketing developments as we enter the new
appeal to
millennium can be summed up in a single theme:
(d) Given financial back up
(a) innovation (b) the Internet
(e) All of the above
(c) virtuality (d) connectedness
6. Achieving marketing objectives through use of electronic
(e) None of these
communications technology is _______.
14. Today marketing is:
(a) E-marketing (b) E-business
(a) Product driven
(c) Internet marketing (d) E-commerce
(b) Services driven
(e) None of the above
(c) Improvement of bottom line (Profitability)
7. Direct online contribution effectiveness is the _______.
(d) Cost conscious
(a) Reach of audience volume of a site
(e) Customer driven market
(b) Proportion of sales influenced by the web site
15. Modern marketing calls for more than developing a good
(c) Proportion of business turnover ach ieved by
product, pricing it attractively, and making it accessible.
e-commerce transactions
Companies must also ________ with present and potential
(d) First and third option above
stakeholders, and the general public.
(e) None of the above
(a) attract (b) reach
8. Modern marketing EXCLUDES _______.
(c) relate to (d) advertise to
(a) Digital marketing
(e) communicate
1. a b c d e 2. a b c d e 3. a b c d e 4. a b c d e 5. a b c d e
RESPONSE 6. a b c d e 7. a b c d e 8. a b c d e 9. a b c d e 10. a b c d e
GRID 11. a b c d e 12. a b c d e 13. a b c d e 14. a b c d e 15. a b c d e
y
o
u
rs
m
a
h
168 SPEED TEST 79

b
o
o
16. In today's changing banking scenario, aggressive promotion 24. Difference between Direct and indirect Bank Marketing is

b
.w
of business is necessary where the competition exists on (a) Direct Marketing is to Bank's employees. Indirect is to

o
(a) branch up-keep outsiders

rd
(b) expeditious service (b) Direct Marketing is to outsiders. Indirect is to

p
re
(c) use of advanced digital technology employees

s
(d) good customer service (c) Direct Marketing is to Bank's owners. Indirect is to

s
.c
(e) All of these outsiders

o
m
17. Bank marketing means (d) Direct Marketing is to other Bank's employees. Indirect
(a) selling by banks is to outsiders
(b) buying by banks (e) None of these
(c) merger of banks 25. Target market for Home Loans is
(d) selling bank's products and services (a) builders (b) housing societies
(e) selling products in banks (c) agriculturists (d) All of these
18. Marketing is not required in ________. (e) None of these
(a) Selling Credit/ Debit Cards 26. The USP of a Credit Card is
(b) Net Banking (c) Corporate Loans (a) cashless operations
(d) Retail Loans (e) All of these (b) only for HNIs
19. Marketing of Internet Banking means (c) only for men
(a) meeting of Banks on the net (d) only for employed persons
(b) net practice (e) transactions through cheque book
(c) marketing usage of Banking transactions through 27. EMI can be a marketing tool if
internet (a) EMI is increasing
(d) transactions with foreign banks (b) It is very high
(e) All of the above (c) It is very low
20. Marketing in banks is (d) EMI has no impact on marketing
(a) a one-day function (b) a one-man function (e) EMI is a flat rate
(c) a one-off affair (d) All of these 28. Banks sell insurance for
(e) None of these (a) increasing deposits
21. Target group for marketing Internet Banking is _______. (b) increasing loans
(a) all customers (c) increasing clients
(b) all literate customers (d) earning more profits
(c) all computer literate customers (e) taking over insurance companies
(d) only borrowers 29. Savings Accounts can be opened by _______
(e) All of these (a) All individuals fulfilling KYC norms
22. Digital Banking is available through _______. (b) All tax payers only
(a) Mobile phones (b) Internet (c) All individuals above the age of 18
(c) Telephones (d) All of these (d) All businessmen only
(e) None of these (e) All students below the age of 18
23. Bancassurance can be sold to _____. 30. A short term loan is repayable within _______
(a) Banks (b) Insurance companies (a) 20 years
(c) Insurance agents (d) Bank customers (b) 3 years
(e) All of the above (c) As per the borrowers’ wish
(d) As per the guarantor’s wish
(e) There is no need to repay short term loans

16. a b c d e 17. a b c d e 18. a b c d e 19. a b c d e 20. a b c d e


RESPONSE 21. a b c d e 22. a b c d e 23. a b c d e 24. a b c d e 25. a b c d e
GRID 26. a b c d e 27. a b c d e 28. a b c d e 29. a b c d e 30. a b c d e
y
o
u
rs
m
a
h
80

b
o
Section Test :

o
b
.w
o
Computer Knowledge/

rd
p
re
Marketing Aptitude

s
s
.c
o
m
Max. Marks : 40 No. of Qs. 40 Time : 25 min. Date : ........./......../................
1. Devices that enter information and let you communicate with the (a) virtual private network (b) LAN
computer are called _______. (c) intranet (d) extranet
(a) Software (b) Output devices (e) internet
(c) Hardware (d) Input devices 10. Tangible, physical computer equipment that can be seen and
(e) Input/Output devices touched is called _______
2. What is the function of the Central Processing Unit of a Computer? (a) hardware (b) software
(a) Creates invoices (c) storage (d) input/output
(b) Performs calculations and processing (e) None of these
(c) Deletes Data 11. Which of the following is the second largest measurement of RAM?
(d) Corrupts the data (a) Terabyte (b) Megabyte
(e) None of these (c) Byte (d) Gigabyte
3. All the characters that a device can use is called its ? (e) Mega Hertz
(a) Skill Set (b) Character Alphabet 12. What resides on the motherboard and connects the CPU to other
(c) Character Codes (d) Keyboard Characters components on the motherboard ?
(e) Character Set (a) Input Unit (b) System Bus
4. If your computer keeps rebooting itself, then it is likely that (c) ALU (d) Primary Memory
_______ (e) None of these
(a) It has a virus 13. Supercomputers _______
(b) It does not have enough memory (a) are smaller in size and processing capability than mainframe
(c) There is no printer computers
(d) There has been a power surge (b) are common in majority of households
(e) It needs a CD-ROM (c) contain thousands of microprocessors
(d) are rarely used by researchers due to their lack of computing
5. What utility to you use to transfer files and exchange messages ?
capacity
(a) Web browsers (b) WWW
(e) are of the same size as laptops
(c) Email (d) Hypertext
14. The basic computer processing cycle consists of _______
(e) search engines
(a) input, processing and output
6. Which unit controls the movement of signals between CPU and I/
(b) systems and application
O?
(c) data, information and applications
(a) ALU (b) Control Unit
(d) hardware, software and storage
(c) Memory Unit (d) Secondary Storage
(e) None of these
(e) None of these 15. The system unit
7. The three main parts of the processor are _______ (a) coordinates input and output devices
(a) ALU, Control Unit and Registers (b) is the container that houses electronic components
(b) ALU, Control Unit and RAM (c) is a combination of hardware and software
(c) Cache, Control Unit and Registers (d) controls and manipulates data
(d) Control Unit, Registers and RAM (e) does the arithmetic operations
(e) RAM, ROM and CD-ROM 16. System software
8. Which of the following does not relate to Input Unit ? (a) allows the user to diagnose and troubleshoot the device
(a) If accepts data from the outside world. (b) is a programming language
(b) It converts data into binary code that is understandable by (c) is part of a productivity suite
the computer (d) is an optional form of software
(c) It converts binary data into the human readable form that is (e) helps the computer manage internal resources
understandable to the users. 17. Microsoft's Messenger allows users to
(d) It sends data in binary form to the computer for further (a) bypass a browser to surf the Web
processing (b) create a blog
(e) None of these (c) communicate via direct live communication
9. Which of the following terms is just the collection of networks that (d) identify and eliminate spam
can be joined together ? (e) make graphic presentations

1. a b c d e 2. a b c d e 3. a b c d e 4. a b c d e 5. a b c d e
RESPONSE 6. a b c d e 7. a b c d e 8. a b c d e 9. a b c d e 10. a b c d e
GRID 11. a b c d e 12. a b c d e 13. a b c d e 14. a b c d e 15. a b c d e
16. a b c d e 17. a b c d e
y
o
u
rs
m
a
h
170 SPEED TEST 80

b
o
o
18. What is the short-cut key to highlight the entire column? (d) Place, People, Product, Policy

b
(a) Ctrl + Page up (b) Ctrl + Page down (e) Promotion, Product, Price, People

.w
(c) Ctrl + Enter (d) Ctrl + Space bar 31. For an economic organization like bank, MIS means

o
(e) Ctrl + C (a) Middle Income Scheme

rd
19. To save the current document or to open a previously saved (b) Management Information System

p
document, _________ is used. (c) Management of Information & Science

re
(a) file menu (b) tools menu (d) Marketing Information System

s
(c) view menu (d) edit (e) Only (b) and (c)

s
.c
(e) review menu 32. Market segmentation is done by firms for all of the following

o
20. In a computer system, _________ device is functionally opposite reasons, except

m
of a keyboard. (a) to increase market share.
(a) joystick (b) mouse (b) to assist new product development.
(c) trackball (d) printer (c) so that they can develop mu lti-purpose advertising
(e) scanner campaigns.
21. Marketing in banks is defined as (d) to extend products into new markets.
(a) Negotiable Instruments Act (e) None of these
(b) Banking Regulation Act 33. Which among the following is not a tool for marketing planning
(c) Reserve Bank of India Act appraisal?
(d) Companies Act (a) External appraisal (b) Internal appraisal
(e) None of these (c) Gap analysis (d) SWOT analysis
22. In marketing terms, attitude can best be defined as a (e) PCOT analysis
(a) rude behaviour of salesperson 34. Cross-selling covers
(b) rude behaviour of consumer (a) identifying customer needs
(c) mental state of consumer (b) matching the products to customer needs
(d) ego of the marketing executive (c) convincing the customers of product benefits
(e) None of these (d) responding to questions and objections of customers
23. According to product life cycle theory, the profit is maximum in (e) All of these
(a) developed Stage (b) early stage 35. Market information means
(c) matured Stage (d) declined Stage (a) knowledge of shops and bazaars
(e) None of these (b) knowledge of shopping malls
24. In banks, loans & advances are considered as (c) knowledge of customer profile and product mix
(a) assets (b) liabilities (d) knowledge of various languages
(c) resources (d) cause of expenditure (e) None of these
(e) None of these 36. Market research is needed for
25. A firm is productively efficient when (a) deciding the market area
(a) it is producing its product or service at the lowest unit cost (b) deciding the right product to be sold
that it can. (c) making proper marketing decisions
(b) it is selling at the lowest price possible. (d) deciding right time to sell
(c) it has the highest labour productivity that it can. (e) All of these
(d) it is making what its customers want. 37. A product life cycle
(e) None of these (a) shows how a products sales or profits, depending on the
26. Vegetable market in India is nearly an example of units used, may rise and fall over its life.
(a) perfect Competition (b) monopoly (b) tells you how long a product will sell for and make a profit.
(c) oligopoly (d) imperfect competition (c) is divided into three stages.
(d) shows how profitable a product will be.
(e) high monopsony
(e) None of these
27. Marketing in banks is
38. The long term objective of marketing is
(a) a one-day function (b) a one-man function
(a) customer satisfaction (b) profit maximisation
(c) a one-off affair (d) All of these
(c) cost cutting
(e) None of these
(d) profit maximisation with customer
28. Modern methods of marketing include
(e) None of these
(a) publicity on the net (b) advertisement on the net
39. Proper pricing is needed for
(c) soliciting business through e-mails (a) extra charges for extra services
(d) telemarketing (e) All of these (b) levy of VAT
29. In a selling process in today’s world (c) good customer service
(a) only standard products are sold (d) putting burden on the customer
(b) no customization required (e) service with extra facilities
(c) the seller need not have product knowledge 40. All of the following are examples of unfair competition, except
(d) the seller should aim at customer satisfaction (a) two firms agreeing to fix their prices.
(e) only quantum of sales matters (b) three companies agreeing to share a market between them.
30. In Marketing Mix 4P’s imply (c) a monopolist charging excess prices for its product.
(a) Product, Price, Place, Promotion (d) exploiting a patent that the firm has on a product it has
(b) Product, Price, Policy, Place developed itself.
(c) Product, Place, Promotion, Policy (e) None of these

18. a b c d e 19. a b c d e 20. a b c d e 21. a b c d e 22. a b c d e

RESPONSE 23. a b c d e 24. a b c d e 25. a b c d e 26. a b c d e 27. a b c d e


28. a b c d e 29. a b c d e 30. a b c d e 31. a b c d e 32. a b c d e
GRID 33. a b c d e 34. a b c d e 35. a b c d e 36. a b c d e 37. a b c d e
38. a b c d e 39. a b c d e 40. a b c d e
y
o
u
rs
m
a
h
b
81

o
o
b
History of Banking

.w
o
rd
and its Development

p
re
s
s
.c
o
m
Max. Marks : 30 No. of Qs. 30 Time : 20 min. Date : ........./......../................
1. Reserve Bank of India was established on 9. Lead Bank system was started on the recommendations of
(a) April 12, 1932 (b) April 1, 1935 (a) Raja Challaia Committee
(c) May 2, 1943 (d) November 13, 1941 (b) Kelkar Committee
(e) None of these (c) Nariman Committee
2. Reserve Bank of India was nationalised on (d) Malhotra Committee
(a) 21 May, 1948 (b) 13 July, 1951 (e) None of these
(c) 1 January, 1949 (d) 12 October, 1951 10. Lead Bank system was started in
(e) None of these (a) 1967 (b) 1968
(c) 1969 (d) 1970
3. Six Commercial banks were nationalised by the Government
(e) 1972
on
11. Reserve Bank of India (RBI) follows the Minimum Reserves
(a) April 15, 1980 (b) May 10, 1981
System in issuing currency since
(c) July 21, 1982 (d) May 13, 1984 (a) 1950 (b) 1952
(e) None of these (c) 1954 (d) 1956
4. The 14 major banks were nationalised by the Government (e) 1951
on 12. Reserve Bank of India started the system of ombudsman to
(a) June 19, 1967 (b) July 19, 1969 resolve grievances of customers in
(c) May 21, 1962 (d) May 12, 1963 (a) 1994 (b) 1995
(e) None of these (c) 1996 (d) 1997
5. Which of the following is the first commercial bank? (e) 1998
(a) State Bank of India 13. The credit control methods adopted by the Reserve Bank are
(b) Oudh Commercial Bank a (a) Quantitative controls
(c) Union Bank of India (b) Qualitative controls
(d) Indian Bank (c) Fixed controls
(e) None of these (d) Both (a) and (b)
6. State Bank of India was established on (e) None of these
(a) May 2, 1951 (b) June 21, 1952 14. Which of the following Committees were appointed by the
(c) July 1, 1955 (d) August 12, 1956 Government for r estr uctur ing th e Regional Rural
(e) None of these Development Banks?
(a) Bhandar Committee (b) K.Basu Committee
7. NABARD (National Bank for Agriculture and Rural
(c) Raj Committee (d) Both (a) and ((b)
Development) was established in
(e) None of these
(a) 1979 (b) 1980
15. The concept of 'Universal Banking' was implemented in India
(c) 1981 (d) 1982 on the recommendations of:
(e) 1985 (a) Abid Hussain Committee
8. Regional Rural Banks were established on (b) R H Khan Committee
(a) July 3, 1970 (b) April 14, 1971 (c) S Padmanabhan Committee
(c) October 2, 1975 (d) November 23, 1978 (d) Y H Malegam Committee
(e) None of these (e) None of these

1. a b c d e 2. a b c d e 3. a b c d e 4. a b c d e 5. a b c d e
RESPONSE 6. a b c d e 7. a b c d e 8. a b c d e 9. a b c d e 10. a b c d e
GRID 11. a b c d e 12. a b c d e 13. a b c d e 14. a b c d e 15. a b c d e
y
o
u
rs
m
a
h
172 SPEED TEST 81

b
o
o
16. Oudh Commercial Bank was established in Which of the following are correct?

b
.w
(a) 1878 (b) 1879 (a) 1 and 3 (b) only 2

o
(c) 1880 (d) 1881 (c) 2 and 3 (d) only 1

rd
(e) None of these (e) All the above are correct.

p
re
17. Which of the following is the first Private bank established 24. Security printing press was established in 1982 at?

s
based on the recommendations of the Narasimhan

s
(a) Kolkata (b) New Delhi

.c
Committee? (c) Bombay (d) Hyderabad

o
m
(a) UTI Bank Ltd. (b) Union Bank (e) Nasik
(c) Bank of Baroda (d) Dena Bank 25. Six private sector banks were nationalised on April 15, 1980,
(e) None of these whose reserves were more than?
18. In 1993, a Nationalized bank was merged with Punjab National
(a) 100 Crores (b) 200 crores
Bank (PNB), what is the name of that Bank?
(c) 300 crores (d) 400 crores
(a) Bank of Baroda (b) Global Trust Bank
(e) 500 crores
(c) New Bank of India (d) Bank of India
26. Scheduled banks are those?
(e) None of these
(a) Includued in the 2nd schedule of the Banking
19. The biggest commercial bank in India is
Regulation Act-1949
(a) RBI (b) SBI
(c) IDBI (d) Exim Bank of India (b) Includued in the 2nd schedule of the Companies Act-
(e) None of these 1956
20. ICICI is a (c) Includued in the 2nd schedule of the Reserve Bank of
(a) Financial Institution India Act -1934
(b) Rural Development Bank (d) Bank Nationalization Act -1969
(c) Cooperative Bank (e) None of these
(d) Space Research Institute 27. When RBI has decided to circulate 'Plastic Currency Notes'
(e) None of these in the market ?
21. Banking amendment bill reduced SBI holding in its seven (a) July 1, 1999 (b) July 1, 2010
subsidiary banks from (c) July 1, 2011 (d) Sept. 1, 2011
(a) 90% to 75% (e) None of these
(b) 85% to 50% 28. When was addopted, New strategy for Rural Lending :
(c) 75% or more to 51% Service Area Approach ?
(d) 65% or more to 49% (a) April 1, 1989 (b) March 1, 2007
(e) None of these (c) April 1, 2010 (d) April 1, 2011
22. When was paper currency first started in India? (e) None of these
(a) 1810 (b) 1715 29. India Brand Equity Fund' was established in the year:
(c) 1635 (d) 1542 (a) 1992 (b) 1998 (c) 1995
(e) 1902 (d) 1996 (e) 1997
23. Consider the following statements. 30. Which was the first Indian Bank to introduce credit card?
(1) Scheduled Banks are those Banks which are included
(a) State Bank of India
in the Second Scheduled of the Reserve Bank Act, 1934.
(b) Central Bank of India
(2) There are 10 Non-Scheduled commercial Banks
(c) Union Bank of India
operating in the country.
(d) ICICI
(3) Co-operative banks are organized &managed on the
(e) None of these
principle of co-operation, self-help and mutual help.

RESPONSE 16. a b c d e 17. a b c d e 18. a b c d e 19. a b c d e 20. a b c d e


21. a b c d e 22. a b c d e 23. a b c d e 24. a b c d e 25. a b c d e
GRID 26. a b c d e 27. a b c d e 28. a b c d e 29. a b c d e 30. a b c d e
y
o
u
rs
m
a
h
b
82

o
o
RBI & its

b
.w
o
rd
Monetary Policies

p
re
s
s
.c
o
m
Max. Marks : 25 No. of Qs. 25 Time : 20 min. Date : ........./......../................
1. The accounting year of RBI occurs between the months of ? (a) ` 85 crore (b) ` 115 crore
(a) April – March (b) March – February (c) ` 200 crore (d) None of above
(c) July – June (d) August – July (e) None of these
(e) None of these 8. In India which agency is entrusted with the collection of
2. In which article RBI permitted to the co-operative Banks for data of capital formation ?
special Account Supervision?
(a) RBI and CSO (b) RBI and SBI
(a) Art – 30 (b) Art – 31 (c) Art – 33
(d) Art – 32 (e) None of these (c) RBI and Other Bank (d) CSO and NSSO

3. Open market operations of RBI refers to? (e) None of these


9. The Bank rate is the rate at which ?
(a) buying and selling of shares
(a) a bank lends to the public
(b) auctioning of foreign exchange
(b) the RBI lends to the public
(c) trading in securities
(c) RBI gives credit to the Commercial Banks
(d) transactions in gold.
(d) the Government of India lends to other countries.
(e) None of these
(e) None of these
4. Monetary policy in India is formulated and implemented by? 10. An increase in CRR by the Reserve Bank of India result in ?
(a) Government of India (a) decrease in debt of the government
(b) Reserve Bank of India (b) reduction in liquidity in the economy
(c) Indian Banks Association (c) attracting more FDI in the country
(d) FICCI (d) more flow of credit to desired sectors
(e) None of these (e) None of these
5. Reserve bank of India follows which system for the issue of 11. Which of the following provides the largest credit to agri-
currency? culture and allied sectors ?
(a) Minimum Reserve System (a) Co-operative Banks with RBI
(b) Proportionate Reserve System (b) Regional Rural Banks
(c) Both of the above (c) Commercial Banks
(d) None of the above (d) Co-operative and Regional Rural Banks
(e) None of these (e) None of these
6. Which of the following controls credit creation by the com- 12. Who published the financial report on currency and finance ?
mercial Banks in India? (a) RBI (b) CSO (c) WTO
(a) Ministry of Finance (b) Reserve Bank of India (d) NSSO (e) None of these
(c) Government of India (d) State Bank of India 13. Who is the custodian of India's foreign exchange Funds?
(a) RBI (b) SBI
(e) None of these
7. Note issuing department of Reserve Bank of India should (c) ICICI (d) Central Bank
always passes the minimum gold stock worth ? (e) None of these

1. a b c d e 2. a b c d e 3. a b c d e 4. a b c d e 5. a b c d e
RESPONSE 6. a b c d e 7. a b c d e 8. a b c d e 9. a b c d e 10. a b c d e
GRID 11. a b c d e 12. a b c d e 13. a b c d e
y
o
u
rs
m
a
h
174 SPEED TEST 82

b
o
o
14. Which of the following authority sanctions foreign exchange (c) Banks holding debt ridden Country Electricity Boards

b
.w
for the import of goods ? bonds

o
(a) Any Nationalised Bank (d) Both a and b

rd
(e) Neither (a) and (b)

p
(b) Exchange Bank

re
20. Which of the following are reasons for adverse impact on

s
(c) Reserve Bank of India card use in India?

s
.c
(d) Ministry of Finance (1) Less ATMs

o
m
(e) None of these (2) Less number of cards
(3) Less Point of Sales terminals
15. How many posts of Deputy Governor in Reserve Bank of
(a) 1, 2 (b) 1, 3
India ?
(c) 2 (d) All of the above
(a) 1 (b) 2 (c) 3 (e) None of these
(d) 4 (e) None of these 21. RBI will purchase back indexed bonds maturing in which
16. Which among the following institutions regulates the exter- year?
nal commercial borrowings ? (a) 2022 (b) 2023
(c) 2024 (d) 2025
(a) SEBI
(e) None of these
(b) Ministry of Finance 22. Which of the following statements is/are correct about
(c) Ministry of Commerce Deputy Governor of RBI?
(d) Reserve Bank of India 1. The tenure of Deputy Governor is five years or till the
age of 62, whichever is earlier.
(e) None of these
2. RBI has provision for three deputy governors.
17. Which of the following statements is not correct ?
3. RBI has provision for four deputy governors.
(a) RBI is the Central Bank of the country (a) Only 1 (b) Only 2
(b) RBI is the Banker of the Central and the state Govern- (c) Only 3 (d) Both 1 and 2
ments (e) None of these
23. Urijit Patel, Deputy Governor of RBI has been reappointed
(c) RBI is the custodian of the country’s Foreign Exchange
for 3 years. How many deputy governors of RBI are there?
Reserve
(a) 3 (b) 4
(d) RBI was established in 1949. (c) 5 (d) 6
(e) None of these (e) None of these
18. RBI has tweaked the rules for lending rates and stipulated 24. Since when has the Reserve Bank of India been successfully
that fixed rate loans of up to how many years will be offered operating the instrument of selective credit control in this
country?
by lenders based on marginal cost of funding?
(a) 1939 (b) 1951
(a) 2 (b) 3
(c) 1956 (d) 1961
(c) 4 (d) 5 (e) None of these
(e) None of these 25. India is a member of the International Monetary Fund since
19. RBI has relaxed norms for which category of banks? _____.
(a) Banks part of the cleaning up exercises of power utilities (a) 1934 (b) 1935
(b) Banks holding debt ridden State Electricity Boards (c) 1947 (d) 1949
bonds (e) None of these

RESPONSE 14. a b c d e 15. a b c d e 16. a b c d e 17. a b c d e 18. a b c d e


19. a b c d e 20. a b c d e 21. a b c d e 22. a b c d e 23. a b c d e
GRID 24. a b c d e 25. a b c d e
y
o
u
rs
m
a
h
b
83

o
o
Banking Product

b
.w
o
rd
and Services

p
re
s
s
.c
o
m
Max. Marks : 21 No. of Qs. 21 Time : 15 min. Date : ........./......../................

1. Which of the following is not the part of the Scheduled 6. Funded Services under corporate banking does not include?
Banking structure in India? (a) Working Capital Finance
(a) Money Lenders (b) Bill Discounting
(b) Public Sector Banks (c) Export Credit
(c) Private Sector Banks (d) Letters of Credit
(d) Regional Rural Banks (e) None of these
(e) State Cooperative Banks
7. ''Swabhiman'' Scheme is related-
2. Small Savings Scheme like national savings certificates,
(a) Rich Customers of the Bank
Public Provident Fund, Monthly Income Schemes are popular
(b) RRBs
among the salaried people. Which financial institutions
(c) To provide basic banking services to bankless villages
manage these schemes?
(a) Public Sector Banks (b) Commercial Banks (d) Both (b) and (c)
(c) Post Offices (d) Co-operative Banks (e) None of these
(e) None of these 8. A centralized database with online connectivity to branches,
3. Which of the following is/are the right(s) of customer internet as well as ATM-network which has been adopted
towards his banker? by almost all major banks of our country is known as?
(a) To receive a statement of his account from a banker (a) Investment Banking
(b) To sue the bank for any loss and damages (b) Core Banking
(c) To sue the banker for not maintaining the secrecy of (c) Mobile Banking
his account (d) National Banking
(d) All of the above (e) Specialized Banking
(e) None of these 9. The Reverse Mortgage scheme is launched to give benefit
4. Which of the following factors is not required to be to which of the following groups of society?
considered to analyze the repayment capacity of a borrower? (a) Persons below 60 yrs
(a) Working capital management (b) Senior Citizens
(b) Personal educational qualifications (c) Unemployed youth
(c) Financial leverage (d) Orphans
(d) Interest rate risk management (e) All of the above
(e) None of these
10. Which of the following scheme is not meant for investment
5. Which of the following is a facilitating service of core loan
purposes?
products of retail banking services?
(a) National saving certificate
(a) Current or savings accounts
(b) Infrastructure bonds
(b) Legal services for documentation
(c) Mutual funds
(c) Delivery of loan at promised time period
(d) Letter of credit
(d) Flexibility in prepayment of loan
(e) None of these
(e) None of these

RESPONSE 1. a b c d e 2. a b c d e 3. a b c d e 4. a b c d e 5. a b c d e

GRID 6. a b c d e 7. a b c d e 8. a b c d e 9. a b c d e 10. a b c d e
y
o
u
rs
m
a
h
176 SPEED TEST 83

b
o
o
11. Systematic investment Plans relates to: 16. If a bank ties up with a retail vendor and then both of them

b
.w
(a) Mutual Funds sponsor a credit card, then such card would be known as __:

o
(b) Life Insurance Companies (a) Retail Credit Card (b) Vendor Card

rd
(c) Commercial Banks (c) Co-branded Card (d) Cash back Card

p
re
(d) Post office savings schemes (e) None of these

s
s
(e) None of these 17. In terms of Banking Terminology, Affinity Card refers to __:

.c
o
12. Which of the following is an example of cash less purchase? (a) The Credit Cards that are linked to special organizations

m
(a) Debit card (b) Credit card (b) The Credit Cards with zero interest rate on repayments
(c) ATM withdrawal (d) All of the above (c) The Credit Cards exclusively for the Bank's employees
(e) None of these (d) All of above
13. Which one of the following is not an electronic banking (e) None of these
delivery channel? 18. From which country, the concept of Credit Card originated?
(a) Mobile Vans (a) United Kingdom (b) United States
(d) Mobile Phone Banking (c) France (d) Australia
(c) Internet Banking (e) None of these
(d) Tele Banking 19. Which is the first credit card facility to be recognised
(e) ATM worldwide?
14. Now-a-days Banks are selling third party products. Example (a) Visa card (b) Maestro card
of third party product is: (c) MasterCard (d) Diner Card
(a) Mutual funds (e) None of these
(b) Term deposits 20. In terms of Credit Cards, what is a Hot Card?
(c) Credit cards (a) Newly issued Card (b) Invalid Card
(d) All of these (c) Stolen Card (d) Unpaid Card Hide
(e) None of these (e) None of these
15. What are White Label ATMs 21. What does the cirrus logo on ATM / debit cards signifies__?
(a) ATMs set up and run by non-banking entities (a) have cash access facility anywhere in India only
(b) ATMs set up and run by banking entities (b) have cash access facility outside the India only
(c) ATMs in rural areas (c) have cash access facility in or outside the India
(d) ATMs in Defence areas (d) have cash access facility in and outside the India
(e) ATM set up in Uttaranchal (e) None of these

RESPONSE 11. a b c d e 12. a b c d e 13. a b c d e 14. a b c d e 15. a b c d e


16. a b c d e 17. a b c d e 18. a b c d e 19. a b c d e 20. a b c d e
GRID 21. a b c d e
y
o
u
rs
m
a
h
84

b
o
o
Banking Term/

b
.w
o
rd
Terminology

p
re
s
s
.c
o
m
Max. Marks : 30 No. of Qs. 30 Time : 20 min. Date : ........./......../................
1. What is the full form of ‘FINO’, a term we see frequently in (a) Company Social Representation
financial newspapers? (b) Company Service Responsibility
(a) Financial Investment Network and Operations (c) Corporate Social Responsibility
(b) Farmers’ Investment in National Organisation (d) Corporate Security Responsibility
(c) Farmers Inclusion News and Operations (e) None of these
(d) Financial Inclusion Network and Operations 8. In a bid to standardise and enhance the security features in
(e) None of these cheque forms, it has now been made mandatory for banks
2. What does the letter 'L' denote in the term ‘LAF’ as referred to issue new format of cheques called
every now and then in relation to monetary policy of the (a) CTS-2010 (b) CTS-2011
RBI? (c) CTS-2012 (d) CTS-2013
(a) Liquidity (b) Liability (e) None of these
(c) Leveraged (d) Longitudinal 9. The “Four Eyes” principle (mentioned by the Reserve Bank
(e) Linear of India) refers to:
3. Which of the following terms is NOT a financial term? (a) Lenders (b) Borrowers
(a) Investment (c) Wealth Managers (d) Micro-Fananciers
(b) El Nino effect (e) None of these
(c) Core banking Solution 10. NRE deposit is
(d) RTGS (a) Non Resident External deposit
(e) All are financial terms (b) Non Resident Extra deposit
4. The term 'Smart Money" refers to __________ . (c) Non Resident Exchange deposit
(a) Foreign Currency (b) Internet Banking (d) Non Refundable External deposit
(c) US Dollars (d) Travelers' cheques (e) Non Resident Extended deposit
(e) Credit Cards 11. Which of the following is NOT a banking-related term?
5. We often come across the term SWIFT in financial (a) SME Finance (b) Overdraft
newspapers. What is the expanded form of this term? (c) Drawing power (d) Sanctioning Authority
(a) Society for Worldwide Interbank Financial (e) Equinox
Telecommunication 12. What does the acronym LAF stand for?
(b) Secure Worldwide Interbank Financial Telecommunica-
(a) Liquidity Adjustment Fund
tion.
(b) Liquidity Adjustment Facility
(c) Society for Worldwide Intr a-bank Finan cial
(c) Liquidity Adjustment Finance
Transaction.
(d) Liquidity Adjustment Factor
(d) Security for Worldwide Interbank Financial
(e) None of these
Transaction
13. Which of the following terms is used in Banking Field?
(e) None of these
6. Many a time we read in financial newspapers about the (a) Interest rate swap (b) Input devices
performance of the “core sectors” in the economy. Which (c) Sedimentary (d) Zero hour
of the following is NOT included in the same? (e) Privilege motion
(a) Coal (b) Automobiles 14. What is “wholesale banking”?
(c) Steel (d) Cement (a) It is a bank-to-bank or B2B dealing.
(e) Oil & Petroleum (b) It is a bank-to-customer dealing.
7. The Standing Committee on Finance headed by Yashwant (c) It is a bank-to-trustworthy customer dealing.
Sinha has recommended that CSR should be mandatory for (d) It is a bank-to-government dealing
all the companies. CSR stands for (e) None of these

1. a b c d e 2. a b c d e 3. a b c d e 4. a b c d e 5. a b c d e
RESPONSE 6. a b c d e 7. a b c d e 8. a b c d e 9. a b c d e 10. a b c d e
GRID 11. a b c d e 12. a b c d e 13. a b c d e 14. a b c d e
y
o
u
rs
m
a
h
178 SPEED TEST 84

b
o
o
15. Trade between India and China is in a state of “Payment (d) Industrial Research and Demands Agency

b
imbalance”. What does this mean in real terms? (e) None of these

.w
(1) China imports less from India but India imports more 22. Hard Currency is defined as currency :

o
from China. (a) which can hardly be used for international transactions

rd
(2) China delays payments to exporters. (b) which is used in times of war

p
re
(3) India wants payments in US Dollars but wants to pay (c) which loses its value very fast

s
in Yuan. (d) traded in foreign exchange market for which demand

s
.c
(a) Only 1 (b) Only 2 is persistently relative to the supply

o
(c) Only 3 (d) All 1, 2 and 3 (e) None of these

m
(e) None of these 23. The terms “bull” and “bear” are used in the :
16. Banks are promoting “Branch less Banking” which means? (a) Bihar Government’s Animal Husbandry Department
(1) Banks will not reduce number of branches. Number of (b) Income Tax Department
branches will be restricted and will concentrate on (c) CBI
specified core business. (d) Stock Exchange
(2) Banks will launch/operate multiple delivery channels (e) None of these
like ATMs, Mobile Banking/Internet Banking etc making 24. We read a term 'ECB' in the financial newspapers. What is
visit to a branch unnecessary. the full form of ECB?
(3) Banks will issue only debit or credit cards for daily (a) Essential Credit and Borrowing
financial transactions. Cheques/Cash payment will not (b) Essential Commercial Borrowing
be allowed. (c) External Credit and Business
(a) Only 1 (b) Only 2 (d) External Commercial Borrowing
(c) Only 1 and 2 (d) Only 2 and 3 (e) None of these
(e) All 1, 2 and 3 25. Green Banking means
17. NBFCs are an important part of the Indian financial system. (a) financing of irrigation projects by banks
What is the full form of this term? (b) development of forestry by banks
(a) New Banking Financial Companies (c) financing of environment friendly projects by banks
(b) Non-Banking Financial Companies (d) development of Railway by banks.
(c) Neo Banking Financial Confederation
(e) None of these
(d) Non-Banking Fiscal Companies
26. DTAA stands for
(e) All of these
(a) Direct Tariff Avoidance Agreement
18. BCSBI stands for
(b) Double Taxation Avoidance Agreement
(a) Banking Codes and Standards Board of India
(c) Direct Taxation Avoidance Agreement
(b) Banking Credit and Standards Board of India
(c) Banking Codes and Service Board of Inida. (d) Double Tariff Avoidance Agreement
(d) Banking Credit and Service Board of India. (e) None of these
(e) None of these 27. The rate at which the Reserve Bank of India lends to the
19. What does the term ‘bancassurance’ mean ? commercial banks in very short term against the backing of
(a) Assurance from the bank to its account holder the Government securities is known as?
regarding safety of his money (a) Bank rate (b) Repo rate
(b) A special product designed by the bank (c) Reverse Repo (d) Discount rate
(c) Selling of insurance policies by banks (e) None of these
(d) Understanding between banks and insurance 28. A letter of credit (L(C) wherein the credit available to the
companies customer gets reinstated after the bill is paid is known as?
(e) None of these (a) Back to back LC (b) Red clause LC
20. SEZ stands for: (c) Back to front LC (d) Revolving LC
(a) Southern Economic Zone (e) None of these
(b) South European zone 29. Which of the following is not an imperfect note?
(c) Special Economic Zone (a) Washed note (b) Bleached note
(d) Special Eastern Zone (c) Mutilated note (d) Oiled note
(e) None of these (e) None of these
21. The abbreviation IRDA stands for: 30. Which of the following is not shown as an asset in the
(a) Industrial Research and Development Authority of balance sheet of a BanK?
India (a) Investment (b) Advances
(b) Insurance Research and Development Authority of (c) Cash Balances with other banks
India (d) Borrowings
(c) Insurance Regulation Development Authority of India (e) None of these

15. a b c d e 16. a b c d e 17. a b c d e 18. a b c d e 19. a b c d e


RESPONSE 20. a b c d e 21. a b c d e 22. a b c d e 23. a b c d e 24. a b c d e
GRID 25. a b c d e 26. a b c d e 27. a b c d e 28. a b c d e 29. a b c d e
30. a b c d e
y
o
u
rs
m
a
h
b
85

o
o
Micro Finance and

b
.w
o
rd
Economics

p
re
s
s
.c
o
m
Max. Marks : 30 No. of Qs. 30 Time : 20 min. Date : ........./......../................
1. Which sector of Indian economy contributes longest to Gross (c) Finance Ministry
National Product ? (d) Reserve Bank of India
(a) Primary sector (b) Secondary sector (e) None of these
(c) Tertiary sector (d) Public sector 9. In which of the following financial years the devaluation of
(e) None of these rupee in India took place twice ?
2. The main source of National Income in India is- (a) 1966-67 (b) 1991-92
(a) Service sector (b) Agriculture (c) 1990-91 (d) 1989-90
(c) Industrial sector (d) Trade sector (e) None of these
(e) None of these 10. VAT is imposed-
3. Which one of the following is not a tax levied by the (a) On first stage of production
government of India ? (b) Directly on consumer
(a) Service tax (b) Education (c) On all stages between production and final sale
(c) Custom duty (d) Toll tax (d) On final stage of production
(e) None of these (e) None of these
4. The most appropriate measure of a country’s economic 11. Term Balance of payment is used in relation to which of the
growth is its- following ?
(a) Gross Domestic Product (GDP) (a) Annual sale of a factory
(b) Net Domestic Product (NDP) (b) Tax collection
(c) Net National Product (NNP) (c) Exports and imports
(d) Per Capita Product (PCP) (d) None of the above
(e) None of these (e) None of these
5. FEMA (Foreign Exchange Management (Act) was finally 12. The Indian economy can be described as
implemented in the year (a) a backward and stagnant economy
(a) 1991 (b) 1997 (b) a developing economy
(c) 2002 (d) 2007 (c) an underdeveloped economy
(e) None of these (d) a developed economy
6. The most common measure of estimating inflation in India is- (e) None of these
(a) Price Index 13. Loans to poor people by banks have many limitations
(b) Wholesale Price Index including lack of security and high operating cost. So to
(c) Consumer Price Index help them which type of finance system developed ?
(d) Price Index of Industrial Goods (a) Ponzi schemes
(e) None of these (b) Micro Finance System
7. The National Income of India is estimated by- (c) Money Laundering Schemes
(a) National Sample Survey Organization (d) Money tampering finance
(b) Ministry of Finance (e) None of these
(c) Reserve Bank of India 14. The beneficiaries of Micro finance business are _______.
(d) Central Statistical Organization (a) Land Less labour
(e) None of these (b) Marginal farmers
8. In India, which one among the following formulates the fiscal (c) Vendors in the small markets
policy?
(d) All the above
(a) Planning Commission
(e) None of these
(b) Finance Commission

1. a b c d e 2. a b c d e 3. a b c d e 4. a b c d e 5. a b c d e
RESPONSE 6. a b c d e 7. a b c d e 8. a b c d e 9. a b c d e 10. a b c d e
GRID 11. a b c d e 12. a b c d e 13. a b c d e 14. a b c d e
y
o
u
rs
m
a
h
180 SPEED TEST 85

b
o
15. One of the delivery channel for Micro Finance is SHG model.

o
24. Which among the following is considered to be the most

b
SHG means ? liquid asset?

.w
(a) Soar Help Group (b) Sake Help Group (a) Gold (b) Money

o
rd
(c) Self Hope Group (d) Self Help Group (c) Land (d) Treasury bonds

p
(e) None of these (e) None of these

re
16. In the Not-For-Profit Micro Finance Institutes, which among 25. Who is author of the ancient book on economics,

s
s
the following are included ? Arthashastra?

.c
(a) Societies

o
(a) Kautilya (b) Chanakya

m
(b) Public Trusts (c) Sushrut (d) Bhattacharya
(c) Non-Profit Companies (e) None of these
(d) All of these
26. Currency notes and coins are called as:
(e) None of these
(a) Flat money (b) Legal tenders
17. Non-banking financial companies, producer companies and
LAB come under the category of For-Profit-MFIs. LAB (c) Fiat money (d) Both b and c
means ? (e) None of these
(a) Loan Area Banks (b) Legal Area Banks 27. What is the currency deposit ratio (cdr)?
(c) Local Axis Banks (d) Local Area Banks (a) ratio of money held by the public in currency to that of
(e) None of these money held in bank deposits
18. On which among the following dates FERA was replaced by (b) ratio of money held by public in bank deposits to that
FEMA? of money held by public in currency
(a) June 1, 2000 (b) June 11, 2000 (c) ratio of money held in demand drafts to that of money
held in treasury bonds
(c) April 1, 2001 (d) April 1, 2002 (d) ratio of money held in demand drafts to that of money
(e) None of these held in mutual bonds
19. What is the minimum net owned funds (NOF) mandatory (e) None of these
for a Infrastructure Finance Company (IFC) in India?
28. What is the reserve deposit ratio (rdr)?
(a) ` 200 Crore (b) ` 300 Crore
(a) the proportion of money RBI lends to commercial banks
(c) ` 500 Crore (d) ` 100 Crore
(b) the proportion of total deposits commercial banks keep
(e) None of these
as reserves
20. In India, Infrastructure Debt Fund can be established as a
Trust or a company. Which of the following regulated the (c) the total proportion of money that commercial banks
Infrastructure Debt Fund set up as a trust? lend to the customers
(a) SEBI (b) RBI (d) the total proportion of money that commercial banks
lend to the money RBI tends
(c) IrDA (d) Ministry of Corporate Affairs
(e) None of these (e) None of these
21. What is the cap on loan amount given out by a Non-Banking 29. What is the Cash Reserve Ratio (CRR)?
Financial Company - Micro Finance Institution (NBFC-MFI)? (a) the fraction of the deposits that commercial banks lend
(a) ` 50000 (b) ` 60000 to the customers
(c) ` 70000 (d) ` 75000 (b) the fraction of the deposits that RBI must keep with
(e) None of these commercial banks
22. Which of the following is are the parties in Factoring (c) the fraction of the deposits that commercial banks must
Business? keep with RBI
(a) buyer (d) the fraction of the deposits that private banks must
(b) seller keep with RBI
(c) buyer and seller (e) None of these
(d) buyer, seller and financial institution 30. In monetary terminology, what is called the 'monetary base'
or 'high powered money'?
(e) None of these
23. What is the minimum tenure of deposits to be taken by (a) the total assets of RBI
NBFCs? (b) the total liability of RBI
(A) 6 months (b) 12 months (c) the total debt of the government
(c) 2 years (d) 3 years (d) the total foreign exchange of RBI
(e) None of these (e) None of these

15. a b c d e 16. a b c d e 17. a b c d e 18. a b c d e 19. a b c d e


RESPONSE 20. a b c d e 21. a b c d e 22. a b c d e 23. a b c d e 24. a b c d e
GRID 25. a b c d e 26. a b c d e 27. a b c d e
28. a b c d e 29. a b c d e 30. a b c d e
y
o
u
rs
m
a
h
b
86

o
o
b
.w
Foreign Trade

o
rd
p
re
s
s
.c
o
m
Max. Marks : 30 No. of Qs. 30 Time : 20 min. Date : ........./......../................
1. ‘Eco mark’ is given to the Indian products that are ? 9. TRIPS and TRIMS are the term associated with ?
(a) pure and unadulterated (a) IMF (b) WTO
(b) rich in proteins
(c) IBRD (d) IDA
(c) Environment Friendly
(d) Economically viable (e) None of these
(e) None of these 10. SEZ act was passed by the parliament in the year ?
2. The earlier name of WTO was ? (a) 2004 (b) 2005
(a) UNCTAD (b) GATT (c) 2006 (d) 2007
(c) UNIDO (d) OECD
(e) None of these
(e) None of these
3. ‘World Development Report’ is an annual publication of ? 11. How many members recently in WTO ?
(a) UNDP (b) IBRD (a) 158 (b) 159
(c) WTO (d) IMF (c) 160 (d) 162
(e) None of these (e) None of these
4. India has the maximum volume of foreign trade with ?
12. Which of the following is a part of capital account?
(a) USA (b) Japan
(c) Germany (d) UAE (a) Private capital (b) Banking capital
(e) None of these (c) Official capital (d) All the above
5. Participatory notes (PNs) are associated with which one of (e) None of these
the followings ? 13. The investment in productive assets and participation in
(a) Consolidated food
management as stake holders in business enterprises is
(b) Foreign Institutional Investors
(c) UNDP (a) FDI
(d) Kyoto protocal (b) FII
(e) None of these (c) Balance of payment
6. What is the purpose of India Brand-Equity Fund ? (d) SDR
(a) To promote in bound tourism
(e) None of these
(b) To make ‘Made in India’ a label of quality
(c) To organise trade fairs 14. The portfolio investment by foreign institutional investors
(d) To provide venture capitals to IT sector is called
(e) None of these (a) FDI
7. A trade policy consists of : (b) FII
(a) Export-Import policy
(c) Balance of payment
(b) Licencing policy
(c) Dumping (d) SDR
(d) Double pricing (e) None of these
(e) None of these 15. Which of the following is international trade:
8. FERA in India has been replaced by ? (a) Trade between provinces
(a) FEPA (b) FEMA (b) Trade between regions
(c) FENA (d) FETA (c) Trade between countries
(d) (b) and (c) of above
(e) None of these
(e) None of these

1. a b c d e 2. a b c d e 3. a b c d e 4. a b c d e 5. a b c d e
RESPONSE 6. a b c d e 7. a b c d e 8. a b c d e 9. a b c d e 10. a b c d e
GRID 11. a b c d e 12. a b c d e 13. a b c d e 14. a b c d e 15. a b c d e
y
o
u
rs
m
a
h
182 SPEED TEST 86

b
o
o
16. Which is NOT an advantage of international trade: 23. Special Economic Zones are :

b
.w
(a) Export of surplus production (a) situated outside India, but subject to RBI control

o
(b) Import of defence material (b) treated as foreign territory and not Indian laws

rd
(c) governed by international and not Indian laws

p
(c) Dependence on foreign countries

re
(d) Availability of cheap raw materials (d) prohibited from buying from Domestic Tariff Area

s
s
(e) None of these

.c
(e) None of these

o
24. The Imports and Exports (Control) Act came into enforcement

m
17. Trade between two countries can be useful if cost ratios of
from
goods are:
(a) 1947 (b) 1950.
(a) Equal (b) Different (c) 1951 (d) 1955
(c) Undetermined (d) Decreasing (e) None of these
(e) None of these 25. The IEC number is issued by the
18. Foreign trade creates among countries: (a) Central Government
(a) Conflicts (b) Cooperation (b) State Government
(c) Hatred (d) Both (a) & (b) (c) Director General of Foreign trade
(e) None of these (d) Ministry of commerce
19. All are advantages of foreign trade EXCEPT: (e) None of these
(a) People get foreign exchange 26. The IEC number is a number with
(a) 7 digits (b) 8 digits
(b) Nations compete
(c) 9 digits (d) 10 digits
(c) Cheaper goods
(e) None of these
(d) Optimum utilisation of country's resources
27. The Director General of Foreign trade is appointed by
(e) None of these (a) Central Government
20. Govt. policy about exports and imports is called: (b) State Government
(a) Monetary policy (c) Ministry of commerce
(b) Fiscal policy (d) Chief justice of the Supreme Court
(c) Commercial policy (e) None of these
(d) Finance policy 28. The foreign Trade (Regulation) Rules was passed in the
(e) None of these year
21. What would encourage trade between two countries: (a) 1991 (b) 1992
(c) 1993 (d) 1994
(a) Different tax system
(e) None of these
(b) Frontier checks
29. The apex body of the Foreign Trade is
(c) National currencies
(a) The Central Government
(d) Reduced tariffs
(b) The State Government
(e) None of these (c) The Ministry of Commerce
22. Foreign trade: (d) All the above
(a) Increases employment opportunities (e) None of these
(b) Increases international mobility of labour 30. The tenure of the Foreign Trade policy is
(c) Increases competition (a) 3 years (b) 5 years
(d) All of the above (c) 1 year (d) 7 years
(e) None of these (e) None of these

16. a b c d e 17. a b c d e 18. a b c d e 19. a b c d e 20. a b c d e


RESPONSE 21. a b c d e 22. a b c d e 23. a b c d e 24. a b c d e 25. a b c d e
GRID 26. a b c d e 27. a b c d e 28. a b c d e 29. a b c d e 30. a b c d e
y
o
u
rs
m
a
h
b
87

o
o
Socio-Eco-Political

b
.w
o
rd
Environment of India

p
re
s
s
.c
o
m
Max. Marks : 25 No. of Qs. 25 Time : 20 min. Date : ........./......../................
1. Swabhiman scheme launched in India is associated with 8. Which one of the following is the objective of National Food
(a) Rural women rights Security Mission ?
(b) Rural old people care (a) To increase production of rice
(c) Rural banking (b) To increase production of wheat
(d) Rural food security (c) To increase production of pulses
(e) None of these (d) All the above
2. In which one of the following years the unorganised workers (e) None of these
social security Act was passed ? 9. Which one of the following durations is related to XII Five
(a) 2004 (b) 2006 Year Plan in India ?
(c) 2008 (d) 2010 (a) 2007-12 (b) 2005-10
(e) None of these (c) 2012-17 (d) 2010-15
3. Swadhar is the scheme for (e) None of these
(a) unique identification 10. Which of the following is not a measure of reducing
(b) self employment for males inequalities ?
(a) Minimum needs programme
(c) women in difficult circumstances
(b) Liberalization of economy
(d) common home of senior citizens. (c) Taxation
(e) None of these (d) Land reforms
4. The aim of Pradhan Mantri Gramodaya Yojana is (e) None of these
(a) Meeting rural needs like primary education, health care, 11. Which one of the following scheme subsumed the Valmiki
drinking water, housing, rural roads. Awas Yojana ?
(b) Alleviating poverty through microenterprises. (a) Integrated Housing and slum development Programme.
(c) Generating employment in rural areas. (b) Sampoorna Grameen Swarozgar Yojana
(d) Strengthening Panchayati Raj system in rural areas. (c) Rajiv Awas Yojana
(e) None of these (d) Integrated Rural development Programme
(e) None of these
5. Twenty Point Economic Programme was first launched in
the year 12. Poverty level in India is established on the basis of-
(a) Per capita income in different states
(a) 1969 (b) 1975
(b) House hold average income
(c) 1977 (d) 1980 (c) House hold consumer expenditure
(e) None of these (d) Slum population in the country.
6. In India disguised unemployment is a prominent feature (e) None of these
mainly of 13. Nirmal Bharat Abhiyan Yojana is associated with-
(a) Primary sector (b) Secondary sector (a) Development of villages
(c) Tertiary sector (d) Social sector (b) Community toilets in slum areas
(e) None of these (c) Construction of house far low income groups
7. Golden Quadrangle project is associated with the (d) Development of roads
development of (e) None of these
14. Crop Insurance Scheme in India was started in
(a) Highways (b) Ports
(a) 1945 (b) 1980
(c) Power Grids (d) Tourism Network (c) 1985 (d) 1988
(e) None of these (e) None of these

1. a b c d e 2. a b c d e 3. a b c d e 4. a b c d e 5. a b c d e
RESPONSE 6. a b c d e 7. a b c d e 8. a b c d e 9. a b c d e 10. a b c d e
GRID 11. a b c d e 12. a b c d e 13. a b c d e 14. a b c d e
y
o
u
rs
m
a
h
184 SPEED TEST 87

b
o
o
15. Which of the following scheme is not for rural development ? 21. India is implementing "Capacity Building for Industrial

b
Pollution Management (CBIPM)" project with the support

.w
(a) PMGSY (b) SGSY
of __?

o
(c) RGGVY (d) SJSRY

rd
(a) Asian Development Bank
(e) None of these

p
(b) Japan Central Bank

re
16. Who is the chairman of 21st law commission ? (c) World Bank

s
s
(a) Justice A.R. Lakshmanan

.c
(d) Reserve Bank of India

o
(b) Justice P.V. Reddy (e) None of these

m
(c) Justice D.K. Jain 22. Who is the chairman of the Panel set up to look into various
(d) Justice Balbir Singh Chauhan contentious issues relating to inter-linking of rivers?
(a) B N Navalawala (b) Hardip Singh Puri
(e) None of these
(c) Prodipto Ghosh (d) Gopalakrishnan
17. According to the Integrated child development services
(e) None of these
which age group of children are focused ?
23. Who has been appointed as new Chief Election
(a) upto 4 years (b) upto 5 years Commissioner of India?
(c) upto 6 years (d) upto 8 years (a) Nasim Zaidi (b) Sayed Nazim
(e) None of these (c) H Hari Shankar (d) V S Chikkamata
18. Which age group of women are eligible for Indira Gandhi (e) None of these
widow Pension Scheme ? 24. What is the name of the new scheme to empower and enable
(a) 40-50 (b) 50-60 minority youths, announcement for which was made in the
budget?
(c) 40-59 (d) 40-79
(a) New Udaan (b) Nayi Soch
(e) None of these
(c) Nayi Umang (d) Nayi Manzil
19. Under which project India is developing guided missile (e) None of these
destroyers? 25. Union Human Resource and Development (HRD) Minister
(a) Project 20B (b) Project 15B Smriti Irani introduced the logo winner of for the New
(c) Project 75B (d) Project 40A
Education Policy (NEP) who emerged as the winner in the
(e) None of these
competition was held through MyGov Platform. What is his
20. E-Samiksha is an online project monitoring system launched
by ____? name?
(a) Indian Railways (b) NHAI (a) Nawab Shaikh (b) Ali Ahmed
(c) RBI (d) FIPB (c) Nawaj Shaikh (d) Mansood Pataudi
(e) None of these (e) None of these

15. a b c d e 16. a b c d e 17. a b c d e 18. a b c d e 19. a b c d e


RESPONSE
20. a b c d e 21. a b c d e 22. a b c d e 23. a b c d e 24. a b c d e
GRID
25. a b c d e
y
o
u
rs
m
a
h
b
88

o
o
b
Appointment/

.w
o
rd
Election/Resignation

p
re
s
s
.c
o
m
Max. Marks : 30 No. of Qs. 30 Time : 20 min. Date : ........./......../................
1. Who among the following has been appointed as permanent 7. Who among th e following has been appointed as
representative of India to UNESCO? Chairperson of National School of Drama?
(a) Ruchir Kamboj (a) Girish Carnad (b) Ratan Thiyam
(b) V S Oberoi (c) U R Anantha Murthy(d) Anupam Khair
(c) Vijay Keshav Ghokhale (e) None of these
(d) Sudeer Kulkarni 8. Who is appointed as the chairman of The Association of
(e) None of these Mutual Funds in India (AMFI).
2. Who among the followings is the chairman of “National (a) Sundeep Sikka (b) Sandesh Kirkire
Commission for Backward Classes”? (c) Pawan Kumar (d) S. M. Patel
(a) Justice V Eswaraih (e) Leo Puri
(b) Justice M N Rao 9. Who took charge as the new commissioner of Delhi Police
(c) Justice Narendra Babu on February 2016?
(d) Justice Gopala Gowda (a) BK Gupta (b) Neeraj Kumar
(e) None of these (c) Bhim Sain Bassi (d) G.V. Prakash Kumar
3. Who among the followings is the Chairperson of the National (e) Alok Kumar Verma
Commission for Minorities? 10. Who is recently appointed as the Chief Justice of Meghalaya
(a) K N Daruwall High Court?
(b) Tsering Namgyal Shanoo (a) M. Subhashini
(c) Wajahat Habibullah (b) Kamal Ganzouri
(d) Ajaib Singh (c) Prafulla Chandra Pant
(e) Shri Naseem Ahmed (d) P. Sathasivam
4. Who among the following is the current Chairperson of (e) T.Nand kumar Singh
“National Green Tribunal” of India? 11. Who among the following is the CEO of PepsiCo Asia,
(a) Justice Jyothimani Middle East and Africa region?
(b) Justice Kingaonkar (a) Abhinav Gupta (b) Sanjay Singh
(c) Justice Swatanter Kumar (c) Sanjeev Chadha (d) Nanda Kishore
(d) Justice U D Salvi (e) Indira Nooyi
(e) None of these 12. Who is appointed as the Vice Chairman of NITI Aayog?
5. Who among the following is the Chairman of “National (a) Jagdish Bhagwati (b) T. N. Srinivasan
Ganga River Basin Authority”? (c) Arvind Panagariya (d) Montek Singh Ahluwalia
(a) President (b) Prime Minister (e) None of these
(c) Vice-president (d) Home Minister 13. Who replaced Sujatha Singh as India's Foreign Secretary
(e) None of these (a) Nirupama Rao (b) Ranjan Mathai
6. Who among the followings has been appointed as Chief (c) Ajit Doval (d) S Jaishankar
Information Commissioner? (e) None of these
(a) Mrs Deepak Sandhu 14. _____ has been appointed as an Election Commissioner of
(b) Mrs Sushma Singh India ?
(c) Rajiv Mathur (a) Anil Swarup (b) C.R. Viswanath
(d) Vijay Sharma (c) Achal Kumar Jyoti (d) Ashok Kumar Gupta
(e) None of these (e) None of These

1. a b c d e 2. a b c d e 3. a b c d e 4. a b c d e 5. a b c d e
RESPONSE 6. a b c d e 7. a b c d e 8. a b c d e 9. a b c d e 10. a b c d e
GRID 11. a c d e 12. a c d e 13. a c d e 14. a c d e
b b b b
y
o
u
rs
m
a
h
186 SPEED TEST 88

b
o
o
15. Who is the incumbent Managing Director of the 24. Who among the following has taken over charge as Deputy

b
International Monetary Fund (IMF)? Chairman of Kolkata Dock System?

.w
(a) Christine Lagarde (b) Dominique Strauss-Kahn (a) Sudheer K Juneja

o
rd
(c) Ban Ki-moon (d) Jim Yong Kim (b) Amit Kundan

p
(e) None of these (c) S Balaji Arunkumar

re
16. Who is the incumbent chairman of the National Bank for (d) K P Oli

s
s
Agriculture and Rural Development (NABARD)? (e) None of these

.c
(a) Atulesh Jindal

o
25. Who among the following has been appointed as Chief

m
(b) Satynarayan Mohanty Executive Officer of its agri-machinery business by Escort
(c) Harsh Kumar Bhanwala Limited?
(d) Upendra Kumar Sinha (a) Ravi A Menon
(e) None of these (b) Ajit Kumar Sinha
17. Who has been appointed as an Asia-Pacific member of the (c) O P Juneja
International Coordinating Committee Bureau of NHRI-ICC? (d) Ram Kumar Adhikari
(a) Arun Jaitley (b) HL Dattu (e) None of these
(c) R M Lodha (d) H L Narayanaswamy 26. Consider the following statements:
(e) None of these 1. Nita Ambani has been named the most powerful
18. Who has been appointed as the advisor to the Khadi and businesswoman in Asia.
Village Industries Commission (KVIC) for promotion of khadi 2. Arundhati Bhattacharya has been ranked third on the
within India and globally?
2016 'Asia's 50 Power Businesswomen' list
(a) Payal Singhal (b) Ritu Beri
3. The list has been released by Forbes.
(c) Tarun Tahiliani (d) Manish Arora
Which of the above statements are correct?
(e) None of these
(a) Only 1 & 2 (b) Only 1 & 3
19. Who is the newly appointed Governor of Central Bank of
(c) Only 2 & 3 (d) All are correct
Bangladesh?
(e) None of these
(a) Fazle Kabir (b) Abdul Hamid
(c) AMA Muhith (d) Shahana Rahman 27. Who among the following persons has appointed as the
(e) None of these CEO of Suzlon Group?
20. Which Indian scholar has been appointed as the Vice (a) Anita Ghosh
Chancellor of the University of Canberra? (b) J P Chalasani
(a) H Deep Saini (b) Sagar Mal (c) Ajit Chaudhary
(c) Mandeep Singh (d) Stephen Parker (d) K S Sharma
(e) None of these (e) None of these
21. Who has been appointed as the new chairman of Bombay 28. Who among the following has taken charge as the General
Stock exchange (BSE)? Manager (GM) of Pune unit of Bharat Electronics Ltd ?
(a) Sudhakar Rao (b) Sethurathnam Ravi (a) M D Yadav
(c) Prem Kumar (d) S Ramadorai (b) M K Sharma
(e) none of these (c) Dinesh Kumar Batra
22. Who of the following Indian-American investor took charge (d) Advesh Singh Sharma
as Alternate Executive Director at International Monetary (e) None of these
Fund (IMF)? 29. Who among the following has been appointed as non-
(a) Nikhil Chopra (b) Sunil Sabharwal official Director of CONCOR?
(c) Vikrant Singh (d) Dheerendra Payasi (a) R K Mishra (b) Sanjeev Shah
(e) None of these (c) M K Chaubey (d) Ajit Shah
23. Which Indian umpire has been promoted as the International (e) None of these
Outdoor Umpire by the International Hockey Federation 30. Who assumed charge as the new secretary of Railway
(FIH)? Board?
(a) Kumar Dixit (b) Jitendra Rai (a) Sanjoy Mookerjee (b) Gangaram Agarwal
(c) Nepoleon Singh (d) Mahendra Singh (c) R. K. Verma (d) P. C. Gajbhiye'
(e) None of these (e) None of these

15. a b c d e 16. a b c d e 17. a b c d e 18. a b c d e 19. a b c d e


RESPONSE 20. a b c d e 21. a b c d e 22. a b c d e 23. a b c d e 24. a b c d e
GRID 25. a b c d e 26. a b c d e 27. a b c d e 28. a b c d e 29. a b c d e
30. a b c d e
y
o
u
rs
m
a
h
b
89

o
o
b
.w
Events/Organisation/

o
rd
p
Summits

re
s
s
.c
o
m
Max. Marks : 30 No. of Qs. 30 Time : 20 min. Date : ........./......../................

1. Which of the following countries is not a member of ASEAN ? 9. In which country the G-20 2016 summit will be organised ?
(a) Thailand (b) Vietnam (a) Mexico (b) South Korea
(c) Myanmar (d) Brazil (c) China (d) USA
(e) None of these (e) None of these
2. Project ‘Sankalp’ is associated with the eradication of - 10. The 42nd summit of G-7 will be held in ________ .
(a) Polio (b) HIV/AIDS (a) USA (b) UK
(c) Illiteracy (d) Tuberculosis (c) Canada (d) Japan
(e) None of these (e) None of these
3. The 19th SAARC summit will be which held in country- 11. Which of the following countries became the new
(a) India (b) Nepal members of NAM ?
(c) Dhaka (d) Pakistan (a) Azerbaijan and Fiji
(e) None of these (b) Syria and Comoras
4. The 21th summit of ASEAN will be organised in- (c) Surinam and Guyana
(a) UAE (b) Brunei (d) Columbia and Cyprus
(c) Mascow (d) Laos (e) None of these
(e) None of these 12. The 2016 NATO summit will be organised in which country ?
5. 8th BRICS summit held in which country- (a) Russia (b) Poland
(a) Brazil (b) Russia (c) Canada (d) France
(c) S. Africa (d) India (e) None of these
(e) None of these 13. In which country first woman elected as a president ?
6. In which country the 2016 Annual Meeting of WEF organised ? (a) North Korea (b) South Korea
(a) Switzerland (b) France (c) Japan (d) China
(c) Germany (d) USA (e) None of these
(e) None of these 14. Who is elected as a new pope of Roman Catholic Church
7. How many members in the APEC nations organisation ? recently ?
(a) 20 (b) 21 (a) Mario Bergoylio (b) George Allensary
(c) 22 (d) 23 (c) Telesfor Toppo (d) Ivan Dias
(e) None of these (e) None of these
8. The 17th NAM summit will be held in the country- 15. The new organisation named “UN-women” created by
(a) Iran (b) Kahira
united nations came into existence on ?
(c) India (d) Venezuela (a) 1 July 2010 (b) 1 July 2011
(e) None of these (c) 1 July 2012 (d) 1 July 2013
(e) None of these

1. a b c d e 2. a b c d e 3. a b c d e 4. a b c d e 5. a b c d e
RESPONSE
6. a b c d e 7. a b c d e 8. a b c d e 9. a b c d e 10. a b c d e
GRID
11. a b c d e 12. a b c d e 13. a b c d e 14. a b c d e 15. a b c d e
y
o
u
rs
m
a
h
188 SPEED TEST 89

b
o
o
16. How many members are associated with BIMSTEC ? 25. 13th Edition of World Spice Congress was held in which

b
.w
(a) 7 (b) 8 state?

o
(c) 9 (d) 10 (a) Gujarat (b) Rajasthan

rd
(c) Kerala (d) Karnataka

p
(e) None of these

re
17. What is the Ranking of India in Global Hunger Index list ? (e) None of these

s
s
.c
(a) 63rd (b) 66th 26. The 12th Conference of Central Council of Health and Family

o
Welfare was held to enforce which policy?

m
(c) 67th (d) 68th
(e) None of these (a) Draft National Health Policy
18. How many members are associated with the organisation (b) Draft National Family Welfare Policy
‘MERCOSUR’ ? (c) Draft National Welfare Policy
(a) 5 (b) 6 (d) Draft National Health Welfare Policy
(c) 7 (d) 8 (e) None of these
(e) None of these 27. Annual Conference of State Minorities Commission which
19. The 9th world Hindi Conference held in which country ? commenced on 24th February 2016 has which theme(s)?
(a) S. Africa (b) Sri Lanka (a) "Sabka Saath,Sabka Vikas"
(c) Russia (d) USA (b) "Minority Welfare Schemes of Government of India-
An Overview"
(e) None of these
(c) "Functioning of State Minorities Commissions-
20. How many points are mentioned to the achievement of
Problems and Challenges
Millionium Development Goal LMDG-2015 of UN?
(d) Both (b) and (c)
(a) 6 (b) 7
(e) None of these
(c) 8 (d) 9
28. Which maritime summit is being held for the first time in
(e) None of these
India in April?
21. Where is the headquarter of F.A.O. ?
(a) Maritime India Summit
(a) Italy (b) Paris
(b) Maritime Cluster Development Summit
(c) Jeneva (d) Moscow
(c) Maritime Smart Cities Summit
(e) None of these
(d) Maritime Inland Waterways Summit
22. (SAMPRITI-III), a special security forces exercise organised
between the countries of - (e) None of these
(a) India-Sri Lanka (b) India-Bangladesh 29. Which website was launched for the Maritime Indian Summit
(c) India-Russia (d) India-USA 2016?
(e) None of these (a) www.maritimeinvestment.com
23. What does EAEU stand for? (b) www.indianmaritime.in
(a) Eurasian Economic Union (c) www.indiamaritimeinvestment.com
(b) European Economic Union (d) www.maritimeinvest.in
(c) European Energy Union (e) None of these
(d) Eurasian Energy Union 30. 10th WTO Ministerial Conference was held in which city?
(e) None of these (a) Nairobi
24. Nuclear Security Summit 2016 took place on which day? (b) Durban
(a) April 1, 2016 (b) April 2, 2016 (c) London
(c) April 3, 2016 (d) April 4, 2016 (d) New York
(e) None of these (e) None of these

16. a b c d e 17. a b c d e 18. a b c d e 19. a b c d e 20. a b c d e


RESPONSE
21. a b c d e 22. a b c d e 23. a b c d e 24. a b c d e 25. a b c d e
GRID
26. a b c d e 27. a b c d e 28. a b c d e 29. a b c d e 30. a b c d e
y
o
u
rs
m
a
h
b
90

o
o
b
Awards and

.w
o
rd
p
Honours

re
s
s
.c
o
m
Max. Marks : 30 No. of Qs. 30 Time : 20 min. Date : ........./......../................

1. Jnanpith Award is given for which field? 9. ‘Ashoka Chakra’ is awarded for
(a) Journalism (b) Music (a) the most conspicuous bravery or self sacrifice on land,
(c) Science (d) Literature air or sea but not in the presence of the enemy
(e) None of these (b) acts of gallantry in the presence of enemy
2. Highest award given to civilian in India is (c) gallantry by children
(a) Bharat Ratna (b) Padma Vibhushan (d) outstanding contribution to literature
(c) Sharam Award (d) Padma Bhushan (e) None of these
(e) None of these 10. Shanthi Swaroop Bhatnagar awards are given for
3. In which year National Film Awards were initiated? (a) exploring new dimensions in creative writing in Indian
(a) 1952 (b) 1953 languages
(c) 1954 (d) 1955 (b) outstanding contribution to science
(e) None of these (c) creating mass awareness on environmental issues
4. Vyas Samman is awarded annually by (d) excellence in film direction
(a) Azim Premji Foundation (e) None of these
(b) Times Group 11. The prestigious Ramon Magsaysay Award was conferred
(c) KK Birla Foundation upon Mr.Arvind Kejriwal in which of the following category?
(d) Ministry of Culture (a) Emergent Leadership
(e) None of these (b) Literature
5. Saraswati Samman is given to which field? (c) Community Welfare
(a) Sanskrit Literature (d) Government Service
(b) Science (e) None of these
(c) Literature 12. B. C. Roy Award is given in the field of
(d) Social Harmony (a) Medicine (b) Music
(e) None of these (c) Journalism (d) Environment
6. The second highest Gallantry award is (e) None of these
(a) Mahavir Chakra 13. The Pampa Prashasti is the highest literacy award given by
(b) Vir Chakra which of the following states?
(c) Arjuna Award (a) Karnataka
(d) Ashok Chakra (b) Kerala
(e) None of these (c) Andhra Pradesh
7. Which of the following states conferred the Bihari Puraskar? (d) Maharashtra
(a) Uttar Pradesh (e) None of these
(b) Bihar 14. Which among the following states has won the 10th Na-
(c) Madhya Pradesh tional Award for Excellence work in Mahatma Gandhi Na-
(d) Rajasthan tional Rural Employment Guarantee Act (MGNREGA)?
(e) None of these (a) Madhya Pradesh
8. Tansen Samman is conferred in the field of : (b) Karnataka
(a) Music (b) Literature (c) West Bengal
(c) Science (d) Journalism (d) Haryana
(e) None of these (e) None of these

1. a b c d e 2. a b c d e 3. a b c d e 4. a b c d e 5. a b c d e
RESPONSE 6. a b c d e 7. a b c d e 8. a b c d e 9. a b c d e 10. a b c d e
GRID 11. a b c d e 12. a b c d e 13. a b c d e 14. a b c d e
y
o
u
rs
m
a
h
190 SPEED TEST 90

b
o
o
15. Which of the following famous financial journals of interna- 23. Golden Globe award is given by

b
.w
tional repute confers ‘Finance minister of the year’ Award? (a) UK (b) France

o
(a) Dalal Street (c) USA (d) China

rd
(b) Euromoney (e) None of these

p
re
(c) Business Standard 24. Palme d’or prize is given by

s
s
(d) Money Matters (a) France (b) USA

.c
(c) UK (d) Indonesia

o
(e) None of these

m
16. The Nobel prize was instituted by which country? (e) None of these
(a) USA (b) UK 25. Which of the following is an award instituted by UNESCO?
(c) Russia (d) Sweden (a) Kalinga Award
(e) None of these (b) Pulitzer prize
17. When did the Nobel prize in the Economics Sciences (c) Stirling prize
launched? (d) Pritzker prize
(a) 1901 (b) 1942 (e) None of these
(c) 1967 (d) 1975 26. International Gandhi Peace prize is instituted in
(a) 1995 (b) 1996
(e) None of these
(c) 1997 (d) 1998
18. The Academy award is also known as
(e) None of these
(a) Oscar Award (b) BAFTA Award
27. The Stirling prize is a British prize for excellence in
(c) Matthews Award (d) Palm d’ore
(a) Medicine (b) Science
(e) None of these
(c) Architecture (d) Literature
19. Confucius peace prize is given by
(e) None of these
(a) Sri Lanka (b) India
28. Which of the following award is given by World Economic
(c) China (d) S. Korea Forum?
(e) None of these (a) Crystal Award
20. Pulitzer prize was established in (b) Kalinga prize
(a) 1917 (b) 1918 (c) Pulitzer Award
(c) 1922 (d) 1928 (d) Abel prize
(e) None of these (e) None of these
21. Nobel prizes are distributed annually at 29. Magsaysay award is given by
(a) Manila (b) New York (a) USA (b) UK
(c) Stockholm (d) Geneva (c) Malaysia (d) Philippines
(e) None of these (e) None of these
22. BAFTA prize is distributed by 30. Booker prize is given to the field of :
(a) UK (b) Russia (a) Fiction (b) Poetry
(c) India (d) USA (c) Drama (d) Essay
(e) None of these (e) None of these

15. a b c d e 16. a b c d e 17. a b c d e 18. a b c d e 19. a b c d e


RESPONSE 20. a b c d e 21. a b c d e 22. a b c d e 23. a b c d e 24. a b c d e
GRID 25. a b c d e 26. a b c d e 27. a b c d e 28. a b c d e 29. a b c d e
30. a b c d e
y
o
u
rs
m
a
h
b
91

o
o
b
.w
Books and Authors

o
rd
p
re
s
s
.c
o
m
Max. Marks : 30 No. of Qs. 30 Time : 20 min. Date : ........./......../................

1. Who is the author of “An Uncertain Glory: India and its 8. Who among the following is the author of “How to Get
Contradictions”? Filthy Rich in Rising Asia”?
(a) Amatya Sen & Michael Bush (a) Mohsin Hamid (b) Tahir Mahmood
(b) Amartya Sen & Satya Paul (c) Mohd Hamid (d) Mohd.Qazi
(c) Amartya Sen & Jean Dreze (e) Mohamed Yusuf
(d) Amartya Sen & Zeenat Shaukat 9. Who has written the book “Walking with Lions: Tales from
(e) Amartya Sen & Salman Rushdie a Diplomatic Past”?
2. Who is the author of “Ambedkar Speaks (Triology)”? (a) P.Chidambaram (b) K.Natwar Singh
(a) Dr Dheerendra (b) Dr Satyendra Singh (c) Sashwant Sinha (d) Jaswant Singh
(c) Dr Kiran Yadav (d) Dr Narendra Jadhav
(e) Manmohan Singh
(e) Akhilesh Yadav
10. A book ‘Fault Lines’ is written by -
3. Pax Indica is recently authored book by?
(a) Raghuram Rajan (b) Subba Rao
(a) Narendra Modi (b) Atal Bihari Vajpayee
(c) Vimal Jalan (d) Montek Singh Ahluwalia
(c) Shashi Tharoor (d) A P J Abdul Kalam
(e) None of these
(e) None of these
11. Which of the following books has been written by Vikram
4. Who is the author of “The Outsider”?
Seth?
(a) Jimmy Connors (b) Amartya Sen
(a) My God Died Young
(c) Jean Dreze (d) Zeenat Shaukat
(b) Islamic Bomb (c) Look Back in Anger
(e) Salman Rushdie
5. Name the book authored by Garima Sanjay which was (d) A Suitable Boy (e) None of these
released by Hamid Ansari. The book highlights how a man 12. Who is the author of the book India 2020?
blames the luck or someone for his failures? (a) Nibal Singh (b) R.K.Narayan
(a) Yaadein (b) Smritiyan (c) Sidney Shelton (d) Dr.A.P.J.Abdul Kalam
(c) Divine Journey (d) Lucky Me (e) None of these
(e) Man by Nature 13. The book ‘My Nation My Life’ was written by
6. Who is the author of “Women of Vision”? (a) L.K.Advani (b) T.N.Seshan
(a) Amish Tripathi (b) Preeti Shenoy (c) Fervez Musharaf (d) Manmohan Singh
(c) Durjoy Dutta (d) Alam Srinivas (e) None of these
(e) Ravinder Singh 14. The famous book ‘Anandmath’ was authored by
7. Who among the following is the author of “Religion, Law & (a) Sarojini Naidu
Society - Across the Globe”? (b) Bankim Chandra Chottapadhya
(a) Salman Rushdie (b) Tahir Mahmood (c) Sri Aurobindo
(c) Jim Herley (d) Mohd.Raza (d) Rabindrnath Tagore
(e) None of these (e) None of these

1. a b c d e 2. a b c d e 3. a b c d e 4. a b c d e 5. a b c d e
RESPONSE 6. a c d e 7. a c d e 8. a c d e 9. a c d e 10. a c d e
b b b b b
GRID 11. a c d e 12. a c d e 13. a c d e 14. a c d e
b b b b
y
o
u
rs
m
a
h
192 SPEED TEST 91

b
o
o
15. Who is the author of “INDIA : The Future is Now”. 23. Which of the following books is written by Panini?

b
.w
(a) Robin Sharma (b) Shashi Tharoor (a) Mudrarakshas (b) Ashtadhyayi

o
(c) Amartya Sen (d) Arundhati Roy (c) Prem Vatika (d) Bijak

rd
(e) None of these (e) None of these

p
re
16. ‘Freedom from Fear’ is a book written by ?
24. The book ‘Mitakshara’ is written by

s
(a) Benzir Bhutto (b) Corazon Aquino

s
(a) Vatsyayana (b) Jeemaatwahan

.c
(c) Aung san suu Kyi (d) Nayantara Seghal

o
(c) Vigyaneshwar (d) Shudrak

m
(e) None of these
17. ‘India of our Dreams’ is a book written by (e) None of these
(a) Dr. S. Radhakrishnan 25. Which of the following books is written by Kautilya?
(b) Dr. C. Subramanian (a) Daybhag (b) Rajtarangini
(c) M.V. Kamath (c) Arthashastra (d) Mitakshara
(d) Dr. Rajendra Prasad (e) None of these
(e) None of these 26. Post Office’ is written by?
18. Which of the following books is written by Sunil Gavaskar? (a) R.K. Narayan
(a) A Brief History of Time
(b) Mulk Raj Anand
(b) A Sense of Time (c) Sunny Days
(c) R.K Laxman
(d) Great Expectations (e) None of these
19. Which of the following is NOT written by Munshi (d) Rabindra Nath Tagore
Premchand? (e) None of these
(a) Gaban (b) Guide 27. Who is called the Father of English Poetry?
(c) Godan (d) Manasorovar (a) Charles Dickens (b) Milton
(e) None of these (c) Chaucer (d) Wordsworth
20. Who is the author of the book ‘Beyond the Lines : An (e) None of these
Autobiography’ ? 28. Who wrote Jungle Book?
(a) General J. J. Singh (b) Kuldip Nayar (a) Mohd. Salim
(c) Ray Bradbury (d) Khushwant Singh
(b) Rudyard Kipling
(e) None of these
(c) Sibhu
21. Who among the following is the author of the book “India
(d) R.K. Narayan
and Malaysia: Intertwined Strands”?
(a) Veena Sikri (b) Naveen Bandopadhya (e) None of these
(c) Sushma Singh (d) Ravichandran Nayak 29. Who wrote ‘War and Peace’?
(e) None of these (a) Leo Tolstoy (b) Mahatma Gandhi
22. The Global Competitiveness Report is published from time (c) Charles Dickens (d) Kipling
to time by ___? (e) None of these
(a) World Bank 30. Who wrote the book ‘The Prince’?
(b) International Monetary Fund (a) Bernard Shaw (b) Niccolo Machiaveli
(c) Yale University
(c) V.S. Sharma (d) Emile Zola
(d) World Economic Forum
(e) None of these
(e) None of these

15. a b c d e 16. a b c d e 17. a b c d e 18. a b c d e 19. a b c d e


RESPONSE 20. a b c d e 21. a b c d e 22. a b c d e 23. a b c d e 24. a b c d e
GRID 25. a b c d e 26. a b c d e 27. a b c d e 28. a b c d e 29. a b c d e
30. a b c d e
y
o
u
rs
m
a
h
b
92

o
o
b
.w
Sports and Games

o
rd
p
re
s
s
.c
o
m
Max. Marks : 30 No. of Qs. 30 Time : 20 min. Date : ........./......../................
1. The Olympic Museum was opened at which of the following 10. ‘Ashes’ is the term associated with which of the following
places? sports?
(a) Rome (b) Berlin (a) Cricket (b) Badminton
(c) Lausanne (d) Athens (c) Basketball (d) Football
(e) None of these (e) None of these
2. With which sport the term’ Caddie’ is associated? 11. National Sports Day is celebrated on
(a) Polo (b) Golf (a) 29th Aug. (b) 4th Dec.
(c) Bridge (d) Billiards (c) 14th Nov. (d) 28th Oct.
(e) None of these
(e) None of these
3. Champion Trophy is associated with
12. The term ‘bogey’ is associated with
(a) Football (b) Hockey
(a) Cricket (b) Chess
(c) Cricket (d) Chess
(c) Golf (d) Baseball
(e) None of these
(e) None of these
4. Rangaswami Cup is associated with
13. FINA is governing body of which sports?
(a) Wrestling (b) Football
(c) Hockey (d) Golf (a) Cricket (b) Archery
(e) None of these (c) Water polo (d) Polo
5. ‘Grand Slam’ is associated with the game of (e) None of these
(a) Lawn Tennis (b) Hockey 14. Which of the following trophies/cups associated with the
(c) Football (d) Swimming game of Hockey?
(e) None of these (a) Derby (b) Aga Khan Cup
6. First youth Olympic games was held in (c) Merdeka (d) Vizzy Trophy
(a) Japan (b) China (e) None of these
(c) North Korea (d) Singapore 15. The ‘Dronacharya Award’ is given to
(e) None of these (a) Coaches (b) Sportspersons
7. ‘Subroto Cup’ is associated with (c) Umpires (d) Sports Editors
(a) Badminton (b) Cricket (e) None of these
(c) Chess (d) Football 16. When did the Wimbledon Grand Slam Tennis tournament
(e) None of these start?
8. Wankhede Stadium is situated in (a) 1857 (b) 1877
(a) Mumbai (b) Delhi (c) 1897 (d) 1898
(c) Lucknow (d) Bangalore (e) None of these
(e) None of these 17. In which year, the Grand Master title of Chess started?
9. The term ‘Gambit’ is associated with (a) 1971 (b) 1972
(a) Chess (b) Tennis (c) 1973 (d) 1974
(c) Basketball (d) Baseball (e) None of these
(e) None of these

1. a b c d e 2. a b c d e 3. a b c d e 4. a b c d e 5. a b c d e
RESPONSE 6. a b c d e 7. a b c d e 8. a b c d e 9. a b c d e 10. a b c d e
GRID 11. a b c d e 12. a b c d e 13. a b c d e 14. a b c d e 15. a b c d e
16. a b c d e 17. a b c d e
y
o
u
rs
m
a
h
194 SPEED TEST 92

b
o
o
18. Duleep Trophy is associated with the game of 25. Indian Sports Research Institute is located at

b
.w
(a) Hockey (b) Badminton (a) Patiala (b) Delhi

o
(c) Football (d) Cricket (c) Cochin (d) Poona

rd
(e) None of these (e) None of these

p
re
19. Which game is associated with Queensberry rules? 26. Who was the first Indian to win an individual medal in Olym-

s
pics?

s
(a) Weight lifting (b) Boxing

.c
(c) Golf (d) Polo (a) PT Usha

o
m
(e) None of these (b) Karnam Malleshwari
20. How many players are there in Kho-Kho? (c) Deepika Kumari
(a) 9 (b) 10 (d) Sania Nehwal
(c) 8 (d) 7 27. Which county did Ravi Shastri play for?
(e) None of these (a) Glamorgan (b) Leicestershire
21. What is the National Game of Russia? (c) Gloucestershire (d) Lancashire
(a) Chess (b) Hockey (e) None of these
(c) Table Tennis (d) Baseball 28. The first World Cup Hockey was played in
(e) None of these (a) Amsterdom, 1972
22. Hurlington stadium is associated with (b) Barcelona, 1971
(a) Polo (b) Cricket (c) Kualalumpur, 1975
(c) Boxing (d) Golf (d) Mumbai, 1976
(e) None of these (e) None of these
23. First Olympic Games were held in- 29. The normal length of a football ground must be
(a) 776 BC. (b) 798 BC. (a) 110 – 120 m (b) 100 – 110 m
(c) 876 BC. (d) 898 BC. (c) 90 – 100 m (d) 120 – 130 m
(e) None of these (e) None of these
24. With which game is Bully associated? 30. India first won the Olympic Hockey gold at
(a) Cricket (b) Football (a) Amsterdam (b) Los Angeles
(c) Golf (d) Hockey (c) Mumbai (d) Tokyo
(e) None of these (e) None of these

18. a b c d e 19. a b c d e 20. a b c d e 21. a b c d e 22. a b c d e


RESPONSE 23. a b c d e 24. a b c d e 25. a b c d e 26. a b c d e 27. a b c d e
GRID 28. a b c d e 29. a b c d e 30. a b c d e
y
o
u
rs
m
a
h
b
93

o
o
Science and

b
.w
o
rd
Technology

p
re
s
s
.c
o
m
Max. Marks : 25 No. of Qs. 25 Time : 20 min. Date : ........./......../................

1. Line connecting the points on a map that have the same 8. What is the name of Australia’s most powerful super
temperature is called? computer ?
(a) Isobar (b) Isotherm (a) Spirit (b) Raijin
(c) Isohyet (d) Isohels (c) Aizen (d) Shinto
(e) None of these
(e) None of these
2. Rearing of silkworms for the production of raw silk is called?
9. Which was the first atomic submarine of India?
(a) Horticulture (b) Sericulture
(c) Viticulture (d) Apiculture (a) INS Chakra (b) INS Vikrant
(e) None of these (c) INS Dhanush (d) INS Viraat
3. In which of the following seas India has building Tsunami (e) None of these
warning device? 10. Vikram Sarabhai Space Centre (VSSC) is at
(a) Arabian Sea (b) South China Sea (a) Thiruvananthapuram
(c) Bay of Bengal (d) Indian Ocean
(b) Mumbai
(e) None of these
(c) Hyderabad
4. GlaxoSmithKline developed world’s first malaria vaccine
called ___? (d) Bengaluru
(a) RTS, S (b) MTS, S (e) None of these
(c) RMS, S (d) MRS, S 11. The first fertilizer plant in India was established in
(e) None of these (a) Trombay (b) Nangal
5. With which of the following Kepler’s Laws are related to (c) Alwaye (d) Sindri
_______?
(e) None of these
(a) Motion of Milkyway
12. Name of first indigenously developed Super Computer of
(b) Motion of planets around sun
India is?
(c) Rotation of Earth on Its own axis
(d) Motion of Moon around earth (a) Param (b) Aryabhatt
(e) None of these (c) Apsara (d) Tejas
6. Where is located the Centre for Wind Energy Technology (e) None of these
(C-WET)? 13. Which of the following is the principal ore of Mercury?
(a) Hyderabad (b) Chennai (a) Bauxite (b) Cinnabar
(c) Kochi (d) Kolkata (c) Hematite (d) Galena
(e) None of these
(e) None of these
7. Which of the following statements is true about “TomTato”?
14. Name the Chinese supercomputer which is declared the
(a) A new mammal species discovered in UK
fastest computer of the world-
(b) A plant produces both tomatoes and potatoes
(c) A man eating fish species (a) Tianhe-2 (b) Chinhane 1
(d) New element discovered under deep sea (c) Kisova (d) Techo-1
(e) None of these (e) None of these

1. a b c d e 2. a b c d e 3. a b c d e 4. a b c d e 5. a b c d e
RESPONSE 6. a b c d e 7. a b c d e 8. a b c d e 9. a b c d e 10. a b c d e
GRID 11. a b c d e 12. a b c d e 13. a b c d e 14. a b c d e
y
o
u
rs
m
a
h
196 SPEED TEST 93

b
o
o
15. Name the organization that launched a series of satellites 21. The Rotavirus vaccine ROTOVAC developed by Indian

b
which is supposed to provide fast, cheap Internet and phone

.w
scientists cures which disease?
service to remote rural areas in 180 countries.

o
(a) Diarrohea (b) Cancer

rd
(a) A2B Networks (b) R2R Networks (c) Diabetes (d) Arthritis

p
re
(c) O3B Networks (d) M2M Networks (e) Pneumonia

s
(e) B2B Networks

s
22. India's first DNA Forensic Laboratory is established in which

.c
16. Name the place in India where Early Tsunami Warning

o
city

m
System have been installed
(a) Gurgaon (b) Mumbai
(a) Rangachang (b) Kanyakumari
(c) Delhi (d) Kanpur
(c) Chilka (d) Mysore
(e) Banglore
(e) Ootkamandalam
23. "A Boy and His Atom" - World's smallest movie made with
17. Researchers have developed Eco-friendly batteries which
can be used in power plants or to store solar energy.These one of the smallest particles of any element in the universe:
batteries are made up using which material atoms is developed by which organization
(a) Carbon, Tin, Sodium (a) Microsoft
(b) Wood, Tin, Carbon (b) Infosys
(c) Wood, Tin and Sodium (c) IBM
(d) Sodium, carbon, Tin (d) Toshiba
(e) Nickel, Carbon, Zinc (e) Sony
18. Which of the following is an indigenously built light combat 24. Which among the following is the First cruise missile test
aircraft of India? fired by India?
(a) Akash (b) Vikrant (a) Aakash
(c) Tejas (d) Arjun (b) Nirbhay
(e) None of these (c) Agni-3
19. Name the broadband and telecommunications provider (d) Aakash-2
which launched world's first Firefox OS smartphone?
(e) Prithvi-3
(a) Telefonica (b) Apple
25. First Indian to go into space
(c) Samsung (d) Telenor
(a) Mohan Sharma
(e) None of these
(b) Rakesh Chaudhary
20. The First Navigation Satellite launched by ISRO
(c) Rakesh Sharma
(a) PSLV C 2 (b) IRNSS-1A
(d) Suresh Sharma
(c) ISS-1A (d) INSAT
(e) None of these
(e) Edusat

15. a b c d e 16. a b c d e 17. a b c d e 18. a b c d e 19. a b c d e


RESPONSE
20. a b c d e 21. a b c d e 22. a b c d e 23. a b c d e 24. a b c d e
GRID
25. a b c d e
y
o
u
rs
m
a
h
b
94

o
o
b
.w
Current Banking

o
rd
p
re
s
s
.c
o
m
Max. Marks : 23 No. of Qs. 23 Time : 15 min. Date : ........./......../................
1. India's first Aadhaar enabled ATM has been launched by 8. Which bank has launched 'Japan Desk' to facilitate Japanese
which bank? corporates investing in India?
(a) State Bank of India (b) ICICI Bank (a) State Bank of India (b) Punjab National Bank
(c) DCB Bank (d) Axis Bank (c) Bank of Baroda (d) Dena Bank
(e) Indian Bank (e) ICICI Bank
2. Which is the first bank to commence the sale of Indian Gold 9. Which of the following agreement is signed between Reserve
Coin (IGC) in the domestic market? Bank of India (RBI) and UAE Central Bank?
(a) State Bank of India (b) Indian Overseas Bank (a) Monetary Exchange Agreement
(c) Punjab National Bank (d) Bank of Baroda (b) Currency Swap Agreement
(c) Anti-Money Laundering Agreement
(e) Indian Bank
(d) Rupee Exchange Agreement
3. Who is the newly appointed Governor of Central Bank of
Bangladesh? (e) Money Exchange Agreement
10. Who has been appointed as Vice-President of Asian
(a) Fazle Kabir (b) Abdul Hamid
Infrastructure Investment Bank (AIIB)?
(c) AMA Muhith (d) Shahana Rahman
(a) Kamal Vatta (b) D J Pandian
(e) None of these (c) Mitali Saran (d) Sreenivasan Kumar
4. India's first contact-less mobile payment solution "iTap" has (e) None of these
been launched by which bank?
11. Who has been appointed from India as Senior Director in
(a) State Bank of India (b) HDFC Bank the World Bank?
(c) Axis Bank Ltd (d) ICICI Bank (a) Amitabh Singh (b) Saroj Kumar Jha
(e) None of these (c) Suvarna Kumar (d) Nikhil Srivastav
5. Which bank has launched iWork@home programme for its (e) None of these
women employees? 12. Which bank has recently raised `900 crore by way of issuing
(a) Yes bank (b) Exim Bank Basel-III compliant tier-II bonds?
(c) ICICI Bank (d) Axis Bank Ltd. (a) IDBI Bank (b) Central Bank of India
(e) HDFC Bank (c) Syndicate Bank (d) Canara Bank
6. Which of the following banks will become the India's first (e) State Bank Of India
Small Finance Bank (SFB)? 13. Which public sector bank has bagged three awards at the
(a) Ujjivan Financial Services Pvt. Ltd 'National Payments Excellence Awards 2015, organized by
(b) Capital Local Area Bank Ltd National Payments Corporation of India (NPCI)?
(a) Bank of Baroda (b) State Bank of India
(c) Au Financiers Ltd
(c) Allahabad Bank (d) Corporation Bank
(d) Janalakshmi Financial Services Pvt. Ltd
(e) None of these
(e) None of these
14. Which bank has recently launched Green PIN facility, under
7. Which bank has won the first ever Green Bond Pioneer Award
which a customer can obtain duplicate PIN for debit card
2016?
instantly through SMS request?
(a) State Bank of India (b) Bank of Baroda
(a) Punjab National Bank (b) ICICI Bank
(c) Punjab National Bank (d) Yes Bank
(c) Dena Bank (d) HDFC Bank
(e) ICICI Bank
(e) None of these

1. a b c d e 2. a b c d e 3. a b c d e 4. a b c d e 5. a b c d e
RESPONSE 6. a b c d e 7. a b c d e 8. a b c d e 9. a b c d e 10. a b c d e
GRID 11. a b c d e 12. a b c d e 13. a b c d e 14. a b c d e
y
o
u
rs
m
a
h
198 SPEED TEST 94

b
o
o
15. Reserve Bank has imposed a penalty of Rs. 1 crore on which (a) South Korea (b) China

b
associate bank of SBI for violation of its instructions

.w
(c) Brazil (d) South Africa
including reporting of data to Central Repository of

o
(e) None of these

rd
Information on Large Credits (CRILC)? 20. Under what name, State Bank of India has launched a

p
(a) State Bank of Patiala

re
specialised branch in Bengaluru that caters to startups?

s
(b) State Bank of Bikaner and Jaipur (a) InCube (b) Foster

s
.c
(c) State Bank of Hyderabad (c) KickStart (d) SBIHatch

o
m
(d) State Bank of Travancore (e) None of these
(e) None of these 21. Which public sector bank has tied up with e-commerce major
16. Who among the following has been reappointed as Deputy Snapdeal to offer instant working capital loans to its online
Governor of RBI for three year term? e-commerce sellers?
(a) Rakesh Mohan (b) Urjit Patel (a) Bank of Baroda (b) State Bank of India
(c) H R Khan (d) S S Mundra (c) Corporation Bank (d) Syndicate Bank
(e) None of these (e) None of these
17. In January 2016, which bank had launched Immediate 22. The 2016 World Development Report by World Bank noted
Payment Service (IMPS) across branches to provide inter- that almost 1.063 billion Indians were offline even though
bank electronic transfer service, capable of processing India ranked among the top five nations in terms of the total
person to account remittances? number of Internet users. What is the title of 2016 World
(a) Central Bank of India (b) Canara Bank Development Report?
(c) Andhra Bank (d) Corporation Bank (a) 'Technology Inclusion'
(e) None of these (b) 'Digital Dividends'
18. Which private sector bank will partner with FINO PayTech (c) 'Sustainable Technology'
to foray into the payments bank space? (d) 'Bridging Technology Divide'
(a) Axis Bank (b) HDFC Bank (e) None of these
(c) Yes Bank (d) ICICI Bank 23. Which Central Bank has imposed negative interest rate in
(e) None of these its monetary policy statement of January 2016?
19. Which country has recently allowed Six central banks and (a) Bank of England (b) People's Bank of China
international financial institutions, including Reserve Bank (c) Bank of Japan (d) European Central bank
of India (RBI), to enter China's interbank foreign exchange (e) None of these
market from January 2016?

RESPONSE 15. a b c d e 16. a b c d e 17. a b c d e 18. a b c d e 19. a b c d e

GRID 20. a b c d e 21. a b c d e 22. a b c d e 23. a b c d e


y
o
u
rs
m
a
h
b
95

o
o
b
.w
o
Current Affairs

rd
p
re
s
s
.c
o
m
Max. Marks : 30 No. of Qs. 30 Time : 15 min. Date : ........./......../................
1. The Pradhan Mantri Ujjwal Yojana will provide - 8. Where was the fourth Nuclear Security Summit (NSS) held?
(a) free electricity to the rural households. (a) London (b) Hong Kong
(b) free higher education to the girl child who belongs to a (c) Washington (d) New Delhi
poor family. (e) None of these
(c) LPG to women of all households below poverty line. 9. Who was appointed as the advisor of the Andhra Pradesh
(d) scholarships to the children of the people below Government by Chief Minister Chandrababu Naidu?
poverty line. (a) Baba Ram dev
(e) None of these (b) Kodali Venkateswara Rao
(c) Hari Krishna
2. Who is the Vice Chairman of NITI AAYOG in India?
(d) Chaganti Koteswara Rao
(a) A P Shan
(e) None of these
(b) Arvind Pangariya 10. Who was sworn-in as the President of Kosovo in April 2016?
(c) Rakesh Tripath (a) Jakup Krasniqi (b) Hashim Thaci
(d) B.P. Jeevan Reddy (c) Atifete Jahjaga (d) Fadil Hoxha
(e) None of these (e) None of these
3. Who is the winner of the prestigious 2016 Abel Prize? 11. Which team topped the April 2016 edition of FIFA or Coca-
(a) Endre Szemeredi (b) Andrew Wiles Cola World Rankings of Soccer?
(c) John Rognes (d) Michael Atiyah (a) Argentina (b) Brazil
(e) None of these (c) France (d) Italy
4. What is the rank of India in the United Nations, World (e) None of these
Happiness Index 2016? 12. Which online furniture store was acquired by the Future
(a) 118 (b) 105 Group in the first week of April 2016?
(c) 98 (d) 125 (a) FabFurnish.com (b) UrbanLadder.com
(e) None of these (c) Pepperfry.com (d) Ezaara.com
(e) None of these
5. Where was 13th meeting of Broadband Commission for
13. The book 'Endurance: My Year in Space and Our Journey to
Sustainable Development held?
Mars' as announced by the publisher Alfred A. Knopf, is a
(a) New York (b) Dubai
story of which famous personality?
(c) Switzerland (d) Mexico
(a) Scott Kelly (b) Mark Kelly
(e) None of these (c) Mikhail Korniyenko (d) Timothy Kopra
6. March 15 is celebrated across the world as? (e) None of these
(a) World Human Rights Day 14. Who is going to succeed Vijay Mallya as the chairman of
(b) World Health Day United Spirits Limited?
(c) World Consumer Rights Day (a) Anand Kripalu (b) M.K. Sharma
(d) World Sleep Day (c) Nicholas Blazquez (d) Angus Mc Dowell
(e) None of these (e) None of these
7. The fifth edition of Indian Aviation show was conducted in 15. What is the theme of 2016 World Health Day (WHD)?
which city? (a) Healthy heart beat, Healthy blood pressure
(a) Odisha (b) Mumbai (b) Beat Diabetes
(c) Bangalore (d) Hyderabad (c) Healthy blood pressure
(e) None of these (d) Save lives: Make hospitals safe in emergencies
(e) None of these

1. a b c d e 2. a b c d e 3. a b c d e 4. a b c d e 5. a b c d e
RESPONSE 6. a b c d e 7. a b c d e 8. a b c d e 9. a b c d e 10. a b c d e
GRID 11. a b c d e 12. a b c d e 13. a b c d e 14. a b c d e 15. a b c d e
y
o
u
rs
m
a
h
200 SPEED TEST 95

b
o
o
16. Who won the 2016 Formula 1 Rolex Australian Grand Prix at 23. India extended 2 billion US dollar line of Credit to which

b
Melbourne Grand Prix? country for implementing Socio-economic development

.w
(a) Sebastian Vettel (b) Nico Rosberg projects?

o
rd
(c) Daniel Ricciardo (d) Lewis Hamilton (a) Bangladesh (b) Nepal

p
(e) None of these (c) Sri Lanka (d) Myanmar

re
17. Which unique Biosphere Reserve of India in the Western (e) None of these

s
s
24. Who was appointed as the Chairman of Central

.c
Ghats is among 20 new sites added by the UNESCO to its

o
World Network of Biosphere Reserves? Administrative Tribunal in April 2016?

m
(a) Great Nicobar (a) Justice (retd) Permod Kohli
(b) Simlipal Biosphere Reserve (b) Justice Syed Rafat Alam
(c) Agasthyamala Biosphere (c) Justice A P Shah
(d) Nanda Devi Biosphere Reserve (d) None of them
(e) None of these (e) None of these
18. Who was conferred with skoch Lifetime Achievement Award 25. Which has become the first state to pass land title bill?
in 2016? (a) Rajasthan (b) Bihar
(a) Venkaiah Naidu (b) Arun Jaitley (c) Punjab (d) Karnataka
(c) Rajnath Singh (d) L K Advani (e) None of these
(e) None of these 26. Who on 6 April 2016 was appointed the Head of Lamborghini
19. Bedaquiline drug launched by the Health Ministry will be India Operations?
used to treat which disease? (a) Vincet Parker (b) Sharad Agarwal
(a) Dengue (b) Tuberculosis (c) Himanshu Sharma (d) Vivek Agnihotri
(c) Malaria (d) Zika Virus (e) None of these
(e) None of these 27. Who was awarded the UNESCO or Guillermo Cano World
Press Freedom Prize 2016?
20. Which veteran singer was awarded the Global Green Hero
Award 2016? (a) Mazen Darwish (b) Khadija Ismayilova
(c) Ahmet ??k (d) None of the above
(a) Stevie Wonder (b) Lionel Richie
(e) None of these
(c) Whitney Houston (d) None of the above
28. Who wrote the book titled "India's War: The Making of
(e) None of these
Modern South Asia 1939-1945"?
21. What was the theme of International women's Day in 2016?
(a) Srinath Raghavan (b) Mini Kapoor
(a) Equality for women is progress for all
(c) Sreedhar Chandan (d) Suhasini Haidar
(b) Planet 50-50 by 2030: Step It up for Gender Equality
(e) None of these
(c) Empowering women, Empowering Humanity: Picture it!
29. Who has won the 2016 US Masters golf tournament?
(d) Empowerment of women for sustainable Development (a) Jordan Spieth (b) Lee Westwood
(e) None of these (c) Danny Willett (d) J Donaldson
22. The Indian women's team on 6 March 2016 defeated which (e) None of these
country to bag gold medal at the Second Division of the 30. Whose birth anniversary was celebrated on April 13, 2016
World Team Table Tennis Championship? at the UN headquarters?
(a) China (b) Brazil (a) B R Ambedkar (b) Annie Besant
(c) Luxembourg (d) Japan (c) Florence Nightingale (d) George Washington
(e) None of these (e) None of these

16. a b c d e 17. a b c d e 18. a b c d e 19. a b c d e 20. a b c d e


RESPONSE
21. a b c d e 22. a b c d e 23. a b c d e 24. a b c d e 25. a b c d e
GRID
26. a b c d e 27. a b c d e 28. a b c d e 29. a b c d e 30. a b c d e
y
o
u
rs
m
a
h
b
96

o
Section Test :

o
b
.w
o
General & Financial

rd
p
re
Awareness

s
s
.c
o
m
Max. Marks : 50 No. of Qs. 50 Time : 30 min. Date : ........./......../................

1. Which of the following games is not included in Olympic 8. Which of the following is not a Public Sector Bank?
games? (a) Corporation Bank (b) United Bank of India
(a) Football (b) Golf (c) Vijaya Bank (d) Bank of Maharashtra
(c) Badminton (d) Hockey (e) Federal Bank
(e) Table Tennis 9. A form of intentional weather modification by changing to
2. Which of the following awards is given only to individuals trying the amount or type of precipitation that falls from clouds
and organisations of Asian countries for excellence in their by dispersing substances into air that serve as cloud
respective fields?
condensation is called ___________.
(a) Booker Prize (b) Nobel Prize
(a) Cloud Computing (b) Cloud Technology
(c) Templeton Prize (d) Ramon Magsaysay Award
(c) Cloud Seeding (d) Cloud Control
(e) Oscar awards
(e) Cloud Engineering
3. Which Indian city is known as the ‘Garden City’?
10. The North Atlantic Treaty Organisation (NATO) is
(a) Udaipur (b) Chandigarh
(c) Srinagar (d) Bangaluru headquartered at _________.
(e) Mysore (a) Camp David, Maryland USA
4. Who amongst the following is the author of the English novel (b) Geneva
‘The White Tiger’? (c) New York
(a) Kiran Desai (b) V. S. Naipaul (d) Sweden
(c) Shashi Tharoor (d) Shobha De (e) Brussels
(e) Aravind Adiga 11. Which of the following is the abbreviated name of a rural
5. Under the RTI Act the time for disposal of request for sports initiative introduced by the Government of India to
information in cases concerning life and liberty is _________. promote youth and social development through sports?
(a) 30 days (b) 15 days (a) NLMP (b) PYKKA
(c) 10 days (d) 7 days (c) CGHS (d) AABY
(e) 48 hours (e) SGSRY
6. World ‘No Tobacco Day’ is observed every year on 12. National Sample Survey Organisation is an organisation of
__________. (a) Ministry of statistics and programme implementation
(a) 31st January (b) 31st May
(b) Planning Commission
(c) 31st July (d) 31st October
(c) Home Affairs
(e) 31st December
(d) RBI
7. Which of the following is not a Consumer Right as per
(e) None of these
Consumer Protection Act 1986?
13. License to establish a commercial bank in India is issued
(a) Right to Consumer Education
(b) Right to Seek Redressal by—
(c) Right to Safety (a) Ministry of Finance (b) Registrar of companies
(d) Right to be Informed (c) Reserve Bank of India (d) Planning Commission
(e) Right to Negotiate (e) None of these

1. a b c d e 2. a b c d e 3. a b c d e 4. a b c d e 5. a b c d e
RESPONSE 6. a b c d e 7. a b c d e 8. a b c d e 9. a b c d e 10. a b c d e
GRID 11. a b c d e 12. a b c d e 13. a b c d e
y
o
u
rs
m
a
h
202 SPEED TEST 96

b
o
o
14. On which one of the following issues can SEBI penalize any 20. India vision-2020 is prepared by

b
.w
company in India? (a) S.C. Gupta Committee

o
(A) Violation of Banking Regulation Act.

rd
(b) Panth Committee

p
(B) Violation of foreign portfolio investment guidelines. (c) Malhotra Commitee

re
(C) For violation of Negotiable Instrument Act.

s
(d) Narasimha Committee

s
.c
(a) Only (A) (b) All (A), (B) & (C)
(e) Kelkar Committee

o
m
(c) Only (A) & (B) (d) Only (B) & (C)
21. The National Maritime Day of India is celebrated on which
(e) Only (B)
day?
15. Which one the following is the west flowing river?
(a) Narmada (b) Krishna (a) April 3 (b) April 4
(c) Godavari (d) Cauvery (c) April 5 (d) April 6
(e) Ganga (e) None of these
16. Which of the following best defines a floating-rate bond? 22. The "Mullaperiyar Dam" has been a subject of controversy
(a) A bond with a fixed interest rate and has better yield between which of the following two states?
than varying interest rate bond (a) U.P and Uttarakhand
(b) A bond with a fixed interest rate and has lower yield (b) Kerala & Tamil Nadu
than varying interest rate bond
(c) Telangana and Andhra Pradesh
(c) A bond with a varying interest rate and has better yield
(d) Kerala & Karnataka
than fixed interest rate bond
(e) None of these
(d) A bond with a varying interest rate and has lower yield
23. The headquarters of the Confederation of Indian Industry
than fixed interest rate bond
(e) None of these (CII) is located at_____.
17. Once a Budget has been presented in the Parliament, the (a) Mumbai (b) Chandigarh
government has to get all money bills related to the union (c) New Delhi (d) Chennai
budget passed within __? (e) None of these
(a) 30 Days (b) 60 Days (c) 75 Days 24. Which of the following is India's first state to link government
(d) 90 Days (e) None of these colleges to national cancer grid?
18. Which among the following regulate the commodity markets (a) Rajasthan (b) Maharashtra
in India?
(c) Madhya Pradesh (d) Odisha Hide
1. RBI
(e) None of these
2. SEBI
25. Which of the following villages has become the first liquor-
3. Forward Market Commission
Choose the correct option from the codes given below: free village in Rajasthan?
(a) Only 1 & 2 (b) Only 2 & 3 (c) Only 1 & 3 (a) Dausa (b) Kachhabali
(d) Only 3 (e) None of these (c) Churu (d) Achalpura
19. The credit policy of a bank does not consists of? (e) None of these
(a) Lending policies 26. Kakrapara Atomic Power Station' is located in which state
(b) Quality control of India?
(c) Loan product mix (a) Gujarat (b) Karnataka
(d) Advertising of loan products (c) Maharashtra (d) Rajasthan
(e) None of these (e) None of these

14. a b c d e 15. a b c d e 16. a b c d e 17. a b c d e 18. a b c d e


RESPONSE 19. a b c d e 20. a b c d e 21. a b c d e 22. a b c d e 23. a b c d e
GRID 24. a b c d e 25. a b c d e 26. a b c d e
y
o
u
rs
m
a
h
SPEED TEST 96 203

b
o
o
27. Which committee has recommended change in the structure 34. Gurmeet Singh is associated with which sports?

b
.w
and ecosystem in the Indian cricket board? (a) Race walk (b) Swimming

o
(a) Kirit Parikh committee (c) Hockey (d) Kabaddi

rd
p
(b) RM Lodha committee (e) None of these

re
35. Which is the first bank to commence the sale of Indian Gold

s
(c) Bibek Debroy committee

s
.c
(d) Naresh Chandra committee Coin (IGC) in the domestic market?

o
(a) State Bank of India

m
(e) None of these
28. Shuklaphanta Wildlife Reserve' is located in which country? (b) Indian Overseas Bank
(a) Nepal (b) Bhutan (c) Punjab National Bank
(d) Bank of Baroda
(c) Myanmar (d) Sri Lanka
(e) None of these
(e) None of these
36. Which organisation is providing loan support for "Madhya
29. Which of the following virus is responsible for diarrhoea
Pradesh Citizen Access to Responsive Services Project"?
among infants and young children?
(a) World Bank (WB)
(a) Parvovirus (b) Norwalk virus
(b) International Monetary Fund (IMF)
(c) Megavirus (d) Rotavirus
(c) Asian Development Bank (ADB)
(e) None of these
(d) Asian Infrastructure Investment Bank (AIIB)
30. The International Day of Remembrance of the Victims of
(e) None of these
Slavery and the Transatlantic Slave Trade", is observed on 37. Which is the National Intelligence Agency of Israel?
which date? (a) GRU (b) NSIS
(a) March 23 (b) March 24 (c) CISEN (d) Mossad
(c) March 25 (d) March 26 (e) None of these
(e) None of these 38. Which organisation has recently released the e-Atlas of
31. Swayam Shikshan Prayog (SSP) is an organisation which Gender Inequality in Education?
aims to: (a) UNESCO (b) UNICEF
(a) promote empowerment of women (c) UNFPA (d) UNSDN
(b) protect the rights of children (e) None of these
(c) protect the human rights of people 39. Songkran festival is celebrated in which country?
(d) take care of Mentally or Physically Challenged Persons (a) India (b) Thailand
(e) None of these (c) Indonesia (d) China
32. What does BEPZA stands for? (e) None of these
(a) Bangladesh Export Processing Zones Authority 40. India's first contact-less mobile payment solution "iTap" has
(b) Brazil Export Processing Zones Authority been launched by which bank?
(c) Belgium Export Processing Zones Authority (a) State Bank of India
(d) Bahamas Export Processing Zones Authority (b) HDFC Bank
(c) Axis Bank Ltd
(e) None of these
(d) ICICI Bank
33. Which is the India's oldest paramilitary force?
(e) None of these
(a) Indian Coast Guard
41. Which state government has launched awareness
(b) Special Frontier Force
programme to protect sparrows?
(c) Assam Rifles
(a) Madhya Pradesh (b) Uttar Pradesh
(d) None of the above (c) Odisha (d) Punjab
(e) None of these (e) None of these

27. a b c d e 28. a b c d e 29. a b c d e 30. a b c d e 31. a b c d e


RESPONSE 32. a b c d e 33. a b c d e 34. a b c d e 35. a b c d e 36. a b c d e
GRID 37. a b c d e 38. a b c d e 39. a b c d e 40. a b c d e 41. a b c d e
y
o
u
rs
m
a
h
204 SPEED TEST 96

b
o
o
42. What is the new name of the LED based Domestic Efficient 47. Which state has launched Mukhyamantri Santwana Harish

b
.w
Lighting Programme (DELP)? Yojana?

o
(a) Jyoti (b) Ujala (a) Himachal Pradesh

rd
(c) Disha (d) Prakash

p
(b) Tamil Nadu

re
43. Who is the first Indian classical musician to perform at

s
(c) Karnataka

s
United Nations?

.c
(d) Odisha

o
(a) Pt. Bhimsen Joshi (b) Pt. Jasraj

m
(c) Ustad Jakir Hussain (d) MS Subbulakshmi (e) None of these
(e) None of these 48. Which of the following banks will become the India's first
44. Which of the following two seas surround the Crimean Small Finance Bank (SFB)?
Peninsula? (a) Ujjivan Financial Services Pvt. Ltd
(a) Black Sea and Sea of Azov (b) Capital Local Area Bank Ltd
(b) Wadden Sea and Black Sea
(c) Au Financiers Ltd
(c) Caspian Sea and Sea of Azov
(d) Janalakshmi Financial Services Pvt. Ltd
(d) Black Sea and Mediterranean Sea
(e) None of these (e) None of these
45. Which of the following is not a correct statement about 49. India's first solar powered ferry will come up in which state?
Infrasound? (a) Gujarat
(a) Infrasound has a frequency lower than 50 hertz (b) Kerala
(b) Infrasound is produced / detected by elephants and (c) Goa
whales (d) Andhra Pradesh
(c) Infrasound can travel longer distances than high
(e) None of these
frequency sound waves
50. India's first self cleaning smart toilets have been installed in
(d) All are correct
(e) None of these which city?
46. Which bank has launched iWork@home programme for its (a) Chennai
women employees? (b) Bengaluru
(a) Yes bank (b) Exim Bank (c) Kolkata
(c) ICICI Bank (d) Axis Bank Ltd. (d) Mumbai a
(e) None of these (e) None of these

RESPONSE 42. a b c d e 43. a b c d e 44. a b c d e 45. a b c d e 46. a b c d e

GRID 47. a b c d e 48. a b c d e 49. a b c d e 50. a b c d e


y
o
u
rs
m
a
h
b
97

o
o
b
.w
Prelim Test - 1

o
rd
p
re
s
s
.c
o
m
Max. Marks : 100 No. of Qs. 100 Time : 1 hr. Date : ........./......../................

Part-I : Numerical Ability 9. 78.45 + 128.85 + 1122.25 = ?


(a) 1329.55 (b) 1239.55
DIRECTIONS (Qs. 1-15): What will come in place of the question (c) 1329.45 (d) 1239.45
mark (?) in the following questions. (e) None of these
5 4 11 10. 8729 – 4376 + 1245 = ? + 2785
1. of of of 848=? (a) 2713 (b) 2823
11 5 6
(c) 2833 (d) 2733
(a) 216 (b) 222 (e) None of these
(c) 208 (d) 212
2 5 7
(e) None of these 11. 17 4 ? 46
2. 1.4% of 750 + 2.2% of 480 = ? 5 8 8
(a) 21.06 (b) 21.16 3 3
(a) 32 (b) 33
(c) 20.88 (d) 21.18 5 5
(e) None of these
2 2
3 2 (c) 33 (d) 32
3. of 116 of 87=? 5 5
4 3
(a) 31 (b) 27 (e) None of these
(c) 29 (d) 26 12. 5616 ÷ 18 ÷ 8 = ?
(e) None of these (a) 36 (b) 76
4. 6.96 ÷ 1.2 – 18.24 ÷ 7.6 = ? (c) 49 (d) 39
(a) 3.4 (b) 3.14 (e) None of these
(c) 3.04 (d) 3.24 13. 420 ÷ 28 × 288 ÷ 32 = ?
(e) None of these (a) 235 (b) 236
5. 32.25 × 2.4 × 1.6 = ? (c) 138 (d) 132
(a) 128.84 (b) 123.84 (e) None of these
(c) 112.88 (d) 112.84 14. 222 ? 516
(e) None of these (a) 1029 (b) 1024
6. 136% of 250 + ? % of 550 = 670 (c) 1124 (d) 1128
(a) 64 (b) 55 (e) None of these
(c) 56 (d) 65 15. 45% of 660 + 28% of 450 = ?
(e) None of these (a) 413 (b) 428
7. 448 ÷ 16 × 35 = ? (c) 423 (d) 418
(e) None of these
(a) 850 (b) 890
(c) 950 (d) 980 DIRECTIONS (Qs. 16–20) : What will come in place of the
(e) None of these question mark (?) in the following number series.
16. 12 16 24 40 ?
14 × 25 – 53 (a) 76 (b) 72
8. ?
24 5 8 9 (c) 84 (d) 88
9 64 (e) None of these
(a) 1 (b)
64 75 17. 9 19 39 79 ?
(a) 139 (b) 129
11 11
(c) 1 (d) 1 (c) 159 (d) 149
64 75
(e) None of these (e) None of these
y
o
u
rs
m
206 SPEED TEST 97

a
h
b
18. 8 17 42 91 ? 28. Number obtained by interchanging the digit of a two digit

o
o
(a) 170 (b) 142 number is more than the original number by 27 and the sum

b
(c) 140 (d) 172

.w
of the digits is 13. What is the original number?
(e) None of these

o
(a) 58 (b) 67

rd
19. 7 8 18 57 ? (c) 76 (d) 85

p
(a) 244 (b) 174

re
(e) None of these
(c) 186 (d) 226

s
29. 22 men can complete a job in 16 days. In how many days will

s
(e) None of these

.c
20. 3840 960 240 60 ? 32 men complete that job?

o
m
(a) 20 (b) 18 (a) 14 (b) 12
(c) 12 (d) 22 (c) 16 (d) 9
(e) None of these (e) None of these
21. 75% of a number is equal to three seventh of another number. 30. A, B, C, D and E are five consecutive odd numbers. Average
What is the ratio between the first number and the second of A and C is 59. What is the smallest number?
number respectively? (a) 65 (b) 63
(a) 4 : 7 (b) 7 : 4 (c) 61 (d) 57
(c) 12 : 7 (d) 7 : 12
(e) None of these
(e) None of these
22. A 275 metre long train crosses a platform of equal length in Direction (Qs. 31-35) : Study the following table carefully and
33 seconds. What is the speed of the train in kmph ? answer accordingly :
(a) 66 (b) 60
(c) 64 (d) 72 The distribution of marks (out of 150) obtained by 180 students in
(e) None of these each of the five subjects.
23. What is compound interest accrued on an amount of ` 45,000
in two years at the rate of 9 p.c.p.a? Marks
0-29 30-59 60-89 90-119 120-150
(a) ` 8,600 (b) ` 8,565.40 Sub
(c) ` 8,464.50 (d) ` 8,540 Maths 22 47 74 25 12
(e) None of these 39 38 67 22 14
Science
9 7 5 4 11 19 59 47 36 19
24. If the fractions , , , and are arranged in Hindi
11 9 6 5 13 24 41 58 34 23
Englis h
ascending order, which one will be the fourth?
Geography 42 32 52 41 13
9 7
(a) (b) Average of
11 9 27 45 60 31 17
five s ubjects
5 4
(c) (d)
6 5 31. If for passing, the student has to obtain minimum 60% marks
11 in the average of five subjects, how many students will
(e) pass?
13
25. Srikant and Vividh started a business investing amounts of (a) 108 (b) 58
` 1, 85,000 and ` 2, 25,000 respectively, If Vividh’s share in (c) 48 (d) 72
the profit earned by them is ` 9,000, what is the total profit (e) None of these
earned by them together? 32. How many students will pass in Geography if minimum
(a) ` 17,400 (b) ` 16,400 passing marks is 40%?
(c) ` 16,800 (d) ` 17,800 (a) 74 (b) 106
(e) None of these (c) 96 (d) Can’t say
26. Present ages of father and son are in the ratio of 6 : 1 (e) None of these
respectively. Four years after the ratio of their ages will 33. How many students have obtained 60 or more marks in at
become 4 : 1 respectively. What is the son’s present age? least one of the five subjects?
(a) 10 years (b) 6 years
(a) 111 (b) 103
(c) 4 years (d) 8 years
(e) None of these (c) 108 (d) 106
(e) Data inadequate
2
27. 65% of a number is more than its th by 140. What is 30% 34. If the criteria for distinction is minimum 75% marks in Maths,
5 how many students will get distinction?
of that number? (a) 37 (b) 27
(a) 186 (b) 168 (c) 12 (d) Can’t say
(c) 164 (d) 182
(e) None of these
(e) None of these
y
o
u
rs
m
SPEED TEST 97

a
207

h
b
35. The no. of students who obtained more than or equal to 43. Four of the following five are alike in a certain way and so

o
o
40% marks in Science is what per cent less than that of form a group. Which is the one that does not belong to that

b
those who scored less than 60% in Hindi? group ?

.w
(a) 17.60% (b) 15.40% (a) Stem (b) Tree

o
rd
(c) 19.80% (d) 24.30% (c) Root (d) Branch

p
(e) None of these (e) Leaf

re
44. If ‘Apple’ is called ‘Orange’, ‘Orange’ is called ‘Peach’,

s
s
Part-II : Reasoning Ability

.c
‘Peach’ is called ‘Patato’, ‘Potato’ is called ‘Banana’,

o
36. How many meaningful three letter English words can be ‘Banana’ is called ‘Papaya’ and ‘Papaya’ is called ‘Guava’,

m
formed with the letters AER, using each letter only once in which of the following grows underground ?
each word ? (a) Potato (b) Guava
(a) None (b) One (c) Apple (d) Banana
(c) three (d) two (e) None of these
(e) Four 45. How many such pairs of letters are there in word ENGLISH,
37. Each vowel of the word ADJECTIVE is substituted with the each of which has as many letters between its two letters as
next letter of the English alphabetical series, and each there are between them in the English alphabets ?
consonant is substituted with the letter preceding it. How (a) None (b) One
many vowels are present in the new arrangement ? (c) Two (d) Three
(a) Four (b) One (e) More than three
(c) Two (d) Three
DIRECTIONS (Qs. 46-50) : In each of the questions below are
(e) None of these
given three statements followed by two conclusions numbered I
38. In a certain code ‘na pa ka so’ means ‘birds fly very high’,
and II. You have to take the given statements to be true even if
‘ri so la pa’ means ‘birds are very beautiful’ and ‘ti me ka
they seem to be at variance from commonly known facts. Read
bo’ means ‘the parrots could fly’. Which of the following is
both of the conclusions and then decide which of the given
the code for ‘high’in that language ?
conclusions logically follows from the given statements
(a) na (b) k a
disregarding commonly known facts.
(c) bo (d) so
(e) None of these Read the statements and the conclusions which follow it and give
39. If the digits in the number 86435192 are arranged in answer
ascending order, what will be the difference between the (a) If only conclusion I is true.
digits which are second from the right and fourth from the (b) If only conclusion II is true.
left in the new arrangement ? (c) If either conclusion I or conclusion II is true.
(a) One (b) Two (d) If neither conclusion I nor conclusion II is true.
(c) Three (d) Four (e) If both conclusions I and II are true.
(e) None 46. Statements :
40. If it is possible to make only one meaningful word with the All stars are suns.
Third, Seventh, Eighth and Tenth letters of the word Some suns are planets.
COMPATIBILITY, which of the following would be the last All planets are satellites.
letter of that word ? If no such word can be made, give ‘X’ Conclusions :
as your answer and if more than one such word can be I. Some satellites are stars.
formed, give your answer as ‘Y’. II. No star is a satellite.
(a) I (b) B 47. Statements :
(c) L (d) X All curtains are rods.
(e) Y Some rods are sheets.
41. In a certain code FINE is written HGPC. How is SLIT written Some sheets are pillows.
in that code ? Conclusions:
(a) UTGR (b) UTKR I. Some pillows are rods.
(c) TUGR (d) RUGT II. Some rods are curtains.
(e) None of these 48. Statements :
42. If in a certain language LATE is coded as 8 & 4 $ and HIRE All switches are plugs.
is coded as 7*3$ then how will HAIL be coded in the same Some plugs are bulbs.
language ? All bulbs are sockets.
(a) 7 & 8* (b) &7*8 Conclusions:
(c) 7*& 8 (d) 7&*8 I. Some sockets are plugs.
(e) None of these II. Some plugs are switches.
y
o
u
rs
m
208 SPEED TEST 97

a
h
b
49. Statements : (iii) If no vowel is present in the group of letters, the second and

o
o
All fishes are birds. the fifth letters are to be coded as ©.

b
All birds are rats. 56. BARNIS

.w
All rats are cows. (a) 9 2 * % # 4 (b) 9 2 4 # * %

o
rd
Conclusions : (c) 9 2 * # % 9 (d) 4 2 * # % 4

p
I. All birds are cows. (e) None of these

re
II. All rats are fishes. 57. DMBNIA

s
s
50. Statements :

.c
(a) 6 @ 9 % # 2 (b) 2 @ 9 % # 6
Some walls are windows.

o
(c) 2 @ 9 % # 6 (d) 2 @ 9 % # 2

m
Some windows are doors. (e) None of these
All doors are roofs.
58. IJBRLG
Conclusions :
(a) # 8 9 * £ $ (b) # 8 9 * £ #
I. Some doors are walls.
(c) $ 8 9 * £ # (d) $ 8 9 * £ $
II. No roof is a window.
(e) None of these
59. BKGQJN
DIRECTIONS (Qs. 51-55) : Read the following information
(a) 9 © $ 7© % (b) © 9 $ 7 % ©
carefully and answer the questions, which follow :
(c) 9 1 $ 7 8 % (d) % 1 $ 7 8 9
‘A - B’ means ‘A is father of B’. (e) None of these
‘A + B’ means ‘A is daughter of B’. 60. EGAKRL
‘A ÷ B’ means ‘A is son of B’. (a) # £ $ 2 1 * (b) £ $ 2 1 * 3
‘A × B’ means ‘A is wife of B’. (c) £ $ 2 1 * # (d) # £ $ 2 1 #
51. How is P related to T in the expression ‘P + S – T’ ? (e) None of these
(a) Sister (b) Wife
(c) Son (d) Daughter DIRECTIONS (Qs. 61-66) : Study the following information
(e) None of these carefully to answer these questions.
52. In the expression ‘P × Q – T’ how is T related to P ? Eight persons A, B, C, D, E, F, G and H work for three different
(a) Daughter (b) Sister companies namely X, Y and Z. Not more than three persons work
(c) Mother (d) Can’t be determined for a company. There are only two ladies in the group who have
(e) None of these different specialisations and work for different companies. Of the
53. Which of the following means T is wife of P ? group of friends, two have specialisation in each HR, Finance and
(a) P × S ÷ T (b) P ÷ S × T Marketing. One member is an engineer and one is a doctor. H is
(c) P – S ÷ T (d) P + T ÷ S an HR specialist and works with a Marketing specialist B who
(e) None of these does not work for company Y. C is an engineer and his sister
54. Which of the following means P is grandson of S ? works in company Z. D is a specialist in HR working in company
(a) P + Q – S (b) P ÷ Q × S X while her friend G is a finance specialist and works for company
(c) P ÷ Q + S (d) P × Q ÷ S Z. No two persons having the same specialisation work together.
(e) None of these Marketing specialist F work for company Y and his friend A who
55. In the expression ‘P + Q × T’ how is T related to P ? is a Finance expert works for company X in which only two
(a) Mother (b) Father specialists work. No lady is a marketing specialist or a doctor.
(c) Son (d) Brother 61. Which of the following combinations is correct ?
(e) None of these (a) C - Z - Engineer (b) E - X - Doctor
DIRECTIONS (Qs. 56-60) : In each question a group of letters is (c) H – X – HR (d) C – Y – Engineer
given followed by four combinations of number/symbol numbered (e) None of these
(a), (b), (c) and (d). Letters are to be coded as per the scheme and 62. For which of the following companies does C work?
conditions given below. You have to find out the serial number of (a) Y (b) X
the combination, which represents the letter group. Serial (c) Z (d) Data inadequate
number of that combination is your answer. If none of the (e) None of these
combinations is correct, your answer is (e) i.e. None of these. 63. Which of the following pairs represents the two ladies in
Letters Q M S I N G D K A L P R B J E the group ?
Number/ 7 @ 4 # % $ 6 1 2 £ 5 * 9 8 3 (a) A and D (b) B and D
Symbol (c) D and G (d) Data inadequate
(e) None of these
Conditions :
64. Which of the following represents the pair working in the
(i) If the first letter is a consonant and the last a vowel, both are
same company ?
to be coded as the code of the vowel.
(a) D and C (b) A and B
(ii) If the first letter is vowel and the last a consonant, the codes
(c) A and E (d) H and F
for the first and the last are to be interchanged.
(e) None of these
y
o
u
rs
m
SPEED TEST 97

a
209

h
b
65. Who amongst the friends is a doctor ? Part-III : English Language

o
o
(a) H (b) E

b
(c) C (d) Either E or C DIRECTIONS (Qs. 71-80) : Read the following passage carefully

.w
(e) None of these and answer the questions given below it. Certain words/phrases

o
rd
66. Four of the following five are alike in a certain way and so have been printed in bold to help you locate them while answering
form a group. Which is the one that does not belong to the

p
some of the questions.

re
group?
Once upon a time a dishonest king had a man called the

s
(a) 56 (b) 35

s
.c
(c) 49 (d) 42 Valuer in his court. The Valuer set the price which ought to be

o
(e) 51 paid for horses and elephants and the other animals. He also set

m
67. If it is possible to make only one meaningful English word the price on jewellery and gold, and things of that kind. This man
from the second, the fifth, the seventh and the eighth letters was honest and just, and set the proper price to be paid to the
of the word PHYSICAL, using each letter only once, second owners of the goods. The king, however, was not pleased with
letter of that word is your answer. If more than one such this Valuer, because he was honest. "If I had another sort of a
word can be formed your answer is M. If no such word can man as Valuer, I might gain more riches," he thought.
be formed your answer is N. One day the king saw a stupid, miserly peasant come into
(a) I (b) A the palace yard. The king sent for the fellow and asked him if he
(c) L (d) M would like to be the Valuer. The peasant said he would like the
(e) N position. So the king had him made Valuer. He sent the honest
Valuer away from the palace.
DIRECTIONS (Qs. 68-70) : Study the following paragraph and Then the peasant began to set the prices on horses and
answer the questions that follow : elephants, upon gold and jewels. He did not know their value, so
Culture in the literal sense of the term is the art and he would say anything he chose. As the king had made him Valuer,
manifestation of human intellectual achievement regarded the people had to sell their goods for the price he set. By and by
collectively. It refers to the cumulative deposit of knowledge, a horse-dealer brought five hundred horses to the court of this
experience, beliefs, values, attitudes, meanings and material king. The Valuer came and said they were worth a mere measure
objects and possessions acquired by a group of people in the of rice. So the king ordered the horse-dealer to be given the measure
course of generations through individual and group striving. The of rice, and the horses to be put in the palace stables.
culture of India refers to the religions, beliefs, customs, traditions, The horse-dealer then went to see the honest man who had
languages, ceremonies, arts, values and the way of life of the been the Valuer, and told him what had happened. "What shall I
people of India. Heritage on the other hand denotes or relates to do?" asked the horse-dealer. "I think you can give a present to
things of special architectural, historical or natural value that are the Valuer which will make him do and say what you want him to
preserved and sustained as valued objects such as historic do and say," said the man. "Go to him and give him a fine present,
buildings and monuments as valued objects such as historic then say to him: "You said the horses are worth a measure of rice
buildings and monuments but now tell what a measure of rice is worth! Can you value that
68. Which of the following conclusion which can be drawn standing in your place by the king?" If the says he can, go wth
from the facts stated in the above passage? him to the king, and I will be there, too."
(a) Culture is an imaginary term by the people of that The horse-dealer thought this was a good idea. So he gave
country. a fine present to the Valuer, and said what the other man had told
(b) Culture is related with our political activities. him to say., The stupid Valuer took the present, and said: "Yes, I
(c) Culture is an integrated sense of art, knowledge, can go before the king with you and tell what a measure of rice is
customs, beliefs, languages, values life-styles etc. worth. I can value that now." Well, let us go at once," said the
(d) Culture is mostly affected with economic activities and horse-dealer. So they went before the king and his ministers in
thoughts. the palace.
(e) None of these The horse-dealer bowed down before the king, and said: "O
69. Which statement strengthens the conclusion of the King, I have learned that a measure of rice is the value of my five
paragraph? hundred horses. But will the king be pleased to ask the Valuer
(a) Life-style, clothing and fooding is also a part of culture. what is the valuer of the measure of rice". The king, not knowing
(b) Geographical situation does not affect the culture. what had happened, asked, "How now, Valuer, what are five
(c) Scientific knowledge is also included in culture. hundred horses worth?" "A measure of rice, O King!" said he.
(d) Culture does not affect one’s life. "Very good, then! If five hundred horses are worth a measure of
(e) None of these rice, what is the measure of rice worth?" "The measure of rice is
70. Which statement weakens the conclusion of the paragraph? worth your whole city," replied the foolish fellow.
(a) Our heritage is also a part of our culture. The ministers clapped their hands, laughing, and saying,
(b) Methods of worship is an important part of culture. "What a foolish Valuer! How can such a man hold that office? We
(c) Culture is a hypothetical and imaginary term. Which used to think this great city was beyond price, but this man says
can not be determined by some points. it is worth only a measure of rice, "Then the king was ashamed,
(d) Culture has an extensive sense which is determined by and drove out the foolish fellow. "I tried to please the king by
life-style, custums, beliefs, languages, values etc. setting a low price on the horses, and now see what has happened
(e) None of these to me!" said the Valuer, as he ran away from the laughing crowd.
y
o
u
rs
m
210 SPEED TEST 97

a
h
b
71. Who did the king appoint as the new Valuer? 79. Why did the horse-dealer go to meet the old Valuer?

o
o
(a) A minsister (b) A horse merchant (a) As the new Valuer had set a very inappropriate price

b
for his five hundred horses

.w
(c) Himself (d) A stingy peasant
(e) None of these (b) As his five hundred horses were stolen from him by

o
rd
72. Why was the king not happy with the old Valuer? the king

p
(a) As the Valuer was not good at his work (c) As he was a very good friend of the old Valuer

re
(b) As he had dishonoured the king (d) As the king head requested him to do so

s
s
(e) None of these

.c
(c) As the Valuer had been dishonest with the king about
80. What advice did the old Valuer give to the horse-dealer?

o
the prices that he set for goods

m
(d) As the king believed that he was not earning much (a) He asked the horse-dealer to inquire with the king about
because of the Valuer's honesty the worth of a measure of rice
(e) None of these (b) He asked the horse-dealer to bribe the new Valuer and
73. Which of the following words can be used to describe the get his horses back
king? (c) He asked the horse-dealer to forget about his horses
(1) Smart (2) Dishonest and go on with his life
(3) Cheat (d) He asked the horse-dealer to publicize his plight and
(a) Only (1) (b) Only (2) thus get his horses back
(c) Only (2) and (3) (d) Only (1) and (3) (e) None of these
(e) All the three (1), (2) and (3) DIRECTIONS (Qs. 81-85) : Rearrange the following six
74. What can possibly be the moral of the story? sentences (A), (B), (C), (D), (E) and (F) in the proper sequence to
(a) Slow and steady wins the race form a meaningful paragraph; then answer the questions given
(b) Change is the only permanent thing in life below them.
(c) An honest answer is the sign of true friendship
(d) Haste makes waste (A) The woodcutter thankfully broke off from work and sat down
(e) No legacy is so rich as honesty to eat the delicious meal that his wife had sent for him.
75. Why did the Ministers laugh at the new Valuer? (B) He was in a good mood that particular morning and soon
(a) As he had sold the king's city at a very low price started singing as he swung his axe at the log of wood in
front of him.
(b) As he had displayed his stupidity by quoting an
(C) After he had eaten his meal and taken rest for a while the
abysmally low price on the king's city
woodcutter got back to work.
(c) As he had cheated the horse dealer
(D) The hours passed and the sun became hotter than ever and
(d) As he had not claculated the price of the five hundred very soon perspiraton started breaking out on the
horses correctly woodcuter's hands and face.
(e) None of these (E) One hot summer's morning a woodcutter was hard at work,
76. What did the new Valuer do when he got the present from chopping wood into small pieces, so that he could sell them
the horse dealer? in the market.
(a) He accepted the present and resigned from his post as (F) As it neared afternoon, his wife sent their little son to him
was requested by the horse-dealer with food for the afternoon.
(b) He accepted the present and agreed to state the worth
of a measure of rice in the presence of the King 81. Which of the following should be the FOURTH sentence
after rearrangement?
(c) He accepted the present and immediately returned the
(a) F (b) E
horse-dealer's horses
(c) D (d) C
(d) He refused to accept the present from the horse-dealer
(e) B
and asked him to leave the premises
82. Which of the following should be the FIRSTsentence after
(e) None of these rearrangement?
77. Which of the following can be said about the old Valuer? (a) A (b) B
(1) He was honest. (2) He was intelligent. (c) C (d) D
(3) He was revengeful. (e) E
(a) Only (1) (b) Only (3) 83. Which of the following should be the SECOND sentence
(c) Only (1) and (2) (d) Only (1) and (3) after rearrangement?
(e) All the three (1), (2) and (3) (a) A (b) B
78. What was the worth of a measure of rice according to the (c) C (d) D
new Valuer? (e) F
(a) The king's entire city 84. Which of the following should be the LAST (SIXTH)
(b) The king's life sentence after rearrangement?
(c) Two horses (a) A (b) B
(d) Not mentioned in the passage (c) C (d) D
(e) None of these (e) E
y
o
u
rs
m
SPEED TEST 97

a
211

h
b
85. Which of the following should be the THIRD sentence after he was amused to see that the monkeys also did (93) what he did.

o
o
rearrangement? The gardener was happy that he had so much unpaid help.

b
(a) A (b) B

.w
One day the gardener wanted to (94) a fair in the city. He
(c) C (d) D had an idea. He called the chief of the monkeys and said to him, "I

o
rd
(e) E have to go out for the day. Can you and your family water my

p
DIRECTIONS (Qs. 86-88) : Which of the phrases (a), (b), (c) and plants like you (95) do? I promise you that if you help me I will

re
(d) given below each sentence should replace the phrase printed (96) sweets for you from the fair.: The monkeys agreed. But after

s
s
the gardener had left, they had a (97). How much water were they

.c
in bold in the sentence to make it grammatically correct? If the

o
sentence is correct as it is given and no correction is required, to pour for each plant? then one of them said. "For plants with big

m
mark (e) as the answer. roots, we must pour (98) of water and for the ones with small
roots, we pour only a litle water." So, the monkeys (99) out each
86. As it was already afternoon, Rohan decided to check out of
plant and then pushed it back again after looking at the root. As
the hotel and go home.
a result, many plants (100) and died. On his return, the gardener
(a) for checking out (b) to checking out
realized that he had been very foolish to trust a bunch of mere
(c) to check outing (d) to checked out
monkeys to do his job.
(e) No correction required
87. Five people which ignored an evacuation order were trapped 91. (a) took (b) was
in a mountain region encircled by a wildfire. (c) great (d) handle
(a) who ignored an (b) those ignoring an (e) mended
(c) who ignores a (d) that ignored a 92. (a) try (b) told
(e) No correction required
88. Since she was the most popular model on the ramp, she (c) were (d) bent
thought no end to herself. (e) learnt
(a) no ending to herself (b) no ends of herself 93. (a) main (b) exactly
(c) no end of herself (d) no end with herself (c) many (d) because
(e) No correction required
(e) too
DIRECTIONS (Qs. 89-90) : In each question below, a sentence 94. (a) call (b) make
with four words printed in bold type is given. These are numbered
as (a), (b), (c) and (d). One of these four words printed in bold may (c) stall (d) go
be either wrongly spelt or inappropriate in the context of the (e) visit
sentence. Find out the word which is wrongly spelt or 95. (a) forcefully (b) hardly
inappropriate, if any. The number of that word is your answer. If (c) usually (d) costly
all the words printed in bold are correctly spelt and also
(e) truly
appropriate in the context of the sentence, mark (e) i.e., All
correct' as your answer. 96. (a) ask (b) bring
89. The city's fashion-conscious ladies (a)/ came together at a (c) got (d) throw
city hotel to check out an exibition (b)/ by various (c)/ (e) create
designers (d)/ and labels. All correct (e) 97. (a) party (b) time
90. The ministry's proposal (a)/ for an autonomous (b)/ (c) answer (d) doubt
overarching authority (c)/ for higher education and research (e) water
was finally approval. (d)/ All correct (e)
98. (a) body (b) many
DIRECTIONS (Qs. 91-100) : In the following passage there are (c) lots (d) weight
blanks, each of which has been numbered. These numbers are
(e) quantity
printed below the passage and against each, five words are
suggested, one of which fits the blank appropriately. Find out the 99. (a) thrashed (b) saw
appropriate word in each case. (c) stick (d) pulled
There was once a gardener who (91) care of the king's (e) splashed
garden. In the garden, lived a family of monkeys. Seeing the 100. (a) withered (b) crushed
gardener at work, the monkeys soon (92) to imitate him. As the (c) killed (d) grew
gardener tended the plants and weeded and watered the garden, (e) smiled
y
o
u
rs
m
212 SPEED TEST 97

a
h
b
o
RESPONSE SHEET

o
b
.w
o
rd
p
Test Code : ............................... Time taken : ......................... Date : ..........................

re
s
(a) (b) (c) (d) (e) (a) (b) (c) (d) (e) (a) (b) (c) (d) (e) (a) (b) (c) (d) (e)

s
.c
o
1. 26. 51. 76.

m
2. 27. 52. 77.
3. 28. 53. 78.
4. 29. 54. 79.
5. 30. 55. 80.
6. 31. 56. 81.
7. 32. 57. 82.
8. 33. 58. 83.
9. 34. 59. 84.
10. 35. 60. 85.
11. 36. 61. 86.
12. 37. 62. 87.
13. 38. 63. 88.
14. 39. 64. 89.
15. 40. 65. 90.
16. 41. 66. 91.
17. 42. 67. 92.
18. 43. 68. 93.
19. 44. 69. 94.
20. 45. 70. 95.
21. 46. 71. 96.
22. 47. 72. 97.
23. 48. 73. 98.
24. 49. 74. 99.
25. 50. 75. 100.
98

y
o
u
rs
m
a
h
b
98

o
o
b
.w
Prelim Test - 2

o
rd
p
re
s
s
.c
o
m
Max. Marks : 100 No. of Qs. 100 Time : 1 hr. Date : ........./......../................

Part-I : Numerical Ability 7 3 1


9. 3 7 1 ?
11 11 2
DIRECTIONS (Qs. 1-10): What will come in place of question
10 6
mark (?) in the following questions? (a) 13 (b) 14
11 11
3 4 5 9 17
1. of of of 1015 = ? (c) 14 (d) 10
5 7 12 11 22
(a) 220 (b) 340 (e) None of these
(c) 240 (d) 145 10. 1080 12 10 = ?
(e) None of these (a) 900 (b) 90
2. 1.5 × 0.025 + (?)2 = 0.1 (c) 120 (d) 12
(a) 0.28 (b) 0.27 (e) None of these
(c) 0.25 (d) 0.235 11. The number zero (0) is surrounded by the same 2-digit
(e) None of these number on both (left and right) the sides; for example, 25025,
67067, etc. The largest number that always divides such a
3. 1.52 0.0225 ? number is
(a) 0.3375 (b) 3.275 (a) 7 (b) 11
(c) 32.75 (d) 0.0375 (c) 13 (d) 1001
(e) None of these (e) None of these
4. 12. The number of 3-digit number exactly divisible by 5 is
0.0289 12 1.5 ?
(a) 181 (b) 180
(a) 1.36 (b) 2.06
(c) 179 (d) 199
(c) 13.90 (d) 14.80
(e) None of these
(e) None of these
13. If a certain sum of money becomes double at simple interest
5. 125% of 260 + ?% of 700 = 500
in 12 years, what would be the rate of interest per annum ?
(a) 32 (b) 56
(c) 23 (d) 46 (a) 8 1 (b) 10
(e) None of these 3
6. 45% of 750 – 25% of 480 = ? (c) 12 (d) 14
(a) 216 (b) 217.50 (e) None of these
(c) 245 (d) 236.50 14. Three successive discounts of 10%, 12% and 15% amount
(e) None of these to a single discount of
(a) 36.28 % (b) 34.68%
7. 758.5 753.8 75 ? (c) 37 % (d) 32.68%
(a) 4.9 (b) 3.6 (e) None of these
(c) 3.3 (d) 4.7 15. The ratio of the prices of two houses A and B was 4 : 5 last
(e) None of these year. This year, the price of A is increased by 25% and that
of B by ` 50000. If their prices are now in the ratio 9 : 10, the
8. 39798 + 3798 + 378 = ?
price of A last year was
(a) 49532 (b) 43984 (a) ` 3,60,000 (b) ` 4,50,000
(c) 43974 (d) 43576
(c) ` 4,80,000 (d) ` 5,00,000
(e) None of these (e) None of these
y
o
u
rs
m
214 SPEED TEST 98

a
h
b
16. During a journey of 80 km a train covers first 60km with a 23. 18.4% of 656 + 12.7% of 864 = ?

o
o
(a) 253 (b) 231 (c) 211

b
speed of 40 km/h and completes the remaining distance with

.w
a speed of 20 km/h. What is the average speed of the train (d) 241 (e) None of these

o
24. (98.4)2 + (33.6)2 = ?

rd
during the whole journey?

p
(a) 10812 (b) 18012 (c) 10910

re
(a) 30 km/h (b) 32 km/h
(d) 18102 (e) None of these

s
s
(c) 36 km/h (d) 40 km/h

.c
25. 8959 ? 4 5 = 26.35

o
(e) None of these

m
(a) 15 (b) 25 (c) 30
17. An aeroplane takes off 30 minutes later than the scheduled (d) 17 (e) None of these
time and in order to reach its destination 1500 km away in 26. 3739 + 164 × 27 = ?
time, it has to increase its speed by 250 km/h from its usual (a) 10540 (b) 4000 (c) 8400
(d) 8200 (e) None of these
speed. Find its usual speed.
27. Rajeev consistently runs 415 meters every day except on
(a) 1000 km/h (b) 750 km/h
Sunday when he runs 500 meters. How many kilometers will
(c) 850 km/h (d) 650 km/h
he run in two weeks-? (in this question week starts from
(e) None of these
Monday)
18. A man arranges to pay off a debt of Rs 3,600 in 40 annual (a) 5.98 kms. (b) 5.86 kms. (c) 5.96 kms.
instalments which form an AP. When 30 of the instalments (d) 5.88 knis. (e) None of these
are paid, he dies leaving one-third of the debt unpaid. Find 28. Amit got 44 marks in Hindi. 55 marks in Science. 77 marks in
the value of the first instalment. Maths. 79 marks in Social Science and 76 marks in English.
(a) 55 (b) 53 The maximum marks of each subject are 100. How much
(c) 51 (d) 49 overall percentage of marks did he get?
(e) None of these (a) 66.2 (b) 64.2 (c) 72.2
19. A tank 30 m long, 20 m wide and 12 m deep is dug in a field (d) 74.2 (e) None of these

500 m long and 30 m wide. By how much will the level of the DIRECTIONS (Qs. 29 - 30) : What will come in place of question
field rise, if the earth dug out of the tank is evenly spread mark (?) in the following number series?
over the field? 29. 7 11 19 35 67 (?)
(a) 0.33 m (b) 0.5 m (a) 121 (b) 153 (c) 141
(c) 0.25 m (d) 0.4 m (d) 133 (e) None of these
(e) None of these 30. 5 6 10 19 35 (?)
DIRECTIONS (Qs. 20-21) : Find the next term in the given series (a) 55 (b) 65 (c) 60
in each of the questions below. (d) 70 (e) None of these
20. 198, 194, 185, 169, .... Directions (Qs. 31-35) : Study the following data carefully and
(a) 136 (b) 144 answer accordingly.
(c) 9 (d) 92
Following chart shows the number of students in
(e) None of these
different universities
21. 6, 9, 7, 10, 8, 11, ....
(a) 12 (b) 13 Delhi Uni
Indraprastha 35%
(c) 9 (d) 14
Uni 13%
(e) None of these
DIRECTIONS (Qs. 22-28): Find out the approximate value which
should replace the question mark (?) in the following questions. Hamdard
14%
(You are not expected to find out the exact value).
J.N.U
22. 10609 7938.81 ? 20%
Jamia
(a) 9200 (b) 81973. 18%
(c) 8553.3 (d) 8682.7
(e) None of these Total no. of students = 120,000
y
o
u
rs
m
SPEED TEST 98

a
215

h
b
Percentage of listeners of different FM channels in 38. If each of the digits in the number 92581473 are arranged in

o
o
National Capital Region ascending order, what will be the difference between the

b
digits which are fourth from the right and third from the left

.w
FM Chann els in the new arrangement ?

o
rd
Un ivers ities Rad io Radio Red FM (a) One (b) Two
Rain bo w

p
M irch i City FM Go ld (c) Three (d) Four

re
Ind rap ras tha 76% 72% 46% 54% 48% (e) None

s
s
39. In a certain code ‘ja ki mo pe’ means ‘at a frog’s leap’, ‘mo la

.c
Hamd ard 63% 64% 59% 47% 53%
ki so’ means ‘take a leap ahead’ and ‘re bo ja na’ means

o
m
JNU 52% 65% 64% 51% 54% ‘insects are frog’s diet’. Which of the following is the code
DU 82% 44% 32% 35% 45% for ‘at’ in that language ?
Jamia 75% 32% 36% 52% 64% (a) ja (b) pe
(c) bo (d) re
31. How many students of JNU listen to Radio city? (e) None of these
(a) 15200 (b) 15600 40. If in a certain language WEAK is coded as 9%2$ and SKIT
(c) 14400 (d) 14600 is coded as #$7@, then how will WAIT be coded in the
same language ?
(e) None of these
(a) 9267 (b) 9276
32. The no. of Indraprastha students listening to Rainbow is
(c) 92 @ 6 (d) 9 @ 67
what per cent of the no. of Jamia students listening FM (e) None of these
Gold? 41. How many such pairs of letters are there in word SENDING,
(a) 65 (b) 56 each of which has as many letters between its two letters as
(c) 68 (d) 58 there are between them in the English alphabets ?
(e) None of these (a) None (b) One
33. From which of the following universities, the no. of students (c) Two (d) Three
liking Red FM is minimum? (e) More than three
(a) Indraprastha (b) Jamia 42. Each vowel of the word GLADIOLUS is substituted with
(c) JNU (d) DU the next letter of the English alphabetical series, and each
(e) Hamdard consonant is substituted with the letter preceding it. How
34. How many students of Indraprastha and Jamia together many vowels are present in the new arrangement ?
(a) Four (b) One
listen to Red FM?
(c) Two (d) Three
(a) 12562 (b) 12872
(e) None of these
(c) 14952 (d) 14272
(e) None of these DIRECTIONS (Qs. 43-47) : In each of these questions a group of
35. Which of the following channels is the most popular among letters is given followed by four combinations of number/symbol
the students of Hamdard and JNU? numbered (a), (b), (c) and (d). Letters are to be coded as per the
(a) Radio Mirchi (b) Radio city scheme and conditions given below. You have to find out the serial
(c) Red FM (d) FM Gold number of the combination, which represents the letter group.
(e) Rainbow Serial number of the combination is your answer. If none of the
combinations is correct, your answer is (e) i.e. ‘None of these’.
Part-II : Reasoning Ability
Letters M B D K Q L I R J S N P A E G
36. If ‘football’ is called cricket, ‘cricket’ is called ‘basketball’
Number/
‘basketball’ is called ‘badminton’, ‘badminton’ is called @ 3 7 % * 4 # 1 2 £ 8 5 9 $ 6
Symbol
‘volleyball’, ‘volleyball’ is called ‘hockey’ and ‘hockey’ is
called ‘golf’, which of the following games is not played Conditions :
using a ball ? (i) If the first letter is a vowel and the last a consonant, both are
(a) Volleyball (b) Basketball to be coded as the code for the consonant.
(ii) If the first letter is a consonant and the last a vowel, the
(c) Hockey (d) Cricket
codes for the first and the last are to be interchanged.
(e) None of these
(iii) If no vowel is present in the group of letters, the first and
37. If it is possible to make only one meaningful word with the
the last letters are to be coded as ©.
First, Second, Third and Fifth letters of the word 43. GQRDBN
TECHNOLOGY, which of the following would be the third (a) ©* 173© (b) 6*1738
letter of that word ? If no such word can be made, give ‘X’ as (c) 6*1736 (d) 8*1738
your answer and if more than one such word can be formed, (e) None of these
give your answer as ‘Y’. 44. IPEBQS
(a) C (b) T (a) #5$3*£ (b) #53$*#
(c) N (d) X (c) £5$3*£ (d) £5$3*#
(e) Y (e) None of these
y
o
u
rs
m
216 SPEED TEST 98

a
h
b
45. RMAPSI 55. If in each number, all the three digits are arranged in

o
o
(a) 1@95£# (b) 1@95£1 descending order, which of the following will be the third

b
(c) #@95£# (d) #@95£1

.w
highest number?
(e) None of these (a) 972 (b) 682

o
rd
46. AREMQN (c) 189 (d) 298

p
(a) 91$@*8 (b) 81$@*8 (e) 751

re
(c) 81$@*9 (d) 91$@*9

s
s
DIRECTIONS (Qs. 56-60) : In each of the questions below are

.c
(e) None of these
given three statements followed by two conclusions numbered I

o
47. KJBPRD

m
(a) 923517 (b) 723517 and II. You have to take the given statements to be true even if
(c) %23519 (d) ©2915© they seem to be at variance from commonly known facts. Read
(e) None of these both of the conclusions and then decide which of the given
conclusions logically follows from the given statements
DIRECTIONS (Qs. 48-52) : Read the following information
disregarding commonly known facts.
carefully and answer the questions, which follow :
‘A – B’ means ‘A is daughter of B,. Read the statements and conclusions which follow it and
‘A + B’ means ‘A is wife of B’ Give answer (a) if only conclusion I is true.
‘A ÷ B’ means ‘A is father of B’ Give answer (b) if only conclusion II is true.
‘A × B’ means ‘A is son of B’. Give answer (c) if either conclusion I or conclusion II is true.
48. In the expression ‘P × R – S’ how is P related to S ? Give answer (d) if neither conclusion I nor conclusion II is true.
(a) Father (b) Grandfather Give answer (e) if both conclusions I and II are true.
(c) Grandson (d) Sister 56. Statements :
(e) None of these No pen is a mobile.
49. Which of the following means S is son-in-law of P? Some mobiles are bottles.
(a) P + R × S (b) P ÷ R × S All bottles are papers.
(c) P + R ÷ S (d) P ÷ R + S Conclusions :
(e) None of these I. Some papers are pens.
50. In the expression ‘P – Q + S’ how is S related to P? II. All bottles are pens.
(a) Mother 57. Statements :
(b) Father All computers are radios.
(c) Brother All radios are televisions.
(d) Cannot be determined Some televisions are watches.
(e) None of these Conclusions :
51. How is P related to S in the expression ‘P × R ÷ S’? I. Some watches are computers.
(a) Brother (b) Wife II Some televisions are computers.
(c) Son (d) Sister 58. Statements :
(e) None of these Some desks are chairs.
52. How is S related to P in the expression ‘P + R ÷ S’? Some chairs are doors.
(a) Son (b) Daughter Some doors are walls.
(c) Daughter- in - law (d) Sister Conclusions :
(e) None of these I. Some walls are chairs.
DIRECTIONS (Qs. 53-55) : Study the sets of numbers given II. No chair is a wall.
below and answer the questions, which follow : 59. Statements :
972 682 189 298 751 All stars are fishes.
53. If one is added to the lowest number and two is added to Some fishes are moons.
the highest number, what will be the difference between the All moons are birds.
second digit of the smallest number and third digit of the Conclusions :
highest number ? I. Some birds are fishes.
(a) 5 (b) 7 II. Some stars are moons.
(c) 9 (d) 8 60. Statements :
(e) None of these All leaves are roots.
54. If in each number, first and the last digits are interchanged, All stems are roots.
which of the following will be the third highest number ? All roots are flowers.
(a) 972 (b) 682 Conclusions :
(c) 189 (d) 298 1. Some flowers are stems.
(e) 751 II. Some flowers are leaves.
y
o
u
rs
m
SPEED TEST 98

a
217

h
b
DIRECTIONS (Qs. 61-67) : Study the following information 68. The passage best support the statement that–

o
o
carefully to answer these questions. (a) History has a short and lucid curriculum to study so it

b
attracts the students.

.w
A group of people has six family members and an advocate.
(b) History has an extensive curriculum and its questions

o
These are L, M, N, O, P, Q and R and having different professions.

rd
are time taking.
Each one of them is a journalist, businessman, architect, doctor

p
(c) History is lucid and easy discipline. So, it does not

re
and pilot but not necessarily in this order. There are three males
need a comprehensive strategic to ensure a sound

s
and three females in the family out of which there are two married

s
preparation.

.c
couples. M is a businessman and is the father of P. N is a housewife
(d) History is an important subject in the examination point

o
and is daughter-in-law of O. L is neither a pilot nor a journalist. R

m
of view.
is an advocate. N is not the mother of P and O is not married to M. (e) None of these
No lady is a journalist. 69. Which of the following assumption which can be implicit in
61. Which of the following groups represents the three ladies the facts stated in the above passage?
in the group ? (a) History is an important subject for the competition
(a) N, P, L (b) P, L, N points of view.
(c) L, N, O (d) O, P, L (b) Nature of this subject is static.
(e) None of these (c) A comprehensive strategy is needed to ensure a sound
62. Who is married to Q ? preparation of history.
(a) N (b) O (d) History attracts the student with its easy nature.
(c) L (d) Can’t be determined (e) None of these
(e) None of these 70. Which of the following is a conclusion which can be drawn
63. Who among the following family members is an architect ? from the facts stated in the above passage.
(a) L (b) O (a) History is an easy subject nevertheless students do
(c) P (d) Can’t be determined not like to opt it as an optional paper.
(e) None of these (b) History is a boring and wearisome subject.
64. Which of the following is the profession of P ? (c) There is not enough study material for the sound
(a) Architect (b) Pilot preparation of history.
(c) Architect or pilot (d) Journalist (d) Questions are not being asked from history in the
(e) None of these competitive exams.
65. How is Q related to O ? (e) None of these
(a) Father (b) Mother
(c) Mother-in- law (d) Son - in - law Part-III : English Language
(e) None of these
66. If in the number 5608391467, the position of the first and the DIRECTIONS (Qs. 71-80) : Read the following passage carefully
sixth digits are interchanged, the second and seventh digits and answer the questions given below it. Certain words have
are interchanged and so on upto the fifth and the tenth been printed in bold to help you locate them while answering
digits. Then which will be the fourth digit from the left end some of the questions.
after interchange? Govind’s father was a rich landlord, who was loved and
(a) 1 (b) 6 respected by all his tenants. When he died, he left large tracts of
(c) 8 (d) 3 land to Govind. But Govind did not spend a single day looking
(e) None of these after his land. He had a funny idea, that there existed a magic
67. What will come next in the following series? potion which, if it was poured on any object, would turn it into
ababcabcdabcdeabcdefabcd gold. He spent all his time trying to learn more about this potion.
(a) g (b) f People took advantage of him and cheated him. His wife grew
(c) e (d) a anxious. Given the amount of money Govind was spending, she
(e) None of these was sure that they would soon be paupers.
One day, a widely respected sage who had been to the
DIRECTIONS (Qs. 68-70) : Study the following paragraph and
Himalayas came to their town. Govind asked him about the potion.
answer the questions that follow :
To his surprise the sage answered, “I have learnt how to brew
History as a discipline has always attracted the imagination such a potion. But it is a difficult process.” “Tell me!” insisted
of the students because of its simple, lucid and easy to Govind, hardly able to believe his luck. “You have to collect the
comprehend nature. At the same time it is vast and time consuming dew which settles on the leaves of a banana tree every morning
therefore it becomes imperative to plan a comprehensive strategy during winter. There is a condition, though. The tree should be
to ensure a sound preparation. The nature of the subject is itself planted and watered regularly with your own hands. Store the
static however the questions asked over the last few years compels collected dew in an earthen vessel and when you have five litres,
us to understand the dynamics of the discipline as well as the bring it to me. I will recite a sacred mantra to transform the dew
forces, trends and patterns including institutions that have shaped into the potion. A drop of the potion will be sufficient to change
history at any given point of time. any object into gold.”
y
o
u
rs
m
218 SPEED TEST 98

a
h
b
Govind was worried “Winter is only for a few months in the 75. Why did Govind decide to cultivate a banana crop?

o
(a) The soil of his land was suitable only for cultivating bananas

o
year. It will take me years to collect the dew.” “You can plant as

b
many trees as you want,” replied the sage. Govind went home (b) It was the most highly priced commodity in the region

.w
and after talking to his wife, began clearing the large fields which (c) It could be grown at any time of the year including winter

o
(d) His wife pressurised him to do so

rd
has been lying vacant for years. He planted rows of banana
(e) The ingredient for the magic potion could only be

p
saplings. He tended them with great care. His wife helped him too.

re
obtained from a banana tree
She would take the banana crop to market and get a good price.

s
76. What made Govind angry with the sage?

s
.c
Over the years the plantation grew and finally after six years (a) The sage had conspired with Govind’s wife against

o
Govind had five litres of dew. He went to the sage who smiled, him

m
uttered a mantra and sprinkled a few drops of dew on a copper (b) He had forgotten the magic spell and all Govind’s hard
vessel. To Govind’s dismay, nothing happened. “you have cheated work was in vain
me!” he shouted at the sage. (c) He had lost a good deal of money in cultivating
The sage however smiled. Govind’s wife then came forward bananas
with a box. The sage opened it and revealed stacks of gold coins (d) The sage had made a fool of him in front of other
inside. Turning to Govind he said, “you worked hard on your villagers
land and created a plantation. Your wife sold the produce in the (e) None of these
market. It was your hard work which created this wealth, not magic.
DIRECTIONS (Qs. 77-78) : Choose the word which is most similar
If I had told you this earlier, you would not have listened.” Govind
in meaning to the word printed in bold as used in the passage.
understood the wisdom behind the sage’s words and worked
even harder from that day on. 77. SPEND
71. Why did Govind’s father give him large tracts of land? (a) pay (b) bought
(a) It was his way of instilling a sense of responsibility in (c) devote (d) settle
his son (e) empty
(b) Govind was his only son and sole heir 78. LYING
(a) sleeping (b) dishonest
(c) To provide Govind with sufficient funds to pursue his
interest of discovering a magic potion (c) relaxing (d) remaining
(e) untruthful
(d) He wanted Govind to continue to look after the tenants
(e) None of these DIRECTIONS (Qs. 79-80) : Choose the word which is most
72. Which of the following can be said about the sage? opposite in meaning to the word printed in bold as used in the
passage.
(a) He was cunning and plotted with Govind’s wife to cheat
him. 79. DISMAY
(b) He had no magical powers as such and used to swindle (a) joy (b) interest
people (c) desire (d) humour
(c) He was a good judge of people (e) luck
(d) He did not deserve his good reputation 80. TENDED
(e) He was dishonest because he had cheated Govind out (a) negligible (b) watched
of his gold (c) inclined (d) ignored
73. Why was Govind’s wife worried ? (e) spoil
(a) Govind had no knowledge of farming and could not DIRECTIONS (Qs. 81-85) : Rearrange the following six
cultivate the land he had inherited from his father sentences (A), (B), (C), (D), (E) and (F) in the proper sequence
(b) Govind had not friends because he was obsessed with
to form a meaningful paragraph; then answer the questions given
finding a potion which would turn any thing into gold
below them.
(c) Govind was only interested in studying under different
sages and neglected his family duties A. The hall was filled with children, teachers, students, family
(d) Since Govind had devoted all his time and wealth to members and those who were close to him.
finding a magic potion, they would soon be poor B. Normally such ceremonies are attended by important people
(e) Govind’s experiments to find a magic potion were like industrialists, politicians and VIP’s.
dangerous C. What I saw when I stepped into the hall amazed me.
74. Why did Govind’s wife help him in the fields? D. I went home with the feeling that it was a most unusual oath
A. To support her husband in his endeavour to find a taking ceremony with only those who were ‘important’ to
magic potion.
him present.
B. The sage had advised her to help her husband succeed.
E. When he was elected President, he invited me to the swearing
C. He needed someone to help him collect the dew.
(a) Only (B) (b) Only (A) in ceremony in the Central Hall of Parliament.
(c) Both (A) and (B) (d) All (A), (B) and (C) F. However in this case everyone who attended the ceremony
(e) None of these seemed to know him personally.
y
o
u
rs
m
SPEED TEST 98

a
219

h
b
81. Which of the following should be the FIRST sentence after mistakes. In Bangladesh, conventional banks and credit co-

o
o
rearrangement ? operatives always 93 lump sum repayments. This created 94

b
(a) A (b) B problems because repaying in a lump sum was a mental hurdle for

.w
(c) C (d) D borrowers. They tended to delay repayment and get further into

o
rd
(e) E debt in the 95. In the end they usually 96 totally on the loan,

p
82. Which of the following should be the SECOND sentence which was a loss to the bank. In structuring our own loans, I

re
after rearrangement ? decided to ask for a daily payment, Monitoring repayment was 97

s
s
(a) B (b) C and it filled people with 98 that they could repay their loans.

.c
(c) D (d) E

o
91. (a) firstly (b) freshly

m
(e) F (c) foremost (d) initially
83. Which of the following should be the THIRD sentence after (e) recently
rearrangement ?
92. (a) copied (b) observed
(a) A (b) B
(c) C (d) D (c) learned (d) understood
(e) E (e) improving
84. Which of the following should be the FIFTH sentence after 93. (a) asked (b) insisted
rearrangement ? (c) demanded (d) settled
(a) B (b) C (e) lend
(c) D (d) E 94. (a) severe (b) no
(e) F
(c) additionally (d) variety
85. Which of the following should be the LAST (SIXTH)
sentence after rearrangement ? (e) plenty
(a) A (b) B 95. (a) time (b) process
(c) C (d) D (c) return (d) event
(e) E (e) action
DIRECTIONS (Qs. 86-90) : Read each sentence to find out 96. (a) neglected (b) abandoned
whether there is any grammatical error or idiomatic error in it. (c) defaulted (d) depended
The error, if any, will be in one part of the sentence. The number (e) disappointed
of that part is the answer. If there is no error, the answer is (e). 97. (a) benefit (b) easier
(Ignore errors of punctuation, if any.) (c) reckless (d) disorganised
86. His proposal had (a) / to be send to (b) / the President of the (e) secure
company (c) / for her approval (d). No error (e).
98. (a) sense (b) confidence
87. Each tuesday evening we visited (a) / the farmers in the
area (b) / and held a meeting (c) / to discuss the problems (c) challenge (d) doubt
they faced (c). No error (e). (e) believe
88. Though our training facilities (a) / are limited only a (b) / few DIRECTIONS (Qs. 99 to 100): In each of the following sentences,
employees have been (c) / selected for training (d). No error an idiomatic expression or a proverb is highlighted. Select the
(e). alternative which best describes its use in the sentence.
89. During the interview (a) / the panel asked me (b) / several
99. Having sold off his factory, he is now a gentleman at large.
technical questions (c) / and I answered all of it (d). No error
(e). (a) Has no serious occupation
90. He decided to work for (a) / an NGO, but most of his (b) / (b) Is living comfortably
classmates opted for high paid (c) / jobs in multinational (c) Is respected by everybody
companies (d). No error (e). (d) Is held in high esteem
DIRECTIONS (Qs. 91-98) : In the following passage there are (e) None of these
blanks each of which has been numbered. These numbers are 100. Though he has lot of money, yet all his plans are built upon
printed below the passage and against each, five words are sand.
suggested, one of which fits the blank appropriately. Find out the (a) established on insecure foundations
approptiate words in each case. (b) based on inexperience
When we 91 started thirty years ago in 1977, we did not (c) resting on cheap material
know anything about how to run a bank for the poor. We therefore (d) resting on immature ideas
looked at how others ran their operations and 92 from their (e) None of these
y
o
u
rs
m
220 SPEED TEST 98

a
h
b
o
RESPONSE SHEET

o
b
.w
o
rd
Test Code : ............................... Time taken : ......................... Date : ..........................

p
re
s
(a) (b) (c) (d) (e) (a) (b) (c) (d) (e) (a) (b) (c) (d) (e) (a) (b) (c) (d) (e)

s
.c
o
1. 26. 51. 76.

m
2. 27. 52. 77.
3. 28. 53. 78.
4. 29. 54. 79.
5. 30. 55. 80.
6. 31. 56. 81.
7. 32. 57. 82.
8. 33. 58. 83.
9. 34. 59. 84.
10. 35. 60. 85.
11. 36. 61. 86.
12. 37. 62. 87.
13. 38. 63. 88.
14. 39. 64. 89.
15. 40. 65. 90.
16. 41. 66. 91.
17. 42. 67. 92.
18. 43. 68. 93.
19. 44. 69. 94.
20. 45. 70. 95.
21. 46. 71. 96.
22. 47. 72. 97.
23. 48. 73. 98.
24. 49. 74. 99.
25. 50. 75. 100.
y
o
u
rs
m
a
h
b
99

o
o
b
.w
Prelim Test - 3

o
rd
p
re
s
s
.c
o
m
Max. Marks : 100 No. of Qs. 100 Time : 1 hr. Date : ........./......../................

Part-I : Numerical Ability 8. 3889 + 12.952 – ? = 3854.002


(a) 47.95 (b) 47.752
DIRECTIONS (Q. 1-10) : What should come in place of the (c) 47.095 (d) 47.932
question mark (?) in the following questions? (e) None of these
1. ? 75 9. ? + 72.64 = 74.64
(a) –5625 (b) 75 (a) 145.28 (b) –2.00
(c) 1500 (d) Cannot be determined (c) –145.28 (d) 147.28
(e) None of these
(e) None of these
10. 6.25 0.0025 = ?
21 7 1 (a) 1800 (b) 2300
2. ?
8 72 171 (c) 1700 (d) 2500
(e) None of these
9 1 11. Which is the smallest of the following numbers ?
(a) (b)
19 3
1
(a) 7 (b)
5 3 7
(c) (d)
19 19
(e) None of these 7 1
(c) (d)
7 7
1 2 1
3. 4 6 5 ? (e) None of these
2 3 3 12. Two equal sums were borrowed at 8% simple interest per
2 annum for 2 years and 3 years, respectively. The difference
1
(a) 15 (b) 16 in the interests was `56. The difference in the interests was
2 3
`56. The sum borrowed were
1 (a) `690 (b) `700
(c) 16 (d) 17 (c) `740 (d) `780
2
(e) None of these
(e) None of these
13. A machine is sold at a profit of 10%. Had it been sold for ` 80
4. 792.02 + 101.32 - 306.76 = ?
less, there would have been a loss of 10%. The cost price of
(a) 893.34 (b) 1200.10 the machine is
(c) 997.11 (d) 586.58 (a) `350 (b) `400
(e) None of these (c) `450 (d) `520
5. 300% of 150 = ?% of 600 (e) None of these
(a) 75 (b) 45 14. A jar of oil was four fifths full. When six bottles of oil were
1 taken out and four bottles of oil were poured into, it was
(c) 450 (d) 133 three fourths full. How many bottles of oil were contained
2
by the jar ?
(e) None of these (a) 10 (b) 20
6. 34.95 + 240.016 + 23.9800 = ? (c) 30 (d) 40
(a) 299.09 (b) 298.0946 (e) None of these
(c) 298.111 (d) 298.946 15. During a journey of 80 km a train covers first 60km with a
(e) None of these speed of 40 km/h and completes the remaining distance with
7. 48.95 – 32.006 = ? a speed of 20 km/h. What is the average speed of the train
(a) 16.089 (b) 16.944 during the whole journey?
(c) 16.35 (d) 16.89 (a) 30 km/h (b) 32 km/h
(e) None of these (c) 36 km/h (d) 40 km/h
(e) None of these
y
o
u
rs
m
222 SPEED TEST 99

a
h
b
16. An aeroplane takes off 30 minutes later than the scheduled 3

o
25. 4096 ?

o
time and in order to reach its destination 1500 km away in

b
(a) 16 (b) 26
time, it has to increase its speed by 250 km/h from its usual

.w
(c) 18 (d) 24
speed. Find its usual speed.

o
(e) None of these

rd
(a) 1000 km/h (b) 750 km/h

p
(c) 850 km/h (d) 650 km/h 26. 6 men can complete a piece of work in 20 days. In how many

re
days will 8 men complete the same piece of work?
(e) None of these

s
s
(a) 12 days (b) 14 days

.c
17. In an examination 35% of the candidates failed in one subject
(c) 15 days (d) 16 days

o
and 42% failed in another subject. While 15% failed in both

m
the subjects. If 2500 candidates appeared at the examination, (e) None of these
how many students passed in either subject but not in both? 27. What will come in place of both the question marks (?) in
the following equation?
(a) 325 (b) 1175
(c) 2125 (d) 1230 363 (?)
(e) None of these (?) 3
18. If the length of a certain rectangle is decreased by 4 cm and (a) 43 (b) 33
the width is increased by 3 cm, a square with the same area (c) 37 (d) 47
as the original rectangle would result. The perimeter of the (e) None of these
original rectangle (in centimetres) is : 28. Raju decided to marry 3 years after he gets a job. He was 17
(a) 44 (b) 46 years old when he passed class 12th. After passing class
(c) 48 (d) 50 12th', he had completed his graduation course in 3 years
(e) None of these and PG Course in 2 years. He got the job exactly 1 year after
completing his PG Course. At what age will he get married?
DIRECTION (Qs. 19-21): What will come in place of the question (a) 27 years (b) 26 years
mark (?) in the following number series? (c) 28 years (d) 23 years
(e) None of these
19. 2 9 30 105 ? 2195
(a) 432 (b) 426 DIRECTIONS (Qs. 29-33): Study the following table carefully to
(c) 440 (d) 436 answer the questions that follow:
(e) None of these Number of Students studying in Six Different Colleges over the
20. 3 4 12 45 ? 1005 Years
(a) 152 (b) 198
(c) 144 (d) 192
College
(e) None of these P Q R S T U
21. 1 3 9 31 ? 651 Year
(a) 97 (b) 127 2004 2500 2250 2450 2150 2020 2300
(c) 129 (d) 109 2005 2040 2300 2400 2200 2090 2120
(e) None of these 2006 2100 2150 2330 2250 2180 2260
DIRECTIONS (Qs. 22-23): Find out the approximate value which 2007 2280 2600 2260 2340 2250 2490
should replace the question mark (?) in the following questions. 2008 2540 2240 2120 2380 2310 2520
(You are not expected to find out the exact value.) 2009 2320 2440 2500 2480 2400 2440
22. 953.7 950.9989 95?
29. What is the total number of students from all the colleges
(a) 1.9 (b) 3 together in the year 2005?
(c) 2.99 (d) 3.6
(a) 10350 (b) 13150
(e) 2.7 (c) 15310 (d) 11350
3.001 (e) None of these
23 1000 of 1891.992 = ? 30. What is the per cent increase in the number of students in
4.987
College T in the year 2007 from the previous year? (rounded
(a) 2500 (b) 1230
off to two digits after decimal)
(c) 1640 (d) 1525
(a) 8.33 (b) 5.18
(e) 2130
(c) 6.63 (d) 3.21
24. 12.25 × ? × 21.6 = 3545 .64 (e) None of these
(a) 20 (b) 12 31. The number of students in college P in the year 2008 forms
(c) 15 (d) 13 approximately what per cent of the total number of students
(e) None of these in the college in all the year together.
y
o
u
rs
m
SPEED TEST 99

a
223

h
b
(a) 11 (b) 31 41. Four of the following five are alike in a certain way and so

o
o
(c) 18 (d) 26 form a group. Which is the one that does not belong to that

b
(e) 23 group?

.w
32. What is the ratio of the total number of students in College (a) 169 (b) 441

o
rd
S in the years 2006 and 2009 together to the total number of (c) 361 (d) 529

p
students in College U in the same years? (e) 289

re
(a) 473 : 470 (b) 470 : 473 42. Among P, Q, R, T and W each having different weight, T is

s
s
(c) 371 : 390 (d) 390 : 371

.c
heavier than W and lighter than only P. Q is not the lightest.

o
(e) None of these Who among them is definitely the lightest ?

m
33. What is the average number of students in all the colleges (a) R (b) W
together in the year 2004? (rounded off to the nearest integer) (c) R or W (d) Data inadequate
(a) 2208 (b) 2196 (e) None of these
(c) 2144 (d) 2324
(e) 2278 DIRECTIONS (Q. 43-47) : In each question below are three
34. The angles of a triangle are in the ratio of 5 : 6 : 7. respectively. statements followed by three conclusions numbered I, II and III.
What is the sum of the smallest angle and the largest angle You have to take the three given statements to be true even if they
together? seem to be at variance from commonly known facts and then
(a) 130° (b) 100° decide which of the given conclusions logically follows from the
(c) 110° (d) 140° three given statements disregarding commonly known facts. Then
(e) None of these decide which of the answers (a), (b), (c), (d) and (e) is the correct
35. What least number should be subtracted from 536 to make it answer and indicate it on the answer sheet.
a perfect square? 43. Statements : Some desks are chairs. All chairs are tables.
(a) 7 (b) 5 Some tables are mats.
(c) 23 (d) 18 Conclusions : I. Some mats are desks.
(e) None of these II. Some tables are desks.
III.Some mats are chairs.
Part-II : Reasoning Ability (a) Only I follows (b) Only II follows
36. How many such pairs of letters are there in the word (c) Only III follows (d) II and III follow
CHANNEL each of which has as many letters between them (e) None of the above
in the word as in the English alphabet? 44. Statements : All sweets are fruits. No fruit is pencil. Some
(a) None pencils are glasses.
(b) One Conclusions : I. Some glasses are sweets.
(c) Two II. Some pencils are sweets.
(d) Three III.No glass is sweet.
(e) More than three (a) Only I follows (b) Only II follows
37. How many meaningful English words can be made with the (c) Only III follows (d) either I or III follows
letters ATLE using each letter only once in each word ?
(e) None of the above
(a) None (b) One 45. Statements : Some books are flowers. Some flowers are
(c) Two (d) Three chains. Some chains are hammers.
(e) More than three
Conclusions : I. Some hammers are flowers.
38. In a certain code GROWN is written as 7 @ % 36 and NAME
II. Some chairs are books.
is written as 64 $. How is GEAR· written in that code?
III.Some hammers are books.
(a) 74$@ (b) 7$4@
(a) None follows (b) Only I follows
(c) 7%4@ (d) 7@$4
(e) None of these (c) Only II follows (d) Only III follows
39. How many pairs of digits are there in the number 6315784 (e) II and III follow
each of which has as many digits between them in the number 46. Statements : All roofs are cameras. Some cameras are
as when the digits are rearranged in descending order? photographs.
(a) None (b) One Some photographs are stores.
(c) Two (d) Three Conclusions : I. Some stores are cameras.
(e) More than three II. Some stores are roofs.
40. What should come next in the following letter series? III.Some cameras are roofs.
B D F H J L N A C E G I K M B D F H J LA C E G I K B D F H J (a) Only I follows (b) Only II follows
(a) B (b) L
(c) Only III follows (d) II and III follow
(c) M (d) F
(e) None of the above
(e) None of these
y
o
u
rs
m
224 SPEED TEST 99

a
h
b
47. Statements : Some nails are horses. All horses are tablets. (a) None is true (b) Only I is true

o
o
All tablets are crows. (c) Only II is true (d) Only III is true

b
Conclusions : I. Some crows are nails. (e) II and III are true

.w
II. Some tablets are nails. 54. Statements : F @ T, T % M, M # R

o
rd
III.Some crows are horses. Conclusions :

p
(a) Only I follows (b) I and II follows I. R © T II. F @M

re
III. F©M

s
(c) I and III follow (d) II and III follow

s
(a) Only I is true (b) Only II is true

.c
(e) All I, II and III follow

o
(c) Only III is true (d) either II or III is true

m
DIRECTIONS (Q. 48-52) : Study the following arrangement (e) II and III are true
carefully and answer the questions given below 55. Statements : J H, H @ B, B % N
D5 R@AK© 3 9 BJE F$ MPI4 H1W Conclusions :
6 2 # UQ8 T N I. N H II. N @ J
48. How many such numbers are there in the above arrangement III. J B
each of which is immediately preceded by a symbol and (a) I and II are true (b) II and III are true
immediately followed by a letter? (c) I and III are true (d) All I, II and III are true
(a) None (b) One (e) None of the above
(c) Two (d) Three 56. Statements : B # T, T © K, K % M
(e) More than three Conclusions :
49. Which of the following is the ninth to the right of the twenty I. K # B
second from the right end of the above arrangement ? II. M # T
(a) E (b) I
III. B # M
(c) D (d) N
(e) None of these (a) Only I is true
50. How many such symbols are there in the above arrangement (b) Only II is true
each of which is immediately preceded by a number and (c) Only III is true
immediately followed by a letter? (d) II and III are true
(a) None (b) One (e) None of the above
(c) Two (d) Three 57. Statements : D % F, F K, K @ R
(e) More than three Conclusions :
51. If all the numbers are dropped from the above arrangement,
I. R % F
which of the following will be the eleventh from the left
end ? II. R % D
(a) B (b) H III. R@ D
(c) $ (d) (a) Only I is true (b) Only II is true
(e) None of these (c) Only III is true (d) I and II are true
52. How many such consonants are there in the above (e) None of the above
arrangement each of which is immediately preceded by a
number and immediately followed by another consonant ? DIRECTIONS (Q. 58-62) : Study the following arrangement
(a) None (b) One carefully and answer the questions given below
(c) Two (d) Three
(e) More than three M, D, K, R, T, H, W and A are sitting around a circle facing
at the centre. D is second to the right of M who is fifth to the left
DIRECTIONS (Q. 53-57) : In the following questions, the symbols of T. K is third to the right of R who is second to the right of D. H
#, %, @, © and are used with the following meanings illustrated. is second to the right of W.
‘P % Q’ means ‘P is not greater than Q’. 58. Who is second to the right of A ?
‘P Q’ means ‘P is not smaller than Q’. (a) M (b) D
‘P # Q’ means ‘P is neither equal to nor smaller than Q’. (c) K (d) Data inadequate
‘P © Q’ means ‘P is neither equal to nor greater than Q’. (e) None of the above
‘P @ Q’ means ‘P is neither smaller than nor greater than Q’. 59. Who is third to the left of M ?
In each question, three statements showing relationships have (a) A (b) T
been given, which are followed by three conclusions I, II and III.
(c) H (d) D
Assuming that the given statements are true, find out which
conclusion(s) is/are definitely true. (e) Data inadequate
53. Statements : M © K, K T, T © J 60. Who is fourth to the right of H ?
Conclusions : (a) A (b) T
I. J # K II. T # M (c) R (d) K
III. M # J (e) None of these
y
o
u
rs
m
SPEED TEST 99

a
225

h
b
61. In which of the following combinations is the first person (d) Department of defense production provides the social

o
o
sitting between the second and the third person ? security to its ex-servicemen.

b
(e) None of these

.w
(a) KMW
(b) MWD 66. Four of the following five are alike in a certain way and so

o
rd
(c) RHT form a group. Which is the one that does not belong to that

p
group?

re
(d) TAK
(a) Water (b) Juice

s
(e) None of the above

s
.c
(c) Petrol (d) Sugar
62. If A and W interchange their positions who will be third to

o
(e) None of these

m
the left of R ?
(a) M (b) D 67. How many meaningful English words can be made with the
letters EIND using each letter only once in each word ?
(c) A (d) K
(a) None (b) One
(e) None of these
(c) Two (d) Three
DIRECTIONS (Qs. 63-65) : Study the following paragraph and (e) None of these
answer the questions that follow : 68. Four of the following five are alike in a certain way and so
The Government of India is responsible for ensuring the form a group. Which of the following does not belong to
defense of India and every part thereof. The Supreme Command of that group?
the Armed Forces vests in the President. The responsibility for (a) 343 (b) 64
national defense rests with the Cabinet. This is discharged through (c) 75 (d) 27
the Ministry of Defense, which provides the policy framework and (e) 216
wherewithal to the Armed Forces to discharge their responsibilities
69. How many such digits are there in the number 586972 each
in the context of the defense of the country. Ministry of Defense
of which is as far away from the beginning of the number as
comprises of four Departments viz. Department of Defense (DOD),
when the digits are arranged in discending order.
Department of Defense Production (DDP), Department of Defense
(a) None (b) One
Research & Development (DDR&D) and Department of Ex-
Servicemen Welfare and also Finance Division (c) Two (d) Three
63. Which of the following is a conclusion which can be drawn (e) More than Three
from the facts stated in the given passage? 70. What should come next in the following number series ?
(a) President of India is responsible for the defense of the 98 7654 3218 7654 3217 6543 21
country. (1) 9 (2) 8
(b) The government of India is responsible for defense of (c) 7 (d) 6
the country and ministry of defense performs the duty (e) None of these
with help of its various departments.
(c) People of India are themselves responsible four their
security. Part-III : English Language
(d) Indians are so bold that there is no need of security. DIRECTIONS (71-80) : Read the following passage carefully
(e) None of these and answer the questions given below it. Certain words/phrases
64. Which statement strengthens the conclusion of the have been printed in bold to help you locate them while answering
passages? some of the questions.
(a) All ministries of the government co-operate to each Once upon a time there lived a vicious king, Raja Shankara–
other and prime minister co-ordinates them. short-tempered and temperamental. "God I am" he said to his
(b) There are many falts in our defense system which image as he stared into the mirror everyday, many times a day, He
come-out time to time. was obsessed with himself. He loved no one but himself. He was
(c) Now-a-days the ministry of defense is not working blinded towards the injustice in his kingdom because he had little
properly consequently internal and external security time for his subjects. He wasted most of his time in pouring milk
is in danger. and honey over himself.
(d) People of the country is themselves aware for their Interruption in his possessed life was dealt with stern
security. reprimanding and sometimes on petty issues he would behead his
(e) None of these servants. Provoked by his evil advisor Twishar, he went on with
65. Which statement weakens the conclusion of the passage. his self indulged life, unaware of the plot his very devoted advisor
(a) President of India is the supreme commander of the was planning. A plot to dethrone the king, rule the kingdom with
Armed Forces. his wicked ways only to harness wealth and the reputation of a
(b) Due to lack of co-ordination ministry of defense is king.
unable to determine the security of people. One morning the king went on his usual moring horseback
(c) Ministry of defense is divided in four various rounds but returned with a very sad look on his face. He locked
departments. himself inside his platial room only to unlock it at sundown. Just
y
o
u
rs
m
226 SPEED TEST 99

a
h
b
as the doors creaked open and Raja Shankara emerged from it, his (d) That his subjects were not good enough to deserve

o
o
wife rushed to embrace him. She feared a damaging incident had better treatment than what was already being meted

b
occurred. out to them

.w
The king spoke seldom that day and awoke the next day to (e) None of these

o
rd
make a proclamation to his servants and subjects. The whole 74. What did Raja Shankara's wife think about the Raja's peculiar

p
kingdom feared what was in store for them from their angry king. behaviour that particular day?

re
But to their surprise he said to all gathered, "From now on I will be

s
(a) She was afraid that something really bad had happened

s
a different king. A softer and a patient king."

.c
(b) She was afraid that the Raja would beat her up because

o
True to his words from that day on, the king had truly turned of his unusual mood

m
on a new leaf; he cleaned out the corruption and injustice in a (c) She thought that he was in his usual sour mood
tender manner with punishments aimed to renew the person from (d) She thought that the Raja had received threats to his
within. life from his servants
One fine day his evil advisor gathered courage to ask the (e) None of these
reason for his paradigm shift. And the king answered. When I
75. What was the reason for Raja Shankara's change in
went on horseback that morning a month ago, I noticed a dog
behaviour?
brutally chasing a cat. The cat managed to sneak into a hole only
(a) His advisor's words had made him realize his mistake
after the dog bit her leg, maiming her for life. soon afterwards, the
dog barked at a farmer who picked up a sharp stone and hit it (b) He had felt bad for a poor family on his tour around his
straight in the dog's eye. Bleeding profusely, the dog yelped in kingdom
pain. As the farmer walked on, he slipped on the edge of the road (c) His wife had betrayed him and hence he was upset
and broke his head. (d) He had realized that doing good to people would bring
All this happened in a matter of minutes before me and then good to him
I realized that evil begets evil. I thought about it deeply and was (e) He had realized that evil begets evil.
ready to give up my worldly life for the betterment of my subjects. DIRECTIONS (76-78) : Choose the word/group of words which
I wanted to give up the evil in me as I did not want evil to encounter is most similar in meaning to the word/group of words printed in
me. bold as used in the passage.
Sniggering away the immoral advisor thought what a perfect 76. BEGETS
time it was to dethrone the king, because the Raja had grown kind (a) produces (b) loses
hearted and patient and would not endeavour a combat. Thinking
(c) expects (d) avoids
how he would plan his attack, he stumbled over a step that took
(e) calls
him hurling down the remaining steps, bringing him to a stop with
a crash. He howled in pain only to discover he had broken the 77. STERN
bones in both his legs. (a) hard (b) tall
71. How can Raja Shankara be described before his (c) easy (d) tight
transformation? (e) severe
(1) He was unjust 78. GATHERED
(2) He was preoccupied with himself (a) partied (b) assembled
(3) He was cruel (c) dispersed (d) pooled
(a) Only (1) (b) Only (2) (e) collated
(c) Only (3) (d) Only (1) adn (2) DIRECTIONS (79-80) : Choose the word/group of words which
(e) All the three (1), (2) and (3) is most opposite in meaning to the word/group of words printed
72. Why was the king not happy with the old Valuer? in bold as used in the passage,
(a) As the Valuer was not good at his work 79. PETTY
(b) As he had dishonoured the king (a) tremendous (b) huge
(c) As the Valuer had been dishonest with the king about (c) vast (d) important
the prices that he set for goods (e) frugal
(d) As the king beloeved that he was not earning much 80. BRUTALLY
because of the Valuer's honesty (a) cruelly (b) partly
(e) None of these (c) gently (d) rarely
73. What proclamation did the Raja make to his subjects? (e) harmfully
(a) That he was giving up his throne for the betterment of DIRECTIONS (81-83) : Which of the phrases (a), (b), (c) and (d)
the kingdom given below each sentence should replace the phrase printed in
(b) That his advisor would be the king from then on bold in the sentence to make it grammatically correct? If the
(c) That he would be a better king to them than he had sentece is correct as it is given and no correction is required,
been all this while mark (e) as the answer.
y
o
u
rs
m
SPEED TEST 99

a
227

h
b
81. Her entry to the office party was restrict as an official walking (92) they found an oasis, where they (93) to take a both.

o
o
enquiry had ben constituted against her. The one, who had been slapped, got (94) in the quicksand and

b
started drowing, but the friend saved him. After the friend (95)

.w
(a) was restricting
(b) is restricted from the near drowning, he wrote on a stone, "The friend who had

o
rd
(c) was restricted slapped and saved his best friend asked him, "After I hurt you,

p
you wrote in the sand and (96) you write on a stone, why?" The

re
(d) is restricting
other friend (97), "When someone hurts us, we should write it

s
(e) No correction required

s
down in sand where winds of forgiveness can erase it away. But,

.c
82. Rima was at her wit's end trying to figure out what to buy when someone does something good for us, we must (98) it in

o
m
for her frind's birthday. stone where no wind can ever erase it."
(a) at her witting end 89. (a) crawling (b) speaking
(b) at her wit ends
(c) swimming (d) walking
(c) to her wit's end
(e) dancing
(d) so wit's end
90. (a) journey (b) sand
(e) No correction required
(c) running (d) border
83. Pritesh while away his time in playing games on the computer
(e) hunt
instead of studying.
91. (a) dead (b) captured
(a) whiled away his time
(c) presentable (d) missing
(b) whiled against his time
(e) hurt
(c) whiling away his time
92. (a) as (b) until
(d) while awayed his time
(c) from (d) with
(e) No correction required
(e) through
DIRECTIONS (84-88) : In each question below, a sentence with 93. (a) decided (b) fell
four words printed in bold type is given. These are numbered as (c) made (d) want
(a), (b), (c) and (d). One of these four words printed in bold may be (e) left
either wrongly spelt or inappropriate in the context of the 94. (a) home (b) stuck
sentence. Find out the word which is wrongly spelt or
(c) blended (d) mixed
inappropriate, if any. The number of that word is your answer. If
(e) sitting
all the words printed in bold are correctly spelt and also
appropriate in the context of the sentence, mark (e) i.e., 'All 95. (a) separated (b) leaked
correct' as your answer. (c) died (d) recovered
(e) saved
84. Discussion (a)/ is an exchange of knowledge (b)/ whereas
arguement (c)/ is a depiction (d)/ of ignorance. All correct 96. (a) s o (b) how
(e). (c) when (d) tomorrow
85. He was arrested (a)/ for the crime (b)/ and was charged (c)/ (e) now
with attempt (d)/ to murder. All correct (e) 97. (a) called (b) tell
86. commit (a)/ yourself to lifelong learning (b)/ as the most (c) replied (d) questioned
valuable (c)/ aset (d)/ you will have is your mind. (e) asked
All correct (e) 98. (a) talk (b) push
87. Belive (a)/ that life is worth (b)/ living and your belief will
(c) engrave (d) add
create (c)/ the fact. (d)/ All correct (e)
(e) bury
88. The best educated (1)/ human bing (2)/ is the one who
understands (3)/ most about the life in which (4)/ he is DIRECTIONS (Qs. 99 to 100): In each of the following sentences,
placed. All correct (e). an idiomatic expression or a proverb is highlighted. Select the
alternative which best describes its use in the sentence.
DIRECTIONS (89-98) : In the following passage there are blanks,
99. Mohan always keeps himself to himself.
each of which has been numbered. These numbers are printed
below the passage and against each, five words are suggested, (a) Is too busy (b) Is selfish
one of of which fits the blank appropriately. Find out the (c) Is unsociable (d) does not take sides
appropriate word in each case. (e) None of these
Once upon a time, two friends were (89) through the desert. 100. While the ladies continued their small talk in the drawing
During some point of the (90) they had an argument, and one room, I felt bored.
friend slapped the other one in the face. The one who got slapped (a) whispering (b) backbiting
was (91), but without saying anything, he wrote in the sand, (c) gossip (d) light conversation
"Today my best friend slapped me in the face." They kept on (e) None of these
y
o
u
rs
m
228 SPEED TEST 99

a
h
RESPONSE SHEET

b
o
o
b
.w
o
Test Code : ............................... Time taken : ......................... Date : ..........................

rd
p
re
s
s
(a) (b) (c) (d) (e) (a) (b) (c) (d) (e) (a) (b) (c) (d) (e) (a) (b) (c) (d) (e)

.c
o
1. 26. 51. 76.

m
2. 27. 52. 77.
3. 28. 53. 78.
4. 29. 54. 79.
5. 30. 55. 80.
6. 31. 56. 81.
7. 32. 57. 82.
8. 33. 58. 83.
9. 34. 59. 84.
10. 35. 60. 85.
11. 36. 61. 86.
12. 37. 62. 87.
13. 38. 63. 88.
14. 39. 64. 89.
15. 40. 65. 90.
16. 41. 66. 91.
17. 42. 67. 92.
18. 43. 68. 93.
19. 44. 69. 94.
20. 45. 70. 95.
21. 46. 71. 96.
22. 47. 72. 97.
23. 48. 73. 98.
24. 49. 74. 99.
25. 50. 75. 100.
y
o
u
rs
m
a
h
b
o
o
100

b
.w
Full Main Test - 4

o
rd
p
re
s
s
.c
o
m
Max. Marks : 200 No. of Qs. 190 Time : 2 hr 40 min. Date : ........./......../................

Part-I : General and Financial Awareness (a) Amul-Gujarat


(b) Parag-Uttar Pradesh
1. Currency Swap is an instrument to manage (c) Gopal-Rajasthan
(a) currency risk (d) Verka-Punjab
(b) interest rate risk (e) None of these
(c) currency and interest rate risk 9. Which of the following terms is used in Banking Field?
(d) cash flows in different currencies (a) Interest Rate Swap (b) Input Devices
(e) All of the above (c) Sedimentary (d) Zero Hour
2. Which of the following voting systems has been adopted (e) Privilege Motion
for the election of Lok Sabha and Legislative assembly 10. The term 'Smart Money" refers to __________ .
elections in India? (a) Foreign Currency (b) Internet Banking
(a) Single transferable vote system (c) US Dollars (d) Travelers' cheques
(b) First-past-the-post system (e) Credit Cards
(c) Hare-clark system 11. Which one of the following is not a 'Money Market
(d) Single non transferable vote system Instrument' ?
(e) None of these (a) Treasury Bills (b) Commercial Paper
3. Money Laundering normally involves (c) Certificate of Deposit (d) Equity Shares
(a) placement of funds (b) layering of funds (e) None of these
(c) integration of funds (d) All of (a), (b) and (c) 12. Which one of the following is a retail banking product ?
(e) None of these (a) Home Loans (b) Working capital finance
4. Capital Market Regulator is (c) Corporate term loans (d) Infrastructure financing
(a) RBI (b) IRDA (e) Export Credit
(c) NSE (d) BSE 13. Which of the following is NOT a function of the Reserve
(e) SEBI Bank of India ?
(a) Fiscal Policy Functions
5. Speaker of Lok Sabha is
(b) Exchange Control Functions
(a) appointed by the President of India on the advice of
(c) Issuance, Exchange and destruction of currency notes
the Cheif Justice of the supreme court
(d) Monetary Authority Functions
(b) appointed by the President of India
(e) Supervisory and Control Functions
(c) elected by the members of the Lok Sabha
14. With reference to a cheque which of the following is the
(d) elected by the members of Lok Sabha and Rajya Sabha
“drawee bank” ?
(e) None of these
(a) The bank that collects the cheque
6. Which of the following is the Regulator of the credit rating
(b) The payee's bank
agencies in India?
(c) The endorsee's bank
(a) RBI (b) SBI
(d) The endorser's bank
(c) SIDBI (d) SEBI
(e) The bank upon which the cheque is drawn
(e) None of these
15. What is a Debit Card ?
7. Lot of Banks in India these days are offering M-Banking
(a) It is a card issued by a Rating Agency
Facility to their customers. What is the full form of 'M' in 'M-
(b) It is a card which can be used for withdrawing cash or
Banking'
(a) Money (b) Marginal making payment even in the absence of any balance in
(c) Message (d) Mutual Fund the account
(e) Mobile phone (c) It is a card which can be used for withdrawing cash or
making payment if there is balance in the account
8. Which of the following is mis-matched in relation to leading (d) It is a card which carries prepaid balance
brands of milk and milk products in India? (e) It is a card which can be used for making STD calls
y
o
u
rs
m
230 SPEED TEST 100

a
h
b
16. Bad advances of a Bank are called __________ . 27. The Indian space agency is soon opening a 100-acre Space

o
o
(a) Bad debt (b) Book debt Park in which of the following cities?

b
(c) Non Performing Asset (d) Out of order accounts (a) Bengaluru (b) Nagpur

.w
(e) Overdrawn accounts (c) Mumbai (d) New Delhi

o
rd
17. The "Sunil Gangopadhyay Memorial Award" is given for (e) None of these

p
excellence in ___? 28. The World Day of War Orphans is observed on which of

re
(a) Telugu (b) Bengali the following day?

s
s
(a) 7th January (b) 6th January

.c
(c) Odisi (d) Rajasthani

o
(e) None of these (c) 8th January (d) 9th January

m
18. National Girl Child Day was observed across India by the (e) None of these
Union Government on 29. Which of the following companies has won 26th Lal Bahadur
(a) 24 January (b) 25 January Shastri Hockey tournament?
(c) 27 January (d) 21 January (a) Coal India Limited
(e) None of these (b) Oil and Natural Gas Corporation (ONGC)
19. Who is the author of the book "My Journey: Transforming (c) Indian Oil Corporation Limited
Dreams into actions"? (d) Steel Authority of India Limited
(a) A P J Abdul Kalam (b) Sachin Tendulkar (e) None of these
(c) Pranab Mukherjee (d) Amartya Sen 30. Which of the following cities have made it to a top 30 list of
(e) None of these the world's most powerful, productive and connected cities,
20. Where is the headquarter of United Nations Children's Fund as per the study conducted by international real estate
(UNICEF) located? consultancy JLL?
(a) Chennai and Bangalore
(a) New York (b) Berlin
(b) Kolkata and pune
(c) Italy (d) Washington D.C
(c) Delhi and Mumbai
(e) None of these
(d) Delhi and Bangalore
21. In which of the following states the Ganga Gram Yojana has
(e) None of these
been launched?
31. Which of the following organisations has launched Jeevan
(a) Uttarakhand (b) Bihar
Labh Scheme?
(c) West Bengal (d) Uttar Pradesh (a) New India Insurance Ltd
(e) None of these (b) National Insurance Company Ltd
22. For what purpose, the Stand Up India Scheme has been (c) State Bank of India
launched? (d) Life Insurance Corporation of India
(a) To promote entrepreneurship among SC/ST (e) None of these
(b) To promote entrepreneurship among Rural Youth 32. Who of the following is the author of the book titled 'Jinnah
(c) To promote entrepreneurship among Woman Often Came to Our House?
(d) To promote entrepreneurship among SC/ST and (a) Jaswant Singh (b) Kiran Doshi
Women (c) Nisid Hajari (d) Jairam Ramesh
(e) None of these (e) None of these
23. Who of the following is the author of the novel titled - Go 33. On what date, the World Braille Day is observed?
Set a Watchman? (a) January 4 (b) December 30
(a) Cynthia Lord (b) Bill Clegg (c) November 9 (d) April 11
(c) Marlon James (d) Harper Lee (e) None of these
(e) None of these 34. Which of the following states became the first state in the
24. Which of the following companies publishes Purchasing country to sign MoU for UDAY scheme?
Managers' Index (PMI)? (a) Rajasthan (b) Gujarat
(a) National Stock Exchange of India Limited (c) Uttar Pradesh (d) Jharkhand
(b) BSE Institute Limited (c) Markit (e) None of these
(d) HSBC Analytics (e) None of these 35. The 14 km long Zojila tunnel will be constructed in which of
25. The 'Fountain of Oneness' is located in which of the the following states?
following cities? (a) Sikkim (b) Jammu & Kashmir
(a) Ajmer (b) Delhi (c) Uttarakhand (d) Himachal Pradesh
(c) Agra (d) Khajuraho (e) None of these
(e) None of these 36. Which of the following person appointed as Chairman of
26. Which of the following cities to get India's first river gallery? Film and Television Institute of India (FTII)?
(a) Bengaluru (b) Mysore (a) Neha Gupta (b) Hrishikesh Kanitkar
(c) Chennai (d) Hyderabad (c) Gajendra Chauhan (d) Arvind Uppal
(e) None of these (e) None of these
y
o
u
rs
m
SPEED TEST 100

a
231

h
b
37. Who has been appointed as the new chairman of 47. Which of the following companies has acquired Shifu for $8

o
o
Competition Commission of India (CCI)? million to enrich customer experience?

b
.w
(a) DK Sikri (b) Amitabh Kant (a) Paytm (b) Flipkart
(c) Ashok Chawla (d) Dhanendra Kumar

o
(c) Amazon (d) Myntra

rd
(e) None of these
(e) None of these

p
38. In which of the following cities the world's first slum museum

re
48. Which of the following financial entity has been penalised

s
will be set up?

s
by the Securities and Exchange Board of India (SEBI) for

.c
(a) Indore (b) Mumbai

o
(c) Lucknow (d) Ahmedabad breaching the stock broker regulations?

m
(e) None of these (a) R. V. Verma (b) Ashwini Kumar
39. Who has been conferred with the 2016 Global and Asia - (c) Ravindra Kumar (d) Anand Rathi
Pacific Central Bank Governor of the Year? (e) None of these
(a) Janet Yellen (b) Mark Carney 49. A friendship bus service between India and was flagged off
(c) Raghuram Rajan (d) Haruhiko Kuroda via Champawat district in Uttarakhand.
(e) None of these (a) Nepal (b) Japan
40. Who has been honoured with the second Harikrishna (c) Australia (d) Canada
Devsare Baalsahitya Award 2015?
(e) None of these
(a) Gangesh Gunjan (b) Sheela Jhunjhunwala
50. Who has been crowned as the FBB 50. Famina Miss India
(c) Balkrishna Garg (d) Sherjung Garg
(e) None of these world 2016?
41. In which state, the Union govt has decided to set up National (a) Namrata Sharma
Organic Farming Research Institute (NOFRI) in 2016? (b) Pankuri Gidwani
(a) Mizoram (b) Meghalaya (c) Priyadarshini chatterjee
(c) Assam (d) Sikkim (d) Sushruthi Krishna
(e) None of these (e) None of these
42. Who was appointed as the CEO of Wipro?
(a) Abid Ali Z. Neemuchwala Part-II : Quantitative Aptitude
(b) T. Kurien
(c) Lalitha Kumaramangalam DIRECTIONS (Qs. 51-61): What will come in place of the question
(d) Ramesh Sippy mark (?) in the following questions ?
(e) None of these 51. 3 × ? + 30 = 0
43. Who of the following has won the 2015 Costa Novel Prize? (a) – 15 (b) 15
(a) Frances Hardinge (b) Kate Atkinson (c) 10 (d) – 30
(c) Andrea Wulf (d) Helen Macdonald (e) None of these
(e) None of these 52. 40.83 × 1.02 × 1.2 = ?
44. Vilmos Zsigmond, who passed away recently, belong to (a) 49.97592 (b) 41.64660
which of the following fields? (c) 58.7952 (d) 42.479532
(a) Politician (b) Scholar (e) None of these
(c) Sportsperson (d) Cinematographer 1 3 1 22
(e) None of these 53. 3 6 1 ?
3 7 2 7
45. National Bank for Agriculture and Rural Development
(NABARD) signed an Memorandum of Understanding 22
(a) 4.4 (b)
(MoU) with which of the following for monitoring Watershed 7
Projects?
5
(a) Central Glass and Ceramic Research Institute (CGCRI) (c) (d) 40.5
(b) National Remote Sensing Centre (NRSC) 22
(c) Central Institute of Technology (CIT) (e) None of these
(d) Indian Association for the Cultivation of Science 54. 1.5625 ?
(IACS) (a) 125 (b) 12.5
(e) None of these (c) 1.05 (d) 1.25
46. Raghunandan Mandal has passed away recently, he related (e) None of these
to? 55. 3978 + 112 × 2 = ? 2
(a) Politics (b) Scholar (a) 8400 (b) 8406
(c) Sportsperson (d) Cinematographer (c) 8600 (d) 8404
(e) None of these (e) None of these
y
o
u
rs
m
232 SPEED TEST 100

a
h
b
66. A, B and C enter into a partnership with investments of

o
2
103.7 101.3 10? ` 3500, ` 4500 and ` 5500, respectively. In the first six months,

o
56.

b
profit is ` 405. What is A’s share in the profit ?

.w
(a) 6 (b) 7 (a) ` 200 (b) ` 105

o
(c) 5 (d) 3 (c) ` 250 (d) ` 151

rd
(e) None of these

p
(e) None of these

re
57. 7589 – ? = 3434 67. A tap can fill a cistern in 8 hours and another tap can empty

s
s
(a) 721 (b) 4055 it in 16 hours. If both the taps are opened simultaneously,

.c
(c) 3246 (d) 11023 the time taken (in hours) to fill the cistern will be :

o
m
(e) None of these (a) 8 (b) 10
(c) 16 (d) 24
58. 1225 ? 5
(e) None of these
(a) 3 (b) 2 68. Pipes A and B can fill a tank in 5 and 6 hours, respectively.
(c) 35 (d) 7 Pipe C can empty it in 12 hours. The tank is half full. All the
(e) None of these three pipes are in operation simultaneously. After how much
59. 300 + 102 × 2 = ? time, the tank will be full ?
(a) 450 (b) 800
9
(c) 550 (d) 320 (a) 3 h (b) 11 h
17
(e) None of these
5 1.6 2 1.4 8 13
60. ? (c) 2 h (d) 1 h
1.3 11 17
(a) 4 (b) 0.4 (e) None of these
(c) 1.4 (d) 1.2 69. If the sum of the digits of an even number is divisible by 9,
(e) None of these then that number is always divisible by :
(a) 24 (b) 12
2 1 1 (c) 18 (d) 27
61. 3 7 5 ?
5 5 4 (e) None of these
3 3 70. A water tank in the form of a cuboid has its base 20 m long,
(a) 5 (b) 5 7 m wide and 10 m deep. Initially, the tank is full but later
10 20
when water is taken out of it, the level of water in the tank
7 11 reduces by 2 m. The volume of water left in the tank is :
(c) 5 (d) 5 (a) 1120 m3 (b) 400 m3
10 20
(c) 280 m 3 (d) 140 m3
(e) None of these
(e) None of these
62. If one-third of a number is 3 more than one -fourth of the
number, then the number is : DIRECTIONS (Qs. 71-75) : Find the next term in the given series
(a) 18 (b) 24 in each of the questions below.
(c) 30 (d) 36 71. 41, 31, ?, 17, 11, 5
(e) None of these (a) 19 (b) 21
(c) 23 (d) 27
63. A boy was asked to write 25 9 2 but he wrote 2592. The
(e) None of these
numerical difference between the two is:
72. 8, 15, 28, 53, ?
(a) 0 (b) 3
(a) 106 (b) 98
(c) 2 (d) 9
(c) 100 (d) 102
(e) None of these
(e) None of these
64. If the two numbers are respectively 20% and 50% of a third
73. 24, 49, ?, 94, 15, 31, 59, 58
number, what is the percentage of the first number to the
(a) 51 (b) 63
second ?
(c) 77 (d) 95
(a) 10 (b) 20
(e) None of these
(c) 30 (d) 40
74. 5, 10, 13, 26, 29, 58, ?, 122
(e) None of these
(a) 60 (b) 61
65. A man gains 10% by selling a certain article for a certain
(c) 111 (d) 91
price. If he sells it at double the price, then the profit made
is: (e) None of these
(a) 120% (b) 60% 75. 2, 3, 10, 15, 26, ?, 55
(c) 100% (d) 80% (a) 32 (b) 33
(e) None of these (c) 34 (d) 35
(e) None of these
y
o
u
rs
m
SPEED TEST 100

a
233

h
b
DIRECTIONS (Qs. 76-80) : What approximate value should come DIRECTION (Qs. 86-90) : Study the following graph carefully to

o
o
in place of the question mark (?) in the following questions? answer the question given below it.

b
.w
(You are not expected to calculate the exact value). Production of paper (in lakh tonnes) by 3 different

o
companies A, B & C over the years

rd
76. 3
860000 ?

p
(a) 75 (b) 80

re
(c) 110 (d) 125 70

s
s
60 60 60 60

.c
(e) 95 60 55 55 55 55

o
50 50 50 50 50

m
5 1 2 50 45 45 45
77. 1 5 2 ? 40 40
8 3 5 40
(a) 15 (b) 4 30
(c) 19 (d) 9 20
(e) 21
10
78. 8769 82 4 ?
0
(a) 27 (b) 44 2010 2011 2012 2013 2014 2015
(c) 429 (d) 12
A B C
(e) 512
79. ? % of 45.999 × 16% of 83.006 = 116.073
(a) 6 (b) 24 86. What is the difference between the production of company
(c) 19 (d) 30 C in 2010 and the production of Company A in 2015?
(e) 11 (a) 50,000 tonnes (b) 5,00,00,000 tonnes
80. 12.998 × 27.059 × 17.999 = ? (c) 50,00,000 tonnes (d) 5,00,000 tonnes
(a) 6020 (b) 6320
(e) None of these
(c) 6800 (d) 6540
87. What is the percentage increase in production of CompanyA
(e) 6150
from 2011 to 2012?
81. The area of a circular plot is twice the area of a rectangular
plot. If the area of the rectangular plot is 11088 sq. metres., (a) 37.5 (b) 38.25
what is the perimeter of the circular plot? (c) 35 (d) 36
(a) 484 metres (b) 572 metres (e) None of these
(c) 528 metres (d) 440 metres 88. For which of the following years the percentage of rise/fall
(e) None of these in production from the previous year the maximum for
82. The sum of the two digits of a two-digit number and the Company B?
difference between the two digits of the two-digit number is (a) 2011 (b) 2012
8. What is the two digit number? (c) 2013 (d) 2014
(a) 80 (b) 88
(e) 2015
(3) 44 (d) Cannot be determined
89. The total production of Company C in 2012 and 2013 is
(e) None of these
what percentage of the total production of Company A in
83. The total number of students studying in a college is 4220.
2010 and 2011?
If the number of girls studying in the college is 2420, what is
(a) 95 (b) 90
the respective ratio of the number of boys to the number of
girls studying in the college? (c) 110 (d) 115
(a) 90 : 131 (b) 90 : 121 (e) None of these
(c) 121 : 70 (d) 121 : 80 90. What is the difference between the average production per
(e) None of these year of the company with highest average production and
84. In how many different ways can the letters of the word that of the company with lowest average production in lakh
'SCENIC' be arranged? tonnes?
(a) 120 (b) 720 (a) 3.17 (b) 4.33
(c) 60 (d) 360 (c) 4.17 (d) 3.33
(e) None of these (e) None of these
85. The cost of 4 bags and 12 purses is ` 1,520, what is the cost 91. The 63% of a number is 2583. What will be 45% of this
of 10 bags and 30 purses? number?
(a) ` 3,600 (b) `3,500 (a) 1845 (b) 1763
(c) `4,000 (d) `3,900 (c) 2255 (d) 1927
(e) None of these (e) None of these
y
o
u
rs
m
234 SPEED TEST 100

a
h
b
column than the number of children seated in each row.

o
4 6 2 9 3

o
92. If the fractions , , , and be arranged in descending How many children are there in each row?

b
5 7 9 11 8
(a) 9 (b) 7

.w
order, the fourth fraction will be (c) 11 (d) 13

o
rd
9 4 (e) None of these

p
(a) (b)
11 5

re
Part-III : Reasoning Ability

s
s
3 6

.c
(c) (d) DIRECTIONS (Qs. 101-103) : Study the following paragraph

o
8 7 and answer the questions that follow :

m
(e) None of these
93. A shopkeeper purchased 245 pieces of an article at `30 per In our society, corruption and abuse of office has been
piece. He spent `980 on transport and `1470 on packing the aggravated by three factors. First, there is a colonial legacy of
articles. He sold the articles at the rate of `50 per piece. unchallenged authority and propensity to exercise power
What is the per cent profit earned? arbitrarily.
In a society which worships power, it is easy for public
(a) 25 (b) 20
officials to deviate from ethical conduct. Second, there is enormous
(c) 28 (d) 22.5
asymmetry of power in our society. Nearly 90% of our people are
(e) None of these
in the unorganized sector. Quite a number of them lead a precarious
94. Difference between the compound interest and simple
existence, depending on subsistence wages with no job security.
interest accrued in two years at 8% per annum is `128.
And nearly 70% of the organized workers with job security and
What is the principal amount?
regular monthly wages are employed by the state directly or
(a) `18000 (b) `16000
through public sector undertakings. Almost all these employees
(c) `20000 (d) Cannot be determined
are 'educated' in a largely illiterate and semiliterate society and
(e) None of these
economically even the lowliest of public servants are better off
95. The number obtained by interchanging the digits of a two
than most people in the country. What is more, their employment
digit number is less than the original number by 18. If sum
in government comes with all the trappings of power. Such
of the digits is 6, what was the original two digit number?
asymmetry of power reduces societal pressure to conform to
(a) 51 (b) 24 ethical behaviour and makes it easy to indulge in corruption
(c) 42 (d) 15 101. The passage best support the statement that–
(e) None of these (a) Corruption is a consequence of hundred, gears slavary
96. Samar spends 52% of his monthly salary on household of the country.
expenditure and 23% on miscellaneous expenditure. If he is (b) Among the many causes socio-economic disparity is
left with `4500, what is his monthly salary? the main cause of corruption.
(a) `16000 (b) `17500 (c) Corruption is a status symbol in our society.
(c) `17000 (d) `18500 (d) Mostly it is found that people themselves incourage
(e) None of these the corruption.
97. Number of girls in a class is 44 which is 55% of the total (e) None of these
number of students. How many boys are there in the class? 102. Which of the following is an inference which can be made
(a) 75 (b) 76 from the facts stated in the above passage?
(c) 84 (d) 36 (a) Corruption is the consequences of can challenged
(e) None of these authority, asymmetry of power and illiteracy etc.
98. Average marks obtained in History by student in ‘A’ division (b) People are helpless before the corruption.
are 75 and the average marks obtained by 40 students in ‘B’ (c) The government should make a strict law against the
division are 78. What are the overall average marks obtained corruption.
in History by the students in both the divisions (rounded (d) People should stir a movement against corruption.
off to two digits after decimal)? (e) None of these
(a) 76.81 (b) 77.11 103. Which of the following is the most effective statement in
(c) 77.41 (d) 76.41 the context of the above passage?
(e) None of these (a) Asymmetry of power reduces societal pressure to
99. The ratio between length and breadth of a rectangular plot conform to ethical behaviour and makes it easy to
is 5 : 3 respectively and its perimeter is 48 m. What will be its indulge in corruption.
area in sq m? (b) Society itself nourishes the corruption.
(a) 120 (b) 116 (c) Power is worshiped in our society which increases the
(c) 115 (d) 76.41 corruption.
(e) None of these (d) 90% people are in unorganised sector and they always
100. In a class of 63 children, the children are seated in rows and feel the insecurity of jobs.
columns such that there are two more children in each (e) None of these.
y
o
u
rs
m
SPEED TEST 100

a
235

h
b
104. If ‘+’ means ‘divided by’; ‘–’ means ‘added to’; ‘×’ means 113. How many such vowels are there in the above arrangement

o
o
‘substracted from’ and ‘÷’ means ‘multiplied by’; then 26 – each of which is immediately preceded by a symbol and

b
immediately followed by a number ?

.w
15 + 5 × 4 ÷ 2 = ?
(a) 15 (b) 20 (a) None (b) One

o
rd
(c) 25 (d) 18 (c) Two (d) Three

p
(e) None of these (e) More than three

re
s
105. Four of the following five are alike in a certain way and so form DIRECTIONS (Qs. 114-119) : In the following questions, the

s
.c
a group. Which is the one that does not belong to that group. symbols , $, *, @ and © are used with the following meanings

o
(a) RPN (b) WSU

m
illustrated.
(c) HDF (d) LHJ
(e) QMO ‘P $ Q’ means ‘P is neither equal to nor greater than Q’.
106. In a row of forty students R is the fifth from right end and ‘P © Q’ means ‘P is neither equal to nor smaller than Q’.
there are ten students between R and D. What is D’s position ‘P Q’ means ‘P is neither greater to nor smaller than Q’.
from the left end of the row ? ‘P @ Q’ means ‘P is not smaller than Q’.
(a) 26th (b) 23rd ‘P * Q’ means ‘P is not greater than Q’.
(c) 24th (d) 25th In each question three statements showing relationship have been
(e) Data inadequate given, which are followed by three conclusions I, II and III.
107. In a certain code ORBITAL is written as CSPHMBU. How is Assuming that the given statements are true, find out which
CHARGER written in that code ? conclusion(s) is/are definitely true.
(a) BIDQSFH (b) BIDSSFH 114. Statements : M @ D, D K, K © R
(c) BIDQQDF (d) DIBQSFH Conclusions : I. R $ M
(e) None of these II. K M
108. Among A, B, C, D and E each having scored different marks III. K $ M
in an examination, B scored more than C and E and less than (a) Only I is true (b) Only II is true
A and D. C’s marks are not the lowest. Who scored the (c) Only III is true (d) Either II or III and I are true
lowest marks ? (e) Either II or III is true
(a) O (b) C 115. Statements : F * T, T $ N, N @ R
(c) B (d) Data inadequate Conclusions : I. R $ T
(e) None of these II. N © F
109. How many such pairs of letters are there in the word
III. F $ R
OVERWHELM each of which has as many letters between
(a) None is true (b) Only I is true
them in the word as in the English alphabet?
(c) Only II is true (d) Only III is true
(a) None (b) One
(e) II and III are true
(c) Two (d) Three
116. Statements : B © N, N @ R, F * R
(e) More than three
Conclusions : I. B © M
110. In a certain code BROWN is written as 531 @% and MEAN
is written as 26©%. How is ROBE written in that code ? II. F * N
(a) 3@16 (b) 3516 III. R $ B
(c) 3156 (d) 3©16 (a) I and II are true (b) I and III are true
(e) None of these (c) II and III are true (d) All I, II and III are true
(e) None of the above
DIRECTIONS (Qs. 111-113) : Study the following arrangement
117. Statements : D $ M, M * B, B J
carefully and answer the questions given below :
Conclusions : I. J © D
R5# 3$MD P14F ©A6EWJ2@ K8Q7
II. B * D
% UT I * V9
III. J $ M
111. Which of the following is the seventh to the left of the
nineteenth from the left end of the above arrangement ? (a) I and II are true (b) I and III are true
(a) D (b) U (c) II and III are true (d) All I, II and III are true
(c) © (d) (e) None of the above
(e) None of these 118. Statements : W K, K © F, F $ M
112. What should come next in the following series based on the Conclusions : I. M © K
above arrangement ? II. W @ F
53$ P4F 6WJ 8Q ? III. F @ W
(a) UI * (b) UT * (a) Only I is true (b) Only II is true
(c) UIV (d) UTV (c) Only III is true (d) II and III are true
(e) None of these (e) None is true
y
o
u
rs
m
236 SPEED TEST 100

a
h
b
119. Statements : F @ T, T K, K * D 127. Statements : Some bikes are cars. Some cars are trains. Some

o
o
Conclusions : I. D @ F trains are buses.

b
Conclusions : I. Some buses are cars.

.w
II. F @ K
III. D @ T II. Some trains are bikes.

o
rd
(a) II and III are true (b) I and III are true III. Some buses are bikes.

p
(c) I and II are true (d) All I, II and III are true (a) None follows (b) Only I follows

re
s
(e) None of the above (c) Only II follows (d) Only III follows

s
.c
DIRECTIONS (Q. 120-125) : Study the following information (e) I and II follow

o
128. Statements : All dogs are cats. Some cats are rats. All rats

m
carefully and answer the questions given below
are mats.
P, A, D, Q, T, M, R and B are sitting around a circle facing at
Conclusions : I. Some mats are cats.
the centre. D is third to the left of T who is fifth to the right of P. A
is third to the right of B, who is second to the right of D. Q is II. Some mats are dogs.
second to the left of M. III. Some rats are cats.
120. Who is to the immediate right of D ? (a) Only I follows (b) Only II follows
(c) Only III follows (d) I and III follow
(a) M (b) Q
(e) None of the above
(c) B (d) Data inadequate
129. Statements : All pens are sticks. All sticks are rings. All
(e) None of these
rings are rods.
121. Who is second to the right of M ?
Conclusions : I. Some rings are pens.
(a) B (b) R
II. Some rods are sticks.
(c) T (d) Q
III. Some rods are pens.
(e) None of these
(a) I and II follow (b) I and III follow
122. Who is second to the left of D ?
(c) II and III follow (d) All I, II and III follow
(a) A (b) Q
(e) None of the above
(c) B (d) P
130. Statements : Some tables are chairs. All chairs are houses.
(e) Data inadequate All houses are tents.
123. Who is third to the right of P ? Conclusions : I. All houses are chairs.
(a) D (b) M II. Some tents are chairs.
(c) R (d) Data inadequate III. Some houses are tables.
(e) None of these (a) I and II follow (b) I and III follow
124. Which of the following pairs represents the immediate (c) II and III follow (d) All I, II and III follow
neighbours of A ? (e) None of the above
(a) PT (b) PB 131. Statements : Some boxes are walls. No wall is road. All roads
(c) TQ (d) PD are rivers.
(e) None of these Conclusions : I. Some rivers are walls.
125. In which of the following pairs the first person is sitting to II. Some roads are boxes.
the immediate right of the second person ? III. No wall is river.
(a) DM (b) BT (a) Only I follows (b) Either I or III follows
(c) RA (d) PQ (c) Only III follows (d) Only II follows
(e) PA (e) II and III follow
DIRECTIONS (Qs. 126-131) : In each of question below are three 132. In a certain code MAIN is written as '9364' and DEAR is
statements followed by three conclusions numbered I, II and III. written as '8532'. How is MEND written in that code?
You have to take the three given statements to be true even if they (a) 9548 (b) 9458
seem to be at variance from commonly known facts and then (c) 9538 (d) 9528
decide which of the given conclusions logically follows from the (e) None of these
three given statements disregarding commonly known facts. Then 133. In a certain code DREAMING is written as BFSEFMHL.
decide which of the answers (a), (b), (c), (d) and (e) is the correct 'How is TREATISE written in that code?
answer and indicate it on the answer sheet. (a) USFBDRHS (b) BFSUDTHS
126. Statements : All cups are benches. Some benches are drums. (c) BFSUSHRD (d) BDQSDRHS
All drums are kites. (e) None of these
Conclusions : I. Some kites are cups. 134. The positions of how many digits in the number 5314697
II. Some kites are benches. will remain unchanged if the digits are rearranged in
III. Some drums are cups. ascending order within the number?
(a) None follows (b) Only I follows (a) None (b) One
(c) Only II follows (d) Only III follows (c) Two (d) Three
(e) II and III follow (e) More than three
y
o
u
rs
m
SPEED TEST 100

a
237

h
b
135. Among A, B, C, D and E each having different amount of 144. A hard copy of a file created on a computer refers to data

o
o
money, C has more money than only E. B has more money (a) saved on a floppy disk

b
than D but less than A. Who among them has the highest

.w
(b) printed on a printer
amount of money? (c) backed up on a tape drive

o
rd
(a) B (b) A (d) sent as an e-mail

p
(c) D (d) Data inadequate (e) None of these

re
s
(e) None of these 145. When you quickely press and release the left mouse button

s
.c
136. Prakash walked 30 metres towards West, took a left turn and twice, you are

o
walked 20metres. He again took a left turn and walked 30 (a) Primary – clicking (b) Pointing

m
metres. He then took a right turn and stopped. Towards (c) Double – clicking (d) Secondary – clicking
which direction was he facing when he stopped? (e) None of these
(a) South (b) North 146. Underlined text, such as text and folder names is referred to
(c) East (d) Data inadequate as a
(e) None of these (a) icon (b) hyperlink
137. How many meaningful English words can be made with the (c) menu (d) source drive
letters RTOU using each letter only once in eachword? (e) None of these
(a) None (b) One 147. To centre a paragraph using shortcut keys, press
(c) Two (d) Three (a) CTRL + C (b) CTRL + E
(c) CTRL + L (d) CTRL + R
(e) More than three
(e) None of these
138. If 'P' denotes '–'; 'Q' denotes ' ', 'R' de notes '×' and 'W'
148. Word processing, spreadsheet, and photo-editing are
denotes '+' then-
examples of
48 Q 12 R 10 P 8 W 4=? (a) application software
(a) 56 (b) 40 (b) system software
(c) 52 (d) 44 (c) operating system software
(e) None of these (d) platform software
139. Which of the following is the middle digit of the second (e) None of these
highest number among the five three-digit numbers given 149. Which of the following displays to the right of the space
below ? where the text will be inserted when you type?
512 739 428 843 654 (a) Screen tip (b) Insertion point
(a) 1 (b) 3 (c) Rulers (d) Office assistant
(c) 2 (d) 4 (e) None of these
(e) 5 150. In a spreadsheet, a cell is defined as the
140. In a certain code language 'green grass everywhere' is written (a) intersection of a table and a tuplet
as 'dik pa sok' and 'cow eats grass' is written as 'nok ta pa'. (b) intersection of a file and a database
How is 'cow' written in that code language? (c) intersection of a row and column
(a) nok (b) ta (d) intersection of a field and a record
(c) nok or ta (d) Data inadequate (e) None of these
(e) None of these
141. A program designed to destroy data on your computer which Part-IV : English Language
can travel to “infect” other computers is called a
DIRECTIONS (Q. 151-160) : Read the following passage
(a) disease (b) torpedo carefully and answer the questions given below it. Certain words/
(c) hurricane (d) virus phrases have been printed in bold to help you locate them while
(e) None of these answering some of the questions.
142. What is the most common way to get a virus in your
There was a country long time ago where the people would
computer’s hard disk ?
change a king every year. The person who would become the
(a) By installing games from their CDROMS
king had to agree to a contract that he would be sent to an island
(b) By uploading pictures from mobile phones to the after one year of his being a king.
computer
One king had finished his term and it was time for him to go
(c) By opening e-mails to the island and live there. The people dressed him up in expensive
(d) By sending e-mails clothes and put him on an elephant and took him around the cities
(e) None of these to say goodbye to all the people. This was a moment of sadness
143. What is the main folder on a storage device called ? for all kings who ruled for one year. After bidding farewell, the
(a) platform (b) interface people took the king to a remote island in a boat and left him there.
(c) root directory (d) device driver On their way back, they discovered a ship that had sunk just
(e) None of these recently. They saw a young man who had survived by holding on
y
o
u
rs
m
238 SPEED TEST 100

a
h
b
to a floating piece of wood. As they needed a new king, they (b) He would be gifted with expensive clothes and

o
o
picked up the young man and took him to their country. They jewellery

b
requested him to be king for a year. First he refused but later he (c) He would be sent on an island which required a lot of

.w
agreed to be the king. People told him about all the rules and work to be done

o
rd
regulations and that how he would be sent to an island after one (d) He would be asked to buy an elephant and go to the

p
year. remote island himself

re
After three days of being a king, he asked the ministers if (e) None of the above

s
s
they could show him the island where all the other kings were 153. What did the young man notice on his visit to the remote

.c
sent. They agreed and took him to the island. The island was island after three days of being king?

o
m
covered with a thick Jungle and sounds of vicious animals were (a) That the animals on the island were too many to be
heard coming out of it. The king went a little bit further to check. killed
Soon he discovered dead bodies of all the past kings. He (b) That the island was very big
understood that as soon as they were left on the island, the wild (c) That the jungle on the island was full of animals that
animals had come and killed them. had killed all the previous kings placed on the island
The king went back to the country and collected 100 strong (d) That the island was very beautiful and clean
workers. He took them to the island and instructed them to clean (e) Not mentioned in the passage
the jungle, remove all the deadly animals and cut down all excess 154. What happened to the island in the first month of the young
trees. He would visit the island every month to see how the work king’s tenure?
was progressing. In the first month, all the animals were removed (A) The wild animals were removed from the island.
and many trees were cut down. In the second month, the whole (B) The whole island was cleaned.
island was cleaned out. The king then told the workers to plant (C) Many unnecessary trees were cut down.
gardens in various parts of the island. He also took with himself (a) Only A (b) Only B
useful animals like chickens, ducks, birds, goats, cows etc. In the (c) Only C (d) B and C
third month, he ordered the workers to build big houses and (e) A and C
docking stations for ships. Over the months, the island turned 155. What could be said about the island after the young king
into a beautiful place. The young king would wear simple clothes had made his servants work on it ?
and spend very little from his earnings as a king. He sent all the (a) The dangerous island had been turned to a beautiful
earnings to the island for storage. When nine months passed like place
this, the king called the ministers and told them: “I know that I (b) The island still remained the same despite all the young
have to go the island after one year, but I would like to go there king’s efforts
right now.” But the ministers didn’t agree to this and said that he (c) The island was barely tolerable now
had to wait for another three months to complete the year. (d) The island was safe from all types of attacks
Three months passed and now it was a full year. The people (e) None of the above
dressed up the young king and put him on an elephant to take him 156. Which of the following describes the young king correctly?
around the country to say goodbye to others. However, this king (A) He was intelligent
was unusually happy to leave the kingdom. People asked him, (B) He had foresight
“All the other kings would cry at this moment. Why is it that you (C) He was cunning
are laughing?” He replied, “Don’t you know what the wise people (a) Only A (b) A and B
say? They say that when you come to this world as a baby, you (c) Only C (d) B and C
are crying and everyone else is smiling. Live such a life that when (e) All A, B and C
you die, you will be smiling and everyone around you will be 157. What was the king’s request to the ministers after the
crying. I have lived that life. While all the other kings were lost completion of nine months?
into the luxuries of the kingdom, I always thought about the future (a) That he should not be sent to the island after
and planned for it. I turned the deadly island into a beautiful completion of one year
abode for me where I can stay peacefully.” (b) That he would like to go to the island immediately
151. Why did the people of the kingdom change the king every (c) That he should be paid more in order to improve work
year? on the island
(a) As their first king had invented this system and had (d) That he should be sent to the island after a year
recorded it in the form of a contract (e) None of the above
(b) As they believed that the new king would bring better 158. How did the young king arrange for money on the island?
ideas to the kingdom. (a) By selling a large amount of his property
(c) As they wanted their king to relax on an island after (b) By spending all the money derived from his income as
one year of hard work a king
(d) Not mentioned in the passage
(c) By borrowing money from the ministers and sending it
(e) None of the above
to the island
152. What would happen to the king once his term of one year
(d) By spending very less of his income as a king and
was over ?
sending it to the island for storage
(a) He would be paraded in the cities as a farewell and
(e) None of the above
then be taken to a remote island
y
o
u
rs
m
SPEED TEST 100

a
239

h
b
159. Why were the people of the kingdom puzzled when the (C) The ducklings hurried towards the lake and the mother duck

o
o
young king was taken around the country to say goodbye began to walk back and forth dragging one wing on the

b
to everyone? ground.

.w
(a) As they could not believe that one year had elapsed (D) A mother duck and her little ducklings were on their way to

o
rd
so soon the lake one day.

p
(b) As they were nor aware that the young king was (E) The fox stared in disbelief at the mother duck and her

re
actually a wise sage ducklings as he could not reach the ducklings because they

s
s
were in the middle of the lake by now.

.c
(c) As the young king was happy to go to the island unlike

o
the previous kings (F) When the fox saw her he became happy as he thought that

m
(d) Not mentioned in the passage the mother duck was hurt and couldn’t fly and that he could
(e) None of the above easily catch and eat her!
160. What can possibly be the moral of the story? 166. Which of the following should be the FIRST sentence after
(a) Always put others before yourself rearrangement?
(b) Give respect to others (a) A (b) B
(c) Live in the present and forget about the future (c) C (d) D
(d) Do not put things off until tomorrow (e) E
(e) Always think and plan ahead 167. Which of the following should be the SECOND sentence
after rearrangement ?
DIRECTIONS (Q. 161-163): Choose the word/group of words (a) A (b) B
which is most similar in meaning to the word/group of words (c) C (d) D
printed in bold as used in the passage. (e) F
161. CONTRACT 168. Which of the following should be the THIRD sentence after
(a) work (b) signature rearrangement?
(c) deal (d) temporary (a) A (b) B
(e) get (c) C (d) D
162. BIDDING (e) E
(a) wishing (b) auctioning 169. Which of the following should be the FOURTH sentence
after rearrangement?
(c) wasting (d) playing
(a) B (b) C
(e) talking
(c) D (d) E
163. ABODE
(e) F
(a) stop (b) mountain
170. Which of the following should be the LAST (SIXTH)
(c) plenty (d) house
sentence after rearrangement?
(e) dwelling
(a) A (b) B
DIRECTIONS (Q. 164-165) : Choose the word / group of words (c) D (d) E
which is most opposite in meaning to the word / group of words (e) F
printed in bold as used in the passage.
DIRECTIONS (Q. 171-175) : Which of the phrases (a), (b), (c)
164. SURVIVED and (d) given below each sentence should replace the phrase
(a) scratched (b) died printed in bold in the sentence to make it grammatically correct?
(c) lived (d) fell If the sentence is correct as it is given and ‘No correction is
(e) suffered required’, mark (e) as the answer.
165. VICIOUS 171. As it was a dark and stormy night, Lata was too scared to go
(a) simple (b) small home alone.
(c) tough (d) harmless (a) very scary to (b) much scared to
(e) ferocious (c) as scared to (d) to scared too
DIRECTIONS (Q. 166-170) : Rearrange the following six (e) No correction required
sentences (A), (B), (C), (D), (E) and (F) in the proper sequence 172. Since it was her engagement party, Riya was dress to kill.
to form a meaningful paragraph; then answer the questions given (a) dresses to kill (b) dressed to kill
below them. (c) dressed to killings (d) dressing to killed
(A) All of a sudden the mother duck saw a fox in the distance, (e) No correction required
was frightened and shouted, “Children, hurry to the lake, 173. Ramesh worries endlessly about his son’s future as he was
there’s a fox !” so poor in studies.
(B) The mother duck ran, leading the fox away from the lake and (a) worry endless (b) worried endless
as soon as the fox came very close, the mother duck quickly (c) worried endlessly (d) worries endless
spread her wings and rose up in the air. (e) No correction required
y
o
u
rs
m
240 SPEED TEST 100

a
h
b
174. Now that the actual criminal had been caught, Kunal was 181. (a) believe (b) thought

o
o
happy that he was finally let of the hook. (c) wished (d) smiled

b
.w
(a) off the hook (b) of the hookings (e) rejoiced

o
(c) off the hooks (d) of the hooks

rd
182. (a) make (b) let
(e) No correction required

p
(c) pay (d) allow

re
175. The little boy appeared all of a sudden out of nowhere and

s
(e) request

s
take everyone by surprise.

.c
183. (a) idea (b) luck

o
(a) took everyone as surprised

m
(c) necklace (d) cry
(b) take everyone with surprised
(e) presence
(c) took everyone by surprises
184. (a) opened (b) snatch
(d) took everyone by surprise
(e) No correction required (c) decorated (d) took
(e) ring
DIRECTIONS (Q. 176-185) : In the following passage there are
blanks, each of which has been numbered. These numbers are 185. (a) smiling (b) hurried
printed below the passage and against each, five words are (c) barking (d) much
suggested, one of which fits the blank appropriately. Find out (e) friendly
the appropriate word in each case. DIRECTION (Qs. 186 to 190): In each of the following sentences,
A lamb was (176) with a flock of sheep one day. She soon an idiomatic expression or a proverb is highlighted. Select the
found some sweet grass at the (177) of the field. Farther and alternative which best describes its use in the sentence.
farther she went, away from the others. She was enjoying herself 186. He went to his friend's house in the evening as was his
so much that she did not (178) a wolf coming nearer to her. wont.
However, when it (179) on her, she was quick to start pleading, (a) as usual (b) as he wanted
“Please, please don’t eat me yet. My stomach is full of grass. If
(c) as his want was (d) as he wanted that day
you wait a while, I will (180) much better.” The wolf (181) that
(e) none of these
was a good idea, so he sat down and waited. After a while, the
187. Why do you wish to tread on the toes?
lamb said, “If you (182) me to dance, the grass in my stomach will
(a) To give offence to them
be digested faster.” Again the wolf agreed. While the lamb was
dancing, she had a new (183). She said, “Please take the bell from (b) To follow them grudgingly
around my neck. If you ring it as hard as you can, I will be able to (c) To treat them indifferently
dance even faster.” The wolf (184) the bell and rang it as hard as (d) To be kicked by them
he could. The shepherd heard the bell ringing and quickly sent (e) None of these
his dogs to find the missing lamb. The (185) dogs frightened the 188. He intends setting up as a lawyer in the adjoining district.
wolf away and saved the lamb’s life. (a) To establish himself (b) To migrate
176. (a) watching (b) laughing (c) To join (d) To settle
(c) willing (d) tiring (e) None of these
(e) grazing 189. The autographed bat from the famous cricketer Sunil
177. (a) height (b) edge Gavaskar is worth a jew's eye.
(c) midst (d) first (a) Not a worthy possession
(e) base
(b) unnecessary
178 (a) notice (b) trust
(c) A costly items
(c) saw (d) worry
(d) A possession of high value
(e) maintain
(e) None of these
179. (a) lunge (b) visited
190. The speaker gave a bird's eye view of the political conditions
(c) ate (d) stand
(e) pounced in the country.
180. (a) walk (b) die (a) a personal view (b) a general view
(c) taste (d) mix (c) a biased view (d) a detailed presentation
(e) reveal (e) None of these
y
o
u
rs
m
a
h
RESPONSE SHEET

b
o
o
b
.w
o
Test Code : ............................... Time taken : ......................... Date : ..........................

rd
p
(a) (b) (c) (d) (e) (a) (b) (c) (d) (e) (a) (b) (c) (d) (e) (a) (b) (c) (d) (e)

re
s
1. 51. 101. 146.

s
.c
2. 52. 102. 147.

o
3. 53.

m
103. 148.
4. 54. 104. 149.
5. 55. 105. 150.
6. 56.
106. 151.
7. 57.
8. 107. 152.
58.
9. 59. 108. 153.
10. 60. 109. 154.
11. 61. 110. 155.
12. 62. 111. 156.
13. 63. 112. 157.
14. 64. 113. 158.
15. 65. 114. 159.
16. 66.
115. 160.
17. 67.
18. 68. 116. 161.
19. 69. 117. 162.
20. 70. 118. 163.
21. 71. 119. 164.
22. 72. 120. 165.
23. 73. 121. 166.
24. 74. 122. 167.
25. 75. 123. 168.
26. 76.
27. 124. 169.
77.
28. 78. 125. 170.
29. 79. 126. 171.
30. 80. 127. 172.
31. 81. 128. 173.
32. 82. 129. 174.
33. 83. 130. 175.
34. 84. 131. 176.
35. 85.
132. 177.
36. 86.
37. 133. 178.
87.
38. 88. 134. 179.
39. 89. 135. 180.
40. 90. 136. 181.
41. 91. 137. 182.
42. 92. 138. 183.
43. 93. 139. 184.
44. 94. 140. 185.
45. 95.
46. 141. 186.
96.
47. 97. 142. 187.
48. 98. 143. 188.
49. 99. 144. 189.
50. 100. 145. 190.
m
o
.c
SPEED TEST 100

s
s
re
p
rd
o
.w
b
o
o
b
h
a
m
rs
u
o
y
242
y
o
u
rs
m
a
h
b
o
o
101

b
.w
Full Main Test - 5

o
rd
p
re
s
s
.c
o
m
Max. Marks : 200 No. of Qs. 190 Time : 2 hr. 40 min. Date : ........./......../................

Part-I : General Awareness 8. Which of the following statement is true?


(a) Banks cannot accept demand and time deposite from
1. On which one of the following issues can SEBI penalize any public.
company in India?
(b) Banks can accept only demand deposits from public.
(A) Violation of Banking Regulation Act.
(c) Banks can accept only time deposits from public.
(B) Violation of foreign portfolio investment guidelines. (d) Banks can accept both demand and time deposits from
(C) For violation of Negotiable Instrument Act. public.
(a) Only (A) (b) All (A), (B) & (C) (e) Banks can accept demand and time deposits only from
(c) Only (A) & (B) (d) Only (B) & (C) government.
(e) Only (B) 9. Interest payable on saving bank accounts is
2. Expand the term ALM as used in Banking/Finance sector?
(a) not regulated by RBI.
(a) Asset Liability Mismatch
(b) regulated by State Governments
(b) Asset Liability Maturity
(c) Asset Liability Management (c) regulated by Central Government
(d) Asset Liability Manpower (d) regulated by RBI
(e) None of the above (e) regulated by Finance minister.
3. What is an ISO series? 10. Which of the following is the correct statement?
(a) Documentation of production processes (a) State bank of India is the sole authority to issue and
(b) Engineering process flowchart manage currency in India.
(c) Quality management and quality assurance standards (b) A nationalized bank is the sole authority to Issue and
(d) All of the above manage currency in India
(e) None of these (c) A cooperative bank is the sole authority to issue and
4. The Federation of Indian Chambers of Commerce and manage currency in India.
Industry (FICCI) was founded in 1927 by (d) RBI is the sole authority to issue and manage currency
(a) Birla and Tata (b) Tata and Thakurdas in India.
(c) Thakurdas and Birla (d) Tata and Godrej
(e) None of these
(e) None of these
11. Accounts are allowed to be operated by cheques in respect
5. Every year March 20 is celebrated as what day?
of
(a) World Sparrow Day
(b) International Women’s Day (a) Both Savings bank accounts and fixed deposit accounts
(c) World Cuckoo Day (b) Savings bank accounts and current accounts
(d) International Child Day (c) Both Savings bank accounts and loan accounts
(e) International Mother’s Day (d) Both Savings bank accounts and cash accounts only
6. Which of the following is not a Central Government tax? (e) Both Current accounts and fixed deposit accounts
(a) Income tax (b) Customs 12. Which of the following is correct statement?
(c) Land revenue (d) Corporation tax (a) Normally no interest is paid on current deposit
(e) None of these accounts
7. The expansion for the BIS, in the context of the banking (b) Interest is paid on current accounts at the same rate as
industry is term deposit accounts
(a) Bank for International Settlements
(c) The rate of interest on current account and savings
(b) Bank for Industrial Settlements account are the same
(c) Bank for Industrial Sectors
(d) No interest is paid on any deposit by the bank
(d) Bank for International Services
(e) Savings deposits are the same as current deposits.
(e) None of these
y
o
u
rs
m
244 SPEED TEST 101

a
h
b
13. Which of the following terms is not associated with the 23. The RV Easwar Committee gave recommendations on:

o
o
game of cricket? (a) Simplification of income tax laws

b
.w
(a) China man (b) Hook (b) Goods and Services Tax in India

o
(c) Pull (d) Love (c) Banking reforms

rd
(e) None of these

p
(d) Crude oil and gas pricing

re
14. Larry page and Sergey Brin are well known as (e) None of these

s
s
(a) Creators of Bluetooth device

.c
24. The RV Easwar Committee gave recommendations on

o
(b) Founders of Google simplification of income tax laws. . 10. Who among the

m
(c) Stem cell researchers following is popularly known as "Krishi ka Rishi"?
(d) Scientists (a) Subhash Palekar
(e) None of these (b) Mankombu Sambasivan Swaminathan
15. The book ‘A Bend in the River’ is written by
(c) Subrahmanyan Chandrasekhar
(a) VS Naipaul (b) Chetan Bhagat
(d) Norman Ernest Borlaug
(c) Vikram Seth (d) Arundhati Roy
(e) None of these
(e) None of these
16. When a bank returns a cheque unpaid. It is called 25. "Maa Annapurna Yojna" has been launched in which state
of India?
(a) payment of the cheque
(b) drawing of the cheque (a) Odisha (b) Tamil Nadu
(c) cancelling of the cheque (c) Gujarat (d) Kerela
(d) dishonour of the cheque (e) None of these
(e) taking of the cheque. 26. What is the maximum speed of the India's first semi high-
17. Distribution of Insurance products and Insurance policies speed train "Gatimaan Express"?
by banks as corporate agents is known as (a) 160 kmph (b) 170 kmph
(a) General Insurance (b) Non-life Insurance (c) 180 kmph (d) 190 kmph
(c) Bancassurance (d) Insurance banking (e) None of these
(e) Deposit Insurance 27. World's first 'White Tiger Safari' has opened in which state
18. What does the letter 'L' denote in term 'LAF' as referred of India?
every now and then in relation to monetary policy of the
(a) Rajasthan (b) Madhya Pradesh
RBI?
(c) Gujarat (d) West Bengal
(a) Liquidity (b) Liability
(c) Leveraged (d) Longitudinal (e) None of these
(e) Linear 28. Unakoti hill,an ancient Shaivite place of worship, is located
19. POSCO is in the process of establishing its plants in India. in which state of India?
What does the letter 'P'. denote in the name POSCO? (a) Uttarakhand (b) Tamil Nadu
(a) Popular (b) Pallin (c) Odisha (d) Tripura
(c) Pohang (d) Paradeep (e) None of these
(e) Petersburg 29. The Central Industrial Security Force ( CISF) Raising Day is
20. The remuneration payable to the Attorney General of India celebrated on which date?
is determined by? (a) February 11 (b) March 10
(a) President of India (b) Prime Minister
(c) April 26 (d) May 3
(c) Law Minister (d) Parliament
(e) None of these
(e) None of these
21. Farzad B gas field is located in which of the following 30. The world's tallest Jain statue is located in which state of
country? India?
(a) Iran (b) Oman (a) Maharashtra (b) Gujarat
(c) Saudi Arabia (d) Afghanistan (c) Karnataka (d) Uttar Pradesh
(e) None of these (e) None of these
22. "Mission Bhagiratha" is the prestigious water grid project 31. Which is the India's first digital state?
in which state of India? (a) Odisha (b) Tamil Nadu
(a) Telangana (b) Uttarakhand (c) Kerala (d) Karnataka
(c) Bihar (d) Uttar Pradesh
(e) None of these
(e) None of these
y
o
u
rs
m
SPEED TEST 101

a
245

h
b
32. The National Science Day is celebrated on which date in 41. The World Day of Social Justice is observed on which date?

o
o
India? (a) February 20 (b) March 12

b
.w
(a) March 1 (b) April 21 (c) April 23 (d) June 14

o
(c) February 28 (d) June 23 (e) None of these

rd
(e) None of these

p
42. Which state government has launched the Pucca Ghar

re
33. The NDDB foundation for Nutrition (NFN) has launched Yojana for construction workers?

s
s
the 'Gift Milk' initiative in which state? (a) West Bengal (b) Gujarat

.c
(a) Telangana (b) Bihar

o
(c) Odisha (d) Madhya Pradesh

m
(c) Uttar Pradesh (d) Odisha (e) None of these
(e) None of these 43. Which of the following states is also known as 'The land of
34. The world's first fast-acting anti-rabies drug "RMAb" will dwan lit mountains'?
be launched in which country?
(a) Manipur (b) Arunachal Pradesh
(a) China (b) France
(c) Nagaland (d) Mizoram
(c) India (d) Japan
(e) None of these
(e) None of these
44. First 'New Silk Road' train from China has completed its trip
35. Who has been conferred with the Germany's prestigious
in which city?
'Cross of the Order of Merit'?
(a) Tehran (b) Kabul
(a) Arun Joshi
(c) Islamabad (d) Ankara
(b) Narendra Singh Rathore
(e) None of these
(c) Goverdhan Mehta
(d) Sushmita Sharma 45. The Indian Institute of Petroleum and Energy (IIPE) has
been established at which of the following city?
(e) None of these
(a) Visakhapatnam (b) Kochi
36. Which company has bagged the Global CSR Excellence &
Leadership Awards in the category of Best Environment- (c) Kolkata (d) Chennai
Friendly Project? (e) None of these
(a) GVK Biosciences 46. Who has won the 2016 Formula 1 Gulf Air Bahrain Grand
(b) Dr. Reddy's Laboratories Prix?
(c) Shantha Biotechnics (a) Kimi Raikkonen (b) Lewis Hamilton
(d) Bharat Biotech (c) Nico Rosberg (d) Daniel Ricciardo
(e) None of these (e) None of these
37. Nargol port is located in which state? 47. In which of the following industry CASA ratio is used?
(a) Maharashtra (b) Gujarat (a) Merchandise Trade (b) Banking & Finance
(c) West Bengal (d) Odisha (c) Software Industry (d) Travel & Tourism
(e) None of these (e) None of these
38. The Khajuraho Dance Festival is celebrated in which state 48. Which among the following is a opposite activity of hedging
of India? in share / currency /future markets?
(a) Rajasthan (b) Uttar Pradesh (a) arbitrage (b) speculation
(c) Madhya Pradesh (d) Chhattisgarh (c) spread (d) short
(e) None of these (e) tall
39. Which of the following committee is related to rejuvenation 49. Which among the following correctly categorizes
of Godavari river? collateralized borrowing and lending obligation?
(a) Ashok Kumar committee (a) A market operation
(b) Nandan Rai committee (b) A money market instrument
(c) Rekha Somalia committee (c) A clearing system
(d) Deepak Naik committee (d) A scheme of Reserve Bank of India
(e) None of these (e) A policy initiative of Government of India
40. India's first ever Gender Park has been inaugurated in which 50. Which of the calling is the oldest joint stock Bank of India
state? (a) Allahbad Bank (b) Bank of Baroda
(a) Odisha (b) Assam (c) Patiala (d) Bank of India
(c) Kerala (d) Tripura
(e) None of these
(e) None of these
y
o
u
rs
m
246 SPEED TEST 101

a
h
b
Part-II : Quantitative Aptitude 61. Which of the following fractions is the least ?

o
o
12 1

b
DIRECTIONS (Qs. 51-69): What will come in place of the question (a) (b)

.w
119 10
mark (?) in the following questions ?

o
rd
5 4 3 4 7
(c) (d)

p
51. of of of 222 = ? 39

re
8 9 5 69

s
(e) None of these

s
(a) 42 (b) 43

.c
(c) 39 (d) 37 62. A number of points are marked on a plane and are connected

o
m
(e) None of these pairwise by a line segment. If the total number of line
52. 56% of 450 + ? = 300 segments is 10, how many points are marked on the plane ?
(a) 52 (b) 48 (a) 4 (b) 10
(c) 42 (d) 56 (c) 5 (d) 9
(e) None of these (e) None of these
63. A sum of money becomes eight times in 3 years if the rate is
53. 271.5 273.5 27?
compounded annually. In how much time, the same amount
(a) 5 (b) 7 at the same compound interest rate will become sixteen
(c) 3 (d) 2
times?
(e) None of these
(a) 6 years (b) 4 years
54. 27.06 × 25 – ? = 600
(c) 8 years (d) 5 years
(a) 76.3 (b) 76.7
(e) None of these
(c) 76.5 (d) 76.2
64. A machine is sold at a profit of 10%. Had it been sold for
(e) None of these
` 40 less, there would have been a loss of 10%. What was
7 4 the cost price ?
55. 4 2 ?
8 13 (a) ` 320 (b) ` 200
(c) ` 225 (d) ` 250
1 1
(a) 11 (b) 11 (e) None of these
3 13
65. Ram spends ` 3620 for buying pants at the rate of ` 480
4 3 each and shirts at the rate of ` 130 each. What will be the
(c) 11 (d) 11 ratio of pants to shirts when maximum number of pants are
13 8
to be bought ?
(e) None of these (a) 7 : 2 (b) 7 : 3
1 (c) 2 : 7 (d) 4 : 5
56. 84 85 82 8?
3
8 (e) None of these
66. Two trains each of 120 m in length, run in opposite directions
(a) 7 (b) 2
with a velocity of 40 m/s and 20 m/s respectively. How long
(c) 3 (d) 4
will it take for the tail ends of the two trains to meet each
(e) None of these
other during the course of their journey ?
57. – (a – b) × ? = b – a (a) 20 s (b) 3 s
(a) – 1 (b) 1 (c) 4 s (d) 5 s
(c) – a (d) a (e) None of these
(e) None of these 67. Ramesh is twice as good a workman as Sunil and finishes a
58. (a + b) = ? × (– a – b) piece of work in 3 hours less than Sunil. In how many hours
(a) 1 (b) – a they together could finish the same piece of work ?
(c) – 1 (d) –b
(e) None of these 1
(a) 2 (b) 2
59. |? + 14| = 11 3
(a) –3 (b) –25 2
(c) 25 (d) 3 (c) 1 (d) 8
3
(e) Either – 3 or –25
(e) None of these
60. 16 + 26 × 2 =?
68. Fifteen years hence, a man will be four times as old as he
(a) 84 (b) 44
was fifteen years ago. His present age is:
(c) 40 (d) 832
(a) 25 years (b) 20 years
(e) None of these
(c) 30 years (d) 45 years
(e) None of these
y
o
u
rs
m
SPEED TEST 101

a
247

h
b
69. The floor of a rectangular room is 15 m long and 12 m wide.

o
3 1 1

o
The room is surrounded by a vrandah of width 2 m on all its 80. 1 2 7 ?

b
5 7 3

.w
sides. The area of the vrandah is :
(a) 124 m2 (b) 120 m2 (a) 17 (b) 3

o
rd
(c) 108 m 2 (d) 58 m2 (c) 9 (d) 29

p
re
(e) None of these (e) 25

s
DIRECTIONS (Qs. 70-74) : Find the next term in the given series 81. The height of four boys is recorded as 142 cms., 156 cms.,

s
.c
in each of the questions below. 162 cms. and 178 cms. What is the average height of all four

o
m
70. 2, 4, ?, 16, 32 boys?
(a) 6 (b) 10 (a) 160 cms. (b) 158.5 cms.
(c) 8 (d) 12 (c) 159.5 cms. (d) 162 cms.
(e) None of these (e) None of these
71. 0, 7, 26, ?, 124, 215 82. Pratul's monthly income is onefourth of Manoj's monthly
(a) 37 (b) 51 income. Manoj's annual income is ` 2.16 lacs. What is
(c) 63 (d) 88 Pratul's annual income? (In some cases monthly income and
(e) None of these in some cases annual income are used.)
72. 4, 15, 16, ?, 36, 63, 64
(a) ` 54.000 (b) ` 5.4 thousand
(a) 25 (b) 30
(c) ` 4.500 (d) ` 45.000
(c) 32 (d) 35
(e) None of these
(e) None of these
73. 1, 8, 9, ?, 25, 216, 49 DIRECTIONS (83-85) : What will come in place of the question
(a) 60 (b) 64 mark (?) in the following number series?
(c) 70 (d) 75 83. 17 98 147 172 181 (?)
(e) None of these (a) 180 (b) 192
74. 336, 210, 120, ?, 24, 6, 0 (c) 184 (d) 182
(a) 40 (b) 50 (e) None of these
(c) 60 (d) 70 84. 11 19 31 47 67 (?)
(e) None of these
(a) 80 (b) 81
DIRECTIONS (Qs. 75-82): Find out the approximate value which (c) 86 (d) 96
is closest to the value that should replace the questions mark (?)
(e) None of these
in the following questions. (You are not expected to find out the
85. 748 460 316 244 208 (?)
exact value.)
(a) 180 (b) 190
75. 1223.9975 ? (c) 172 (d) 182
(a) 110 (b) 144 (e) None of these
(c) 34 (d) 12.55
(e) 125 DIRECTIONS (Q. 86-90): Study the tables carefully to answer
76. 503 × 201 = ? the questions that follow:
(a) 101100 (b) 1000000 Number of candidates (in lakhs) appearing in an entrance
(c) 110000 (d) 100003 examination from six different cities and the ratio of candidates
passing and failing the same
(e) 1000103
77. 1205 2.5 = ? City A B C D E F
(a) 3000 (b) 4800 Number of
1.25 3.14 1.08 2.27 1.85 2.73
(c) 300 (d) 480 Candidates
(e) 500 Ratio of candidates passing and failing within the city.
78. 22020 0.011 = ?
City Passing Failing
(a) 20020 (b) 2002000
A 7 3
(c) 200200 (d) 20002
B 5 3
(e) 2000020
C 4 5
79. 20800 ? D 1 3
(a) 12 (b) 120 E 3 2
(c) 140 (d) 102 F 7 5
(e) 1020
y
o
u
rs
m
248 SPEED TEST 101

a
h
b
86. What is the ratio of number of candidates failing the exam 96. If the value of x + y = 18 and xy = 72, what is the valueof (x)2

o
+ (y)2 ?

o
from City D to that of those failing the exam from City A?

b
(a) 324 (b) 54

.w
(a) 289 : 42 (b) 42 : 289
(c) 227 : 50 (d) 50 : 227 (c) 180 (d) Cannot be determined

o
rd
(e) None of these (e) None of these

p
87. The number of candidates appearing for the exam from City 97. Rashmi obtained a total of 484 marks out of 750 in an

re
examination. What is her approximate percentage in the

s
C is what per cent of the number of candidates appearing

s
examination?

.c
for the exam from City B? (rounded off to the nearest integer)
(a) 61 (b) 56

o
(a) 27 (b) 34

m
(c) 72 (d) 65
(c) 42 (d) 21 (e) 70
(e) 38 98. What approximate amount of compound interest can be
88. The number of candidates passing in the exam from City F is obtained on an amount of `5000 at the rate of 3% per annum
what per cent of the total number of candidates appearing at the end of 3 yr?
from all the cities together ? (rounded off to two digits after (a) `482 (b) `464
the decimal). (c) `450 (e) `425
(a) 12.93 (b) 14.46 (e) `478
(c) 10.84 (d) 11.37 99. Girish attends to 7 customer calls daily. Approximately, how
(e) None of these many calls would he attend to in the span of two months?
(a) 8400 (b) 420
89. Which city has the highest number of students failing the
(c) 217 (d) 7012
entrance exam?
(e) 336
(a) F (b) C 100. Two number are such that the sum of twice the first number
(c) B (d) D and thrice the second number is 126 and the sum of thrice
(e) None of these the first number and twice the second number is 144. What
90. What is the number of candidates passing the exam from is the smaller number?
City E? (a) 18 (b) 24
(a) 13,000 (b) 11,10,000 (c) 32 (d) 36
(c) 1,13,000 (d) 11,000 (e) None of these
(e) None of these
91. Sitaram invested an amount of `7450@ 6% per annum rate Part-III : Reasoning Ability
of simple interest. After how many years will he obtain the 101. In a certain code language ‘tree is very beautiful’ is written
total amount of `8791? as ‘ka na da ta’ and ‘this is stong tree’ is written as ‘na pa sa
(a) 5 yr (b) 3 yr ka’. How is beautiful written in that code language?
(c) 2 yr (d) 6 yr (a) da (b) ta
(e) None of these (c) sa (d) Data inadequate
92. The difference between 36% of a number and 12% of the (e) None of these
same number is 82.32. What is the number? 102. In a certain code ‘GIVE’ is written as 51@© and ‘FAIL’ is
(a) 324 (b) 382 written as ‘% 219’. How is LEAF written in that code?
(c) 343 (d) 336 (a) 5©2% (b) 9©2%
(e) None of these
(c) 9@2% (d) 9©1%
93. The cost of 16 watches and 21 calculators is 30480. What is
(e) None of these
the cost of 32 watches and 42 calculators?
(a) `60970 (b) `60480 103. The positions of the first and the sixth digits in the number
(c) `80960 (d) Cannot be determined 5109238674 and interchanged, similarly the positions of the
(e) None of these second and the seventh digits are interchanged and so on.
94. If (66)2 added to the square of a number, the answer so Which of the following will be the third digit from the right
obtained is 4840. What is the number? end after the rearrangement?
(a) 32 (b) 28 (a) 9 (b) 0
(c) 24 (d) 18 (c) 6 (d) 3
(e) None of these (e) None of these
95. A sum of money is divided amongst A, B and C in the ratio 104. Four of the following five are alike in a certain way and so
of 3 : 4 : 5. Another amount is divided amongst P and Q in form a group. Which is the one that does not belong to that
the respective ratio of 2 : 1. If Q got `1050 less than B, what group?
is the amount received by C? (a) Wheel (b) Tyre
(a) `2850 (b) `1000 (c) Car (d) Door
(c) `1840 (d) Cannot be determined
(e) Gear
(e) None of these
y
o
u
rs
m
SPEED TEST 101

a
249

h
b
105. In a certain code SUBSTANCE is written as RATRUFDOB. DIRECTIONS (114-118): In the following questions, the symbols

o
o
How is TENTHOUSE written in that code? @, S, , © and # are used with the following meaning as

b
.w
(a) SMDSIFTVP (b) UOFUIDRTN illustrated below:

o
(c) UOFUIFTVP (d) SMDSIDRTN ‘P Q’ means ‘P is neither greater than nor smaller than Q’.

rd
(e) None of these

p
‘P S Q’ means ‘P is neither greater than nor equal to Q’.

re
DIRECTIONS (106-108): Following questions are based on the ‘P @ Q’ means ‘P is not smaller than Q’.

s
s
five three digit numbers given below:

.c
‘P © Q’ means ‘P is not greater than Q’.
519 364 287 158 835

o
‘P # Q’ means ‘P is neither smaller than nor equal to Q.’

m
106. If the positions of the first and the third digits within each
number are interchanged, which of the following will be the Now is each of the following questions assuming the given
third digit of the second lowest number? statements to be true. Find which of the three conclusions I, II
(a) 9 (b) 4 and III given below them is/are definitely true and give your answer
(c) 7 (d) 8 accordingly.
(e) 5 114. Statements: W @ T, T © M, M S D
107. If the positions of the first and the third digits within each Conclusions: I. W # D
number are interchanged, which of the following will be the II W @ M
middle digit of the second highest number? III. D # T
(a) 1 (b) 6 (a) Only I is true
(c) 8 (d) 5
(b) Only II is true
(e) 3
(c) Only III is true
108. Which of the following is the difference between the second
digits of the highest and the lowest of these numbers? (d) Only II and III are true
(a) 3 (b) 1 (e) None of these
(c) 2 (d) 0 115. Statements: F R, R © M, M S D
(e) None of these Conclusions: I. D # R
DIRECTIONS (109-113) : Study the following information II. D # F
carefully and answer the questions given below: III. M @ F
A, M, P, D, Q, R, W and B are sitting around a circle facing at the (a) Only I and II are true
centre. D is fourth to the left of A who is third to the right of M. P (b) Only I and III are true
is third to the left of Q who is third to the left of M. R is third to the (c) Only II and III are true
right of W who is second to the right of B.
(d) All I, II and III are true
109. Who is second to the left of D?
(e) None of these
(a) W (b) B
(c) Q (d) Data inadequate 116. Statements: V © M, M B, B S F
(e) None of these Conclusions: I. F # M
110. Who is third to the left of P? II. B @ V
(a) M (b) D III. F # V
(c) R (d) Data inadequate (a) Only I and II are true
(e) None of these (b) Only II and III are true
111. Who is to the immediate right of Q? (c) Only I and III are ture
(a) W (b) D (d) All I, II and III are true
(c) B (d) Data inadequate (e) None of these
(e) None of these
117. Statements: D # N, N @ B, B F
112. Which of the following pairs represents the first and second
Conclusions: I. F S D
respectively to the right of W?
II. N # F
(a) DM (b) QB
III N F
(c) MR (d) Data inadequate
(e) None of these (a) Only I is true
113. In which of the following pairs is the second person sitting (b) Only II is true
to the immediate right of the first person? (c) Only III is true
(a) MD (b) RM (d) Only either II or III is true
(c) AB (d) QB (e) Only I and either II or III are true
(e) None of these
y
o
u
rs
m
250 SPEED TEST 101

a
h
b
118. Statements: R S T, T # K, K @ M Conclusions :

o
o
Conclusions: I. R S M I. No bag is key.

b
.w
II. T # M II. Some bags are keys.

o
III. R S K III. Some toys are keys.

rd
(a) None follows (b) Only I follows

p
(a) None is true (b) Only I is true

re
(c) Only II follows (d) Only III follows
(c) Only II is true (d) Only III is true

s
(e) Only I and II follow

s
.c
(e) Only II and III are true 125. Statements :

o
m
DIRECTIONS (Qs. 119-123): Study the following arrangement Some days are nights.
carefully and answer the questions given below: Some nights are months.
W3# R@ EJ KT 4B 91 DU8 1 H% AV 5 7MP 2Q S6 Some months are years.
119. Which of the following is the sixth to the right of the twenty Conclusions :
first from the right end of the above arrangement? I. Some years are nights.
(a) 8 (b) D II. Some months are days.
(c) P (d) @ III. No year is night.
(e) None of these (a) Only I follows (b) Only II follows
120. How many such vowels are there in the above arrangement (c) Only III follows (d) Only either I or II follows
each of which is immediately preceded by a symbol and (e) None of these
immediately followed by a consonant? 126. Statements :
All cycles are tyres.
(a) None (b) One
Some tyres are wheels.
(c) Two (d) Three
All wheels are buses.
(e) Four
Conclusions :
121. How many such symbols are there in the above arrangement,
I. Some buses are tyres.
each of which is immediately followed by a number but not
II. Some wheels are tyres.
immediately preceded by a number?
IIi. Some buses are cycles.
(a) None (b) One (a) Only I and II follow (b) Only I and III follow
(c) Two (d) Three (c) Only II and III follow (d) All I, II and III follow
(e) more than three (e) None of these
122. How many such consonants are there in the above 127. Statements :
arrangement, each of which is immediately preceded by a Some dogs are cats.
number and immediately followed by a symbol? Some cats are horses.
(a) None (b) One All horses are tigers.
(c) Two (d) Three Conclusions :
(e) More than three I. Some tigers are cats.
123. Four of the following five are alike in a certain way based on II. Some horses are dogs.
their positions in the above arrangement and so form a group. III. Some tigers are dogs.
Which is the one that does not belong to that group? (a) None follows (b) Only I follows
(a) 8 1 D (b) 7 5 (c) Only II follows (d) Only III follows
(c) P 2 7 (d) E J R (e) Only II and III follow
(e) T 4 J 128. Statements :
All ropes are sticks.
DIRECTIONS (124-128) : In each of the questions below are
given three statements followed by three conclusions numbered Some sticks are hammers.
I, II and III. You have to take the given statements to be true even Some hammers are lakes.
if they seem to be at variance from commonly known facts. Read Conclusions :
all the conclusions and then decide which of the given conclusions I. Some lakes are ropes.
logically follows from the given statements disregarding
II. Some hammers are ropes.
commonly known facts.
III. Some lakes are sticks.
124. Statements :
(a) None follows (b) Only I follows
All keys are locks.
(c) Only II follows (d) Only III follows
No lock is toy.
(e) Only I and III follow
All bags are toys.
y
o
u
rs
m
SPEED TEST 101

a
251

h
b
DIRECTIONS (129 – 130) : In each of the questions given below 132. Which of the following statement strengthens the

o
o
which one of the five answer figures on the right should come conclusion of paragraph?

b
.w
(a) Political parties should patch-up the disputes amongst
after the problem figures on the left, if the sequence were
them.

o
continued?

rd
(b) Election disputes should be disposed the parliament

p
129. Problem Figures or concerned house of states.

re
(c) The supreme Court should establish a special judicial

s
C C S S =

s
.c
Z S C = S authority for the disposal of election petitions.

o
S Z = C Z (d) Existing system is appropriate for the disposal of

m
= = Z Z C election-petitions.
Answer Figures (e) None of these
133. Which of the following statement weakens the conclusion
= = S S = of the paragraph?
S Z = = Z (a) The government should follow the high level
Z S C C S committees and eminent persons.
C C Z Z C (b) The government should follow the recommendation
(a) (b) (c) (d) (e) of NCRWC.
130. Problem Figures (c) The new system may be unsuccessful so, the
C = = C government should continue the existing system.
(d) Fast disposal of election petitions is necessary.
S = C (e) None of these
= C = 134. In a certain code OMNIBUS is written as SUBINMO. How
Answer Figures is TROUBLE written in that code?
(a) ELBUORT (b) ELRTBOU
D CD D C = (c) EBOURLT (d) ELBTROU
=C = = =C (e) None of these
C 135. How many such pairs of letters arethere in the word
(a) (b) (c) (d) (e) CORPORATE each of which has as many letters between
them in the word as in the English alphabet?
DIRECTIONS (Qs. 131-140) : Study the following paragraph (a) None (b) One
and answer the questions that follow : (c) Two (d) Three
Election petitions in India are at present to be filed in the (e) More than three
High Court. Under the Representation of the People Act, such 136. If it is possible to make only one meaningful word with the
petitions should be disposed of within a period of 6 months. In first, second, fifth, and sixth letters of the word
actual practice however, such petitions remain pending for years 'EDUCATION', which would be the last letter of the word?
and in the meanwhile, even the full term of the House expires thus If more than one such word can be formed, give X as the
rendering the election petition infructuous. There have been answer. If
suggestions from other high level committees and eminent persons no such word can be formed, give K as your answer.
that a separate judicial set-up may be required. The National (a) T (b) X
Commission to Review the Working of the Constitution (NCRWC) (c) A (d) E
recommended that special election benches should be constituted
(e) K
in the High Courts earmarked exclusively for the disposal of
137. Rohan walks a distance of 10 km towards North, then turns
election petitions.
to his left and walks 20 km. He again turns left and walks 10
131. Which of the following conclusion which can be drawn
km and then, he takes a right turn and walks 5 km. How far
from the facts stated in the above paragraph?
he is from the starting point?
(a) The election commission should itself. make the
(a) 10km (b) 20 km
disposal of election disputes.
(c) 30 km (d) 25 km
(b) The election commission should hand-over the
(e) 15 km
disputes to an authority.
138. If 'a' means '–', 'B' means '×', 'A' means ' 'and 'F' means '+',
(c) The election petitions remain pending for years while
then-
it should be disposed within six years.
(d) To fast disposal of election petitions a separate judicial 12 B 20 A 4 Q 10 F 30 = ?
set-up should be made or special benches should be (a) 60 (b) 80
constituted in the High Courts. (c) 70 (d) 90
(e) None of these (e) None of these
y
o
u
rs
m
252 SPEED TEST 101

a
h
b
139. The positions of the first and eighth digits of the number (a) repeater (b) router

o
o
92753481 are interchanged. Similarly, the positions of second (c) gateway (d) switch

b
.w
and the seventh digits are interchanged and so on, which of (e) None of these
the following will be the fifth digit to the left of the second

o
150. Something which has easily understood instruction is said

rd
digit from the right end after the rearrangement? to be

p
(a) 8 (b) 2

re
(a) user friendly (b) information

s
(c) 4 (d) 1 (c) word processing (d) icon

s
.c
(e) 3 (e) None of these

o
140. What should come next in the following letter series?

m
Z Y X W V U T Z Y X W V U Z YX W V Z Y X W Part-IV : English Language
(a) V (b) Y
(c) Z (d) U DIRECTIONS (Q. 151-165): Read the following passage carefully
(e) None of these and answer the questions given below it. Certain words/ phrases
141. Most of the commonly available personal computers / have been printed in bold to help you locate them while answering
laptops have a keyboard popularly known as some of the questions.
(a) QWERTY (b) QOLTY Long ago there was a poor Brahmin named Krishnan. He
(c) ALTER (d) UCLIF could not find enough work to do. Sometimes, he and his family
(e) None of these had to go without food. At last Krishnan decided to leave his
142. What are the default file extension for all word documents ? village in search of work. Early next morning, he left the house. He
walked the whole day until he came to a thick jungle. He was tired,
(a) WRD (b) TXT
thirsty and hungry. While looking around for water to drink, he
(c) DOC (d) FIL
found a well. He went to the well and looked in. There he saw a
(e) None of these jaguar, a monkey, a snake and a man. They had all fallen into the
143. Manipulating data to create information is known as well. “O, noble Brahmin”, the jaguar called out to him, “Please
(a) feedback (b) programming help me out, so that I can go back to my family.”
(c) processing (d) analysis “But you are a jaguar”, said Krishnan. “How do I know you
(e) None of these will not kill me?” “Don’t be afraid of me, I promise I will not do you
144. A _________ is an organised colection of data about a any harm”, replied the jaguar. Krishnan reached into the well and
single entity. pulled out the jaguar. The jaguar thanked him and said, “I’m
(a) file (b) library Shersingh. I live in a cave in the mountains. I shall be most delighted
if I can repay my debt to you someday.” Krishnan then heard the
(c) database (d) dictionary
monkey calling out to him from the well. The Brahmin at once
(e) None of these pulled the monkey out. The monkey thanked the Brahmin. “If you
145. The horizontal and vertical lines on a worksheet are called are ever in need of food, just drop in at my place below that big
(a) cells (b) sheets mountain. Bali is my name.” Now the snake called out to him for
(c) blocking (d) gridlines help. “Help you!” exclaimed Krishnan. “You are a snake. What if
(e) None of these you bite me?” “I shall never bite you”, said the snake. So Krishnan
146. _________ is when the computer is turned on and the pulled the snake out of the well. The snake said, “Remember, if
operating system is loading. you are ever in any difficulty, just call out my name-Naagesh, and
wherever you are, I shall find you.” The jaguar, the monkey and
(a) Booting (b) Flashing
the snake took leave of the Brahmin. But before they left, they
(c) Tracking (d) Taping
spoke to him about the man in the well. “Please do not help him,”
(e) None of these said Shersingh. “If you do”, said Naagesh, “you will be in trouble
147. To print a document, press ________ then press ENTER yourself.” As soon as they left, the man in the well began to call
(a) SHIFT + P (b) CTRL + P out for help. Krishnan felt sorry for the man and pulled him out of
(c) ALT + P (d) ESC + P the well. “Thank you for your kindness”, said the man. “I am Seth
(e) None of these Ghanshyamdas. I am a goldsmith. If you ever need my help, don’t
148. Which of the following are valid Min. & Max. zoom sizes in hesitate to visit my humble house near the city.” The goldsmith
MS Office then left for home.
(a) 10, 100 (b) 20, 250 After some time, the Brahmin continued his journey. But he
could not find any work. He then remembered Shersingh, Bali,
(c) 10, 500 (d) 10, 1000
Naagesh and Seth Ghanshyamdas. He thought it was time to
(e) None of these seek their help. He first went to Bali. The monkey was overjoyed
149. If you wish to extend the length of the network without to see him. He gave him a warm welcome and offered him some
having the signal degrade, you would use a really delicious fruits. The Brahmin told him how grateful he was.
y
o
u
rs
m
SPEED TEST 101

a
253

h
b
Now Krishnan went to see Shersingh, the jaguar. As soon as (c) As his village people had asked him to leave their village

o
o
Shersingh saw Krishnan coming, he ran out to welcome him. He and look for work somewhere else.

b
.w
gave Krishnan a beautiful gold necklace and other precious (d) As he wanted to search for food in a village different
from his own.

o
jewellery. Krishnan thanked Shersingh for the jewellery and

rd
departed. His journey had at last brought him luck, he thought. He (e) None of the above

p
would be able to sell the ornaments for a good price. But who could 152. Why did the jaguar, the monkey and the snake tell Krishnan

re
not to save the man in the well?

s
help him to sell the ornaments? He then remembered Seth

s
.c
Ghanshyamdas. He went to him. The goldsmith was glad to see (a) As the man in the well was a goldsmith

o
Krishnan. “I have come to ask for your help”, said Krishnan. “Here (b) As the man in the well had cheated the snake, the

m
are some ornaments. Please give me a good price for them.” Seth monkey and the jaguar
Ghanshyamdas took the jewellery and examined it carefully. “I shall (c) As the man in the well was a thief
certainly help you”, he said. “But let me show them to another (d) As the snake, the monkey and the jaguar hated the man
goldsmith. Please wait here, I will be right back.” He then went out as they had known him for a very long time
with the ornaments. Seth at once rushed to the Palace of the King. (e) None of the above
He said, “A man brought these ornaments to me and asked me to 153. Why was krishnan afraid to save Naagesh from the well?
sell them. But they are the ornaments I made for the Prince who is (a) As Naagesh had threatened him with dire
missing.” “Who is this man? Where is he?”, thundered the King. consequences.
This rogue must have murdered my little Prince and robbed his (b) As he thought Naagesh would eat him.
jewels!” “He is a Brahmin named Krishnan, your Majesty”, replied (c) As he thought Naagesh would bite him once he was
the goldsmith, and he is there, in my house. The king called for his out of the well.
most dreaded soldiers. “Arrest the Brahmin who is in the goldsmith’s (d) As he thought that Naagesh would capture him as soon
house and throw him into the darkest dungeons of the kingdom”, as he got out of the well.
roared the King. The King’s guard stormed into the goldsmith’s (e) None of the above.
house and seized Krishnan. Krishnan was thrown into a dark 154. Why did Krishnan go to meet Seth Ghanshyamdas?
dungeon to await his execution. He then remembered the words of (a) As he thought that Seth Ghanshyamdas could help
Naagesh, the snake. So he called out to him. him in selling the ornaments gifted to him by Shersingh.
(b) As he knew that Seth Ghanshyamdas had contact with
Suddenly, almost like magic, Naagesh slithered his way
the King which could prove to be beneficial.
down a narrow window into the dingy cell. “O, Lord!” hissed
(c) As Seth Ghanshyamdas had requested krishnan to sell
Naagesh, “how did you manage to get yourself arrested?”
ornaments only to him
Krishnan cried and then told the snake what had happened. “I
(d) As Krishnan was extremely fond of Seth Ghanshyamdas
have a plan”, hissed Naagesh. “I shall creep into the Queen’s
(e) None of the above.
room and bite her”, said Naagesh. “She will faint. No matter what
155. What did Bali do after seeing Krishnan at his house?
they do, she will remain asleep. The poison will remain in her
(1) He gave Krishnan directions to Shersingh’s house.
body until you place your hand on her forehead”, explained
(2) He welcomed Krishnan to his house.
Naagesh. He then left Krishnan and went to the palace. He crept
(3) He offered tasty fruits to Krishnan.
into the Queen’s room and bit her. The Queen fainted. The sad
(a) Only 1 (b) Only 2
news that the Queen had been bitten by a snake spread all over
(c) Only 3 (d) Only 2 and 3
the Kingdom. Vaidyas came from far and near, but their medicines
(e) 1 and 3
had no effect. No one could revive the Queen. Finally, the King
156. What plan did Naagesh have to save Krishnan from the
declared that anyone who could cure the Queen would be
dungeon?
handsomely rewarded. Many people went to the palace but all of
(a) That he would sneak Krishnan out of the dungeon
them failed. “I can cure the Queen”, Krishnan told the guards. At
without anyone noticing
once they took him to the Queen. Krishnan sat beside the Queen
(b) That he would bite the King and make him unconscious
and placed his hand on her forehead. Soon, she opened her eyes
(c) That he would bite Krishnan and make everyone believe
and sat up. The King was overjoyed and shed tears of happiness.
that he was dead
He embraced Krishnan and thanked him. “Your Majesty”, said
(d) That he would enter the Queen’s chamber and scare her
Krishnan. “I was sent to prison for a crime I did not commit.”
(e) None of the above
Krishnan told the King the whole story. The King was fuming 157. What did Seth Ghanshyamdas tell the King about Krishnan?
with rage when he heard what the goldsmith had done. He at once (a) That Krishnan had brought fake ornaments for selling
had the goldsmith arrested. The King then presented Krishnan (b) That krishnan was an honest Brahmin who had left his
with a large house and a thousand pieces of gold. Krishnan sent
village
for his family and they all lived happily ever after.
(c) That Krishnan had killed the Prince
151. Why did Krishnan decide to leave his village?
(d) That Krishnan had brought those ornaments for selling
(a) As he could not find much work in his own willage and
his family had to starve sometimes because of it. which had been made for the missing Prince
(b) As his family had requested him to do so. (e) None of the above
y
o
u
rs
m
254 SPEED TEST 101

a
h
b
158. What did the King do on learning the truth about Krishnan

o
DIRECTIONS (Q. 166-170): Which of the phrases (a), (b), (c)

o
and Seth Ghanshyamdas?

b
and (d) given below each sentence should replace the phrase

.w
(a) He put Krishnan back in the dungeon as he still held
printed in bold in the sentence to make it grammatically correct?

o
Krishnan responsible for the Prince’s death

rd
If the sentence is correct as it is given and no correction is
(b) He called for Krishnan’s wife and family

p
required, mark (e) as the answer.

re
(c) He presented gold to Krishnan and also a house to live in

s
166. A young and successful executive was travelling down a

s
(d) He congratulated the snake on his efforts to save

.c
neighbourhood street, going a bit to fast in his new car.

o
Krishnan

m
(a) a bit too fastly (b) a bit as fast
(e) None of the above
(c) a bit to fastly (d) a bit too fast
159. What did the King do to save the Queen after even the
(e) No correction required
Vaidyas failed to revive her? 167. All she could think about was the beautiful dress and how
(a) He punished the snake for having harmed the Queen she could earn enough money to buy it.
(b) He announced a reward to anyone who could cure the (a) All she can think (b) All she could thought
Queen (c) All she can thought (d) All she can thinking
(c) He immediately called for Krishnan to cure the Queen (e) No correction required
(d) He asked his guards to immediately look for someone 168. He told his employer of his plans to leave the business to
who could cure the Queen lead a more leisure life.
(e) None of the above (a) more leisurely life (b) many leisurely life
160. What can possibly be the moral of the story? (c) many leisured life (d) more leisurely live
(a) Trust oneself before trusting overs (e) No correction required
(b) A good deed never goes in vain 169. Padma could convince anyone with her talk as she had the
(c) You cannot change people but you can change yourself gift of the gabbing.
(a) gifting of the gabbing (b) gift of the gab
(d) Try and try until you succed
(c) gifting of the gab (d) gift of the gab
(e) One must be the change one wishes to see in this world
(e) No correction required
DIRECTIONS (Q. 161-163): Choose the word/group of words 170. For countries undergoing a recession, large cuts in public
which is most similar in meaning to the word/group of words spending seem to be the ordering of the day.
printed in bold as used in the passage. (a) be the ordering of days
(b) being the order of the day
161. GO
(c) be the order of the day
(a) report (b) live
(d) being the ordering of days
(c) send (d) leave
(e) depart (e) No correction required
162. HUMBLE DIRECTIONS (Q. 171-175): In each question below, a sentence
(a) elegant (b) polite with four words printed in bold type is given. These are numbered
(c) modest (d) real as (a), (b), (c) and (d). One of these four words printed in bold may
(e) vast be either wrongly spelt or inappropriate in the context of the
163. SEEK sentence. Find out the word which is wrongly spelt or
(a) hunt for (b) watch for inappropriate, if any. The number of that word is your answer. If
(c) mention (d) ask for all the words printed in bold are correctly spelt and also
(e) force appropriate in the context of the sentence, mark (e) ie. ‘All
DIRECTIONS (Q. 164-165): Choose the word/group of words correct’ as your answer.
which is most opposite in meaning to the word/group of words 171. The whole (a)/ time she walked with her child in her arms,
printed in bold as used in the passage. the only thing (b)/ that worried (c)/ her was her son’s
164. CONTINUED feature. (d)/ All correct (e)
(a) remanded (b) presented 172. When the young artist returned (a)/ to his village, his family
(c) rested (d) carried on held a festive (b)/ dinner on its lawn to celebrate his
(e) stopped triumpant (c)/ homecoming. (d)/ All correct (e)
165. HANDSOMELY 173. Had she not suppressed (a)/ all the details of her Company’s
(a) Meagrely (b) tidily project (b)/ her Company would have bagged (c)/ the
(c) ugly (d) raggedly contract. (d)/ All correct (e)
(e) plenty
y
o
u
rs
m
SPEED TEST 101

a
255

h
b
174. She trusted Mira with all her heart (a)/ and thus handled 183. (a) more (b) scene

o
o
(b)/ over her life’s (c)/ savings to her instantly. (d)/ All (c) whole (d) last

b
.w
correct (e). (e) lucky

o
175. It is difficullt (a)/ to see the picture (b)/ when you are inside

rd
184. (a) servants (b) mother
(c)/ the frame. (d)/ All correct (e)

p
(c) computers (d) relatives

re
s
DIRECTIONS (Q. 176-190): In the following passage there are (e) man

s
.c
blanks, each of which has been numbered. these numbers are 185. (a) minds (b) selves

o
m
printed below the passage and against each, five words are (c) property (d) pillars
suggested, one of which fits the blank appropriately. Find out the
(e) country
appropriate word in each case.
One day a father of a very wealthy family (176) his son on a DIRECTIONS (Qs. 186-190): In each of the following sentences,
trip to the country with the purpose of (177) his son how the poor an idiomatic expression or a proverb is highlighted. Select the
people live so he could be thankful for his wealth. They spent a alternative which best describes its use in the sentence.
(178) of days and nights on the farm of what would be considered
186. He resigned the post of his own accord.
a (179) poor family. On their (180) from the trip, the father asked
(a) which he liked
his son, “How was the trip?" "It was great, Dad.” “Did you see
how poor people can be?”, the father asked. “Oh yeah”, said the (b) according to his convenience
son. So what did you (181) from the trip?”, asked the father. The (c) voluntarily and willingly
son answered, “I saw that we have one dog and they had four. (d) according to his judgement
We have a pool that (182) in the middle of our garden and they (e) None of these
have a creek that has no end.” “We have imported lanterns in our
187. As a politician he is used to being in the limelight all the
garden and they have the stars at night.” “Our patio reaches to
time.
the front yard and they have the (183) horizon.” “we have a small
(a) giving speeches
piece of land to live on and they have fields that go beyound our
sight.” “We have (184) who serve us, but they serve others.” (b) the object of admiration
“We buy our food, but they grow theirs.” “We have walls around (c) the centre of attraction
our (185) to protect us; they have friends to protect them.” (d) an object of public notice
With this the boy’s father was speechless. Then his son (e) None of these
added, “Thanks dad for showing me how poor we are”. 188. I ran out of money on my European tour.
176. (a) took (b) beat
(a) exhausted my stock of
(c) drag (d) mould
(b) did not have enough
(e) showed
(c) lost
177. (a) presenting (b) requesting
(d) carried a lot
(c) tell (d) trusting
(e) showing (e) None of these
178. (a) two (b) couple 189. Madhuri might scream blue murder, but I feel Deepali
(c) much (d) few should get the promotion since she is better qualified for
the job.
(e) many
179. (a) major (b) some (a) someone has been murdered with some blue liquid
(c) sorrow (d) very (b) someone is being murdered and has become blue
(e) astutely (c) suffer from persecution complex
180. (a) lane (b) journey (d) make a great deal of noise and object vehemently
(c) leave (d) return (e) None of these
(e) walking 190. In modern democratic societies lynch law seems to have
181. (a) reveal (b) think become the spheres of life.
(c) saw (d) believe (a) law of the mob
(e) learn (b) law of the underworld
182. (a) stands (b) reaches (c) law of the constitution

(c) swims (d) leak (d) law of the parliament

(e) watery (e) None of these


y
o
u
rs
m
256 SPEED TEST 101

a
h
RESPONSE SHEET

b
o
o
b
.w
o
Test Code : ............................... Time taken : ......................... Date : ..........................

rd
p
(a) (b) (c) (d) (e) (a) (b) (c) (d) (e) (a) (b) (c) (d) (e) (a) (b) (c) (d) (e)

re
s
1. 51. 101. 151.

s
.c
2. 52. 102. 152.

o
3. 53. 103. 153.

m
4. 54. 104. 154.
5. 55. 105. 155.
6. 56. 106. 156.
7. 57. 107. 157.
8. 58. 108. 158.
9. 59. 109. 159.
10. 60. 110. 160.
11. 61. 111. 161.
12. 62. 112. 162.
13. 63. 113. 163.
14. 64. 114. 164.
15. 65. 115. 165.
16. 66. 116. 166.
17. 67. 117. 167.
18. 68. 118. 168.
19. 69. 119. 169.
20. 70. 120. 170.
21. 71. 121. 171.
22. 72. 122. 172.
23. 73. 123. 173.
24. 74. 124. 174.
25. 75. 125. 175.
26. 76. 126. 176.
27. 77. 127. 177.
28. 78. 128. 178.
29. 79. 129. 179.
30. 80. 130. 180.
31. 81. 131. 181.
32. 82. 132. 182.
33. 83. 133. 183.
34. 84. 134. 184.
35. 85. 135. 185.
36. 86. 136. 186.
37. 87. 137. 187.
38. 88. 138. 188.
39. 89. 139. 189.
40. 90. 140. 190.
41. 91. 141. 191.
42. 92. 142. 192.
43. 93. 143. 193.
44. 94. 144. 194.
45. 95. 145. 195.
46. 96. 146. 196.
47. 97. 147. 197.
48. 98. 148. 198.
49. 99. 149. 199.
50. 100. 150. 200.
m
o
.c
s
s
re
p
rd
o
.w
b
o
o
b
h
a
m
rs
u
o
y
m
o
.c
s
s
re
p
rd
o
.w
b
o
o
b
h
a
m
rs
u
o
y
y
o
u
rs
m
a
h
SOLUTIONS 1

b
o
19. (c) First term = 32 – 1.

o
SPEED TEST 1

b
Second term = 33 + 1

.w
1. (a) A square is a two-dimensional figure consisting of sides Third term = 42 – 1

o
whereas a cube is a three- dimensional figure. Similarly, \ Fourth term = 43 + 1

rd
circle is a two-dimensional figure and a sphere is a 20. (b) First term = 23.

p
three-dimensional figure.

re
Second term = 32

s
B R I G H T Third term = 43

s
.c
\ Fourth term = 52

o
+1 –1 21. (d) Second term = 4 × First term

m
2. (e) \ Fourth term = 4 × Third term
J S C S G F 22. (a) Second term = (First term)2 – 1
Similarly, \ Fourth term = (Third term)2 – 1
23. (e) Second term = (First term)3
J O I N E D \ Fourth term = (Third term)3.
24. (a) Second term = First term + 1/8 First term.
+1 –1
\ Fourth term = Third term + 1/8 Third term.
25. (e)
J P K C D M 26. (c) 1 and 4 are what happens after a disease. 5 is its
3. (a) The second number is the product of the digits of the symptom but not a definite one. 2 is a probable cause.
first. 27. (c) The first is the force fighting on/in the second.
4. (d) The first is found in the form of the second. 28. (e) More of a test of your English.
5. (c) In first term, two letters are missing between first two 29. (c) Secretly is the opposite of openly, and silently is the
letters while last two are continous but in second term opposite of noisily. Choices a and b are clearly not the
first two letters are continuous and two letters are opposites of silently. (Choice d) means the same thing
missing between last two letters. as silently.
6. (a) The third letter of second term is the next letter 30. (b) A spring forms or has the shape of a coil, and a ring
according to alphabet to the third letter of first term. forms a loop.
7. (a) The letters of each group are in reverse order.
8. (d) First two letters of the first term are in reverse order in SPEED TEST 2
the second term and so are the next two letters.
1. (a) +2 -3
9. (b) Fifth and third letters of the first term are first and H ¾¾® J ¾¾® G
second letters of the second term and first two letters +1 -3
P ¾¾® Q ¾¾® N
of the first term are third and fourth letters of the second
+1 -3
term. D ¾¾® E ¾¾® B
10. (a) There is a gap of one letter between third and fourth, +1 -3
T ¾¾® U ¾¾® R
fourth and first, and first and second letters of each
+1 -3
group. K ¾¾® L ¾¾® I
11. (a) The letters in first and second terms are in reverse order 2. (e) Lotus is grown in water (Mud).
of alphabet. 3. (b) P R O B L E M
12. (e) There is a gap of one letter between each corresponding
¯ ¯ ¯ ¯ ¯ ¯ ¯
letters of 'QYGO' and 'SAIQ'
13. (d) There is a gap of three letters between each 2 9 4 8 3 7 5
corresponding letters of 'YAWC' and 'UESG'. B O R E M O E P
14. (c) There is a gap of two letters between the two ¯ ¯ ¯ ¯ ¯ ¯ ¯ ¯
consecutive letters of each term. ;
8 4 9 7 5 9 (4) 7 2
15. (e) First, Second, and third each term is one more than the
L B O R O M E P
square of prime number. Hence the fourth term
¯ ¯ ¯ ¯
= (19)2 + 1 ;¯ ¯ ¯ ¯
16. (a) First term = (6)2 + 6. 3 8 4 9 4 5 7 2
Second term = (7)2 + 7 E R O L
Third term = (10)2 + 10
¯ ¯ ¯ ¯
\ Fourth term = (11)2 + 11
17. (d) First term = 32 + 3. 7 9 4 3
Second term = 42 + 4 4. (e)
Third term = 52 + 5 5. (d) River is a water body.
\ Fourth term = 62 + 6 6. (d) Except 255 all other numbers are one more than perfect
18. (c) First term = 22 – 1 square.
Second term = 32 + 1 50 = (7)2 + 1, 65 = (8)2 + 1;
Third term = 32 – 1 170 = (13)2 + 1, 290 = (27)2 + 1
\ Fourth term = 42 + 1 But, 255 = (16)2 – 1
y
o
u
rs
m
a
h
2 101 SPEED TEST

b
o
In each pair if first letter is mth from the beginning of

o
7. (e) The number 49 is a perfect square of a natural 23. (d)

b
number. alphabet the second letter is mth from end.

.w
8. (e) Except Brinjal, all others grow underground. 24. (e) In all others,

o
rd
9. (d) All others are parts of a car. 1st letter + 1 = 4th letter.
10. (c) Except 529, all others are perfect squares of even

p
4th letter + 1 = 2nd letter

re
numbers. The number 529 is a perfect square of an odd
and 2nd letter +2 = 3rd letter

s
number.

s
.c
196 = 14 × 14; 256 = 16 × 16 25. (c) In all others

o
529 = 23 × 23; 576 = 24 × 24 4th letter + 1 = 1st letter

m
324 = 18 × 18 1st letter + 2 = 2nd letter
-2 -2 2nd letter + 1 = 3rd letter.
11. (a) R ¾¾® P ¾¾® N 26. (b) All others are synonyms.
-4 +2 27. (d) All others are synonyms.
W ¾¾® S ¾¾® U
28. (a) All others imply ‘UP’.
-4 +2
H ¾¾® D ¾¾® F 29. (c) ‘Large’ is an adjective whereas others are noun.
-4 +2
30. (d) All others are negative.
L ¾¾® H ¾¾® J
-4 +2
Q ¾¾® M ¾¾® O SPEED TEST 3
12. (b) The number 441 is a multiple of 3
1. (a) 1050 420 168 67.2 26.88 10.752
13. (e) -3 -3
P ¾¾® M; E ¾¾® B;
¸ 2.5 ¸ 2.5 ¸ 2.5 ¸ 2.5 ¸ 2.5
-3 -3
T ¾¾® Q; I ¾¾® F; 0 6 24 60 120 210 ?
2. (e)
+3
V ¾¾® Y +6 +18 +36 +60 +90 +126
14. (d) 115 = 5 × 23 ;
85 = 5 × 17; +12 +18 +24 +30 +36
95 = 5 × 19;
\ ? = 210 + 126 = 336
155 = 5 × 31;
3. (a) The pattern of the series is :
But, 75 = 5 × 15
19 – 15 = 4 = 22
One factor of 75 is not a Prime Number.
83 – 19 = 64 = 43
15. (e) Except number 345, all other numbers are product of 23
119 – 83 = 36 = 62
and a Prime Number.
631 – 119 = 512 = 83
115 = 23 × 5;
\ ? = 631 + 102 = 631 + 100 = 731
161 = 23 × 7
4. (c) The pattern of the series is :
253 = 23 × 11;
19 + 1 × 7 = 19 + 7 = 26
391 = 23 × 17 26 + 2 × 7 = 26 + 14 = 40
But 345 = 23 × 15. 40 + 4 × 7 = 40 + 28 = 68
The number 15 is not a Prime Number. 68 + 8 × 7 = 68 + 56 = 124
16. (c) -2
O ¾¾® M ¾¾® Q
+4 124 + 16 × 7 = 124 + 112 = 236
5. (d) The pattern of the number series is as given below:
-2
H ¾¾® F ¾¾® J
+4 11 × 1 – 1 = 10
10 × 2 – 2 = 18
-4
T ¾¾® P ¾¾® R
+2 18 × 3 – 3 = 51
51 × 4 – 4 = 200
-2 +4 200 × 5 – 5 = 995
T ¾¾® R ¾¾® V
\?= d
-2 +4 6. (b) The pattern of the number series is as given below:
V ¾¾® T ¾¾® X
14 + 10 = 24
17. (c) In all others, 1st letter – 1 = 2nd letter, and 2nd letter 24 + 19 (= 10 + 9) = 43
– 2 = 3rd letter. 43 + 28 (= 19 + 9) = 71
18. (a) Except Diabetes, all others are infectious diseases. 71 + 37 (= 28 + 9) = 108
19. (a) Except Mustard, all others are grains. Mustard is an 108 + 46 (= 37 + 9) = 154
oilseed. 7. (e) The pattern of the number series is as given below:
20 (d) All the numbers are multiples of 5. But 25 is a perfect 144 + 29 = 173
square. 173 – 33 = 140
21. (d) All others are synonyms 140 + 29 = 169
169 – 33 = 136
22. (c) All others are synonyms
136 + 29 = 165
y
o
u
rs
m
a
h
SOLUTIONS 3

b
o
o
8. (a) The pattern of the number series is as given below: 25. (e) The letters are in reverse order while one letter is

b
missing between two consecutive letters.

.w
656
+ 24 = 328 + 24 = 352 26. (b) There is a gap of one letter between two consecutive

o
2

rd
letters. Besides, this the letters are capital and lower
352

p
respectively.
+ 24 = 176 + 24 = 200

re
2 27. (b) m m l l

s
s
mmll

.c
200
+ 24 = 100 + 24 = 124 mmll

o
2

m
28. (e) 1, 12, 123, 1234, 12345, 123456, 123456 7
124
+ 24 = 62 + 24 = 86 29. (c) ABCD, ABCDE, ABCDEF, PQRS, PQRST, PQRST U
2
86 30. (e) N + 3 = Q, Q + 3 = Z, Z + 3 = S
+ 24 = 43 + 24 = 67 D – 2 = W, W – 2 = F, F – 2 = V
2
9. (b) The pattern of the number series is as given below: P + 3 = B, B + 3 = R, R + 3 = I Hence, ? = SVI
12 × 4 – 30 = 48 – 30 = 18
18 × 4 – 36 = 72 – 36 = 36 SPEED TEST 4
36 × 4 – 42 = 144 – 42 = 102
102 × 4 – 48 = 408 – 48 = 360
1. (e) E X C U R S I O N
360 × 4 – 54 = 1440 – 54 = 1386
10. (c) 71 78 99 134 183 246
2. (c) G O L D E N
+(7 × 1) +(7 × 3) +(7 × 5) +(7 × 7) +(7 × 9)
342 337.5 328.5 315 297 274.5 3. (a)
11. (b) S T R I V E
–(4.5× 1) –(4.5 × 2) –(4.5 × 3) –(4.5 × 4) –(4.5 × 5) 4. (b) Meaningful words are % TAN and ANT
12. (c) 161 164 179 242 497 1520 C L I E N T
+3 +15 +63 +255 +1023
5. (a)
C E I L N T
(3 × 4)+3 (15 × 4)+3 (63 × 4)+3 (255 × 4)+3 6. (c) ELAN, LEAN and LANE
13. (b) 3 + 32, 5 + 52, 7 + 72. 7. (b) Meaningful words are % ARE, ART, ATE
14. (a) 92 + 10, (19)2 + 20, (29)2 + 30 8. (a) Given word : A R G U M E N T
15. (e) 100 × 1.1, 200 × 2.2, 300 × 3.3, 400 × 4.4 Alphabetically the sequence is
16. (b) + 12 – 02, + 32 – 22, + 52 – 42, + 72 – 62.
17. (e) × 3 + 1, × 3 + 3, × 3 + 5, × 3 + 7 ........ A E G M+1 N+1 R+1 T+1 U+1
+1 +1 +1
18. (e) The given series is ¸ 2 – 6
B F H N O S U V
Reqd no. = 50 ¸ 2 – 6 = 25 – 6 = 19
19. (d) (1015 + 1) ¸ 2 = 508; 9. (d) New order of letters : GANCARROE
(508 + 2) ¸ 2 = 255; (255 + 3) ¸ 2 = 129; 10. (c) According to english alphabet, resultant group will be
(129 + 4) ¸ 2 = 66.5; (66.5 + 5) ¸ 2 = 35.75; as follows:
(35.75 + 6) ¸ 2 = 20.875
20. (a) The series is P R I N C E
× 4, ¸ 8, × 12, ¸ 16, × 20 C E I N P R
21. (d) The series is
Only two letters 'I and N' will remain unchanged.
–1.1, –2.2, –4.4, –8.8, –17.6
22. (e) The series is 11. (e) Meaningful words are : ROSE, SORE, EROS and ORES.
+112, + 122, + 132, + 142, + 152. 3 15 14 19 20 1 2 12 5
23. (d) The series is abcab, bcabc, cabca. 12. (d) C O N S T A B L E
24. (c) First two letters of each term are in reverse order.
Similarly third and fourth letters are also in reverse order. 13. (c) P H Y S I C A L
Besides this, second letter of the second term is the
next letter after the first letter of the first term. 14. (d) S H I F T E D
Second Method -
15. (b) Meaningful words are : GO and TO
–1 –1 –1 –1 –1 –1 –1 –1
16. (a) Meaningful words are : TON, TOE, TEN
D C X W, F E V U, H G T S, ...
J ...
I ...
R Q... 17. (b) According to order of alphabet
–1 –1
C E F I L M R U
+1 B D E H K L Q T
+1
+1 18. (d) Meaningful word are : TIRE, TIER and RITE.
y
o
u
rs
m
a
h
4 101 SPEED TEST

b
o
o
19. (a) D I S T A N C E 5. (b) As, Similarly,

b
+2

.w
N ¬¾¾ L ü G ¾¾® I
-2
C J R S B M B F ï +2

o
-2
A ¬¾¾ Y ï L ¾¾® N

rd
B B C F J M R S +2
X ¬¾¾ V ïï R ¾¾® T
-2

p
re
20. (d) FOLK ® FKLO ® EJKN +2
-2
A ¬¾¾ Y ï C ¾¾® E

s
21. (c) MULE = 1 + 3 + 3 + 2 = 9 ýÞ

s
+2

.c
22. (e) Meaningful words are : TEARS, STARE, RATES and -2
L ¬¾¾ J ï E ¾¾® G

o
+2
I ¬¾¾ G ï P ¾¾® R
ASTER. -2

m
23. (b) Word : WEBPAGE ï +2
-2
T ¬¾¾ R ï Y ¾¾® A
Changed word : +2
-2 ï R ¾¾® T
VFAOBFF E ¬¾¾ C þ +2
C ¾¾® E
So, F appears thrice.
6. (a) Going through information provided, we get codes for
24. (c) Words : TIPS, SPIT and PITS.
G ® 3, R ® 8, E ® 1, C ® 9.
25. (b) SKILL, KILLS Therefore, Greece will be coded as 381191.
26. (c) Original word : 7. (b) CAT ® SATC & DEAR ® SEARD
HABITUAL Clearly, the first letter is transferred to last place and S
Changed word : taken its place. In the same way SINGS would be coded
GBAJSVBK as SINGS.
So, fourth from the left is J. 8. (a) PROMISE
27. (c) Word : 1 2 3 4 567
WA LK I N G Þ MISER will be coded as 45672.
Alphabetical order : 9. (d) In the code, night is called sunshine. As we sleep in
A G I K LN W night, the correct answer is sunshine.
So, the positions of K and N remain unchanged. 10. (a) Position of D alphabetically = 4
28. (d) Clearly, we have : Position of F alphabetically = 6
COMPREHENSION ® (COM) (PREHENS) (ION) Thus D + F = 10 = position of J in alphabet.
® COMIONSNEHERP 11. (d) ‘ 2 4 7 ’ ® ‘spread red carpet ’
The middle letter is the seventh letter, which is S.
29. (d) ‘ 2 3 6 ’ ® ‘dust one carpet ’

‘ 2 3 4 ’ ® ‘ one red carpet ’


D E P R E S S I O N
Hence, ‘2’ ® ‘carpet’; ‘4’ ® ‘red’; ‘7’ ® ‘spread’;
E D R P S E I S N O ‘3’ ® ‘one’; ‘6’ ® ‘dust’.
1 2 3 4 5 6 7 8 9 10 12. (b) A person sits on a chair. Since ‘chair’ is called ‘cot’, our
answer is ‘cot’.
30. (a) Random, Restaurant, Restrict, Robber, Rocket.
O V E R V I S T
13. (a) $ # % * # + ´ –
SPEED TEST 5
1. (c) Second and fourth letters of the word PROSE are moved S O R E
two steps backwards in alphabatical order. Similarly, From above table, SORE is coded as : ´ $ * %
LIGHT can be coded as LGGFT.
14. (a) As,
2. (d) Z = 52 = 26 × 2
P U L S E and N E W
ACT = 1 × 2 + 3 × 2 + 20 × 2 = 48 [Alphabetical position
numbers has been -1 -1 -1 -1 -1 -1 -1 -1
doubled] O T K R D M D V
Þ BAT = 2 × 2 + 1 × 2 + 20 × 2 = 46 reverse order reverse order
3. (c) Here, 2 ® A, 3 ® L, 5 ® G,, 4 ® U and 9 ® T..
D R K T O V D M
Hence, 23549 will be having the code ALGUT. Similarly,
4. (a) RBM STD BRO PUS º the cat is beautiful ....(i)
P R O B E S
TNH PUS DIM STD º the dog is brown ....(ii)
-1 -1 -1 -1 -1 -1
PUS DIM BRO PUS CUS º the dog has the cat ....(iii)
(i) and (ii) Þ STD PUS º is O Q N A D R
(ii) and (iii) Þ PUS DIM º the dog reverse order
(i) and (iii) Þ PUS BRO º the cat R D A N Q O
\ From (iii), CUS º has Hence, required code : RDANQO
y
o
u
rs
m
a
h
SOLUTIONS 5

b
o
o
15. (b) As, 21. (a) As,

b
.w
R E M I T +2
D ¾¾ ®F +2
I ¾¾ ®K

o
and

rd
+2
O ¾¾ ®Q +2
N ¾¾ ®P

p
£ 3 7

re
Similarly,
and

s
s
+2

.c
C O N S U L A ¾¾ ®C

o
+2
T ¾¾ ®V

m
= % 8 b $ 5 22. (b) W R O M B T ® 7 1 9 4 8 3
Similarly, 23. (b) As,
+1
P ¾¾® and B ¾¾® C
+1
O C E L O T Q
-1 -1
I ¾¾® H O ¾¾ ®N
% = £ 5 % 7 +1
N ¾¾®
+1
O L ¾¾ ®M
16. (c) -1 -1
K ¾¾® J T ¾¾ ®S
1 2 3 4 5 4 1 3 5 2
A M O N G ® N A O G M Similarly,
1 2 3 4 5 4 1 3 5 2 +1
M ¾¾ ®N
S P I N E ® N S I E P -1
U ¾¾ ®T
Hence, +1
S ¾¾ ®T
1 2 3 4 5 4 1 3 5 2 -1
T ¾¾ ®S
L A M O N ® O L M N A
24. (b) As,
17. (a) S E A L and D O S E
L O C K
¯ ¯ ¯ ¯ ¯ ¯ ¯ ¯ +1 +1 -1 -1
$ 7 5 @ # 8 $ 7 M P B J
Hence, and

S O L D B L O W
+1 +1 -1 -1
¯ ¯ ¯ ¯
C M N V
$ 8 @ #
Similarly,
18. (a) Teacher write on blackboard with chalk, here chalk is
W I N E
called book, hence here the code of chalk is book. +1 +1 -1 -1
19. (e) As,
X J M D
M O T H E R
25. (b) SOLDIER JFSCRNK
+1 +1 +1
+1
O M H U R F +1
–1
Similarly, –1
–1
A N S W E R –1

+1 +1 Similarly,

N A W T R F GENIOUS PVTHFDM
+1
G R O W W I T H I N +1
+1
20. (a) and –1
= @ % # –1
# ÷ + © ÷ r –1
W I N G –1

26. (e) The colour of blood is red and here red means orange.
# ÷ r =
y
o
u
rs
m
a
h
6 101 SPEED TEST

b
o
o
7. (c) The only son of Vineet’s grandfather is Vineet’s

b
(27-28) : colours of the sky father. The lady is the daughter of Vineet’s father.

.w
Thus, the lady in the park is the sister of Vineet.

o
Þ ki 8. (a) As X is the son of Y’s father and Y is the sister of X and

rd
la fa so
thus, he has to be the brother of Y.

p
re
rainbow colours Þ ro ki Father

s
9. (b)

s
.c
Only Daughter
sky high rocket Þ la pe jo

o
m
Monika
the rocket world Þ pe so ne
Mother
27. (d) colours sky high Þ ki la jo
28. (c) The code of ‘the’ is ‘so’. Rahul
D R E A M I N G 10. (a)
29. (e) +1 –1 Daizy Binny Aruna
B F S E F M H L (Binny’s sister (Binny’s sister)
and Chinky’s Mother)
Similarly,
T R E A T I S E
+1 –1
B F S U D R H S
Chinky
30. (b) A person sits on a chair. Since ‘chair’ is called ‘cot’, our
answer is ‘cot’. 11. (d)
Saroj Rajesh
SPEED TEST 6 mother father
1. (c) Clearly, Arun is uncle of the lady in the photograph.
Arun — Brother Vani Deepak Ramesh
| Sister-in-law only brother
Son + Wife mother = lady
|
daugher wife

Mother — B
|
A-C 12. (c)
2. (a) A’s mother = B’s sister
Clearly, A can be a neice of B.
3. (b) The only son of Mahesh’s father is Mahesh himself.
Father of Kamla is Mahesh and Mahesh is father of 13. (c)
Kamla. Mother/Photograph
4. (d) Clearly, the grandson of Anil’s mother is son of Anil law
h er-in-
and wife of Anil’s son is daughter in-law of Anil. Thus, Mot
Anil is the father-in-law of the girl.
5. (b) Clearly, from the relationship diagram. Y is the brother- Husband Sister Man Brother Wife Son
in-law of B.
Ne

Married couple
A B
ph
ew

Son
Brother Brother-in-law
Brother
X Y
Brothers For Qs. 14 to 16
6. (c) Woman’s Mother’s husband
(Couples) (Couples)
Roma Mohan Madhu Jeevan
(Couples) (Couples)
Smita Devika Aman Krishna Sunil
Woman’s father
Woman’s father’s sister ¾¾® Woman’s Aunt. Daughter
Anuj Ankur
Since, woman’s aunt is man’s aunt Romila
\ woman is sister of man. There are in all four married couples.
y
o
u
rs
m
a
h
SOLUTIONS 7

b
o
o
14. (a) Since, Devika is wife of Aman and Krishna is sister 25. (a) R(+) V(–) S

b
of Aman, therefore, Krishna is the sister-in-law of

.w
Devika.

o
15. (d) Since, Anuj is son of Krishna and Romila is daughter

rd
T(+)
of Aman and Aman is brother of Krishna, therefore

p
re
Anuj and Romila are cousins. Hence V is the aunt of T.

s
16. (c) Since, Madhu is mother of Krishna who is wife of

s
26. (c) K(+) T(–)

.c
Sunil. Therefore, Madhu is Sunil’s mother-in-law.

o
For Qs. 17-20.

m
The information given in the question can be
summarised as: D(+) B M(+)

(Father) Wife (Mother)


For Qs. 27 to 30
E D
Adhir Mishra Mr. & Mrs. Mohan
Married
Urmila Raghu Sumit Roma Roshan Bimla Shilla
Sons (eldest) (youngest)
Wife
C B (Son)
Mr. & Mrs Sharma
Sohan Shivendar
Daughter Son/daughter
Shilla Sandeep Shaifali
F A
Leela (daughter)
17. (c) C is the mother to A and F.
18. (d) Since we do not know whether A is male or female, 27. (b) Sumit’s mother-in-law = Mrs. Mohan.
therefore (d). 28. (c) As Sohan is son of Sumit and Sumit is son of Mishra.
19. (d) We cannot tell from the given facts whether A is a male The surname of Sohan is Mishra.
or female. 29. (a) Leela is the grand daughter of Mr. Sharma. Hence, the
20. (b) The two couples in the question are DE (grandmother surname of Leela is Sharma.
and grandfather) and BC (father and mother). 30. (d) Shivendar is son of Roma. Therefore, he is grandson
21. (e) S of Roma’s father.

SPEED TEST 7
T(–) R(+) V(–)
1. (c) Clearly, I am to the north of my house.
Hence T is the daughter of S.
22. (b) V(–) 5 N

5 W E
T(–) R(+) S
10 5
23. (d) T(+) R(–)
End Point S

Starting point
V(+) S (My house)
S is either the nephew or niece of T.
2. (b) 70 m
24. (c) T(+) R(–) T(–) V(+)(–) S
100 m
B' A'
A B
V(+) S R(+) 50 m 10 30 My friend
My self 10
(1) (2)
20
S is nephew or niece of T. Absurd relationship.
When my friend reaches on the previous track
T(–) S(+) (i.e. on B') again, he had travelled a distance of
(30 + 10 + 20 + 10) = 70 m. As I walk with the same speed
R(+) V(+)
as that of my friend I have walked 70m, but on the
(3) straight track. Now, he is just [100 – (30 + 20)] = 50m
S is husband of T. from my starting point.
We need not go further. Hence, the distance between us = (70 – 50) = 20m
y
o
u
rs
m
a
h
8 101 SPEED TEST

b
o
o
3. (a) Clearly the direction of the hour’s hand is North-east. 8. (d) Left of Kunal is Atul.

b
North East of Atul is Prashant (obvious from the above

.w
H N diagrams).
2 3 4

o
9. (c) Total distance travelled = 25 + 40 + 60 + 90 = 215 mts.

rd
1 5

p
12 6 M W E 10. (d) West North

re
8 km/h
Rohan

s
s
B A

.c
S

o
m
3 N
1
90 m
W E
4. (b) 3
3
S C D
1 Rahul
Required distance = 3 + 1 = 4 km 10 km/h
South East
5. (a) Here, O is the starting point.
20
Rohan takes the route BADCB at 8 km/hr = m/sec.
9
25
4 Rahul takes the route CDABC at 10 km/hr = m/sec.
9
3 AB = BC = CD = DA = 90 m.
First time they meet when Rohan travels 120 m and
A O B Rahul travels 150 m.
3 4
For the second time, Rohan is 10 m towards B from C or
Rahul is 80 m towards C from B. Hence on BC at a
Both A and B are 32 + 42 = 5 km from the starting distance of 10 m from C.
point.
11. (a)
6. (b) Clearly the school is in north-east E

5
School N N S

40
Ram’s house W E W
Reeta Kavita

S
25
7. (c)
West
Kavita’s shadow is right to Reeta i.e., Kavita’s shadow
is left to Kavita.
\ Kavita is facing North.
12. (a) 45°+180° = 225° clockwise direction
River is finally flowing in east direction. 270° anticlockwise direction.
For (Qs. 8-9) 225 – 270° = – 45°
Prashant 1st ent
vem
Mo

180°
30m
45°
Initial
Position 45°
Nitin Atul
25m 40m Kunal
Mo Fina 2nd
60m vem l Movement
en
t

i.e., 45° anticlockwise from initial position.


Dinesh Hence, the required direction is south-west.
y
o
u
rs
m
a
h
SOLUTIONS 9

b
o
o
13. (d) 50 m 18. (b) P O

b
N

.w
20 m

o
10m

10 km
10 km

rd
30 m P
Bhavika W E

p
Sunaina
50 m Q 100 m

re
s
S

s
6 km

.c
70 m

o
19. (a) 15 m
When Bhavika is at P, Sunaina who is walking at the

m
N
same rate will be at Q which is at a distance of 70 m from
Sunaina’s point of start.

10 m
\ Required distance between them is PQ W E
PQ = (70 + 50) – 100 = 20 m
O
5 km S
14. (c)
N 20. (a) PQ = 10 + 6 = 16 km
A F B 21. (d)
1 km 1 km P
W E
C 2 km E
D 22. (a)
5 km
S N
3 km Q
5 km

P (Starting point) 5 km
W E

Find the distance PB.


By Pythagorus theorem, we have S S
R 5 km
PB2 = PF2 + FB2
PB2 = (4)2 + (3)2 = 25 Obviously, Q lies North of S.
Þ PB = 5 km N
23. (d)
15. (c) M 15 m X D M
W E
20 m

R K

15 m Y S
N 12 m 24. (b) O
Z
XZ = XY + YZ = MN + YZ = 20 m + 12 m = 32 m
30 m

16. (d) C 15 km D
N
12 km

15 m
W E

B 15 km A S
20 m

Obviously, CB = AD = 12 km and B is south of C.


17. (b) B 7 km C

P
15 m
5 km

F OP = 30 m + 20 m = 50 m
8 km
3 km

25. (b) D A 40 m
O
N
E 7 km D
30 m

30 m

W E
A C B
20 m
BF = AB – (AE + EF) S
OD = OA + AD = OA + BC
= AB – {(AB – CD) + EF} = 40 m + 20 m = 60 m
= 8 – {(8 – 5) + 3} = 8 – {3 + 3}= 8 – 6 = 2 km
y
o
u
rs
m
a
h
10 101 SPEED TEST

b
o
o
26. (d) 21 km 2. (b) Time between 1 p.m. on Tuesday to 1 p.m. on Thursday

b
B C = 48 hrs.

.w
The watch gains (1 + 2) = 3 minutes in 48 hrs.

o
rd
Þ it gains 1 min, in 16 hrs.
5 km

5 km

p
Hence, it will show correct time at 5 a.m. on Wednesday.

re
3. (b) Anuj reached at = 8 : 15 AM

s
s
Time when the other man came = 8:15 + 0:30=8:45 AM

.c
A D
8 km 13 km (who was 40 minutes late)

o
m
We have to find out C to D. \ scheduled time of meeting = 8:45 – 0: 40 = 8 : 05 AM
27. (a) N 4. (d) First clock will gain 11 × 2 minutes in 11 hrs., and second
15 km clock will lose 11 × 1 minutes in 11 hrs.
Q
Hence difference will be 33 minutes.
P
5. (d) His birthday will be in common date of 15 to 18 and 16
W E 12km to 19 i.e., 17th
18 km
O 6. (b) The heights of A, B, C, D and E in ascending order
is E < D < A < B < C
R
S Clearly, E is the shortest.
let O be the starting point and P, Q and R the positions 7. (a) Position of Kiran from the top = [35 – 7] + 1 = 29 th
after every movement. Hence, Distance from the starting Position of Sohan from the top = 9th.
point = Difference of their positions = 29 – 9 = 20
Distance of final position R from O = OR = 18 – 12 = 6 km. \ Mohan’s position from top = 9 + 10 = 19th
Hence, Kiran’s position from Mohan = 29 – 19 = 10th
28. (b) N For (Qs. 8 to 11)
South West
According to the information provided, the order in which
the boys stand according to their heights is as follows :
W E

East North
S D E C A F B
From the figure, it is clear that ‘S’ becomes ‘North-east’ 8. (c) Clearly, form the above diagram, E is between D and C.
in the new figure (dotted line) 9. (b) D is the tallest.
5 km 10. (d) Counting from the shortest, C is the fourth one in the
29. (e) line.
N
2 km 11. (c) Clearly, from the above diagram, B is the shortest.
8 km School 12. (b) Even if we cannot determine the exact sequence of the
5 km
W E weights of the children, we can conclude on the basis
of the information provided that D is the heaviest.
1 km 13. (c) The arrangement in the parking
5 km S
C + S + C + 2S + C + 3S + C + 4S + C + 5S + C +
Remaining distance = 8 – (2 + 1) = 5 km 6S + C + 7S + C = 36 vehicles
Hence, the drivers require to travel 5 km towards north
Q in the second half of the row = 18 vehicles
to reach the school again.
\ C + 7S + C + 6S + 2S = 18 vehicles
30. (c) Hence, no. of scooters= 7 + 6 + 2 = 15
14. (b) After interchanging their positions, position of A from
25 m left = 11
then positions of A form right = 9.
\ The total no. of people in the row = (9 + 11) – 1 = 19.
15 m 15. (a) Clearly, C comes at the second place – in the
descending order.
South- west direction 1B
25 m
2 C
3 A ¯ Decreasing weights
4E
SPEED TEST 8 5 D
1. (b) Next train for N. Delhi leaves at 8:30 p.m. Since time For (Qs. 16 & 17) :
interval between two trains for N. Delhi is 45 minutes. A The age wise order is as follows :
train for New Delhi has left 15 minutes ago. Gita > Kusum > Arti > Archana > Suman
\ Time of information = 8:30 – 45 + 15 = 8 P.M. 16. (b) 17. (c)
y
o
u
rs
m
a
h
SOLUTIONS 11

b
o
o
18. (b) The order in which the five boys reach the finishing

b
SPEED TEST 9
line is Gaurav, Raj, Mohit, Ashish, Sanchit.

.w
Hence Gaurav won the race. 1. (b) The sitting order of the persons are as follows :

o
19. (b) From the information given in the question, the E

rd
newspaper was read in the following order

p
N S

re
B, C, E, A, D.

s
Hence B passed the newspaper to C. BEGFDCA W

s
.c
20. (d) There are 18 persons in a row. Thus, A and B are sitting at the extreme ends.

o
2. (c) Clearly, J is sitting in the middle.

m
Previous B M
3. (d) At one end, we have k and at the other end, we have N.
4. (b) Only statement (b) is true.
Present M B 5. (a) The sitting arrangement can be shown as follows:
7th from left 15th from left C
12th from right 4th from right
(Motilal’s position) B D
21. (d) Positions as given in the question are
E F
A
7th 23rd I
My friend exactly Clearly, F is opposite to B.
in between 6. (b) Sitting arrangement is as follows :
G1 B1 G 2 B 2 G 3 B 3 G 4
6 7th 23rd 15 39th 1st 2 nd 3 rd 4 th 5 th 6 th 7 th
persons persons
The number of boys are less than girls, so we should
\ My position is 6 + 1 + 15 + 1 + 15 + 1 = 39. begin with girl.
22. (c) According to Pratap: 20, 21 or 22 ...(i) (Qs. 7-10) : Seating arrangement is as follows:
According to his sister: not 22 ...(ii) T
From (i) and (ii), the birthday falls on Apr 20 or 21.
R H
23. (a) R's position = 11th from right
\ M's position = (11 + 15 + 1 = ) 27th from right
= (40 – 27 + 1 =) 14th from left D W
24. (a) R = 16th from the right.
\ w = (16 – 5 =) 11th from the right.
Y
25. (e) Time at which the train arrived = 7 : 14 PM M
But Seema's watch is 6 minutes fast, hence the time in 7. (a) 8. (b) 9. (c) 10. (c)
the Seema's watch = 7 : 20 PM. (Qs. 11-15): Seating arrangement is as follows:
26. (c) 12th 16th
K
R S
J P
22nd
Total number of children in the row = 16 + 22 – 1 = 37 Q O
27. (d) B > C > D, A, E
28. (a) R < M; Q < R, N; N < M L
M>N / R>Q M
N
29. (e) Total number of boys = 41.
11. (b) 12. (c) 13. (e) 14. (a)
30. (d) Geeta > Shilpa
Deepa > Geeta 15. (d)
Reepa > Gayatri (Qs. 16-20): Seating arrangement is as follows:
J
Fatima is the seniormost
But no other data is there to find who is the juniormost. L O
31. (e) Correct order can’t be determined.
32. (c) 3
Q P
Neela
Kamal Sunita
K M
11th 22nd N
(From front) 19th (From front) 16. (e) 17. (c) 18. (d) 19. (a)
So there are seven girls between Kamal and Neela. 20. (b)
y
o
u
rs
m
a
h
12 101 SPEED TEST

b
o
For (Qs. 21 to 25) : Seating arrangement is as follows:

o
8. (d) Clearly, B and D are the females members in the family.

b
H 9. (b) From the above table C and D the married couple.

.w
A 10. (c) A, C and E are the male members in the family.

o
G

rd
11. (d) D, the wife of the trader C is the doctor in the family.

p
F 12. (c) C is the trader in the family.

re
E

s
For (Qs. 13-16)

s
.c
D From the given facts, we can summarize,
C

o
m
B
21. (a) 22. (b) 23. (e) 24. (c) Friends Game Subjects
For (Qs.25 to 30) A Football Maths, Physics / Accounts
V B Football Maths, Accounts / Physics
S Q C Cricket Chemistry, Biology
D Boxing Maths, Accounts.

R W 13. (c) 14. (d) 15. (a) 16. (d)


For (Qs. 17-21)
T The Organization of the plays will be as follows
P
25. (d) 26. (b) 27. (c) 28. (d) 5th 6th 7th 8th 9th 10th
29. (a) 30. (c)
Mon Tue Wed Thu Fri Sat
SPEED TEST 10 D B E C F A
For (Qs. 1 to 5) : 17. (c) The organisation would start from play D which is clear
The given information can be summerised in a table
that follows: from above.
18. (b) E is to be organised on 7th.
Students Math Athletics Studies GK Arts 19. (a) The organisation would end with the play A on 10th.
Asha × × 20. (a) The play B is organised on 6th which is a Tuesday.
Charu × × 21. (d) The correct sequence of the plays is DBECFA.
Deepa × For (Qs. 22-25)
Beena × ×
Ela × ×
Teacher Subjects
shows good A Hindi, Eng, Math
× shows not good
1. (d) From the table above, it is clear that Deepa is good B Hindi, Eng, Geo, History, French
in Studies, General Knowledge and Arts. C Eng, Geo
2. (b) Clearly, Beena is good in Studies, General Knowledge
D Math, Hindi
and Mathematics.
3. (a) Obviously, Asha is good in Studies, Mathematics and E History, French
Athletics.
4. (c) Charu is good in Athletics, General Knowledge and 22. (c) 23. (d) 24. (c) 25. (b)
Mathematics.
5. (d) From the last row of the above table, it is clear that (Qs. 26-30) :
Ela is good in Studies, General Knowledge and Arts The given information can be summarized as follows :
but not in Athletics.
6. (c) A maximum of five cources can be taken – History,
Chemistry, Pschycology, Astronomy and Flights Day
Mathematics. Jet Airways Tuesday
7. (a) There is only one way – History and Psychology.
For (Qs. 8 to 12) British Airways Thursday
A Delta Wednesday
(Farmer) Quantas Saturday
Emirates Monday
Lufthansa Sunday
Daughter Sons Air India Friday

B (Unmarried)
(Teacher) 26. (c) 27. (c) 28. (a)
Husband E (Unmarried)
D C 29. (e) 30. (d)
(lawyer)
(Doctor) (Trader)
y
o
u
rs
m
a
h
SOLUTIONS 13

b
o
o
For questions 21 to 25:

b
SPEED TEST 11

.w
Employee Department Sport
For questions 1 to 5:

o
A Pers TT

rd
B Admin Football

p
re
Friend Bank Occupation
C Admin Hockey

s
A S Forex

s
D Admin Basketball

.c
B M Agriculture

o
E Mktg Cricket

m
C N Economist
F Pers Volleyball
D L TO
G Mktg LT
E R IT
H Mktg Badminton
F Q Clerk
21. (c) 22. (b) 23. (e)
G P Research 24. (a) 25. (d)
(26-30) : Ascending order of fatness :-
1. (b) 2. (c) 3. (a) Q < T < V < W < R < S/P < P/S
E555F
4. (d) 5. (e) Ascending order of height
For questions 6 to 10: V<W< T<S<R< P<Q
26. (c)
Mon P 27. (e) Even all the conditions are not sufficient to find the
thickest.
Tue R
28. (b)
Wed W
29. (d) P or S can come.
Thu V 30. (c) R is 5th from the left in both arrangements.
Fri S
Sat Q SPEED TEST 12
Sun T
1. (b) Area common to singer and poets.
2. (b)
6. (c) 7. (a) 8. (b)
3. (c) Area common to and
9. (c) 10. (e)
4. (b) circle only
For questions 11 to 15: 5. (c) Area not common to rectangle but common to square
and circle.
Friend Shift Day off 6. (a) Area common to Rectangle, Circle, Square.
P II Tuesday 7. (d) Area common to Rectangle and Circle.
8. (a) Only area of square.
Q I Monday 9. (c) Area common to rectangle, square and circle.
R II Wednesday 10. (d) Sun is star. Moon is a satellite.
S I Sunday
Star
T III Friday
V III Thursday Sun
Moon
W I Saturday

11. (c) 12. (d) 13. (a) 11. (a) The required set of students is denoted by region
14. (d) 15. (c) common to any three circle only
For questions 16 to 20: \ Required number = (13 + 13 + 18 + 18) = 62.
A > D > G ...(ii); C > E > H ...(iii) 12. (a) The required set of students is denoted by regions
D > B > F ...(iv); G > C ...(v); F > G ...(vii) lying inside the circles representing History,
Combining these, we get Mathematics and Science. \ Required number = (9 +
A > D > B> F> G >C > E > H 14 + 18 + 15 + 16 + 13 + 13 + 20 + 18 + 13 + 16 + 19) = 183.
16. (e) 13. (b) The required set of students is denoted by the regions
common to the circles representing History and
17. (b) A > D > B > F > G. > C , J > E > H
Geography.
18. (a) \ Required number = (20 + 13 + 12 + 18) = 63.
19. (b) G, C, E and H 14. (b) Number of students who took History
20. (e) = (16 + 12 + 18 + 20 + 18 + 14 + 13) = 111.
y
o
u
rs
m
a
h
14 101 SPEED TEST

b
o
o
Number of students who took Geography Fingers

b
= (9 + 16 + 13 + 20 + 13 + 12 + 18) = 101. Neclaces

.w
Number of students who took science

o
Rings Ears

rd
= (19 + 15 + 18 + 20 + 18 + 16 + 13) = 119.
Number of students who took mathematics

p
re
= (9 + 14 + 13 + 20 + 13 + 15 + 18) = 102.

s
15. (c) both students and teachers are parts of college and is

s
.c
differents. Conclusions :

o
16. (b) I. True

m
II. Not True

3. (d) Bottles
Cups
Mother Homosapien

Woman Plates

Spoon
17. (b) Home minister is a minister, minister is part of cabinet.
18. (d) All Parrots are birds, but mice is entirely different. or
19. (a) Some professor may be scientist or researcher.
Bottles
20. (c) Men, Rodents are entirely differents, but both are living Cups
beings.
21. (c) Mother and Father are entirely different but both are
parents.
22. (d) Nitrogen is air but ice is differents. Spoon
23. (a) All three items are partly related. Plates
24. (d) Tiger is a Carnivore, while elephants is not.
25. (b) Herring is type of fish, fish belongs to the class of Conclusions :
animals. I. Not True
26. (c) Nurse and Patient are differents but both are parts of II. Not True
Hospitals. Both conclusions form complementary pair.
27. (c) Nose and hand are differents but both are parts of body. 4. (a) Erasers
28. (b) All diamonds rings are rings, all rings are ornaments.
29. (d) Table are furniture but book are differents.
Pens
30. (c) Chess and table tennis are differents but both are indoor
games. Sharpners

SPEED TEST 13 Staples


or
Erasers
Radios
1. (c) Computers Televisions
Pens Sharpners
Phones

Staples

Conclusions :
Conclusions : I. Not True
I. Not True II. Not True
II. True
5. (d) Jungles Bushes
2. (b) Fingers Trees
Neclaces
Hills
Rings

Ears Conclusions :
I. True
or II. True
y
o
u
rs
m
a
h
SOLUTIONS 15

b
o
o
b
or,

.w
Grills

o
6. (d) Windows or

rd
Glasses holidays trip vacations

p
re
If neither conclusion I and nor II follow.

s
s
12. (d) According to statement

.c
Grills

o
m
Windows
Glasses

kites bird kites aeroplane


Conclusion-I : False
II: False or

Painters Artists kites bird Aeroplane


7. (d) Painters Artists or
If neither conclusion I and II follows.
Dancers 13. (a) According to statement
Dancers
plastics fibres
metals plastics
Conclusion-I : False
II: False
fibres
Buildings or,
plastics
Room metal

8. (b) Cabins
Only conclusion I follows.
14. (a) According to statement
streets
roads
Conclusion-I : False
II: True
street highway
Necklace streets
or,
9. (a) Rings Bracelet highway
roads
Hence only conclusion I follow.
Conclusion-I : True 15. (b) According to statement I
II: False Rocks
animals plants plants
Arms
Rocks
Hands Muscles
10. (a) plants
or,
animals
Conclusion-I : True Hence, only conclusion II follows.
II: False
11. (d) According to statement
16. (d) Holiday
Vacation
Holiday
or Trips
Trips
holidays vacation holidays trip Vacation
y
o
u
rs
m
a
h
16 101 SPEED TEST

b
o
o
Conclusions : 22-23.

b
I. Not True II. Not True

.w
Kites Power

o
Birds
17. (d)

rd
Kites Birds or

p
Energies Heat

re
s
s
.c
Aeroplane
Force

o
Aeroplane

m
Conclusions :
I. Not True II. Not True
Fibres
18. (a)

22. (b) Conclusion I : False


Metals Conclusion II : True
23. (a) Conclusion I : True
Plastics Conclusion II : False
24-25.
Conclusions :
Coin
I. True II. Not True Metals
Rocks Note
19. (b)
Animals Plants

Plastics
OR
Conclusions :
I. Not True II. True Note Metals Coin
20. (b) Institutes

Banks
Plastics
School OR
Academics Note Metals
Win
Conclusions :
I. Not True II. True

21. (d)
Test Exams
Plastics
24. (e) Conclusion I : True
Conclusion II: True
25. (d) Conclusion I : False
Question Conclusion II : False
OR
26. (e)
Test Exams Picture

Symbols
Figures
Question
Graphics
Conclusion I : False Conclusion II : False
y
o
u
rs
m
a
h
SOLUTIONS 17

b
o
OR 28. (a) Hence, only conclusion I follows.

o
b
Symbols 29. (b) Hence, conclusion II follows.

.w
30. (d) According to statements.

o
Figures

rd
p
re
s
Mails Updates

s
.c
o
m
Chats
Picture
or

Graphics
Conclusion I : True Conclusion II : True Mails
27. (d) Jobs or
Mails

Chats Chats

Hence, conclusion I follows.

SPEED TEST 14
Occupations
Vacancies Bins
OR
Jobs 1. (e) Flowers
Occupations

Handles

Sticks
Vacancies Bins
Conclusion I : False Conclusion II : False
For questions 27 -29 :
According to statements:
or
Flowers Handles

Gliders Airplane Sticks


Conclusions I : True II : Not true
Parachutes Helicopters III : Not true
or 2. (b) Houses

towers
Gliders Airplane windows
temples
Parachutes s
opte r
Helic Houses
or
temples

Gliders Airplane towers


windows
Parachutes Conclusions I : Not true II : True
III : Not true
y
o
u
rs
m
a
h
18 101 SPEED TEST

b
o
o
3. (c) cots Pens

b
6. (d)

.w
o
rd
p
re
walls Toys
doors chairs

s
Desks

s
.c
walls

o
doors Roads

m
or Roads
Pens
or
cots
chairs
walls doors
Toys Desks
or Conclusions I : Not true
II : Not true
chairs Bangles
7. (c)

cots
Conclusions I : Not true
II : Not true
III : Not true Table Ring
As conclusions I and III complement to each other. Huts
Ring

4. (d) or
Bangles
trees
fences
gardens stones Table Huts
Conclusions I : True
II : True or Bangles
III : True
5. (a)
Box Leaves Ring
Table Huts
Conclusions I : Not true
Books II : Not true
Since, conclusion I and II form complementary pair.
Therefore, either (I) or (II) follows.
Jungles
8. (e)
Trees
or Jungles Chairs
Poles
Rooms
Books Conclusions I : True
Leaves
II : True
(Qs. 9-10) House Flats
Box
Conclusions I : Not true
II : Not true Buildings
III : Not true
Appartment
y
o
u
rs
m
a
h
SOLUTIONS 19

b
o
o
Flats 16. (d) Conclusions I : Not true

b
II : Not true

.w
or
17. (b)

o
Circles

rd
Rings ellipse

p
Buildings Appartment

re
House Squares

s
s
9. (b) Conclusions I : Not true

.c
II : True

o
m
10. (d) Conclusions I : Not true
Conclusions I : Not True
II : Not true
(Qs. 11-12) II : True
Rivers 18. (d)
Appar- Bun-
House glows
Canal tment
or
Oceans Bun- Appar-
Seas House glows tment
or
Seas
Conclusions I : Not True
II : Not True
Oceans
19. (a)
Rivers Canal Gases Liquids
11. (d) Conclusions I : Not true Water
II : Not true
12. (e) Conclusions I : True or
II : True
(Qs. 13-14)
evening
Gases Liquids
Noon
Water

Day Conclusions I : True


Night II : Not True
Noon 20. (b)
or Hours
Seconds

Day
Day Night

evening
or
13. (d) Conclusions I : Not true Hours
II : Not true Seconds
14. (a) Conclusions I : True
II : Not true Day

(Qs. 15-16)
card

Conclusions I : Not True


papers boards II : True
or papers (21 -22)
boards r s
so
Teachers

fes
o
card Pr
rs
15. (c) Conclusions I : Not true ture
Lec
II : Not true
or
Since conclusions I and II form complementary pair.
y
o
u
rs
m
a
h
20 101 SPEED TEST

b
o
o
3. (d) Letter series T M E I U F

b
.w
Professors

o
Lecturers

rd
Teachers Code 7 $ 2 % d 9

p
re
or According to

s
9 $ 2 % d 9

s
condition (ii)

.c
rs

o
so
es

m
of Teachers, 4. (a) Letter series J T A E R I
Pr Lecturers
Code 3 7 1 2 @ %
21. (a) Conclusions I : True II : Not True
According to
22. (b) Conclusions I : Not True II : True
condition (i) % 7 1 2 @ 3
(23 - 24) : Possible venn diagram are
5. (c) Letter series U K T M I H
Roses Flower Red
Code d # 7 $ % 6
or
According to
Red
condition (iii) « # 7 $ % «
Roses Flower 2 8 % 9 5 6
6. (d)
or R N A X H S
Red Condition (iii) is applicable.
Flowers Red
@ 6 2 + 7 4
7. (a)

23. (d) P S R G F T
24. (a) Condition (i) is applicable.
25. (a) A possible Venn diagram is + 5 9 6 3 %
8. (a)
High
G H X S D G
Hills Condition (ii) is applicable.
9. (a) As, M O D E and D E A F

# 8 % 6 % 6 7 $
Similarly, F O A M

SPEED TEST 15 $ 8 7 #
10. (e) W E A K W H E N
1. (a) Letter series E R W H K A

5 % 9 $ 5 * % 7
Code 2 @ © 6 # 1 Therefore,
without H A N K
condition
2 @ © 6 # 1

2. (e) Letter series M P E K D U * 9 7 $


11. (c) 12. (a) 13. (e)
14. (b) 15. (d)
Code $ 4 2 # 5 d 16. (b) 2 7 0 5 1 4

According to
condition (i) d 4 2 # 5 $ $%L T KQ
condition (ii) is applied
y
o
u
rs
m
a
h
SOLUTIONS 21

b
o
o
17. (a) 3 6 4 2 7 9 8. (a) New arrangement

b
.w
H F 3 U 6 G I T P L 8 9 S 2 7 A M K .......

o
©H$ Q%©

rd
condition (i) is applied

p
12th from left

re
18. (c) 8 7 5 3 0 6

s
+2 –1

s
9. (d) L S 8

.c
H%T # L J

o
+2 –1

m
condition (ii) is applied A K M
19. (e) 5 9 2 4 7 6 @ +2 –1
I F
6 +1 +1
T@Q $%H % G
20. (d) 4 6 8 9 1 0 J +2 –1
D ©

$ H J @ K« F +5 +6 +7
% L
condition (iii) is applied 7
10. (c) 3 +5 +6 +7
21. (a) 2976581®B@$©P=B I S A
22. (b) 7 2 6 9 5 3 4 ® K B © @P T $ +5 +6 +7 M
23. (c) 81 3524 6®©CTP BK© U T ^
24. (e) 4352718®= TPB$ C= 11. (a) D F J T $ # PR ZQ * C MAB @ HK LS + ?
25. (a) 4 3 5 9 7 1 only $ # P is the required answer.
12. (e) * Q Z RP# $ TJ FD C MAB@ HK LS + ?
? + % « # $ 13. (d) Number of total symbols = 6; Number of total letters
= 16. Since, all the symbols are denoted by 7 and all
SPEED TEST 16 letters are denoted by 5, sum of the elements of the
-3 +4
sequence = 6 × 7 + 16 × 5 = 122
1. (e) 2 ¾¾® 3 ¾¾® # 14. (b) When all the symbols are dropped the series becomes
-3 +4 as follows:
O ¾¾® I ¾¾® C
D F JT PR ZQ C MAB HK LS
-3 +4 Now, seventh to the right of twelfth letter from the right
K ¾¾® O ¾¾® 5
-3 +4
= (12 – 7 =) 5th letter from the right, i.e., B.
# ¾¾® P ¾¾® I 15. (d) Compare ‘DJ’ and ‘?S’. ‘D’ is the first element from left
-2 +3 end of the series and ‘?’ is the first element from right
B ¾¾® $ ¾¾® <
end. Similarly, ‘J’ and ‘S’ are third elements from left
2. (a) +3 +3 +3 +3 and right end respectively. Hence, ‘FT’ is related
P ¾¾® # ¾¾® 7 ¾¾® @ ¾¾® D
to' + L'.
+3 +3 +3 +3
R ¾¾® L ¾¾® I ¾¾® O ¾¾® K 16. (b) D F J T $ #P R ZQ * C M A B @ H K L S + ?
+3 +3 +3 +3 ABC DE F G HI JK LMNO P QRS T UV
J ¾¾® 3 ¾¾® 2 ¾¾® N ¾¾® C 17. (e) Except it there is only one element between first and
3. (c) 11th to the left of 16th from left means 5th from the left. second letter of each group of words when the position
But the sequence has been reversed. Therefore, required of the letters in the series is taken into consideration.
element will be 5th from right in the original sequence. 18. (c) Eighth element from right = (22 + 1 – 8 =)15th element
5th from right Þ B from left.
4. (c) Consonant Number Symbol Hence, the required element which is exactly midway
Such combinations are between 5th element from left and 15th element from
R3P , N7O , K5D , Q4 æ 15 + 5 ö
left, is ç = ÷ 10th element from left, i.e., Q.
è 2 ø
5. (e) Number Symbol Consonant 19. (d) Clearly, the given letters, when arranged in the order 5,
There is no such combination. 1, 2, 3, 4 from the word ‘TRACE’.
6. (b) Ninth to the right of the 20th from the right means 11th 20. (b) Clearly the given letters, when arranged in the order 4,
from the right, i.e., M. 5, 2, 3, 1, 6 form the word 'STRAVE'.
Symbol Symbol 21. (d) We have to look for Vowel-Number and Number-Vowel
7. (c) Consonant
sequences.
Such combinations are : J 1 # P 4 E K 3 A D $ R U M 9 N 51 % T V * H 2 ¸ F 6
@ F ! : + J C G8QW
4, 3 and 5 are the required numbers.
y
o
u
rs
m
a
h
22 101 SPEED TEST

b
o
o
(d) 4 + 2 K – 51; 4+2=K–5=1

b
22. (d) D $ RU M 9 N 5 1 % T V * H 2 (e) 6 + 3 Q – 5 ¸ ; 6 +2=8–5=2
E5555555555F E555555555F

.w
7 elements 7 elements Note that the difference between two successive

o
rd
23. (a) After the changing, the series becomes as follows; elements in 5 is not similar to others.
J 1 # P4E K3AD$ RUM9 NWQ8 G6 F ¸2H 26. (d) After changing the series becomes as follows :

p
re
* V T % 15 I D 7 1 J P $ 3 E RT 5 £ M 2 N A4 F H 6 H U9 # VB@ W

s
Now, twenty-second element from the right end is 3.

s
Now, ninth to the right of the eleventh element from the

.c
left® (11 + 9 =) 20th element from the left, i.e., G. 27. (c) We have to look for

o
Vowel-number-consonant sequence.

m
24. (b) We have to look for Symbol Consonant – Consonant
sequence and Symbol–Consonant–Symbol sequences. M £ 5TRE 3$ PJ1 7D I 2NA4 F H6 H U9 #VB@W
J I # P 4 E K 3 A D $ R U M 9 N 51 % T V H Only 2 and 4 are such numbers.
2¸ F6G8QW 28. (e) D2 J
Only T is such a consonant. R +4 P +4 D +4A
25. (e) See the difference between each two successive 3 +4 1 +4 2 +4F
element. £ +4 E +4 J +4I
(a) A +2 $ – 5 E; A+2=$–5=E 29. (d) M £ 5T RE 3 $ PJ 17 D I 2 NA4 F H 6 U9 # VB@ W
(b) % + 2V – 5 N; %+2=V–5=N 30. (b) Fifth element towards right of the seventeenth element
(c) 2 + 2 F – 5 V; 2+2 = F–5 =V from the right end implies twelfth element from the right
end. Hence, the required element is 4.

SPEED TEST 17

Solution (1 to 5)
After analysis of the given input and various steps of rearrangement it is clear that words and numbers are being rearranged
alternately. The words get rearranged in alphabetical order while the numbers get rearranged in descending order.
1. (c) input : won 13 now 25 72 please go 47
Step I : go won 13 now 25 72 please 47
Step II : go 72 won 13 now 25 please 47
Step III : go 72 now won 13 25 please 47
Step IV : go 72 now 47 won 13 25 please
Step V : go 72 now 47 please won 13 25
Step VI : go 72 now 47 please 25 won 13
Step VI is last step.
2. (c) Step III : car 81 desk 15 42 39 tall more
Step IV : car 81 desk 42 15 39 tall more
Step V : car 81 desk 42 more 15 39 tall
Step VI : car 81 desk 42 more 39 15 tall
Step VII : car 81 desk 42 more 39 tall 15
Step VII is last steps.
3. (b) Step II : bell 53 town hall near 27 43 12
Step III : bell 53 hall town near 27 43 12
Step IV : bell 53 hall 43 town near 27 12
Step V : bell 53 hall 43 near town 27 12
Step VI : bell 53 hall 43 near 27 town 12
Step VI is last steps.
4. (d) Step II : box 93 25 year end 41 32 value
input :
Hence step II can not determine input.
5. (a) input : paper dry 37 23 height call 62 51
Step I : call paper dry 37 23 height 62 51
Step II : call 62 paper dry 37 23 height 51
Step III : call 62 dry paper 37 23 height 51
Step IV : call 62 dry 51 paper 37 23 height
Step V : call 62 dry 51 height paper 37 23
Step VI : call 62 dry 51 height 37 paper 23
Step VI is last step.
y
o
u
rs
m
a
h
SOLUTIONS 23

b
o
o
(Sol. 6-10): 16. (c) Step II : 18 task bear cold dish 81 63 31

b
In step I, the word that comes last in the alphabetical order Step III : 18 task 31 bear cold dish 81 63

.w
comes to the first place, pushing the rest of the line rightward. Step IV : 18 task 31 dish bear cold 81 63

o
Step V : 18 task 31 dish 63 bear cold 81

rd
In step II, the largest number comes at the second place,
pushing the line rightward. Thus, words and numbers get Step VI : 18 task 31 dish 63 cold bear 81

p
re
arranged alternately till all the words are in reverse Step VII : 18 task 31 dish 63 cold 81 bear

s
alphabetically order and numbers in descending order. 17. (d) Input : 72 59 37 go for picnic 24 journey

s
.c
6. (d) Input: glass full 15 37 water now 85 67 Step I : 24 72 59 37 go for picnic journey

o
Step I : water glass full 15 37 now 85 67 Step II : 24 picnic 72 59 37 go for journey

m
Step III : 24 picnic 37 72 59 go for journey
Step II : water 85 glass full 15 37 now 67
Step IV : 24 picnic 37 journey 72 59 go for
Step III : water 85 now glass full 15 37 67
Step V : 24 picnic 37 journey 59 72 go for
Step IV : water 85 now 67 glass full 15 37 Step VI : 24 picnic 37 journey 59 go 72 for
Step V : water 85 now 67 glass 37 full 15 18. (a) Input : nice flower 34 12 costly height 41 56
Since the arrangement has been done, there will be no Step I : 12 nice flower 34 costly height 41 56
step VI. Step II : 12 nice 34 flower costly height 41 56
7. (d) Step II : ultra 73 12 16 mail sort 39 kite Step III : 12 nice 34 height flower costly 41 56
Step III : ultra 73 sort 12 16 mail 39 kite 19. (d) Step II : 16 victory 19 36 53 store lake town
Step IV : ultra 73 sort 39 12 16 mail kite Step III : 16 victory 19 town 36 53 store lake
Step V : ultra 73 sort 39 mail 12 16 kite Step IV : 16 victory 19 town 36 store 53 lake
Step VI : ultra 73 sort 39 mail 16 12 kite Since the line is already arranged, there will be no fifth
Step VII : ultra 73 sort 39 mail 16 kite 12 step.
Hence, Step VI will be the last but one. 20. (d) We can't work out backward.
8. (d) We can't work backward in an arrangement type. 21. (b) Input : milk pot 18 24 over goal 36 53
9. (c) Step II : tube 83 49 34 garden flower rat 56 Step I : 18 milk pot 24 over goal 36 53
Step III : tube 83 rat 49 34 garden flower 56 Step II : 18 pot milk 24 over goal 36 53
Step IV : tube 83 rat 56 49 34 garden flower Step III : 18 pot 24 milk over goal 36 53
Step V : tube 83 rat 56 garden 49 34 flower Step IV : 18 pot 24 over milk goal 36 53
Step VI : tube 83 rate 56 garden 49 flower 34 Step V : 18 pot 24 over 36 milk goal 53
10. (a) Input : hunt for 94 37 good 29 48 book. Step VI : 18 pot 24 over 36 milk 53 goal
Step I : hunt 94 for 37 good 29 48 book Hence step V is the last but one.
Step II : hunt 94 good for 37 29 48 book 22. (a) Step III : 36 win 44 95 86 ultra box queen
Step III : hunt 94 good 48 for 37 29 book Step IV : 36 win 44 ultra 95 86 box queen
Step IV : hunt 94 good 48 for 37 book 29 Step V : 36 win 44 ultra 86 95 box queen
(Sol. 11-15): Step VI : 36 win 44 ultra 86 queen 95 box
Input : for 52 all 96 25 jam road 15 hut 73 bus stop 38 46 Hence 6 – 3 = 3 more steps will be required
Step I : all for 52 25 jam road 15 hut 73 bus stop 38 46 96 23. (a) Input : new 22 model 27 pump 38 11 join
Step II : bus all for 52 25 jam road 15 hut stop 38 46 96 73 Step I : 11 new 22 model 27 pump 38 join
Step III : for bus all 25 jam road 15 hut stop 38 46 96 73 52 Step II : 11 pump new 22 model 27 38 join
Step IV : hut for bus all 25 jam road 15 stop 38 96 73 52 46 Step III : 11 pump 22 new model 27 38 join
Step V : jam hut for bus all 25 road 15 stop 96 73 52 46 38 Step IV : 11 pump 22 new 27 model 38 join
Step VI : road jam hut for bus all 15 stop 96 73 52 46 38 25
Step VII : stop road jam hut for bus all 96 73 52 46 38 25 15 SPEED TEST 18
11. (b) Step IV : hut for bus all 25 jam road 15 stop 38 96 73 52
46 Eighth from the right – road. +Þ – – Þ´
12. (c) 13. (c) 1. (a) ¸Þ + ´Þ ¸
14. (a) Step V : jam hut for bus all 25 road 15 stop 96 73 52 46 38
Sixth from the left is 25 10 × 5 ¸ 3 – 2 + 3 = ?
15. (d) or, ? = 10 ¸ 5 + 3 × 2 – 3
(Sol. 16-23): or, ? = 2 + 6 – 3 = 5
In step I the least number comes to the left most position,
pushing the rest of the line rightward . In step II the word +Þ ¸ – Þ´
2. (b)
that comes last in the alphabetical order shifts to second ¸Þ + ´Þ –
from left, pushing again the rest of the line rightward. Similarly,
in step III the second least number shifts to third from left. In 63 × 24 + 8 ¸ 4 + 2 – 3 = ?
step IV the second from last in the alphabetical order comes or, ? = 63 – 24 ¸ 8 + 4 ¸ 2 × 3
to the fourth position. And this goes on alternately till all the or, ? = 63 – 3 + 2 × 3
numbers are arranged in ascending order and the words in or, ? = 66
reverse alphabetical order. 3. (b) 6 × 4 – 9 = 15
Step V : jam hut for bus all 25 road 15 stop 96 73 52 46 38 4. (c)
Step VI : road jam hut for bus all 15 stop 96 73 52 46 38 25 5. (a) 6 × 5 = 30, 30 × 3 + 1 = 91, 8 × 7 = 56, 56 × 3 + 1 =
Step VIII : stop road jam hut for bus all 96 73 52 46 38 25 15 169, 10 × 7 = 70, 70 × 3 + 1 = 211
Similarly 11 × 10 = 110, 110 × 3 + 1 = 331
y
o
u
rs
m
a
h
24 101 SPEED TEST

b
o
o
6. (a) 21. (b) Since, 20 × 10 = 200, therefore, – means ×
– Þ +, + Þ ´

b
8 + 4 = 12, therefore, ÷ means +.

.w
+ Þ –, ´ Þ + 6 – 2 = 4, therefore, × means – .

o
rd
and 12 ÷ 3 = 4, therefore, + means ÷.
Option (a) : 6 ¸ 20 + 12 + 7 – 1 = 70
Now, given expression

p
L.H.S. = 6 – 20 + 12 × 7 ¸ 1 = 6 – 20 + 84

re
= 100 × 10 – 1000 + 1000 ÷ 100 – 10
= 90 – 20 = 70 R. H.S.

s
= 100 0 – 1000 + 10 – 10 = 0

s
.c
7. (c) + Þ ´, ´ Þ – 22. (b) Using the given symbols, we have:

o
Given expression = 8 + 7 × 8 ÷ 40 – 2

m
¸ Þ +, – Þ ¸
1
Given expression ® 175 – 25 ¸ 5 + 20 × 3 + 10 =8 + 7× –2
5
After conversion Þ 175 ¸ 25 + 5 × 20 – 3 × 10
= 7 + 100 – 30 = 77 37 2
= =7 .
8. (a) Using the proper signs, we get 5 5
Given expression = 14 × 10 + 42 ¸ 2 – 8 23. (c) 9 × 8 + 8 ÷ 4 – 9 = 65
= 140 + 21 – 8 = 153 24. (b) 20 + 12 – 4 ÷ 8 × 6 = 29
9. (c) Using proper notations, we have: 25. (d) 40 + 12 ÷ 3 × 6 – 60 = 4
26. (d) Using the correct symbols, we have
11 Given expression = 24 x 12 + 18 ÷ 9 = 288 + 2 = 290.
(a) Given statement is 3 ¸ 2 + 4 < 9 ¸ 3 – 2 or < 1 not true
2 27. (d) Using the proper notations in (4) we get the statement
(b) 3 + 2 + 4 < 18 ¸ 3 – 1 or 9 < 5, which is not true. as 2 × 5 – 6 + 2 = 6 or 10 – 6 + 2 = 6 or 6 = 6, which is true.
(c) 3 + 2 – 4 > 8 ¸ 4 – 2 or 1 > 0, which is true 28. (c) Using the proper notations in (3), we get the statement
5 as 5 × 2 ¸ 2 < 10 – 4 + 8 or 5 × 1 < 18 – 4 or 5 < 14,
(d) 3 ¸ 2 – 4 > 9 ¸ 3 – 3 or - > 0, which is not true. which is true.
2 29. (a) Using the correct symbols, we have
10. (d) Using the proper notation in (d), we get the statement Given expression
as:
8 × 8 + 8 ¸ 8 – 8 = 64 + 1 – 8 = 57 (36 - 4 ) ¸ 8 - 432 ¸ 8 - 4 4-4
11. (c) Using the proper notations in (c), we get the statement = = = = 0.
4x8 - 2x16 + 1 32 - 32 + 1 0 +1
as:- 30. (b) Using the correct symbols, we have
5 × 2 ¸ 2 < 10 – 4 + 8 Given expression = (3 × 15 + 19) ÷ 8 – 6
or, 5 × 1 < 18 – 4 = 64 ÷ 8 – 6 = 8 – 6 = 2.
or 5 < 12 ® which is true.
12. (b) Using the correct symbols, we have SPEED TEST 19
Given expression = (23 + 45) × 12 = 68 × 12 = 816.
13. (c) Given expression = 32 × 5 – (15 – 3) × 3 1. (b) No of right angles in one hour = 2
= 160 – 12 × 3 \ No of right angles in 24 hours = 24 × 2 = 48
= 160 – 36 = 124 = bce 2. (a) Hour hand covers an angle of 360° in 12 hours.
14. (c) Given expression = 105 + 56 – (20 × 7/14) \ Time taken to cover an angle of 135°
12
= 105 + 56 – 10 = 151 = ´135 =4.5 h \ Required time = 3 + 4.5 = 7.5 = 7:30
15. (b) Given expression = 105 ¸ 15 × 3 = 7 × 3 = 21 = cb 360
3. (d) Angle made by hour hand for 12 hours = 360°
16. (a) Using the proper signs in the given expressions, we 360°
get Angle made by hour hand for 1 hour =
12
175 ¸ 25 + 5 × 20 – 3 × 10 360°
= 7 + 5 × 20 – 3 × 10 = 7 + 100 – 30 = 107 – 30 = 77 \ Angle made by hour hand for 6 hours = 12 × (6) = 180°
17. (c) Using the proper signs, we get: 4. (b) In a year, number of weeks = 52 extra day = 1
36 – 8 4 + 6 ¸ 2× 3 = 36 – 2 + 3 × 3 From 2002 to 2008, there are 6 years.
= 36 – 2 + 9 = 45 – 2 = 43 So number of extra days = 6 (1) = 6
18. (d) Interchanging (+ and ÷) and (2 and 4), we get : While 2004 and 2008 are leap years, having one more extra
day apart from the normal extra day.
(1) 4 ÷ 2 + 3 = 3 or 5 = 3, which is false
Thus, number of extra days = 6 + 1 + 1 = 8
(2) 2 ÷ 4 + 6 = 1.5 or 6.5 = 1.5, which is false. Out of these 8 extra days, 7 days form a week and so 1 day
10 remains.
(3) 2 + 4 ÷ 3 = 4 or = 4, which is false. Hence, March 1, 2002 is 1 day less then March 1, 2008 i.e.,
3 it is Friday.
(4) 4 ÷ 2 + 6 = 8 or 8 = 8, which is true. 5. (c) In one hour, hour hand and minute hand are at right angles 2
19. (d) Using the correct symbols, we have: times.
Given expression = 8 + 36 ÷ 6 – 6 ÷ 2 × 3 Time = 10 p.m – 1 p.m = 9 hr.
\ No. of times, when both hands are perpendicular to each
=8+6–3×3=5
other in 9 hr = 9 × 2 = 18
20. (b) Using the proper notations in (2), we get the statement 6. (b) Here H × 30 = 4 × 30 = 120 0 .
as 5 × 2 ÷ 2 < 10 – 4 + 2 or 5 < 8 , which is true. (Since initially the hour hand is at 4. \ H = 4).
y
o
u
rs
m
a
h
SOLUTIONS 25

b
o
Required angle A = 90 0 and since, H × 30 > A° so,

o
15. (c) The year 2004 is a leap year. It has 2 odd days.

b
there will be two timings. \ The day on 8th Feb, 2004 is 2 days before the day on 8th

.w
2 Feb, 2005. Hence, this day is Sunday.

o
Required time T = (H × 30 ± A) minutes past H. 16. (d) Count the number of odd days from the year 2007 onwards
11

rd
from the year 2007 onwards to get the sum equal to 0 odd day.

p
2
\ One timing = (4 × 30 + 90) minutes past 4

re
Year 2007 2008 2009 2010 2011 2012 2013 2014 2015 2016 2017
11

s
Odd day 1 2 1 1 1 2 1 1 1 2 1

s
2

.c
= 38 minutes past 4. Or 4 : 38 approx. 17. (b) Each day of the week is repeated after 7 days
11

o
So, after 63 days, it will be Monday.

m
7. (a) Since, in one hour, two hands of a clock coincide only once,
so, there will be value. \ After 61 days, it will be Saturday.
18. (c) 17th June, 1998 = (1997 years + Period from 1.1.1998 to
2 17.6.1998)
Required time T = (H ´ 30 + A°) minutes past H.
11 Odd days in 1600 years = 0
Here H = initial position of hour hand = 3 Odd days in 300 years = (5 × 3) º 1
(Since 3 o’clock) 97 years has 24 leap years + 73 ordinary years.
A° = required angle = 0° (Since it coincides) Number of odd days in 97 years = (24 × 2 + 73) = 121 = 2
2 odd days.
T= (3 ´ 30 + 0) minutes past 3 Jan. Feb. March April May June
11
4 (31 + 28 + 31 + 30 + 31 + 17) = 168 days
= 16 minutes past 3. = 24 weeks = 0 odd day
11 Total number of odd days = (0 + 1 + 2 + 0) = 3
8. (b) At 5 o’clock, the hands are 25 min. spaces apart. Given day is Wednesday.
To be at right angles and that too between 5.30 and 6, the 19. (d) No. of days between 21st July, 1947 and 21 st July, 1999
minute hand has to gain (25 + 15) = 40 min. spaces = 52 years + 366 days.
55 min. spaces are gained in 60 min. = 13 beap years + 39 ordinary years + 366 days
æ 60 ö 7 = (13 × 2) odd days + 39 odd days + 2 odd days
40 min. spaces are gained in ç ´ 40 ÷ min . = 43 min . = (26 + 39 + 2) odd days = 67 odd years = 4 odd days.
è 55 ø 11
= (7 – 4) = 3 days before the week day on 21st July,
7 1999 = Saturday.
\ Required time = 43 min. past 5
11 20. (b) Next train for N. Delhi leaves at 8:30 p.m. Since time
9. (d) At 4 o’clock, the hands of the watch are 20 min. spaces apart. interval between two trains for N. Delhi is 45 minutes.
To be in opposite directions, they must be 30 min. spaces apart. A train for New Delhi has left 15 minutes ago.
\ Minute hand will have to gain 50 min. spaces Time of information = 8:30 – 45 + 15 = 8 P.M.
55 min. spaces are gained in 60 min. 21. (b) Time between 1 p.m. on Tuesday to 1 p.m. on Thursday
æ 60 ö 6 = 48 hrs. The watch gains (1 + 2) = 3 minutes in 48 hrs.
50 min. spaces are gained in ç ´ 50 ÷ min . or 54 min . it gains 1 min, in 16 hrs.
è 55 ø 11 Hence, it will show correct time at 5 a.m. on Wednesday.
6 22. (b) A reverse flow chart will look as follows:
\ Required time = 54 min. past 4
11 Desk officer - Friday
10. (a) 55 min. spaces are covered in 60 min. Same day
Senior clerk - Friday
æ 60 ö 5 Senior clerk’s leave -Thursday
60 min. spaces are covered in ç ´ 60 ÷ min. = 65 min. Next day
è 55 ø 11 Inward clerk - Wednesday
Loss in 64 min. = æ 65 5 - 64 ö = 16 min . 23. (b) Ashish leaves his house at 6:40 AM.
ç ÷ Ashish reaches Kunal’s house at 7:05 AM.
è 11 ø 11
They finish Breakfast at 7:05 + 0:15 = 7:20 AM.
Loss in 24 hrs. = çæ ´ ´ 24 ´ 60 ÷ö min = 32 min .
16 1 8 That’s the time when they leave Kunal’s house for thier
è 11 64 ø 11 office.
11. (c) 100 years contain 5 odd days. 24. (b) Anuj reached at = 8 : 15 AM
\ Last day of 1st century is Friday Time when the other man came = 8:15 + 0:30=8:45 AM
200 years contain (5 × 2) º 3 odd days. (who was 40 minutes late)
\ Last day of 2nd century is Wednesday. \ scheduled time of meeting = 8:45 – 0: 40 = 8 : 05 AM
300 years contain (5 × 3) = 15 º 1 odd day. 25. (d) First clock will gain 11 × 2 minutes in 11 hrs., and second
\ Last day of 3rd century is Monday. clock will lose 11 × 1 minutes in 11 hrs.
400 years contain 0 odd day. Hence difference will be 33 minutes.
\ Last day of 4th century is Sunday. 26. (d) 1st of month was Tuesday, hence the date on first Saturday
This cycle is repeated. was 5th.
\ Last day of a century cannot be Tuesday or Thursday or Hence the other Saturdays of the month are 12, 19, 26.
Saturday. Rama met her brother on 26th.
12. (a) The century divisible by 400 is a leap year. 1
\ The year 700 is not a leap year. 27. (d) 2 hrs = 150 min
2
13. (b) x weeks x days = (7x + x) days = 8x days

14. (c) On 31st December, 2005 it was Saturday. \ Angle covered by hour hand in 1 min =
Number of odd days from the year 2006 to the year 2009 2
= (1 + 1 + 2 + 1) = 5 days \ Angle covered by hour hand in 2½ hrs.
\ On 31st December 2009, it was Thursday. 1°
Thus, on 1st Jan, 2010 it is Friday. = 150 min =150 × = 75°
2
y
o
u
rs
m
a
h
26 101 SPEED TEST

b
o
o
28. (c) Total no. of days between 27.3.1995 7. (d) From both the statements:

b
and 1.11. 1994 = 27 + 28 + 31 + 31 + 29 = 146

.w
B D
Now, 146 is not completely divided by 7. It we have one + 10

o
day more then we have 147 days which is completely divided

rd
by 7. Thus, the days of the week on 1 Nov, 1994 was Left 20th Right

p
re
Monday. But the day will be Tuesday ( We have 146 days)
D’s position from the left end cannot be determined.

s
29. (a) 16 - 1- 1997—Thursday.

s
8. (e) From both the statements:

.c
Number of normal year between 1997 and 2000 = 2

o
We know every year has 1odd day.

m
N
Now, number of leap year = 1
K
Leap year has 2 odd days NW NE
\ odd days = 2 + 2 = 4 \ 4 Jan, 2000 was Tuesday.
30. (b) Day is on 28th Feb = Tuesday W E D
Since, the leap year is excluded
\ The day is on 28th March = Tuesday SW SE
S
M
SPEED TEST 20
Clearly, Town M is towards South-East direction of Town K.
1. (c) Age of C = Total age – age of (A + B + D + E) 9. (d) From both the statements K, M and T are siblings of P.
= Total age – 2 × Average ages of (A + B) – 2 × Average ages 10. (c) From I. M
of (D + E) PT
2. (c) From I Thus, it can be found that M is to the north-west of T. So, I
alone is sufficient to answer the question.
From II. M K
Q N T
Thus, it can be deduced that M is to the north-west of T. So,
II alone is also sufficient to answer the question. Thus, the
answer can be found by using either of the statements alone.
W E 11. (e) From I. It is clear that K, D and R are siblings while K and D
are females. But no relation can be found between D and M.
From II. It can be found that R’s father is certainly the father
S-W of K and D as well. Since M is married to R’s father, it means
P H S M is the mother of R.
Thus, after combining I and II, we can definitely say that M
P is to the South-West of Q. is the mother of D. So, both statements I and II are required
From II to answer.
12. (e) D’s position = 15th from right
Q \ R’s position = (15 – 10 =) 5th from right
F = (40 – 5 + 1 = ) 36th from left

13. (c) 20 m
P
Again, P is to the South-West of Q.
30 m
3. (e) From I : K

(+)N M(–)
From II : F (+) Û K (–) D was facing East when he started his journey, from
Combining both: F(+) Û K(–) statement I.

N(+) M(–)
Hence K is the mother of N. 30 m
4. (e) Statement II:
Rani’s age = x yrs.
Þ 2x + x = 72 Þ 3x = 72 Þ x = 24 yrs. 20 m
Using this with Statement I, we get
D was facing East when he started his journey, from statement
24 II.
Age of Nidhi = 3 times younger than Rani’s age = = 8 yrs.
3 14. (d) From statement I and II.
\ Both statements I and II are Sufficient. A
5. (a)
6. (e) Since the ages of none of them is given, no conclusion can be – – +
drawn through both the statements. B C D
y
o
u
rs
m
a
h
SOLUTIONS 27

b
o
o
15. (c) From Statement I

b
B D

.w
o
R 10 km S From Right
10 Girls

rd
20th

p
or

re
2 km 2 km D B

s
s
T 10 Girls

.c
P 5 km Q

o
North

m
From Right
20th
West East Both statements are not sufficient to answer the
question.
South 22. (e) According to statement I
Town K is in north-west with respect to town D.
Required distance = PT = PQ + QT = (5 + 10) km N
= 15 km From Statements II N-W N-E
K
2 km Q
T
P E
W
2 km 2 km

R 13 km S D
W-S S-E
Require distance = PT = PQ + QT = (2 + 13) km = 15 km S
16. (e) From the both statements According to statement II.
4th Town M is in S-E w.r.t to D
21
R S
15th N
N-W N-E
There are 21 students between R and S. D
17. (d) From the the statements
Suneeta has two children. However, she may have more sons.
18. (c) From Statement I W E

M S-E S-E
H
S
Combining statement I and II
N
K W-N N-E
N
X Y NW NE
From statement II W E D W E
X R SE
SW
S
M S-W S-E
S
Town M is in S-E w.r.t town, K.
Y Both statements required to answer the question
23. (d) From statement I
K and M are sister of T
19. (e) T
20. (a)
21. (d) According to statements I sister sister
D is the 20th from right end.
According to statement (ii) K M
10 girls are in between B and D. From statement II.
Combining statement (i) and (ii). T’s father is husband of P’s mother.
y
o
u
rs
m
a
h
28 101 SPEED TEST

b
o
o
husband \ F £ N >R ³ H

b
Conclusion : I. H d N Þ H > N [not true]

.w
II. F # R Þ F < R [not true]

o
rd
father mother If neither conclusion I not II is true.

p
6. (b) Accordingly,

re
M # T Þ M< T

s
T P

s
T@ K Þ T £ K

.c
From statements - I and II

o
K$N Þ K ³ N

m
husband \ M<T £ K ³ N
Conclusion : I. M # N Þ M < N [not true]
II. K d M Þ K > M [not true]
father mother Only conclusion II is true.
brother or sister 7. (c) Accordingly,
T%H Þ T=H
T P
H$W Þ H ³ W
sister sister
\ T=H ³ W
Conclusion : I. W # T Þ W < T [true]
K M
II. W % T Þ W = T or
Both statements are not sufficient to answer the If either conclusion I or II is true. [true]
question. 8. (a) Accordingly,
24. (b) According to Statement I NdK Þ N>K
Arun did not went London on sunday. K# D Þ K<D
According to statement II:- D%M Þ D=M
Arun’s brother went London on Friday \ N> K< D= M
Hence only statement II are sufficient to answer the Conclusion : I. M d K Þ M > K [true]
question. II. D d N Þ D > N [not true]
25. (e) From statement I Only conclusion I is true.
9. (e) Accordingly,
new good clothes Þ 5 3 9
J$ B Þ J³B
From statement II B% RÞ B=R
good clothes are costly Þ 9 6 7 3 RdF Þ R> F
Combining statement - I and II \ J ³ B= R> F
Conclusion : I. F # B Þ F < B [true]
new Þ 5
II. R @ J Þ R £ J [true]
Both statements are required to answer the question.
Both conclusion I and II are true.
SPEED TEST 21 10. (e) V # S Þ V £ S
S© LÞS<L
1. (a) H > K ³ R > T > L L© JÞL<J
Conclusions:
I. H > L : True hence V £ S < L < J
II. K > T : Not True Conclusions
2. (e) P = N > D ³ G < B = J I. V © L Þ V < L (True)
Conclusions:
I. G < P : True II. S © J Þ S < J (True)
II. G < J : True 11. (b) M # R Þ M £ R
3. (d) F £ C ³ V = Z > X = U R©JÞR<J
Conclusions: J#HÞJ£H
I. V < U : Not True
II. Z < F : Not True hence M £ R < J £ H
4. (b) Q £ E = 1 > N ³ R ³ S Conclusions
Conclusions : I. M # H Þ M £ H (False)
I. E = S : Not True
II. S £ N : True II. R © H Þ R < H (True)
5. (d) Accordingly, 12. (a) H $ F Þ H ³ F
F@ N Þ F £ N F@ GÞF=G
NdR Þ N >R G «MÞG>M
H@ R Þ H £ R hence H ³ F = G > M
y
o
u
rs
m
a
h
SOLUTIONS 29

b
o
5. (c) It is mentioned in the statement that Mr. X has been

o
Conclusions

b
I. H « M Þ H > M (True) declared successful in the preliminary screening for

.w
II. H « G Þ H > G ( False) the post of Director of KLM Institute. Therefore, either

o
he will be selected or will not be selected as Director of

rd
13. (d) R©JÞR<J KLM Institute.

p
J«TÞJ>T 6. (d) II may be an assumption which the professor is

re
T#LÞT£L assuming before passing his statement but it definitely

s
s
cannot be a conclusion. Hence II does not follow. I

.c
hence R < J > T £ L

o
may or may not be possible. Hence I does not follow.

m
Conclusions 7. (b) It is clear that either there is no facility for health
I. R @ T Þ R = T ( False) insurance available or it is available for only affluent
II. J @ L Þ J = L (True) sections. Hence I cannot be definitely concluded. II
14. (a) W@TÞW=T follows from the given statement, as ‘limited resources’
of the person suggests that he will go to a hospital
T $ K ÞT ³ K
which provides treatment on nominal charges or free.
K«FÞK>F 8. (a) Only conclusion I follows. The statement talks about
hence W = T ³ K > F dedicated ordinary doctros but that in no way infers
Conclusions that extra ordinary specialists are not dedicated to their
profession. So conclusion II follows. I is true in the
I. W $ K Þ W ³ K (True) context of the scenario prevailing in the country.
II. W @ K Þ W = K (False) 9. (c) Either I or II can follow. As the government would be
15. (d) R £ D ...(i); D > W ...(ii); B ³ W ...(iii) reviewing the diesel prices in light of the spurt in the
None of the inequations can be combined. international oil prices, the gove can either decide to
However, either I (W < R) or III (W ³ R) must be true. increase or keep the price stagnant (increasing subsidy.)
16. (c) H ³ V ...(i); V = M ...(ii); K > M ...(iii) 10. (d) The availability of vegetables is not mentioned in the
Combining these, we get H ³ V = M < K given statement. So, I does not follow. Also, II is not
Hence K > V and I follows. directly related to the statement and so it also does not
Also, M £ H and II follows. follow. Probably the demand is surpassing the supply.
11. (d) I and II are assumptions and not conclusions.
But H and K can't be compared. Hence III does not follow.
12. (d) The statement does not say why the poor societies
17. (a) K < T ...(i); T ³ B ...(ii); B £ F ...(iii) suffer. Hence I does not follow. II also does not follow
Clearly, the inequations can't be combined. because the statement merely states a fact; it does not
18. (a) Z < F ...(i); R £ F ...(ii); D > R ...(iii) look into the merits of the fact.
Clearly, the inequations can't be combined. 13. (c) As Praveen has not yet returned, he might have got
19. (b) M > R ...(i); R = D ...(ii); D £ N ...(iii) killed or might have survived. Hence (c) is the correct
Combining these, we get M > R = D £ N option.
I does not follow as M and N can't be related. 14. (b) Nothing has been said in the statements which imply
N ³ R and II follows. that VCRs and being now manufactured indigenously.
M > D and III follows. Therefore, I is invalid. Since import licence on VCR’s
20. (b) 21. (d) 22. (e) has been withdrawn, they can be now freely imported.
15. (d) Nothing of the sort can be concluded as given in two
SPEED TEST 22 conclusions on the basis of the statements.
16. (d) Both of the conclusion are invalid.
1. (a) Only conclusion I seems to be reasonable. Considering 17. (b) Unless absolute figures are given, no conclusin of the
the different nature of IT Companies different type I can be made. Since average no. of students per
parameters should be employed for rating. It is not teacher (60) in rural areas is higher than the average
necessary that if separate rating agency is established no. of students per teacher (50) in urban areas, we can
for IT companies the investors will get protection of conclude that more students study with the same
their investment. Therefore, conclusion II does not teacher in the rural areas as compared to those in the
follow. urban areas.
18. (d) This statement does not mention anything about
2. (a) By increasing the manufacturing capacity the Company healthy people. Neither does it mention about evening
"Y" would compete reasonably on the cost front. And, walks. Hence none of the conclusions follows.
as such it can improve the quality of its products. Hence, 19. (a) Only this can balance the equation.
conclusion I follows. Conclusion II seems to be an 20. (e) All the given choices would lead to an increase in the
assumption. number of visits to health facilities.
3. (b) Considering the amount of loss incurred by Public
Sector Units it seems to be true that the Government SPEED TEST 23
did not take care in the matter of investment in the
Public Sector Units. The use of term "only" in the 1. (b) Read the last two sentences of the paragraph.
conclusion I makes it invalid. 2. (b) Read the line-"If the agriculture sector does well and
4. (b) Clearly, only conclusion II follows. It is not clear how world trade conditions improve."
the population of developing countries will not increase 3. (c) The Indian economy depends on agricultural sector
in the future. which depends a lot on monsoon.
y
o
u
rs
m
a
h
30 101 SPEED TEST

b
o
o
4. (c) Process of poverty measurement needs to take into

b
account various factors to tackle its dynamic nature. 3. (a) 1 R

.w
5. (a) It may not be possible to have an accurate poverty G B

o
rd
measurement in India. W Y 2
6. (b) Increase in number of persons falling into poverty varies

p
O 3

re
considerably across the country over a period of time. G– O

s
7. (c) 8. (e) 9. (c) 10. (b)

s
R–W

.c
11. (d) 12. (e)

o
B– Y
13. (c) The argument boils down to the following, including

m
4. (a) B & K can't opposite to A
the unstated assumption provided by (c): M & K '' '' to H
Premise: Students get enough reading practice already. B & P '' '' to H
Unstated assumption (c): The reading program From above statements
provides only reading practice. H can’t be opposite to B, K, M, P
Conclusion: The reading program is unnecessary. Thus H will in opposite of A.
(a) is not a necessary assumption. The argument is not (Qs. 5-8). Since, there are 64 smaller cubes of equal size, therefore,
concerned with whether improved reading skills would n = no. of divisions on the face of undivided cube = 4
help the students learn history and science. Rather, the 5. (c) no. of cubes with no face coloured = (n – 2)3
argument involves whether the new program would = (4 – 2)3 = 8
help improve reading skills. 6. (d) no. of cubes with one face painted = (n – 2)2 × 6
(b) is not a necessary assumption. The argument is = (4 – 2)2 × 6 = 24
that no additional reading practice is needed, regardless 7. (a) Number of cubes with two red opposite faces = 0
of which program provides that practice. (none of the cubes can have its opposite faces
(d) is not a necessary assumption. The argument does coloured)
not aim to compare the importance of one discipline 8. (c) Number of cubes with three faces coloured
over another. = 4(cubes at top corners) + 4(cubes at bottom corners)
14. (a) (a) is the correct choice as the passage says that =8
“efficiency is present everywhere, this makes it all 9. (d)
pervading”. The passage does not suggest that 10. (d) From figure, 6 is opposite 4
efficiency does not pay or can be more of a torture. 1 is opposite 2
15. (c) Only this follows by combining the two statements. 3 is opposite 5
11. (c) is opposite to =
SPEED TEST 24 × will opposite to +
¸ will opposite to –
1. (d) 2. (b) 3. (b) hence (a) (b) (d) are not identical.
4. (c) 3 goes in the opposite direction: it talks about the utility 12. (a) When the sheet shown in fig. (X) is folded to from a
of fashion whereas the author does not talk of fashion cube, then the face bearing the dot lies opposite to the
approvingly. shaded face, the face bearing a circle (With ‘+’ sign
5. (b) Read the last sentence of the paragraph. inside it) lies opposite to a blank face and the remain-
6. (d) 7. (a) 8. (a) 9. (a) ing two blank faces lie opposite to each other. Clearly,
10. (b) 11. (d) 12. (d) 13. (c) the cubes shown in figures (B) and (D) cannot be
14. (d) formed since they have the shaded face adjacent to the
15. (d) The use of word ‘hassle-free’ suggests that the face bearing a dot and the cube shown in fig. (C) can-
company assumed that people seek convenience and not be formed since it shown all the three blank face
comfort. adjacent to each other. Hence, only the cube shown in
fig.(A) can be formed.
SPEED TEST 25
1. (d) 1 is correct as it clearly shows that 1 and 5 are the top 13. (c) The symbols adjacent to D are d , ×, and .
and bottom. 2 and 6 are on the sides of 4. So 2 is opposite Hence, the remaining symbol (?) will be opposite
6. 2 is correct on the same lines. 3 is opposite 4. 3 is to D.
correct as I & IV clearly tells that 4 is opposite 3. All the 14. (a) The numbers adjacent to 6 are 2, 3, 4 and 5 (from the
three statements regarding the figure given in the first three figure). Hence, number 1 will be opposite
question are correct. Hence, the answer is (d). to 6.
2. (a) As it is clear from the figure that face 2 is adjacent to
face 3. 15. (a)

2
5
1 4 3
The total no. of all such smaller cubes
6 = 4 × 2 (layers) = 8.
y
o
u
rs
m
a
h
SOLUTIONS 31

b
o
o
16. (b) There are 10 cubes. SPEED TEST 27

b
17. (c) The number on the fall opposite to face having 1 is

.w
6 because 3, 2 and 4 are adjacent faces of 6. 1. (a) From problem figure (4) to (5) the lower design is

o
18. (a) The number opposite to 4 is 2 because 5, 1, 6 and 3 reversed laterally while the other design moves to the

rd
are adjacent faces of 4 opposite side. Similar changes would occur from

p
re
19. (a) Bottom face of figure I means opposite face of 5. It will problem figure (2) to (3).

s
be 3 because 4, 1, 6 and 2 are adjaceent faces of 5. 2. (a) Symbol changes positions as shown in both the

s
.c
20. (a) Symbols adjacent to are –, ×, ¸ , +. Therefore, diagrams alternately and symbols in place of the sign

o
symbol D will be opposite to . ‘·’ remain unchanged in each of the successive figure.

m
SPEED TEST 26 · · ·
1. (d) The shifting of the elements takes place in such a way
that the change is completed in four steps. So the
· ·
change from figure five to six will be similar to the Figure (1) to (2) Figure (2)
change from figure one to figure two. 3. (c) Each symbol of the figure rotates in Anticlockwise and
2. (a) In each step, two pairs of elements get changed, a new symbol replaces the symbol at the top which is
beginning from the upper left and lower right. The inverted alternatively in each of the successive figures.
change takes place in three successive steps. 4. (e) From problem figure (1) to (2), first and second figure
3. (c) The main design rotates respectively through 45° interchange their position and are reversed at the new
clockwise, 180° and 90° anticlockwise after every two positions and at the same time, third figure remains
figures. The shaded leaflet rotates through 45° unchanged. From problem figure (2) to (3), second and
clockwise, 90° clockwise, 135° clockwise, 45° third figures interchange their positions and are
anticlockwise and 90° anticlockwise. reversed at the new position and the figure (1) remain
4. (d) Watch the rotation of each element separetely. The unchanged. The same problem is repeated alternatively.
triangel rotates by 90° ACW and 180°. The circle rotates 5. (a) In each step, both the line segments close to the sides
45°, 135°, 90° CW 45° CW ... ‘C’ rotates 45°, 90°, 135°, of the hexagon, move to the adjacent sides in a
180° CW. Clockwise. Also the line segments at the corner move
5. (b) In each step elements interchange in pairs while one to the adjacent corner Clockwise and their number
element beginning from one end is replaced by a new increases by one, in the first, third, fifth steps.
one. The line of orientation rotates by 45° ACW. 6. (e) From element I to II the design rotates through 180°
6. (d) In each step the elements of the upper row shift from 7. (b) From element I to II the design is mirror image after
left to right in cyclic order while elements of the lower being rotated through 90° clockwise.
row shift from right to left in cyclic order. 8. (c) From element I to II the design is enclosed by another
7. (b) In each step, the whole figure rotates by 45° ACW. The design.
middle element interchanges with elements on either 9. (e) From element I to II the upper design encloses the lower
side alternately while the third element is replaced by a design.
new one. 10. (d) From element I to II the design is divided into four
8. (c) In each step the whole figure rotates by 90° ACW while equal parts and the lower left part becomes shaded.
one of the end elements is replaced alternately on either 11. (c) For the square follow if 1 = 5 then 2 = 6 rule. In alternate
side. steps the lower-row elements go to the upper row and
9. (a) In the first step the elements shift from the upper left to new elements appear in the lower row.
lower right ® middle left ® upper right ® lower left ® 12. (b)
upper left. In the next step the elements shift one step 13. (b) In each step the corner elements rotate 90° CW and
CW in cyclic order. shift one side ACW alternately. The middle figure
10. (e) In each step the upper element rotates by 90° ACW. rotates 180° and 90° CW alternately.
The lower element gets inverted and a curve is added 14. (e) In alternate steps the upper left shifts to centre ® lower
to it on the upper side. right ® upper left. The upper right shifts to right middle
11. (d) In alternate steps the elements shift one-and-a-half sides ® upper-middle ® upper right and lower left ® left
CW while one of the elements beginning from the ACW middle ® lower middle ® lower left.
end gets replaced by a new one in each step. 15. (e) In each step the side arrow rotates 90° CW and shifts
12. (b) In each step the whole figure rotates by 90° CW while one side CW. The smaller arrow rotates 45° ACW and
one element is added in each step alternately on CW the larger arrow 45° CW alternately.
and ACW end. SPEED TEST 28
13. (b) In each step the whole figure rotates by 90° ACW and
1. (c) Meaningful words : ARE, EAR
an arc is added on the CW side.
14. (b) In each step the triangles rotate by 90° CW. The shading 2. (d) As L®8 and H ® 7
of the right triangle changes alternately. The shadings A®& I ®*
of the middle and left triangles change in each step in a
set order. T®4 R ®3
15. (a) In each step the quadrilateral rotates by 90° ACW while E®$ E®$
it shifts half a side CW alternately.
y
o
u
rs
m
a
h
32 101 SPEED TEST

b
o
o
Similarly, 21. (c) Play A is staged on Wednesday. So plays F, B and D are

b
H ¾¾ ®7 A ¾¾
®& staged after play A is staged.

.w
I ¾¾ ®* L ¾¾
®8 22. (b) All others are staged one after the other.

o
rd
3. (b) Others relate to ‘parts of tree’. 23. (e) Play E is staged on Monday, the first day.

p
4. (d) All others are parts of a car. 24. (c) 25. (d)

re
5. (d) DG, IG and SN 26. (a) B ® 9; A ® 2; R ® *; N ® %; I ® #; S ® 4

s
s
6. (b) 7. (c)

.c
27. (b) D ® 2; M ® @; B ® 9; N ® %; I ® #; A ® 6

o
8. (c) 9. (b) Condition (ii) is applied.

m
10. (e) 11. (a) 28. (c) I ® $; J ® 8; B ® 9; R ® *; L ® £; G ® #
12. (b) 13. (d) Condition (ii) is applied.
14. (c) 15. (d) 29. (c) 30. (c)
16. (a) Some hens are fish. (I-Type) 31. (a)
32. (d) V $ W ÞV < W
W@TÞW³T
T#H ÞT£H
All fish are birds. (A - Type) hence V < W ³ T £ H
I + A Þ I-type Conclusions %
\ Some hens are birds. I. V © T Þ V > T ( False)
This is Conclusion I. II. H % W Þ H = W ( False)
17. (c) Both the Premises are Particular Affirmative. No 33. (b) H © M Þ H > M
Conclusion follows from Particular Premises. M@ EÞM³E
Conclusions I and II form Complementary Pair. Therefore, E$ CÞE<C
either Conclusion I or II follows. hence] H > M ³ E < C
18. (e) All bats are boys. (A-Type) Conclusions %
I. C @ M Þ C ³ M ( False)
II. H © E Þ H > E (True)
34. (e) N @ J Þ N ³ J
All boy are gloves. (A-Type) J % RÞJ =R
A + A Þ A type Conclusion. R©HÞR>H
\ All bats are gloves. hence] N ³ J = R > H
This is Conclusion II. Conclusions %
Conclusion I is Converse of this Conclusion. I. R # N Þ R £ N (True)
19. (d) II. N © H Þ N > H (True)
20. (b) Some doctors are nurses. (I-Type) 35. (d) L @ K Þ L ³ K
K ©AÞ K >A
A$WÞA<W
hence, L ³ K > A < W
All nurses are patients. (A-Type) Conclusions % I. W $ L Þ W < L ( False)
I + A Þ I -type Conclusion.
II. L # W Þ L £ W ( False)
\ Some doctors are patients.
36. (a) 37. (b)
Conclusion II is Converse of this Conclusion. 38. (c) 39. (d)
(Qs. 21-25) : 40. (e)
The given information can be summarized as follows.

Plays Days
A Wednesday
B Friday
C Tuesday
D Saturday
E Monday
F Thursday
y
o
u
rs
m
a
h
SOLUTIONS 33

b
o
o
SPEED TEST 29 11. (d) None of options gives us a four digit perfect square.

b
12. (a) Clearly, 1000 + 1> 1000 × 1

.w
1. (b) – 1.5 will lie leftmost on the number line, hence it is the Hence, the required positive integer is 1.

o
rd
smallest. 3
13. (a) Number of women = 120 ´ = 72

p
2. (b) According to the question, 5

re
s
2 3 Number of men = 48

s
the required value = ´ 114 - ´ 68 = 76 - 51 = 25

.c
3 4 Þ number of married persons = 80

o
and the number of unmarried persons = (120 – 80) = 40

m
3. (d) Let the number be x.
If all the men are assumed to be married, then the number
x x x of married women could be 80 - 48 = 32
Now - = 3 or = 3 or x = 36
3 4 12 Þ maximum number of unmarried women
= 72 - 32 = 40
4. (d) Clearly, 1 < 1 = 7 < 7
7 7 7 1089 + 1
14. (a) Number of odd numbered pages = = 545
2
1
Þ is the smallest number.. 15. (b) Let p = 5, 7,11,13, ............
7
For p = 5, (p2 - 1) = 24
1
5. (c) th part is black. Half of the remaining
8 For p = 7, (p2 - 1) = 48

i.e.
1 7 7
´ = is yellow.. For p = 11, (p2 - 1) = 120
2 8 16
For p= 13, (p2 - 1) = 168
7 ...................................................................
Therefore, the part left = ,
16 ...................................................................
...................................................................
1
which is equal to 3 = 3.5 cm. Clearly, all the above numbers are divisible by 24.
2 16. (a) 9 stubs are used to make 1 candle
3.5 1044
Hence, length of the pencil = = 8 cm \ 1044 stubs will be used to make or 116 candles
7 9
16 17. (c) Let the whole number be x
According to question
6. (a) 25 ´ 92 = 32 ´ 81 = 2592 .
\ Difference = 25 × 92 – 2592 = 2592 – 2592 = 0 69
x + 20 =
Hence, the numerical difference is 0. x
7. (b) Let the number of buffaloes be x and number of ducks Þ x2 + 20x = 69
be y. Then number of legs = 4x + 2y and the number of Þ x2 + 20x – 69 =0
heads = x + y Þ x2 + 23x – 3x – 69 = 0
Now, 4x + 2y = 2(x + y) + 24 Þ x ( x + 23) – 3 ( x + 23) = 0 Þ ( x + 23) ( x – 3) = 0
or 2x = 24 or x = 12 \ x = 3 or – 23, Hence, 3 is only whole number.
8. (a) The numbers– 264, 396, 792, and 6336 are divisible by 18. (b) A clock strikes 4 in 9 seconds.
132. 9
\ clock strikes 1 in seconds.
9. (c) Let the number be x. 4
112 9
Now, x + 13x = 112 or 14x = 112 or x = =8 Þ clock strikes 12 in ´ 12 seconds
14 4
10. (d) We test i.e., clock strikes 12 in 27 seconds.
19. (c) Given, numbers are 50, 35 and 35.
3600 Now, place value of 3 is 30 and 30 in the numbers 35
= 400, which is not a perfect cube.
9 and 35 respectivvely.
\ Sum of the place values = 30 + 30 = 60
3600 20. (b) Below given are those numbers between 500 and 600 in
= 72, which is not a perfect cube.
50 which 9 occurs only once.
3600 509, 519, 529, 539, 549, 559, 569, 579, 589, 590, 591, 592,
= 12, which is not a perfect cube 593, 594, 595, 596, 597, 598.
300 These are 18 numbers.
3600 21. (c) From the given alternatives,
Now, = 8 = (2)3 , which is a perfect cube
450 112 × 114 = 12768
Hence, (d) is the correct option. \ Larger number = 114
y
o
u
rs
m
a
h
34 101 SPEED TEST

b
o
o
22. (a) Out of the given alternatives, 2. (d) Let the number be x.

b
Q x2 – (12)3 = 976

.w
137 × 139 = 19043
\ x2 = 976 + 1728 = 2704

o
\ Required smaller number = 137

rd
23. (d) Let the numbers be x and (x + 1), \ x = 2704 = 52

p
\ x(x + 1) = 8556 3. (c) Q 5 chairs + 8 tables = `6574

re
or, x2 + x – 8556 = 0 \ 10 chairs + 16 tables = 6574 × 2 = ` 13148

s
s
or, x2 + 93x – 92 x – 8556 = 0

.c
4. (b) Let the number be x.

o
or, (x2 + 93) (x – 92) = 0 Q x2 + (56)2 = 4985

m
\ x = 92 Þ x2 = 4985 – 3136 = 1849
24. (b) 382 = 1444 \ x = 1849 = 43
392 = 1521
\ Required number = 1521 – 1500 = 21 5. (a) Units digit in (7 4 ) = 1 . Therefore, units digit in (7 4 )8
455.8 i..e. 7 32 will be 1. Hence, units digit in
25. (d) Number of pieces = = 53
8.6 (7)35 = 1´ 7 ´ 7 ´ 7 = 3
26. (c) Amount received by each student
Again, units digit in (3) 4 = 1
15487
= = » `204 Therefore, units digit in the expansion of
76
(34 )17 = (3) 68 = 1
æ 1ö
27. (c) ç 1 - ÷ of the number = 84 Þ Units digit in the expansion of
è 5ø
(371 ) = 1´ 3 ´ 3 ´ 3 = 7
æ 84 ´ 5 ö
\ number = ç ÷ = 105 and units digit in the expanison of (1135 ) = 1
è 4 ø
28. (d) A + C = 146 Hence, units digit in the expansion of
or A + A + 4 = 146 7 35 ´ 371 ´ 1155 = 3 ´ 7 ´1 = 1
146 - 4 1
or A = = 71 6. (b,c) Here, 0 < p < 1, so let p =
2 2
\ E = A + 8 = 71 + 8 = 79
1 æ 1 1 1 ö
280 Clearly, p < çèQ < or < 2÷
ø
29. (d) Lowest number of set A = – 4 = 52 p 2 1/ 2 2
5
Lowest number of other set = 52 × 2 – 71 = 33 1 1 1
Also, > p Q > or 4 > 0.707
\ Required sum = 33 + 34 + 35 + 36 + 37 = 175 p2 2 2
30. (c) Let total number of goats be x. æ 1ö
çè ÷ø
Then, total number of hens = (90 – x) 2
So, x × 4 + (90 – x) × 2 = 248 48 ´ 9
Þ 4x – 2x = 248 – 180 7. (a) 9 lemons cost = = 144 paise = cost of 4 mangoes
3
68
x= = 34 144
2 Þ cost of 3 mangoes = ´ 3 = 108 paise
4
SPEED TEST 30 = cost of 5 apples
or cost of 9 oranges = 108 paise
1. (c) Let Farah’s age at the time of her marriage be x. 108
Þ cost of one orange = = 12 paise
9
Then, (x + 8) = x ´ 9 8. (a) Let x = 1 × 2 × 3 × 4 = 24
7
Therefore, n = 1 + 24 = 25 = odd
9x Clearly, n is an odd integer and a perfect square.
Þ -x=8 This is true for all values of x (product of any four
7
consecutive integers).
8´7 9. (a) By remainder theorem,
Þx= = 28 years
2 96 will have the remainder 1 as 9 has the remainder 1.
\ Farah’s present age = 28 + 8 = 36 years
96 + 7
1 Also will have the same remainder as
\ Daughter’s age 3 years ago = 36 ´ -3 8
6
= 3 years (1) 6 + 7
which has the remainder equal to 0.
8
y
o
u
rs
m
a
h
SOLUTIONS 35

b
o
o
It means that number of cats must be divisible by 5 and
784 (73 )28 (343)28

b
10. (b) = = number of dogs must be divisible by 4.

.w
342 342 342 This condition follow, if cat, C = 15, dog, D = 40

o
or C = 35, D = 20.

rd
(343) 28 In other cases, (1) will not be satisfied.

p
By remainder theorem, will have the same

re
342 If C = 15, D = 40, then 3 + 10 = 13 pets were adopted.

s
If C = 35, D = 20, then 7 + 5 = 12 pets were adopted.

s
128

.c
remainder as i.e. the remainder is 1. But 13 > 12.

o
342 17. (d) 6n2 + 6n = 6(n2 + n) = 6n(n + 1)

m
Alternatively : n(n + 1) is always divisible by 2, as the product of two
consecutive natural numbers is always divisible by 2.
(343) 28 (342 + 1) 28
= Þ (6n2 + 6n) is divisible by 6 and 12.
342 342 Hence, option (d) is correct.
(342)28 + 28 C1 (342) 27 + ... + 28C 27 342 + 1 M
= 18. (d) W= , M – 10 = W + 5
342 2
Clearly, 1 is the remainder. [where M ® men, W ® women]
11. (a) Let the number of students in each row is n and the On solving, we get M = 30, W = 15.
number of row is r. Then the number of students in the \ M + W = 45.
class will be nr. 19. (b) Let the number of boys = B.
According to the question, 3 2
(n + 4) (r – 2) = nr ............(1) \ B = 18 Þ B = 24 = of class
4 3
and (n – 4) (r + 4) = nr ............(2)
\ Strength of the class = 36
on simplifying equations (1) and (2), we get the system
\ No. of girls = 36 – 24 = 12.
of equations
20. (d) Suppose husband’s age be H years.
n – 2r + 4 = 0
Then wife’s age W = H – 9
n–r–4=0
On solving this system, we obtain r = 8; n = 12 H-9
Hence, nr = 96 Son’s age S =
2
12. (d) Let each of them had x bullets after division. Then,
total number of bullets they had after using 4 bullets H
Daughter’s age D =
each = (x - 4) + (x - 4) + (x - 4) = 3x - 12 3
According to question,
Now, 3x - 12 = x or x = 6
Original no. of bullets = 6 × 3 = 18 H H -9
+7 = Þ 2H + 42 = 3H - 27
13. (b) I is false as sum of two negative integers is always 3 2
negative. Þ H = 42 + 27 = 69
II is true, as product of two negative integers is always \ W = 60.
positive. 21. (d) Let the face value of the National Savings Certificates
III is not always true as x –y can be positive or negative purchased by Soma in the first year be Rs.x.
according as x > y or x < y. \ x + (x + 400) + (x + 800) + (x + 1200) + (x + 1600) +
14. (a) Let the number be x. Then, as per the operation (x + 2000) + (x + 2400) + (x + 2800) = 48000
undertook by the student, we have Þ 8x + 11200 = 48000
x + 12 Þ 8x = 36800 Þ x = 4600
= 112 Þ x = 660 22. (c) T = P + 4 = 9; N = T – 3 = 6.
6
23. (a) Let the Number be N
660 \ N = 765x + 42
Hence, the correct answer = + 12 = 122
6 765 is divisible by 17
15. (d) Let the present age of Harry and George be x and y, \ if N is divided by 17, remainder will be 42 – 34 = 8.
respectively. 24. (a) From option (a),
Then, 210 – 2n = 960 Þ 1024 – 2n = 960 Þ 2n = 64
\ n = 6.
1
x= y ...........(i) Similarly we can try for options (b) and (c). Hence, any
3 other option does not satisfy the given equation.
1 25. (c) Let the total number of packages be x.
and ( x - 5) =( y - 5) ...........(ii)
4 2
After uploading x packages remaining packages are
From (i) and (ii), y = 45 years 5
Hence, the required age = (5 + y) = 50 years
2 3
16. (c) Total no. of cats and dogs = 55 .............. (1) x– x= x
Out of these, 1/5 of cats + 1/4 of dogs had been adopted. 5 5
y
o
u
rs
m
a
h
36 101 SPEED TEST

b
o
o
According to the question, 6. (d) To find the capacity we have to take the HCF of 279,

b
341 and 465.

.w
1
When he uploaded another 3 packages then of 279 = 31 × 9 × 3

o
2 341 = 31 × 11

rd
original no. of packages remained. 465 = 31 × 3 × 5

p
re
3x x 3 1x Þ HCF (279, 341, 465) = 31
\ –3= Þ x – =3

s
Capacity of the measuring can be = 31 ml.

s
5 2 5 2

.c
7. (b) H.C.F of co-prime numbers is 1.
Þ 6x – 5x = 30 Þ x = 30

o
So, L.C.M. = 117/1 = 117.

m
Hence, 30 packages were in the van before the first delivery.
8. (a) Required time = LCM of 200, 300, 360, 450 sec
26. (c) Given : Divisor = 2 × remainder
= 1800 sec.
\ Divisor = 2 × 75 = 150 9. (c) The LCM of 18, 22, 30 is 990.
2 So, they will meet each other after 990, ie, 16 min and 30 sec.
Also, Divisor = ´ dividend
3 10. (d) The required number must be a factor of (11284 – 7655) or
3 3629.
Þ dividend = 150 ´ = 225 Now, 3629 = 19 × 191
2 \ 191 is the required number.
27. (a) 24162 = 89x + 43
LCM ´ HCF 2079 ´ 27
Þ x = (24162 – 43) ÷ 89 = 271 11. (c) The required number = = = 297
28. (c) By actual division, we find that 999999 is exactly divisible First number 189
by 13. The quotient 76923 is the required number. 12. (a) Clearly, the required number must be greater than the LCM
29. (b) Clearly, unit’s digit in the given product = unit’s digit in of 18, 24, 30 and 42 by 1.
7153 × 172. Now, 18 = 2 × 32
24 = 23 × 3
Now, 74 gives unit digit 1.
30 = 2 × 3 × 5
\ 7153 gives unit digit (1 × 7) = 7. Also 172 gives unit 42 = 2 × 3 × 7
digit 1. \ LCM = 32 ×23 × 5 × 7 = 2520
Hence, unit’s digit in the product = (7 × 1) = 7. \ the required number = 2520 + 1 = 2521
30. (b) The digit in the unit’s place of 251 is equal to the remainder 13. (d) The first number = 2 × 44 = 88
when 251 is divided by 10. 25 = 32 leaves the remainder 2 HCF ´ LCM 44 ´ 264
when divided by 10. Then 250 = (25)10 leaves the remainder \ The second number = = = 132
88 88
210 = (25)2 which in turn leaves the remainder 22 = 4. Then 14. (b) HCF = 12. Then let the numbrs be 12x and 12y.
251 = 250×2, when divided by 10, leaves the remainder 4×2 = 8. Now 12x × 12y = 2160 \ xy = 15
Possible values of x and y are (1, 15); (3, 5); (5, 3); (15, 1)
SPEED TEST 31 \ the possible pairs of numbers (12, 180) and (36, 60)
15. (c) Bells will toll together again at a time, which is obtained by
1. (b) Q Product of numbers = (LCM × HCF) taking L.C.M. of their individual tolling intervals.
Þ 480 × second number = 2400 × 16 L.C.M. of 9, 12 and 15 = 180 min
Þ second number = 80 They will toll together again after 180 min, i.e. 3 hours.
Time = 8 + 3 = 11 a.m.
2. (c) Let numbers be x and y. 16. (b) Since each rod must be cut into parts of equal length and each
Q Product of two numbers = their (LCM × HCF) part must be as long as possible, so HCF should be taken.
Þ xy = 630 × 9 HCF of 78, 104, 117 and 169 = 13.
Also, x + y = 153 (given)
78
No. of parts from 78cm. rod = =6
since x – y = = (x + y) 2 - 4xy 13
104
Þ x - y = (153) 2 - 4(630 ´ 9) No. of parts from 104 cm. rod =
13
=8

= 23409 - 22680 = 729 = 27 117


No. of parts from 117 cm. rol = =9
3. (a) Product of the numbers 13
= HCF × LCM = 21 × 4641 169
= 21 × 3 × 7 × 13 × 17 No. of parts from 169 cm. rod = = 13 .
13
= 3 × 7 × 3 × 7 × 13 × 17 \ Maximum no. of pieces = 6 + 8 + 9 + 13 = 36
\ The required numbers can be 17. (c) Let the numbers be x and 4x.
3 × 7 × 13 and 3 × 7 × 17 = 273 and 357 Then, 84 ´ 21 = x ´ 4x
4. (b) Required number = HCF of 108 and 144 = 36
5. (c) Time gap between two consecutive ticks or 4 x 2 = 1764
58 609 or x 2 = 441 or x = 21
sec. and sec.
57 608 Þ 4x = 4 × 21 = 84
58 609 Thus the larger number = 84
\ Required time = LCM of and 18. (b) Let us check each of the options here starting with (a)
57 608
LCM of 58 and 609 1218 13 11 133 11 8 65
= = sec (a) + = <5 (b) + = >5
HCF of 57 and 608 19 5 6 30 4 3 12
y
o
u
rs
m
a
h
SOLUTIONS 37

b
o
o
12 1 Therefore, in one hour (60 minutes), then will fall

b
19. (b) = 0.1008, = 0.1

.w
119 10 æ 60 ö
together 8 times ç ÷ excluding the one at the start.

o
è 7 ø

rd
4 7
= 0.102 and = 0.101 28. 2
(b) HCF = 2 × 3 2

p
39 69

re
LCM = 24 × 35 × 52 × 72

s
1 Ist number = 23 × 34 × 5

s
Thus, is the least.

.c
10 2nd number = 24 × 32 × 52

o
observing the above situation, we conclude that the

m
4 4 -4
æ2ö æ3ö 34 æ 2ö third number must be
20. (c) Reciprocal of ç ÷ = ç ÷ = 4 = ç ÷
è3ø è2ø 2 è 3ø x = 22 ´ 32 ´ 33 ´ 7 2 = 22 ´ 35 ´ 7 2
29. (d) Product of numbers = HCF × LCM
34
(a), (b), (d) are all equivalent to 4800 ´ 160
24 Þ The other number = = 1600
480
21. (c) 1.65m = 165 cm
Required length = LCM of 25 and 165 30. (c) The traffic lights will again change at three different
= 825 cm = 8.25 m road crossings simultaneously after the LCM of 48, 72
and 108
5 6 7 i.e., after every (432 sec) 7 minutes and 12 seconds, i.e.
22. (c) = 0.312 , = 0.352 , = 0.388
16 17 18 the earliest at 8 : 27 : 12 hours.
5 6 7
Therefore, < < SPEED TEST 32
16 17 18
29 15 116 + 45 161 1190 1190 ´ ?
23. (d) + = = 1. (a) ´ ? = 3094 or, = 3094
12 16 48 48 7225 85
3094 ´ 85
161 31 192 or, ? = = 221
Therefore, + = = 4 = a whole number 1190
48 48 48 2. (a) Q ?- 2 5 = ( 5 -1 )2 = 5 + 1 - 2 5
161 17 \?=6
And - = 3 = whole number
48 48 ? 25 ? 5 5 1
3. (a) = Þ = = =
31 17 17 36 (11 ´ 3 - 18) 6 33 - 18 15 3
Between and ; is the least fraction. 6
48 48 48 \= = 2
Clearly, the least fraction among the given fractions in 3
4. (c)
17
options is . 5. (e) 169 – 25 – 26 + 7 = (?)2 = 125 = ?2 Þ ? = 125 = 5 5
48 2 2
24. (b) Correct asecending order is 6. (e) (16) - 5 + 169 = (? )
3

256 – 125 + 13 = (?)2


2 9 5 8 144 = (?)2
< < <
3 13 7 11 ? = ± 12
1 7. (b) 2
? = 225 + 2304 + (12 ) = 15 + 48 + 144 = 207
25. (a) Let us take a proper fraction, such as .
2
8. (b) ? = 450 + 890 + 685 = 2025 = 45
1+ 2 3
Now, the new fraction = = 9. (c) ? = ( 6859 )
13
+ 4 = 19 + 4 = 23
2+ 2 4
3 1 447
Thus, > 10. (e) ? = 7´ + 73 - 26 = 149 + 73 - 26 = 196 = 14
4 2 21
x æ 60 - 40 ö 20
26. (b) As x is a positive number, we have =1 ? = 5´ ç = 5´ = 50
x
11. (d) è 2 ÷ø 2
x +1 1 x 12. (d) ? = 42 = 16
=1+ >1, <1
x x x +1 \ ? = 256
x + 2 x + 3 -1 1 255
= =1- <1 13. (b) (?)2 = =3 \ ?= 3
x+3 x +3 x+3 17 ´ 5
27. (b) LCM of 6, 5, 7, 10 and 12 = 420 seconds 9 ´ 16 ´ 5
14. (d) ( ?) 2 = 80 + = 80 + 20 = 100
420 36
= = 7 minutes .
60 Q ? = 100 = 10
y
o
u
rs
m
a
h
38 101 SPEED TEST

b
o
o
( )2 - 2 Þ 34969 = 187

b
15. (a) ?= 6 +1 6 = 6 +1+ 2 6 - 2 6 = 7

.w
Hence, the number of rows = 187
31. (c) 93 × 812 ¸ 273 = (3)?

o
184
32 × 3 × 34 × 2 ¸ 33 × 3 = (3)?

rd
16. (c) 12 ´ + 26 - 73 = ?
23 36 × 38 ¸ 39 = (3)? or 36 + 8 – 9 = (3)?

p
re
= 96 + 26 - 73 = ? = 122 - 73 = 49 = 7 35 = (3)? or ? = 5

s
( 9 ) ´ ( 81 ) ¸ ( 27 ) = ( 3 )( )
3 5

s
17. (b) 169 – 64 – 676 + 2 = (?)2 3 ?
32. (c)

.c
= 169 – 64 – 26 + 2 = (?)2 = 171 – 90 = 81

o
(3)3 × (9)5 ¸ (3)3 × 2 = (3)? or (3)3 × (3)2 × 5 ¸ (3)6 = (3)?

m
\?=9
(3)3 + 10 – 6 = (3)? or (3)7 = (3)?
18. (c) (74 ´ )
676 - 42 ´ ? = 496
33. (e)
?=7
81.1 × 42.7 × 23.3 = 2?
( 74 ´ 26 - 496) 2? = (23)1.1 × (22)2.7 × 23.3 or 2? = (2)3.3 × (2)5.4 × 23.3
?= = 1428 ¸ 42 = 34 = (34)2 = 1156 2? = (2)3.3 + 5.4 + 3.3 or 2? = (2)12.0
42
19. (e) 312 ? = 12
49 ´ 49 ´ 49
20. (e) ?2 = 2 48 ´ 8 = – 21 + 8 + 49 – 14 8 34. (a) ? = (49)3 ¸ (7)2 = = 2401
7´7
= 14 8 – 21 + 57 – 14 8 = 36 = 62 35. (a) 643.1 × 84.3 = 8?
\?=6 Þ (82)3.1 × (8)4.3 = 8? Þ 86.2 × 84.3 = 8?
21. (e) 7365 + 29.16 + ? = 7437.16 Þ 86.2 + 4.3 = 8? Þ 810.5 = 8?
? = 10.5
? = 473.16 – 7394.16 36. (d) 8? = 87 × 26 ¸ 82.4
? = 43 = 1849
82
3 8? = 87 × or 8? = 87 + 2 – 2.4
æ 756 ´ 67 ö 82.4
22. (b) çè ÷ = 250047
804 ø 8? = 86.6 or ? = 6.6
37. (a) ? = (31)31 × (31)– 27
23. (b) 17 + 51 + 152 + 289 ? = (31)31 – 27
? = (31)4
= 17 + 51 + 152 + 17 = 17 + 51 + 169 ? = (961)2

{(12) }
= 17 + 51 + 13 = 17 + 8 = 25 = 5 -2 2
(12 )-4
24. (d) 217 + 64 Þ 217 + 8 = 225 = 15 38. (e) = =1
{(12) } (12 )-4
-2
25. (a) 2

210.25 21025 145 145


26. (d) + Þ + Þ 14.5 + 1.45 = 15.95
100 10000 10 100 64 ´ 64
39. (c) = 6? Þ 68– 3 = 6? Þ 6? = 65 Þ ? = 5
27. (a) 4 2000 45 63
4 16
( 8)
6 3+ 6- 4
85 400 40. (c) 8? =
3 3 6 4
´ ( 64 ) ¸ 84 = 8 ´ 8 ¸ 8 = 8 = 85
5 425
-25 \?=5
Clearly, the required least number is 25.
28. (b) Let the number be x. SPEED TEST 33
Now, according to the question,
x2 – (22)3 = 9516 1. (b)
or, x2 = 9516 + (22)3 = 9516 + 10648 = 20164 2. (e) ? = 456.675 + 35.7683 × 67.909 – 58.876
Þ x = 20164 = 142 = 456.675 + 2428.98 – 58.876
29. (a) Let the number be x. = 2885.66 – 66 – 58.876 = 2826.78 » 2830
According to the question.
(4052 – x2) × 15 = 41340 3. (c) ?=
{(52) 2
+ ( 45 )
2
} = 2704 + 2025 = 4729 = 591.125
8 8 8
41340
Þ 4052 - x 2 = = 2756 Þ x2 = 4052 – 2756 = 1296
15 4. (b) ?= (12.25)2
– 625 = – 25 (12.25)2
\x= = 150.0625 – 25 = 125.0625
1296 = 36
5. (e) ? = 572 + 38 × 0.50 – 16
30. (a) Number of rows = 34969 = 572 + 19 – 16 = 591 – 16 = 575

187 1056 ´ 7 ´ 5 ´ 13
6. (a) ?= = 7280
3 ´ 2 ´ 11
1 34969
1
249
7. (b) ? ´ 9 = 6318 + 26 = 243
28
224 243
2569 \ ? = = 27
367 9
2569 2
\ ? = (27) = 729
´
y
o
u
rs
m
a
h
SOLUTIONS 39

b
o
o
78700 24. (a) ? × 40 = 8059 – 7263 = 796
+ 4 ´ 120 = 45 + 460 = 505 ; 525

b
8. (c) ?»

.w
1750 796
\ ?= = 19.9
9. (c) ? ; 78 + 42 + 9 = 129 ; 130

o
40

rd
1 2 3 1

p
10. (b) ´ ´ ´1715 = 85.75 » 85 4 ´ ? = 4062 ¸ 5 = 4062 ´ = 812.4
25. (a)

re
8 3 5
5

s
561

s
11. (a) ´ 20 = 320.5 » 320

.c
812.4
35 \ ?= = 203.1

o
4

m
12. (d) (15)2 ´ 730 = 225 ´ 27 = 6075
æ 1 ö
3325 152 26. (e) ? = 3.5 ´ (80 ¸ 2.5) = 3.5 ´ ç 80 ´ ÷
13. (d) Given expression implies ? = ´ è 2.5 ø
25 16
= 133 × 9.5 = 1263.5 = 3.5 ´ 32 = 112
27. (c) 13% of 258 – ? = 10
1 2 2 \ ? = 13% of 258 – 10
14. (e) ? =5+ + 2 + + 3+
5 15 3
13
1 2 2 = 258 ´ - 10 = 33.54 - 10 = 23.54
= 10 + + + 100
5 15 3 4 3 5 4 11 5
28. (d) ?= ´2 ¸ = ´ ¸
3 + 2 + 10 15 5 4 8 5 4 8
= 10 + = 10 +
15 15 4 11 8 88 13
= 10 + 1 = 11 =´ ´ = =3
5 4 5 25 25
15. (c) ? = – 15 – 27 – 88 – 63 + 255
= –193 + 255 = 62 29. (e) ? = 623.15 – 218.82 – 321.43 = 623.15 – 540.25 = 82.9
16. (b) Given expression can be written as 30. (b) ? × 50 = 5437 – 3153 + 2284 = 7721 – 3153 = 4568

2525 ´ 0.25 ´ 7 4568


?=
\ ?= = 91.36
= 883.75 50
5
14 57 20 2 3 20 2 1 2 4 SPEED TEST 34
17. (b) ?= ´ ´ = ´ ´ = ´ ´ =
19 70 21 1 10 21 1 1 7 7
1. (a) Assume the third number = x
500 ´ 32 50 ´ 162
18. (e) ?= + = 160 + 81 = 241 According to question
100 100
2 × 280 + x + 178.5 × 2 = 281 × 5
19. (d) 45316 + 52131 – 65229
= ? + 15151 or, 560 + x + 357 = 1405
Þ 32218 = ? + 15151 or, x + 917 = 1405
\ ? = 32218 – 15151 = 17067 or, x = 1405 - 917 = 488
184 ´ 4 184 ´ 4 2. (b) Age of the fourth friend = 31 × 4 - 32 × 3
20. (c) ?= = =8 = 124 - 96 = 28 years
400 ´ 23 4 ´ 23
100 3. (a) Required average
21. (a) ? 4 /3 ´ ?5 / 3 = 23 ´ 128 965 + 362 + 189 + 248 + 461 + 825 + 524 + 234
Þ ?3 = 25 × 27 = 2 12 =
8
\ ? = (212)1/3 = 24 = 16
22. (b) 16% of 450 ¸ ?% of 250 = 4.8 3808
= = 476
16 ? 8
Þ 450 ´ ¸ 250 ´ = 4.8 4. (b) 21a + 21b = 1134
100 100
or, 21 (a + b) = 1134
Þ 72 ¸ 2.5 ´ ? = 4.8
1134
72 a+b= = 54
Þ 2.5 ´ ? = 21
4.8 a + b 54
\ Required average = = = 27
72 2 2
\ ?= =6
4.8 ´ 2.5 5. (e) Let the first number be = 6x
23. (e) \ Second number = 3x
? - 11 = 1521
and the third number = 2x
Þ ? - 11 = 39
According to the question,
Þ ? = 39 + 11 = 50 6x + 3x + 2x = 154 × 3
or, 11x = 154 × 3
\ ? = (50)2 = 2500
y
o
u
rs
m
a
h
40 101 SPEED TEST

b
o
o
16. (c) Total of 10 innnings = 21.5 × 10 = 215

b
154 ´ 3 Suppose he needs a score of x in 11th innings; then average in

.w
\x= = 42
11 215 + x

o
11 innings = = 24 or, x = 264 – 215 = 49

rd
\ Required difference = 6x – 2x = 4x = 4 × 42 = 168 11

p
6. (d)

re
x
7. (c) Average score 17. (a) Let the total journey be x km. Then km at the speed of 25

s
3

s
.c
1
= [221 + 231 + 441 + 359 + 665 + 525] x

o
6 km/hr and km at 30 km/hr and the rest distance

m
4
1
= [2442] = 407 æ x xö 5
6 çè x - - ÷ø = x at the speed of 50 km/hr..
8. (e) Let A = x 3 4 12
According to the question Total time taken during the journey of x km
x + x +1 + x + 2 + x + 3 + x +4 x x 5x 18 x 3x
= 5 × 48 = hrs + hrs + hrs = hrs = hrs
3 ´ 25 4 ´ 30 12 ´ 50 600 100
Þ 5x + 10 = 240
Þ 5x = 230 x 100 1
\ average speed = = = 33 km/hr
\ x = 46 18 x 3 3
\ E = 46 + 4 = 50 600
\ A × E = 46 × 50 = 2300 18. (a) Let the total no. of workers be x.
9. (b) 16a + 16b = 672 Now, 8000 x = 7 × 12000 + (x – 7) × 6000
or, 16 (a + b) = 672 42000
Þ x= = 21
672 2000
\a+b= = 42 19. (d) Let the average score of 19 innings be x.
16
18x + 98
a+b 42 Then, = x + 4 Þ x = 22
Required average = = = 21 19
2 2
The average score after 19th innings
10. (e) Third number = 290
= x + 4 = 22 + 4 = 26
– (48.5 × 2) – (53.5 × 2)
20. (b) Total weight of 45 students
= 290 – 97 – 107 = 86
11. (b) Total age of the family of five members = 24 × 5 = 120
= 45 × 52 = 2340 kg
Total age of the family of five members before 8 years Total weight of 5 students who leave
= 120 – 5 × 8 = 120 – 40 = 80 = 5 × 48 = 240 kg
Total weight of 5 students who join
80
So, Required average age = = 16yr = 5 × 54 = 270 kg
5
Therefore, new total weight of 45 students
12. (e) x + x + 2 + x + 4 + x + 6 = 4 × 36
= 2340 – 240 + 270 = 2370
Þ 4x + 12 = 144 Þ 4x = 144 – 12
2370 2
x=
132
= 33
Þ New average weight = = 52 kg
Þ 4x = 132 Þ 45 3
4
13. (b) Let the score of Ajay = x 21. (c) Total score of 40 innings = 40 × 50 = 2000
Rahul = x – 15 Total score of 38 innings = 38 × 48 = 1824
Manish = x – 25 Let the highest score be x and the lowest score be y.
According to question, x = 63 + 30 Sum of the highest and the lowest score
\ x = 93 = x + y = 2000 – 1824
\ Score of Ajay = 93 Þ x + y = 176 ...(i)
then Rahul = 93 – 15 = 78
then Manish = 93 – 25 = 68
and by question, x – y = 172 ...(ii)
Total marks of Rahul, Manish and Suresh Solving (i) and (ii), we get x = 174
= 3 × 63 = 189 22. (c) Let average cost of petrol per litre be Rs x
\ Suresh = 189 – (78 + 68) = 43
12000
\ Manish + Suresh = 68 + 43 = 111 \ x=
14. (a) Suppose marks got in physics, chemistry and mathematics 4000 4000 4000
+ +
are P, C and M. 7.5 8 8.5
P + C + M = C + 120
\ P + M = 120 12000 3 3
= = =
P+M æ 1 1 1 ö 10 + 1 + 10 2 1 2
= 60 4000 ç + + ÷ + +
2 è 7.5 8 8.5 ø 75 8 85 15 8 17
15. (c) By Direct Formula :
2 ´ 60 ´ 30 2 ´ 60 ´ 30 6120
Average = = = 40 km/hr = = Rs 7.98 per litre
60 + 30 90 767
y
o
u
rs
m
a
h
SOLUTIONS 41

b
o
64.5% of 800 + 36.4% of 1500 = (?)2 + 38

o
23. (d) Let the three numbers are x1, x2, x3 and x3 be the largest 3. (a)

b
number. 516 + 546 = (?)2 + 38

.w
Given, average of three numbers = 135 1062 – 38 = (?)2

o
1024 = (?)2
\ x1 + x2 + x3 = 3 × 135 = 405

rd
(32)2 = (?)2
Since, x3 = 180

p
? = 32

re
x1 + x2 = 225 .......... (i) 4. (c) 41% of 601 – 250.17 = ? – 77% of 910

s
Given x1 – x2 = 25 .......... (ii) » 246 – 250 = ? – 701

s
.c
Solving (i) and (ii), x1 = 125, x2 = 100 » 701 + 246 – 250 = ?

o
Hence, the smallest number = 125 – 25 = 100 » 700 = ?

m
24. (d) Total age of 10 students =12.5 × 10 = 125 years 5. (a) 40.005% of 439.998 + ?% of 655.011 = 228.5
Total age of 20 students = 13.1 × 20 = 262 years 40 x
» ´ 440 + ´ 655 = 229
100 100
\ Average age of 30 students = 125 + 262 = 12.9 years
30 655x 655x
» 176 + = 229 » = 229 - 176
25. (d) Replacing the two numbers and arranging them in 100 100
ascending order, we get 53 ´ 100
31, 35, 35, 36, 38, 45, 46, 52, 55, 60 x= »8
655
Since no. of observation is even
6. (d) 25% of 84 × 24% of 85 = ?
éN æN ö ù 21 × 204 = ?
ê 2 th + çè 2 + 1÷ø th ú 428.4 = ?
ë û 7. (b) 20.06%of 599 + 10.01% of 901 = ?
\ Median = observation
2 20 10
» 600 ´ + 900 ´ =?
( 5th + 6th ) obs. 100 100
= » 120 + 90 = ?
2 \ ? » 210
38 + 45 83 8. (a) 14.2% of 5500 + 15.6% of ? = 1795
= = = 41.5
2 2 15.6
781 + ´ ? = 1795
26. (b) Total of x and y = 40 × 2 = 80 100
x+y+z 80 + 10 90 15.6
\ Average of x, y and z = = = = 30 ´ ? = 1014
3 3 3 100
27. (a) Sum of 11 numbers = 11 × 10.9 = 119.9 1014 ´ 100
?= = 6500
Sum of first 6 numbers = 6 × 10.5 = 63 15.6
Sum of last 6 numbers = 6 × 11.4 = 68.4 9. (e) 36% of 245 – 40% of 210 = 10 – ?
The middle number = Sum of the first six + Sum of the 88.2 – 84 = 10 – ?
the last six – Sum of all the 11 4.2 = 10 – ?
? = 10 – 4.2 = 5.8
= 63 + 68.4 – 119.9
= 11.5 1
10. (a) of 3842 + 15% of ? = 2449
28. (a) Sum of 10 numbers = 402 2
Corrected sum of 10 numbers = 402 – 13 + 31 – 18 = 402 15
1921 + x + = 2449
Hence, correct average = 402 = 40.2 100
10 15
29. (d) Required average number of microwave x´ = 2449 - 1921
100
42 ´ 12 + 20 ´ 10 504 + 200 704 528 ´100
= = = = 32 x= = 3520
12 + 10 22 22 15
30. (d) Temperature on the fourth day 11. (c) 57% of 394 – 2.5% of 996
= 40.2 × 4 + 41.3 × 4 – 40.6 × 7 57 2.5
= 160.8 + 165.2 – 284.2 = 41.8°C = 394 ´ - 996 ´
100 100
= 225 – 25 = 200
SPEED TEST 35 12. (a) 40% of 265 + 35% of 180 = 50% of ?
Þ 265 × 0.4 + 180 × 0.35 = ? × 0.5
1. (b) 76% of 1285 = 35% of 1256 + ? 169
Þ 976.6 = 439.6 + ? Þ 976.6 – 439.6 = ? Þ 106 + 63 = ? × 0.5 Þ ? = = 338
0.5
\ ? = 537
2. (b) (21.5% of 999)1/3 + (43% of 601)1/2 = ? 1 1 21 10
13. (b) 4 ´ 3 + ? = 20% of 120 Þ ´ + ? = 120 ´ 0.2
(1000 × 21.5%)1/3 + (600 × 43%)1/2 = ? 5 3 5 3
(215)1/3 + (258)1/2 = ? Þ 7 × 2 + ? = 24 Þ ? = 10
(216)1/3 + (256)1/2 = ? 14. (c) 14% of 250 × ? % of 150 = 840
(6)3 ×1/3 + (16)2 × 1/2 = ?
6 + 16 = ? 14 150 ´ ?
250 ´ ´ = 840
? = 22 100 100
y
o
u
rs
m
a
h
42 101 SPEED TEST

b
o
o
35 × 1.5 × ? = 840 23. (e) Let the maximum aggregate marks = x

b
840

.w
?=
According to the question,
35 ´ 15

o
40% of x – 4% of x = 261

rd
? = 16
(40 - 4)

p
18 27.5 or x × = 261

re
15. (b) ? = 609 ´ + 450 ´
100
100 100

s
s
= 109.62 + 123.75 = 233.37 » 233 261

.c
125 85 \x= × 100 = 725

o
16. (c) 3060 ´ -?´ = 408 36

m
100 100
24. (b) Population of the town after 2 years
85
3825 – 408 = ? × æ 7 öæ 5 ö
100
= 198000 ç 1 + ÷ç1 - ÷
3417 ´ 100 è 100 øè 100 ø
=?
85 198000 ´ 107 ´ 95
4020 = ? = = 201267
100 ´ 100
x y
17. (d) 500 × = 300 ´ 25. (b) Total expenditure = 44668 + 56732 = 101400
100 100 Total percentage expenditure
Þ 5x = 3y = 100 – 22 = 78 %
5x
Þ y= .... (i) 101400 ´ 100
3 \ Total amount = = ` 130000
78
xy ´ 200
Þ = 60 Þ xy = 3000 26. (b) 75 % of 16 = 12
100 ´100 \ 12% of monthly salary = 6567
5x é 5x ù 6567 ´ 100
Þ x´ = 3000 êQ y = 3 ú \ 100% = = ` 54725
3 ë û
12
3000 ´ 3 27. (d) Total valid votes = 85% of 15200 = 12920
Þ 5x2 = 3000 × 3 Þ x2 =
5 \ Number of valid votes to other candidate
Þ x 2 = 1800 = 45% of 12920 = 5814
28. (c) If the radius is diminised by r%, then
Þ x = 1800 = 2 ´ 3 ´ 3 ´10 ´10
x = 30 2 æ r2 ö 102
Area is diminished by çç 2r - ÷÷ % = 2 ´ 10 - = 19%
185 35 ? è 100 ø 100
18. (d) 400 ´ + 240 ´ = 1648 ´
100 100 100 29. (b) Let B’s salary be ` 100,
? ? then A’s salary = ` 125
Þ 740 + 84 = 1648 × Þ 824 = 1648 ×
100 100 125 - 100 25
% lesser = ´ 100 = ´100
824 ´ 100 125 125
Þ ?= or ? = 50
1648 1
2 50 25
= ´100 = 20%
19. (b) ? = 630 ´ ´ ´ 5
3 100 100 30. (b) Let the original price of apple be ` x /dozen
210 ´ 50 ´ 25 ´ 2 210 54 54 10
Þ ?= Þ ?= = 52.5 4x
100 ´ 100 4 New price ` = /dozen = 4 x - =
20. (b) Number of transferred employees 5 x 12
5
1225 ´ 40
= 40% of 1225 = = 490 æ 5 1ö 5
100 Þ 54 ç - ÷ = Þ 54 çæ 1 ÷ö = 5
x è 4x x ø 6 è 4x ø 6
21. (b) Let the original fraction be .
y 54 ´ 6 4x
Þ 4x = Þ = 12.96
5 5
x + 5x 4 6x 18 x 72 12
Then, =2 Þ = Þ = =
y + 3y 7 4y 7 y 42 7 SPEED TEST 36
x
22. (c) Let the original fraction be = y 1. (d) Total number of students in the school = 819
Number of girls = 364
According to the question,
\ Number of boys = 819 - 364 = 455
350 \ Required ratio = 435 : 364 = 5 : 4

100 = 7 7x 7 x 7 8 8
400 9 Þ 8 y = 9 Þ y = 9 ´ 7 = 9 æ2 ö
y´ 2. (d) Share of Urmila in dividend = ç ´ 57834 ÷ = ` 19278
100 è6 ø
y
o
u
rs
m
a
h
SOLUTIONS 43

b
o
o
3. (b) Let the present ages of Richa and Shelly be 13. (a) Iron Copper

b
5x and 8x years. Alloy I 8 : 6 14 kg.

.w
According to the question, Alloy II 36 : 6 42 kg.

o
44 : 12 56 kg

rd
5 x + 10 7
=

p
After 10 years, 1 1 1
8 x + 10 10 : : = 6:4:3

re
14. (b) Ratio of sides =
2 3 4

s
or, 56x + 70 = 50x + 100 or, 56x – 50x = 100 – 70

s
æ 6ö

.c
or, 6x = 30 Largest side = ç104 ´ ÷ cm = 48 cm

o
è 13 ø

m
30 15. (d) Let the numbers be x and y.
\x= =5
6
x 4
\ Shelly's present age = 8x = 8 × 5 = 40 years \ y=7
4. (d) The sum of money is not known.
\ 7x = 4y ... (a)
5. (a) Let the age of woman be 2x years and that of her
daughter be x years. x + 30 5
=
According to the question, y + 30 8
\ 8x – 5y = – 90 ...(b)
84 From eqn (b), 32x – 20y = – 360
2x + x = 2 × 42 or, 3x = 84 or, x = = 28
3 From eqn (a), 35x = 20y
\ Daughter's age = 28 years \ 32x – 35x = – 360
6. (e) Let Arun’s present age be x years. 360
Then, Deepak’s present age = (x + 14) years \ x= = 120
3
x-7 5 y = 210
Then, =
x + 14 - 7 7 330
= 165
\ Avg =
2
84
Þ 7x - 5x = 35 + 49 Þ x = = 42 1 2 3
2 16. (d) Given the ratio = : : = 6 : 8 : 9.
\ Deepak’s present age = 42 + 14 = 56 years 2 3 4
7. (d) Data is given in ratio. So age can’t be determined. æ 6ö
8. (a) Let number of students in Arts and Commerce were 4x \ 1st part = ` ç 782 ´ ÷ = ` 204.
è 23 ø
and 5x respectively. Then, 17. (b) Let actual distance be x km. Then,
4x 8 520 3
= Þ 44x – 40x = 520 Þ x = = 130 :1 :: 60 : x
5x + 65 11 4 4
\ Number of students in Arts = 4 × 130 = 520 3 240
9. (c) Let the original number of boys and girls be 6x and 5x Þ x = 60 Þ x = = 80 km
4 3
respectively. 18. (d) Let A = 2x, B = 3x and C = 4x. Then,
6x + 8 11 A 2x 2 B 3x 3 C 4x 2
Then, = = = , = = and = =
5x - 2 7 B 3x 3 C 4x 4 A 2x 1
78 A B C 2 3 2
Þ 55x - 42 x = 56 + 22 Þ x = =6 Þ : : = : : = 8 : 9 : 24.
13 B C A 3 4 1
\ Number of boys = 6 × 6 + 8 = 44 19. (c) Total age of 3 boys = (25 × 3) years = 75 years
10. (a) Present age of Meena Ratio of their ages = 3 : 5 : 7.
8 ´ 8 ´ (10 - 3) 8´ 8´ 7
Age of the youngest boy = æç 75 ´ ö÷ years = 15 years
= = = 32 years 3
24 - 10 14
è 15 ø
11. (b) Let the present age of father and son be 17x and 7x 20. (b) Let income of A = ` 3x, income of B = ` 2x
respectively. and expenditure of A = ` 5y,
17x – 6 3 expenditure of B = ` 3y
Then, =
7x – 6 1 Now, saving = income – expenditure
Þ 21x – 17x = 18 – 6 Þ x = 12 ÷ 4 =3 \ 3x – 5y = 2x – 3y = 200
Þ x = 2y and y = 200
\ Father’s present age = 17 × 3 = 51 years.
12. (c) Rita : Sita : Kavita \ x = 400
7 : 15 \ A’s income = ` 1200
7 : 16 21. (c) Since, A : B = 2 : 3 and B : C = 6 : 5
49 : 105 : 240 \A: B: C = 4: 6 : 5
The ratio of money with Rita, Sita and Kavita is
Then, A’s share = 4
49 : 105 : 240 ´ 750 =` 200
We see that 49 º ` 490 \ 240 º ` 2400 4+ 6+5
y
o
u
rs
m
a
h
44 101 SPEED TEST

b
o
o
22. (c) Let the numbers be 5x and 4x. 5

b
Now, difference of numbers = 10 30. (b) In the mixture, milk = ´ 28 = 20 litres, and

.w
i.e. 5x – 4x = 10 Þ x = 10 7

o
\ Larger number = 10 × 5 = 50

rd
2
Water = ´ 28 = 8 litres
23. (b) Give, Ratio of distribution = 3 : 1 : 5

p
7

re
Let the share of P, Q and R be 3x, x and 5x, respectively 2 ltrs of water is added, Hence the new ratio of

s
Since, Difference between Q’s and R’s share is Rs. 3600.

s
.c
\ 5x – x = 3600 Milk 20 20
= = = 2 :1

o
Þ x = 900 Water 8 + 2 10

m
\ P’s share = 3 × 900 = 2700
and Q’s share = 900 SPEED TEST 37
Hence, Total of P’s and Q’s share
= Rs 3600 1. (a) New % of sugar in (3 + 1) litre solution
24. (b) Mrs. X spends = Rs 535 0.04 ´ 3
\ Total cost = 43 shirt + 21 ties = 535 = = 0.03 = 3%
By hit and trial, S = 10, T = 5 (3 + 1)
Þ Total cost = 43 × 10 + 21 × 5 = 535 80 ´ 15 + 20 ´ 20
Hence, Ratio of shirts to ties = 10 : 5 = 2 : 1 2. (c) C.P. of mixture = = ` 16
80 + 20
a 2
25. (a) Given = (100 + 25)
b 5 \ S.P. = ´ 16 =` 20
100
2a 2 3. (b) By the rule of alligation, we have
+3 2´ + 3
2a + 3b 5 Cost of Alcohol Cost of Kerosene Oil
Consider = b = Rs 3.50 ` 2.50
7a + 5b 7a 2
+5 7´ + 5
b 5
a cost of
(By putting value of ) mixture
b ` 2.75
19
19
= 5 =
39 39
5 2.75 – 2.50 = 0.25 3.50 – 2.75 = 0.75
26. (c) Let the numbers be 6x and 13x. 0.25 1
LCM (6x, 13x) = 13 × 6 × x = 312 (given) \ Required ratio = = i.e. 1: 3
0.75 3
312 4. (b) In mixture,
Þ x= Þ x=4
13 ´ 6 Quantity of pure milk 3- 0 3 5
= = =
So, the numbers are 24 and 52. Quantity of water 3.6 - 3 0.6 1
\ Sum of the numbers = 24 + 52 = 76
Since in every 5 litres of milk, he adds 1 litre of water.
5a + 3b 23 \ In every 25 litres of milk, he adds 5 litres of water.
27. (d) We have, =
2a - 3b 5 5. (b) Let C.P. of milk be Re. 1 per litre.
Þ 25a + 15 b = 46a – 69b Then, S.P. of 1 litre of mixture = Re. 1.
Gain = 20%
a 84 4
Þ 21a = 84 b Þ = = æ 100 ö 5
b 21 1 \ C.P. of 1 litre of mixture = ` ç ´1 =` .
è 120 ÷ø 6
28. (b) Let the no. of one rupee, 50 paise and 25 paise coins be
2x, 3x and 4x respectively. By the rule of alligation, we have :
According to question, C.P. of 1 litre C.P. of 1 litre
of water of milk
æ 3x 4x ö
Rs. ç 2x + + ÷ = Rs. 216 0 Re. 1
è 2 4ø Mean
price
8x + 6x + 4x Re. 5
Þ = 216
4 6
\ x = 48
\ Number of 50 paise coins = 48 × 3 = 144 5 1 5 5
29. (c) Let the ages of two persons be X and Y respectively 1– = –0 =
6 6 6 6
X 5 X – 16 3
\ = and =
Y 7 Y – 16 5 1 5
\ Ratio of water and milk = : = 1: 5.
On solving we get X = 40, Y = 56. 6 6
y
o
u
rs
m
a
h
SOLUTIONS 45

b
o
6. (a) Out of 10 litre of solution, there is 1 litres of nitric acid

o
12. (d) By the rule of alligation,

b
and 9 litres of water. Alcohol concentration :

.w
Let x litres of water be added to the solution so that the

o
diluted solution is of 4% strength.

rd
Original mixture Pure alcohol
\ 4% of (10 + x) = 1 Þ x = 15.

p
15% 100%
7. (a) Let C.P. of 1 litre milk be Re. 1.

re
s
50

s
S.P. of 1 litre of mixture = Re. 1, Gain = %.

.c
3

o
m
6 25%
\ C.P. of 1 litre of mixture = Re. .
7
By the rule of alligation, we have :
C.P. of 1 litre C.P. of 1 litre
of water of milk
0 Mean Re. 1 75% 10%
price
\ Alcohol must be added in the ratio of 10 : 75 or 2 : 15
Re. 6
7 \ Quantity of alcohol to be added in 10 litres
6 2 4
1 = ´ 10 =
7 15 3
7
1 6 100 ´ 60
\ Ratio of water and milk = : = 1 : 6. 13. (b) C. P. of mixture = = ` 48
7 7 (100 + 25)
8. (b) The first alloy does not have tin. Therefore, quantity of
Let x kg be mixed. Then,
5
tin in 2 units of the resulting alloy = 40 ´ x + 55 ´ 16
13 48 =
Þ Quantity of tin in one unit of the resulting alloy 16 + x
5 Þ 8x = 16[55 - 48] Þ x = 14 kg
5 14. (c) The existing solution has 40% sugar. And sugar is to be mixed;
= 13 = so the other solution has 100% sugar. So, by alligation method:
2 26
9. (b) By the rule of alligation, 40% 100%
water concentration, 50%
Original solution Water
4 50% 10%
1
11 \ The two mixtures should be added in the ratio 5 : 1.
300
Therefore, required sugar = ´ 1 = 60 gm
5
6 (mixture) 15. (b) Gold Copper
13 19 9
15
7 14 6 4
13 143 \ Gold : Copper = 6 : 4 = 3 : 2
\ water must be added to the mixture in the ratio 16. (c) Apply the alligation on fracfion of milk in each mixture.
14 7 Mixture Water
: i.e. 2 : 11
143 13 2 0
3
2
Quantity of water to be added = ´ 55 = 10 litres 1
11 3
10. (c) Gold Copper 1 1
Type A 14 4 3 3
Type B 7 11 Ratio of mixture to water = 1 : 1
Type C 21 15 Therefore, if there is 60 liture of solution, 60 litres of water
\ The ratio of the Gold and Copper in the type C should be added.
alloy = 7 : 5. 17. (e) Initially water (weight) = 45 gm & milk 15 gm. After added
11. (c) C. P. of mixture of 18 kg = 10 × 45 + 8 × 50 = ` 850 15 gm
\ S. P. = C. P. + Profit = 850 + 32 = ` 882 water the percentage of water
\ S. P. = ` 882 for 18 kg
weight of water 60
882 = = ´ 100 = 80%
\ S. P. for 1 kg. = Rs =` 49 total weight of mixture 75
18
y
o
u
rs
m
a
h
46 101 SPEED TEST

b
o
o
18. (a) By the rule of alligation: 23. (c) Let the quantity of pure milk be x litres.

b
Cost of 1 kg tea of 1st kind If 5 litres of water is added to it, then the cost of (5 + x)

.w
litres = Rs. 3x

o
rd
S.P. of ( 5 + x) litres = Rs. (3x + 15)

p
\ Profit = Rs. 15

re
Given 20% of 3x = 15

s
s
.c
3x
Þ = 15 Þ x = 25

o
5

m
\ The amount of pure milk in the mixture is 25 litre
\ Required ratio = 750 : 250 = 3 : 1. 24. (c) Let the weight of tea worth Rs 25 per kg = x kg.
19. (d) Let the C.P. of spirit be Re. 1 per litre. According to question
5 5
Spirit in 1 litre mix. of A = litre; C.p. of a litre mix. in A = Re. . é x ´ 25 + 30 ´ 30 ù
7 7 \ 110% of ê x + 30 ú = 30
7 ë û
Spirit in 1 litre mix. of B = litre;
13 110 é 25x + 900 ù
5 Þ = 30
C.P. of 1 litre mix. in B = Re. . 100 êë x + 30 úû
13
8 8 Þ 11 (25x + 900) = 300 (x+30)
Spirit in 1 litre mix. of C= litre; Mean price = Re. .
13 13 Þ 275x + 9900 = 300 x + 9000 or 25x = 900
By the rule of alligation, we have : \ x = 36 kg.
C.P. of 1 litre mixture in A C.P. of 1 litre mixture in B 25. (b) Let, weight of sugar costing Rs 5.75 per kg = x kg
æ5ö æ7ö x ´ 5.75 + 75 ´ 4.50 = 5.50 ´ (x + 75)
ç ÷ Mean price ç ÷ Þ 5.75x + 337.50 = 5.50x + 412.50
è7ø è 13 ø
æ8ö Þ 0.25x = 75
ç ÷
æ1ö
è 13 ø \ x = 300 kg
9
ç ÷
è 13 ø 91 SPEED TEST 38
1 9
\ Required ratio = : = 7 : 9. 1. (b) Let S.P. = ` 100. Then, C.P. = ` 96; Profit = ` 4.
13 13
20. (c) Let the quantity of milk and water be 40 litres and 60 æ 4 ö 25
litres, respectively. \ Profit % = ç ´100 ÷ % = % = 4.17%. » 4.2%
è 96 ø 6
(Q Ratio of milk to water = 2 : 3)
After removing 50% of solution 2. (a) Let the cost of production of the table be ` x.
Quantity of milk = 20 litres and Then, 125% of 115% of 110% of x = 1265
Quantity of water = 30 litres 125 115 110
Þ ´ ´ ´ x = 1265
Therefore, the concentration of the solution is reduced 100 100 100
from 40 to 20 i.e. a reduction of 50%.
253 æ 1265 ´ 160 ö
3 Þ x = 1265 Þ x = ç ÷ = ` 800
21. (b) Quantity of milk = ´ 45 = 27 litres 160 è 253 ø
5
3. (b) Let C. P. = ` x then profit = S.P. – C. P.
2
Quantity of water = ´ 45 = 18 litres 1 11x
5 Þ ´ x = 891 – x Þ = 891
10 10
Let x litres of water be added to make the ratio 9 : 11.
891´10
18 + x 11 Þ Þx= =` 810
\ = 18 + x = 33 Þ x =15l 11
27 9
4. (d) C. P. for 50 pencils = ` 100
22. (c) Ratio of milk in the containers are,
100
1 3 5 5 3 25 \ C. P. for 45 pencils = ´ 45 = ` 90
5´ : 4´ : 5´ = : : 50
6 8 12 6 2 12 = S.P. of 45 pencils
and the ratio of water in the containers are, \ No gain , no loss
5 5 7 25 5 35 5. (b) (100 – loss) : S1 : : (100 + gain) : S2
5´ : 4´ :5´ = : : \ (100 – 12.5) : 420 : : (100 + 12.5) : S2
6 8 12 6 2 12
Ratio of mixture of milk and water in the containers 87.5 : 420 : : 112.5 : S2
Þ 87.5 × S2 = 420 × 112.5
æ1 3 5 ö æ5 5 7 ö
= ç ´5+ ´4 + ´ 5 ÷: ç ´ 5 + ´ 4 + ´ 5÷ 420 ´1125
è6 8 12 ø è 6 8 12 ø Þ S2 = = 540
= 106 : 230 = 53 : 115 875
y
o
u
rs
m
a
h
SOLUTIONS 47

b
o
o
5 1 14. (d) Let C.P. = ` x. Then,

b
6. (b) S.P. for 1 egg = ` = Rs

.w
10 2 (100 - 7) 93
S.P. = ´x = x

o
100 1 5 100 100

rd
\ C. P. for 1 egg = ´ =`
(100 + 20) 2 æ 93 ö 100

p
12
Also, ç 100 x + 48 ÷ (100 5) = x

re
Þ He bought 12 eggs for 5 rupees. è ø +

s
s
150 3 Þ 93x + 4800 = 105x

.c
7. (b) C.P. for one coconut = Rs = Rs Þ 12x = 4800 Þ x = ` 400

o
100 2

m
S.P. for one coconut = ` 2 15. (e) CP of DVD player = ` x
According to the question,
3 1
Profit on one coconut = 2 - = ` æ 22 ö
2 2 Q x ç1 + ÷ = ` 10980
1 è 100 ø
\ Profit on 2000 coconut = ´ 2000 = ` 1000
2 50
8. (a) Let C.P. = Rs 100, then M. P. = ` 150 \ x = 10980 × = ` 9000
61
S.P. = 70% of 150 = ` 105 16. (c) 24 bats + 32 sticks = ` 5600
105 - 100 \ 8(3 bats + 4 sticks) = ` 5600
\ % profit = ´ 100 = 5%
100 5600
9. (b) C.P. of one litre = ` 6 Þ 3 bats + 4 sticks = = ` 700
8
After adding water to it
1754 + 1492
2 17. (a) Cost price = = ` 1623
One has to pay Rs 7.20 for litre of milk. 2
3
10500 ´ 100 ´ 100
2 18. (c) Required amount = = ` 6250
So S.P. of litre of milk = Rs 7.20 120 ´ 140
3 19. (e) 15 pendants + 24 chains
7.20 ´ 3 = 3 (5 pendants + 8 chains)
Þ S.P. of 1 litre of milk = Rs =` 10.80
= 3 × 145785 = ` 437355
2
Q S.P. > C.P. 20. (c) CP = Rs 153, desired gain 20%
Þ SP = 153 × 1.2 = Rs 183.60
10.80 - 6 4.80 Let the marked price be Rs x
Hence gain = ´ 100 = ´ 100
6 6 Then, x ´ 0.85 = 183.60
= 0.80 × 100 = 80%
10. (a) He gives 800 grams but charges the price of 1000 grams 183.60
Þ x= = Rs. 216
(1 kg) 0.85
Þ on every 800 grams, he gains (1000 – 800) grams i.e. 21. (d) Applying successive discounts of 10%, 12% and 15%
200 grams. on 100, we get 100 ´ 0.9 ´ 0.88 ´ 0.85 = 67.32
200 Þ Single discount = 100 – 67.32=32.68
\ His gain % = ´ 100% = 25%
800 Hence, none of the given options is correct.
error 22. (b) Let the cost price of the machine be Rs x.
Short cut : Gain % = 11x
true weight–error Then, selling price at a profit of 10% = Rs
10
200
= ´ 100 = 25%. 9x
1000 - 200 And the selling price at a loss of 10% = Rs
11. (d) Here, SP1 = SP2 10
CP1 6 3 Consequently, we find that
Þ 140 CP1 = 60CP2 Þ = =
CP2 14 7 æ 11x 9x ö
çè - ÷=
3 10 10 ø
\ CP1 = ´ 8000 = ` 2400
(3 + 7) x
and CP2 = 8000 – 2400 = ` 5600 Þ = 80 Þ x = Rs 400
5
12. (b) Let the C.P. of the goods be ` 100 23. (b) Let CP be Rs x and SP be Rs y.
Þ Marked price of the goods = ` 120 Then, profit = Rs (y – x)
Discount = 10% Þ S.P. is 90% of Rs 120 = ` 108 If SP = 2y, then profit = 3(y – x)
\ Gain% = (108 – 100) = 8%. Now, 2 y - x = 3( y - x )
13. (d) Let his loss = ` x. Then,
C.P. = 5000 + x = 5600 – 2x Þ y = 2x
Þ 3x = 600 Þ x = 200 y-x 2x - x
\ C.P. =5000 + 200 = Rs 5200 Now, profit = ´ 100 = ´ 100 = 100%
x x
y
o
u
rs
m
a
h
48 101 SPEED TEST

b
o
As, we know, profit = selling price – cost price

o
24. (c) Let CP of 12 pencils = SP of 10 pencils = Re 1

b
\ Cost price of 12 note book = 12x – 2x = 10 x

.w
1
Therefore, CP of 1 pencil = Rs and 2x

o
12 \ Profit ( percentage) = ´ 100 = 20%

rd
10x
1

p
SP of 1 pencil = Rs

re
10 SPEED TEST 39

s
s
.c
1 1 2 1
Profit on one pencil = - = = Rs P× R × T

o
10 12 120 60

m
1. (b) Simple Interest =
100
% profit = 1/ 60 ´ 100 = 20% 31400 ´ 8 ´12
1/12 =` 30144
25. (d) If any two transactions of SP is the same and also gain 100
% and loss % are same then there is always a loss \ Required amount = ` (31400 + 30144)
2 2
= ` 61544
æ Common gain or loss% ö æ 10 ö (26350 - 21250) ´ 100
\ loss % = ç ÷ = ç ÷ = 1% 2. (e) Rate =
è 10 ø è 10 ø 21250 ´ 6
26. (a) Let the cost price of geyser be Rs x. Then
x × 1.1 × 1.15 × 1.25 = Rs 1265 510000
= = 4%
127500
1265
Þ x= = Rs.800 3. (c) Let the required time = t years
1.58125
Simple interest = (11442 – 9535) = `1907
27. (a) Money paid to buy watch = Rs. 1950.
Money paid as interest of 10% on Rs. 1950 = Rs. 195 P×T×R
Simple =
\ Total money paid = Rs. 2145 100
Since S.P. = Rs. 2200, therefore the man gained Rs. 55. 9535 ´ 4 ´ t
28. (c) Let the cost price be Rs 100. 1907 =
\ Marked price is Rs 135. 100
At 10% discount, the customer has to pay 1907 ´ 100
= Marked price – discount = 135 – 13.5 = 121.5. \t= = 5 years
9535 ´ 4
1 4. (b) Let the principal be = `100
\ % profit = 21.5% = 21 %. \ Simple interest
2
29. (a) S.P. of the 1st chair = Rs. 500 100 ´ 8 ´ 6
Gain = 20% = = `48
100
500 ´ 100 500 ´ 100 \ Amount (100 + 48) = `148
\ C.P. of the 1st chair = = \ When the amount is = `148, the principal = `100
100 + 20 120
\ When amount = `28046, the principal
1250
= 100
3 = × 28046 = `18950
48
S.P. of the 2nd chair = Rs. 500
\ Simple interest = (`28046 – 18950) = `9096
Loss = 12%
P×R×T
500 ´ 100 500 ´ 100 5. (c) Simple interest =
\ C.P. of the 2nd chair = = 100
100 - 12 88
56500 × 3 × 12
6250 = ` = 20340
= 100
11
\ Required amount = ` (56500 + 20340) = `76840
Now S.P. of both the chairs = Rs. 1000
C.P. of both the chairs 8376 ´ 100
6. (d) Amount invested =
1250 6250 13750 + 18750 32500 8´ 6
= + = = = ` 17450
3 11 33 33 P´ 4´ 5
7. (d) 2000 =
\ Net gain = 1000 – 32500 = 500 100
\ P = 10000
33 33
éæ 4 ö
2 ù
500 33 500 Now, CI = 10000 êç1 + ÷ - 1ú = 10000 × 0.0816 = 816
Þ Gain % = ´ 100 = ´ 100
ëêè
100 ø
32500 33 32500 ûú
1120 ´ 8.5 ´ 3
100 20 8. (a) Amount = 11200 + = 11200 + 2856 = 14056
= = = 1.5% (To one place of decimal) 100
65 13 9. (b) Ratio of two parts = r2 t2 : r1 t1 = 54 : 50 = 27 : 25
30. (b) Let x be the selling price of 1 notebook. 2600
\ Selling price of 2 note book = 2x = profit \ Sum lent out at 10% = ´ 27 = ` 1350
52
y
o
u
rs
m
a
h
SOLUTIONS 49

b
o
o
10. (d) It dobles in 10 yrs.
(c) Difference in S.I. = P ´ T (R1 - R 2 )

b
Then trebles in 20 yrs. 17.

.w
11. (b) Suppose the rate of interest = r% and the sum = ` A 100

o
A+ r´4 P ´ 4´ 2

rd
Now, A + = 600 ; Þ 56 = (Q R1 – R2 = 2)

p
100 100

re
Ar 56 ´ 100

s
or, A + = 600 Þ P= = ` 700

s
.c
25 4´ 2

o
é r ù 18. (a) Q Rate = 5 paise per rupee = 5%

m
or, A ê1 + ú = 600 ...(1)
ë 25 û 200 ´ 5 ´ 7
\ S.I. = = ` 70
A´ r ´ 6 100
And, A + = 650 ;
100 19. (c) Let one gets = ` x
é 3r ù
then, second gets = ` (68,000 – x)
or, A ê1 + ú = 650 ...(2) Given : A1 = A2
ë 50 û
Dividing (1) by (2), we have x ´10 ´ 8 (68000 - x) ´10 ´ 6
x+ = (68, 000 - x) +
r 100 100
1+ (25 + r ) ´ 2 12
25 = 600 ; or, = Þ x[100 + 80] = (68, 000 - x)[100 + 60]
3r 650 50 + 3r 13
1+ 180x
50 Þ = 68, 000 – x
160
(25 + r ) ´ 2 12
or, = Þ 34x = 68000 ´16 Þ x = Rs32,000
50 ´ 3r 13
or, (50 + 2r) × 13 = (50 + 3r) × 12 \ second gets = ` 36,000
or, 650 + 26r = 600 + 36r; or, 10r = 50 3 51
\ r = 5% 20. (b) Interest for one year = ` 212.50 ´ ´1 = `
12. (a) Let the sum be ` x. 100 8
x ´ 8 ´ 4 32 x
Thus in 8 years, the interest is ` 51.
\ Interest = = 21. (c) Shortcut method :
100 100
If borrowed amount be ` M and it is to be paid in equal
32 x 68 x instalments, then
x- =
100 100 ra n(n - 1)
M = na + ´
When interest is
68 x
less, the sum is ` x. 100 ´ Y 2
100 where Y = no. of instalments per annum
x a = annual instalment
\ when interest is 340 less, the sum is ´100 ´ 340 = ` 500 Here, M = 4200, y = 1, r = 10, n = 5, a = ?
68x
13. (b) S. I. = 81 – 72 = ` 9 10a 5(5 - 1)
4200 = 5a + ´
9 ´100 ´ 4 100 2
\T= = 2 years
72 ´ 25 Þ 4200 = a [5 + 1] Þ 6a = 4200
14. (a) S.I. for 1½ years = ` (1164 – 1008) = ` 156 Þ a = ` 700
æ 156 ´ 2 ´ 2 ö 22. (d) We need to know the S.I., principal and time to find the
S. I. for 2 years = ` ç
3 ÷ = Rs 208 rate. Since the principal is not given, so data is
è ø inadequate.
\ Principal = ` (1008 – 208) = ` 800 23. (c) Let the principal be P and rate of interest be R%.
Now , P = 800, T= 2, S.I. = 208
éæ P´ R ´ 6 öù
æ 100 ´ 208 ö ê ç 100 ÷ ú 6PR 6
\ Rate = ç ÷ % = 13% è øú =
è 800 ´ 2 ø \ Required ratio = ê = = 2 : 3.
ê æ P ´ R ´ 9 ö ú 9PR 9
15. (b) S.I. for 5 years = ` (1020 –720) = ` 300 ê çè 100 ÷ø ú
ë û
300
SI. for 2 years = Rs ´ 2 = Rs 120 24. (a) Let the sum be ` x. Then,
5
\ Principal = ` (720 – 120) = ` 600 æ x ´ 6 ´ 3 ö æ x ´ 9 ´ 5 ö æ x ´ 13 ´ 3 ö
ç 100 ÷ + ç 100 ÷ + ç 100 ÷ = 8160
Now, P = 600, T = 2, S.I. = 120 è ø è ø è ø
120 ´100 Þ 18 x + 45x + 39x = (8160 × 100) Þ102x = 816000
\R= = 10% Þx = 8000.
600 ´ 2 25. (b) Let the sum be ` 100. Then,
5
1.53 ´10 ´ 20 æ 100 ´ 10 ´ 1ö
16. (b) Let S.I. = ` x = = `2500. S.I. for first 6 months = ` ç = Rs. `
100 è 100 ´ 2 ÷ø
y
o
u
rs
m
a
h
50 101 SPEED TEST

b
o
o
æ 105 ´ 10 ´ 1ö 4000 ´ 5 ´ 2

b
S.I. for last 6 months = ` ç =` 5.25. 5. (a) Simple interest = = ` 400
è 100 ´ 2 ÷ø

.w
100

o
So, amount at the end of 1 year = ` (100 + 5 + 5.25) Compound interest

rd
= ` 110.25. 5 ö
2
= 4000 æç 1 +

p
\ Effective rate = ` (110.25 – 100) = 10.25%. - 4000

re
è ÷
26. (d) Let the rate be R% p.a. Then, 100 ø

s
s
4000 ´ 105 ´ 105

.c
æ 5000 ´ R ´ 2 ö æ 3000 ´ R ´ 4 ö = - 4000 = 4410 – 4000 = ` 410
÷+ç ÷ = 2200

o
ç 100 ´ 100

m
è 100 ø è 100 ø
\ Difference = 410 – 400 = ` 10
æ 2200 ö 2
Þ 100R + 120R = 2200 Þ R = ç ÷ = 10. 5 ö
è 220 ø 6. (a) Compound interest = 35000 æç 1 + ÷ – 35000
è 100 ø
27. (b) Let the savings be X and Y and the rates of simple
= 38587.50 – 35000 = ` 3587.50
interest be 5x and 4x respectively.
8730 ´ 100
1 1 1 1 X 4 7. (d) Principle = = 48500
Then, X ´ 5x ´ ´ = Y ´ 4x ´ ´ or = , 6´ 3
2 100 2 100 Y 5
i.e., X : Y = 4 : 5. éæ 6 ö
2 ù
Compound Interest = 48500 êç1 + ÷ - 1ú
28. (a) Let the principal be P, then amount after 12 years = 2P
ëêè
100 ø
ûú
Þ SI = (2 P - P ) = P = 48500 × 0.1236 = ` 5994.60
P´r ´t P ´ r ´ 12 éæ r ö
2 ù 594.5 æ r ö
2
Now, I = ÞP= 8. (a) 594.5 = 5800 êç1 + ÷ - 1ú = = ç1 + ÷ -1
100 100 êëè 100 ø úû 5800 è 100 ø
100 25 1 2
or r = = =8 % æ r ö
12 3 3 0.1025 + 1 = ç1 + ÷
29. (b) Let the sum be Rs x è 100 ø

x ´ 8 ´ (3 - 2) (100 + r )2
Now, 56 = Þ x = Rs 700 1.1025 =
100 10000
1.1025 × 10000 = (100 + r)2.
30. (a) SI = 10, 000 ´ 6 ´ 9 = Rs 450 11025 = (100 + r)2.
12 ´ 100 105 = 100 + r
r = 5%
SPEED TEST 40 éæ 27 ö
2 ù
9. (d) Compound interest = 7400 êç 1 + ÷ - 1ú
êëè 200 ø úû
2
æ 5 ö
1. (a) Required Amount = 15000 ç 1 + ÷ = ` 16537.50 éæ 227 ö 2 ù é 227 ´ 227 - 200 ´ 200 ù
è 100 ø = 7400 êç ÷ - 1ú = 7400 ê ú
Time êëè 200 ø úû ë 200 ´ 200 û
æ Rate ö
2. (b) Amount = Principal ç1 + ÷ é 51529 - 40000 ù 11529
è 100 ø = 7400 ê
40000 ú = 7400 ´ 40000
ë û
éæ 8 ö
2
ù = 2132.865 = ` 2132.87
=1250 êç1 + ÷ - 1ú éæ T ù
ëêè 100 ø ûú 10. (c) C. I. êç 1 +
r ö
÷ - 1ú
æ 108 ´108 -100 ´100 ö 1250 ´1664 êëè 100 ø úû
=1250 ´ ç ÷ = = ` 208
è 100 ´100 ø 10000 éæ r ö
2 ù
4676.25 = 14500 ê ç 1 + ÷ - 1ú
Time êëè 100 ø úû
æ Rate ö
3. (e) Amount = Principal = çè1 + ÷
100 ø 4676.25 æ r ö
2
4676.25 æ r ö
2
Þ = ç1 + ÷ -1 Þ + 1 = ç1 + ÷
3 14500 è 100 ø 14500 è 100 ø
æ 5 ö 21 21 21
= 5690 ç 1 + ÷ = 5690 × 20 ´ 20 ´ 20 = `6586.90 2
è 100 ø 4676.25 ´ 14500 æ r ö
Þ = ç1 + ÷
\ Compound interest = `(6586.9 – 5690) = `896.9 » 897 14500 è 100 ø
30240 ´ 100 19176.25 r r
4. (e) Rate = = 12% Þ = 1+ Þ 1.3225 = 1 +
84000 ´ 3 14500 100 100
Compound interest
13225 r 115 r r 115
3 Þ = 1+ Þ = 1+ Þ = -1
æ 12 ö 10000 100 100 100 100 100
= 84000 ç 1 + ÷ – 84000
è 100 ø r
=
115 - 100 r
=
115
Þ Þ Þ r = 15%
= 118013.95 – 84000 = ` 34013.95 100 100 100 100
y
o
u
rs
m
a
h
SOLUTIONS 51

b
o
o
11. (c) Compound interest accrued half-yearly. 16. (b) Let the sum of money be Rs x.

b
R = 20% yearly = 10% half-yearly

.w
3
n = 2 years = 4 half-yearly æ r ö
Now, 8x = x ç1 +

o
è 100 ÷ø

rd
éæ r ö
n ù
CI = P êç 1 + ÷ - 1ú

p
êëè 100 ø úû

re
3
r
or, æç1 + r ö÷ = (2)3

s
or 1 + =2

s
éæ ù 4 éæ 11 ö4 ù è 100 ø 100

.c
10 ö
= 10000 êç1 + ÷ - 1ú = 10000 êç ÷ - 1ú

o
êëè 100 ø úû êëè 10 ø úû Again, let the sum becomes 16 times in n years.

m
n
é11´ 11 ´ 11 ´ 11 - 10 ´ 10 ´10 ´10 ù æ r ö
= 10000 ê Then, 16 x = x ç1 +
è 100 ÷ø
10 ´ 10 ´ 10 ´ 10 ú
ë û
é14641 - 1000 ù é 4641 ù
= 10000 ê ú = 10000 ê10000 ú = ` 4641 Þ 16 = 2 n or 2 4 = 2 n or n = 4
ë 10000 û ë û 17. (d) Required difference
2 2
æ R ö æ 14 ö é 2 ù 5000 ´ 12 ´ 2
12. (e) Required difference = P ç ÷ = 985 çè 100 ÷ø æ 12 ö
è 100 ø = ê5000ç1 + ÷ - 5000ú -
êë è 100 ø úû 100
196
= 985 ´ = 19.306
10000 æ 28 28 ö
= 5000ç ´ - 1÷ - 1200
13. (d) Difference in amounts = 2977.54 – 2809 = ` 168.54
è 25 25 ø
Now, we see that ` 168.54 is the interest on ` 2809 in one
year (it is either simple or compound interest because both 784 - 625 ö
are the same for a year). = 5000 æç ÷ - 1200 = Rs.72
è 625 ø
168.54 ´ 100 18. (d) Let the money borrowed be Rs x and the rate of interest
Hence, rate of interest = = 6%
2809 charged = r%
Now, for the original sum, Time = 2 years
2 2
æ 6 ö æ 53 ö x´r´2
2809 = x ç 1 + ÷ or, 2809 = x ç ÷ Now, 4000 =
è 100 ø è 50 ø 100
2809 ´ 50 ´ 50 Þ r x = 200000 ............. (i)
\ x= = ` 2500
53 ´ 53 2
æ r ö
t Again, x ç1 + ÷ = x + 4200
æ 2 ö è 100 ø
14. (b) 6632.55 = 6250 ç1 + ÷
è 100 ø
Þ xr 2 2 xr
6632.55 æ 51 ö
t
663255 æ 51 ö
t + = 4200
or, =ç ÷ or, =ç ÷ 10000 100
625000 è 50 ø 625000 è 50 ø
or 20r + 4000 = 4200 [from (i)]
t
132651 æ 51 ö æ 51 ö
t or r = 10%
or, =ç ÷ =ç ÷ \ t=3
125000 è 50 ø è 50 ø n
æ r ö
19. (c) A = Pç1 + ÷ ; A = Amount
t 3 è 100 ø
Hence, the time is =
2 2 P = Principal ; r = rate of interest ; n = time
15. (a) Whenever the relationship between CI and SI is asked for 3
years of time, we use the formula: 3
æ 105 ö
Required amount = 1000ç ÷
rt
´ CI è 100 ø
SI =
éæ r ö
r ù
100 êç 1 + ÷ - 1ú 21 21 21
ëè 100 ø û = 1000 ´ ´ ´
20 20 20
5´ 3 = Rs 1157.625
150 = ´ CI 20. (b) Let the rate of interest be r %.
éæ r ö
3 ù
100 êç1 + ÷ - 1ú
ë è 100 ø û 2520 (1 + r /100) 4
Therefore, =
2400 3
é 9261 - 8000 ù æ r ö
150 ´100 ê çè1 + ÷ø
CI = ë 8000 úû 100
5´ 3
r 21
150 ´ 100 ´ 1261 1261 Þ 1+ = or r = 5%
= = = ` 157.62 100 20
5 ´ 3 ´ 8000 8
y
o
u
rs
m
a
h
52 101 SPEED TEST

b
o
\ let the time in which the principal amount becomes

o
21. (d) Cash Price (C.P.)= Rs. 39,000

b
Cash Down Payment (D.P.) = Rs. 17000 16 times be n

.w
Balance Due (B.D) = Rs. 22000 n

o
P = Value of Instalment = Rs. 4800 æ 100 ö

rd
Then 16P = P ç1 + ÷
n = No. of Instalments = 5 è 100 ø

p
re
R = Rate of interest
Þ 16 = 2n Þ n = 4yrs.

s
s
æ nR ö ì (n - 1)R ü

.c
Q ç1 + ÷ BD = í1 + ý nP 24. (a) Amount = Rs. 176040, Principal = Rs. 16000

o
è 1200 ø î 2400 þ Time = 2 yrs, Rate = R

m
æ 5R ö ì 4R ü 2
æ R ö
\ çè 1 + ÷ø 22000 = í1 + 2400 ý5 ´ 4800 17640 = 16000 ç 1 + ÷
1200 î þ è 100 ø
æ 5R ö ì 4R ü 17640 æ R ö
2
æ R ö
2
Þ ç1 + ÷11 = í1 + 2400 ý12 Þ = ç1 +
è 1200 ø î þ ÷ Þ 1.1025 = ç1 + ÷
16000 è 100 ø è 100 ø
55R 24R
Þ 11 + = 12 + R R
1200 1200 Þ 1+ = 1.05 Þ = 1.05 – 1 = 0.05 Þ R = 5%
100 100
55R 24R 31R 25. (a) Let A lent Rs. x and B lent Rs. y
Þ - = 1Þ =1
1200 1200 1200 Since, A and B together lent out Rs. 81600
1200 \ x + y = 81,600
Þ R= = 38.71% Now, given (r) Rate = 4%
31
22. (c) Let the amount of each instalment be Rs. x. 4 26
\ 1+ r = 1 + =
3 100 25
Amount of Rs. 100 for 3 years = 100 æç1 + 5 ö÷ According to the question, we have
è 100 ø 3–2
x æ 26 ö 26
21 21 21 9261 =ç ÷ =
= 100 ´ ´ ´ = Rs. y è 25 ø 25
20 20 20 80
25
9261 \ Investment made by B = 81600 ´ = 40, 000
Present value of Rs. due after 3 years = Rs. 100 51
80
Present value of Rs. x due after 3 years
SPEED TEST 41
100 ´ 80 8000x
= ´ x = Rs.
9261 9261 5 10
1. (b) Relative speed = 42 – 30 = 12 km/hr = 12 ´ = m/s
2 18 3
æ 5 ö
Amount of Rs. 100 for 2 years = 100 ç1 +
è 100 ÷ø Time =
Total length of both the trains 84 + 60
=
Relative speed 10
21 21 441 3
= 100 ´ ´ =
20 20 4 144 ´ 3
= = 43.2 seconds
400 10
Present value of Rs. x due after 2 years = Rs. x
441 120
Similarly, present value of Rs. x due after 1 year 2. (a) Speed of the train = = 24 m/s
5
21 \ time taken by the train to pass the platform
= Rs. x
20 120 + 180
= = 12.5 seconds
8000 400 20 24
\ x+ x+ x = 126100 3. (c) Length of the train = Relative speed × time
9261 441 21
Þ 8000x + 8400x + 8820x = 126100 × 9261 æ5ö 15 ´ 5 ´ 48
= (40 - 25) ç ÷ ´ 48 = = 200 m
Þ 25220x = 126100 × 9261 è ø
18 18
4. (c) Let the length of the train = x m
126100 ´ 9261
Þ x= = Rs 46305 x x + 100
25220 Then, speed of the train ==
15 25
3 or, 25x = 15x + 1500 or, 10x = 1500
æ r ö
23. (b) Given 8P = P ç 1 + ÷ \ x = 150 m
è 100 ø
Where P = Principal amount, Distance covered 2924
5. (c) Speed of bus = = = 68 kmph.
r = Compound interest rate Time taken 43
Þ r = 100% 6. (a) Distance covered = Speed × Time = 49 × 7 = 343km
y
o
u
rs
m
a
h
SOLUTIONS 53

b
o
o
7. (e) Let the distance between the village and the school be x x 20

b
xkm. and = + ...(ii)

.w
According to the question, y - 2 y 60

o
On solving (i) and (ii), we get x = 3y

rd
x x x + 2x Put x = 3y in (i) we get
+ = 6 or,, = 6 or,, 3x=6×4

p
re
4 2 4
3y 1

s
6´4 = 3- Þ y = 20

s
\x= = 8 km y+4 2

.c
3

o
Hence, distance = 20 × 3 = 60 km.

m
8. (a) Speed of train = (200 + 400) ´ 18 = 60 km/hr.. æ 16 ö
17. (b) Rate downstream = ç ÷ kmph = 8 kmph;
36 5 è2ø
9. (c) Distance covered in 18 seconds
æ 16 ö
5 Rate upstream = ç ÷ kmph = 4 kmph.
= 90 ´ ´ 18 = 450 m è 4 ø
18 1
\ length of platform = 450 – 160 = 290 m \ Speed in still water = (8 + 4) = 6 km/h.
2
(64 - 48) ´ 60 18. (c) After 5 minutes (before meeting), the top runner covers
10. (b) Stoppage minutes per hour = 2 rounds i.e., 400 m and the last runner covers 1 round
64
= 15 minutes. i.e., 200 m.
\ Top runner covers 800 m race in 10 minutes.
480
11. (c) Speed of bus = = 40 km/hr Total distance
12 19. (a) Average speed =
Total time
Speed of train = 40 ´ 9 = 72 km/hr 400 ´ 4 ´ 9 400 ´ 4 ´ 9
5 = =
88 + 96 + 89 + 87 360
72 = 40 metres /minutes
Speed of car = ´ 13 = 52 km/hr
18 æ 750 ö
Distance covered by car = 52 × 5 = 260 km 20. (a) Speed = ç ÷ m / sec = 5 m / sec
è 150 ø
5
12. (a) Length of platform = 126 ´ × 24 – 300 = 540 meter æ 18 ö
18 = ç 5 ´ ÷ km / hr = 18 km / hr.
è 5ø
540 21. (a) Let each side of the square be x km and let the average
\ Speed of man = = 1.8 meter/second
5 ´ 60 speed of the plane around the field be y km/h. Then,
13. (b) Distance = 64 × 8 = 512 km x x x x 4x
512 + + + =
\ Speed = = 85 km/hr (approx.) 200 400 600 800 y
6
14. (d) Net distance gained by car over the bus 25x 4x æ 2400 ´ 4 ö
Þ = Þ y=ç ÷ = 384.
= 40 + 60 = 100m, in 20 sec. 2400 y è 25 ø
\ Average speed = 384 km/h.
Distance 100
Time = Þ 20 = 22. (d) Let after t hours they meet then,
Relative speed æ 5ö 3t + 4t = 17.5 Þ t = 2.5
ç 36 ´ ÷ - S2
è 18 ø \ Time = 10 am + 2.5 h = 12 : 30pm
Þ S2 = 5 m/s = 18 kmph. 23. (a) Let original speed = S km/h
15. (b) Relative speed of the trains Here, distance to be covered is constant
= (72 – 54) km/h = 18 km/h æ 20 ö
\ S ´ 8 = (S + 5) ç ÷
æ 5ö è 3 ø
= ç 18 ´ ÷ m/sec = 5 m/sec.
è 18 ø 20 100 100
Þ 8S - S= ÞS= = 25 km / h
Time taken by the trains to cross each other 3 3 4
= Time taken to cover (100 + 120) m at 5 m/sec 24. (c) Let the speed of the bus be x km / h.
æ 220 ö then speed of the car = (x + 25) km / h
=ç ÷ sec = 44 sec. 500 500
è 5 ø \ = + 10
16. (a) Let the distance be x km. Let speed of train be y km/h. x x + 25
Then by question, we have Þ x2 + 25x – 1250 = 0 Þ x = 25
Thus speed of the bus = 25 km/h
x x 30 Speed of the car = 50 km/h
= - ...(i)
y + 4 y 60 Alternative:
Difference in speeds 25 km / hr is in only option (c).
y
o
u
rs
m
a
h
54 101 SPEED TEST

b
o
o
25. (a) Distance to be covered by the thief and by the owner is 1 1

b
same. 3. (a) A’s 1 day’s work = and B’s 1 day’s work = .

.w
Let after time 't', owner catches the thief. 18 9

o
æ 1 1ö 1

rd
æ 1ö \ (A + B)’s 1 day’s work = ç + ÷ = .
\ 40 ´ t = 50 ç t – ÷

p
è 2ø è 18 9 ø 6

re
s
s
5 1 4. (c) Let C completes the work in x days.

.c
Þ 10t = 25 Þ t = hr = 2 hr

o
2 2 1

m
26. (a) Let the duration of the flight be x hours. Then, Work done by (A + B) in 1 day =
10
600 600 600 1200 1
- = 200 Þ - = 200
x 1 x 2x + 1 Work done by (B +C) in 1 day =
x+ 18
2 A’s 5 days’ work + B’s 10 days’ work
Þ x (2x + 1) = 3 Þ 2x2 + x – 3 = 0 Þ (2x + 3) (x – 1) = 0 + C’s 15 days’ work = 1
Þ x = 1 hr. [neglecting the –ve value of x]. or (A + B)’s 5 days’ work + (B + C)’s 5 days’ work
180 180 + C’s 10 days’ work = 1
27. (d) Required difference = - = 15 km
3 4 5 5 10
or + + =1
28. (c) Let the husband and the wife meet after x minutes. 4500 10 18 x
metres are covered by Pradeep in 60 minutes. \ x = 45 days
4500 1 1 1
In x minutes, he will cover x metres. 5. (b) In one min, (A + B) fill the cistern = + = th
60 10 15 6
Similarily,
In 3 mins, (A + B) fill the cistern = 3 = 1 th
3750 6 2
In x minutes, his wife will cover x m.
60
1 1
4500 3750 Remaining part = 1 - =
Now, x+ x = 726 2 2
60 60 1
726 ´ 60
Q th part is filled by A in one min.
10
Þ x= = 5.28 min
8250 1 1
29. (b) Downstream speed = 15 + 5 = 20 km/h. \ nd part is filled by A in 10 ´ = 5 min .
2 2
\ Required distance = 20 ´ 24 = 8km. \ Total time = 3 + 5 = 8 min.
60 6. (b) Ratio of times taken by A and B = 100 : 130 = 10 : 13.
30. (c) Let the distance travelled during both upward and Suppose B takes x days to do the work.
downward journey be x km. Then, 10 : 13 : : 23 : x
Total distance covered æ 23 ´ 13 ö 299
Average speed = Þx =ç ÷ Þx= .
Total time taken è 10 ø 10
x+x 2 2 ´ 28 ´16 1 10
= = = = 20.36 km / h A’s 1 day’s work = ; B’s 1 day's work = .
x x 28 + 16 44 23 299
+
16 28 28 ´16
æ 1 10 ö 23 1
(A + B)’s 1 day’s work = ç + ÷ = = .
SPEED TEST 42 è 23 299 ø 299 13
\ A and B together can complete the job in 13 days.
7. (a) Sunil takes 5 days and Pradeep takes 15 days to do the
12 ´ 8 work.
1. (c) Number of days = = 24 days
12 - 8
1 1 4 th
2. (a) In an hour, George and Sonia together can copy In a day they would complete + i.e., work.
5 15 15
1 1 7 11
+ = of a 50-page manuscript. The remaining th work would be completed by
6 8 24 15
7 11
i.e. In an hour they together can copy of the Pradeep in ´ 15 i.e. 11 days.
48 15
100-page manuscript. 8. (a) Let 1 man’s 1 day’s work = x and
i.e. They together can copy a 100-page manuscript in 1 boy’s 1 day’s work = y.
1 1
48 6 Then, 6x + 8y = and 26x + 48y = .
hours, i.e. 6 hours. 10 2
7 7
y
o
u
rs
m
a
h
SOLUTIONS 55

b
o
Solving these two equations, we get :

o
Part filled by (A + B + C) in 1 min.

b
.w
1 1 æ 1 1 1 ö 1
x= and y = . =ç + - ÷= .

o
100 200 è 36 45 30 ø 60

rd
\ (15 men + 20 boys)’s 1 day’s work 1

p
13. (d) (Man + Son)’s one day’s work =

re
æ 15 20 ö 1 8
=ç + ÷= .

s
è 100 200 ø 4 1

s
.c
Man’s one day’s work =
\ 15 men and 20 boys can do the work in 4 days. 10

o
m
1 1 1 1
9. (a) 1 man’s 1 day’s work = . Þ Son’s one day’s work = - =
108 8 10 40
\ Son can do it in 40 days.
æ1 ö 2 3
12 men’s 6 day’s work = ç ´ 6 ÷ = . 14. (d) Q A can do of the work in 12 days
è9 ø 3 4
æ 2ö 1 1 4 1
Remaining work = ç 1 - ÷ = . \ A can do of the work in 12 ´ ´ days = 2 days
è 3ø 3 8 13 8
15. (b) (A + B)’s 5 days’ work
æ 1 ö 1
18 men’s 1 day’s work = ç ´18 ÷ = . æ 1 1 ö 45 9
è 108 ø 6 =5 ç + ÷ = =
è 25 20 ø 100 20
1
work is done by them in 1 day..
6 æ 9 ö 11
Remaining work = ç1 - ÷ =
1 è 20 ø 20
\ work is done by them in 6 ´ 1 = 2 days
3 3 11
of the work would be finished by B in
1 20
10. (c) A’s one day’s work = th work 11
16
20 = 11days.
1
B’s one day’s work = th work 1
12 20
Let the number of days B has worked alone = x days.
Then, 16. (a) 50 men complete 0.4 work in 25 days.
A’s amount of work + B’s amount of work = 1 Applying the work rule, m1 ´ d1 ´ w 2 = m 2 ´ d 2 ´ w1
æ1 ö æ 1ö we have,
Þ 4 ç ÷ + (x + 4) ç ÷ = 1 50 ´ 25 ´ 0.6 = m 2 ´ 25 ´ 0.4
16
è ø è 12 ø
1 x+4 3 50 ´ 25 ´ 0.6
Þ + = 1 Þ x = ´ 12 - 4 Þ x = 5 days or m2 = = 75 men
4 12 4 25 ´ 0.4
æ 1 1ö 9 3 Number of additional men required = (75 – 50) = 25
11. (c) (A + B)'s 1 hour's work = ç + ÷ = = 17. (c) Let C completes the work in x days.
è 12 15 ø 60 20
1
æ 1 1 ö 8 2 Work done by (A + B) in 1 day =
(A + C)'s 1 hour's work = ç + ÷ = = 10
è 12 20 ø 60 15
1
æ 3 2 ö 17 Work done by (B +C) in 1 day =
Part filled in 2 hrs = ç + ÷ = 18
è 20 15 ø 60 A’s 5 days’ work + B’s 10 days’ work + C’s 15 days’
æ 17 ö 17 work = 1
Part filled in 6 hrs = ç 3 ´ ÷ = or (A + B)’s 5 days’ work + (B + C)’s 5 days’ work
è 60 ø 20
+ C’s 10 days’ work = 1
æ 17 ö 3
Remaining part = ç 1 - ÷ = 5 5 10
è 20 ø 20 or + + = 1 or x = 45 days
3 10 18 x
Now, it is the turn of A and B and part is filled by 1
20 18. (d) In 1 day, work done by 12 men =
A and B in 1 hour. 18
\ Total time taken to fill the tank = (6 + 1) hrs = 7 hrs. 6 1
In 6 days, work done by 12 men = =
æ 1 1 ö 7 18 3
12. (a) Part filled in 7 min. = 7 × ç + ÷ = 2
è 36 45 ø 20 Remaining work =
3
æ 7 ö 13
Remaining part = ç1 - ÷ = Now, m1 ´ d1 ´ w 2 = m 2 ´ d 2 ´ w1
è 20 ø 20
y
o
u
rs
m
a
h
56 101 SPEED TEST

b
o
24. (a) Let 1 man’s 1 days’ work= x & 1 boy’s 1 day’s work = y

o
2

b
or 12 ´ 18 ´ = 16 ´ d 2 ´ 1

.w
3 1 1
Then, 2x + 3y = and 3x + 2y =

o
4 ´ 18 ´ 2 10 8

rd
or d2 = = 9 days
16 7 1

p
Solving, we get : x = and y =

re
1 200 100

s
19. (c) 10 men’s 1 day’s work = ; \ (2 men + 1 boy)’s 1 day’s work

s
15

.c
æ 7 1 ö 16 2

o
1 = çè 2 ´ + 1´ ÷= =

m
15 women’s 1 day’s work = . 200 100 ø 200 25
12
(10 men + 15 women)’s 1 day’s work So, 2 men and 1 boy together can finish the work in
æ 1 1ö 9 3 1
= ç + ÷= = . 12 days.
è 15 12 ø 60 20 2
\ 10 men and 15 women will complete the work in 25. (a) Ratio of time taken by A and B = 160 : 100
=8:5
20 2 Suppose, B alone takes x days to do the job
= 6 days.
3 3 then, 8 : 5 : : 12 : x
æ 1 1ö 5 8x = 5 × 12
20. (a) Work done by A and B in 5 days = ç + ÷ ´ 5 =
è 10 15 ø 6 5 ´ 12 1
x= = 7 days.
5 1 8 2
Work remaining = 1 - =
6 6 æ 1 1 ö 15 1
\ C alone can do the work in 6 × 2 = 12 days 26. (c) Son’s 1 day’s work = ç - ÷ = = 7 days
è 3 5ø 2 2
5 5 2 27. (a) If x complete a work in x days. y will do the same task in
Ratio of their share work = : : = 3 : 2 :1
10 15 12 3x days.
Share of wages = ` 225, ` 150, ` 75. 3x – x = 40
1 Þ x = 20
21. (d) 1 man’s 1 day’s work = . y will finish the task in 60 days
100
(x + y)’s 1 days work
1
(10 men + 15 women)’s 1 day’s work = . 1 1 1
6 = + =
15 women’s 1 day’s work 20 60 15
æ 1 10 ö æ 1 1 ö 1 Both of them will complete the work in 15 days.
=ç - ÷=ç - ÷= . æ1 1 1 ö 1
è 6 100 ø è 6 10 ø 15
28. (c) Part filled by (A + B + C) in 1 hour = ç + + ÷ = .
1 è 5 10 30 ø 3
\ 1 woman’s 1 day’s work = . \ All the three pipes together will fill the tank in
225
\ 1 woman alone can complete the work in 225 days. 3 hours.
22. (a) Let the number of men originally employed be x. 1
9x = 15(x – 6) 29. (d) Part filled by first tap in one min = th
12
or x = 15
1
1 Part filled by second tap in one min = th
23. (c) In 8 days, Anil does = rd work . 18
3
Now, 2 éê + ùú + unfilled part = 1
1 1
1
\ in 1 day, he does = th work. ë12 18 û
24
1 1 13
\ Rakesh’s one day’s work = 60% of = th work . Þ unfilled part = th
24 40 18
1 2 Q 1 th part of tank is filled by second tap in 1min.
Remaining work = 1 - = 18
3 3
(Anil and Rakesh)’s one day’s work
\ 13 th part of tank is filled by second tap in 1 min.
1 1 1 18
= + = th work
24 40 15 13
= 18 ´ min = 13 min.
1 18
Now, th work is done by them in one day..
15 30. (c) Hint : Let the time be t hours after 1 a.m.
2 2 t ( t - 1) ( t - 2 )
\ rd work is done by them in 15 ´ = 10 days \ + - =1
3 3 4 5 2
y
o
u
rs
m
a
h
SOLUTIONS 57

b
o
o
11. (c) Let the ratio of proportionality be x, then

b
SPEED TEST 43 4x × x = 196 or, 4x2 = 196 or, x = 7

.w
Thus, Father’s age = 28 yrs, Son’s age = 7 yrs

o
1. (c) Let the age of father and son be 15x years and x years After 5 yrs, Father’s age = 33 yrs.

rd
respectively. Son’s age = 12 yrs

p
15x + x \ Ratio = 33 : 12 = 11 : 4

re
Now, according to the question, = 16 12. (b) Difference in ratios = 8
2

s
Then 8 º 24 \ 1 º 3

s
16 ´ 2

.c
or, x = = 2years i.e., value of 1 in ratio is equivalent to 3 yrs
16

o
Thus, Rita’s age = 3 × 3 = 9 yrs

m
Hence age of the son = 2 years
Mother’s age = 11 × 3 = 33 yrs.
2. (b) Average age = 28.5
After 3 years, the ratio = 12 : 36 = 1 : 3
\ Total age = 28.5 × 2 = 57
13. (c) Let the present age be x yrs. Then
5
\ Daughter’s age = ´ 57 = 15 years 1
19 125% of (x – 10) = x; and 83 % of (x + 10) = x
3
3. (a) Let Sudha’s and Neeta’s present ages be 6x and 7x years
1
respectively. \ 125% of (x – 10) = 83 % of (x + 10)
According to the question. 3
6x - 5 5 5 5
= (x = 10) = (x + 10)
7x - 5 6 4 6
Þ 36x – 30 = 35x – 25 5 5 50 50
or, x - x = +
Þx=5 4 6 6 4
\ Sudha’s present age = 6 × 5 = 30 years 5x 250
or, = \ x = 50 yrs.
æ 15 ´ 36 + 12 ´ 16 ö 12 12
4. (a) Required average age = ç ÷ years 14. (b) Let the father’s present age be x and son’s age be x1 and x2.
è 36 + 12 ø
æ 540 + 192 ö æ 732 ö Now, x = 3(x1 + x2) .....(i)
= ç ÷ years = ç ÷ years = 15.25 years. Also, x + 5 = 2(x1 + 5 + x2 + 5)
è 48 ø è 48 ø x + 5 = 2(x1 + x2 + 10) .....(ii)
5. (a) Let the ages of Swati and Khyati two years ago be 5x and 7x x
years respectively. Putting value of (x1 + x2) = from (i) in equation (ii)
According to the question, 3
5x + 4 7 æx ö
= x + 5 = 2 ç + 10 ÷ = 45
7x + 4 9 è3 ø
Þ 49x + 28 = 45x + 36 15. (a) Let the present ages of P and Q be 3x and 4x respectively.
Þ 4x = 8 Þ x = 2 After 4 years
\ Khyati’s present age = 7x + 2 = 7 × 2 + 2 = 16 years 4x – 3x = 5
6. (b) Shortcut method : \x=5
\ P’s present age = 3x = 3 × 5 = 15 years
5(9 - 1) 16. (c) Let the present ages of Rama and Shyama be 4x and 5x years
Son’s age = = 8 yrs
(9 - 4) respectively,
\ Father’s age = 4 × 8 = 32 yrs 4x + 5 5
7. (b) Shortcut method : \ =
5x + 5 6
5(7 - 1) + 5(3 - 1) Þ 25x + 25 = 24x + 30
Son’s age = = 10 yrs Þ x = 30 – 25 = 5
7 -3
From the first relationship of ages, if F is the age of the father \ Rama’s present age = 4 × 5 = 20 years
then F + 5 = 3 (10 + 5) 17. (d) Let the mother’s age be y years.
\ F = 40 yrs \ The age of father = (y + 9) years
8. (c) Shortcut method : y
10(4 - 1) + 10(2 - 1) The age of son = years
Daughter’s age = = 20 yrs 2
4-2 æy ö
1 The age of daughter = ç - 7 ÷ years
9. (e) 10 yrs ago, A was of B’s ago. è2 ø
2 Now according to the given condition,
3 æy ö
AT present, A is
4
of B’s age. (y + 9) = 3ç - 7 ÷
è2 ø
æ1 ö 3 y - 42
10 ç - 1÷ Þ y+9=
è2 ø 2
\ B’s age = = 20 yrs
1 3 Þ 2y + 18 = 3y – 42
- Þ y = 60 years
2 4
18. (d) Suppose age of Ram = R
3
A’s age = of 20 = 15 yrs his son’s age = S
4 and his father’s age = F
10. (b) Let the age of the daugher be x yrs.
R 2
Then, the age of the mother is (50 – x) yrs. According to question, S = and R = F ´
5 yrs ago, 7(x – 5) = 50 – x – 5 3 5
or, 8x = 50 – 5 + 35 = 80 5R R +S+ F
\ F= and = 46
\ x = 10 2 3
Therefore, daughter’s age = 10 yrs R + S + F = 46 × 3
and mother’s age = 40 yrs
y
o
u
rs
m
a
h
58 101 SPEED TEST

b
o
o
R 5R 30. (a) Let the present age of the father be ‘x’ and that of the son be
R+ + = 138

b
3 2

.w
x 8
R = 36 ‘y’. Then =
y 3

o
36

rd
S= = 12 \ 3x = 8y ...(i)
3

p
5 ´ 36 x + 12 2

re
F= = 90 Further, y + 12 = 1 \ x + 12 = 2y + 24

s
2

s
19. (c) Let the ages of Abhay and his father 10 years ago be x and 5x

.c
\ x – 2y = 12 ...(ii)
years respectively. Then,

o
From eqn (i) and (ii), x = 48, y = 18

m
Abhay’s age after 6 years = (x + 10) + 6 = (x + 16) years. \ sum = 66 yrs.
Father’s age after 6 years = (5x + 10) + 6 = (5x + 16) years.
3 SPEED TEST 44
\ (x + 16) = ( 5x + 16 ) Û 7 (x + 16) = 3 (5x + 16)
7
Û 7x + 112 = 15x + 48 1. (c) Required number of ways
Û 8x = 64 Û x = 8. = ways of selecting 4 objects out of 6 given objects
Hence, Abhay’s father’s present age = (5x + 10) = 50 years. 6´5
= 6C4 = = 15
20. (b) Let their present ages be 4x, 7x and 9x years respectively. 2
Then, (4x – 8) + (7x – 8) + (9x – 8) = 56 Û 20x = 80 Û x = 4. 2. (c) Total no. of unrestricted arrangements = (7 – 1) ! = 6 !
When two particular person always sit together, the total no.
\ Their present ages are 16 yrs, 28 yrs. and 36 yrs. respectively.
of arrangements = 6! – 2 × 5!
21. (d) 16 years ago, let T = x years and G = 8x years Required no. of arrangements = 6! – 2 × 5!
After 8 years from now, T = (x + 16 + 8) years and = 5! (6 – 2) = 5 × 4 × 3 × 2 × 4 = 480.
G = (8x + 16 + 8) years. 3. (c) In MATHEMATICS, the consonants M and T are repeated
\ 8x + 24 = 3(x + 24) Û 5x = 48. two times each.
48 Also the vowel A is repeated two times.
+8
T x +8 88 11 Since there are four vowels, A, A, E and I; A being repeated,
8 years ago, = = 5 = =
48
G 8x + 8 8 ´ + 8 424 53 4
5 therefore vowels can be arranged in = 12 ways.
2
22. (d) R – Q = R – T Þ Q = T. Also, R + T = 50 Now remaining 7 consonants, with M, T being repeated, can
Þ R + Q = 50
So, (R – Q) cannot be determined. 7
be written in = 7 × 6 × 5 × 3 × 2 = 1260 ways.
23. (e) Let the ages of father and son be x and (45 – x) years 2´2
respectively. Now four vowels together can take any of the 8 places as
Then, (x – 5) (45 – x – 5) = 34 shown below:
VC VC VC VC VC VC VC V
Û (x – 5) (40 – x) = 34 Û x2 – 45x + 234 = 0
\ Total number of ways in which the letters of the word
Û (x – 39) (x – 6) = 0 Û x = 39 or x = 6. MATHEMATICS can be arranged such that vowels always
\ Father’s age = 39 years and son’s age = 6 years come together = 1260 × 8 × 12 = 120960.
24. (a) Let the ages of children be x, (x + 3), (x + 6) 4. (c) There are 8 different letters in the word MATHEMATICS;
and (x + 12) years. three letters M, A and T being repeated.
Then, x + (x + 3) + (x + 6) + (x + 9) + (x + 12) = 50 The number of ways in which four letters of the word
Û 5x = 20 Û x = 4. MATHEMATICS can be arranged = 8 P4
\ Age of the youngest child = x = 4 years. = 8 × 7 ×6 × 5 = 1680
25. (b) Anup’s age = (5 – 2) years = 3 years. Let Gagan’s age be x 6!
years. 5. (c) Number of ways = (Q T letter comes in two time)
2!
x -6
Then, = 3 Û x – 6 = 54 Û x = 60. 6 ´ 5 ´ 4 ´ 3 ´ 2 ´1
18 = = 360
26. (c) Let the school ages of Neelam and Shaan be 5x and 6x 2 ´1
respectively. Then, 6. (e) CYCLE whereas C comes two times.
1 5! 5 ´ 4 ´ 3 ´ 2
´ 5x æ1 ö æ5 ö So, arrangements are = = = 60 ways
3 5 2! 2
= Û ç ´ 9 ´ 5x ÷ = ç ´ 6x ÷ Û 15 = 15.
1 9 è3 ø è2 ø 5!
´ 6x 7. (e) Required no. of ways = = 60 is
2 2!
Thus, Shaan’s age cannot be determined. Total no. of letters in the word is 5; T is repeated twice.
27. (d) Let C’s age be x years. Then, B’s age = 2x years. A’s age = (2x 8. (c) The committee of 4 persons is to be so formed that it has at
+ 2) years. least 1 woman.
\ (2x + 2) = 2x + x = 27 Û 5x = 25 Û x = 5. The different ways that we can choose to form such a
Hence, B’s age = 2x = 10 years. committee are:
28. (d) Let the present ages of the father and son be 2x and x years 6´ 5´ 4
(i) 1w. 3 m in 4C1 × 6C3 = 4 ´ = 80
respectively. 3 ´ 2 ´1
Then, (2x – 18) = 3 (x – 18) Û x = 36. 4´ 3 6´ 5
(ii) 2w. 2 m in 4C2 × 6C2 = ´ = 90
\ Required sum = (2x + x) = 3x = 108 years. 2 ´1 2 ´1
29. (d) Let the ages of Preeti and Sonal 1 year ago be 4x and 4 6
(iii) 3w. 1 m in C3 × C1 = 4 × 6 = 24
x years respectively. (iv) 4w in 4C4 = 1
Then, [(4x + 1) + 6] – [(x + 1) + 6] = 9 Û 3x = 9 Û x = 3. \ Total no. of different ways in which a committee of 4
\ Required ratio = (4x + 1) : (x + 1) = 13 : 4. persons can be formed so that it has at least one woman.
= 80 + 90 + 24 + 1 = 195
y
o
u
rs
m
a
h
SOLUTIONS 59

b
o
o
9. (d) The committee of 4 persons is to be so formed that it has at
\ Reqd probability = 7 = 1

b
least 2 men. The different ways that we can choose to form

.w
such a committee are: 21 3

o
6´5 3´3 22. (c) Taking all vowels (IEO) as a single letter (since they

rd
(i) 2m. 2w in 6C2 × 4C2 = ´ = 90
2 ´1 2 ´1 come together) there are six letters

p
re
6´ 5´ 4 6!
(ii) 3m. 1w in 6C3 × 4C1 ´ 4 = 80 Hence no. of arrangements = ´ 3! = 2160

s
3 ´ 2 ´1

s
2!

.c
6´5 [Three vowels can be arranged 3! ways among

o
(iii) 4m in 6C4 = = 15
themselves, hence multiplied with 3!.]

m
2 ´1
\ Total no. of different ways in which a committee of 4 23. (b) When 0 is the repeated digit like
persons can be formed so that it has at least 2 men. 100, 200, ...., 9 in number
= 90 + 18 + 15 = 185 When 0 occurs only once like
10. (e) One girl can be chosen in 4C1 = 4 ways 110, 220, ....., 9 in number
and 4 boys can be chosen in 6C4 = 15 ways When 0 does not occur like
\ Total number of ways = 4 × 15 = 60 ways
11. (a) CORPORATION= 11 letters
112, 211, ....., 2 × (8 × 9) = 144 in number.
‘O’ comes thrice, ‘R’ twice. Hence, total = 9 + 9 + 144 = 162.
11! 24. (b) Required number of possible outcomes
\ total no. of ways = = 3326400 = Total number of possible outcomes –
3!2!
Number of possible outcomes in which all vowels are
12. (b) There are seven letters in the word “COUNTRY” and two
vowels O and U. Considering two vowels as one unit, total together
number of letters will be 5 + 1 = 6. So, number of arrangements = 6 ! – 4 ! × 3 != 576
= 6! 25. (d) The required number of ways
Now, the two vowels can be arranged in 21 ways among = (10 + 1)(9 + 1)(7 + 1) - 1 = 879 .
themselves.
26. (c) Three digit number less then 600 will have first element
\ Total number of ways = 6! × 2! = 1440
13. (c) The word PROBLEM consists of 7 distinct letters. 100, and last element 599. First place will not have digit
\ Number of arrangements = 7! more than 6, hence, 7 and 9 can not be taken : So, first
= 70 × 6 × 5 × 4 × 3 × 2 × 1 = 5040 digit can be selected in 4 ways. Second digit can be
14. (e) There are 6 letter inthe word ‘ATTEND’ whereas, T comes selected in 6 ways and since repetition of digits are
two times. allowed, third digit can also be selected in 6 ways :
6! 720 So, number of ways are 4 × 6 × 6 = 144.
So, required number of ways = = = 360 27. (b) Selection of 2 members out of 11 has 11C2 number of
2! 2
15. (a) In word ‘offices’, there are 7 letters and F comes two times. ways
11C = 55
7! 7 ´ 6 ´ 5 ´ 4 ´ 3 ´ 2! 2
Required number of ways = = = 2520 28. (b) From each railway station, there are 19 different tickets
2! 2!
16. (e) ARMOUR = 6 letter whereas R repeated twice to be issued. There are 20 railway station
6! 6 ´ 5 ´ 4 ´ 3 ´ 2 ´ 1 So, total number of tickets = 20 × 19 = 380.
\ = = 360 29. (c) To make a 5 digit number, 0 can not come in the bagining.
2! 2 ´1
17. (c) Total number of ways to stand boys and girls together So, it can be filled in 4 ways. Rest of the places can be
= 4! × 3! × 2! = 4 × 3 × 2 × 3 × 2 × 2 = 288 filled in 4! ways. So total number of digit formed = 4 × 4!
18. (e) = 2 × 24 = 96
30. (b) Total no. of digits = 6
O, A, E S F T W R To form a odd numbers we have only 3 choice for the
When the vowels are always together, then treat all unit digits.
the vowels as a single letter and then all the letters Now, Extreme left place can be filled in 6 ways the
can be arranged in 6! ways and also all three vowels middle place can be filled in 6 ways.
can be arranged in 3! ways. Hence, required no. of \ Required number of numbers = 6 × 6 × 3 = 108
arrangements = 6! × 3! = 4320.
5 SPEED TEST 45
5 ´ 4 10
C2
19. (b) Reqd probability = 7 = =
C2 7 ´ 6 21 1. (a) Here, S = {1, 2, 3, 4, ...., 19, 20}.
Let E = event of getting a multiple of 3
20. (b) Treat B and T as a single letter. Then the remaining
= {3, 6, 9, 12, 15, 18}
letters (5 + 1 = 6) can be arranged in 6! ways. Since, O is
repeated twice, we have to divide by 2 and the B and T n(E) 6 3
\ P(E) = = = .
letters can be arranged in 2! ways. n(S) 20 10
6! ´ 2! 10 10 2
Total no. of ways = = 720
2 2. (c) P (getting a prize) = = = .
(10 + 25) 35 7
21. (e) If the drawn ball is neither red nor green, then it must be
3. (b) Clearly, there are 52 cards, out of which there are 16
blue, which can be picked in 7C1 = 7 ways. One ball can
face cards.
be picked from the total
(8 + 7 + 6 = 21) in 21C1 = 21 ways. 16 4
\ P (getting a face card) = = .
52 13
y
o
u
rs
m
a
h
60 101 SPEED TEST

b
o
o
4. (c) Here, n(S) = 52. n(E) 6 1

b
There are 26 red cards (including 2 kings) and there are \ P(E) = = = .

.w
2 more kings. n(S) 36 6

o
Let E = event of getting a red card or a king. 14. (d) In a simultaneous throw of two dice, we have

rd
Then, n(E) = 28. n(S) = (6 × 6) = 36.

p
re
Let E = event of getting a total of 10 or 11
n(E) 28 7

s
\ P(E) = = = . = {(4, 6), (5, 5), (6, 4), (5, 6), (6, 5)}

s
.c
n(S) 52 13
n(E) 5

o
5. (b) Total number of balls = (6 + 8) = 14 \ P(E) = = .

m
Number of white balls = 8 n(S) 36
15. (a) Total possible outcomes, S ={HHH, HHT, HTH, THT,
8 4 TTH, THH, TTT, HTT} and desired outcomes E ={HTH,
P (drawing a white ball) = = .
14 7 THT}
6. (d) Total number of balls = (8 + 7 + 6) = 21. Þ n(E) = 2 and n(S) = 8
Let E = event that the ball drawn is neither red nor green. n(E) 2 1
= event that the ball drawn is red. Hence, required probability = P(E) = = =
n(S) 8 4
\ n(E) = 8 16. (c) 16 tickets are sold and 4 prizes are awarded. A person
8 4 1
\ P(E) = . =
21 buys 4 tickets, then required probability =
16 4
7. (b) Clearly, n(S) = (6 × 6) = 36. 17. (c) Total number of letters = 4
Let E = Event that the sum is a prime number. Total number of vowels = 2 (O and E)
Then, E = {(1, 1), (1, 2), (1, 4), (1, 6), (2, 1), (2, 3), (2, 5), (3,
2), (3, 4), (4, 1), (4, 3), (5, 2), (5, 6), (6, 1), (6, 5)} 2 1
Required Probability = =
\ n(E) = 15 4 2
18. (d) Total no. of case = 63 = 216
n(E) 15 5 Favourable cases = {(1, 1, 1), (2, 2, 2), (3, 3, 3), (4, 4, 4),
\ P(E) = = = .
n(S) 36 12 (5, 5, 5), (6, 6, 6)}.
8. (d) Here S = {HH, HT, TH, TT}. 6 1
Probability = =
Let E = event of getting at least one head 216 36
= {HT, TH, HH} 19. (b) No. of days in leap year = 366
n(E) 3 No. of complete week = 52
\ P(E) = = .
n(S) 4 (Q 366 ¸ 7 gives 2 as remainder )
9. (b) Here S = {TTT, TTH, THT, HTT, THH, HTH, HHT, HHH} \ No. of days left = 2
Let E = event of getting at least two heads 2
= {THH, HTH, HHT, HHH} Required probability =
7
n(E) 4 1 20. (a) Total case = 6 × 6 = 36
\ P(E) = = = .
n(S) 8 2 Favourable = (1, 6), (2, 5), (3, 4), (4, 3), (5, 2), (6, 1)
10. (b) When a die is thrown, we have S = {1, 2, 3, 4, 5, 6} 6 1
Let E = event of getting a number greater than 4 Probability = =
36 6
= {5, 6}.
21. (a) Total number of balls = 5 + 7 + 8 = 20
n(E) 2 1 Probability that the first ball drawn is white
\ P(E) = = = .
n(S) 6 3 5
C 1
11. (a) We know that in a simultaneous throw of two dice, = 20 1 =
C1 4
n(S) = 6 × 6 = 36.
Let E = event of getting a total f 7 If balls are drawn with replacement, all the four events
= {(1, 6), (2, 5), (3, 4), (4, 3), (5, 2), (6, 1)} will have equal probability.
Therefore, required probability
n(E) 6 1 1 1 1 1 1
\ P(E) = = = . = ´ ´ ´ =
n(S) 36 6 4 4 4 4 256
12. (c) In two throws of a die, n(S) = (6 × 6) = 36. 22. (b) Total no. of outcomes when two dices are thrown = n
Let E = event of getting a sum 9 (S) = 36 and the possible cases for the event that the
= {(3, 6), (4, 5), (5, 4), (6, 3)} sum of numbers on two dice is a prime number, are
(1, 1), (1, 2), (1, 4), (1, 6), (2, 1), (2, 3), (2, 5), (3, 2), (3, 4),
n(E) 4 1
\ P(E) = = = . (4, 1), (4, 3), (5, 2), (5, 6), (6, 1), (6, 5)
n(S) 36 9 Number of outcomes favouring the event = n (A) = 15
13. (a) In a simultaneous throw of two dice, n(S) = 6 × 6 = 36.
n ( A) 15 5
Let E = event of getting a doublet Required probability = = =
= {(1, 2), (2, 2), (3, 3), (4, 4), (5, 5), (6, 6)} n(S ) 36 12
y
o
u
rs
m
a
h
SOLUTIONS 61

b
o
o
1 Area of the room = Area of the carpet

b
23. (d) The probability of selecting any bag =

.w
2 æ 75 ö 2 2
= ç 180 ´ ÷ m = 135 m .

o
Now, probability of getting a white ball from the first è 100 ø

rd
1 3 3

p
bag = ´ = æ Area ö æ 135 ö

re
2 5 10 \ Breadth of the room = ç ÷ =ç ÷ m = 7.5 m.
è Length ø è 18 ø

s
and probability of getting a white ball from the second

s
.c
5. (a) In a rectangle,
bag = 1 ´ 2 = 1

o
m
2 6 6 (perimeter) 2
Required Probability = The probability that a white ball = (diagonal) 2 + 2 ´ area
4
is drawn either from the first or the second bag
(14)2
=
3 1 7
+ = Þ = 52 + 2 ´ area
10 6 15 4
24. (b) Out of 20 consecutive numbers there are 10 even and 49 = 25 + 2 × area
10 odd 49 - 25 24
We have to choose 2 numbers out of 20 \ Area = = = 12cm 2
2 2
20 20 ´ 9 6. (a) In an isoscele right angled triangle,
\ Total outcomes = C2 = = 190 Area = 23.3 × perimeter2
2
We can odd sum only when one even and one odd is = 23.3 × 202 = 9320 m2
added. 7. (b) Required area covered in 5 revolutions
\ No. of ways of choosing 1 even and 1 odd 22
Out of 10 even and 10 odd = 10C1 × 10 C1 = 10 × 10 = 100 = 5 × 2prh = 5 × 2 × × 0.7 × 2 = 44 m2
7
100 10 8. (c) In a triangle,
\ Required probability = =
190 19 1
25. (b) Here S = {1, 2, 3, 4, 5} Area = ´ length of perpendicular × base
2
Let E be the event of getting a multiple of 3.
E = {3, 6} 1
or 615 = ´ length of perpendicular × 123
2
2 1
\ P(E) = = . 615 ´ 2
6 3 \ Length of perpendicular = = 10 m.
123
SPEED TEST 46 14 m
9. (a) D C
1. (a) In a circle, circumference = 2pr
14 m
44
Hence, 44 = 2pr \ r= 24 m
2p
44 44
Now, area of circle = p r2 = p ´ ´ = 154 m 2 40 m
2p 2p A B
2. (a) Let the length and breadth of a rectangle are 9 xm and Area of the shaded portion
5 xm respectively.
1
In a rectangle, area = length × breadth = ´ p(14 )2
\ 720 = 9x × 5x 4
or x2 = 16 Þ x = 4 = 154 m2
Thus, length = 9 × 4 = 36 m 10. (a) Circumference of circular bed = 30 cm
and breadth = 5 × 4 = 20 m (30)2
Therefore, perimeter of rectangle = 2(36 + 20) = 112 m Area of circular bed =
4p
3. (d) Perimeter of the circle = 2pr = 2(18 + 26) Space for each plant = 4 cm2
22 \ Required number of plants
Þ 2´ ´ r = 88 Þ r = 14
7 (30)2
= ¸ 4 = 17.89 = 18 (Approx)
\ Area of the circle 4p
2 22 11. (c)
= pr = ´ 14 ´14 = 616 cm 2 .
7
4. (b) Length of the carpet 10 W
æ Total cost ö = æ 8100 ö m = 180 m. 15
=ç ÷ ç ÷
è Rate / m ø è 45 ø
y
o
u
rs
m
a
h
62 101 SPEED TEST

b
o
o
Let the width of the path = W m
A

b
then, length of plot with path = (15 + 2W) m

.w
and breadth of plot with path = (10 + 2 W) m

o
rd
Therefore, Area of rectangular plot (wihout path)
= 15 × 10 = 150 m2

p
x x

re
and Area of rectangular plot (with path)

s
= 150 + 54 = 204 m2

s
.c
Hence, (15 + 2W) × (10 + 2W) = 204

o
B D C

m
Þ 4W2 + 50 W – 54 = 0
Þ 2W2 + 25 W – 27 = 0 Since, in an isosceles triangle, the altitude bisects the
Þ (W – 2) (W + 27) = 0 base. So, BD = DC = (16 – x).
Thus W = 2 or –27 In DADC, AC2 = AD2 + DC2
\ with of the path = 2 m Þ x2 = (8)2 + (16 – x)2
12. (a) If area of a circle decreased by x % then the radius of a Þ 32x = 320 Þ x = 10.
circle decreases by \ BC = (32 – 2x) = (32 – 20) cm = 12 cm.

(100 - 10 100 - x )% = (100 - 10 100 - 36)% æ1 ö


Hence, required area = ç ´ BC ´ AD ÷
è 2 ø
= (100 - 10 64)%
æ1 ö
= 100 - 80 = 20% = ç ´12 ´10 ÷ cm 2 = 60 cm 2 .
13. (a) Area of the outer rectangle = 19 × 16 = 304 m2
è2 ø
17. (c) Area of field = 576 km2. Then,
2m each side of field = 576 = 24 km
Distance covered by the horse
= Perimeter of square field
= 24 × 4 = 96 km
12 distance 96
2m =
2m \ Time taken by horse = =8h
speed 12
15 18. (c) Let the length and breadth of the original rectangular
2m field be x m and y m respectively.
Area of the original field = x × y = 144 m2
Area of the inner rectangle = 15 × 12 = 180 m2 144
\x= … (i)
Required area = (304 – 180) = 124 m2 y
14. (a) Radius of a circular grass lawn (without path) = 35 m If the length had been 6 m more, then area will be
\ Area = pr2 = p (35)2 (x + 6) y = 144 + 54
Radius of a circular grass lawn ( with path) Þ (x + 6) y = 198 … (ii)
= 35 + 7 = 42 m Putting the value of x from eq (i) in eq (ii), we get
\ Area = pr2 = p(42)2
\ Area of path = p(42)2 – p(35)2 æ 144 ö
ç + 6 ÷ y = 198
= p(422 – 352) è y ø
= p( 42 + 35) (42 –35) Þ 144 + 6y = 198
22 2 Þ 6y = 54 Þ y = 9 m
= p × 77 × 7 = ´ 77 ´ 7 = 1694 m Putting the value of y in eq (i) we get x = 16 m
7
15. (b) Radius of the wheel of bus = 70 cm. Then, 19. (b) Perimeter = Distance covered in 8 min.
circumference of wheel = 2pr = 140 p = 440 cm æ 12000 ö
Distance covered by bus in 1 minute =ç ´ 8 ÷ m = 1600 m.
è 60 ø
66 Let length = 3x metres and breadth = 2x metres.
= ´ 1000 ´100 cms
60 Then, 2 (3x + 2x) = 1600 or x = 160.
Distance covered by one revolution of wheel \ Length = 480 m and Breadth = 320 m.
\ Area = (480 × 320) m2 = 153600 m2.
= circumference of wheel
= 440 cm æ 22 ö
20. (c) Length of wire = 2p´ R = ç 2 ´ ´ 56 ÷ cm = 352 cm.
6600000 è 7 ø
\ Revolutions per minute = = 250
60 ´ 440 352
16. (b) Let ABC be the isosceles triangle and AD be the altitude. Side of the square = cm = 88 cm.
4
Let AB = AC = x. Then, BC = (32 – 2x). Area of the square = (88 × 88) cm2 = 7744 cm2.
y
o
u
rs
m
a
h
SOLUTIONS 63

b
o
21. (a) Let the length of the room be l m

o
5

b
Then its, breadth = l /2 27. (a) 79.2 km/hr = 79.2 ´ = 22 m/s

.w
18

o
l 5000 2 min 40 sec = 2 ´ 60 + 40 = 120 + 40 = 160 sec.

rd
Therefore, l ´ = Circumference of circular field = speed × time
2 25

p
re
l 2 = 400 = 22 ´ 160 = 3520m
or

s
s
or l = 20 m circumference

.c
Radius of circular field (r) =

o
2p
Also, 2lh + 2 ´ l ´ h = 64800

m
2 240 3520 ´ 7
= = 560m
Þ 3 l h = 270 2 ´ 22
22
270 270 Area of circular field = pr 2 = ´ (560) 2
or h= = = 4.5 m 7
3 ´ 20 60
22. (a) In a cube, 22
= ´ 560 ´ 560
Area = 6 (side)2 7
or 150 = 6 (side)2 = 985600 m2
28. (d) Area of rectangle = l × b = 240
\ side = 25 = 5 m Either length or breadth should be clear, then answer can be
determined.
Length of diagonal = 3 ´ side = 5 3 m 29. (c) Area of the circle = 39424 sq cm
23. (b) Given, playground is rectangular. 22 2
pr2 = 39424 Þ ´ r = 39424
Length = 36 m, Breadth = 21 m 7
Now, perimeter of playground = 2( 21 + 36) = 114 39424 ´ 7
2
Now, poles are fixed along the boundary at a distance Þ r = Þ r2 = 1792 × 7
22
3m.
Þ r = 12544 r = 112 cm 4a = r 4a = 112 a = 28
114 \ Area of the square = a2 = (28)2 = 784 sq cm
\ Required no. of poles = = 38 . 30. (b) From the figure, it is required to find the length CD.
3
We have CA = LB = 15m
1 \ LD = BD – LB = 15m
24. (d) Area of square = ´ (diagonal)2 D
2
1 1 15 m
= ´ ( 28) = ´ 28 ´ 28
2
2 2 C L
36 m
15 m
= 392 cm2 A B
36 m
circumference 44
25. (c) One side of square = = = 11 cm \ CD = CL2 + DL2 = 362 + 152 = 1521 = 39 cm
4 4
31. (d) Total area = Area of square + 4 (Area of a semi-circle)
Circumference of rectangle = 4 × perimeter of square
2 æ 1 2ö é 2 ù
= 4 × 44 = 176 cm = 2 + 4 ç pr ÷ = (4 + 2p) m2 ê radius = = 1ú
è2 ø ë 2 û
width of rectangle
32. (b) In any quadrilateral,
circumference of rectangle Area of the quadrilateral
= – length
2 1
= × any diagonal × (sum of perpendiculars drawn on
2
176
= - 51 = 88 - 51 = 37cm. 1
2 diagonal from two vertices) = ´ D ´ (P1 + P2 )
2
\ Required difference = width – side = 37 – 11 = 26 cm.
1
= ´ 23 ´ (17 + 7) = 12 × 23 = 276 sq cm
circumference 220× 7 2
26. (c) Radius of circle (r) = = = 35 m. 33. (b) Length of the wire = circumference of the circle
2p 2× 22
2 ´ 22 ´ 42
22 22 = 2p × 42 = = 264 cm
area of circle = pr 2 = ´ (35)2 = ´ 35 ´ 35 7
7 7 Now, perimeter of the rectangle = 264 cm.
= 3850 m2 = area of rectangle Since, perimeter includes double the length and breadth, while
finding the sides we divide by double the sum of ratio.
\ Length of rectangle = area of rectangle 264
´ 6 = 72 cm
Therefore, length =
width 2(6 + 5)
3850 264
= = 77 m. and breadth = ´ 5 = 60 cm
50 2(6 + 5)
y
o
u
rs
m
a
h
64 101 SPEED TEST

b
o
o
34. (a) Let the final length of the side of the smaller square be a.
Surface area of bigger cube = 6 A2

b
Now, a + 10 + 7 = 19

.w
a = 19 – 17 = 2 or 384 = 6A2
\ A = 8 cm.

o
\ Area of the smaller square = (2)2 = 4

rd
\ Decrease in the area of the smaller square = 16 – 4 Surface area of smaller cube = 6 a 2

p
= 12 sq. units. 96 = 6a2

re
35. (c) Let the breadth be b. \ a = 4 mm = 0.4 cm

s
s
Then, length = b + 13 Perimeter = 50 = 2(l + b)

.c
Volume of bigger cube
2(b + 13 + b) = 50 b = 6 m l = 6 + 13 = 19 m So, Number of small cube =

o
\ Area = length × breadth = 19 (6) = 114 m2 Volume of smaller cube

m
SPEED TEST 47 (8)3 512
= 3
= = 8, 000
(0.4) 0.064
1. (a) Volume of the bucket = volume of the sand emptied
9. (c) Volume of the liquid in the cylindrical vessel
Volume of sand = p (21)2 × 36 = Volume of the conical vessel
Let r be the radius of the conical heap.
æ 1 22 ö 3
Then,
1 2
pr ´ 12 = p(21)2 ´ 36 = ç ´ ´12 ´12 ´ 50 ÷ cm
3 è3 7 ø
or r2 = (21)2 × 9 or r = 21 × 3 = 63 æ 22 ´ 4 ´ 12 ´ 50 ö 3
2. (a) Let the edge of the third cube be x cm. =ç
7 ÷ cm .
Then, x3 + 63 + 83 = 123 è ø
Þ x3 + 216 + 512 = 1728 Let the height of the liquid in the vessel be h.
Þ x3 = 1000 Þ x = 10. 22 22 ´ 4 ´12 ´ 50
Then, ´ 10 ´ 10 ´ h =
Thus the edge of third cube = 10 cm. 7 7
3. (b) Area of the inner curved surface of the well dug
æ 4 ´ 12 ´ 50 ö
22 or h = ç ÷ = 24 cm.
= [2p × 3.5 × 22.5] = 2 ´ ´ 3.5 ´ 22.5 è 10 ´10 ø
7 10. (b) Volume of material in the sphere
= 44 × 0.5 × 22.5 = 495 sq. m.

4.
\ Total cost = 495 × 3 = ` 1485.
(d) Let the length of the wire be h cm.
é4
{ 3
}
3 ù 3 æ4 ö 3
= ê p´ (4) - (2) ú cm = ç p´ 56 ÷ cm .
ë 3 û è 3 ø
and radius of sphere and wire are R and r respectively. Let the height of the cone be h cm.
Then, volume of sphere = volume of wire (cylinder)
1 æ4 ö
4 4 Then, p´ 4 ´ 4 ´ h = ç p´ 56 ÷
or p R 3 = p r 2h or R 3 = r 2h 3 è 3 ø
3 3
æ 4 ´ 56 ö
4 Þ h =ç ÷ = 14 cm.
or (3)3 = (0.1)2 h è 4´ 4 ø
3
æ4 ö 3
4 ´ (3)3 108 11. (d) Volume of sphere = ç p´ 9 ´ 9 ´ 9 ÷ cm .
\ h= = = 3600 cm = 36 m è 3 ø
2 0.03
3 ´ (0.1)
æ1 ö 3
Volume of cone = ç p´ 9 ´ 9 ´ 9 ÷ cm .
5. (c) In a sphere, volume = 4 p r3 è 3 ø
3 Volume of wood wasted
and surface area = 4p r 2 éæ 4 ö æ1 öù 3
= êç p ´ 9 ´ 9 ´ 9 ÷ - ç p ´ 9 ´ 9 ´ 9 ÷ ú cm .
4 3 ëè 3 ø è 3 øû
According to question, p r ¸ 4p r 2 = 27 = (p × 9 × 9 × 9) cm3
3
or r = 27 × 3 = 81 cms æ ö
6. (a) Area of the wet surface = [2(lb + bh + lh) – lb] ç p´ 9 ´ 9 ´ 9 ÷
= 2(bh + lh) + lb \ Required percentage = ç ´ 100 ÷ %
çç 4 ÷÷
= [2(4 × 1.25 + 6 × 1.25) +6 × 4] m2 = 49 m2. ´ p´ 9 ´ 9 ´ 9
è 3 ø
7. (b) Let l be the length and b be the breadth of cold
storage. æ3 ö
L = 2B, H = 3 metres = ç ´100 ÷ % = 75%.
è4 ø
Area of four walls = 2[L × H + B × H] = 108 12. (b) Curved surface area of cylinder = 2prh
Þ 6BH = 108 Þ B = 6 \ Surface area of 50 cylindrical pillars = 50 × 2prh
\ L = 12, B = 6, H = 3 Now, Diameter of each cylindrical pillar = 50 cm
Volume = 12 × 6 × 3 = 216 m3
8. (c) Let 'A' be the side of bigger cube and 'a' be the side of 50
\ Radius = = 25 cm » .25 m
smaller cube 2
y
o
u
rs
m
a
h
SOLUTIONS 65

b
o
Also, height = 4m 1 cm3 weighs 10 g. Þ 2618 cm3 will weigh

o
b
\ Surface area = 50 × 2 × 3.14 ×. 25 × 4

.w
2618 ´10
= 314 × 1 sq m. = kg = 26.18 kg

o
= 314 sq. m. 1000

rd
Now, labour charges at the rate of 50 paise 22. (b) Volume of the steel used in the hemispherical bowl

p
per sq. m = 314 ×.5 = 157.0 º ` 157
[ ]

re
2 2 22
13. (c) Let the kerosene level of cylindrical jar be h. = p (4.5)3 – 43 = ´ ´ 27.125 = 56.83 cm3

s
3 3 7

s
.c
1 2 23. (d) Volume of the parallelopiped = 5 × 4 × 3 = 60 cm3
Now, Volume of conical vessel = pr h

o
Volume of the cube = 4 × 4 × 4 = 64 cm3

m
3
Since, radius (r) = 2 cm and height(h) = 3cm of conical Volume of the cylinder = p × 3 × 3 × 3 = 27p
vessel. = 84.8 cm3
1 4
\ Volume = p ´ 4 ´ 3 = 4p Volume of the sphere = p(3)3 = 36p = 113 cm3
3 3
Now, Volume of cylinderical jar = pr2h The required decreasing order is D, C, B and A.
= p (2)2h 24. (d) Let R be the radius of bigger sphere. Volume of one
= 4ph spheres + Volume of other sphere = Volume of bigger
Now, Volume of conical vessel = Volume of cylindrical sphere
Jar 4 3 4 3 4 3
Þ 4 p = 4 ph Then, p (1) + p ( 2 ) = pR
3 3 3
h = 1cm
Hence, kerosene level in Jar is 1 cm. 4 3 4
Þ p(1 + 23 ) = pR 3
14. (a) Let the rise in water level = x m 3 3
Now, volume of pool = 40 × 90 × x = 3600 x Þ R3 = 9
When 150 men take a dip, then displacement of 1
water = 8m3
\ R = 93
3600 x 900 25. (b) Given, Diameter of building = 54 cm
\ =8 Þ x = 2 Þ x = .33m
150 150 \ Radius = 27 cm.
Þ x = 33.33 cm Also, given area of the base of the wall = 352 cm2
15. (d) Let edge of the new cube = x cm. Let Dx be the thickness of the wall
Volume of the newly formed figure (cube) \ Area of the Base = 2 p r× D x
= sum of volume of smaller cubes. Þ 352 = 2 × p × 27 ×D x
i.e. (x)3 = (3)3 + (4)3 + (5)3 = 27 + 64 + 125 = 216 352 ´ 7 8 ´ 7 56
Þ x = 6 cm ÞDx= = = »2
2 ´ 22 ´ 27 27 27
16. (b) Volume of the spherical ball = volume of the water Hence, thickness of the wall is 2 cm.
displaced.
4 SPEED TEST 48
Þ pr 3 = p (12)2 × 6.75
3
144 ´ 6.75 ´ 3 1
Þ r3 = = 729 Þ r = 9 cm, 1. (c) In a right angled D, the length of the median is the
4 2
17. (d) Let x be the length of a side (edge of the cube) 1
Now, x3 = 12 x Þ x2 = 12 length of the hypotenuse . Hence BD = AC = 3cm.
2
Total surface area = 6x2 = 6 × 12 = 72 square units 2. (a) ÐD = 180 - ÐB = 180 - 70 = 110°
18. (b) Let an edge of the cube be ‘a’cm.
Now, 6a2 = 726 Þ a2 = 121 Þ a = 11m \ ÐACD = 180 - ÐD - ÐCAD
Volume of the cube = (11)3 = 1331 m3. 180 - 110 - 30 = 40°
19. (c) 1 litre = 1 (dm)3 3. (b) The sum of the interior angles of a polygon of n sides
Length of the cistern = 3m = 30 dm ( Q 1 m = 10 dm) p
Breadth of the cistern = 2m = 20 dm is given by the expression (2n – 4)
Height of the cistern = 1 m = 10 dm 2
\ Volume of the cistern = 30 × 20 × 10 = 6000 (dm)3 p p
Þ ( 2n – 4) ´ = 1620 ´
20. (c) Here r = 8 cm, h = 15 cm Þ l = 8 + 15 = 17
2 2 2 180
Curved surface area of the cone 1620 ´ 2
( 2n – 4) = 180 = 18
= pr l = p ´ 8 ´ 17 = 136 p cm 2
or 2n = 22
21. (c) Volume of the cone = 1 pr 2 h or n = 11
3 4. (c) Tangent at any point of a circle is ^ to the radius
In DOPT, OP2=PT2+OT2
1 22 (13)2 = (12)2 + OT2 Þ 169 – 144 = OT2
= ´ ´ 49 ´ 51 = 2618cm 3
3 7 Þ 25 = OT2 Þ 5 = OT
y
o
u
rs
m
a
h
66 101 SPEED TEST

b
o
o
5. (c) Let the angles of the triangle be 5x, 3x and 2x.

b
Now, 5x + 3x + 2x = 180° AB = 32 - 12 = 2 2 cm

.w
or 10x = 180 or x = 18

o
\ AC = 4 2 cm

rd
or Angles are 36, 54 and 90°
Given D is right angled. 13. (d) Let the angles be x and 3x

p
re
6. (c) m Ð ABM = 180º –120º = 60º We know, sum of the supplementry angles = 180°

s
\ D AMB is a 30º – 60º – 90º triangle. \ x + 3x = 180°

s
.c
Þ 4x = 180° Þ x = 45°

o
3 3 14. (d)
\ AM

m
AB = ×8=4 3
2 2 15. (c) Supplementary angles are pairs of angles whose
measures upto 180 degrees.
1 1
MB = AB = x 8 = 4 Hence, let one angle be x. Since they are equal, \ the
2 2 other angle is also equal to x.
(AC)2 = (AM)2 + (MC)2 = (4 3 )2 + (4 +7)2 So, x + x = 180 Þ 2x = 180
\ x = 90
= 48 + 121 = 169 ; AC = 169 = 13. 16. (b)
7. (d) A P 1
17. (b) Four right angle + rd right angle
3
10 cm 1
= 4 × 90 + × 90 = 360 + 30 = 390
3
18. (b) Two right angles = 2 × 90° = 180°
B R 1
C Q Half right angles = ´ 90° = 45°
2
D ABC and D PQR are similar.. Total two right angles and one half right angle
AB Perimeter of DABC AB 36
= Þ = = 180°
PQ Perimeter of DPQR PQ 24
+ 45°
36 225°
or AB = ´ 10 = 15
24
19. (a) Five and two-third of a right angle
8. (a) For the two similar triangles, we have
2
h12 Area of 1st D 9 = 5 × 90° + ´ 90° = 450° + 60° = 510°
= = 3
h 22 Area of IInd D 16 20. (c) Let the angle be q.
According to question
Þ h1 : h2 = 3 : 4
9. (c) Circumcentre of a triangle is the point of the intersection 2
of the perpendicular bisectors of its sides.
q= ( 90 - q ) ...(1)
3
10. (a) We have, x + y + (y + 20) = 180
or x + 2y = 160 ...(i) 1
and 4x – y = 10 ...(ii)
q= (180 - q ) ...(2)
4
From (i) and (ii), y = 70, x = 20 From eq. (1)
Angles of the triangles are 20°, 70°, 90°. Hence the
2
triangle is a right angled. q= ( 90 - q)
11. (a) Clearly (x + 1) will be the hypotenuse of the right 3
triangle [ x ³ 0] 3q = 180 – 2q
5q = 180 Þ q = 36º
Now, (x + 1)2 = x2 + (x – 1)2 21. (a) Let the angles are x, 2x, 3x and 4x
or x2 + 2x + 1 = x2 + x2 – 2x + 1 x + 2x + 3x + 4x = 360º
or x2 – 4x = 0 or x (x – 4) = 0 or x = 4
10x = 360º
The side of a triangle cannot be equal to zero.
x = 36º
There fore, x = 4
4th angle = 4 × x = 4 × 36 = 144º
\ Hypotenuse = (4 + 1) = 5
22. (b)
12. (d)
2 cm
O¢ O
O 5
1 cm 3 cm 3
C B 2 2cm A A B
M
y
o
u
rs
m
a
h
SOLUTIONS 67

b
o
In DOAM

o
11. (d) Pattern of the series would be as follows

b
52 = 32 + AM2 7 9 12 16 21

.w
25 – 9 = AM2

o
16 = AM2

rd
+2 +3 +4 +5
AM = 4

p
12. (c) Pattern of the series would be as follows

re
\ Length of chord AB
384 192 96 48 24

s
= 2 × AM = 2 × 4 = 8 cm

s
.c
23. (a) 2x + 3x = 180º ÷2 ÷2 ÷2 ÷2

o
5x = 180º 13. (e) Pattern of the series would be as follows

m
x = 36º 5× 1+1=6
Hence, angles are 72º and 108º respectively. As opposite 6 × 2 + 2 = 14
angle of parallelogram are equal. Therefore, measure of
14 × 3 + 3 = 45
all angles are 72º, 108º, 72º, 108º.
24. (d) Let the angles be 4x, 3x and 2 x. \ 45 × 4 + 4 = 184
4x + 3x + 2x = 180° x = 20° 14. (e) Pattern of the series would be as follows
\ angles are 80°, 60° and 40°. 8 9 13 22 38
25. (c) Second angle of parallelogram
= 180° – 45° = 135° +(1)2 +(2)2 +(3)2 +(4)2
\ Required value 15. (a) Pattern of the series would be as follows
= 135 + 2 × 45
= 135 + 90 = 225° 6 11 21 41 81

+5 +10 +20 +40


SPEED TEST 49
16. (b) The series pattern would be as follows :
7 13 25 49 97
121 117 108 92 67 31
1. (a)
2 2 2 2 2
–2 –3 –4 –5 –6 +6 +12 +24 +48
So, the above series is progressing of double
50 26 14 8 5 3.5 differences.
2. (b) 17. (d) The series pattern would be as follows :
÷2+1 ÷2+1 ÷2+1 ÷2+1 ÷2+1 5 6 10 19 35
3 23 43 63 83 103
3. (c) 2 2 2 2
(1) (2) (3) (4)
+20 +20 +20 +20 +20 18. (a) The series pattern would be as follows :
748 737 715 682 638 583 8× 1+1=9
4. (e) 9 × 2 + 2 = 20
–11 –22 –33 –44 –55 63 × 4 + 4 = 256
19. (e) The series pattern would be as follows :
1 9 25 49 81 121 169
11 13 16 20 25
5. (d)
12 32 52 72 92 112 132
+2 +3 +4 +5
6. (b) 36 20 12 8 6 5 20. (c) The series pattern would be as follows :
E55555F E55555
F E555F E5555F E55555
F 608 304 152 76 38
¸ 2+ 2 ¸2 + 2 ¸ 2+ 2 ¸2 + 2 ¸2 + 2

7. (c) 668 656 632 584 448 296


E55555F E55555 F E55555F E55555F E55555 F ÷2 ÷2 ÷2 ÷2
-12 -24 -48 -96 -192
E55555F E55555 F E55555F E55555F 21. (d) The series is based on the following pattern:
´2 ´2 ´2 ´2 2 × 3 + 5 = 11
11 × 4 – 6 = 38
8. (e) 1 121 441 961 1681 2601 38 × 5 + 7 = 197
­ ­ ­ ­ ­ ­
12 112 212 312 412 512 197 × 6 – 8 = 1174 ; not 1172
1174 × 7 + 9 = 8227
9. (d) 9 49 201 1009 4041 20209 80841 8227 × 8 – 10 = 65806
E55F E555F E555F E555F E55555F E555F Clearly, 1172 is the wrong number and it should be
´5 + 4 ´4 +5 ´5 + 4 ´4 + 5 ´5+ 4 ´4 + 5
replaced by 1174.
10. (a) 31 35 44 60 85 121 22. (a) The series is based on the following pattern :
E55555F E55555
F E5555F E5555F E55555F
+ 22 +32 +42 +52 +62 16 + 12 = 17; not 19
17 + 22 = 21
y
o
u
rs
m
a
h
68 101 SPEED TEST

b
o
21 + 32 = 30

o
3. (e) The pattern of the number series is :

b
30 + 42 = 46

.w
45 + 1 × 12 = 45 + 12 = 57
46 + 52 = 71 57 + 2 × 12 = 57 + 24 = 81

o
71 + 62 = 107

rd
81 + 3 × 12 = 81 + 36 = 117
Clearly, 19 should replaced by 17.

p
117 + 4 × 12 = 117 + 48 = 165

re
23. (d) The series is based on the following pattern :
165 + 5 × 12 = 165 + 60 = 225

s
7 + 9 = 16

s
.c
9 + 16 = 25 4. (e) The pattern of the number series is :

o
16 + 25 = 41 17 + 13 = 17 + 1 = 18

m
25 + 41 = 66; 68 18 + 23 = 18 + 8 = 26
26 + 33 = 26 + 27 = 53
41 + 66 = 107 53 + 43 = 53 + 64 = 117
66 + 107 = 173
Clearly, 68 should be replaced by 66 117 + 53 = 117 + 125 = 242
24. (c) The series is based on the following pattern 5. (a) The pattern of the number series is :
4 × .5 = 2
1 1 1
2 × 1.5 = 3 ; not 3.5 + =
4 4 2
3 × 2.5 = 7.5
1 1 3
7.5 × 3.5 = 26.25 + =
26.25 × 4.5 = 118.125 2 4 4
Clearly, 3.5 should be replaced by 3. 3 1
25. (b) The series is based on the following pattern: + =1
16 × 0.25 = 4 4 4
4 × 0.50 = 2 1 1
2 × 0.75 = 1.5 1+ =1
4 4
1.5 × 1.00 = 1.5 ; not 1.75 3 1
1.5 × 1.25 = 1.875 \? = 1 + = 2
4 4
Clearly, 1.75 should be replaced by 1.5. 6. (c) The pattern of the number series is :
26. (c) The series is
× 1 + 2, × 2 + 3, × 3 + 4, × 4 + 5, × 5 + 6 4 + 1 × 15 = 19
The wrong number is 18. 19 + 2 × 15 = 49
It should be 6 × 2 + 3 = 15 49 + 3 × 15 = 94
27. (e) The series is × 1.5 94 + 4 × 15 = 154
The wrong number is 366
It should be 243 × 1.5 = 364.5 154 + 5 × 15 = 229
28. (a) The series is 7. (a) The pattern of the number series is :
× 6 + 42, × 5 + 30, × 4 + 20, × 3 + 12, × 2 + 6,
1 1
The wrong number is 3674 + =1
It should be 1220 × 3 + 12 = 3672 2 2
29. (b) The series is (2)3, (3)3, (4)3, (5)3, (6)3, (7)3, 1 1
The wrong number is 218 1+ =1
2 2
It should be (6)3 = 216
30. (d) The series is + (7)2, + (6)2, + (5)2, + (4)2, + (3)2 1 1
The wrong number is 102. 1 + =2
2 2
It should be 68 + (6)2 = 104
1 1
\ ?=3+ = 3
SPEED TEST 50 2 2
8. (d) The pattern of the number series is :
1. (b) The pattern of the number series is : 101 + 2 = 103
353 + 1 = 354
103 – 4 = 99
354 – 3 = 351
99 + 6 = 105
351 + 5 = 356
105 – 8 = 97
356 – 7 = 349
97 + 10 = 107
346 + 9 = 358
2. (c) The pattern of the number series is : 9. (b) The pattern of the number series is :
219 – 3 = 216 = 63
1 + 22 = 1 + 4 = 5 344 – 219 = 125 = 53
5 + 23 = 5 + 8 = 13 408 – 344 = 64 = 43
13 + 24 = 13 + 16 = 29 \ ? = 408 + 33
29 + 25 = 29 + 32 = 61 = 408 + 27 435
61 + 26 = 61 + 64 = 125
y
o
u
rs
m
a
h
SOLUTIONS 69

b
o
o
22. (a)
10. (a) The pattern of the number series is :

b
45

.w
7 + 3 = 10
10 + 12 (= 2 × 3) = 16 4 12 42 196 1005 6066 42511

o
rd
16 + 12 (= 2 × 6) = 28

p
28 + 24 (= 2 × 12) = 52

re
42 is written in place of 45.

s
52 + 48 (= 2 × 24 ) = 100

s
23. (a)

.c
11. (a) 1 × 1 = 1

o
10
1×2=2

m
2 8 12 20 30 42 56
2×3=6
6 × 4 = 24 +4 +6 +8 + 10 + 12 + 14
24 × 5 = 120 8 is written in place of 6.
12. (b) 7 – 8 = 1 = 13 24. (e)
16 – 8 = 8 = 23 60
43 – 16 = 27 = 33 32 16 24 65 210 945 5197.5
? – 43 = 107 = 43 × 0.5 × 1.5 × 2.5 × 3.5 × 4.5 × 5.5
232 – 107 = 125 = 53 65 is written in place of 60.
13. (b) 4 × 3 + 1 = 13 25. (d)
4 × 4 + 1 = 17
193
4 × 5 + 1 = 21 7 13 25 49 97 194 385
14. (c) 982 – 977 = 5
977 – 952 = 25 +6 + 12 + 24 + 48 + 96 + 192
952 – 827 = 125 194 is written in place of 193
827 – 822 = 5 26. (a) The series is based on the following pattern :
822 – ? = 25 16 + 12 = 17; not 19
? = 822 – 25 17 + 22 = 21
= 797 21 + 32 = 30
15. (c) 1 ×8=8 30 + 42 = 46
46 + 52 = 71
8 × 9 = 72
71 + 62 = 107
72 × 8 = 576
Clearly, 19 should replaced by 17.
576 × 9 = 5184
27. (d) The series is based on the following pattern :
5184 × 8 = 41472
7 + 9 = 16
16. (e) The series is as follows: 9 + 16 = 25
64 + 5 = 69; 16 + 25 = 41
69 + 5 = 74;
25 + 41 = 66; 68
74 + 5 = 79
41 + 66 = 107
54 – 5 = 49; 66 + 107 = 173
49 – 5 = 44 Clearly, 68 should be replaced by 66
17. (b) The series is as follows: 32 16 24 65 210 945 5197.5
÷2 +8 28. (e)
×0.5 ×1.5 ×2.5 ×3.5 ×4.5 ×5.5
Hence, ? = 1012 ÷ 2 + 8 = 514 29. (a) The series is
18. (c) The series is as follows: – 200, –100, –50, –25, – 12.5, –6.25, ......
× 1, × 3, × 5, × 7, × 9, × 11 30. (c) The series is
×1.5, × 2, ×1.5, ×2, ×1.5, ×2, .....
Hence, ? = 75 × 7 = 525
19. (a) The series is as follows: SPEED TEST 51
1 1 1
× , × 1, × 1 , × 2, × 2 , × 3 1. (d) Total value of the quantity sold for item D
2 2 2
1 40 ´ 150 12.5 ´ 90
Hence, ? = 78 × 2 = 195 = ´ ´ 100 = 60 × 11.25 × 100 = ` 67500
2 100 100
20. (a) This is a series of prime numbers : 2. (e) Average price per kg. of items A, B and C
21. (d)
(17.5 + 10 + 7.5 ) 35
155 = = = ` 11.667 » 10.50 (approx)
3 3
3601 3602 1803 604 154 36 12 3. (a) Total value of quantity sold for item E = 15 × 25 × 100 = 37500
÷1+1 ÷2+2 ÷3+3 ÷4+4 ÷5+5 ÷6+6 Total value of quantity sold for item F
154 is written in place of 155. = 10 × 35 × 100 = 35000
Required ratio = 37500 : 35000 = 15 : 14
y
o
u
rs
m
a
h
70 101 SPEED TEST

b
o
o
4. (e) Total value of the quantity sold for item C
12 ´ 2000

b
= 45 × 100 × 7.50 = 33750

.w
Total value of the quantity sold for item E 100 12
15. (d) Required ratio = = = 4 :5
15 ´ 2000 15

o
= 22.5 × 100 × 15 = 33750

rd
33750 100
´ 100 = 100%

p
\ Required percentage = 16. (e) Required difference = 680 – 258 = 422

re
33750
5. (d) Required price

s
550 - 430

s
120 17.5 ´ 120 17. (b) Required percentage = ´100 » 27%

.c
= 20 ´ 100 ´ ´ = 2400 × 21 = ` 50400 430

o
100 100

m
6. (d) Total number of students studying in college H 160 + 708 + 550 + 586
18. (b) Required average = = 501
= 51.2 + 40 + 36.5 = 127.7 thousand 4
Total number of students studying in college K. 19. (a) Number of flights cancelled by airline R in 2010 due to
= 30 + 56 + 25 = 111 thousand
Required difference = (127.7 – 111) thousand 880 ´ 60
technical fault = = 528
= 16.7 thousand = 16700 100
7. (b) Total number of students studying in all the colleges together
( 600 + 546)
= [(51.2 + 40 + 36.5) + ( 65 + 50 + 33) 20. (e) Required percentage = ´ 100
+ (44 + 30 + 60) + 30 + 56 + 25)] thousand 365
(127.7 + 148 + 134 + 111) = 520.7 thousand = 520700 1146
8. (c) Number of students from the faculty of science from college = ´ 100 = 314 (approx.)
H and I together = 40 + 50 = 90 365
Number of students from the faculty of science from college
J and K together = 30 + 56 = 86 SPEED TEST 52
90 45
Required ratio = = = 45 : 43 1. (a) 48 % of 525 + ? % of 350 = 399
86 43
9. (a) Number of students from the faculty of science from college 48 ?
I = 50 Þ ´ 525 + ´ 350 = 399
Total number of students from college I = 65 + 50 + 33 = 148 100 100
50
Þ 25200 + ? × 350 = 399 × 100
Required percentage = ´ 100 = 33.78% » 34% (approx.) Þ ? × 350 = 39900 – 25200 = 14700
148
10. (e) Average number of students from the faculty of commerce 14700
from all the colleges together. Þ ?= = 42
350
(36.5 + 33 + 60 + 25 )
= thousand 3 4 5
4 2. (b) ?= of of of 490
154.4 7 5 8
= thousand = 38.625 thousand = 38625
4 3 4 5
11. (a) Required percentage Þ ? = × × × 490
7 5 8
æ 5 ö 14 2 14
2000 ´ ç 1 - ÷ ´ 2000 ´ ´ Þ ? = 35 × 3 = 105
è 7 ø 100
´ 100 = 7 100 ´ 100
=
2000 ´
7 140 3. (d) ? + 17 2 = 335
100
Þ ? + 289 = 335
80
= ´ 100 » 57% Þ
140 ? = 335 - 289 = 46
12. (d) Total number of teachers who teach English and History Þ ? = 46 × 46 = 2116
( 7 + 27 ) 4. (a) ? = 125% of 560 +22% of 450
together = 2000 ´ = 680
100 125 22
Þ ?= ´ 560 + ´ 450
Total number of teachers who teach Mathematics and Biology 100 100
(14 + 12 ) 70000 9900
together = 2000 ´ = 520 Þ ?= +
100 100 100
Required difference = 680 – 520 = 160 Þ ? = 700 + 99 = 799
13. (e) Total number of Biology and History teachers
12 100 + 40 ö æ 27 100 - 20 ö 28 ´ 5 - 15 ´ 6
æ 5. (c) ?=
= ç 2000 ´ ´ ÷ + ç 2000 ´ ´ ÷ 7 + 256 + (13) 2
2
è 100 100 ø è 100 100 ø
æ 12 140 ö æ 27 80 ö 140 - 90
= ç 2000 ´ ´ ÷ + ç 2000 ´ ´ ÷ Þ ?=
è 100 100 ø è 100 100 ø 49 + 16 + 169
= 336 + 432 = 768
14. (b) Required average 50 25
Þ ?= =
2000 ´ ( 25 + 27 + 12 )
234 117
1280 6. (b) ? = 18.76 + 222.24 + 3242.15
100 » 420
= = Þ ? = 3483.15
3 3
y
o
u
rs
m
a
h
SOLUTIONS 71

b
o
? = 784 ¸ 16 ¸ 7

o
7. (d) 15. (b) Suppose 16 men can complete the same work in x days

b
Then, Men days

.w
784
Þ ?= ¸7 10 8

o
16

rd
Þ ? = 49 ¸ 7 = 7 16 x

p
16 : 10 : : 8 : x

re
3 5 Þ 16 × x = 10 × 8

s
8. (c) ?= of 455 + of 456

s
.c
7 8 10 ´ 8
Þ

o
x= = 5 days

m
3 5 16
Þ ? = × 455 + × 456
7 8 16. (c)
Þ ? = 195 + 285
x
Þ ? = 480 17. (a) Let the original fraction be = .
9. (b) ? = 1.05% of 2500 + 2.5% of 440 y
1.05 2.5 x ´ 200 4 x 4 3 2
Þ ?= ´ 2500 + ´ 440 \ = Þ = ´ =
100 100 y ´ 300 21 y 21 2 7
2625 1100 18. (e) The word SIMPLE consists of 6 distinct letters
Þ ?= +
100 100 \ Number of arrangements = 6!
3725 = 6 × 5 × 4 × 3 × 2 × 1 = 720
Þ ?= = 37.25 19. (d) Let the present age of A = x and B = y years
100 According to first condition
10. (b) ? = 4900 ¸ 28 × 444 ¸ 12
Þ ? = 175 × 37 x -7 3
= Þ 4 x - 28 = 3 y - 21Þ 4 x - 3 y = 7 ........ (i)
Þ ? = 6475 y -7 4
11. (b) Q Cost price of (12 belts + 30 wallers) = ` 8940 According to second condition
Q Cost price of 3 × (4 belts + 10 wallets) = ` 8940 x+9 7
8940 = Þ 8x + 72 = 7 y + 63
y+9 8
Q Cost price of 4 belts + 10 wallets = = ` 2980
3 Þ 7y – 8x = 9 ......... (ii)
12. (c) Q Cost price of an article = ` 1850
8 x - 6 y =14
For 30% profit, selling price of this article
7 y - 8x =9
130
= 1850 ´= ` 2405 y = 23 years.
100
20. (e) x + (x + 1) + (x + 2) = 1383
13. (e) Compound Interest after two years Þ 3x + 3 = 1383 Þ 3x = 1380
2 1380
æ 10 ö Þ x= = 460
= 8500 ç1 + - 8500
è 100 ÷ø 3
Largest number = x + 2 = 462
11 11 21. (b) 8 × 6.5 = 52
= 8500 ´ ´ - 8500
10 10 52 × 5.5 = 286
= 10285 – 8500 = ` 1785
286 × 4.5 = 1287.
14. (a) Let length of the train be x m 22. (d) 3 × 14 = 42
5 50 42 × 12 = 504
Speed of the train be 60 km/h = 60 ´ = m/s
18 3 504 × 10 = 5040
x + 200 5040 × 8 = 40320.
Then, = 27
50 23. (c) 403 – 3 = 400
3 400 – 6 = 394
394 – 12 = 382
3( x + 200) 382 – 24 = 358
Þ = 27
50 358 – 48 = 310
Þ 3x + 600 = 1350 310 – 96 = 214 .
Þ 3x = 1350 – 600
Þ 3x = 750 +5 –9

750 24. (d) 7 8 4 13 –3 22 –14


Þ x= = 250 m
3 –3 –7 –11
y
o
u
rs
m
a
h
72 101 SPEED TEST

b
o
o
25. (a) 250000 ¸ 4 = 62500 148

b
62500 ¸ 5 = 12500 or, x = = 37

.w
12500 ¸ 4 = 3125 4

o
\ Numbers are A = 37, B = 39, C = 41, D = 43

rd
3125 ¸ 5 = 625
\ Product of B and D = 39 × 43 = 1677.

p
625 ¸ 4 = 156.25

re
33. (a) 2.31 km = 2.31 × 1000 = 2310 m

s
156.25 ¸ 5 = 31.25. Total number of days = 3 × 7 = 21

s
.c
7 2310

o
26. (b) Largest fraction = \ Distance covered by Anu each day = = 110

m
8 21
m.
1
Smallest fraction = 5
2 34. (b) 43.2 m/hr = 43.2 ´ = 12 m/s
18
7 1 7-4 3
Difference = - = = . Total distance covered = 12 × 80 = 960 m.
8 2 8 8 Perimeter of the square = 960 m.
27. (e) (?)0.6 × (?)1.4 = 26 × 104 Side of the square = 240 m.
or, ?2 = 2704 Area = (240)2 = 57600 sqm.
or, ? = ±52. 35. (b) Let the number of children be x.
28. (a) Perimeter of the square = 84 cm Now, according to the question
Perimeter of the rectangle = 28 cm
Perimeter of the rectangle = 2(1 + b) æ 4800 ö
çè - 100÷ (x + 4) = 4800
or, 2(8 + b) = 28 cm x ø
or, b = 14 – 8 = 6 cm.
\ Breadth of the rectangle = 6 cm æ 48 ö
or, çè - 1÷ø (x + 4) = 48
x
84
Side of the square = = 21 cm or, (48 – x)(x + 4) = 48x
4
or, x2 + 4x – 192 = 0
Difference = 21 – 6 =15 cm. or, (x + 16)(x – 12) = 0
29. (e) Perimeter of the rectangle = 42 m \ x = 12 sweets
2(l + b) = 42 m
or, l + 8.5 = 21m 4800
Number of students = = 400 .
or, l = 12.5 m. 12
Area of the rectangle = 12.5 × 8.5 = 106.25 sq.m.
342000
\ Area of the circle. = 106.25 sq.m. 36. (c) Sneha's monthly income = = 28500
30. (d) Let the positive number be x. 12
5x 3x 28500
Then, ´ = 504.6 \ Akruti's monthly income = ´ 116 = 34800
100 100 95
Akruti's annual income = 417600.
5 3
\x ´ ´x´ = 504.6 256
100 100 37. (b) Time taken by the truck = = 8 hr.
32
2 504.6 ´ 100 ´ 100 Distance covered by the car = (256 + 160) = 416 km.
or, x =
15 Time = 8 hr.
\ x = 580.
416
31. (b) Two women alone can complete a piece of work in 16 \ Speed of the car = = 52 km / hr.
days. 8
\ Four women can complete the same work in 8 days. 663 - 612
Since 12 children can complete the work in 38. (a) Required percentage = ´ 100 = 5% .
1020
4 ´8 4 ´8 39. (c) Sum of the heights of all the girls = 148 × 21 = 3108 cm
= = 8 days.
8-4 4 Sum of the heights of the teacher and all the girls = 149
× 22 = 3278 cm.
12 ´ 8 Teacher's height = 3278 – 3108 = 170 cm.
\ Four children can complete the work in = 24
4 40. (a) Radius = 17.5 cm.
days.
22
32. (d) Let the four odd consecutive numbers be x, x + 2, x + 4 Area of the circle = ´ 17.5 ´ 17.5 = 962.5 sq cm.
and x + 6. 7
Also, A = x, B = x + 2, C = x + 4 and D = x + 6.
\ 4x + 12 = 4 × 40
or, 4x = 160 – 12 = 148
y
o
u
rs
m
a
h
SOLUTIONS 73

b
o
o
SPEED TEST 53 9. (e) 10. (c) 11. (b) 12. (b)

b
.w
13. (a) 14. (d) 15. (d)
1. (a)

o
16. (c) The author wants us to stop debating and implement

rd
2. (c) They are responsible for national disintegration. policies.

p
3. (d) They are harmful to national integrity. 17. (c) Stated in the first paragraph.

re
4. (b) India was forged into a nation on account of a common 18. (d) Uncertainty about payment is mentioned, hence option

s
s
culture evolved over the centuries. (d).

.c
5. (a) The author wants India to remain as an ideal nation

o
19. (b) Refer to the second paragraph. “State governments

m
and the passage has certainly a message behind it. have not implemented agr eed plans to ensure
6. (e) 7. (b) 8. (a) repayment when due...”
9. (e) India’s insurgence stood for gaining freedom by 20. (a) All the factors are mentioned in the passage.
adopting the path of non-violent struggle. 21. (a) Refer to the third paragraph. “The Delhi model has
10. (c) 11. (d) 12. (e) worked. But it receives no public support.”
13. (a) 14. (e) 22. (d) Clearly, populist measures would go against financial
15. (b) wellbeing.
16. (a) Option (c) can be rejected as it is out of the context. 23. (d) It is stated in the passage that the enforcement of the
Option (d) is also not true as they like to invest abroad reforms was inadequate.
than in India. Option (e) is true in parts. Only option 24. (c) Eminent British economists and political scientists have
which is in sync with major portion of passage is (a). strongly attacked the tradition of budget secrecy.
17. (a) Other options are not mentioned in the passage. 25. (e) It leads to the control of public expenditure in order to
18. (c) Last part of the passage chiefly describes it. Other set realistic taxation implications.
options show these sectors in positive light which is 26. (b) He has presented the example of both the open budget
against passage content. system and the secret budget system, practised by
19. (b) Option (B) is true in its completeness, encompassing various countries and has looked into all their aspects.
the whole of the passage. But other options are true in 27. (d) 28. (e)
parts only. Option (E) is false in parts. 29. (a) Sir Richard Clarke was the originating genius of nearly
20. (c) Both (1) and (3) are directly mentioned in passage. every important development in the British budgeting
Statement 2 is just not true. techniques during the last two decades.
21. (c) Meaning of Clout is force. 30. (b)
22. (e) Marked means distinguished and different.
Imperceptible is that cannot be distinguished or SPEED TEST 55
percepted.
23. (d) Option (D) can be related to the experts as mentioned 1. (b) 2. (e) 3. (c) 4. (c)
in the passage. 5. (b) 6. (a) 7. (b) 8. (d)
24. (d) Option (D) has plenty of illustrations, references and 9. (d) 10. (d) 11. (e) 12. (c)
structure to support it in the passage. Other options 13. (c) 14. (a) 15. (b) 16. (c)
like E and B are true but not the central theme of the 17. (d) 18. (b) 19. (e) 20. (a)
passage. Option (A) and (C) are not true. 21. (d) 22. (e) 23. (e) 24. (b)
25. (d) PROMOTES means to advocate a particular cause. 25. (a) 26. (b) 27. (d) 28. (b)
26. (e) A rise from the 4% level to 7% says that there is rise in 29. (b) 30. (a)
education in Egypt.
27. (a) All of the options are mentioned in one or other part of SPEED TEST 56
the passage.
28. (c) First line of the passage is self explanatory. 1. (b) The word ' vicarious' means 'Endured or done by one
29. (b) Only option which can be linked with the passage is 1.
person substituting for another' which is nearest in
Their people will agitate for greater political freedom,
meaning to phrase 'Not experienced personally' which
culminating in a shift to a more democratic form of
is option (b), therefore, (b) is the correct answer. The
government.
other 3 words have following meanings: Ambitious -
30. (c) Option A is wrong as this is not intended from the use
strongly desirous; Nostalgic - homesickness; Vindictive
of this phrase. Last part of passage has this phrase
- vengeful.
and poor-rich divide has been discussed their in voting
2. (b) The word 'craven' means 'cowardly, contemptibly timid
pattern or why rich people even educated do not vote.
pusillaninuous which is nearest in meaning to cowardly'
Option (D) is out of the context. Option E can also be
which is option (b), therefore, (b) is the correct answers.
rejected as corruption is not the issue here.
3. (d) The word ' Tepid' means moderately warm; Luke warm
which is nearest in meaning to 'Lukewarm' which is option
SPEED TEST 54
(d), therefore, (d) is the correct answer. The other 3 words
have following meanings: Irreversible - uncapable of
1. (b) 2. (b) 3. (d) 4. (d) being changed; Causing fatigue - something that leads
5. (c) 6. (c) 7. (e) 8. (d) to mental exertion; Fast moving - speedy.
y
o
u
rs
m
a
h
74 101 SPEED TEST

b
o
o
4. (c) The word 'Tenuous' means thin or slender in form which 20. (b) Thw word ‘exaggerate’ means amplify, inflate, elaborate

b
is nearest in meaning to 'slender' which is option (c), etc. which is similar in meaning to ‘overstate’.

.w
therefore, (c) is the correct answer. The other three words 21. (d) The word ‘evident’ means obvious, clear, tangible,

o
rd
have followings meanings: Contentious - quarrelsome; distinct etc. which is nearest in meaning to ‘quite clear’.
Dark - little as no light; Malfunctioning - to fail to

p
22. (b) The word ‘penalize’ means inflict a penalty on, which

re
function.

s
5. (a) The word 'probity' means integrity or uprignt ness, is quite similar in meaning to ‘punish’.

s
.c
honesty' which is nearest in meaning to integrity which 23. (c) The word ‘remedial’ means aimed at solving a problem

o
is option (a), therefore, (a) is the correct answer. The

m
which is similar in meaning to ‘corrective’.
other 3 words have following meanings: Impudence - 24. (a) The word ‘trivial’ means not worth considering,
in solence; Profane - irreligious; Preface - an
introductory past. insignificant etc., which is similar in meaning to
6. (b) The word 'Musty' means absolute, out dated or stale ‘unimportant’.
food' which is nearest in meaning to 'stale' which is 25. (a) The word ‘ Incredible’ means beyond belief,
option (b), therefore, (b) is the correct answer. The other unbelievable, unimaginable etc. which is similar in
3 words have following meanings: Certainty - something
meaning to ‘hard to believe’.
certain; Modern - not ancient or remote; Mysterious -
implying or suggesting a mystery. 26. (d) The word ‘frenetic’ means ‘wildly excited or active’,
7. (b) The word 'Alleviate' means 'to diminish or lessen ' which energetic, hectic, fast and furious etc.
is nearest in meaning to 'lessen' which is option (b), 27. (c) The word ‘elicit’ means draw out, extract, obtain,
therefore, (b) is the correct answer. The other three provoke etc.
words have following meanings: To release - to free
from confinement; To deprive - to remove from the 28. (b) The word ‘Lucrative’ means gainful, remunerative, well-
possession; To deceive - delude, to be unfaithful to. paid which is most similar in meaning to ‘profitable’.
8. (c) Morose means ‘depressed and pessimistic’. Some more 29. (c) Vivid means presented in very clear/distinct way. From
synonyms are : Cantankerous, gloomy, in a bad mood, the options detailed is the other option which can
mournful, moody, splenetic.
replace but only if 'clear' as option is not available.
9. (c) Protagonist ‘means person who takes the lead or the
central figure of the narrative’. Some more synonyms Also 'detailed' can be confusing as well. So it is not the
are: Central character, Prime mover, Exponent, Hero, best choice. Ambiguous is opposite for the Vivid.
Exemplar, Mainstay. 'Categorical' is without any condition.
10. (c) Factitous means ‘unnatural, false or artificial’ so in the 30. (d) Deplore means to disapprove /to condemn /to regret.
given option artificial would be the right synonym of
Deprive means lack of something. Implored is to beg
factitious. Other synonyms : false, artificial, sham.
11. (d) The word ‘hospitable’ means genial, welcoming, impatiently. Deny is to refuse. So clear choice is
congenial, friendly, convivial, but welcoming is the most 'regretted' as this is direct meaning of the word.
suitable word. 31. (c) Vindictive is revengeful. Other options like violent and
12. (a) The word ‘scarcely’ means hardly, barely, almost not, cruel can be actions of revenge but not the right answer
only just. as synonym. Irritable is not correct clearly.
13. (c) The word ‘disdain’ means contempt, scorn, disrespect, 32. (a) Pragmatic is practical. Pragmatic is replacing 'practical'
dislike. From the given options ‘hate’ is the most suitable in English usage for common use.
word. 33. (d) Soporific means sleep inducing. It can not be replaced
14. (a) The word ‘absurd’ means stupid not logical and with other options. There is not much use of context in
sensible which is nearest in meaning to senseless. these types of words which have direct meaning with
15. (a) The word ‘philanthropy’ means the practice of helping so clear options.
the poor and those in need especially by giving money. 34. (b) Evoke means to 'call for' or 'to rise by a reason'. From
Hence generosity (willing to give somebody money, the options, (b) is the direct meaning of the word.
gifts etc) is nearest in meaning to it. 35. (a) Abate is to reduce but not to completely remove. So
16. (a) The word ‘mutual’ is used to describe feelings that two Vanished as answer is rejected. Increased is just the
or more people have for each other equally. Hence, opposite in sense. Stabilized is to remain at same status
reciprocal is similar word in meaning to it. or as it is.
17. (d) The word ‘weird’ means very strange or unusual and 36. (c) Insulting behavior is insolent behavior. Violent behavior
difficult to explain which is similar in meaning to can not be counted as disrespectful it is extreme
unnatural. one.Yes; disrespect can be a reason for violent behavior.
18. (d) The word ‘pessimistic’ means bleak, distrustful, 37. (d) Mendacious is intentionally untrue or false. Options
hopeless, depressed etc. which is nearest in meaning are not confusing either.
to ‘not hopeful’. 38. (c) Induces means persuades, insist and provoke.
19. (c) The word ‘analogous’ means like, equivalent, related 39. (d) Authentic means genuine, real, valid and dependable.
etc. which is nearest in meaning to ‘similar’. 40. (c) Scarcity means insufficiency, shortage, lack, dearth or
paucity.
y
o
u
rs
m
a
h
SOLUTIONS 75

b
o
o
SPEED TEST 57 28. (a) Enrich is making richer /more valuable/more

b
meaningful/more nourished. The opposite of this word

.w
1. (a) 2. (e) 3. (d) 4. (e) is to make poor - impoverished.

o
rd
5. (c) 29. (a) Credible is loyal or believable or something in which
faith can be reposed. Incredible is just opposite of the

p
6. (b) The word ‘obligatory’ means compulsory, mandatory,

re
required whose opposite is optional or voluntary. word. Although from the options Believable is close

s
but not the best one to choose. If credible is not in the

s
7. (d) The word ‘obscure’ means unclear whose opposite is

.c
clear. options then this is the best among the rest.

o
30. (a) Inert is without motion. From the options active is clear

m
8. (a) The word ‘repulsive’ means repellent, nasty whose
opposite is attractive. choice. As Lazy is similar in meaning.While Resolute
9. (a) The word ‘vital’ means essential, urgent while trivial (Firm) is irrelevant in context. Strong does not mean
means insignificant. activeness or motion. He has very strong emotions for
10. (d) The word ‘inhibit’ means hinder, hold back whose her.
opposite word is encourage or promote. 31. (a) Affluent is extra rich. If richness is a scale then Affluent
11. (c) The word ‘exploit’ means treat unfairly, utilize, misuse is on one end and poor is on the other. Ordinary lies in
or take advantage of which is opposite in meaning to between the two. Backwardness is just the result of
support. poorness. So 'poor' is the best option.
12. (b) The word ‘sharp’ means razor-edged, fierce, shrill etc. 32. (d) Energetic is full of activity and lethargic means
which is opposite in meaning to blunt. inactivity/slowness/ laziness/dull. Gloomy is sadness.
13. (d) The word ‘condemn’ means censure, criticize, blame 33. (b) Here secure means to make safe and precarious is having
etc. which is opposite in meaning to praise. a sense of insecurity. So it can be a proper antonym for
14. (d) The word ‘reluctant’ means timid, resistant, opposed the word from the available options.
etc. which is opposite in meaning to eager. 34. (d) 'Various' word derives from the variety. Variety is range
15. (a) The word ‘scarcity’ means shortage, lack, paucity, of choices and similarity is opposite of it.
meagreness, dearth etc. Its opposite word is ‘plenty’ in 35. (b) Recession means depression, slump, downturn,
the given options. collapse, decline so its antonym can be inflation, boom,
16. (a) The word ‘bleak’ means bare, exposed, denuded, dim rise increase.
etc which is quite opposite in meaning to ‘bright’. 36. (d) Denied means deprive, starve, unused, shorn of so its
17. (d) The word ‘stern’ means serious, austere, unforgiving, antonym can be affirmed, avowed, confirmed, declared,
harsh etc. Its opposite word will be ‘forgiving’. stated.
18. (a) The word ‘superficial’ means shallow, casual, hasty, 37. (c) Collective means together, mutually, group, joint or
trivial, silly, inane etc. which is opposite in meaning to cooperative so its antonym can be individual, single or
‘profound’. alone.
19. (a) The word ‘elegance’ means grace, stylishness, charm, 38. (b) Abducted means kidnap, snatch or seize so its antonym
cleverness which is opposite in meaning to ‘balance’. can be set free, release or freedom.
20. (d) The word ‘coarse’ means ugly, rough, ill-mannered, 39. (c) Enhanced means increase, augment, boost or amplify
rude, crude etc. which is opposite in meaning to ‘soft’ so its antonym can be decreased, reduce or lessen.
from the given options. 40. (b) Condemn. Adore means esteem, respect and admire so
21. (c) The word ‘extravagant’ means prodigal, lavish, costly its antonym can be disapprove, criticize, revile, attack
etc. which is opposite in meaning to the word or condemn.
‘economical’.
22. (b) The word ‘diligent’ means industrious, hard-working, SPEED TEST 58
rigorous etc. which is opposite in meaning to the word
‘lazy’. 1. (c) shown
23. (c) The word ‘fictitious’ means fake, spurious etc. Its 2. (d) drawn to
opposite word will be genuine or real. Certain Verbs, Nouns, Adjectives, and Participles are
24. (b) The word ‘generous’ means liberal, charitable, always followed by certain Prepositions.
benevolent etc. Hence, uncharitable is the opposite 3. (d) were
word. The Past Subjunctive ‘were ‘is used after tile verb wish,
25. (b) Ignored is to overlook /to avoid /not considered/set to indicate a situation, which is contrary to fact or unreal;
aside. If something is ignored then it can be removed. as in, I wish I were a millionaire.
So 'remove' gives the same sense as 'ignored'. 4. (c) had seen me
26. (c) Lethargic is slow/lazy/inactive/dull. Active is antonym. 5. (c) concur with
Aggressive is second best option. Hungry is to make a 6. (b) swum
guessing student go on sense of the sentence and The Past Participle “swum” is to be used and not the
deviate from the right answer. past tense “swam”.
27. (d) Hazardous is harmful/perilous/marked by danger/risky; The Past Participle represents a completed action or state
opposite for it is 'safe'. Other options are not confusing of the thing spoken of.
so it is easy to answer. 7. (b) The sentence gives a condition, so the principle clause
will use ‘would’ not ‘will’.
y
o
u
rs
m
a
h
76 101 SPEED TEST

b
o
o
8. (b) Since there is a comparison between two, a comparative 28. (b) would not have been misunderstood.

b
degree verb must be used. 29. (d) reformed.

.w
9. (a) Gravity is the singular subject which will have the present 30. (c) When he ran across the road

o
The past tense is needed here as the sentence indicates.

rd
indefinite verb i.e., verb + s/es form.
10. (b) Since a comparison is being made there must be a subject

p
re
in both parts of sentence devided by than. There must SPEED TEST 59

s
be ‘that’ in the second part therefore, (d) is incorrect

s
.c
because of incorrect usage of article ‘the’. 1. (c) 2. (a) 3. (e) 4. (e) 5. (c)

o
11. (b) There is no need for ‘who’ or ‘which’ (both of which are 6. (b) 7. (c) 8. (e) 9. (d) 10. (e)

m
incorrect ‘whom’ will be the right pronoun) the clause 11. (c) 12. (b) 13. (b) 14. (d) 15. (b)
follows the subject ‘person’ directly and need not be 16. (c) 17. (b) 18. (c) 19. (e) 20. (c)
connected through a pronoun. 21. (b) 22. (b) 23. (e) 24. (c) 25. (c)
12. (d) It is a passive voice sentence and the phrase before 26. (c) 27. (d) 28. (e) 29. (c) 30. (e)
‘that’ should be a complete passive form of verb and not
31. (d) 32. (c) 33. (e) 34. (b) 35. (d)
a gerund.
13. (d) ‘of’ is the preposition used with frightened.
14. (c) Capitalist society is the singular subject and the statement SPEED TEST 60
made is a dictum, so pesent infinitive tense will be used.
1. (d) 2. (c) 3. (d) 4. (a)
15. (a) The use of the word here is as a ‘noun’ not as a verb thus
5. (a) 6. (c) 7. (c) 8. (e)
damage will be the right answer.
9. (a) 10. (a) 11. (b) 12. (e)
Tip : A sentence of form this can cause, will always be
13. (d) 14. (d) 15. (e) 16. (e)
followed by a noun.
17. (a) 18. (c) 19. (b) 20. (e)
16. (d) The right expression is ‘conditions necessary for’, since
21. (b) Engaged is the word which fits in both the sentences.
the gap is followed by, this. ‘complete’ should be used in
In the first sentence it means that couple has made on
the noun form and not adjective so it will be ‘completion agreement or a pledge to get married and in statement
of this’. II it means occupy the attention or efforts of (a person
17. (b) The right use is as + adjective + as or persons).
18. (a) There cannot be a subject in the first part of the sentence 22. (b) Option (b) is correct. Application in the first statement
as the same subject is given on the second part so (c) means a written request for employment. In the II
and (d) are eliminated. To study is the infinitive verb statement application means the act of putting to a
which will not be used here, because it suggests the special use or means the act of putting to a special use
action which is the affect and not the cause. e.g. ‘To gain or purpose.
something you have to lose something else’. 23. (e) hearing
By studying is the right answer because this gives the Statement I - an instance or a session in which testimony
cause for the verb in the latter part of the sentence. and arguments are presented, especially before an
19. (b) In the given sentence there is no subject or verb, so the official, as a judge in a lawsuit.
added phrase should be of the form subject + infinitive Statement II - the act of perceiving sound.
verb. 24. (a) resort
20. (c) Modals such as must or should cannot be used with Statement I - to have recourse for use help, or
‘hope’. accomplishing something of ten as a final available
21. (a) There must be the preposition ‘to’ to connect the verb option or resource.
‘tend’ with the noun ‘result’, (b) is not correct because Statement II - a place to which people frequently or
first form of verb should be used and not continuous generally go for relaxation or pleasure.
since its a simple statement made suggesting no action 25. (d) Statement I - salary
taking place at the current moment. Statement II - to carry on (a bottle, war, conflict,
22. (b) The subject of the sentence is candidate, but in the first argument, etc).
part there is no subject, it should be passive or have an
active subject. (b) is the only option with passive verb. SPEED TEST 61
23. (d) ‘because’ cannot be used as the conjunction in this
sentence because for the sentence to be complete it 1. (b) up
should be followed by an ‘of’, i.e., because of their, since 2. (d) up with
this is not the option ‘of their’ is the most appropriate 3. (b) ‘with
use. 4. (b) of
24. (c) had better see 5. (c) on
25. (c) had known 6. (c) in———with
26. (c) than one’s speed. 7. (c) on for
Here ‘than’, used as a preposition, as; 8. (b) within——to
I need more than fifty rupees for this magazine. 9. (c) for
27. (b) which 10. (a) from
11. (c) for
y
o
u
rs
m
a
h
SOLUTIONS 77

b
o
o
12. (b) in ——— around 18. (a) Replace leisure by the adverbial word leisurely

b
13. (a) to 19. (b) The phrase ‘gift of the gab’ means ‘to have a talent for

.w
14. (c) off speaking’.

o
rd
15. (d) Though —— in 20. (c) The phrase ‘the order of the day’ means common,
16. (d) to

p
popular or suitable at a particular time. For example :

re
17. (c) Although — from Pessimism seems to be the order of the day.

s
18. (d) among

s
21. (d) started carrying out their plan.

.c
19. (b) in 22. (a) started the discussion.

o
20. (d) in

m
23. (c) A succession of unexpected events.
21. (b) towards 24. (c) be implemented next month
22. (c) to 25. (b) Idiom put your foot down means : to be very strict in
23. (c) concur with
opposing what somebody wishes to do.
24. (d) of
26. (d) In most probability
25. (b) on
26. (c) with 27. (a) ignorant about
27. (d) to 28. (a) in communication with
28. (c) upon 29. (d) discovered by chance
29. (d) of 30. (b) made a record.
30. (a) at——in——at
In is used with names of countries and large towns, at is SPEED TEST 63
more often used when speaking of small towns and
villages. 1. (b) ‘Sent’ is the third form of verb ‘send’ in passive voice.
31. (b) In the perfect continuous tense only ‘for’ and ‘since’ 2. (e)
are used. ‘since’ is used to suggest a particular time 3. (a) Replace ‘though’ by ‘As’ to express cause and effect
in past and ‘for’ is used to suggest a time period, ‘a in the sentence.
long time’ suggest a time period so ‘for’ will be the 4. (d) Use ‘and I answered’ properly to express cause and
answer. effect in the sentence.
32. (b) ‘With’ is used with overwhelmed 5. (c) Apply the adverb ‘highly’ before the adjective ‘paid’.
33. (b) Preposition ‘about’ is used before the word 6. (a) Replace adverb ‘necessarily’ by adjective ‘necessary’.
‘prohibition’. 7. (e)
34. (b) This will be the right preposition. 8. (c) Use conjunction ‘and/therefore’ in place of ‘yet’.
35. (b) The right usage is ‘comes to mind’. 9. (b) ‘Any’ shows uncertainty of selection. So, use ‘one’
instead of ‘any’.
SPEED TEST 62 10. (d) Use third form of verb ‘signed’ in passive voice.
11. (a) Here, Nuclear waste will still remain/be .... should be
1. (a)
used.
2. (b)
12. (c) Here, whose attic had been should be used. Whose is
3. (a)
used to say which person or thing you mean.
4. (b)
Look at the sentence :
5. (d)
6. (d) It is the house whose door is painted red.
7. (c) 13. (c) Here, subject i.e. A public safety advertising campaign
8. (a) is singular. Hence, hopes to draw attention ... should
9. (e) be used here.
10. (c) 14. (b) Look at the structure of the sentence in Passive Voice
11. (e) Here, too is used as emphatic word. Lata was so scared of Past Simple.
that she could not go home alone. Hence, no correction Subject + was/were + V3 (Past Participle)
is required. Hence, awarded to the most .... should be used.
12. (b) The structure of sentence is subject + was/were + third 15. (b) Here, a time of reawakening .... should be used.
form of verb + object. Thus, Riya was dressed to kill. 16. (c) Here, and guidance to the mind tortured by doubt
13. (c) The given sentence is the statement of simple past should be used.
tense. Hence it should be ‘worried’ instead of worries. Look at the sentence:
14. (a) The phrase ‘let off’ means to give them only a light All activities take place under the guidance of an
punishment. experienced tutor.
15. (d) It should be ‘took’ instead of ‘take’. 17. (d) Here, Past Simple i.e. the educated class did not support
16. (d) Replace to by ‘too’ to make a correct phrase ‘a bit too him .... should be used as the sentence shows past
fast’ which means slightly or to a small extent. time.
17. (e) The word think about refers to consider. Hence no 18. (a) Here, Depletion (Noun) of the Ozone layer .... should
correction is required. be used.
y
o
u
rs
m
a
h
78 101 SPEED TEST

b
o
o
19. (a) Here, Most of the people who should be used. Who is 19. (d) : Replace 'for' by 'of

b
used to show which person or people you mean. 20. (e) : no error

.w
Look at the sentence: 21. (c) : Replace 'enough' by 'any'

o
rd
The people who called yesterday want to buy the 22. (c) : Remove 'not'

p
house. 23. (a) : Add 'having' after 'after'

re
20. (b) Here, in reducing human suffering .... should be used. 24. (c) : Replace 'was able to ' by 'could'

s
s
21. (d) Since the sentence begins in past tense. It should end 25. (d) : Replace 'for' by 'to'

.c
in past tense also since it the subject is singular the 26. (a) : Replace 'had' by 'would have'

o
m
verb will also be singular. Hence “them is missing” 27. (c) : Replace 'pumping' by 'pump'
should be “them was missing”. 28. (a) : Replace 'ential' by 'entails'
22. (a) The teacher that is the subject is singular so “were” 29. (b) : Replace 'sang' by 'sung'
will be replaced with “was”. 30. (b) : Replace 'at' by 'with'
23. (b) “Son for help her” should be “son to help her”. 31. (c) : Replace 'would have' by 'had'
24. (b) The verb “ask” will be in the past tense it will become 32. (c) : Replace 'those' by 'which'
“asked”. 33. (a) : Replace 'for' by 'two'
25. (c) “Saving” will be replaced with past tense of the verb 34. (d) : Add 'they' before 'assume'
“Save” that is saved because it is preceded by had. 35. (b) : Add 'which are' before 'available'
26. (a) Change ‘decline of’ to ‘decline in’.
36. (a) : Replace 'being intelligent' by 'intelligence'
27. (b) Delete ‘for’ after order.
37. (a) : The correct form is 'The judges not onlyacquitted
28. (c) Place on before ‘land’.
38. (e) : No error
29. (c) Delete ‘to’ before Chennai.
39. (b) : The correct form is ' different people diferentjobs .'
30. (d) Replace ‘for’ by ‘to’.
40. (d) : Replace 'promising' by 'promissed'. 366 (c): Replace
31. (c) Change ‘look for’ by ‘look after’.
32. (a) Delete for after await. 'done' by 'made'
33. (c) debarred from sending is correct.
34. (a) Do not use of with despite. Despite means inspite of. SPEED TEST 65
35. (a) Change it to ‘yielded to’
36. (b) dying of hunger is correct. 1. (a) 2. (d) 3. (b) 4. (c) 5. (d)
37. (c) Change than by ‘to’. 6. (c) 7. (d) 8. (a) 9. (d) 10. (b)
38. (d) Change ‘by’ to ‘for’. 11. (d) 12. (c) 13. (c) 14. (d) 15. (b)
39. (b) Change ‘to stay in’ by ‘rather than stay in’. She prefers 16. (a) 17. (d) 18. (b) 19. (c) 20. (c)
to write rather than to speak on telephone.
21. (e)
40. (c) Change ‘startled by’ to ‘startled at’.
22. (d) Use ‘divided’ instead of ‘dividend’.
41. (b) Change ‘with’ to ‘from’.
23. (a) Use ‘agreed’ properly for ‘consented’.
SPEED TEST 64 24. (b) Use ‘file’ instead of ‘fill’.
25. (c) Apply ‘funds’ for ‘investment’.
1. (d) Replace ‘their’ by ‘its’. The banker’s association – 26. (e)
collective noun – so pronoun ‘its’ singular. 27. (b) Use ‘able’ instead of ‘enable’.
2. (b) Replace ‘are’ by ‘is’. ‘five quintals’ refers a definite 28. (d) Use ‘overcome’ instead of ‘resort’.
quantity (as collective noun) so verb will be singular. 29. (b) Use ‘spread’ instead of ‘spend’.
3. (a) Replace ‘have’ by ‘has’. Dickens is the name of a person.
30. (a) Use ‘spent’ properly for ‘initiated’.
4. (d) Insert ‘are’ after ‘hopes’. Hopes is plural, so verb will
be plural. 31. (c) The correct spelling is souvenirs which mean memento,
5. (a) Place be after may. reminder, memorial etc.
6. (d) : Remove' a matter' 32. (a) It should be wasted instead of vested.
7. (c) : Remove' for preparing' 33. (d) It should be griping which means a sudden strong pain
8. (b) : Either' must' or' 'have to' alone should be used in stomach.
9. (c) : Raplace' were' by 'are' . 34. (e) No correction is required.
10. (c) : Replace' indefinite' by , indefinitely' . 35. (b) The correct spelling is charming.
11. (d) : Replace' days' by daily' . 36. (d) It should be ‘here was her son’s future’.
12. (d) : Replace' needs' by' need'
37. (c) The correct spelling should be ‘triumphant’.
13. (d) : Replace' of' by in'
38. (a) The word suppressed should be replaced by revealed
14. (c) : Replace' appreciating' by' appreciated
15. (a) : Remove' being' or leaked in the sentence.
16. (b) : Replace 'are' by 'is' 39. (b) It should be ‘handed over’ which means the act of
17. (d) : Remove 'independent' moving power or responsibility from one person to
18. (d) : Replace 'does' by 'did' another.
40. (a) The correct spelling is difficult.
y
o
u
rs
m
a
h
SOLUTIONS 79

b
o
o
SPEED TEST 66 32. (e) Piece of cake means something easy to do. Therefore,

b
option (e) is the correct choice.

.w
1. (b) 2. (d) 3. (d) 4. (b) 33. (b) Took to one’s heels means to run away. Therefore,

o
rd
5. (c) 6. (d) 7. (a) 8. (a) option (b) is the correct choice.

p
9. (a) 10. (c) 11. (d) 12. (d) 34. (e) To pledged means to make a promise. Therefore, option

re
13. (c) 14. (d) 15. (c) 16. (d) (e) is the correct choice.

s
s
17. (a) 18. (c) 19. (c) 20. (a) 35. (d) Crying need means a definite or desparate need for

.c
21. (a) 22. (b) 23. (b) 24. (a)

o
someone or something.

m
25. (d) 26. (b) 27. (d) 28. (c) Therefore, option (d) is the correct choice.
29. (c) 30. (b)
36. (d) Light upon means to arrive at something by chance.
Therefore, option (d) is the correct choice.
SPEED TEST 67

Sol: (Qs. 1-5): Clearly C must be followed by D, which must be SPEED TEST 69
further followed by the E as E reitrates the housing shortage and
says that the real deficit will be even higher. D and E provide the 1. (d) 2. (c) 3. (a) 4. (a)
statistical proof of the staggering task mentioned in C. So this 5. (b) 6. (c) 7. (b) 8. (b)
leads us to two options (b) and (d). Among them (b) seems to be 9. (e) 10. (a) 11. (b) 12. (e)
more appropriate as B again emphasises on but is being said is A 13. (b) 14. (a) 15. (c) 16. (b)
and also that B cannot be the concluding statement of the 17. (e) 18. (b) 19. (a) 20. (e)
paragraph. Hence, ABCDE gives the correct arrangement. 21. (c) 22. (b) 23. (d) 24. (a)
1. (a) 2. (b) 3. (c) 4. (d) 25. (e) 26. (a) 27. (d) 28. (b)
5. (e) 29. (c) 30. (a) 31. (c) 32. (d)
Sol. For (Qs.6-10) : The arrangement EABDC is correct. 33. (e) 34. (b)
The paragraph is clearly taking about Goa state and hence E has
to be opening sentence is the paragraph. This is followed by A SPEED TEST 70
where the phrase, ‘is an impressive case in point’, which is an
example of what is being said in E. A is followed by E as ‘a similar 1. (b) 2. (d) 3. (e) 4. (b)
agitation’ mentioned in B refers to the public activism mentioned
5. (c) 6. (c) 7. (a) 8. (d)
in A. B is further followed by D and C.
9. (c) 10. (b) 11. (d) 12. (a)
6. (e) 7. (a) 8. (b) 9. (d)
13. (e) 14. (b) 15. (a) 16. (b)
10. (c)
17. (d) 18. (e) 19. (c) 20. (c)
Sol For. (Q.11-15): The required arrangement is ABDCE.
A is the opening sentence as is clear from the given options. A is 21. (d) into 22. (a) around
followed by B as ‘the proposal’ mentioned in B is reffering to 23. (b) translating 24. (c) practice
whatever has been talked in A. B is followed by D as D continues 25. (c) chance 26. (d) provided
to talk about the response mentioned in B. Also note that B 27. (a) other 28. (e) off
mentions a 2 : 1 response against the proposal which is also clear 29. (e) hard 30. (b) equilibrium
by the 68 : 31 mentioned in D. D is followed by C and C is followed 31. (a) provides 32. (e) form
by E. The ‘other immigration organizations’ mentioned in E clearly 33. (e) symbol 34. (b) made
states that the previous sentence must have a statement from 35. (d) emphasis
some other organization, which is the immigration lawyers as
mentioned in C. SPEED TEST 71
11. (a) 12. (b) 13. (d) 14. (c)
15. (e) 16. (e) 17. (b) 18. (e)
1. (b) 2. (a) 3. (c) 4. (b)
19. (a) 20. (c) 21. (a) 22. (b)
5. (d) 6. (c) 7. (b) 8. (c)
23. (c) 24. (d) 25. (d) 26. (d)
27. (a) 28. (a) 29. (a) 30. (d) 9. (b) 10. (d) 11. (c) 12. (c)
13. (a) 14. (e) 15. (b) 16. (e)
SPEED TEST 68 17. (b) 18. (c) 19. (e) 20. (d)
21. (e) 22. (d) 23. (e) 24. (b)
1. (b) 2. (b) 3. (a) 4. (b) 25. (b) 26. (d) 27. (d) 28. (a)
5. (c) 6. (a) 7. (d) 8. (d) 29. (c) 30. (c) 31. (e) 32. (a)
9. (d) 10. (c) 11. (c) 12. (b) 33. (c) 34. (a) 35. (d)
13. (b) 14. (a) 15. (b) 16. (a) 36. (b) substitute has for are.
17. (b) 18. (d) 19. (c) 20. (b) 37. (b) replace should by will.
21. (b) 22. (b) 23. (a) 24. (d)
38. (a) sale should come in place of selling
25. (b) 26. (b) 27. (a) 28. (d)
29. (a) 30. (d) 31. (b) 39. (c) earned.
40. (d) replace since with from.
y
o
u
rs
m
a
h
80 101 SPEED TEST

b
o
o
SPEED TEST 72 21. (a) 22. (a) 23. (a) 24. (b)

b
25. (c) 26. (a) 27. (e) 28. (e)

.w
29. (c) 30. (d)

o
1. (a) 2. (c) 3. (a) 4. (b)

rd
5. (e) 6. (b) 7. (b) 8. (d)

p
SPEED TEST 77

re
9. (a) 10. (e) 11. (e) 12. (b)

s
13. (a) 14. (b) 15. (a) 16. (e)

s
1. (b) 2. (b) 3. (b) 4. (c)

.c
17. (d) 18. (a) 19. (e) 20. (b)

o
21. (b) 22. (e) 23. (d) 24. (a) 5. (c) 6. (a) 7. (a) 8. (d)

m
25. (d) 26. (b) 27. (a) 28. (c) 9. (a) 10. (d) 11. (b) 12. (d)
29. (b) 30. (c) 13. (d) 14. (d) 15. (b) 16. (e)
17. (b) 18. (e) 19. (e) 20. (e)
SPEED TEST 73 21. (b) 22. (b) 23. (e) 24. (c)
25. (d) 26. (c) 27. (e) 28. (c)
1. (e) 2. (a) 3. (d) 4. (a) 29. (d) 30. (a)
5. (b) 6. (d) 7. (e) 8. (e)
9. (a) 10. (e) 11. (c) 12. (c) SPEED TEST 78
13. (b) 14. (a) 15. (b) 16. (d)
17. (d) 18. (c) 19. (c) 20. (d) 1. (b) 2. (e) 3. (e) 4. (e)
21. (b) 22. (c) 23. (c) 24. (d) 5. (a) 6. (b) 7. (a) 8. (c)
25. (b) 26. (e) 27. (e) 28. (c) 9. (a) 10. (d) 11 (a) 12. (c)
29. (c) 30. (c) 13. (e) 14. (a) 15. (b) 16. (d)
17. (e) 18. (c) 19. (d) 19. (c)
SPEED TEST 74 21. (d) 22. (d) 23. (e) 24. (e)
25. (b) 26. (a) 27. (d) 28. (a)
1. (e) 2. (a) 3. (d) 4. (b) 29. (e) 30. (a)
5. (b) 6. (c) 7. (b) 8. (a)
9. (a) 10. (c) 11. (a) 12. (d) SPEED TEST 79
13. (d) 14. (a) 15. (b) 16. (d)
17. (c) 18. (c) 19. (c) 20. (d) 1. (e) 2. (b) 3. (c) 4. (a)
21. (a) 22. (d) 23. (a) 24. (a) 5. (b) 6. (a) 7. (d) 8. (e)
25. (a) 26. (c) 27. (b) 28. (a) 9. (b) 10. (c) 11. (c) 12. (c)
29. (b) 30. (c) 13. (d) 14. (e) 15. (e) 16. (c)
17. (d) 18. (e) 19. (c) 20. (e)
21. (c) 22. (d) 23. (d) 24. (e)
SPEED TEST 75 25. (a) 26. (a) 27. (c) 28. (d)
1. (c) 2. (a) 3. (c) 4. (c) 29. (c) 30. (b)
5. (a) 6. (b) 7. (c) 8. (b)
9. (a) 10. (c) 11. (a) 12. (b) SPEED TEST 80
13. (a) 14. (c) 15. (d) 16. (d)
1. (e) 2. (b) 3. (d) 4. (a)
17. (e) 18. (a) 19. (b) 20. (a)
21. (b) 22. (c) 23. (b) 24. (a) 5. (c) 6. (c) 7. (a) 8. (c)
25. (a) 26. (a) 27. (e) 28. (a) 9. (c) 10. (a) 11. (d) 12. (b)
29. (a) 30. (c) 13. (c) 14. (a) 15. (c) 16. (e)
17. (c) 18. (d) 19. (a) 20. (d)
SPEED TEST 76 21. (e) 22. (c) 23. (a) 24. (a)
25. (a) 26. (a) 27. (e) 28. (e)
1. (e) 2. (d) 3. (c) 4. (e) 29. (d) 30. (a) 31. (b) 32. (c)
5. (d) 6. (e) 7. (d) 8. (c) 33. (e) 34. (e) 35. (c) 36. (e)
9. (c) 10. (b) 11. (c) 12. (b)
37. (a) 38. (d) 39. (c) 40. (d)
13. (d) 14. (e) 15. (b) 16. (d)
17. (b) 18. (a) 19. (d) 20. (a)
y
o
u
rs
m
a
h
SOLUTIONS 81

b
o
o
SPEED TEST 81 20. (b) As per RBI, growth of card acceptance infrastructure like

b
.w
automated teller machines (ATMs) and point-of-sale (PoS)
terminals is not on a par with that of card issuance.

o
1. (b) 2. (c) 3. (a) 4. (b) 5. (b) 6. (c)

rd
7. (d) 8. (c) 9. (c) 10. (c) 11. (d) 12. (b) Debit cards vastly outnumber the volume of credit cards

p
13. (d) 14. (d) 15. (b) 16. (d) 17. (a) 18. (c) issued in the country. Further, a high number of debit

re
19. (b) 20. (a) 21. (c) 22. (d) 23. (d) 24. (d) cards have been issued in recent times under the Prime

s
s
25. (b) 26. (c) 27. (b) 28. (a) 29. (d) 30. (b) Minister's Jan Dhan Yogana, especially to customers

.c
in rural areas and smaller towns.

o
m
SPEED TEST 82 Cash continues to be the predominant mode of payment
as it appears to be "costless" in comparison to the
1. (c) The accounting year of RBI starts the month between visible costs associated with card/electronic payments.
July-June. Some of the factors that have inhibited growth in the
2. (a) In Article-30, RBI permitted to the co-operative Bank acceptance infrastructure are lack of adequate and low-
for special account supervision. cost telecom infrastructure and lack of incentive for
3. (c) Open market operations of RBI refers to trading in merchants for acceptance of cards, among others.
securities. 21. (b) Reserve Bank of India said the government will buy
4. (b) Monetary policy in India is formulated and implemented back inflation indexed bonds/IIBs maturing in 2023 via
by RBI. reverse auction in February 11th. Not so effective bonds
5. (a) Reserve Bank of India follows minimum reserve system have received a below par response due to lack of
for the issuing of currency. marketing and associated tax issues. GoI has also
6. (b) RBI controls credit creation by the Commercial Bank in notified repurchase of 1.44 percent inflation indexed
India. government stock 2023 via a reverse auction for total
7. (b) Note issuing department of RBI should always possess amount of INR 6500 crore. Bonds were launched as
the minimum gold stock worth ` 115 crore. alternative to gold as an investment in high current
8. (b) RBI and CSO in India is entrusted with the collection account deficit. Repurchase will be prematurely held to
of data of capital formation. redeem government stock through utilization of surplus
9. (c) The Bank rate is the rate at which RBI gives credit to cash balance and it was an ad-hoc move.
the commercial Banks. 22. (a) RBI has four Deputy Governors. Two are from outside,
10. (b) An increase in CRR by the RBI results in reduction in a commercial banker and an economist and two others
liquidity in the economy. are promoted from within its ranks.
11. (c) Commercial Banks provide the largest-credit to - Deputy Governor of RBI can be appointed for a period
agriculture and allied sectors. of five years or till the age of 62, whichever is earlier.
12. (a) RBI publishes the financial report on currency and Current other three Deputy Governors of RBI are:R.
finance. Khan, S.S. Mundra and R. Gandhi.
13. (a) RBI is the custodian of India’s foreign exchange funds. - The Union Government has reappointed Dr Urjit Patel
14. (c) RBI sanctions foreign exchange for the import of goods. as Deputy Governor of Reserve Bank of India (RBI).
15. (d) There are 4 posts of deputy Governor in Reserve Bank - Dr Urjit Patel headed committee to review the monetary
of India. policy framework.
16. (d) RBI regulates the external commercial borrowings. 23. (b) Urijit Patel, Deputy Governor of RBI in charge of the
17. (d) All the statements given above are correct except that, monetary policy department, has been appointed for a
RBI was established in 1949. term of three years for the second time. Patel, 52, will
18. (b) RBI has tweaked upcoming new rules for lending rates. finish his 3 year term on January 10, and has become
The central bank said on 29th March 2016 the fixed rate the longest serving deputy governor in 2015-2016 if he
loans of close to three years offered by lenders will be serves the full three year term. Many of the recent deputy
linked to the marginal cost of funding. Loans above governors served a maximum period of only 5 years. A
that tenor should be exempt. Earlier, all fixed rate loans deputy governor can serve for the period of three years
have been exempted from being set based on marginal w.e.f taking on the charge on or after January 11,2016
cost of funding. The change will apply to new rules or till further orders, whichever is earlier. A PhD from
implemented from April 1, 2016.New rules will force Yale University in Economics and graduate of the
lenders to adjust lending rates in relation to market University of London and Oxford, he has served as the
rates removing the sector's discretion in making a head of the U. Patel Committee proposing inflation
decision as to how much to charge for loans. targeting as the prime objective of the central bank. As
19. (d) Reserve Bank has provided relief to banks holding per the agreement, RBI will target 4 percent inflation at
bonds of State Electricity Boards that were debt ridden. the close of the financial year 2016-2017 and for
Banks have been allowed to keep Ujwal Discom subsequent years, with a band of +/- 2 percent. Other
Assurance Yojana scheme bonds under held to deputy governors of the RBI are H. R. Khan, R. Gandhi
maturity category. This will lessen pressure on bond and S. S. Mundra.
yields and debt market. HTM is part of the debt holdings 24. (c)
of the bank not subjected to daily price movement 25. (c) Since March, 1934 India is a member of the International
which can be held by banks till maturity. Monetary Fund. It has to, therefore, maintain its rate of
y
o
u
rs
m
a
h
82 101 SPEED TEST

b
o
o
exchange at the level which it has declared to the IMF. 10. (c) VAT is imposed on all stages between production and

b
.w
The Reserve Bank takes suitable measures to maintain final sale.
the value of the rupee at this declared level. 11. (c) Balance of payment is used in terms of Exports and

o
rd
Imports.

p
SPEED TEST 83 12. (b) The Indian Economy can be described as a developing

re
economy.

s
s
1. (a) 2. (c) 3. (d) 4. (b) 13. (b) 14. (d) 15. (d) 16. (d)

.c
5. (a) 6. (d) 17. (a) 18. (a)

o
m
7. (c) To provide basic banking services to bankless villages 19 (b) A company which has net owned funds of at least
8. (b) 9. (b) 10. (d) 11. (a) ` 300 Crore and has deployed 75% of its total assets in
12. (d) 13. (a) 14. (a) Infrastructure loans is called IFC provided it has credit
15. (a) RBI has given nod to Muthoot Finance to set up White rating of A or above and has a CRAR of 15%.
Label ATMs .ATMs set up and run by non-banking 20. (a) If the IDF is set up as a trust, it would be a mutual fund,
entities are called White Label ATMs (WLAs). regulated by SEBI.
16. (c) It is a special type of credit card which is sponsored by 21. (a) NBFC-MFI is a non-deposit taking NBFC which has at
both the credit card issuing company and the least 85% of its assets in the form of m microfinance.
participating retail company or vendor. Co-branded Such microfinance should be in the form of loan given
credit card carries special deals and savings from the to those who have annual income of `60,000 in rural
participating merchants. areas and `120,000 in urban areas. Such loans should
17. (a) Credit cards linked to special organizations like sports not exceed `50000 and its tenure should not be less
clubs, exclusive clubs and charities. Affinity credit cards than 24 months. Further, the loan has to be given without
can also help raise funds, when a part of income from collateral. Loan repayment is done on weekly,
every transaction goes toward the benefit of relevant fortnightly or monthly instalments at the choice of the
organization. borrower.
18. (b) 22. (d) Factoring refers to the process of managing the sales
19. (a) Visa card In 1977, Visa was adopted internationally and register of a client by a financial services company.
became the first credit card facility to be recognised Basically, there are three parties involved in a factoring
worldwide. It is a conditional authorization given by a transaction: 1. The buyer of the goods 2. The seller of
competent authority of a country for a person who is the goods 3. The factor, i.e. The financial institution.
not a citizen of that country to enter its territory and to 23. (b) All NBFCs are not allowed to take deposits. Only those
remain there for limited duration. NBFCs which have specific authorization from RBI are
20. (c) A hot card is a lost or stolen card. A hot list is the list of allowed to accept/hold public deposits. NBFCs cannot
caution against the use of a credit card by a defaulter take demand deposits. They can accept only term
holder. deposits with a tenure of minimum 12 months.
21. (d) 24. (b) Liquid assets are those assets that can be exchanged
most readily with minimum number of obstacles and
SPEED TEST 84 minimum time.
25. (b) Chanakya
1. (e) 2. (a) 3. (b) 4. (e) 5. (a) 6. (b) 26. (d) Both (b) and (c)
7. (c) 8. (a) 9. (c) 10. (a) 11. (e) 12. (b) 27. (a) Ratio of money held by the public in currency to that of
13. (a) 14. (a) 15. (a) 16. (b) 17. (b) 18. (a) money held in bank deposits.
19. (c) 20. (c) 21. (c) 22. (d) 23. (d) 24. (d) 28. (b) The proportion of total deposits commercial banks keep
25. (c) 26. (b) 27. (b) 28. (d) 29. (c) 30. (d) as reserves.
29. (c) The fraction of the deposits that commercial banks must
SPEED TEST 85 keep with RBI
30. (b) This includes the currency (notes and coins in
1. (c) Tertiary sector of Indian economy contributes largest circulation and vault cash of commercial banks) along
to the Gross National Product. with the deposits held by the Government of India ad
2. (a) Service sector is the main source of National Income in commercial banks with RBI.
India.
3. (d) Toll tax is not a tax levied by the government of India. SPEED TEST 86
4. (d) The most appropriate measure of a country’s economic
growth is it’s per capita product. 1. (c) ‘Eco Mark’ is given to the Indian products for
5. (c) Foreign Exchange Management Act (FEMA) was finally Environment Friendly purpose.
implemented in the year 2002. 2. (b) The Earlier name of WTO was GATT before 1995.
6. (b) The most common measure of estimating inflation in 3. (b) World Development Report is an annual publication of
India is WPI. IBRD.
7. (d) The national income of India is estimated by CSO. 4. (a) India has the maximum volume of foreign trade with
8. (c) Finance Ministry formulates the fiscal policy in India. USA.
9. (b) The devaluation of rupee in India took place twice in 5. (b) Participatory notes (PNs) are associated with Foreign
the financial year 1991-92. Institutional Investors (FII’s).
y
o
u
rs
m
a
h
SOLUTIONS 83

b
o
o
6. (b) The purpose of India Brand Equity Fund to make ‘Made

b
incorporating state of the art weaponry and sensors

.w
in India’ a label of quality. including the extended range Barak 8 surface-to- air
7. (a) A trade policy consists of Export-Import policy.

o
missiles.

rd
8. (b) FERA in India has been replaced by FEMA. 20. (a) Indian railway has launched the E-Samiksha an online

p
9. (b) TRIPS and TRIMS are the terms associated with WTO. project monitoring system with an aim of monitoring

re
10. (c) In the year 2006, SEZ act was passed by the parliament. implementation of various ongoing projects including

s
s
11. (d) There are total 162 members recently in WTO. Rail Budget proposals. Apart from budget-related

.c
12. (d) Capital account is classified into 3 parts in India-

o
projects, the E-Samiksha can also be used for

m
private, banking and official capital. monitoring the infrastructure target and board meeting
13. (a) The investment in productive assets and participation follow-up.
in management as stake holders in business enterprises 21. (c) The Government of India, through its Ministry of
is foreign direct investment. Environment & Forests, is implementing a project titled
14. (b) The portfolio investment by foreign institutional "Capacity Building for Industrial Pollution Management"
investors is called foreign institutional investment. under the assistance of World Bank. The project
15. (c) 16. (c) 17. (b) 18. (b) objectives are strengthening the environmental
19. (a) 20. (c) 21. (d) 22. (d) management capacity of central and state level
23. (b) 24. (a) 25. (c) 26. (d) regulatory authorities with emphasis on rehabilitation
27. (c) 28. (c) 29. (c) 30. (b) of polluted sites and for undertaking area-based
demonstration projects on remediation of contaminated
SPEED TEST 87 sites. The project also aims at developing a "National
Program for the Rehabilitation of Polluted Sites" to
1. (c) Swabhiman scheme is associated with Rural Banking reduce or eliminate the environmental and health risks
in India. associated with legacy pollution.
2. (c) The unorganised workers social security Act was 22. (a) The union government has set up a panel headed by B
passed in 2008. N Navalawala to look into various contentious issues
3. (c) Swadhar scheme launched for the women in difficult relating to inter-linking of rivers. The panel will works
circumstances. towards speedy implementation of the inter-linking of
4. (a) The main objective of Pradhan Mantri Gramodaya river projects. The panel comprising of water experts
Yojana is meeting rural needs like housing, drinking, and senior officials of different ministries will facilitate
water, healthcare, etc. interlinking of intra-state and intra- basin rivers.
5. (b) Twenty point Economic programme was first launched 23. (a) Dr Nasim Zaidi has been appointed a next Chief Election
in 1975. Commissioner of India. He will assume the charge of
6. (a) The disguished unemployment is 9 prominent feature the office from April 19, after the incumbent H S Brahma
mainly of primary sector. will retire on April 18.
7. (a) Golden Quadrangle project is associated with Highways 24. (d) This scheme was announced to enable minority youth
development. to obtain school leaving certificate and gain better
8. (d) All the above given statements is the objective of employment.
National Food security Mission. 25. (c) Nawaj Shaikh is from National AIDS Research Institute
9. (c) The period 2012-17 is related to 12th Five year plan in (NARI) and belongs to Pune. The said competition was
India. held through the MyGov Platform for suggesting the
10. (b) Liberalization of Economy is not a measure of reducing Logo, Slogan and Tagline for the New Education Policy
inequalities. of the Government of India. More than 3000 entries
11. (a) Valmiki Awas Yojana subsumed Integrated Housing were received. It was for the first time ever that this
and slum development programme. particular logo was designed by a common man of the
12. (c) Poverty level in India is established on the basis of country and not by any advertising agency or a
house hold consumer expenditure. corporate house. A cash prize of `10,000 will be
13. (b) Nirmal Bharat Abhiyan Yojana is associated with awarded to Shaikh.
community toilets in slum areas.
SPEED TEST 88
14. (c) Crop Insurance scheme in India was started in 1985.
15. (d) 'SJSRY' Scheme is not related to the rural development.
1. (a) Ruchir Kamboj, currently the Chief of Protocol, has
16. (d)
been appointed as the next Permanent Representative
17. (c) ICDS programme focused on the age group of children
of India to UNESCO, Paris, with the rank of ambassador.
upto 6 years.
The 1987-batch IFS officer, Kamboj will succeed V S
18. (d) The age group of 40-79 years old women are eligible for Oberoi.
India Gandhi Widow Pension Scheme. 2. (a) Government of India has appointed Justice Vangala
19. (b) The Project 15B of Indian navy aims to develop stealth Eswaraih, former acting Chief Justice of Andhra
guided missile destroyers. Under the project Mazagon Pradesh as the chairperson of the National Commission
Dock Limited will construct four stealth guided missile for Backward Classes. The National Commission of
destroyers. Project 15B destroyers will feature Backward Classes has been set up under the National
enhanced stealth characteristics as well as Commission for Backward Classes Act 1993.
y
o
u
rs
m
a
h
84 101 SPEED TEST

b
o
o
3. (e) 24. (a) Nuclear Security Summit 2016 took place in Washington

b
.w
4. (c) Justice Swatanter Kumar is the present Chairperson of DC on April 1, 2016 and leaders comprised PM Narendra
National Green Tribunal of India. Modi, UK PM David Cameron, Canadian PM Justin

o
rd
5. (b) Prime Minister of India chairs the National Ganga River Trudeau, French President Francois Hollande and other

p
Basin Authority. world leaders. World leaders have stressed the

re
6. (b) importance of Convention on Physical Protection of

s
s
7. (b) Shri Ratan Thiyam has been appointed as the Nuclear Material and its amendment in 2005 as well as

.c
chairperson of the National School of Drama Society. the International Convention on the suppression of

o
m
The President of India has appointed Shri Thiyam for a acts of nuclear terrorism.
period of four years as per the relevant rules and 25. (a) The 13th edition of the World Spice Congress began in
regulations of the Society. Ratan is a playwright and Ahmedabad, Gujarat and the theme of the two yearly
theatre director. three day event is 'Target 2020: Clean, Safe and
8. (e) 9. (e) 10. (c) 11. (c) Sustainable Supply Chain'. The World Spice Congress
12. (e) 13. (d) 14. (c) 15. (a) has been organised jointly by Spices Board, Cochin
16. (c) 17. (b) 18. (b) 19. (a) Hill Produce Merchant's Association and IndiaPepper
20. (a) 21. (a) 22. (b) 23. (c) and Spice Trade Association. It is given support by All
24. (c) 25. (a) 26. (d) 27. (b) India Spices Importers & Distributors Association and
28. (c) 29. (b) 30. (c) Indian Spices & Foodstuff Exporters' Association.
26. (a) The Union Health and Family Welfare Minister Shri JP
SPEED TEST 89 Nadda chaired the 12th Conference of the Central
Council of Health and Family Welfare which discussed
1. (d) Brazil is not a member of ASEAN. the Draft of the National Health Policy 2016. Some of
2. (b) ‘Sanklap’ project is associated with the eradication of the resolutions adopted towards the close of the
HIV/AIDS. meeting were as followed:
3. (b) 19th SAARC Summit will held in Islamabad, Pakistan in (i) Health needs of country have changed over time
since the last National Health policy
2016.
(ii) Draft National Policy formulated by the Ministry
4. (d) The 29th summit of ASEAN will be organised in Laos.
of Health and Family Welfare, GoI has been
5. (d) The 2016 BRICS Summit will be held in Panaji, Goa in
formulated keeping this in mind
India.
(iii) Endorsement of the Draft National Health Policy
6. (a) The 2016 annual meeting of WEF organised in
by the Central Council for Health and Family
Switzerland.
Welfare.
7. (b) There are 21 members in the APEC nations organisation. 27. (d) One day Annual Conference of the State Minorities
8. (d) The 17th NAM summit proposes to be held in Commission commenced with the focus on inclusive
Venezuela. growth with the motto "Sabka Saath, Sabka Vikas." The
9. (c) The G-20 2016 Summit will be hosted in China. one day Conference focusing on Development of
10. (a) The 42nd summit of G-7 will be held in Japan. Minorities had two Technical sessions besides the
11. (a) Azerbaijan and Fiji became the new members of NAM. inaugural. Valedictory. Mr. Praveen Davar addressed
12. (b) The 2016 NATO summit organised in the Poland. the Technical I session on "Minority Welfare Schemes
13. (a) A lady president elected first time in South Korea. of Government of India-An Overview".
14. (a) George Mario Bergoglio elected as a new pope of Roman Technical II session on "Functioning of State Minorities
Catholic Church. Commissions-Problems and Challenges" was
15. (a) The ‘UN Women’ came into existence on 1 July 2010. addressed by Ms. Mabel Rebello. Last year, the themes
16. (d) There are 10 members associated with BIMSTEC. of the conference were "Capacity building of Minority
17. (a) The Ranking of India in Global Hunger Index list is 63rd Educational Institutions in India" and "Corporate Social
last year. Responsibility and Development of Minorities in India".
18. (d) There are 8 members associated with MERCOSUR. 28. (a) This is the flagship initiative of the Shipping Ministry.
19. (a) The 9th World Hindi Conference held in South Africa. It provides an excellent platform for participants to fund
20. (c) There 8 point mentioned in MDG-2015 of United Nation. business opportunities. The Maritime India Summit will
21. (a) The headquarter of FAO (Food and Agriculture showcase investment opportunities in the maritime
organisation) is situated in Italy. sector for different activities such as port modernisation,
22. (b) SAMPRITI-III, 9 a special security forces exercise shipbuilding, ship repair and cycling. The aim is to
organised between India and Bangladesh. provide a platform for interacting closely with
23. (a) Shanghai Cooperation Organisation has set to pioneering international maritime organisations and
showcase alignment between China and Russia create awareness about trends and potential for
through integration of Beijing marshalled Silk Road maritime sector in India.
Economic Belt and Moscow driven Eurasian Economic 29. (d) Mr. Nitin Gadkari, Minister of Shopping, Road
Union. SCO is part of the emerging Eurasia-centred Transport and Highways launched a website entitled
Silk Road geopolitical architecture pillared by China www.maritimeinvest.in to commemorate the Maritime
and Russia, along with the Central Asian Republics. India Summit within the Make in India Summit. User
y
o
u
rs
m
a
h
SOLUTIONS 85

b
o
o
friendly website will facilitate access to detailed Rajasthani by a Rajasthani writer. It carries a citation, a

b
.w
information about the summit including registrations. plaque and prize money of ‘ 1 lakh.
Maritime India Summit 2016 is a global maiden summit 8. (a) Tansen Samman is conferred in the field of music which

o
rd
being carried out by the Shipping Ministry in April. For carries a cash prize of ‘ 2 lakh and a citation.

p
MIS 2016, India's partner country is the Republic of 9. (a)

re
Korea. More than 50 maritime nations have also been 10. (b) The Shanti Swarup Bhatnagar award for Science and

s
s
invited to attend the summit. Technology (SSB) is an award in India given annually

.c
30. (a) 5 day long 10th WTO Ministerial Conference was by the CSIR. It is named after the founder Director of

o
m
concluded on 19th December 2015 in Nairobi, Kenya the CSIR and carries an award money of ‘ 5 lakh each.
aimed at adoption of the Nairobi package for benefitting 11. (a) In 2006, Kejriwal was awarded the Ramon Magsaysay
the poorest members of the organisation. The Award for Emergent Leadership recognising his in-
conference was attended by trade ministers of 162 volvement in a grassroots movement Parivartan using
member countries of the WTO. India was represented right-to-information legislation in a campaign against
by Minister of State (Independent Charge) for corruption. The same year, after resigning from the IRS,
Commerce & Industry Nirmala Sitharaman. This marks he donated his Magsaysay award money as a corpus
the first time a meeting has been held in Africa by WTO fund to found the Public Cause Research Foundation,
in this capacity. a non-governmental organisation (NGO).
12. (a) Bidhan Chandra Roy Award was instituted in 1976 in
SPEED TEST 90
memory of B. C. Roy by Medical Council of India. The
Award is given annually in the categories of States-
1. (d) The Jnanpith award is a literary award which along
manship of the Highest Order in India, Medical man-
with the Sahitya Akademi Fellowship is one of the two
cum-Statesman, Eminent Medical Person, Eminent per-
most prestigious literary honours in the country. The
son in Philosophy and Eminent person in Arts. It is
award was instituted in 1961. Any Indian citizen who
presented by President of India in New Delhi on July 1,
writes in any of the official languages of India is eli-
the National Doctors’ Day.
gible for the honour.
13. (a) Pampa Prashasti is the highest literary honour con-
2. (a) Bharat Ratna is India’s highest civilian award. The offi- ferred by the Karnataka government on a litterateur for
cial criteria for awarding the Bharat Ratna stipulated it
his/her lifetime contribution to Kannada literature. The
is to be conferred “for the highest degrees of national award is named after Adikavi Pampa of 10th Century.
service which includes artistic, literary, and scientific The award carries award money of ‘ 3 lakh and a cita-
achievements, as well as “recognition of public service tion.
of the highest order”. The last recipient of the award is 14. (a) Madhya Pradesh has been awarded 10th National
the cricketer Sachin Tendulkar for the year 2014. Award for Excellence work in Mahatma Gandhi National
3. (c) The National Film awards, one of the most prominent Rural Employment Guarantee Act (MGNREGA). The
film awards in India, were established in 1954. Every award was given to Madhya Pradesh for convergence
year, a national panel appointed by the government of MGNREGA and other schemes for construction of
selects the winning entry, and the award ceremony is permanent assets and for excellent work in providing
held in New Delhi where the President of India pre- opportunities for earning permanent income.
sents the awards. 15. (b)
4. (c) The Vyas Samman is a literary award which was first 16. (d) The Nobel prize is a set of an international awards be-
awarded in 1991. It is awarded annually by the K.K. stowed in a number of categories which is given annu-
Birla Foundation and includes a cash of ally to the winners by Swedish and Norwegian Com-
‘ 250,000 (as of 2005). To be eligible for the award, the mittees in recognition of cultural and/or scientific ad-
literary work must be in the Hindi language and has vances. It was the will of the Swedish inventor Alfred
been published in the past 10 years. Nobel that established the Nobel prizes in 1895 in Swe-
5. (c) The Saraswati Samman is an annual award for outstand- den.
ing prose or poetry literary works in any Indian lan- 17. (c) The Nobel prize in Economics or Economic sciences
guage. It was instituted in 1991 by the K. K. Birla Foun- was established in 1968 and endowed by Sweden’s
dation. The award contains ‘ 10 lakh, a citation and a central bank, the Sveriges Riksbank, on the occasion
plaque. Candidates are selected from literary works of the bank’s 300th anniversary. While the Nobel Prize
published in the previous ten years by a panel that in particular was established in 1895.
includes scholars and former award winners. 18. (a) The Academy award is also known as the Oscar award
6. (a) The Maha Vir Chakra is the second military decoration in which is presented for various categories in the Film
India and is awarded for acts of conspicuous gallantry in industry. It was first given in 1929.
the presence of the enemy, whether on land, at sea or in the 19. (c) The Confucius Peace prize is a prize established in 2010
air. The medal may be awarded posthumously. in the People’s Republic of China (PRC). The Confucius
7. (d) Bihari Puraskar conferred by Rajasthan is a literary Peace Prize’s first winner was former Vice President of
award instituted by K. K. Birla Foundation. The award the Republic of China and Kuomintang Chairman Lien
is named after the famous Hindi poet Bihari and is Chan, for his contribution to developing positive ties
awarded to an outstanding work published in Hindi or between Taiwan and mainland China.
y
o
u
rs
m
a
h
86 101 SPEED TEST

b
o
o
20. (a) The Pulitzer Prize is a U.S. award for achievements in 14. (b) Anand Math is a 1952 Hindi patriotic-historical film

b
.w
newspaper and online journalism, literature, and musi- directed by Hemen Gupta, based on ‘Anandamath’,
cal composition. It was established in 1917 and admin- the famous Bengali novel written by Bankim Chandra

o
rd
istered by Columbia University in New York City by Chattopadhyay in 1882. The novel and film are set in

p
provisions in the will of American publisher Joseph the events of the Sannyasi Rebellion, which took place

re
Pulitzer. in the late 18th century in eastern India, especially

s
s
21. (c) The Nobel awards in literature, medicine, physics, chem- Bengal.

.c
istry, peace, and economics are given in Stockholm, 15. (b) 16. (b) 17. (c) 18. (c)

o
m
Sweden. The Peace prize is awarded in Oslo, Norway. 19. (b) 20. (b)
22. (a) The British Academy Film awards are presented in an 21. (a) The Vice President of India Shri M. Hamid Ansari
annual award show hosted by the British Academy of released a book titled “India and Malaysia: Interwined
Film and Television Arts (BAFTA). It is given by UK Strands” authored by former diplomat Smt Veena Sikri.
and is considered to be the counter awards for Oscars. The book offers a panoramic yet in-depth historical
23. (c) Golden Globe award is given in the field of film and analysis of the inter-linkages between India and
television by Hollywood Foreign Press Association in Malaysia, which are a microcosm of the much larger
United States of America. relationship between South Asia and South East Asia,
24. (a) The Palme d’Or is the highest prize awarded at the as these have evolved more than two millennia.
Cannes Film Festival and is presented to the director of 22. (d) The Global Competitiveness Report is a yearly report
the best feature film of the official competition. It is published by the World Economic Forum. The report
presented by Festival International du film de, France. assesses the ability of countries to provide high levels
25. (a) The Kalinga Prize for popularization of Science is an of prosperity to their citizens. This in turn depends on
international distinction instituted by UNESCO. It was how productively a country uses available resources.
started in 1951 by donation from Mr Bijoyanand Patnaik, Since 2010, Switzerland has led the ranking as the most
founder and president of the Kalinga Foundation Trust competitive economy in the world India is at 60th
in India. position.
26. (a) International Gandhi Peace prize is given annually by 23. (b) Ashtadhyayi is written by Panini. Panini a sage who is
Government of India to those individuals and organi- believed to have lived around the fifth century B.C.,
zations which contribute towards changes in the po- although other claims trace him to the 4th, 6th and 7th
litical, social or economic reforms via non-violence. It centuries, is credited with having created the
was instituted in 1995. Ashtadhyayi, or Ashtak, which is a grammar defining
27. (c) The Royal Institute of British Architects Stirling Prize the structure and syntax of the Sanskrit language.
is a British prize for excellence in architecture. It is 24. (c) Mitakshara is written by Vigyaneshwar. It was consid-
organised and awarded annually by the Royal Insti- ered one of the main authorities on Hindu Law from the
time the British began administering laws in India.
tute of British Architects (RIBA).It is named after the
25. (c) Kautilya’s Arthashastra is an excellent treatise on state-
architect James Stirling. craft, economic policy and military strategy. it is said to
28. (a) The World Economic Forum gives Crystal award to have been written by Kautilya, also known by the name
those artists who have improved the state of the world Chanakya or Vishnugupta, the prime minister of India’s
through their art. first great emperor, Chandragupta Maurya.
29. (d) Ramon Magsaysay award is given annually to those 26. (d) The Post Office (Bengali: Dak Ghar) is a 1912 play by
Asian people who have contributed extraordinary ser- Rabindranath Tagore. It concerns Amal, a child confined to
vice in their respective fields. This award is given by his adopted uncle’s home by an incurable disease.
Philippine in the memoir of Philippine President Ramon 27. (c)
Magsaysay. He is considered to be one of the great 28. (b) English author Rudyard Kipling wrote the Jungle Book
examples of integrity, courage, and idealistic democrat. in 1894. It is a collection of stories.
30. (a) 29. (a) War and Peace is a novel by the Russian author Leo
Tolstoy, first published in 1869. The work is epic in
SPEED TEST 91 scale and is regarded as one of the most important
works of world literature. It is considered as Tolstoy’s
1. (c) The book presents the booming maintenance of the finest literary achievement, along with his other major
world’s largest democracy and achievements of India prose work, Anna Karenina (1873–1877).
since independence. 30. (b) The Prince is a 16th-century political treatise by the
2. (d) Ambedkar Speaks (Trilogy) is authored by Dr. Narendra Italian diplomat and political theorist Niccolò
Machiavelli. The descriptions within The Prince have
Jadhav.It is an attempt to develop a comprehensive
the general theme of accepting that the aims of princes—
bibliography on the speeches delivered by Ambedkar.
such as glory and survival—can justify the use of im-
3. (c) 4. (a) 5. (b) moral means to achieve those ends.
6. (d) A pictorial coffee table book by Alam Srinivas was
launched on 24 June 2013 in Mumbai by former Bombay SPEED TEST 92
High Court Chief Justice C S Dharmadhikari.
7. (b) 8. (a) 9. (b) 10. (a) 1. (c) The Olympic Museum is located in Lausanne, Switzer-
11. (d) 12. (d) 13. (a) land. The museum was founded on 23 June 1993, on
y
o
u
rs
m
a
h
SOLUTIONS 87

b
o
o
the initiative of Juan Antonio Samaranch. The Olympic 25. (a) Founded in 1961, Netaji Subhas National Institute of

b
.w
Museum was opened again on 21 December 2013 after Sports commonly known as National Institute of Sports
23 months of renovation. (NIS), is the Academic Wing of the Sports Authority of

o
rd
2. (b) In golf, a caddy or caddie is the person who handles a India (SAI) and Asia’s largest Sports Institute located

p
golf player’s bag and clubs, and gives also some in- in city of Patiala.

re
sightful advice and moral support to him. 26. (b) Karnam Malleshwari is an Indian weightlifter. She is

s
3. (c) The ICC Champions Trophy is a One Day International the first Indian to win an individual medal in Olympics.

s
.c
(ODI) cricket tournament organised by the International 27. (a)

o
Cricket Council (ICC). It is second in importance only 28. (b) The first world cup Hockey was played in Barcelona in

m
to the Cricket World Cup. 1971 whose final winner was Pakistan.
4. (c) 29. (b) The average length of the football field is
5. (a) The four Grand Slam tournaments, also called Majors, 100 – 110m (110 – 120 yards) with width is in the range
are the most important annual tennis events. The Grand of 64 to 75 m (70–80 yd).
Slam itinerary consists of the Australian Open in mid 30. (a) The India national field hockey team had won its first
January, the French Open in May/June, Wimbledon in Gold in 1928 at Amsterdam, Nederlands in which India
June/July, and the US Open in August/September. defeated the Nederlands by 3-0. India also won Gold in
6. (d) The first Youth Olympic Games (YOG), was held in 1932, 1936, 1948, 1952, 1956, 1964, and 1980.
Singapore from 14 to 26 August 2010. The age limit for
the games is 14-18. The next Summer Youth Olympics SPEED TEST 93
will be held in Nanjing in 2014.
7. (d) Subroto Cup Football Tournament is an inter-school 1. (b) 2. (b)
football tournament in India, named after the Indian Air 3. (b) With an aim of avoiding a repeat of the 2004 catastrophe,
Force Air Marshal Subroto Mukerjee. Subroto Cup is India is building Tsunami warning device in the South
conducted by the Indian Air Force, with support from China Sea, which is likely to operate in the next 10
India’s Ministry of Youth Affairs & Sports. months.
8. (a) Wankhede stadium is in Mumbai. It is in this stadium 4. (a) British drug maker GlaxoSmithKline is seeking
that India had won the World cup cricket in 2011 against regulatory approval for the world’s first malaria vaccine
Sri Lanka. after trial data showed that it had reduced the number
9. (a) A Gambit is a term associated with the game of Chess.
of cases in African children. The vaccine known as
Gambit is a chess opening in which a player sacrifices
RTS, S was found to have almost halved the number of
material, usually a pawn, with the hope of achieving a
resulting advantageous position. malaria cases in young children in the trial and to have
10. (a) The term ‘ashes’ is associated with cricket. reduced by about 25% the number of malaria cases in
11. (a) National Sports Day is celebrated on 29th August in the infants. GSK is developing RTS,S with non-profit Path
memory of the great hockey player Major Dhyan Chand. Malaria Vaccine Initiative supported by funding from
12. (c) A bogey is a score of 1-over par on any individual hole the Bill & Melinda Gates Foundation.
on a golf course. Golf holes are typically rated as par 3, 5. (b) Kepler’s laws of planetary motion are three scientific
par 4 or par 5. laws describing the motion of planets around the Sun -
13. (c) FINA or Fédération Internationale de Natation or Inter- The orbit of a planet is an ellipse with the Sun at one of
national Swimming Federation is the International Fed- the two foci, a line segment joining a planet and the Sun
eration (IF) recognized by the International Olympic sweeps out equal areas during equal intervals of time and
Committee (IOC) for administering international com- the square of the orbital period of a planet is proportional to
petition in Aquatics. the cube of the semi-major axis of its orbit.
14. (b) 6. (b)
15. (a) The Dronacharya award is presented by Indian Gov- 7. (b) A plant that produces both tomatoes and potatoes,
ernment to people showing excellence in sports coach- called the TomTato, has been developed for the UK
ing. B.I. Fernandez is the first foreign Coach who was market. Ipswich-based horticulture firm Thompson and
awarded by Dronacharya Award in 2012.
Morgan said the hybrid plants were not genetically
16. (b) 17. (b)
modified
18. (d) The Duleep Trophy is a domestic first-class cricket
competition played in India between teams represent- 8. (b) Australia unveiled its most powerful computer Raijin
ing geographical zones of India. The competition is named after the Japanese God of thunder and rain. Raijin
named after Kumar Shri Duleepsinhji. is considered the 27th most powerful computer in the
19. (b) The Marquess of Queensberry rules is a code of gen- world.
erally accepted rules in the sport of boxing. 9. (a)
20. (a) 21. (a) 22. (a) 10. (b) The Vikram Sarabhai Space Centre (VSSC) is a major
23. (a) The Olympic Games were a series of athletic competi- space research centre of the Indian Space Research
tions among representatives of city-states and one of Organisation (ISRO), which focuses on rocket and space
the Panhellenic Games of Ancient Greece. They were vehicles for India's satellite programme. It is located in
held in honor of Zeus, and the Greeks gave them a Thiruvananthapuram, Kerala.
mythological origin. The first Olympics is traditionally 11. (c) The first fertilizers’ plant was established in Sindri which
dated to 776 BC. is an industrial township within the Dhanbad municipal
24. (d) limits of the Dhanbad District of Jharkhand state.
y
o
u
rs
m
a
h
88 101 SPEED TEST

b
o
o
12. (a) PARAM is a series of supercomputers designed and 5. (b) The two day meet of 13th Broadband Commission for

b
.w
assembled by the Centre for Development of Advanced Sustainable Development was held in Dubai (UAE). It
Computing (C-DAC) in Pune, India. The latest machine

o
aims to promote the power of broadband networks &

rd
in the series is the PARAM Yuva II. services for the 2030 Agenda for sustainable

p
13. (b) Cinnabar is the common ore of mercury.Generally it Development.

re
occurs as a vein-filling mineral associated with recent 6. (c) World Consumer day is celebrated across the world on

s
s
volcanic activity and alkaline hot springs. 15th March 2016. The theme for 2016 is- antibiotics off

.c
14. (a) 15. (c) the menu.

o
m
16. (a) ETWS installed in Rangachang in Andaman and 7. (d) President Pranab Mukherjee inaugurated the 5th
Nicobar Islands to predict Tsunami within three minutes edition of India Aviation 2016 in Hyderabad which was
of being triggered. jointly organised by the Union Civil Aviation Ministry
17. (c) & the federation of Indian Chambers of Commerce &
18. (c) Hindustan Aeronautics Limited (HAL): Tejas is India’s Industry (FICCI).
indigenously built light combat aircraft. It has come 8. (c) The fourth Nuclear Security Summit (NSS) is being held
from Light Combat Aircraft (LCA) programme. in Washington, D.C. on March 31-April 1, 2016.
19. (a) Telefonica, Spanish broadband & telecommunications 9. (d) Divine speaker and spiritualist Chaganti Koteswara Rao
provider has launched the world's first smartphone with has been appointed Adviser to Andhra Pradesh
the Firefox Operating System (OS) to compete with Government.
devices running Google's Android and Apple's IOS. 10. (b) Hashim Thaci was sworn-in as the President of Kosovo
20. (b) Indian Space Research Organization (ISRO) has in April 2016.
successfully launched IRNSS-1A on PSLV C 22 from 11. (a) Argentina topped the April 2016 edition of FIFA or
Satish Dhawan Space Centre, Sriharikota in Andhra Coca-Cola World Rankings of Soccer.
Pradesh. 12. (a) Shardul Amarchand Mangaldas advised Future Group
21. (a) on its acquisition of 100% stake in FabFurnish.com.
22. (a) Global biotechnology company 'Life Technologies' has 13. (a) NASA astronaut Scott Kelly will write a memoir titled
launched India's first private DNA forensics laboratory Endurance: My Year in Space and Our Journey to Mars,
in Gurgaon which is expected to accelerate sampling as announced on 6 April 2016. The book will be
process thereby reducing the burden on existing published by Alfred A. Knopf.
forensic laboratories. 14. (b)
23. (c) IBM have made the world's smallest movie "A Boy and 15. (b) The World Health Day (WHD) 2016 theme is "Beat
Diabetes" to scale up diabetes prevention, strengthen
His Atom" using atoms. The movie has been verified
care and enhance surveillance.
by the Guinness World Records
16. (b) The Australian grand prix was won by Nico Rosberg
24. (b) "Nirbhay", First cruise missile of India has a Long-
overtaking Sebastian Vettel & Lewis Hamilton.
range (1,000-2,000 km) & strikes targets more than 700 17. (c) Agasthyamala Biosphere Reserve shared by Kerala &
km away carrying nuclear warheads. Tamil Nadu has been placed in the new 20 Biosphere
25. (c) Sqn Ldr Rakesh Sharma was a test pilot in the IAF. In Reserve list released by UNESCO.
1984 he became the first citizen of India to go into space 18. (a) Union Urban Development, Housing & Urban Poverty
when he flew aboard the Soviet rocket Soyuz T-11. Alleviation & Parliamentary Affairs Minister M.
Venkaiah Naidu was bestowed with Skoch Lifetime
SPEED TEST 94 Achievement Award at 43rd SKOCH Summit held in
New Delhi for his contributions to inclusive growth &
1. (c) 2. (b) 3. (a) 4. (d) towards poverty alleviation in India.
5. (c) 6. (b) 7. (d) 8. (a) 19. (b) Union Health Ministry launched Bedaquiline new anti-
9. (b) 10. (b) 11. (b) 12. (a) TB drug for Drug Resistant TB as part of the Revised
13. (d) 14. (a) 15. (d) 16. (b) National Tuberculosis Control Program on the eve of
17. (c) 18. (d) 19. (b) 20. (a) World TB Day.
21. (b) 22. (b) 23. (c) 20. (a) Veteran Singer Stevie Wonder was honoured with
Global Green Hero Award at the 13th Annual Global
SPEED TEST 95 Green US ceremony.
21. (b) The International Women's Day is celebrated on 8th
1. (c) The Pradhan Mantri Ujjwal Yojana is a scheme to provide March globally. 2016 Theme is "Planet 50-50 by 2030:
LPG to women of all households below poverty line. Step It up for Gender Equality."
2. (b) 22. (c) Luxembourg 23. (a) Bangladesh
3. (b) The British mathematician Andrew J.Wiles won the 2016 24. (a) Justice Permod Kohli was appointed as the new
Abel prize for solving a centuries-old hypothesis- chairman of the Central Administrative Tribunal (CAT).
Fermat's Last theorem. 25. (a) Rajasthan has become the first state to pass land title
4. (a) The World Happiness Report, 2016 released by bill.
sustainable Development Solutions Network, a global 26. (b) Italian super-car brand Lamborghini appointed former
initiative of the United Nations placed India in 118th Audi India head of field forces, Sharad Agarwal, as its
rank in terms of Happiness Index. new Head of India operations.
y
o
u
rs
m
a
h
SOLUTIONS 89

b
o
o
27. (b) Khadija Ismayilova became the 2016 recipient of the 27. (b) RM Lodha committee The Lodha committee, which was

b
.w
UNESCO/Guillermo Cano World Press Freedom Prize. primarily assigned with the task of determining the
28. (a) Historian Srinath Raghavan wrote the book titled "India's quantum of punishment for players and others involved

o
rd
War: The Making of Modern South Asia 1939-1945" which in spot fixing in IPL, also gave recommendation on

p
details India's contribution to World War II. change in the structure and ecosystem in the Indian

re
29. (c) Danny Willett has won the 80th edition of US Masters cricket board saying no to politicians' involvement in

s
s
golf tournament 2016 at Augusta National in Georgia, BCCI.

.c
United States. 28. (a) The Shuklaphanta Wildlife Reserve is a protected area

o
m
30. (a) UN commemorated the 125th birth anniversary of B. R. in the Terai of the Far-Western Region, Nepal, covering
Ambedkar on April 13 at the UN headquarters. 305 km2 (118 sq mi) of open grassland, forests, riverbeds
and tropical wetlands at an altitude of 174 to 1,386
SPEED TEST 96 metres. It was gazetted in 1976 as Royal Shuklaphanta
Wildlife Reserve.
1. (b) 2. (d) 3. (d) 4. (e) 29. (d) Diarrhoea caused by Rotavirus is one of the leading
5. (e) 6. (b) 7. (e) 8. (e) causes of severe diarrhoea and death among children
9. (c) 10. (e) 11. (b) 12. (a) less than five years of age in India. Rotavac is the new
13. (c) 14. (d) vacccine launched by Health Ministry to co
15. (a) Narmada is a west flowing river. It flows westward over 30. (c) The International Day of Remembrance of the Victims
a length of 1,312 km before draining through Gulf of of Slavery and the Transatlantic Slave Trade is observed
Cambay into the Arabian Sea. every year on March 25 to offer the opportunity to
16. (d) Floating rate bonds have variable interest rate and honour and remember those who suffered and died at
protect investors against a rise in interest rates (which the hands of the brutal slavery system. The 2016 theme
have an inverse relationship with bond prices) . They is "Remember Slavery: Celebrating the Heritage and
also carry lower yields than fixed notes of the same Culture of the African Diaspora and its Roots".
maturity. 31. (a) Swayam Shikshan Prayog (SSP) aims to promote
17. (c) As per rules, government has to get all money bills empowerment of women as leaders and entrepreneurs
related to the union budget passed within 75 days of through self help groups, social enterprises and
the presentation of the budget. community led initiatives.
18. (d) Commodity markets in India are regulated by Forward 32. (a) Bangladesh Export Processing Zones Authority BEPZA
Markets Commission (FMC) headquartered at Mumbai. stands for "Bangladesh Export Processing Zones
19. (d) 20. (a) Authority", which is a government organization, that
21. (c) The National Maritime Day of India is celebrated every includes information of investment opportunities and
year on April 5. On this day, in 1919 navigation history for investors, proposals, reports, export promotion
was created when SS Loyalty, the first ship of the zones (EPZ) in Bangladesh.
Scindia Steam Navigation Company, journeyed to the 33. (c) Assam rifles is India's oldest paramilitary force, raised
United Kingdom. originally in 1835 as Cachar Levy. They perform many
22. (b) The Mullaiperiyar Dam is a masonry gravity dam on roles including the provision of internal security under
thePeriyar River in the Indian state of Kerala. It is located the control of the army through the conduct of counter
insurgency and border security operations. Assam
881 mabove mean sea level, on the Cardamom Hills of
Rifles is known as "Sentinels of the Northeast". Its
the Western Ghats in Thekkady, Idukki District of Kerala.
headquarters are located at Laitkor (Shillong). The motto
It has been asubject of controversy between Tamil
of Assam Rifles is "Friends of the Hill People". While
Nadu and Kerala.
Assam Rifles functions under the Ministry of Defence,
23. (c) The Confederation of Indian Industry (CII) is an
its administrative control is under the Ministry of Home
association of Indian businesses which works to create
Affairs.
an environment conducive to the growth of industry in
34. (a) Gurmeet Singh has become the first Indian athlete to
the country. CII is located at New Delhi. clinch a gold medal at the 2016 Asian 20km Race Walk
24. (b) The Maharashtra government has recently tied up with Championships in Nomi, Japan. It is the first time in 34
the Tata Trusts to link all government colleges and years that an Indian has won a gold in either an Asian
medical colleges with he latter's national cancer grid. Championships or Asian Games in this event. Before
With this, Maharashtra has become the India's first him, Hakam Singh and Chand Ram won a gold each in
state to join the initiative. The national cancer grid links 20km road walk in 1978 and 1982 Asian Games
all existing and proposed cancer care centres to create respectively.
a uniform line of treatment for cancer patients across 35. (b) Bank The Indian Overseas Bank (IOB) has become the
India. first bank to commence the sale of Indian Gold Coin
25. (b) Kachhabali village in Bhim tehsil of Rajsamand district (IGC) in the domestic market. The bank has tied up with
has become the first liquor-free village in Rajasthan. Metals and Minerals Trading Corporation (MMTC) for
26. (a) Kakrapara Atomic Power Station (KAPS) in Gujarat was the sale of IGC of 24 carat purity. IGC is the first ever
recently making news, when its one unit was shut down national gold offering by the Centre. The IGC has the
after leakage of heavy water from its coolant system. national emblem, the Ashok Chakra, engraved on one
y
o
u
rs
m
a
h
90 101 SPEED TEST

b
o
o
side and the face of Mahatma Gandhi on the other. The is a land mass towards the north of the Black Sea and

b
.w
coin carries advanced anti-counterfeit features and has Ukrainian region of Kherson to the south and
comes in tamper-proof packaging. The current Russian region of Kuban to the west. Crimea and

o
rd
denominations available are 5, 10 and 20 gm. adjacent territories were united in the Crimean Khanate

p
36. (a) India has recently signed a loan agreement of $35 before becoming part of Russia during 1783. Its

re
Million with the World Bank (WB) for "Madhya economy primarily depends on tourism and agriculture,

s
s
Pradesh Citizen Access to Responsive Services but since Russian annexation of Crimea in 2014, there

.c
Project". The purpose of the project is to improve access

o
has been a considerable drop in the number of people

m
and quality of public services in Madhya Pradesh visiting it. With an area of 27,000 square kilometres,
through implementation of the 2010 Public Service Crimea has a population of 2.2 million people. .
Delivery Guarantee Act. The duration of project is 5 45. (a) Infrasound has a frequency lower than 50 hertz
years. Infrasound has a frequency lower than 20 hertz . Though
37. (d) Mossad is the national intelligence agency of Israel. It it cannot be heard by humans, animals such as
is one of the most powerful secret service agencies in elephants, rhinos, alligators and whales produce
the world. It's headquarters is in Tel Aviv, Israel. The
infrasound. One interesting feature of these sound
agency has active agents spread across the world and
waves is that they can travel longer distances than
are involved in intelligence gathering, covert operations
high frequency waves. Avalanches and earthquakes
and protecting Jews and Jewish interests.
38. (a) The United Nations Educational, Scientific and Cultural also produce infrasound.
Organization (UNESCO) Institute of Physics has 46. (c) The ICICI Bank has launched "iWork@home", a first-
recently released the e-Atlas of Gender Inequality in of-its-kind programme that allows women employees
Education. The e-Atlas shows gender gaps from primary to work from home for up to a year. The company has
to tertiary education and educational pathways of girls deployed face recognition technology to facilitate
and boys in more than 200 countries and territories. women to work from home and provide access to the
39. (b) The Songkran festival, also known as the World's bank's core banking servers.
biggest water festival, is celebrated every year in 47. (c) The Karnataka government has launched
Thailand from 13 to 15 April. Recently, it is in news Mukhyamantri Santwana Harish Yojana for free medical
because Thailand government has decided to impose treatment to accident victims. As per the scheme, the
a curfew during 2016 Songkran festival to show accident victims will be given free medical treatment
solidarity with farmers hit by drought. for the first 48 hours at any hospital and financial aid
40. (d) India's first contact-less mobile payment solution "iTap" upto Rs. 25,000 will also be provided. The scheme has
has been launched by ICICI Bank. Through "iTap" its been named after Harish Nanjappa who had donated
customers will be able to make payments by waving his eyes moments before his death. .
their cell phones near an NFC- enabled merchant 48. (b) The Jalandhar-based Capital Local Area Bank Ltd
terminal. In it, ICICI bank has used the Host Card (CLABL) will become the India's first small area finance
Emulation (HCE) technology which creates 'virtual' bank (SFB) by flagging-off its operations on April 13,
cards for 'physical' credit or debit cards (Visa/ 2016. The small bank will launch operations under a
MasterCard) of the bank, as selected by the customer. new name "Capital Small Finance Bank Limited". Apart
41. (b) The Uttar Pradesh has launched awareness campaign from CLABL, the Reserve Bank of India (RBI) has
at school levels to create awareness about granted 9 entities in-principle licences to open small
conservation of birds on March 14, 2016. The awareness finance banks to expand access to financial services in
campaign would also be spread among the students to rural and semi-urban areas. These are Ujjivan Financial
make the state clean and green under "CleanUP - Green Services Pvt. Ltd, Janalakshmi Financial Services Pvt.
UP".
Ltd, Au Financiers (India) Ltd, etc.
42. (b) The Union Power Ministry has recently named the LED
49. (b) India's first 75-seater solar powered ferry will come up
based Domestic Efficient Lighting Programme (DELP)
in the favourite tourism spot of Kerala "Allepey (the
as "UJALA". Presently, DELP is running successfully
Venice of the East)".These boats will be commissioned
in over 120 cities across India. UJALA, an acronym for
Unnat Jyoti by Affordable LEDs for All, is being within the next three months and will run on the 2.5 km-
implemented by Energy Efficiency Services Limited long Vaikkom-Thavanakkadavu route. The boat is 20 m
(EESL). The National LED programme was launched long, 7 m wide with a maximum cruising speed of 7.5
by the Prime Minister on January 2015 with a target of knots. The boat is built by a Kochi-based NavAlt, in
replacing 77 crore incandescent lamps with LED bulbs. collaboration with a French firm AltEn, which makes
43. (d) In 1966, MS Subbulakshmi was invited by Secretary- solar powered ferries across the world.
General of the United Nations, U Thant to give a special 50. (a) India's first self cleaning smart toilets have been
concert at the United Nations. This was the first installed in Chennai, Tamil Nadu. Around 180 self-
performance by any Indian classical musician at the cleaning public e-Toilets have been installed and are
UN. . free for public use. Each toilet automatically cleans itself
44. (a) Black Sea and Sea of Azov Crimean Peninsula is before entry and after use; and sensors in it enable
surrounded by two seas - Black Sea and Sea of Azov. It consumption of less water and power with each use.
y
o
u
rs
m
a
h
SOLUTIONS 91

b
o
o
SPEED TEST 97 : PRELIM TEST - 1

b
.w
o
Answer Key

rd
p
1 (d) 16 (b) 31 (c) 46 (c) 61 (d) 76 (b) 91 (a)

re
2 (a) 17 (c) 32 (b) 47 (b) 62 (a) 77 (c) 92 (e)

s
s
.c
3 (c) 18 (d) 33 (e) 48 (e) 63 (c) 78 (a) 93 (b)

o
4 (a) 19 (e) 34 (d) 49 (a) 64 (e) 79 (a) 94 (e)

m
5 (b) 20 (e) 35 (a) 50 (d) 65 (b) 80 (e) 95 (c)
6 (e) 21 (a) 36 (c) 51 (a) 66 (c) 81 (a) 96 (b)
7 (d) 22 (b) 37 (c) 52 (d) 67 (b) 82 (e) 97 (d)
8 (c) 23 (c) 38 (a) 53 (e) 68 (c) 83 (b) 98 (c)
9 (a) 24 (c) 39 (d) 54 (c) 69 (a) 84 (c) 99 (d)
10 (e) 25 (b) 40 (b) 55 (b) 70 (c) 85 (d) 100 (a)
11 (b) 26 (b) 41 (e) 56 (a) 71 (d) 86 (e)
12 (d) 27 (b) 42 (d) 57 (d) 72 (d) 87 (a)
13 (e) 28 (a) 43 (b) 58 (c) 73 (c) 88 (c)
14 (b) 29 (e) 44 (d) 59 (a) 74 (e) 89 (b)
15 (c) 30 (d) 45 (e) 60 (b) 75 (b) 90 (d)

Hints & Solutions


11 4 5 9. (a) ? = 78.45 + 128.85 + 1122.25 = 1392 . 55
1. (d) ? 848 212 10. (e) 5598 = ? + 2785
16 5 11
? = 5598 – 2785 = 2813
150 1.4 480 2.2
2. (a) ? 87 37 375
100 100 11. (b) ?
5 8 8
= 10.50 + 10.56 = 21.06
3219 375
116 3 87 2 ?
3. (c) ? 40 8
4 3
= 87 – 58 = 29 3219 1875 1344
40 40
6.96 18.24
4. (a) ? 168 3
1.2 7.6 33
5 5
= 5.8 – 2.4 = 3.4
5. (b) ? = 32.25 × 2.4 × 1.6 = 123.84 5616
12. (d) ? 39
250 136 550 ? 18 8
6. (e) 670
100 100 420 288
13. (e) ? 135
340 + 5.5 × ? = 670 28 32
5.5 × ? = 670 – 340 = 330 14. (b) 484 ? 516
330
? 60 ? 516 484 32
5.5
? 32 32 1024
448
7. (d) ? 35 980
16 660 45 450 28
15. (c) ?
100 100
14 25 125 225 75 11
8. (c) ? = 1 = 297 + 126 = 423
120 72 192 64 64
16. (b) The pattern of the number series is:
y
o
u
rs
m
a
h
92 101 SPEED TEST

b
o
o
12 + 22 = 16

b
11

.w
16 + 23 = 24 0.85
13

o
24 + 24 = 40

rd
40 + 25 = 72 7 4 9 5 11

p
Clearly,

re
17. (c) The patern of the number series is : 9 5 11 6 13

s
25. (b) Ratio of the profit of Srikant and Vividh

s
9 + 10 = 19

.c
= 185000 : 225000 = 37 : 45

o
19 + 20 = 39

m
39 + 40 = 79 Sum of the ratios = 37 + 45 = 82
Total profit earned
79 + 80 = 159
18. (d) The pattern of the number series is: 82
= 9000 = ` 16400
8 + 32 = 17 45
17 + 52 = 42 26. (b) Father’s present age = 6x years
42 + 72 = 91 Son's present age = x years
After four years
91 + 92 = 172
19. (e) The pattern of the number series is: 6x 4 4
7× 1+1=8 x 4 1
8 × 2 + 2 = 18 6x + 4 = 4x + 16
18 × 3 + 3 = 57 12
57 × 4 + 4 = 232 2x = 12 x 6
2
20. (e) The pattern of the number series is: Son’s present age = 6 years
3840 4 = 960 27. (b) Let the number be x.
960 4 = 240
x × 65 2x 13 x 2x
240 4 = 60 140 140
100 5 20 5
60 4 = 15
13 x 8 x x
x 75 3 3x 3 140 140
21. (a) y y 20 4
100 7 4 7
x = 4 × 140 = 560
x 3 4 4 560 30
y 7 3 7 30% of 560 168
100
22. (b) Speed of the train 28. (a) Let the original number be 10x + y where y > x.
Length of train and platform 10y + x – 10x – y = 27
= Time taken to cross each other 9(y – x) = 27
y–x=3 ....(i)
275 275 550 18 and x + y = 13 ....(ii)
m/sec. kmph = 60 kmph
33 33 5 From equations (i) and (ii),
y = 8 and x = 5
Time
Rate Original number = 58
23. (c) CI P 1
100
1
29. (e) M1D1 = M2D2
22 × 16 = 32 × D2
2
9 22 16
45000 1 1 = 45000 × [(1.09)2 –1] D2 = = 11 days
100 32
30. (d) Let the smallest odd number A be x
= 45000 × 0.1881 = ` 8464.5
x + x + 4 = 2 × 59
9 7 2x = 118 – 4 = 114
24. (c) 0.82; 0.78
11 9 114
x 57
5 4 2
0.83; 0.8 31. (c) 60% of 150 = 90. It means those who obtained either
6 5
90 or more than 90 marks in the average of five
subjects will be declared as passed. Therefore,
the required number = 31 + 17 = 48
y
o
u
rs
m
a
h
SOLUTIONS 93

b
o
o
32. (b) Reqd no. = 52 + 41 + 13 = 106 Conclusions I and II form Complementary Pair.

b
.w
35. (a) No. of students who obtained more than or equal to Therefore, either I or II follows.

o
40% marks in Science 47. (b) All curtains are rods.

rd
= 67 + 22 + 14 = 103

p
re
No. of students who obtained less than 60% marks in

s
Hindi = 19 + 59 + 47 = 125

s
.c
o
125 103

m
Reqd % 100 17.60% Some rods are sheets.
125
(A + I No Conclusion)
36. (c) Meaningful words : ARE, EAR, ERA
48. (e) Some plugs are bulbs
37. (c) A D J E C T I V E
1 1 1 1 1 1 1
B C I F B S J U F
38. (a) na pa ka so birds fly very high All bulbs are sockets.
ri so la pa birds are very beautiful (I + A I-type)
ti me ka bo the parrots could fly “Some plugs are sockets”.
Thus high is coded as na. Conclusion I is Converse of thi3s Conclusion.
39. (d) 1 2 3 4 5 6 7 8 9 Conclusion II is Converse of the first Premise.
Difference = 8 – 4 = 4 49. (a) All fishes are birds. (conversion)
40. (b) 1 2 3 4 5 6 7 8 9 10 11 12 13
CO M PA T I B I L I T Y
Meaningful word LI M B
All birds are rats.
2 2
41. (e) As F H I G (A + A A-type)
2 2 “All fishes are rats”.
N P E C
All birds are rats. (conversion)
Similarly,
2 2
S U L J
2 2
I K T R
All rats are cows.
42. (d) As L 8 and H 7 (A + A A-type)
A & I * “All birds are cows”.
T 4 R 3 This is Conclusion I.
E $ E $ 50. (d) Some windows are doors.

Similarly,
H 7 A &
I * L 8 All doors are roofs.
43. (b) Others relate to ‘parts of tree’. (I + A I-type)
44. (d) Since ‘potato’ is called Banana. Thus, ‘Banana’ grows “Some windows are roots”.
underground. 51. (a) P + S P is daughter of S.
45. (e) EI, EG, GI and NL. S – T S is father of T.
46. (c) Some suns are planets. Therefore, P is sister of T.
52. (d) P × Q P is wife of Q.
Q – T Q is father of T.
T is child of P and Q.
All planets are satellites. The sex of T is not known.
(I + A I-type) T is either son or daughter of P.
“Some suns are satellites”.
y
o
u
rs
m
a
h
94 101 SPEED TEST

b
o
o
53. (e) P × S P is wife of S. 60. (b) E £; G $; A 2; K 1; R *; L 3

b
.w
S ÷ T S is son of T. Condition (ii) is applied.

o
T is either father-in-law or mother-in-law of P. For (Qs. 61-65): Given information can be tabulated as follows

rd
P ÷ S P is son of S.

p
re
S × T S is daughter of T

s
Person Sex Company Specialisation

s
Therefore, T is father of P.

.c
A Male X Finance

o
P – S P is father of T. B Male Z Marketing

m
P + T P is daughter of T C Male Y Engineer
T ÷ S T is son of S. D Female X HR
Therefore, T is father of P. E Male Y Doctor
54. (c) P + Q P is daughter of Q. F Male Y Marketing
Q – S Q is father of S. G Female Z Finance
H Male Z HR
Therefore, P is sister of S.
P ÷ Q P is son of Q.
Q × S Q is wife of S. Thus, ‘G’ is a sister of ‘C’.
Therefore, P is son of S. 66. (c) The number 49 is a perfect square of a natural number.
P ÷ Q P is son of Q. 67. (b) 1 3 3 4 5 6 7 8
Q + S Q is daughter of S.
P H Y S 1 C A L
Therefore, P is grandson of S.
55. (b) P + Q P is daughter of Q. Meaningful word HAIL
Q × T Q is wife of P. 68. (c) Obviously option (c) may be the conclusion of the
Therefore, T is father of P. passage. Because the passage also talks about the
56. (a) B 9; A 2; R *;N %; I #; S 4 literal the actual sense of culture.
57. (d) D 2; M @; B 9; N %; I #; A 6 69. (a) Option (a) have the same summary hense it strengthens
Condition (i) is applied. the conclusion of the passage.
58. (c) I $; J 8; B 9; R *; L £; G # 70. (c) Option (c) has the opposite sense of the passage. Thus
Condition (ii) is applied. it weakens the conclusion of the passage.
59. (a) B 9; K ©; G $; Q 7; J ©; N %
Condition (iii) is applied.
y
o
u
rs
m
a
h
SOLUTIONS 95

b
o
o
SPEED TEST 98 : PRELIM TEST - 2

b
.w
o
Answe r Key

rd
p
1 (d) 16 (b) 31 (b) 46 (b) 61 (c) 76 (b) 91 (d)

re
2 (c) 17 (b) 32 (e) 47 (e) 62 (b) 77 (c) 92 (c)

s
s
.c
3 (a) 18 (c) 33 (a) 48 (c) 63 (a) 78 (d) 93 (a)

o
4 (a) 19 (b) 34 49 (d) 64 (d) 79 (a) 94 (a)

m
(c)
5 (e) 20 (b) 35 (b) 50 (b) 65 (e) 80 (d) 95 (b)
6 (b) 21 (c) 36 (a) 51 (a) 66 (b) 81 (e) 96 (c)
7 (d) 22 (a) 37 (c) 52 (e) 67 (c) 82 (a) 97 (b)
8 (c) 23 (b) 38 (b) 53 (a) 68 (b) 83 (c) 98 (b)
9 (b) 24 (a) 39 (b) 54 (b) 69 (c) 84 (e) 99 (a)
10 (e) 25 (d) 40 (e) 55 (a) 70 (e) 85 (d) 100 (d)
11 (d) 26 (d) 41 (d) 56 (d) 71 (b) 86 (b)
12 (b) 27 (a) 42 (e) 57 (b) 72 (c) 87 (e)
13 (a) 28 (a) 43 (a) 58 (c) 73 (d) 88 (a)
14 (d) 29 (e) 44 (c) 59 (a) 74 (e) 89 (d)
15 (a) 30 (c) 45 (d) 60 (e) 75 (e) 90 (c)

Hints & Solutions


Thus 1001 is the largest number which divides the
3 4 5 3 4 5 1015
1. (d) of of of 1015 = 1015 145 numbers of the type 25025, 67067 etc.
5 7 12 5 7 12 7
12. (b) A three digit number to be exactly divisible by 5 must
have either 0 or 5 at its units place.
2. (c) 1.5 0.025 (?) 2 0.1 (?) 2 0.1 1.5 0.025
Such numbers will be 100, 105, 110, ..........., 995.
(?) 2 0.1 0.0375 ? .0625 0.25 First term = 100, last term = 995
Let the required number be n.
3. (a) 1.5 2 0.0375 2.25 0.15 0.3375 To find the value of n, we may use the following formula
5. (e) 125% of 260 + ?% of 700 = 500 of arithmetic progression,
Tn = a + (n – 1) d.......... (1)
?% of 700 = 500 – 125% of 260
Where d = common difference = 5
?% of 700 = 175 Tn = 995
175 100 a = 100
? 25 Hence from (1)
700
995 = 100 + (n – 1) 5 5n = 900
6. (b) 45% of 750 – 25% of 480 n = 180
45 750 25 480 Digits to be used = 0, 1, 2, 3, 4, 5, 6, 7, 8, 9.
= 337.5 120 217.5 13. (a) Let the principal be P, then amount after 12 years = 2P
100 100
SI (2P P ) P
7. (d) 758.5 753.8 75(8.5 3.8) 754.7
P r t P r 12
8. (c) 39798 + 3798 + 378 = 43974 Now, I P=
100 100
7 3 1 40 80 3 160 6 100 25 1
9. (b) 3 7 1 = 14 or r 8 %
11 11 2 11 11 2 11 11 12 3 3
14. (d) Applying successive discounts of 10%, 12% and 15%
1080
10. (e) 1080 12 10 9 on 100, we get 100 0.9 0.88 0.85 67.32
12 10
Single discount = 100 – 67.32=32.68
11. (d) First start with the option (d). 15. (a) Let the prices of two houses A and B be Rs 4x and
1001× 25 = 25025 Rs 5x, respectively for the last year.
1001× 67 = 67067 etc.
y
o
u
rs
m
a
h
96 101 SPEED TEST

b
o
o
Then, the prices of A this year = `(1.25 × 4x) and that of

b
103

.w
B = ` (5x + 50,000) 10 10609

o
This year, Ratio of their prices = 9 : 10 100

rd
203 609
1.25 4x 9

p
609

re
5x 50,000 10 0

s
s
5x = 4,50,000

.c
50x 45x 450000 Also, 7938.81 89.1 , by long division method, as

o
x = 90,000 below:

m
Hence, the price of A last year was 89.1
4x = `3,60,000 8 79388.81
Totaldistance 64
16. (b) Average speed = 169 1538
Total time 1521
1781 1781
80 80 1781
= = = 32 km/h
60 20 2.5 0
40 20 Hence 103 × 89.1 = 9177.3 9200
17. (b) Let the usual speed of the aeroplane be x km/h. 23. (b) 18.4% of 656 + 12.7% of 864 = 0.184 × 656 + 0.127 × 864
= 120.704 + 109.728 = 230.432 231
Then, 1500 1 1500
24. (a) (98.4)2 + (33.6)2 = 9682.56 + 1128.96 = 10811.52 10812
x 2 (x 250) 25. (d) Let there be x in place of question mark
Solving, we get x = 750 km/h
18. (c) Sum of 40 instalments = S40 = 3600 = 20 (2a + 39d) 8959 8959
So, 26.35 x 17
or 2a + 39d = 180 ...(i) x 4 5 26.35 20
Sum of 30 instalments = S30 = 2400 = 15 (2a + 29d) 26. (d) ? = 3739 + 164 × 27 = 3739 + 4428 8200
or 2a + 29d = 160 ...(ii) 27. (a) Required distance covered = 2(6 × 415 + 500) metre
From (i) and (ii), we get a = 51 and d = 2 = 2(2490 + 500) metre = 5980 metre = 5.98 km
The value of first instalment = ` 51 28. (a) Total marks obtained by Amit
19. (b) Volume of the earth taken out = 30 × 20 × 12 = 7200 m3 = 44 + 55 + 77 + 79 + 76 = 331
331
Field
Required percentage = × 100 = 66.2
500
470 m
30 m 29. (e) The pattern of the number series is:
7 × 2 – 3 = 11
30 m 11 × 2 – 3 = 19
19 × 2 – 3 = 35
20 m 35 × 2 – 3 = 67
67 × 2 – 3 = 134 – 3 = 131
470 m 30 m
Tank 30. (c) The pattern of the number series is:
5 + 12 = 6
Area of the remaining portion (leaving the area of dug 6 + 22 = 10
out portion) 10 + 32 = 19
= 470 × 30 + 30 × 10 = 14100 + 300 = 14400 m3 19 + 42 = 35
Let h be the height to which the field is raised when the 35 + 52 = 35 + 25 = 60
earth dug out is spread. 31. (b) No. of students (of JNU) listening to Radio City
Then, 14400 × h = 7200
h = 0.5 m 20 65
120000 15600
100 100
198 194 185 169 144
32. (e) The no. of Indraprastha students listening to FM
20. (b)
2 2 2 2 Rainbow = 120000 × 13% × 48%
–2 –3 –4 –5
The no. of Jamia students listening to
21. (c) The first, third, fifth .... and second, fourth .... terms are
FM Gold = 120000 × 18% × 52%
groups of consecutive natural numbers.
Reqd. percentage
22. (a) 10609 7938.81 120000 13% 48%
100 66.66%
10609 103, by long division method, as below: 120000 18% 52%
y
o
u
rs
m
a
h
SOLUTIONS 97

b
o
o
33. (a) Indraprastha University 43. (a) G ©

b
.w
34. (c) The no. of Indraprastha students listening to
Q *

o
13 46

rd
Red FM 120000 12 598 R 1

p
100 100

re
D 7
The no. of Jamia students listening to Red FM

s
s
B 3

.c
18 36
120000 12 648

o
100 100 N ©

m
Condition III is applied.
Total students = (12 × 598) + (12 × 648)
= 12 × 1246 = 14952 44. (c) I £
35. (b) Radio City P 5
36. (a) Since Badminton is not played by using a ball and also
E $
called Volleyball. So, Volleyball is required answer.
B 3
37. (c) T E C H N O L O G Y Q *
1 2 3 4 5 6 7 8 9 10 S £
Condition I is applied.
Word formed CE N T 45. (d) R #
38. (b) Arranging the number in ascending order, we get M @
1 23 45 78 9
4th number (Right) = 5 A 9
3rd number (left) = 3 Difference = 5 – 3 = 2 P 5
S £
39. (b) Ja Ki mo pe at a frog's leap
I 1
mo la Ki so take a leap ahead Condition II is applied.
46. (b) A 8
re bo Ja na insects are frog's diet
Thus, at is coded as pe. R 1
E $
40. (e) As W 9 and S #
K $ M @
E %
A 2 I 7 Q *
K $ T @ 8
N
Condition I is applied.
Thus, W 9
47. (e) K ©
A 2
I 7 J 2
T @ B 3
41. (d) DG, IG and SN P 5
42. (e) 1
G F R 1
1 D ©
L K
1 Condition III is applied.
A B 48. (c) P × R P is son of R.
1 R – S R is daughter of S.
D C
Therefore, P is grandson of S.
1 49. (d) P + R P is wife of R.
I J
R × S R is son of S.
1
O P Therefore, P is daughter-in-law of S.
1 P R P is father of R.
L K R × S R is son of S.
1 Therefore, S is wife of P.
U V
P + R P is wife of R.
1 R S R is father of S.
S R
Therefore, S is child of P.
y
o
u
rs
m
a
h
98 101 SPEED TEST

b
o
o
P R P is father of R. 60. (e) All leaves are roots. (conversion)

b
.w
R S R is wife of S.

o
Therefore, S is son-in-law of P.

rd
50. (b) P – Q P is daughter of Q.

p
Q S Q is wife of S.

re
All roots are flowers.

s
Therefore, S is father of P.

s
A+A A-tpye of conclusion

.c
51. (a) P × Q P is son of Q.

o
Q S Q is father of S. “All leaves are flowers”.

m
Therefore, P is brother of S. Conclusion II is Converse of this Conclusion.
52. (e) P R P is wife of R. All stems are roots. (conversion)
R S R is father of S.
The sex of S is not clear.
S is either son or daughter of P.
53. (a) Lowest number = 189 + 1 = 190
All roots are flowers.
Highest number = 972 + 2 = 974
A+A A-type of conclusion
difference = 9 – 4 = 5
“All stems are flowers”.
54. (b) Interchanging the digits of given number we get Conclusion I is Converse of this Conclusion.
279, 286, 981, 892 and 157. For (Qs. 61-67) : The given information can be tabulated as
3rd highest number = 286 or 682 follows:
55. (a) Arranging all three digits of given number in
descending order, we get Pers on S ex Relations hip Profes sion
972, 862, 981, 982 and 751.
3rd highest number = 972 L Female W ife of M A rchitect
56. (d) No pen is a mobile. (conversion) Father of P. Business man
M M ale Husband of L.
Son of Q and O.
Some mobiles are bottles. Female Daughter-in-law of
N Hous ewife
E+I O-type of conclusion O and Q.
“Some bottles are not pens”. O Female W ife of Q Pilot
Some mobiles are bottles. (conversion)
P M ale Son of M and L. Journalis t
Grandfather of P. Doctor

All bottles are papers. Q M ale Husband of Q.


I+A I-tpye of conclusion Father of M .
“Some mobiles are papers”. R – – A dvocate
57. (b) All computers are radios. (conversion)
Two married couples : LM and QO.
66. (b) According to question
9146756083
All radios are televisions. 67. (c) ab, abc, abcd, abcde, abcdef, abcd e
A+A A-type of conclusion
68. (b) Clearly option (b) is best support by the passage.
“All computers are televisions”. 69. (c) The passage provides the facts about the preparation
Conclusion II is Converse of this Conclusion. of history. Hence option (c) is the assumption which
58. (c) Conclusion I and II from Complementary Pair. Therefore, can be implicit in the facts of the passage.
either Conclusion I or II follows. 70. (e) Obviously. None of the given option is conclusion
59. (a) Some fishes are moons. (conversion)
which can be drawn from the facts of the passage.
71. (b) Refer to the Ist sentence of para 1 ‘Govind’s father
........................ left large tract of land to Govind’.
All moons are birds. 72. (c) Refer to the 3rd sentence of para 2 “To his
I+A I-type of conclusion surprise.................into gold”.
“Some fishes are birds”. 73. (d) Refer to the last two sentences of para 1 of the passage.
Conclusion I is Converse of this Conclusion. 74 (e)
y
o
u
rs
m
a
h
SOLUTIONS 99

b
o
o
75. (e) Refer to the sixth sentence of para 2 of the passage 79. (a) The word dismay (noun) means feeling of fear and

b
.w
‘You have..................during winter’. discouragement. For instance,

o
76. (b) Refer to the third last sentence of para 3 of the passage. They were struck with dismay at the news.

rd
77. (c) The word spend (verb) means to pay out, to employ While, the word joy (noun) means feeling of happiness.

p
(labour, time etc.) on some objects. For instance, Thus, both are antonyms.

re
s
How do you spend your leisure-time? 80. (d) The word tend (verb) means look after. For instance,

s
.c
Similarly, the word devote (verb) means to give time and The nurse tended the injured persons.

o
energy to something. For instance, While, the word ignore (verb) means take no notice of.

m
He is devoted to his studies. For instance,
Thus, spend and devote are synonymous. Raman ignored the advice of his elders.
78. (d) The word lie (verb) means to be at rest. For instance, Thus, both are antonyms.
She is lying on the bed. 86. (b) ‘Sent’ is the third form of verb ‘send’ in passive voice.
Similarly, the word remain (verb) means continue to 88. (a) Replace ‘though’ by ‘As’ to express cause and effect
stay. For instance, in the sentence.
Twenty pens were distributed and the remaining were 89. (d) It should be ‘and I answered all of them’ to properly
returned. express cause and effect in the sentence.
Thus, lying and remaining are synonymous. 90. (c) Apply the adverb ‘highly’ before the adjective ‘paid’.
y
o
u
rs
m
a
h
100 101 SPEED TEST

b
o
o
SPEED TEST 99 : PRELIM TEST - 3

b
.w
o
Answer Ke y

rd
p
1 (e) 16 (b) 31 (c) 46 (c) 61 (a) 76 (a) 91 (e)

re
2 (d) 17 (b) 32 (a) 47 (e) 62 (b) 77 (e) 92 (b)

s
s
.c
3 (c) 18 (d) 33 (e) 48 (a) 63 (b) 78 (b) 93 (a)

o
4 (d) 19 (d) 34 (e) 49 (b) 64 (a) 79 (d) 94 (b)

m
5 (a) 20 (e) 35 (a) 50 (c) 65 (b) 80 (c) 95 (d)
6 (d) 21 (c) 36 (c) 51 (d) 66 (d) 81 (c) 96 (e)
7 (b) 22 (e) 37 (c) 52 (c) 67 (b) 82 (e) 97 (c)
8 (a) 23 (b) 38 (b) 53 (a) 68 (c) 83 (a) 98 (c)
9 (e) 24 (d) 39 (b) 54 (d) 69 (c) 84 (c) 99 (c)
10 (d) 25 (a) 40 (e) 55 (b) 70 (d) 85 (e) 100 (d)
11 (d) 26 (c) 41 (b) 56 (a) 71 (e) 86 (d)
12 (b) 27 (b) 42 (c) 57 (b) 72 (d) 87 (a)
13 (b) 28 (b) 43 (b) 58 (c) 73 (c) 88 (b)
14 (d) 29 (b) 44 (d) 59 (b) 74 (a) 89 (d)
15 (b) 30 (d) 45 (a) 60 (a) 75 (e) 90 (a)

Hints & Solutions


1. (e) ? 75
Squaring on both the sides, we get 11. (d) Clearly, 1 1 7
7
? = 75 × 75 = 5625 7 7 7
21 7 1
2. (d) ? 1
8 72 171 is the smallest number..
7
21 7 1 3 12. (b) Let the sum be Rs x
or ? =
8 72 171 19
x 8 (3 2)
Now, 56 x = `700
1 2 1 100
3. (c) ? 4 6 5
2 3 3 13. (b) Let the cost price of the machine be `x.

3 4 2 9 1 11x
(4 6 5) 15 16 Then, selling price at a profit of 10% = `
6 6 2 10
4. (d) ? = 792.02 + 101.32 – 306.76 = 586.58 9x
5. (a) 300% of 150 = ? % of 600 And the selling price at a loss of 10% = `
10
or , ? of 600 = 45000 or, ? = 75
6. (d) 34.95 + 240.016 + 23.9800 = 298.946 Consequently, we find that
7. (b) 48.95 – 32.006 = ? 11x 9x
or, ? = 48.95 – 32.006 = 16.944 80
10 10
8. (a) 3889 + 12.952 – ? = 3854.002
or ? = 3889 + 12.952 – 3854.002 = 47.95 x
80 x `400
9. (e) ? + 72.64 = 74.64 or ? = 74.64 – 72.64 = 2.00 5
14. (d) Let the capacity of the jar be of x bottles.
10. (d) 6.25 0.0025 = ? or ? 6.25 2500 since 6 bottles were taken out from jar and 4 bottles of
0.0025
oil poured into it 2 bottles were taken out
y
o
u
rs
m
a
h
SOLUTIONS 101

b
o
o
Therefore, we have

b
22. (e) 963.7 950.9989 95?

.w
4 3

o
x 2 x or 95? 953.7 0.9989

rd
5 4

p
or 95? 9527 or , ? = 2.70

re
4 3

s
x x 2 x = 40 3.001

s
5 4

.c
23. (b) 1000 of 1891.992 = ?
4.987

o
m
Total distance 3
15. (b) Average speed = or ? = 100 of 1900 = 100 + 1140 = 1230
Total time 5
24. (d) 12.25 × ? × 21.6 = 3545.64
80 80
= = 32 km/h 3545.64
60 20 2.5 ?= 13.4 13
264.6
40 20
3
16. (b) Let the usual speed of the aeroplane be x km/h. 25. (a) ? = 4096 = 3 16 16 16 16
26. (c) M1D1 = M2D2
1500 1 1500
Then, 6 × 20 = 8 × D2
x 2 (x 250)
6 20
Solving, we get x = 750 km/h D2 = = 15 days
8
17. (b) Using Venn Diagram
363 ?
27. (b)
? 3
35-15 15 42-15 ?2 = 363 × 3 = 121 × 32 = 112 × 32
=20 =27
? = 11 × 3 = 33
28. (b) Raju's age at the time of marriage
Failed in only the Failed Failed in only the = 17 + 3 + 2 + 1 + 3 = 26 years
first subject in both second subject
the subjects 29. (b) Required number
= 2040 + 2300 + 2400 + 2200 + 2090 + 2120 = 13150
Thus, percentage of students who passed in both 30. (d) Required percentage growth
subjects
2250 2180
= 100 – [(35 – 15) + (42 – 15) + 15] = 100 – (35 + 42 – 15) = 100 3.21%
2180
= 100 – (62) = 38%
and percentage of students who failed in both subject 31. (c) Required %
= 15% 2540
Therefore, the percentage of students who passed in = 100
2500 2040 2100 2280 2540 2320
either subject = 100 – (38 + 15) = 100 – 53 = 47%
Hence, required no. of students who passed in either 254000
= % 18.43% 18%
47 13780
subject but not in both = 2500 × = 1175 32. (a) Required ratio = (2250 + 2480) : (2260 + 2440)
100
18. (d) Let the length and breadth of the rectangle be x and = 4730 : 4700 = 473 : 470
y cm, respectively. 33. (e) Required average number
Then, (x – 4) (y + 3) = xy 3x – 4y = 12 ... (i)
Also, (x – 4) = (y + 3) [sides of square] 2500 2250 2450 2150 2020 2300
=
x–y=7 ... (ii) 6
From (i) and (ii),
13670
x = 16 and y = 9 = 2278.33 2278
Perimeter of the original rectangle = 2(x + y) = 50 cm 6
19. (d) The series is × 1 + 1 × 7, × 2 + 2 × 6, × 3 + 3 × 5... 34. (e) 5x + 6x + 7x = 180°
20. (e) The series is × 1 + 12, × 2 + 22, × 3 + 32.., ... 18x = 180°
21. (c) The series is × 1 + 2, × 2 + 3, × 3 + 4, ...
y
o
u
rs
m
a
h
102 101 SPEED TEST

b
o
o
II. False. From both figures it is clear that either I or III

b
180

.w
x= 10 follows.
18

o
rd
Sum of the smallest and the largest angles

p
re
= 12x = 12 × 10 = 120°
er

s
mm

s
35. (a) 232 = 529 Books

.c
45. (a) Chair Ha

o
242 = 576 Flowers

m
Required number = 536 – 529 = 7

Fl
ow
Mats er
s
or Books
r
Chair
mme
43. (b) Desks Ha
Tables Chair

I. False II. False III. False

Cameras Photo
46. (c) Stores
Chair
Desks
Roofs
or
les
Tab
Mats

eras
I. False II. True III. False Ca m Pho
to
Roofs
or

Store
Pencil
Sweets
I. False II. False III. True
44. (d)
Fruits Glasses Crows
Tablets

47. (e) Nails Horses

Sweets
or sses
Gla Pencil I. True II. True III. True
63. (b) Since the passage talks about the defense system of
Fruits
our country and role of the government option (b) may
be a conclusion of the passage.
y
o
u
rs
m
a
h
SOLUTIONS 103

b
o
o
64. (a) Obviously option (a) strengthens the conclusion of

b
5 8 6 9 7 2

.w
the passage. 69. (c)

o
65. (b) Option (b) has the opposite sense. Thus it weakens 9 8 7 6 5 2

rd
the conclusion of the passage.

p
70. (d) 9 8 7 6 5 4 3 2 1

re
66. (d) Except sugar, all others are liquid.

s
8 7 6 5 4 3 2 1

s
67. (b) Meaningful Word DINE

.c
7 6 5 4 3 2 1

o
68. (c) Except 75, all other numbers are perfect cubes.

m
6 5 4 3 2 1
343 = 7 × 7 × 7
64 = 4 × 4 × 4
27 = 3 × 3 × 3
216 = 6 × 6 × 6
y
o
u
rs
m
a
h
104 101 SPEED TEST

b
o
o
SPEED TEST 100 : FULL MAIN TEST - 4

b
.w
Answer Key

o
rd
1 (d) 26 (b) 51 (e) 76 (e) 101 (b) 126 (c) 151 (d) 176 (e)

p
2 (b) 27 (a) 52 (a) 77 (d) 102 (a) 127 (a) 152 (a) 177 (b)

re
s
3 (d) 28 (b) 53 (e) 78 (a) 103 (a) 128 (d) 153 (c) 178 (a)

s
.c
4 (e) 29 (b) 54 (d) 79 (c) 104 (e) 129 (d) 154 (e) 179 (e)

o
m
5 (c) 30 (c) 55 (d) 80 (b) 105 (a) 130 (c) 155 (a) 180 (c)
6 (d) 31 (d) 56 (e) 81 (c) 106 (d) 131 (c) 156 (e) 181 (b)
7 (e) 32 (b) 57 (b) 82 (a) 107 (a) 132 (a) 157 (b) 182 (d)
8 (c) 33 (a) 58 (d) 83 (b) 108 (e) 133 (e) 158 (d) 183 (a)
9 (a) 34 (d) 59 (e) 84 (d) 109 (e) 134 (c) 159 (c) 184 (d)
10 (e) 35 (b) 60 (a) 85 (e) 110 (c) 135 (b) 160 (e) 185 (c)
11 (d) 36 (c) 61 (e) 86 (d) 111 (c) 136 (b) 161 (c) 186 (a)
12 (b) 37 (a) 62 (d) 87 (a) 112 (a) 137 (c) 162 (a) 187 (a)
13 (a) 38 (b) 63 (a) 88 (b) 113 (b) 138 (e) 163 (e) 188 (a)
14 (e) 39 (c) 64 (d) 89 (e) 114 (d) 139 (b) 164 (b) 189 (d)
15 (c) 40 (c) 65 (a) 90 (c) 115 (c) 140 (c) 165 (d) 190 (b)
16 (c) 41 (d) 66 (b) 91 (a) 116 (b) 141 (d) 166 (d)
17 (b) 42 (a) 67 (c) 92 (c) 117 (b) 142 (c) 167 (a)
18 (a) 43 (b) 68 (d) 93 (a) 118 (e) 143 (c) 168 (c)
19 (a) 44 (d) 69 (c) 94 (c) 119 (a) 144 (b) 169 (e)
20 (a) 45 (b) 70 (a) 95 (c) 120 (a) 145 (c) 170 (d)
21 (d) 46 (a) 71 (c) 96 (e) 121 (c) 146 (b) 171 (e)
22 (d) 47 (a) 72 (d) 97 (d) 122 (d) 147 (b) 172 (b)
23 (d) 48 (d) 73 (d) 98 (d) 123 (b) 148 (a) 173 (c)
24 (c) 49 (a) 74 (b) 99 (e) 124 (e) 149 (e) 174 (a)
25 (b) 50 (c) 75 (d) 100 (d) 125 (e) 150 (c) 175 (d)

Hints & Solutions


51. (e) 3 × ? + 30 = 0 c
105 2
ab a bc 1010
30
? 10
3 57. (b) ? = 7589 – 3534 = 4055
52. (a) ? = 40.83 × 1.02 × 1.2 = 49.97592
1225 35
1 3 1 22 58. (d) ? = 7
53. (e) ? = 3 6 1 5 5
3 7 2 7 59. (e) ? = 300 + (100 × 2) = 300 + 200 = 500
10 45 3 22 5 1.6 2 1.4 8 2.8 5.2
2.44 60. (a) ? 4
3 7 2 7 1.3 1.3 1.3
54. (d) 1.5625 1.25 2 1 1 2 1 1
61. (e) 3 7 5 = (3 + 7 – 5) + =
55. (d) 3978 + 112 × 2 = ? 2 5 5 4 5 5 4
? = (3978 +224) × 2 = 8404
8 4 5 7
5 5
3.7 1.3 2 ? 20 20
56. (e) 10 10 10
62. (d) Let the number be x.
1.3 2 x x x
103.7 10? [ a b ac ab c
] Now 3 or 3 or x = 36
3 4 12

10? 105
2 63. (a) 25 92 32 81 2592 .
y
o
u
rs
m
a
h
SOLUTIONS 105

b
o
o
Difference = 25 × 92 – 2592 74. (b) Add 3 after doubling the previous number.

b
(d) The series exhibits the pattern of n2 + 1, n2 – 1,

.w
= 2592 – 2592 = 0 75.
Hence, the numerical difference is 0. alternatively, n taking values 1, 2.............

o
rd
64. (d) Let the third number be 100. Then, the first and second 76. (e) Let x be there in the question mark.

p
numbers will be 20 and 50, respectively.

re
So, 3
860000 x x3 860000

s
20

s
Required % = 100 40 Taking log10 on boths the sides 3log10x = 5.9345

.c
50 log x = 1.9782, Taking antilog we get x 95

o
m
65. (a) Let the cost price of an article be `100 [log x is nearly 2 so, x will be near to but less than 100]
then, S.P. = 100 + 10 = `110
5 1 2 13 16 12
If S.P. = 2 × 110 = `220 77. (d) 1 5 2
8 3 5 8 3 5
(220 100)
then, profit % = 100 = 120% 15 13 40 16 12 24 195 640 288
100 =
66. (b) Ratio of investment 120 120
= 3500 : 4500 : 5500 = 35 : 45 : 55 = 7 : 9 : 11 1123
Since, Ratio of investment is same as ratio of profit. = 9.35 9.
120
Ratio of profit = 7 : 9 : 11
Now, profit = ` 405 8769 8769
78. (a) 8769 82 4 = 26.73 27
4 82 328
7
A’s share = 405 = `Rs 105
105 79. (c) Let x be there in place of question mark so, x% of
27
45.999 × 16% of 83.006 = 116.073.
1 1 1 x 16
67. (c) Part of the tank filled in one hour = We take 46 83 116
8 16 16 100 100
Hence, the tank will be filled in 16 hours. x × 0.46 × 13.28 = 116
68. (d) Part of the tank filled by the three pipes working or x × 6.11 = 116
1 1 1 17 x = 18.98 19.
simultaneously in one hour is = 80. (b) 12.998 × 27.059 × 17.999
5 6 12 60
Can be taken as near to 13 × 27 × 18 6318
60 81. (c) r2 = 2 × 11088
i.e. it takes hours to fill up the tank completely..
17 22
r 2 = 2 × 11088
1 7
Now, of the tank is filled with all the pipes open,
2 2 11088 7
r2 = 7056
60 1 13 22
simultaneously together in = 1 hours
17 2 17 r= 7056 = 84 metre
69. (c) Any even number is given by 2n for all n Z, where Z
is a set of integers. This is divisible by 9 if it form 9 × 2n 22
Circumference = 2 r = 2 × × 84 = 528 metre
= 18n, which is divisible by 18. 7
For example, number 36 is even and sum of digits 82. (a) x + y = 8
(3 + 6) is 9, which is divisible by 9. Hence, the number
36 is divisible by 18. x y 8
2 x 16
70. (a) Volume of water left in the tank = × b × h
= 20 × 7 × (10 – 2) = 1120 m3 x=8
71. (c) This is a series of prime number y=0
72. (d) Let x = 8 Two digit number = 80
then 15 = 2x – 1 = y 83. (b) Required ratio
28 = 2y – 2 = z
= (4220 – 2420) : 2420
53 = 2z – 3 = m
Next term in the pattern should be 2m – 4 = 2 x 53 – 4 = 1800 : 2420 = 90 : 121
= 102 84. (d) The word SCENIC consists of 6 letters in which C comes
73. (d) It is a combination of two series, namely twice.
24, 49, – 94; and 15, 31, 59, 58 6!
The two series correspond to Number of arrangements =
2!
x, (2x + 1), (4x – 1), (4x – 1), (4x – 2)
Hence the missing term is 6 5 4 3 2 1
4 × 24 – 1 = 95 = 360
2
y
o
u
rs
m
a
h
106 101 SPEED TEST

b
o
o
85. (e) 4x + 12y = 1520

b
10

.w
Multiplying both sides by 2.5, Gain per cent 100 25
40

o
10x + 30y = 1520 × 2.5

rd
94. (c) If difference between the compound interest and simple
= ` 3800

p
interest on a certain sum of money for 2 yr at r% rate is

re
86. (d) Difference of production of C in 2010 and A in 2015 = `x.

s
5,00,000 tonnes

s
2

.c
87. (a) Percentage increase of A from 2011 to 2012 100

o
Sum x

m
r
55 40
100 37.5% 2 2
40 100 25
128 128
88. (b) Percentage rise/fall in production for B 8 2
128 25 25
` 20000
2011 2012 2013 2014 2015 2 2
95. (c) Let the original number be 10x + y.
9% –16.6% 10% – 9% 10% Number obtained by interchanging the digits
Here, the maximum difference is from 2011 to 2012, which = 10y + x
is 10. And the second nearest to it is fall or rise of 5. So, 10x + y – 10y – x = 18
undoubtedly the answer is 2012. 9x – 9y = 18
x–y=2
120 Again, x+y=6
89. (e) Percentage production = 100 133.3%
90 x = 4 and y = 2
90. (c) Average production of A = 50 Original number
= 10 × 4 + 2 = 42
Average production of B = 54.17 96. (e) Let Samar’s monthly salary be `x.
Average production of C = 50 According to the question,
Difference of production = 54.17 – 50 = 4.17 x – (52 + 23)% of x = 4500
91. (a) Let the number be x. x – 75% of x = 4500
25% of x = 4500
x 63
2583 4500 100
100 x `18000
25
2583 100 97. (d) 55% of total students = 44
x 4100
63
44 100
45 Total number of student 80
45% of this number 4100 1845 55
100 Number of boys = 80 – 44 = 36
92. (c) Decimal equivalents of the fractions
45 75 40 78
4 6 98. (d) Required average marks
0.8, 0.857 45 40
5 7
3375 3120 6495
2 9 76.41
0.22, 0.82 85 85
9 11 99. (e) Perimeter = 2(Length + Breadth)
3 48 = 2(5x + 3x)
0.375
8 48
x 3
Clearly, 16
0.857 > 0.82 > 0.8 > 0.375 > 0.22 Area = (5 × 3) × (3 × 3) = 15 × 9 = 135 m2
6 9 4 3 2 100. (b) Let the children in row = x
The children in a column = (x + 2)
7 11 5 8 9
According to the question,
93. (a) Purchasing price of articles = `(245 × 30) = `7350 x(x + 2) = 63
Total cost = `(7350 + 980 + 1470) = `9800 x2 + 2x – 63 = 0
9800 x2 + 9x – 7x – 63 = 0
CP per piece ` 40 x(x + 9) – 7(x + 9) = 0
245
SP = `50 per piece (x + 9) (x – 7) = 0
Hence, there are 7 children in each row.
y
o
u
rs
m
a
h
SOLUTIONS 107

b
o
o
101. (b) Clearly option (b) is best supported by the passage. For Q.No.114-119

b
.w
102. (a) The given passage talks about the cause of corruption P$Q P<Q

o
and option (a) also has the same sense. Hence, it may P©Q P>Q

rd
be the inference.

p
P Q P=Q
103. (a) Obviously, option (a) is the most effective statement in

re
P@Q P Q

s
the context of the given passage.

s
.c
104. (e) According to the question, after interchanging the P*Q P Q

o
symbols, equations becomes 114. (d) Given statements, M@D M K,

m
26 + 15 ÷ 5 – 4 × 2 = 26 + 3 – 8 = 29 – 8 = 21 D K D = K, K© R K >R
+4 +2 +2 +2
Conclusions :
+2
I. R $ M R < M (True)
105. (a) R P N W S U H D F L H J Q M O
II. K M K= M (True)
–2 +2 +2 +2 +2 III. K $ M K < M (True)
So, RPN is different from other four. Hence, only either II or III and I is true from the given
Left Right statements.
10 students 5th
D R 115. (c) Given statements F*T F< T,,
106. (d)
40 students T $ N T < N, N@ R N R
So, D’s position from the left end of the row Conclusions :
I. R $ T R < T (False)
= 40 – (10 + 1 + 5 ) + 1 = 41 – 16 = 25 th
II. N ©F N>F (True)
R I R III. F $ R F < R (False)
107. (a) O B T L
Hence, only conclusion II is true from the given
statements.
+1 +1 –1 +1 +1
+1 +1 116. (b) Given statements, B © N B> N,
C P M
N@ R N R, F * R, F R
S H Conclusions :
Similarly, I. B © R B > R (True)
H E II. F * N F N (False)
C A G R III. R $ B R < B (True)
–1
Hence, only I and III conclusion are true from the given
+1 +1 +1 +1 statements.
+1 +1
117. (b) Given statements, D $ M D< M,
B I D Q S F H M*B M < B, B J B =J
108. (e) According to the question, Scored order among the Conclusions :
students = A > D or D > A > B > C > E I. J © D J > D (True)
II. B @ D B D (False)
So, E student is the lowest marked student.
III. J @ M J M (True)
O V E R W H E L M Hence, only I and III conclusion are true from the given
109. (e) statements.
118. (e) Given statements, W K W =K,
So, such number of pairs are OR, EH, LM, MR.
K©F K >F, F $ M F<M
110. (c) B R O W N and M E A N Conclusions :
I. M © K M > K (False)
5 3 1 @ % 2 6 © %
II. W @ F W F (False)
R O B E
III. F @ W F W (False)
3 1 5 6 Hence, any conclusion is not true from the given
111. (c) According to the question, in the above arrangements statements.
seventh to the left of the nineteenth from the left is ©. 119. (a) Given statements, F @ T F T,
112. (a) According to the question, in the above arrangements T K T =K, K * D K<D
following series is progressing (+ 2) and ( + l)step in his Conclusions :
group and next group comes (+ 3) step, I. D @ F D F (False)
So, next series comes in the group is U I *. II. F @ K F K (True)
113. (b) According to the question, in the above arrangements III. D @ T D T (True)
one vowel is immediately preceded by a symbol and Hence, conclusion II and III are true from the given
immediately followed by a number, Vowel is © A6 statements.
y
o
u
rs
m
a
h
108 101 SPEED TEST

b
o
o
For Q. 120-125. Conclusions :

b
.w
Sitting arrangements of 8 persons would be as follows : I. Some buses are cars. (False)
II. Some trains are bikes. (False)

o
D

rd
M Q III. Some buses are bikes. (False)

p
So, none conclusion follows given statements.

re
128. (d) According to the question, Venn-Diagram is

s
s
P

.c
B

o
m
T A Rats
Dogs Rats Dogs
R or ts
Ma
120. (a) From the above arrangements, M is to the immediate Cats Ma
ts
Cats
right of D.
121. (c) From the above arrangements, T is second to the right Conclusions :
of M. I. Some mats are cats. (True)
122. (d) From the above arrangements, P is second to the left of D. II. Some mats are dogs. (False)
123. (b) From the above arrangements, M is third to the right of P. III. Some rats are cats. (True)
124. (e) From the above arrangements, P and R are the immediate So, conclusion I and III follow given statements.
neighbours of A. 129. (d) According to the question, Venn-Diagram is
125. (e) From the above arrangements, in the following pairs, P
is immediate to the right of A.
For Q. No. 126-131.
126. (c) According to the question, Venn-Diagram is

Pins
Dr
um

cups Drums
s

Sticks
or cups
tes

Rings
Ki
s

Benches
ite

Benches Rods
K

Conclusions :
I. Some kites are cups. (False) Conclusions :
II. Some kites are benches. (True) I. Some rings are pens. (True)
III. Some drums are·cups. (False) II. Some roads are sticks. (True)
So, only conclusion II follows, given statements. III. Some roads are pens. (True)
127. (a) According to the question, Venn·Diagram is So, all I, II, III conclusions follow, given statements.
130. (c) According to the question, Venn-Diagram is

Bikes Cars Trains Buses


Tables
es

Chairs
Hous

ts
Ten

Cars

Bikes
or
Conclusions :
Buses I. All houses are chairs. (False)
Trains II. Some tents are chairs. (True)
III. Some houses are tables. (True)
So, conclusion II and III follow given statements.
Cars
131. (c) According to the question, Venn-Diagram is

Bikes
Boxes Walls Roads

or
Trains Rivers
Buses
or
y
o
u
rs
m
a
h
SOLUTIONS 109

b
o
o
b
140. (c) green grass everywhere dik pa sok

.w
o
cow eats grass nok ta pa

rd
Boxes Walls Roads

p
The code for 'cow' is 'nok' or 'ta'

re
Rivers 151. (d) The reason behind the question is not mentioned in

s
s
.c
the passage.
Conclusions :

o
152. (a) Refer to the second sentence of the first para of the

m
I. Some rivers are walls. (False) passage that the king had to agree to a contract.......
II. Some roads are boxes. (False) being a king.
III. No wall is river. (True) 153. (c) Refer to the sentence that the island was covered.......
So, only conclusion III follows given statements. discovered dead bodies.......past kings of the second
132. (a) M A I N D E A R last sentence of the third para of the passage.
154. (e) Refer to the fourth sentence that in the first
9 3 6 4 8 5 3 2 month.......trees were cut down.......of the fourth para
Therefore, of the passage.
M E N D 155. (a) Refer to the last sentence “I turned the deadly
island..............a beautiful abode.......peacefully” of the
9 5 4 8 sixth para of the passage.
133. (e) D R E A M I N G 156. (e) From reading the passage thoroughly we come to the
conclusion that the king was intelligent, foresight and
cunning as he made deadly island a beautiful place to
+1 –1 live in with all luxuries of the kingdom.
157. (b) Refer to the sentence that ‘I know.......complete the
year’ of the statement made by king in the fourth para
B F S E F M H L of the passage.
Similarly, 158. (d) Refer to the second last sentence of the fourth para
T R E A T I S E that he sent all the.......for storage.......of the passage.
159. (c) Refer to the third sentence that however, the king.......leave
the kingdom.......of the last para of the passage.
+1 –1 160. (e) The moral of the story is ‘Always think and plan
ahead’.
161. (c) The word contract means agreement. Compact, bond,
B F S U D R H S
deal, bargain etc. are the most similar word in meaning.
162. (a) The word bidding means order, command, wish, desire,
134. (c) 5 3 1 4 6 9 7 request, direction etc. Hence wishing is most similar in
meaning of the given word.
1 3 4 5 6 7 9 163. (e) The word abode means house, dwelling, residence and
135. (b) C > E, A > B > D habitation etc.
Now, A > B > D > C > E 164. (b) The word survived means remained alive, lived,
endured etc. Hence died is the most opposite word in
136. (b) meaning.
N 165. (d) The word vicious means brutal, savage, dangerous,
30 m
cruel etc. Its opposite word will be gentle, kindly,
E harmless etc.
W
20 m 166-170. Sentences are re-arranged in order as DACFBE. A mother
duck is the clue and qualifier of a sentence which is
S followed by A, C, F and finally B and E.
171. (e) Here, too is used as emphatic word. Lata was so scared
137. (c) Meaningful words that she could not go home alone. Hence, no correction
TOUR, ROUT is required.
138. (e) 48 Q 12 R 10 P 8 W 4 = ? 172. (b) The structure of sentence is subject + was/were + third
? = 48 ÷ 12 × 10 – 8 + 4 form of verb + object. Thus, Riya was dressed to kill.
? = 4 × 10 – 8 + 4 173. (c) The given sentence is the statement of simple past
tense. Hence it should be ‘worried’ instead of worries.
? = 40 – 8 + 4 = 36
174. (a) The phrase ‘let off’ means to give them only a light
139. (b) Second Highest Number punishment.
7 39 175. (d) It should be ‘took’ instead of ‘take’.
y
o
u
rs
m
a
h
110 101 SPEED TEST

b
o
o
b
SPEED TEST 101 : MAIN TEST - 5

.w
Answer Key

o
rd
1 (d) 26 (a) 51 (d) 76 (a) 101 (d) 126 (a) 151 (a) 176 (a)

p
re
2 (c) 27 (b) 52 (b) 77 (d) 102 (b) 127 (b) 152 (e) 177 (e)

s
3 (c) 28 (d) 53 (a) 78 (b) 103 (b) 128 (a 153 © 178 (b)

s
.c
4 (c) 29 (b) 54 (c) 79 (a) 104 (c) 129 (e) 154 (a) 179 (d)

o
m
5 (a) 30 (a) 55 (e) 80 (e) 105 (a) 130 (a) 155 (d) 180 (d)
6 (c) 31 (c) 56 (d) 81 (c) 106 (e) 131 (d 156 (e) 181 (e)
7 (a) 32 (c) 57 (b) 82 (a) 107 (d) 132 (c) 157 (d) 182 (a)
8 (d) 33 (a) 58 (c) 83 (d) 108 (c) 133 (c) 158 (c) 183 (c)
9 (d) 34 (c) 59 (e) 84 (e) 109 (c) 134 (a) 159 (b) 184 (a)
10 (d) 35 (c) 60 (e) 85 (b) 110 (b) 135 (e) 160 (b) 185 (c)
11 (b) 36 (a) 61 (b) 86 (c) 111 (a) 136 (d) 161 (b) 186 (c)
12 (a) 37 (b) 62 (c) 87 (b) 112 (a) 137 (d) 162 (c) 187 (c)
13 (d) 38 (c) 63 (b) 88 (a) 113 (c) 138 (b) 163 (d) 188 (a)
14 (b) 39 (d) 64 (b) 89 (d) 114 (c) 139 (a) 164 (e) 189 (d)
15 (a) 40 (c) 65 (a) 90 (e) 115 (d) 140 (c) 165 (a) 190 (a)
16 (d) 41 (a) 66 (c) 91 (b) 116 (d) 141 (a) 166 (d)
17 (c) 42 (c) 67 (b) 92 (c) 117 (e) 142 (c) 167 (e)
18 (e) 43 (b) 68 (a) 93 (a) 118 (c) 143 (d) 168 (a)
19 (c) 44 (a) 69 (a) 94 (e) 119 (a) 144 (c) 169 (b)
20 (a) 45 (a) 70 (c) 95 (d) 120 (c) 145 (a) 170 (c)
21 (a) 46 (c) 71 (c) 96 (c) 121 (b) 146 (a) 171 (d)
22 (a) 47 (b) 72 (d) 97 (d) 122 (c) 147 (b) 172 (c)
23 (a) 48 (b) 73 (b) 98 (b) 123 (b) 148 (c) 173 (a)
24 (a) 49 (b) 74 (c) 99 (b) 124 (b) 149 (d) 174 (b)
25 (c) 50 (a) 75 (c) 100 (a) 125 (d) 150 (a) 175 (a)

Hints & Solutions


21. (a) Iran Recently, India has offered to invest up to 20 billion Railway station to Agra Cantt. Station. The train
US dollars in Iran's energy industry, subject to provision completed its 200 km journey to Agra within 100
of concessional rights.The two nations, also discussed minutes. It is capable of running at a maximum speed of
developments regarding Farzad-B gas field. . 160 kmph.
22. (a) Telangana Mission Bhagiratha is the water grid project 27. (b) The world's first 'White Tiger Safari' has recently opened
in Telangana State of India, which aims at reaching out for public at Mukundpur in Satna district, Madhya
drinking water supply to even remotest place in the Pradesh. The first of its kind safari has cost Rs. 50 crore
State. and is spread over an area of 25 hectares. It is home to
23. (a) simplification of income tax laws three white tigers including one male, two females and
24. (a) Subhash Palekar, popularly known as Krishi ka Rishi two Royal Bengal tigers.
(the "sage of agriculture"), is a famous exponent of 28. (d) Unakoti hill is an ancient Shaivite place of worship with
natural farming. huge rock reliefs celebrating Shiva. It is the prime tourist
25. (c) The Gujarat Government has launched the state-wide spot of Unakoti Tripura District in the Kailashahar
"Maa Annapurna Yojna (Scheme)" at Sanand, Subdivision in the North-eastern Indian state of Tripura.
Ahmedabad to provide cereals at concessional rates .
to poor and middle class families under the National 29. (b) The Central Industrial Security Force (CISF) Raising
Food Security Act (NFSA). This scheme will cover BPL, Day is celebrated every year on March 10. On this day,
Antyodaya and APL families. in 1969, the CISF was set up under an act of the
26. (a) India's fastest train Gatimaan Express "Gatimaan Parliament of India with a strength of 2,800. It is directly
Express" has been flagged off from Hazrat Nizamuddin under the Union Ministry of Home Affairs and its
headquarters are at New Delhi.
y
o
u
rs
m
a
h
SOLUTIONS 111

b
o
o
30. (a) A 108-feet tall idol of Lord Rishabhdeva, the first 37. (b) Gujarat Nargol port is located in Gujarat. Indian Railways

b
.w
Teerthankar of Jains, carved out of a single rock, has proposes to undertake implementation of rail connectivity

o
entered the "Guinness World Records" as the world's for the ports of Nargol and Hazira under PPP

rd
tallest Jain statue. The impressive statue is located atop 38. (c) Madhya Pradesh The 42nd edition of world fame

p
re
Mangi Tungi mountain near Teharabad village of Khajuraho Dance Festival has started at Khajuraho in

s
Baglan tehsil in Nashik district of Maharashtra. Before Chhatrapur District, Madhya Pradesh. This cultural

s
.c
this statue, the 57 feet idol of Lord Bahubali in festival highlights the richness of the various Indian

o
Shravanabelagola in Karnataka was considered the

m
classical dance styles like Kathak, Bharathanatyam,
world's tallest Jain statue. Odissi, Kuchipudi, Manipuri and Kathakali with
31. (c) The President of India Pranab Mukherjee has declared performances of some of the best exponents in the field.
Kerala as India's first digital state during the launch of 39. (d) The Maharashtra State Economic Development Council
digital empowerment campaign at Kozhikode, Kerala. and Naik Environmental Research Institute Ltd in
The campaign aims to develop better digital Nashik have formed Deepak Naik committee to work
communication across the state by 2020. Kerala is the towards reviving the river Godavari. Naik is the
first Indian state to successfully link its villages with chairman of Maharashtra state Economic development
broadband connectivity under the National Optical
council. He is an expert in river issues.
Fibre Network (NOFN) programme.
40. (c) India's first ever Gender Park has been inaugurated by
32. (c) February 28 The National Science Day is celebrated in
President Pranab Mukherjee at Kozhikode, Kerala. The
India on February 28 every year to mark the discovery
park is set up to promote research and other initiatives
of the Raman effect by Indian physicist C V Raman.
to enable total gender equality. The Gender Institute at
The 2016 theme is "Scientific Issues for Development
the park would focus on learning, research and capacity
of the Nation" that aims at raising public appreciation.
development, as part of supporting the efforts of the
State and Central governments in ensuring an inclusive,
33. (a) The NDDB foundation for Nutrition (NFN) has
discrimination free society.
launched its novel 'Gift Milk' initiative to provide
nutritional support for school children. NDDB's wholly 41. (a) The World Day of Social Justice is observed on
owned subsidiary, Indian Immunologicals Limited (IIL) February 20 to recognize the need to promote efforts to
partnered with NFN and launched the initiative in the tackle issues such as poverty, exclusion and
Z.P. School, Laxmapur village, Ranga Reddy District, unemployment. The 2016 theme is "A Just Transition -
Telangana. IIL has adopted this school as part of its environmentally sustainable economies and societies".
CSR activity. . 42. (c) The Odisha Government has launched the Pucca Ghar
34. (c) India The World's first fast-acting anti-rabies drug Yojana for construction workers. The pucca houses
"Rabies Human Monoclonal Antibody (RMAb)" will will be built for 50,000 construction workers during the
be launched in India. The drug has been developed by current financial year with a cost of Rs. 500 crore. The
Pune-based Serum Institute of India (SII) and US-based state government has decided to provide an assistance
MassBiologics of the University of Massachusetts of Rs.1 lakh to each registered construction worker of
Medical School. the state for construction of his/her house under the
35. (c) Prof. Goverdhan Mehta has been conferred with the Yojana. A construction worker registered with the
Federal Republic of Germany's prestigious Odisha Buildings and Construction Workers' Welfare
"Bundesverdienstkreuz" (the Cross of the Order of Board for the three years can avail the benefit.
Merit). He is a well-known researcher in Chemical 43. (b) Arunachal Pradesh is also known as the "land of the
Sciences & former Vice-Chancellor of the University of dawn-lit mountains". Arunachal Pradesh was granted
Hyderabad (UoH). He is a Fellow of the Royal Society statehood on 20th February, 1987. . 7. Utkarsha Bangla
(FRS), a Foreign Member of the Russian Academy of scheme has been launched by the West Bengal
Sciences and has been conferred with 'Chevalier de la government, for what purpose? [A]To train school
Legion d'Honneur' by the President of France. dropouts [B]To stop human trafficking [C]To promote
36. (a) GVK Biosciences The Hyderabad-based GVK entrepreneurship among women [D]To promote organic
Biosciences has received the Global CSR Excellence & farming Hide Answer To train school dropouts The
Leadersh ip Awards in the category of Best West Bengal government has recently launched
Environment-Friendly Project for its "Go Green, Grow Utkarsh Bangla Scheme, a welfare scheme for school-
Green" plantation drive. The global CSR awards are dropouts. The scheme is aimed at giving vocational
constituted by the World CSR Congress every year on training to school dropouts who will be trained free of
the World CSR Day and are presented to companies charge in tailoring, driving, repairing television and
who believe in the value of sustainable corporate social others. Each youth will get training ranging from 400 to
responsibility. . 1200 hours.
y
o
u
rs
m
a
h
112 101 SPEED TEST

b
o
o
44. (a) The first train connecting China to Iran through the ancient 63. (b) Let the sum of money be ` x.

b
Silk Road has arrived in the Iranian capital after traveling

.w
3
over 10,000 kilometers. The train had started its journey r
Now, 8x

o
x 1

rd
from China's eastern Zhejiang province and it took 14 days 100

p
to reach final destination. It had passed through two Central 3

re
Asian countries Kazakhstan and Turkmenistan r r
or, 1 (2)3 or 1 2

s
100

s
45. (a) The Indian Institute of Petroleum and Energy (IIPE) 100

.c
has been established at Visakhapatnam by the Ministry Again, let the sum becomes 16 times in n years. Then,

o
of Petroleum and Natural Gas (MoPNG), Government

m
n
of India. This institute has the mandate to lead the r
16 x x 1
nation forward in education, research and training 100
related to all areas of Petroleum and Energy.
46. (c) 47. (b) 48. (b) 49. (b) 50. (a) 16 2 n or 2 4 2 n or n 4
64. (b) Let the cost price of machine be ` 100
5 4 3 SP of machine at a profit of 10% = ` 110
51. (d) ? 222 37
8 9 5 SP of machine at a loss of 10% = ` 90
52. (b) Let the number be x If SP is (110 – 90) = ` 20 less then CP = ` 100
Therefore, if SP is ` 40 less, then
56
450 x 300 or x = 300 – 252 = 48 100
100 CP = 40 ` 200
20
53. (a) (27)1.5 (21)3.5 (27) ? 65. (a) Let us work with the options.
For (a), total cost = 7 480 2 130 3620
?=5 ax ay a(x y)
For (b), total cost = 7 480 3 130 3750
54. (c) Let the number be x. For (c), total cost = 2 480 7 130 1870
27.06 × 25 – x = 600 Hence, option (a) is correct.
or, x = 676.5 – 600 = 76.5 66. (c) Relative speed of the trains = (40 + 20) = 60 m/s
39 30 45 1 Distance = (120 + 120) = 240 m
55. (e) ? 11 Time taken by trains to cross each other completely
8 13 4 4
240
1 1 = 4s
56. (d) 84 3
85 84 3 5 2 84 ? = 4 60
8 82 Larger the no. of cogs (tooth of wheel) of wheel,
57. (b) –(a – b) . x = b – a lesser will be that no. of revolution made by it.
Put x replacing ‘?’ (question mark) 67. (b) Let Sunil finishes the job in x hours.
or [ (a b) x] [b a] or (a b) x a b x
Then, Ramesh will finish the job in hours.
a b 2
or x 1 x
a b We have, x 3 x 6
2
58. (c) a b ? ( a b) Therefore, Sunil finishes the job in 6 hours and Ramesh
or a b x.( a b) in 3 hours.
[Put x replacing ‘?’ (question mark)] 1 1 1
or a + = x. (a + b) Work done by both of them in 1 hour =
6 3 2
or x = – 1 They together finish the piece of work in 2 hours.
59. (e) |? + 14| = 11 or ? + 14 = 11 or – 11 68. (a) Let the present age of the man = x years
? = – 25 or – 3 Now, (x + 15) = 4 (x – 15)
60. (e) 16 + 26 × 2 = 16 + 52 = 68 or 3x = 75 or x = 25 years
12 1 69. (a) Area of the outer rectangle = 19 × 16 = 304 m 2
61. (b) 0.1008, 0.1
119 10 2m
Verandah
4 7
0.102 and 0.101
39 69
1 12
Thus, is the least. 2m
10 2m
62. (c) Let there be n points marked on the plane.
15 Floor of room
Total number of line segments = n C 2 =10
2m
n(n 1)
10 or n 2 n 20 0 Area of the inner rectangle = 15 × 12 = 180 m2
2
Required area = (304 – 180) = 124 m2
or (n 5)(n 4) 0 or n = 5 [n = – 4 is rejected]
y
o
u
rs
m
a
h
SOLUTIONS 113

b
o
o
70. (c) The terms exhibit the pattern 21, 22, 23 and so on.

b
1.08

.w
71. (c) Try the pattern n 3 – 1, n = 1, 2, ....... 87. (b) Required percentage = 100 34
3.14
Pattern is 22, 42 – 1,42 , 62 – 1, 62 and so on.

o
72. (d)

rd
73. (b) Can you see that the pattern is 88. (a) Total number of candidates appearing from all the cities

p
together
12 , 2 3 , 3 2 , 4 3 , 5 2 , 6 3 , 7 2

re
= (1.25 + 3.14 + 1.08 + 2.27 + 1.85 + 2.73) lakhs

s
74. (c) Note that

s
= 12.32 lakhs

.c
0 = 13 – 1 6 = 2 3 – 2

o
Number of candidates passing from City F

m
24 = 33 – 3
7
75. (c) ? 1223.9975 34 = 2.73 1.5925 lakh
12
76. (a) ? = 503 × 201 = 101103 101100
77. (d) ? = 1205 2.5 = 482 480 1.5925
Required percentage = 100 12.93
78. (b) ? = 22020 0.011 = 2001818 2002000 12.32
89. (d) Number of failures:
79. (a) ? 20800 144 12
3
8 15 22 176 City A × 1025 lakhs = 0.375 lakh
80. (e) ? 25.14 10
25
5 7 3 7
81. (c) Required average height 3
City B × 3.14 lakhs = 1.1775 lakh
8
142 156 162 178 638
159.5 cm 5
4 4 City C × 1.08 lakhs = 0.6 lakh
82. (a) Manoj's monthly income 9

3
= 2.16 100000 = ` 18000 City D × 2.27 lakh = 1.7025 lakh
12 4
Pratul's monthly income 2
City E × 1.85 lakhs = 0.74 lakh
1 5
= 18000 × = ` 4500
4 5
City F × 2.73 lakh = 0.455 lakh
Pratul's annual income 12
= 12 × 4500 = ` 54000 90. (e) Number of passed students from City E
83. (d) The pattern of the number series is: 3
17 + 92 = 17 + 81 = 98 = 1.85 lakhs = 111000.
5
98 + 72 = 98 + 49 = 147
147 + 52 = 147 + 25 = 172 91. (b) Interest = `8791 – 7450 = `1341
172 + 32 = 172 + 9 = 181 I 100 1341 100 2235
Time 3 yr
181 + 12 = 181 + 1 = 182 P R 7450 6 745
92. (c) 36% of x – 12% of x = 82.82
84. (e) The pattern of the number series is:
11 + 8 = 19 36 12
or x –x 82.32
19 + 12 (= 8 + 4) = 31 100 100
31 + 16 (= 12 + 4) = 47
9 x 3x
47 + 20 (= 16 + 4) = 67 or – 82.32
25 25
67 + 24 (= 20 + 4) = 91
6x
85. (b) The pattern of the number series is: or 82.32
25
748 – 460 = 288
or 6x = 82.32 × 25
460 – 316 = 144
316 – 244 = 72 82.32 25
or x 343
6
244 – 208 = 36 ? = 208 – 18 = 190
93. (a) 16 watches + 21 calculators = `30485
3 3 32 watches + 42 calculators = 2 × `30485 = `60970
86 (c) Required ratio = 2.27 : 1.25
4 10 94. (e) x2 + (66)2 = 4840
= 1.7025 : 0.375 = 227 : 50 or x2 + 4356 = 4840
or x2 = 4840 – 4356
y
o
u
rs
m
a
h
114 101 SPEED TEST

b
o
o
or x2 = 484

b
102. (b) G I V E F A I L

.w
or x = 22
(d) Let the money received by A, B and C be `3x, 4x and

o
95.

rd
5x, respectively and money received by P and Q be `2y 5 1 @ © % 2 1 9

p
and y, respectively.

re
4x – y = 1050 …(i) Therefore, L E A F

s
s
.c
Since, we cannot from another equation here. So, we cannot

o
solve it. 9 © 2 %

m
96. (c) x2 + y2 = (x + y)2 – 2xy
= (18)2 – 2 × 72 103. (b) 3 8 6 7 4 5 1 0 9 2
= 324 – 144
3rd from right
= 180
104. (c) All others are parts of a Car.
484
97. (d) 100 64.533 105. (a) S U B S T A N C E
750
= 65% about
–1 +1 +1
t
r
98. (b) CI = p 1 –1
100 U F O
R A T R D B
Similarly,
3
3
`5000 1 –1 T E N T H O U S E
100

103 103 103 –1 +1 +1


`5000 –1
100 100 100

1092727 –1000000 S M D S I F T V P
`5000 106. (e) 519 915; 364 463;
1000000
287 782; 158 851;
92727 835 538
`5000
1000000 Second lowest number
= `5000 × 0.092727 538 83 5
= `463.635
107. (d) Second highest number
= `464 (about)
99. (b) In 1 day, Girish attends 7 customer's call 581 158
In 60 days, Girish attends 60 × 7 = 420 calls 108. (c) Highest number 835
100. (a) Suppose that numbers are x and y Lowest number 158
From question Required difference 5–3=2
2x + 3y = 126 …(i) (109–113) : Sitting arrangement
3x + 2y = 144 …(ii) D
Eq. (i) multiple by 3 and Eq. (ii) multiple by 2 and then
M W
subtract Eq. (ii) from Eq. (i), we get
5y = 90
or y = 18 R Q
Put the value of y in Eq. (i)
2x + 3y = 126 P B
or 2x + 3 × 18 = 126 A
or 2x + 54 = 126 109. (c) Q is second to the left of D.
or 2x = 126 – 54 = 72 110. (b) D is third to the left of P.
or x = 36 111. (a) W is to the immediate right of W.
x = 36, y = 18 112. (a) D and M are the first and second respectively to the
right of W.
Smaller number = 18
113. (c) B is sitting to the immediate immediate right of A.
101. (d) treeis very beautiful ka na da ta = $ @
(114– 118) : #
this is strong tree na pa sa ka
The code for ‘beautiful’ is either ‘da’ or ‘ta’.
y
o
u
rs
m
a
h
SOLUTIONS 115

b
o
o
114. (c) W @ T W T

b
121. (b) Number Symbol Number

.w
T©M T M

o
There is only one such combination: Q $ 6

rd
M$D M < D Therefore, W T M < D

p
Conclusions

re
122. (c) Number Consonant Symbol
I. W # D W D : Not True

s
s
Such combinations are:

.c
II. W @ M W M : Not True

o
1H % ; 2 Q $

m
III. D # T D T : True
1 3
115. (d) F R F R 123. (b) 8 1 D
R M R M 1 2
7 5
M$D M D
1 3
Therefore, F= R M D P 2 7
Conclusions E
1
J
3
R
I. D # R D > R : True
II. D # F D > F : True T
1
4
3
J
III. M @ F M F : True (124– 130).
116. (d) V © M V M (i) All petals are trees Universal Affirmative (A-type).
M B M B (ii) Some days are nights Particular Affirmative (I-type).
(iii) No lock is toy Universal Negative (E-type).
B$ F B F
(iv) Some locks are not toys Particular Negative (O-type)
Therefore, V M B F 124. (b) All keys are locks.
Conclusions
I. F # M F > M : True
II. B @V B V : True No lock is toy.
A + E E-type of Conclusion
III. F #V F V : True “No key is toy.”
117. (e) D # N D>N All bags are toys.
N@B N B
B F B F
No toy is lock.
Therefore, D> N B = F A + E E-type of Conclusion
Conclusions “No bag is lock.”
I. F$ D F D : True All bags are toys.
II. N# F N > F : Not True
III. N F N = F : Not True No toy is key.
N is either greater than or equal to F. Therefore, either I A + E E-type of Conclusion
or II is true. “No bag is key.
118. (c) R $ T R T This is Conclusion I.
T# K T>K 125. (d) All the three Premises are Particular Affirmative (I-type).
No Conclusion follows from the two Particular Premises.
K@M K M Conclusions I and III from complementary pair.
Therefore, R < T > K M Therefore, either I or III follows.
Conclusions 126. (a) Some tyres are wheels.
I. R $ M R M : Not True
II. T# M T > M : True
All wheels are buses.
III. R$ K R < K : Not True I + A I-type of Conclusion
119. (a) 21st from the right end is B and sixth to the right of B “Some tyres are buses.”
is 8. Conclusion I is Converse of it.
Trick : Required answer = 21 – 6 = 15th from the right, Conclusion II is Converse of the second premise.
i.e., 8. 127. (b) Some cats are horses.
120. (c) Symbol Vowel Consonant
Such combinations are:
@EJ ; %AV All horses are tigers.
y
o
u
rs
m
a
h
116 101 SPEED TEST

b
o
o
I + A I-type of Conclusion

b
“Some cats are tigers.” 139. (a) 1 8 4 3 5 7 2 9

.w
Conclusion I is Converse of it.

o
5th of the left of 2

rd
128. (a) All ropes are sticks.
140. (c) Z Y X W V U T

p
re
Z Y X W V U

s
Z Y X W V

s
Some sticks are hammers.

.c
A + I No Conclusion Z Y X W

o
m
129. (e) The following changes occur in the subsequent figures.
Z Y X
(1) to (2) (1) to (3)
151. (a) Refer to the sentence ‘He could not find.........in search
of work’.
152. (e) Refer to the sentence “Please do not help him”..........of
the second para of the passage.
153. (c) Refer to the sentence “You are...........bite me?”........of
These two steps are continued in the subsequent the second para of the passage.
figures alternately. 154. (a) Refer to the sentence “He would be.......Ghanshyamdas”
130. (a) The following changes occur in the subsequent figures: .......of the third para of the passage.
(1) to (2) (2) to (3) 155. (d) Refer to the sentence, “He gave him........delicious
fruits”........of the third para of the passage.
156. (e) Refer to the sentence “I shall creep........bite her........
hand on her forehead” of the fourth para of the passage.
157. (d) Refer to the sentence, “A man brought .......prince who
is missing”........of the third para of the passage.
158. (c) Refer to the sentence “He at once........pieces of gold”
These two steps are continued in the subsequent of the second last sentence of fourth para of the
figures alternately. passage.
131. (d) The given passage talks about the disposal of election 159. (b) Refer to the sentence “Finally, the king
petitions. It also gives the resolution of the disposal of declared........handsomely rewarded” of the fourth para
the election petition. While option (d) also have the of the passage.
same sanse, Hence it may be a conclusion. 160. (b) The moral of the story can possibly be “A good deed
132. (c) Obviously, option (c) strengthens the conclusion of never goes in vain”.
the passage. 161. (b) The word ‘live’ is most similar in meaning to the word
133. (c) The summary of option (c) opposes the conclusion of the ‘go’ in the context of the passage.
passes. Thus it weakens the conclusion of the passage. 162. (c) The word ‘modest’ is a synonym of humble, while other
134. (a) The code has been generated by writing the letters in words are plain, simple, ordinary and unpretentious.
reverse order OMNIBUS SUBINMO 163. (d) The most similar word is ‘ask for’ in the context of the
passage.
Therefore, TROUBLE ELBUORT
164. (e) The word ‘stopped’ is most opposite in meaning like
break off.
135. (e) 3 15 18 16 15 18 1 20 5 165. (a) The word ‘meagerly’ is the antonym of the word
handsomely in the context of the passage.
C O R P O R A T E 166. (d) Replace to by ‘too’ to make a correct phrase ‘a bit too
fast’ which means slightly or to a small extent.
167. (e) The word think about refers to consider. Hence no
136. (d) 1 2 3 4 5 6 7 8 9 correction is required.
E D U C A T I 0 N 168. (a) Replace leisure by the adverbial word leisurely.
169. (b) The phrase ‘gift of the gab’ means ‘to have a talent for
Meaningful word DATE speaking’.
170. (c) The phrase ‘the order of the day’ means common,
137. (d) C 20 m B popular or suitable at a particular time. For example :
Pessimism seems to be the order of the day.
10 m 10 m 171. (d) It should be ‘here was her son’s future’.
172. (c) The correct spelling should be ‘triumphant’.
E 173. (a) The word suppressed should be replaced by revealed
5 km D A or leaked in the sentence.
Required distance = AE = AD + DE = 20 km + 5 km = 25 km 174. (b) It should be ‘handed over’ which means the act of
138. (b) 12 B 20 A 4 Q 10 F 30 = ? moving power or responsibility from one person to
? = 12 × 20 ÷ 4 – 10 + 30 another.
? = 12 × 5 – 10 + 30 ? = 90 – 10 = 80 175. (a) The correct spelling is difficult.

S-ar putea să vă placă și